Skip to main content

Full text of "Mathematics Of Accounting Third Edition"

See other formats


OU_166514  >m 

CD  ^  CO 

> 


MATHEMATICS 

OF  ACCOUNTING 

BY 

ARTHUR  B.  CURTIS,  B.C.S.,  C.P.A. 

AND 

JOHN  H.  COOPER,  B.  Accts.,  C.P.A. 


THIRD  EDITION 


PRENTICE-HALL,  INC. 

Englewood  Cliffs 


PRENTJOT^ALL  ACCOUNTING  SERIES 
//.  A.  Finney,  Editor 


COPYRIGHT,  1925,  1934,  1947,  BY 

PRENTICE-HALL,  INC. 

ENGLEWOOD  CLIFFS,  N  J. 

ALL  RIGHTS   RESERVED.      NO   PART  OF  THIS  BOOK   MAY  BE 

REPRODUCED  IN  ANY  FORM,  BY  MIMEOGRAPH  OR  ANY  OTHER 

MEANS,     WITHOUT     PERMISSION     IN     WRITING     FROM     THE 

PUBLISHERS. 


First  Printing 

.  .  .June,  1925 

Second  Printing 

September,  1925 

Third  Printing   .... 

June,  1926 

Fourth  Flint  ing  . 

.February,  1928 

Fifth  Printing  . 

Juno,  1929 

Sixth  Printing 

August,  1930 

Seventh  Printing 

November,  1931 

Eighth  Printing 

September,  1933 

Revised  Edition 

First  "»  '     :   .• 

.January,  1934 

Secon  .  1  '  .   •  .   * 

.  .   August,  1934 

Third  !' 

May,  1935 

Fourt'-    !'      •:  £     .. 

February,  1936 

Fifth  Printing 

.February,  1938 

Sixth  Printing 

.    ..    May,  1940 

Seventh  Printing 

January,  1941 

Eighth  Printing     . 

September,  1941 

Ninth  Printing  

February,  1942 

Tenth  Printing 

.   Octobei,  1942 

Eleventh  Printing 

March,  1943 

Twelfth  Piiniing 

February,  19-14 

Thirteenth  I'MMIIIIK 

August,  1944 

Fourteenth  Printing 

February,  194  5 

Fifteenth  Printing     . 

.  ,   August,  1945 

Sixteenth  Printing 

.April,  1940 

Se\ont<MMith  Prinfing 

Mav.  1946 

Eighteenth  Printing 

August,  1946 

Nineteenth  Printing. September,  1946 

Thud   Edition 

Fust    Printing        .  January,  1047 

Second    Piintmg     .  August,  1047 

Third     Printing  October,  1947 

Fourth  Printing  .  .  June,  19J9 
Fifth  Printing  .  October,  1949 

Sixth    Printing  September,  1952 

Seventh  P    •  •    ^      ....    O-  "        1952 

Eighth    Pinning May,  1954 

Ninth  Punting November,  1954 

Tenth  Printing August,  1955 

Eleventh  Printing  ....  Match,  1936 
Twelfth  Printfnsr.  Senfpmbpr.  195ft 
Thirteenth  Printing,  June,  1957 


Preface 

It  is  now  somewhat  more  than  twenty  years  since  the  publica- 
tion of  the  first  edition  of  Mathematics  of  Accounting.  Those 
familiar  with  the  original  edition,  and  the  revised  edition  ten  years 
later,  will  find  that  in  the  present  revision  the  sequence  of  subjects 
has  been  considerably  changed;  the  treatment  of  some  subjects 
has  been  amplified;  certain  other  subjects  have  been  added.  Some 
problems  have  been  changed;  new  problems  and  review  problems 
have  been  included. 

Subjects  new  to  Part  1  are:  Factors  and  Multiples,  Business 
Insurance,  and  Payroll  Records  and  Procedure.  The  chapter  on 
Graphs  has  been  expanded  to  include  Index  Numbers. 

Part  2  has  been  enlarged  to  include  chapters  on  the  following: 
Permutations  and  Combinations,  Probability,  Probability  and 
Mortality,  Life  Annuities,  Net  Premiums,  and  Valuation  of  Life 
Insurance  Policies. 

Algebraic  formulas  have  been  changed  to  conform  to  standard 
usage,  while  arithmetical  substitutions  and  detailed  solutions  have 
been  retained.  Those  desiring  to  work  entirely  from  the  tables  in 
the  appendix  will  find  table  references  in  the  detailed  solutions. 

Grateful  recognition  is  here  given  to  those  who  have  taught 
from  the  previous  editions  and  who  have  responded  with  sugges- 
tions for  this  revision, 

ARTHUR  B.  CURTIS 
JOHN  H.  COOPER 


Contents 

PART  I 

2SAPTEB  PAGB 

1.  FUNDAMENTAL  PROCESSES  AND  SHORT  METHODS  FOR  THE 
ACCOUNTANT 1 

Addition;  Drill  tables;  Streamline  addition;  Drill  table;  Combinations 
whose  sum  is  10;  Drill  table;  Adding  where  the  same  number  is  repeated 
many  times;  Drill  table;  Group  addition;  Drill  table;  Addition  of  two 
columns  at  a  time;  Drill  table;  Recording  addition  by  columns;  Practical 
applications;  Subtraction;  Avoid  errors;  Difference  between  a  given  minu- 
end and  several  subtrahends;  Balancing  an  account;  Complement  method; 
Subtracting  on  an  adding  machine;  Practical  problems;  Multiplication; 
Contractions  in  multiplication;  To  multiply  by  factors  of  the  multiplier; 
To  multiply  when  a  part  of  the  multiplier  is  a  factor  or  multiple  of  another 
part;  To  multiply  a  number  of  two  figures  by  11 ;  To  multiply  any  number 
by  11;  Multiplying  by  25;  Multiplying  by  15;  Multiplying  numbers  ending 
with  ciphers;  Multiplication  by  numbers  near  100,  and  by  numbers  near 
1,000;  Multiplication  of  two  numbers  each  near  100,  1,000,  and  so  forth; 
Multiplying  by  numbers  a  little  larger  than  100;  Multiplication  of  two 
numbers  each  a  little  more  than  100;  Cross  multiplication;  To  cross-multi- 
ply a  number  of  three  digits  by  a  number  of  two  digits;  To  cross-multiply 
a  number  of  three  digits  by  another  number  of  three  digits;  Preparation  of 
a  table  of  multiples  of  a  number;  Division;  To  divide  by  25,  50,  or  125; 
Abbreviated  division;  Use  of  tables  in  division;  Reciprocals  in  division. 

2.  CHECKING  COMPUTATIONS 29 

Methods;  Rough  check;  Absolute  check;  Check  numbers  obtained  by 
casting  out  the  nines:  Verification  of  addition;  Verification  of  subtraction; 
Verification  of  multiplication;  Verification  of  division;  Verification  of 
division  where  there  is  a  remainder.  Check  numbers  obtained  by  casting 
out  the  elevens:  Verification  of  addition;  Verification  of  subtraction; 
Verification  of  multiplication;  Verification  of  division.  Check  number 
thirteen. 

3.  FACTORS  AND  MULTIPLES 35 

Factors;  Tests  of  divisibility;  Greatest  common  divisor;  Least  common 
multiple;  Cancellation. 

4.  COMMON  FRACTIONS 39 

Terms  explained;  Reduction  of  fractions;  Principle;  Mixed  numbers;  To 
change  a  mixed  number  to  an  improper  fraction;  Addition  and  subtraction 
of  fractions;  Multiplication  of  fractions;  Division  of  fractions;  To  find  the 
product  of  any  two  mixed  numbers  ending  in  J ;  To  multiply  a  mixed  num- 
ber by  a  mixed  number;  Decimal  fractions;  Addition  and  subtraction;  Mul- 
tiplication; Division;  To  abbreviate  decimal  multiplication  when  a  given 
number  of  decimal  places  is  required;  Division  of  decimals;  To  change  a 
decimal  fraction  to  an  equivalent  common  fraction;  To  change  a  common 
fraction  to  a  decimal;  Aliquot  parts;  The  use  of  aliquot  parts;  Multiplica- 
tion by  aliquot  parts;  Division  by  aliquot  parts. 


vi  CONTENTS 

CHAPTEB  PAGH 

5.  PERCENTAGE 53 

Relation  between  percentage  and  common  decimal  fractions;  Applica- 
tions; Definitions;  Fundamental  processes;  Computations;  Daily  record 
of  departmental  sales ;  Per  cent  of  returned  sales  by  departments ;  Clerk's 
per  cent  of  average  sales;  Per  cent  of  income  by  source;  Per  cent  of  expense; 
Per  cent  of  increase  or  decrease;  Operating  statistics;  Budgeting;  Profits 
based  on  sales;  Marking  goods;  Commissions. 

6.  COMMERCIAL  DISCOUNTS 71 

Cash  discount;  Trade  discount;  Single  discount  equivalent  to  a  series; 
To  find  the  net  price;  Transportation  charges  on  discount  invoices; 
Anticipation. 

7.  SIMPLE  INTEREST 77 

Definition;  Short  method  of  calculating;  Sixty-day  method;  Method 
using  aliquot  parts;  The  cancellation  method;  Dollars-times-days  method, 
6%;  Interchanging  principal  and  time;  Exact  or  accurate  interest;  Accu- 
mulation of  simple  interest;  Symbols;  Simple  amount;  Rate;  Time;  Present 
worth;  Comparison  of  simple  amount  and  simple  present  worth;  True 
discount. 

8.  BANK  DISCOUNT 87 

Definition;  Counting  time;  Finding  the  difference  between  dates  by  use 
of  a  table;  Proceeds;  To  find  bank  discount  and  proceeds;  To  find  the  face 
of  a  note  when  the  proceeds,  time,  and  rate  of  discount  are  given. 

9.  PARTIAL  PAYMENTS 91 

Partial  payments  on  debts;  Methods;  United  States  Rule;  Merchants'  Rule. 

10.  BUSINESS  INSURANCE 95 

Kinds  of  insurance;  Policy;  Fire  insurance;  Form  of  policy;  Rates;  To  find 
the  premium;  Agent's  commission;  Cancellation  of  policies;  Coinsurance; 
Use  and  occupancy  insurance;  Group  life  insurance;  Health  insurance; 
Workmen's  compensation  insurance. 

11.  PAYROLL  RECORDS  AND  PROCEDURE 107 

Requirements;  Payroll  procedure;  Timebooks;  Time-clock  cards;  Deduc- 
tions; Withholding  exemptions;  Payroll  sheets;  Piecework  system;  Pay 
checks;  Pay  envelopes  and  receipts;  Coin  sheet  and  currency  memorandum. 

12.  AVERAGE 123 

Simple  average;  Moving  averages;  Progressive  average;  Periodic  average; 
Weighted  average. 

13.  AVERAGING  DATES  OF  INVOICES 131 

Definition;  Use;  Term  of  credit;  Average  due  date;  Focal  date;  Methods; 
Rule  for  product  method. 

14.  EQUATION  OF  ACCOUNTS,  OR  COMPOUND  AVERAGE  .   .   .   135 

Definition;  Rule  for  the  product  method;  When  to  date  forward  or 
backward. 


CONTENTS  vii 

CHAPTER  PAGL 

15.  ACCOUNT  CURRENT 139 

Definition;  Methods. 

16.  STORAGE 141 

Definition;  Running  account. 

17.  INVENTORIES 143 

Valuation  of  inventories;  Cost  or  market,  whichever  is  lower;  Average  cost 
method;  "First-ill,  first-out"  method  of  inventory;  "Last-in,  first-out" 
method  of  inventory;  Merchandise  turnover;  Number  of  turnovers;  Per 
cent  of  mark-down  to  net  cost;  Computation  of  inventory  by  the  retail 
method;  Determining  the  ratio  of  cost  to  retail. 

18.  GROSS  PROFIT  COMPUTATIONS 155 

Gross  profit;  Rate  per  cent  of  gross  profit;  Procedure;  Uses;  Cost  of  goods 
sold;  Rate  per  cent  of  cost  of  sales;  Fire  Losses;  Use  of  gross  profit  test  in 
verification  of  taxpayer's  inventory;  Installment  sales  of  personal  property; 
Computation  of  gross  profit ;  Reserve  for  unearned  gross  profit ;  Bad  debts ; 
Deferring  income;  its  effect  on  tax. 

19.  ANALYSIS  OF  STATEMENTS 169 

Financial  and  operating  ratios;  Costs,  expenses,  and  profits;  Ratio  of  gross 
profit  to  net  sales;  Ratio  of  operating  profit  to  net  sales;  Ratio  of  net  profit 
to  net  sales;  Ratio  of  operating  profit  to  total  capital  employed;  Ratio  of 
net  profit  to  net  worth;  Earnings  on  common  stockholders'  investments; 
Working  capital  ratio ;  Sources  of  capital;  Manner  in  which  capital  is  invest- 
ed; Turnover  of  total  capital  employed;  Turnover  of  inventories;  Turnover 
of  accounts  receivable;  Turnover  of  fixed  property  investment. 

20.  PARTNERSHIP 181 

Definition;  Mathematical  calculations;  Goodwill;  Profit-sharing  agreements; 
Lack  of  agreement;  Losses;  Arbitrary  ratio;  Ratio  of  investment;  Division 
of  profits  by  first  deducting  interest  on  capital;  Profits  insufficient  to  cover 
interest  on  investment;  Adjustments  of  capital  contribution;  Profit  sharing 
in  ratio  of  average  investment;  Liquidation  of  partnership;  Methods; 
Total  distribution;  Periodic  distribution. 

21.  GOODWILL 201 

Definition;  Basis  of  valuation;  Earning  power  determined  from  profit  and 
loss  statements;  Methods  of  valuing  goodwill;  Case  illustrations;  Valuation 
by  appraisal;  Valuation  by  number  of  years'  purchase  price  of  net  profits; 
Valuation  on  basis  of  excess  of  profits  over  interest  on  net  assets;  Basis  of 
stock  allotment;  Common  stock  only;  Preferred  stock  for  net  assets;  Bonds 
preferred  stock,  and  common  stock. 

22.  BUSINESS  FINANCE 213 

Stock  rights;  Sale  of  stock  and  rights,  federal  income  tax;  Working  capital; 
Cumulative  voting;  Book  value  of  shares  of  stock;  Profits  distribution. 

23.  PUBLIC  FINANCE  AND  TAXATION 223 

Governmental  functions;  Purposes  of  taxes;  Appropriations;  Kinds  of 
taxes.  Property  Tax:  Determination  of  tax  rate;  To  find  the  amount 
of  tax. 


Viii  CONTENTS 

CHAPTER  PAGE 

24.  FUNDAMENTALS  OF  ALGEBRA '  .    .    .    .  229 

Explanation ;  Symbols  and  terms ;  Positive  and  negative  numbers ;  Addition 
of  positive  and  negative  numbers;  The  coefficient;  Parentheses,  brackets, 
and  braces;  Subtraction;  Multiplication;  Division. 

25.  EQUATIONS 235 

Simple  equations;  Fractions;  Clearing  of  complex  fractions;  Simultaneous 
equations  with  two  or  more  unknowns;  Arithmetical  solution  of  problems 
containing  unknown  quantities. 

26.  LOGARITHMS 249 

Uses  of  logarithms;  Exponents;  Parts  of  a  logarithm;  Characteristic;  Posi- 
tive characteristic;  Negative  characteristic;  Mantissa;  How  to  use  a  table 
of  logarithms;  To  find  a  number  when  the  logarithm  is  given;  To  find  a 
number  whose  mantissa  is  not  in  the  table;  Rules  for  computation  by  loga- 
rithms; Multiplication  by  logarithms;  Division  by  logarithms;  Powers  of 
numbers;  Process  with  a  negative  characteristic;  Roots  of  numbers,  Process 
with  negative  characteristics;  The  slide  rule;  Use  of  slide  rule;  Accuracy  of 
calculations  made  by  the  slide  rule;  Theory  of  the  slide  rule;  How  to  learn  to 
use  the  slide  rule;  Reading  the  slide  rule;  Construction  of  model  slide  rule; 
Multiplication  on  the  slide  rule;  Division  on  the  slide  rule. 

27.  GRAPHS  AND  INDEX  NUMBERS 265 

Charts  and  graphs;  Circle  chart;  Comparison  of  circles;  Bar  chart;  Line  or 
curve  chart;  Rules  for  coordinate  charts;  Logarithmic  chart;  Ratio  charts. 
Index  Numbers:  Uses  of  index  numbers;  Index  numbers;  Economic  position 
of  agriculture;  Construction  of  index  numbers;  Composite  price  indexes; 
Weighted  index  numbers ;  Farm  evaluation  on  the  basis  of  crop  production 
index;  Computation  of  the  crop  production  index. 

28.  PROGRESSION 285 

Definition ;  Increasing  series ;  Decreasing  series ;  Arithmetical  progression ; 
Symbols;  Relation  of  elements.  Increasing  Series:  To  find  the  number  of 
terms ;  To  find  the  first  term ;  To  find  the  last  term ;  To  find  the  common  dif- 
ference; To  find  the  sum.  Decreasing  Series :  To  find  the  number  of  terms; 
To  find  the  first  term;  To  find  the  last  term;  To  find  the  common  difference; 
To  find  the  sum.  Geometrical  Progression :  Elements ;  Increasing  series ;  To 
find  the  first  term;  To  find  the  last  term;  To  find  the  sum;  To  find  the 
ratio;  Decreasing  series;  To  find  the  first  term;  To  find  the  last  term;  To 
find  the  sum;  To  find  the  ratio;  Progression  problems  solved  by  the  use  of 
logarithms. 

29.  FOREIGN  EXCHANGE 293 

Foreign  trade;  Rate  of  exchange;  Par  of  exchange;  Current  rate  of 
exchange;  Six  classes  of  problems;  Conversion  of  one  monetary  unit  into 
terms  of  another;  Conversion  of  decimals  of  one  monetary  unit  into  mone- 
tary units  of  a  smaller  denomination;  Interest  on  foreign  exchange;  To  find 
the  value  of  a  time  bill  of  exchange;  Foreign  exchange  accounts;  Averaging 
accounts  in  foreign  exchange;  Conversion  of  foreign  branch  accounts. 

PART  II 

30.  COMPOUND  INTEREST 311 

Compound  interest;  Compound  interest  method;  Actuarial  science;  Sym- 
bols: Principal;  Time;  Rate;  Ratio  of  increase;  Compound  amount  tables; 


CONTENTS  •* 

CHAPTER  '  PAGE 

30.  COMPOUND  INTEREST  (Cont.} 

Calculation  of  compound  amount;  Compound  amount  of  given  principal; 
Compound  interest;  Results  of  frequent  conversions  of  interest;  Nominal 
and  effective  rates;  Effective  interest;  Compound  present  worth;  Com- 
pound discount;  Rate;  Time;  Compound  amount  for  fractional  part  of  con- 
version period. 

31.  ORDINARY  ANNUITIES 327 

Definition ;  Kinds  of  annuities ;  Rent  of  an  annuity ;  Amount  of  an  ordinary 
annuity ;  Analysis  of  compound  interest ;  Relation  of  compound  interest  and 
annuities ;  Procedure  in  computing  the  amount  of  an  annuity ;  Semiannual 
or  quarterly  basis;  Rent  of  ordinary  annuity;  Use  of  effective  interest  in 
annuities;  Sinking  fund  contributions;  Present  value  of  an  ordinary 
annuity;  Amortization;  Computation  of  the  rents  or  periodic  payments  of 
the  present  value  of  an  ordinary  annuity ;  Payment  of  debt  by  installments ; 
Computation  of  the  term  of  an  annuity;  Use  of  effective  rate  in  annuities; 
Computation  of  the  rate  of  an  annuity;  Selection  of  rate  by  calculation  of 
amounts  of  annuities;  Solution  of  annuity  problem  with  limited  data. 

32.  SPECIAL  ANNUITIES 349 

Annuity  due;  To  find  the  amount  of  an  annuity  due;  Present  value  of  an 
annuity  due;  Comparison  of  present  value  of  an  ordinary  annuity  and  that 
of  an  annuity  due ;  To  find  the  present  value  of  an  annuity  due ;  Rents  of  the 
amount  of  an  annuity  due;  Rent  of  the  present  value  of  an  annuity  due; 
Effective  interest  on  annuity  due;  Deferred  annuity;  Perpetuity;  Perpetui- 
ties payable  at  intervals  longer  than  a  year. 

33.  BOND  AND  BOND  INTEREST  VALUATION 367 

Definitions;  Bonds  sold  at  par;  Bonds  purchased  at  a  discount  or  at  a  pre- 
mium; Price  and  rate  of  yield;  Use  of  bond  tables;  Bond  table,  first  form; 
Bond  table,  second  form;  Interpolating  in  bond  tables;  Bond  values  com- 
puted without  tables;  Bonds  sold  at  a  discount;  Bonds  sold  at  a  premium; 
Values  of  bonds  between  interest  dates ;  Interest  accrued  between  interest 
dates ;  Bond  discount  or  premium  between  interest  dates ;  Illustration  of  the 
practical  process  of  calculating  the  value  of  a  bond  bought  at  a  discount ; 
Theoretical  procedure  illustrated;  Illustration  of  the  practical  process  of 
calculating  the  value  of  a  bond  bought  at  a  premium;  Bonds  bought  on 
a  yield  basis ;  Bonds  to  be  redeemed  above  par ;  Serial  redemption  bonds ; 
Frequency  of  redemption  periods;  Alternative  solution;  Bonds  redeemed 
by  other  than  equal  annual  payments;  Bonds  redeemable  from  a  fund; 
Effective  rate  of  interest  on  bonds ;  Effective  rate  of  interest  on  bonds  sold 
at  a  premium;  Effective  rate  on  bonds  sold  at  a  discount;  Computation 
when  bond  table  is  not  available ;  Approximation  by  averages ;  Amortization 
of  discount,  premium,  or  discount  and  expense  on  serial  redemption  bonds; 
Bonds  outstanding  method ;  Scientific  method. 

34.  ASSET  VALUATION  ACCOUNTS 40£ 

Asset  valuation;  Depreciation;  Depletion;  Depreciation  methods;  Straight- 
line  method;  Working-hours  or  unit-product  method;  Sum-of-digits 
method;  Sinking-fund  method;  Annuity  method  of  depreciation;  Fixed- 
percentage-of-diminishing-value  method;  Composite  life;  Depletion;  Cal- 
culation of  depletion;  Capitalized  cost;  Perpetuity  providing  for  ordinary 
annual  expenses  and  for  replacement  of  asset;  Capitalization  of  a  wasting 
jsset. 


x  CONTENTS 

CHAPTER  PAGE 

35.  BUILDING  AND  LOAN  ASSOCIATIONS 425 

Control;  Classes  of  stock;  Withdrawal  of  funds;  Plans  of  organization; 
Terminating  plan;  Serial  plan;  Dayton  or  Ohio  plan;  To  find  the  time 
required  for  stock  to  mature  (rate  of  interest  given) ;  To  find  the  effective  , 
rate  of  interest  on  money  invested  in  installment  shares. 

36.  PERMUTATIONS  AND  COMBINATIONS 439 

Permutation;  Number  of  ways  of  doing  two  or  more  things  together; 
Combinations. 

37.  PROBABILITY 445 

Probability;  Permutations  and  combinations  in  probability;  Compound 
events;  Independent  events;  Mutually  exclusive  events;  Empirical 
probability. 

38.  PROBABILITY  AND  MORTALITY 453 

Life  insurance;  Mortality  table;  Notation;  Probability  of  living;  Proba- 
bility of  dying;  Joint  life  probabilities. 

39.  LIFE  ANNUITIES 459 

Factors  involved;  Pure  endowment;  Life  annuity;  Commutation  columns; 
Life  annuities  due;  Use  of  commutation  table;  Deferred  annuity;  Deferred 
We  annuity  due;  Temporary  life  annuities;  Temporary  annuities  due;  Life 
annuities  with  payments  m  times  a  year;  Forborne  temporary  annuity  due. 

40.  NET  PREMIUMS 469 

Net  single  premium;  Annual  premiums;  Term  insurance;  Annual  premium 
for  term  insurance;  Net  single  premium  for  endowment  insurance;  Annual 
premium  for  endowment  insurance. 

41.  VALUATION  OF  LIFE  INSURANCE  POLICIES 473 

Mortality  and  the  level  premium;  Policy  reserves;  Interest  and  the  pre- 
mium; Loading;  Dividends  and  net  cost;  Terminal  reserves;  Retrospec- 
tive method;  Transformation;  Reserve  valuation  for  limited  payment  life 
insurance;  Preliminary  term  valuation. 

APPENDIX                                   APPENDIXES 
I.  Practical  Business  Measurements 481 

II.  Tables  of  Weights,  Measures,  and  Values 489 

III.  Tables  .   .    .    .  • 497 

INDEX 540 


PARTI 


CHAPTER(lp 

Fundamental  Processes  and  Short 
Methods  for  the  Accountant 

Addition.  Addition  is  the  process  of  combining  numbers  of  the 
same  denomination.  Quantities  of  such  unlike  measures  as  dollars 
and  yards  cannot  be  added;  but  quantities  like  yards,  feet,  &i\dinches 
can  be  changed  to  like  numbers  and  then  added.  Like  numbers  are 
numbers  that  express  the  same  kind  of  units.  The  sum  is  the 
number  resulting  from  adding  two  or  more  like  numbers,  and  the 
addends  are  the  different  numbers  to  be  added. 

Addition  is  the  most  fundamental  of  all  numerical  operations. 
It  is  essential  that  the  clerk,  the  businessman,  arid  the  accountant 
be  able  to  add  with  precision  and  rapidity.  The  ability  to  recog- 
nize the  sums  of  numbers  instantly  is  acquired  by  constant  practice 
and  careful  study. 

Drill  tables.  Practice  adding  the  columns  of  numbers  in  the 
following  table  until  you  can  complete  the  operation  in  twenty-five 
seconds,  without  error.  State  sums  only;  that  is,  do  not  repeat  the 
numbers  to  be  added. 

581654596729489* 
1?1<*322.  747l'6462 

796375943959848  • 
49^125432845817 

786689278876379 


Practice  stating  the  sums  of  the  following  columns  of  numbers 
until  you  can  do  all  of  them  correctly  in  less  than  two  and  a  half 
minutes. 

34234335576523538 
22233323323322232 


[Drill  iable  continued  on  next  page.] 
3 


FUNDAMENTAL  PROCESSES  AND  SHORT  METHODS 

567232-72847446332 
431222222-32,422222 
668.9  7.  68796754  6'  635 

45849635345484568 
4432123.  2232212323 


67457576886346396 
54226414413331321 
676988798995-79998 

95886487865577776 
8,28633567  £.5  5  375464 


76747589645695966 
44314644346^55551 


78879657689727445 
3  4  5  5365467652  3<  '4  .4  5 


Streamline  addition.     Omit  unnecessary  words:  that  is,  do  not 
name  the  number  to  be  added;  name  only  the  sum. 

6  In  the  example  at  the  left,  a  common  way  of  adding  would  be  (com- 

7  mencing  at  the  top):  5  and  7  are  12,  12  and  8  are  20,  20  and  4  are  24, 

8  arid  so  forth.     Instead  of  adding  in  this  manner,  proceed  to  the  answer 
4        by  saying  (mentally),  "  12,  20,  24,  27,  29,  38." 

3 
2 


Drill  table. 

3     4267     5  .1835 

2    5    9    8    9.    3    4  .  7    2    8    ' 

7377268671 

40  6.  9586963 

5933872499 

6842625746, 

1584397537 

Combinations  whose  sum  is  10.  Combinations  of  two  or  more 
numbers  whose  sum  is  10  are  of  frequent  occurrence.  When  these 
combinations  are  recognized,  addition  may  be  shortened  by  adding 
such  combinations  as  10. 


FUNDAMENTAL  PROCESSES  AND  SHORT  METHODS        5 

4  In  this  example,  the  addition  may  be  performed  as  follows:  (com- 
7,        mencing  at  the  top)    14,  16,  21,  30,  40. 

3  Or,  it  may  be  added  in  this  manner:  11,  21,  30,  40. 

2  Do  not  try  to  form  combinations.     Unless  they  are  instantly  recog- 

5  nized,  add  the  numbers  in  the  regular  manner. 
9 

5) 

3 

J? 
40 

Drill  table. 


7 
2 
1 
5 
6 
4 
9 
8 
2 

3 

5 
6 
4 
,9 

8 
2 

7 
4 

,5 
fr. 
2 

7.' 
1 
4< 
8 
2. 
5 

3 

8 
2 

7 

3 

1 

8 

7 

2 

5 

4 

5 
2 
3 

) 
\ 

6 

!) 

lj 

7, 

2/ 
9 

4 

£ 

3^ 

1 
5 
9" 

8 
3 

7 

4* 
2. 

9 

7' 
3 
6 
3 
t 
2 
5 

2 

7 
4 
5" 
1. 
7. 
5 
5 
9 

Adding  where  the  same  number  is  repeated  many  times.     In 

obtaining  averages,  in  adding  statistics,  and  in  other  work  involv- 
ing addition,  often  the  same  number  is  repeated  many  times.  Use 
multiplication  to  save  time  in  adding. 

724  In  this  example,  7  occurs  four  times  and  6  occurs  three  times  in  the 

785         third  column.     The  sum  of  the  third  column  may  be  found  as  follows: 

773  Carried                  4 

748  4X7                 28 

696  3X6                 18 

687  «o 

679  5U 


5092 

-7 
^ 

The  work  is  actually  performed  mentally  thus:  4  (28)  32,  (18) 
50.  Where  the  columns  are  long,  a  side  calculation  may  be 
necessary. 

Drilftable.  .  .      ..  ,  ,  , 


68 
63 
64 
,67 
59 
54 
57 
•IS 

284 
273 
281 
311 
314- 
321 
318 
W 

V4 
33 
32 
31 
34 
32 
33 
36 

.86 
.75 
86 
.29 
36 
.75 
.95 

W 

23, 
23. 
24. 
25 
26 
31 
32, 
33 

56 
95 
72 
31 
54 
72 
69 
47 

47. 
39 
38. 
45 
39 
42. 
38,. 

56 

85 
64, 
58. 
95 
74 
56 

4l. 

72, 
72 
69 

68 
67 
71. 

53 
37 

48 
95 
83 
44 
,93 
59 

•-1  0  CLA 


FUNDAMENTAL  PROCESSES  AND  SHORT  METHODS 


Group  addition.  The  most  practical  method  of  adding  is  to 
group  or  combine  two  or  more  figures  mentally,  and  to  name 
results  only. 


5 

4 
7 
3 
8 
6 
1 

J2 
36 


Mental  Steps 


10     19 


14 


36 


Drill  tabl 

t    -v 

/ 


Instead  of  saying,  "5  and 
4  are  9,  and  7  are  16,  and  3 
are  19,  and  8  are  27,  and  6 
are  33,  and  1  are  34,  arid  2  are 
30,"  simply  think,  ''9,  19, 
33,  36." 


ib 


t 

6  -'4  8  5  3"  7 ;  0*  3  8  6  4  5  7  3j  8  7  4  2  9 
3652487632576393847 
7'  3  6  2  9  8-  4  6  7  4  5  7  6  2  4-  •  6  9  7  4 
Z  6  3.  5875325973846827 
8  2  4  .8  3  7  4  6.  5  4,  9  7  6  3  5  8  7  3  6 
386247  8  635863476248 
5737358237634862359 
437427864-7648395821 
55998636777753421  73 


Addition  of  two  columns  at  a  time.     Two  columns  of  figures  may 
he  added  at  the  same  time,  as  shown  in  the  following  illustration: 


Mental  titeps 


56     Tens 


Units 


Tens 


Explanation 


Units 


28(7)  76  (2)  84 

43  (3)  124  (4)  127 

_21    (5)  147  (6)  148 

148 


(1)  56  and  20  =  76  (2)  76  and  8  =  84 
(3)  84  and  40  =  124  (4)  124  and  3  =  127 
(5)  127  and  20  =  147  (6)  147  and  1  =  148 


Drill  table. 


s 


^  ^ 

79  82  24  37  65  39,  28  28. 

48  84  33  44  81  58  39  59- 

81  95  46  53  42  48  23  86 

1£  83  52  66  73  73  37  63 


Recording  addition  by  columns.  ^Accountants  are  subject  to 
interruptions,  but  the  time  required  to  re-add  a  column  of  figures 
for  the  purpose  of  picking  up  the  carrying  figure  may  be  saved  if 
the  total  of  each  column  is  recorded  separately.  The  separate 
column  totals  are  also  convenient  to  use  in  checking  the  work  ;  for 
instance,  if  in  a  final  summary  of  additions  there  is  an  error  of 
$100.00,  the  hundreds'  columns  of  the  subtotals  may  be  verified 
quickly  without  the  necessity  of  re-adding  all  the  columns. 


FUNDAMENTAL  PROCESSES  AND  SHOR1  METHODS         7 

Fxample — Method  1    Example — Method  2    Example— Method  3 

4572  4572  4572 

39S6  3986  39S6 

2173  2173  2173 

5911  5911  5911 

2765  2765  2765 

4937  4937  4937 

24  24                 24 

32  ^            34                 34 

40  43                  43 

20 24  24 

24344  24344 

Explanation  1.  Add  each  column  separately,  setting  the  sums  one  place  to 
the  left,  as  in  the  example.  After  the  last  column  has  been  added,  add  the 
individual  sums  in  regular  order;  that  is,  from  right  to  left. 

Explanation  2.  In  Method  2,  a  little  time  is  saved  by  adding  to  each  column 
the  number  carried  from  the  column  at  the  right. 

Explanation  3.  Method  3  differs  from  Method  2  in  the  writing  of  the  columns' 
sums.  It  is  somewhat  easier  to  write  the  sums  one  below  the  other.  This 
cannot  be  done  in  Method  1  because  carrying  figures  are  not  used,  and  another 
step  is  required  to  complete  the  answer:  that  is,  finding  the  grand  total  of  the 
units,  tens,  hundreds,  and  so  forth. 

A  modification  of  the  third  method  is  useful  in  adding  columns  of  dollars 
and  cents. 

$    644  22  The  total,  $4,062.08,  is  obtained  by  adding  each  column  sepa- 

821    94         rately    as    explained    under    Method    3.     The    computation    will 
314  26         appear  as  follows,  the  purpose  of  the  horizontal  lines  being  to 
712  84        separate  cents  from  dollars,  and  hundreds  from  thousands. 
976.54 
592  28 

$4~062~08 

Sum  of  the  first  column 28 

Bum  of  the  second  column,  28  plus  2,  carrying  number     .       .  .  30 

Sum  of  the  third  column,  19  plus  3,  carrying  number 22 

Sum  of  the  fourth  column,  24  plus  2,  carrying  number       .      .  26 

Sum  of  the  fifth  column,  38  plus  2,  carrying  number     40 

As  there  are  no  more  columns,  write  the  carrying  number 4 

The  total,  $4,062.08,  is  obtained  by  reading  the  numbers  at  the  right,  com- 
mencing at  the  bottom. 


y 

> 

,  l.s 

-  ;/5273 
/fc2191 
,7  8437 
"~,v  3426 
,«!  7139 
'  >s7895 

/ 

2. 

5126 
8497 
7934 
9783 
9126 
8751 

^x 

7952 
2975 

8675 
8437' 
2975' 
3826 

/ 

4. 

1395 
2764 
8351 
6248 
5347 
4586 

Problems 
'V?*    6' 

36S8  $367 
4932,  421. 
7A63;  281  . 
2898  •  633 
6598   855. 
887*7   769 

98 
74 
34 
46 
91 
25 

v 

$786 
518 
946, 
881. 
542: 
787. 

42 
49 
72 
92, 
37 
6ft 

8. 

$498. 
822 
753 
629, 
367. 
521 

57 
56 
86 
75 
43 
.54 

8          FUNDAMENTAL  PROCESSES  AND  SHORT  METHODS 

Practice  Problems 

Average  weekly  earnings  from  payroll  reports. 


^  ^f 

2. 

<<•  fc 

\t 

4. 

VK-     a 
<#  0.7     b. 

-  17;' 

8. 

^33  20 

35  72 

30 

85 

33.20 

28 

28 

37 

61 

22 

.53 

23.25 

25.13 

29  88 

21. 

.99 

35  72 

28 

24 

29 

.97 

35 

62 

35  25 

37.41 

39  24 

35 

31 

42  28 

22 

61 

31 

.36 

16 

.22 

32  18 

31  65 

33  47 

28 

89 

31.56 

21 

46 

21 

.34 

31 

.91 

37  17 

31.40 

35.29 

27 

.89 

29.82 

23 

.91 

33, 

.34 

14 

.16 

35  99 

22  93 

23.22 

19 

.11* 

37.33 

22 

41 

31, 

.78 

27 

.14 

36  66 

32.16 

33.49 

19. 

.15 

28  97 

17 

99 

34 

73 

31 

62 

35  96 

26  37 

28  34 

18. 

.96. 

54.61 

25 

21 

26 

74 

33 

84 

37  37 

36  52* 

33.64 

30 

73' 

39  06 

33. 

25 

22 

88 

30 

89 

18.17 

32  05  , 

34  60 

30. 

70 

28.29 

28 

72. 

28 

09 

39 

04 

29  64 

32  58. 

26  49 

18. 

,95 

26  87 

19 

49- 

28 

20 

15 

76 

15  82 

23  60 

28.81 

33, 

,45' 

27.01 

18. 

64 

20 

80 

20 

87^ 

29  87 

23  44 

37.92 

31 

(TO 

39.52 

37. 

12 

37 

53 

38 

72 

27  84 

36  37 

41.54 

30 

41" 

41.86 

31. 

04 

28 

94 

37. 

16  , 

36  83 

Tabulation  of  advertising  lineage. 


7,  ^\9.  , 

10. 

^tQLlj 

12. 

'  "-I?-, 

14. 

J>  "^5.^ 

16. 

26,228 

29,207 

22,107- 

14,849 

10,049 

57,104 

13,022 

10,755 

13,818 

17,588 

1  5,977-  • 

'11,966 

14,745 

71,075 

15,223 

16,850 

27,122- 

28,267 

39,082" 

36,021 

8,562 

119,035 

17,058- 

15,573 

17,077 

15,095 

9,644 

7,888 

5,575 

28,857 

18,048- 

10,259 

32,094- 

36,072 

23,449» 

19,634 

12,376 

39,190 

26,174 

19,635 

32,936 

32,835 

18,930 

15,033 

2,175 

16,085 

28,169 

24,572 

21,499 

18,116 

46,520 

43,778 

7,531 

15,484 

14,949 

15,057 

20,655 

24,094 

25,140 

19,271 

8,650 

28,192 

24,478  ' 

19,445 

22,338 

10,365 

8,015  4 

13,412 

15,530 

14,711 

22,175 

24,493 

13,412 

60,475 

38,795 

93,323 

23,680 

22,865 

23,680 

37,335 

Addition  of  dollars  and  cents,  irregular  items. 


f  t. 


'jVfl-S   is.   0\j$r4 

20.    *|21£t) 

22. 

13p. 

10 

86 

.35 

80 

45. 

40 

$6 

.35 

1 

,955.05 

12. 

65 

52 

.67 

44 

.82 

34 

20 

*'  52* 

67- 

531.03 

10 

57 

44 

00 

37 

45 

28 

66 

42 

57 

442  .  85 

50. 

05 

208 

.33 

127 

.29 

135 

68 

208 

33 

2 

,148  74 

1,275. 

48 

394 

.68 

4,151 

.36 

945 

21 

878 

52k 

30 

,149  39 

260, 

73 

64 

.72 

24.94- 

2 

.72 

111 

56 

112  64 

7, 

81 

6 

.29 

.72 

118, 

.61 

,6 

46 

509.74 

78, 

.13 

27 

.33 

71 

.97 

.75 

44 

.77 

27.02 

2 

.50 

.62 

12 

.09 

32 

.49 

.69 

153  07 

111 

82 

27 

.65 

.35' 

8 

.58 

3 

07 

1 

,512.34 

29 

.53 

7 

.33 

160 

.31- 

33 

55 

70 

63 

1 

,002.90 

54 

.53: 

16 

.29 

45 

15: 

8 

45 

5 

37 

282  51 

15 

.36- 

4 

.59 

128 

.60 

24 

.85 

35 

,15 

66  66 

147 

62* 

111 

52 

41 

.51. 

138 

.34 

9.98 

146.43 

5 

27 

59 

.68 

46 

.43 

92 

.54 

214 

34 

641.51 

FUNDAMENTAL  PROCESSES  AND  SHORT  METHODS 


Practical  applications.  In  the  following  problems  will  be  found 
examples  of  business  records  requiring  addition  for  the  completion 
of  the  record. 

Problem  1 

In  this  problem,  cash  register  tapes  provided  the  source  of  the  entries  on 
Form  1.  As  the  sales  were  registered,  the  classification  was  imprinted  on  the 
tape.  At  the  end  of  the  day,  the  classified  items  appearing  on  the  tape  were 
accumulated  on  Form  1,  and  the  totals  transferred  to  Form  2.  At  the  end  ot 
the  week,  Form  2  was  added;  at  the  end  of  the  month,  the  weekly  totals  were 
accumulated  to  monthly  totals.  Thus,  sales  for  the  month  were  analyzed  by 
departments  or  classes. 

Add  the  columns  on  Form  1  (Saturday's  sales),  transfer  the  totals  to  Form  2, 
and  find  the  total  sales  for  the  week. 

Form  1 


Candy 

Cigars 

Soda 

Drugs 

Own 
Remedies 

Patent 
Medicines 

Toilet 
Articles 

X3? 

.10 

.10 

.45 

75 

1.25 

.35 

1.25 

.25 

.15 

1.64 

.45 

.50 

1.15 

.80 

.15 

.20 

.10 

.15 

1   50 

.80 

.45-- 

.25 

.45 

.75* 

1.25 

.89 

2.65 

.75* 

.50 

.10 

1  50 

.90 

33 

.75 

90 

25. 

.50 

U75 

1.65 

2,35 

1.85 

W 

/•'" 

1,0' 

*<\ 

4  .  />,    ~ 

£    v    <- 

Iff 

Form  2 


Day 

Candy 

Cigars 

Soda 

Drugs 

Own 

Heine- 

•t  • 

Patent 
Medi- 

Toilet 
Articles 

Totals 

^ 

.^ 

dies 

cines 

IVlon. 

12  65 

19  15 

T 
3  95 

27.63 

Vis 

/9  85 

5.00 

^4( 

Tues. 

8  50 

16.10 

6  80 

33  98 

2.47 

12.20 

3  65 

..S^  70 

Wed. 

11  25 

8  75 

4  50 

15.20 

1  75 

2  55 

10.45 

...$*.'£>" 

Thurs. 

9  65 

4  25 

2.55 

7  65 

2  85 

4  86 

4.63 

..2&.ifv 

Fri. 

10  35 

5.55 

3.75 

12.84 

3.68 

5.49 

3.85 

...j*»\5f  J 

Sat. 

..4<<;:<? 

I'^O 

(•& 

1^1$ 

47/4 

£•£'£ 

7':Ty 

"  jryjir 

1>*C 

s.^. 

&A- 

(&-.n 

*"*' 

ai..o 

^.^ 

^,lf 

Form  2  is  self-proving — that  is,  the  sum  of  the  daily  totals  must  equal  the 
sum  of  the  departmental  or  classification  totals. 

Problem  2 

The  "peg  board'7  is  used  for  accumulating  numbers  having  to  do  with  many 
kinds  of  information.  The  numbers  are  entered  on  narrow  forms  which  are 
attached  to  the  "peg  board."  The  forms  are  held  in  place  and  cross  extension 
as  well  as  "footings"  are  thus  permitted. 


10       FUNDAMENTAL  PROCESSES  AND  SHORT  METHODS 


In  the  following  example,   this  arrangement  is  used  to   accumulate  total 
departmental  sales  made  by  a  salesman. 


Salesman 
R.  F. 

Salesman 
R.  F. 

Salesman 
R.  F. 

Salesman 
R.  F. 

Salesman 

u.  F.       rotaz  »Srtte« 

Dept. 

,f>qte  4/2 

/j^  4/3 

JCtyte  4/4 

£)it^  4/5 

7^%  y6 

138  &7 

.587  .  23 

;347  58 

£o  f    0  1 

f  637  82         

1 

645.39 

321.69 

123  63 

563  85 

495  71                  .....      . 

2 

362  45 

847.86 

219  23 

149  27 

826  45 

3 

472  31 

123.45 

547  81 

462  38 

718  26 

4 

45  97 

671.17 

359  34 

326  49 

534  58 

5 

273.14 

372  45 

135  67 

857  62 

149  17 

6 

928,.  63 

436  49 

569  81 

318  48 

529  32 

7 

7  *V   l_J   VJ    *^\ld 

"\  trfi   S^i 

i  *i  i  vl 

.     A      ,  jj"    LV 

,  j.    i' 

V^  L*™  ^?M 

']/)  **^     • 

J.H  r     ^ 

>  »  •  l 

(a)  Find  the  total  of  eacli  day's  sales. 

(b)  Find  the  total  sales  for  each  department. 

The  answer  in  the  lower  right  corner  proves  the  work. 

Subtraction.  Subtraction  is  the  process  of  finding  the  differ- 
ence between  two  like  numbers.  The  minuend  is  the  number  to  be 
diminished,  and  the  subtrahend  is  the  number  to  be  taken  from  the 
minuend.  ""- 

Addition  and  subtraction  are  closely  related.  Subtraction  by 
adding  is  the  method  used  by  the  expert  cashier  and  by  money 
changers.  The  " making  change"  method  of  subtraction  consists 
in  adding  to  the  amount  of  the  purchase  enough  to  make  the  sum 
equal  to  the  amount  tendered  in  payment. 

Example 

Y  buys  groceries  to  the  value  of  $1.34  and  gives  the  cashier  two  one-dollar 
bills  in  payment.  How  much  change  should  he  receive? 


FUNDAMENTAL  PROCESSES  AND  SHORT  METHODS       11 

Solution 

The  cashier  in  making  change  may  return  to  Y  a  penny,  a  nickel,  a  dime, 
and  a  half  dollar,  saying:  "$1.34,  35,  40,  50,  $2.00,"  which  means  $1.34  +  .01 
=  $1.35;  $1.35  +  .05  =  $1.40;  $1.40  +  .10  =  $1.50;  and  $1.50  +  .50  =  $2.00. 
Other  coins  than  those  mentioned  may  be  returned  by  the  cashier,  but  it  is 
customary  to  make  change  in  the  largest  coins  possible. 

Exercise 

As  the  cashier,  make  change,  using  the  largest  denominations  possible, 
assuming  the  following  purchases  were  made  and  two  one-dollar  bills  were  offered 
in  payment. 

1.  $1.44  5.  $1.64  9.  $1.17  13.  $1.43 

2.  1.67  6.     1.32  10.     1.29  14.     1.38 

3.  1.27  7.     1.82  11.     1.54  16.     1.49 

4.  1.41  8.     1.1 1  12.     1.56  16.     1.05 

Avoid  errors.  Many  errors  in  subtraction  are  made  in  borrow- 
ing from  the  next  higher  order.  When  that  order  is  reached,  it  is 
not  uncommon  to  overlook  the  fact  that  borrowing  has  taken  place. 
Errors  of  this  kind  can  be  avoided  by  changing  subtraction  to  the 
process  of  addition;  that  is,  by  adding  to  the  subtrahend  the  num- 
ber required  to  make  the  subtrahend  equal  to  the  minuend. 

Explanation.     Instead  of  thinking,  "7  from  16  is  9,"  think,  "7  +  9  «=  16." 
Write  the  9.     Add  1,  the  digit  carried  over,  to  the 
8,  making  9.     9  -f  8  =  17.     Write  8,  and  add  1,  Example 

the  digit  carried  over,  to  1,  making  2.     2  +  0  =  2.         Minuend  8276 

Write  0.     3  +  5  =  8.     Write  5.     Answer:  5,089.  Subtrahend 3187 

Difference 5089 

Problems 

1.  9574  2.  7436         3.  6175         4.  8147  5.  6328         6.  5317 

5886  3569  2897  4368  2549  3428 

Difference  between  a  given  minuend  and  several  subtrahends. 

In  instances  similar  to  the  following  example,  the  final  result  can 
be  found  in  one  operation  by  the  application  of  the  foregoing 
method  of  subtraction. 

Example 

From  a  fund  of  $3,456,  the  following  disbursements  were  made:  $594,  $375, 
and  $286.  What  was  the  balance  left  in  the  fund? 

Explanation.    Write  the  problem  as  shown  in  the  solution.    Begin  at  the 
right,  and  add  the  units'  column  of  subtrahends,  (6  +  5  +  4),  adding  (and 
setting  down)  enough  (in  this  instance,  1)  to  make  the  units'  figure 
of  the  sum  the  same  as  the  units'  figure  of  the  minuend.    Add  the 
tens'  column  of  the  subtrahends,  including  the  carrying  figure,         .     ? 
(I  +8  +  7  +  9),  adding  (and  setting  down)  enough  (in  this  in-  594 

stance,  0)  to  make  the  tens'  figure  of  the  sum  equal  the  tens'  figure  375 

of  the  minuend.    Add  the  hundreds'  column  of  the  subtrahends,  286 

including  the  carrying  figure,  (2  +  2  +  3  +  5),  adding  (and  setting         $2,201 


12       FUNDAMENTAL  PROCESSES  AND  SHORT  METHODS 

down)  enough  (in  this  instance,  2)  to  make  the  hundreds'  figure  of  the  minuend. 
To  the  carrying  figure,  1 ,  add  enough  (in  this  case.  2)  to  make  the  thousands' 
figure  of  the  minuend;  set  down  2. 

Problems 

1.  $1,562            2.  $2,756  28      3.  $5,987  4.  $4,875  6.  $2,975 

437                        52770                235  365  762 

122                           7  55                789  1,529  194 

254                       528  75              1,526  284  275 


Balancing  an  account.  In  most  cases,  inspection  will  tell  which 
side  of  the  account  is  the  greater  in  amount.  Add  the  larger  side, 
and  put  the  same  footing  on  the  smaller  side,  leaving  space  for  the 
balance;  then  add  from  the  top  downward,  Mipply'mg  the  figures 
necessary  to  make  the  column  total  equal  to  the  footing  previously 

placed  there. 

Example 

Debits  Credits 

$  1,956.18  $      134.26 

3,452  75  258  19 

289.34  764  83 

5,726.31  2,375  94 
_     Balance,      7,891   36          * 

jjMgjjS  $U2424.58 


Explanation.  The  balance,  $7,891.36,  was  found  as  follows:  Inspection 
showed  the  debit  side  to  be  the  larger  in  amount.  It  was  therefore  added,  and 
the  footing  of  the  account,  $11,424.58,  was  placed  under  both  debit  and  credit 
columns.  The  first  order  of  the  credits  —  that  is,  the  cents  —  addsjto  22.  Insert 
6  to  make  28.  With  2,  the  digit  carried  over,  the  second  order,  the  dimes,  adds 
to  22.  Insert  3  to  make  25.  The  third  order,  the  dollars,  with  the  digit  carried 
over,  adds  to  23.  Insert  1  to  make  24.  The  fourth  order,  the  tens  of  dollars, 
with  the  digit  carried  over,  adds  to  23.  Insert  9  to  make  32.  The  fifth  order, 
the  hundreds  of  dollars,  with  the  digit  carried  over,  adds  to  16.  Insert  8  to 
make  24.  The  sixth  order,  the  thousands  of  dollars,  with  the  digit  carried  over, 
adds  to  4.  Insert  7  to  make  11. 

Problems 

1.     Debits      Credits  2.     Debits        Credits  3.      Debits         Credits 

$856.73  $298.56  $725  14  $1,356.17  $3,586.28  $  591.18 

34596  264.39  23951   69135  192.75  2,751.26 

298.85    6.15  64.28   256.38  38472   185.35 

142  31  ..............  ..............    75.19  265  54  .................. 

^Complement  method.    The  complement  of  a  number  is  the 
difference  between  that  number  and  the  unit  of  a  next  higher  order 
Thus,  the  complement  of  6  is  4;  the  complement  of  8  is  2;  and  the 
complement  of  68  is  32. 

If,  in  subtracting  a  number  less  than  10  from  a  given  number, 
Its  complement  is  added,  the  result  will  be  10  too  large.  If  two 


FUNDAMENTAL  PROCESSES  AND  SHORT  METHODS       13 

complements  are  added,  the  result  will  be  20  too  large ;  and  if  three 
complements  are  added,  the  result  will  be  30  too  large. 

To  find  the  sum  of  a  column  containing  numbers  to  be  sub- 
tracted, add  the  complements  of  the  subtractive  items,  and  from 
the  sum  of  each  order  deduct  as  many  tens  as  there  are  subtractive 
items  in  the  order. 

Example 

A  practical  application  of  the  complement  method  of  subtraction  is  that  of 
finding  the  net  increase  in  a  statistical  record  such  as  the  following: 


Sales 

Sales 

Increase 

Dept. 

This  Mo. 

Last  Mo. 

Decrease* 

1 

$    427  95 

$    346  29 

$  81  66 

2 

515  86 

457  75 

58  11 

3 

395  57 

385.86 

9.71 

4 

402.75 

416  87 

14  12* 

$1,742.13 

$1,606.77 

$135.36 

Solution 

The  difference  between  the  sales  this  month  and  the  sales  last  month  for 
each  department  is  shown  as  an  increase  or  a  decrease.  The  difference  between 
the  total  sales  this  month  and  the  total  sales  last  month  is  $135.36.  To  prove 
that  the  departmental  increases  and  decrease  are  correct,  add  the  third  column, 
beginning  at  the  top  and  adding  downward,  using  the  complement  each  time 
on  the  last  number.  Thus,  8  and  8  are  16;  write  6,  and  drop  the  10,  as  one 
complement  was  added  and  the  answer  is  10  too  large.  14  and  9  are  23;  write  3 
and  carry  10,  dropping  one  10.  19  and  6  are  25;  write  5  and  carry  1,  again 
dropping  one  10.  14  and  9  are  23;  write  13,  dropping  one  10  as  before. 

Example  Solution 

Find  the  net  increase  of  the  fol-  In   this   problem   there   are   four 

lowing  items:  items   showing    decreases;    therefore, 

each  time  a  complement  is  added,  the 

Increase  final  result  will  be  10  too  large,  and 

Decrease*  in  this  case,  the  final  result  will  be  40 

15  60  too  large,  so  40  is  deducted  each  time. 

4  51*  Begin  at  the  top  and  add  downward: 

17  20  9  (comp.),  15,  23  (comp.  was  8),  30, 

61   96  31,  37,  46,  51,  subtract  40,  write  1 

29  00  and  carry  1. 

8  62*  Now  the  next  column.     7(6andl), 

124  20  12,  14,  23,  27,  29,  33,  35,  38,  41,  45, 

59  40  54,    60,    subtract   40,    write   0   and 

89  83*  carry  2.     Next  column,  7,  13,  20,  21, 

199.30  30,  32,  36,  45,  46,  55,  62,  64,  68,  70, 

113  79*  subtract  40,  write  0  and  carry  3. 

132  46  Adding  the  tens:  4  (I  and  3  car- 

34.99  ried),  5,  11,  13,  15,  20,  22,  31,  40,  43, 

122 . 65  46,  48,  but  subtract  20  as  only  two 

580.01  complements  were  used,  write  8  and 

carry  2,     The  complement  10  may  be 


14       FUNDAMENTAL  PROCESSES  AND  SHORT  METHODS 


added  each  time  there  is  no  item,  making  the  answer  68,  then  subtract  40,  leaving 
28  as  before.  Remember,  subtract  as  many  10's  as  there  are  complements  added. 
Finally  the  hundreds'  column.  There  are  but  five  items  in  this  column; 
therefore,  with  the  2  carried,  proceed  as  follows:  3,  4,  13,  15,  subtract  10  (only  one 
complement  was  added)  and  write  5.  Answer:  580.01. 

Problems 

The  items  to  be  subtracted  are  marked  (*)  in  Problems  1  and  2. 


1.  $58  10 
19  66 
45  55 
77  28 
9  01* 
16.11 
14  12* 


2.  $122  65 
175  50 

89  88* 

17  20 

1  48 

8  62* 

36  95 


3.  $48.75  Gain        4.  $20  25  Gain 


3 1.25  Gain 
3 . 20  Loss 
65.50  Gain 
15. 25  Loss 
16. 38  Gain 
26  65  Gain 


4. 50  Loss 
41  50  Gain 
28  45  Gain 
38  47  Gain 
12  34  Loss 
49  82  Gain 


Subtracting  on  an  adding  machine.  If  increase  or  decrease 
columns^ are  being  verified  on  an  adding  machine  that  does  not  have 
a  direct  subtraction  device,  add  the  complements  of  the  numbers 
to  be  subtracted. 

To  subtract  $219.48,  set  780.52  on  the  keyboard  and  strike  all 
nines  to  the  left  of  the  number;  and  to  subtract  $102.79,  set  897.21 
and  strike  all  nines  to  the  left  of  the  number.  Striking  of  the 
nines  eliminates  from  the  totalizers  the  number  1  that  would 
otherwise  be  included  in  the  answer. 

Practical  problems.  In  the  following  problems,  both  addition 
and  subtraction  have  to  be  performed  in  order  to  complete  the 
records. 

Problem  1 

This  problem  illustrates  a  section  of  a  twelve-month  moving-average  schedule 
used  in  cost  accounting  and  other  cumulative  work.  Assuming  that  twelve 
months  covers  a  cycle  of  business  changes  due  to  seasonal  variations,  and  so 
forth,  the  moving  twelve  months'  total  provides  a  fairly  reliable  amount  for 
comparative  purposes. 

The  earliest  month's  results  are  subtracted  from  the  twelve  months'  total 
and  the  current  month's  results  are  added,  making  a  current  twelve-month 
accumulation.  The  record  is  self-proving. 


Total,  12/31/43..   J 

Dept.  1 
^125,275.93 

'  Dept.  2 
$56,472.29    ! 

Dept.  3 
$4,207.23     ! 

Dept.  4            Total 
$7,200.49    

Deduct  Jan.,  1943 

9,495.79 

4,907.63 

368.80 

502.50 

Add  Jan.,  1944... 

8,805.67 

4,480.25 

358.79 

588.79 

12  mos.  totals  

Deduct  Feb.,  1943 

8,933.07 

4,093.19 

293.67 

496.68 

Add  Feb.,  1944... 

9,033.48 

4,123.97 

235.80 

517.90    .... 

12  mos.  totals  

Deduct  Mar.,  1943 

10,854.92 

4,837.07 

331.04 

480.09    

Add  Mar.,  1944.. 

8,588.37 

4,001.18 

334.17 

521.72     

12  mos.  totals.     .    . 

FUNDAMENTAL  PROCESSES  AND  SHORT  METHODS       15 


Problem  2 

From  the  following  sales  record,  find  the  increase  or  decrease  in  sales  by 
departments. 


COMPARATIVE  SALES  RECORD 


Dept.  No. 
1  

February,  2nd  Year 
$  7,134.95 

February,  1st  Year 
$  6,834.79 

2 

6,225  19 

5,764  87 

3  

7,934  97 

8,375  16 

4 

6,354  76 

5,986  35 

5  

3,695.15 

3,756  89 

6  

.    .             9,767  98 

9,475  18 

7  

8,567  39 

8,467  .  57 

8 

5,607  18 

4,865  84 

9  

11,365  39 

10,785  65 

14,572  86 

13,764  16 

Increase  or 
Decrease  f 


Total  . 


Problem  3 


A  daily  business  record  may  be  prepared  from  cash  register  totals  and  other 
information.  With  the  aid  of  the  amounts  given,  complete  the  record  for  the 
day.  Some  of  the  sections  contain  items  that  are  needed  to  complete  other 
sections. 


Cash  Receipts 
Rec'd.  on  Acc't.   $234  56 
Other  Receipts  .       59  32 
Cash  Sales 497.85 

Total  Receipts 

Cash  on  Hand 
Opening  Balance  $250 . 75 

Receipts   

Total       

Paid  Out  

Closing  balance 

Accounts  Payable 

Bal.  for'd $315.20 

Invoices  Today.     262.35 

Total 

Paid  Today....     136.57 
Balance 


Sales 

Cash  Sales .  .  .   $ 

Credit  Sales..         152.35 


Total  Sales 

Bank  Account 
Bal.  for'd....   $2,872.63 

Today's  Dep       __ 

Total _ 

Today's  Cks..        175.32 
Balance 


Cash  Sales  Summary 
Total  for'd...   $2,542.75 
Today's  Cash 

Sales „ 

Total   to   for- 
ward   „ 

Problem  4 


Cash  Paid  Out 

For  Stock $  85  42 

For  Expenses .  .        19  56 

Personal 27.50 

Deposit 652.80 

Total 

Accounts  Receivable 

Hal.  for'd $481.52 

Credit  Sales 

Total 

Rec'd.  on  Acct 

Balance 

Credit  Sales  Summary 

Total  for'd $638.47 

Today's    Credit 

Sales 

Total  to  forward    


In  the  following  table  of  Gross  Profits  by  Departments,  add  the  Goods  on 
Hand,  March  1,  1st  Year,  to  the  Purchases  for  the  Year,  and  from  this  sum 
subtract  the  Goods  on  Hand,  March  1,  2nd  Year.  This  gives  the  Cost  of  Goods 
Sold.  The  operation  should  be  performed  without  transferring  any  of  the 
figures.  Use  the  complements  of  the  numbers  in  the  column  Goods  on  Hand, 
March  1,  2nd  Year. 


16       FUNDAMENTAL  PROCESSES  AND  SHORT  METHODS 

The  difference  between  the  Cost  of  Goods  Sold  and  the  Sales  will  give  the 
Profit  or  Loss. 

To  verify  the  work,  add  all  the  columns,  and  deal  with  the  totals  in  the  same 
way  as  with  the  figures  for  the  departments.  The  difference  between  the  total 
Cost  of  Goods  Sold  column  and  the  Sales  column  should  equal  the  difference 
between  the  Profit  and  the  Loss  columns,  showing  the  Net  Profit  of  the  ten 
departments  for  the  year. 

GROSS  PROFITS  BY  DEPARTMENTS 

Goods  Goods 

On  Hand  Purchases  On  Hand     Cost  of 

March  1 ,  for  the  March  1 ,      Goods 

Dept.     1st  Year  Year  2nd  Year      Sold  Sales        Profit    Loss 

\       $3,475  86  $  9,846  37  $2,347    11   $12,678  92  . 

2  1,357.10  6,72540  1,47586.. 6,18890 

3  3,276  84  10,326  85  3,827  84  8,297  63                   ..    . 

4  5,475.90  11,176  98  5,874   13  13,586  47  .  .. 

5  4,276  83  9,798  34  4,207  16  10,508  92  .  .. 

6  3,785  47  8,376.41  3,648  10  8,756  13  ......       ....... 

7  2,98617  9,38657  3,01474 8,964  85  ...  ..    .       __    . 

8  3,275.83  8,724   18  2,817  56  9,575  34  .    ..  ... 

9  2,976  95  9,543  34  2,734  15 10,789  18  

10         3,532  25  10,217  60  3,375  89  ..................  12,756  84  ......  ..      .__„_. 

Footings -. 1LI1 _^    -.-- 

Multiplication.  Multiplication  is  a  short  process  of  addition; 
that  is,  a  number  is  to  be  taken  as  an  addend  a  given  number  of 
times. 

How  many  bushels  of  grain  are  in  three  bins  each  containing 
146  bu.? 

A  ddition  Multiplication 
146  146 

146  __3 

146  438 

438 

Multiplication  involves  three  numbers,  the  multiplicand  (the 
number  to  be  repeated,  146) ;  the  multiplier  (the  number  showing 
the  number  of  repetitions,  3) ;  and  the  product  (the  number  show- 
ing the  result,  438). 

The  multiplicand  and  the  product  are  always  like  numbers. 
146  bushels  multiplied  by  3  equals  438  bushels. 

Problems 

1.  What  is  the  cost  of  640  acres  of  land  at  $42.50  an  acre? 

2.  How  many  minutes  are  there  in  an  ordinary  year? 

3.  A  barrel  of  flour  contains  196  pounds.     What  is  the  weight  of  flour  produced 
in  one  day  by  a  mill  that  produces  375  barrels? 


FUNDAMENTAL  PROCESSES  AND  SHORT  METHODS       17 


4.  Sound  travels  about  1,120  feet  in  a  second.    How  far  will  it  travel  in 
15  seconds? 

5.  How  many  peaches  are  in  12  crates,  if  there  are  84  peaches  in  each  crate? 

Accuracy  and  speed  in  multiplication  depend  largely  upon  a 
thorough  mastery  of  the  multiplication  tables.  Tables  previously 
learned  should  be  reviewed.  Continue  with  frequent  drills  011 
combinations  up  to  25  X  25.  The  following  table  of  multiples 
from  12  X  12  to  25  X  25  is  given  for  reference  and  drill.  Tables 
of  multiples  prepared  in  this  manner  facilitate  the  work  of  pay  roll 
extension,  inventory  extension,  billing,  and  so  forth. 

TABLE  OF  MULTIPLES 


J4^  13 

14 

16 

16 

17 

18 

19 

20 

21 

22 

23 

24 

/-2& 

12(144^)156 

168 

180 

192 

204 

216 

228 

240 

252 

264 

276 

288 

(300 

13  TSG  169 

182 

195 

208 

221 

234 

247 

260 

273 

286 

299 

312 

325 

14  168  182 

196 

210 

224 

238 

252 

266 

280 

294 

308 

322 

336 

350 

15  180  195 

210 

225 

240 

255 

270 

;2S5 

300 

315 

330 

345 

360 

375 

16  192  208 

224 

240 

256 

272 

288 

304 

320 

336 

352 

368 

384 

400 

17  204  221 

238 

255 

272 

289 

31)6 

323 

340 

357 

374 

391 

408 

425 

18  216  234 

252 

270 

288 

306  i 

(323f 

>342 

360 

378 

396 

414 

432 

450 

19  228  247 

266 

285 

304 

323 

342" 

361 

380 

399 

418 

437 

456 

475 

20  240  260 

280 

300 

320 

340 

360 

380 

400 

420 

440 

460 

480 

500 

21  252  273 

294 

315 

336 

357 

378 

399 

420 

441 

462 

483 

504 

525 

22  264  286 

308 

330 

352 

374 

396 

418 

440 

462 

484 

506 

528 

550 

23  276  299 

322 

345 

368 

391 

414 

437 

460 

483 

506 

529 

552 

575 

24  288  312 

336 

360 

384 

408 

432 

456 

480 

504 

528 

552 

576 

600. 

25^300  )*25 

350 

375 

400 

425 

450 

475 

500 

525 

550 

575 

600 

625 

Contractions  in  multiplication.  Contractions  in  multiplication 
may  often  be  made  by  observing  the  peculiarities  of  the  multiplier 
and  the  multiplicand  and  calling  into  use  factors,  multiples, 
complements,  supplements,  reciprocals,  aliquots,  and  the  like. 

To  multiply  by  factors  of  the  multiplier.  The  ordinary  method 
and  the  shorter  method  of  multiplying  by  factors  are  shown  in  the 
following  example.  Observe  that  in  the  ordinary  method  there 
are  two  multiplications  and  an  addition,  while  in  the  shorter 
method  there  are  only  two  multiplications. 


Multiply  567  by  27. 

Ordinary  Method 
567 
27 

3969 
1134 

15309 


Example 

Solution 


Shorter  Method 
567    27  =  9  X  3 
9 


5103 

3 

15309 


18       FUNDAMENTAL  PROCESSES  AND  SHORT  METHODS 

Problems 

Multiply: 

1.  4,584  by  64.  3.  1,459  by  35.  5.  8,756  by  42. 

2.  8,359  by  54.  4.  2,684  by  27.  6.  6,123  by  45. 

To  multiply  when  a  part  of  the  multiplier  is  a  factor  or  multiple 
of  another  part. 

Example 

Multiply  34,768  by  4H8. 

Solution 
34768 
__488 

27HT44  product  by  8 
16688640  product  of  60  times  product  by  8 

16966784 

Problems 

Multiply: 

1.  45,692  by  549.  3.  21,347  by  497.  6.  84,123  by  248. 

2.  49,871  by  648.  4.  33,546  by  355.  6.  13,456  by  153. 

To  multiply  a  number  of  two  figures  by  11.  Observation  of  the 
ordinary  method  shows  that,  in  the  answer,  the  sum  of  the  two 
digits  is  written  between  the  two  digits. 

Ordinary  Method  Shorter  Method 

54  54 

11  Jl 

54  594 

54 

594 

When  the  sum  of  the  two  digits  is  10  or  more,  1  must  be  carried 
to  the  digit  at  the  left;  for  example,  64  X  11  =  704,  and  93  X  11 
-  1,023. 

To  multiply  any  number  by  11.  Observation  of  the  ordinary 
method  shows  that,  in  the  answer,  the  units'  digit  of  the  multipli- 
cand is  the  units'  digit  of  the  product;  that  the  tens'  digit  of  the 
product  is  the  sum  of  the  units'  digit  and  the  tens'  digit  of  the 
multiplicand;  that  the  hundreds'  digit  of  the  product  is  the  sum  of 
the  tens'  digit  and  the  hundreds'  digit  of  the  multiplicand;  and  so 
on.  When  the  sum  of  two  digits  is  10  or  more,  1  must  be  carried. 

Ordinary  Method  Shorter  Method 
8937  8937 

11  _11 

8937  98307 

8937 
98307 


FUNDAMENTAL  PROCESSES  AND  SHORT  METHODS       19 

Multiplying  by  26.  Annex  two  ciphers  to  the  multiplicand, 
and  divide  by  4. 

Example 
Multiply  7,562  by  25. 

Solution 

4)756200 

"189050 

Problems 

Multiply  each  of  the  following  by  25: 

1.  3,874,; 6  2.  3,948.  3.  7,981.  4.  5,426. 

Multiplying  by  15.  Annex  a  cipher  to  the  multiplicand,  and 
increase  the  result  by  one-half  of  the  multiplicand. 

Example 

Multiply  8,435  by  15. 

Solution 

84350 

42175 

126525 

Problems 
Multiply  each  of  the  following  by  15: 

1.  7,432.  2.  8,397.  3.  3,926.  4.  9,536. 

Multiplying  numbers  ending  with  ciphers.  Multiply  the  sig- 
nificant figures  in  each  number,  and  to  the  product  annex  as  many 
ciphers  as  there  are  final  ciphers  in  both  the  multiplier  and  the 
multiplicand. 

Example 

Multiply  756,000  by  4,200. 

Solution 

756 

42 

31752cvoOo  (  ' 

Annex  five  ciphers.     Answer:  3,175,200,000. 

Problems 
Multiply: 

1.  325,000  by  2,300.  3.  24,100  by  4,200. 

2.  370  by  480.  4,  8,300  by  2,100. 

Multiplication  by  numbers  near  100,  as  98,  97,  96,  and  so  forth, 
and  by  numbers  near  1,000,  as  997,  996,  and  so  forth.  This 
method  is  of  value  in  finding  the  net  proceeds  of  some  amount  less 
2%,  3%,  and  so  forth,  and  also  in  many  other  situations. 


20       FUNDAMENTAL  PROCESSES  AND  SHORT  METHODS 

Example 
Multiply  3,247  by  97. 

Solution 
Multiply  the  number  by  100,  and  subtract  3  times  the  number. 

324,700  =  3,247  X  100 

9,741  =  3,247  X  3 
314,959  =  3,247  X  97 

Multiplication  by  a  number  near  1,000  is  accomplished  in  the 
name  manner  by  multiplying  by  1,000  instead  of  by  100. 

Problems 

Multiply: 

1.  2,450  by  98.        2.  7,318  by  97.        3.  5,438  by  90.        4.  8,752  by  95. 

Multiplication  of  two  numbers  each  near  100,  1,000,  and  so 
forth.  Products  of  numbers  in  this  class  may  be  calculated 

mentally. 

Example 

Multiply  90  by  98. 

Explanation.     Stop  1.     Multiply  the  complements  of  the  two  numbers,  and 
if  the  product  occupies  units'  place  only,  prefix  a  cipher. 
Result,  08.  Solution 

Step  2.     Subtract  the  complement  of  one  number  Complement 

from  the  other  number,  and  write  the  result  at  the  left  CH\  4 

of  the  result  in  Step  1.     The  complement  of  either  mini-  ()s  2 

her  subtracted  from  the  other  number  leaves  the  same         ()4()s 
remainder;  as,  96  —  2  or  98  —  4  each  equals  94.     Answer: 
9,408. 

Example 

Multiply  92  by  88. 

Solution 

(Complement 
92  8 

88  1 2 

8096 

Explanation.  The  product  of  the  complements  is  90,  the  last  two  figures  of 
the  answer.  88  —  8  or  92  —  12  =  80,  the  lirst  two  figures  of  the  answer. 
Answer:  8,090. 

Example 

Multiply  996  by  988. 

Solution 

Complement 
996  4 

988  12 

984,048 


FUNDAMENTAL  PROCESSES  AND  SHORT  METHODS       21 

Explanation.  When  numbers  near  1,000  are  multiplied,  ciphers  are  prefixed 
to  the  product  of  the  complements,  so  that  the  product  occupies  three  places. 

Problems 

Multiply: 

1.  97  by  96.  2.  88  by  98.  3.  995  by  992.         4.  997  by  994. 

Multiplying  by  numbers  a  little  larger  than  100,  as  101,  102, 
and  so  forth.  Annex  two  ciphers  to  the  multiplicand,  and  to  this 
add  the  product  of  the  multiplicand  and  the  units'  figure  of  the 
multiplier.  Annex  three  ciphers  for  multipliers  over  1,000. 

Example 
Multiply  3,475  by  104. 

Solution 

347500 

13900  (4  X  3,475) 
361400 

Problems 

Multiply : 

1.  2,875  by  102.      2.  3,490  by  105.      3.  2,972  by  1,004.      4.  4,508  by  1,006. 

Multiplication  of  two  numbers  each  a  little  more  than  100.     To 

the  sum  of  the  numbers  (omitting  one  digit  in  the  hundreds' 
column),  annex  two  ciphers,  and  add  the  product  of  the  supple- 
ments (excess  over  100). 

Example 

Multiply  112  by  113. 

Solution 
112 
113 

12500     (sum  of  numbers,  with  one  digit  in  the  hundreds*  column  omitted) 
156     (product  of  supplements,  12  X  13) 

1265(5 

Explanation.  In  instances  similar  to  the  foregoing,  a  knowledge  of  the 
multiplication  tables  to  20  X  20  makes  mental  results  possible,  and  is  invaluable 
in  inventory  and  other  extensions. 

Problems 
Multiply: 

1.114  by  112.          2.  106  by  108.         3.  116  by  111.         4.  118  by  115. 

Cross  multiplication.  When  the  multiplicand  and  the  multi- 
plier are  each  numbers  of  two  figures,  the  work  may  easily  be  kept 
in  mind  and  the  partial  products  added  without  being  written 
down. 


22       FUNDAMENTAL  PROCESSES  AND  SHORT  METHODS 

Example 

Multiply  47  by  38. 

Solution  Graphic  Solution 

47 


38  3; 


1786 


*     o        i  - 

A      *« 


Explanation.  8X7  =  56.  Write  6,  carry  5.  (8  X  4)  +  (3  X  7)  +  5  =  58, 
Write  8,  carry  5.  (3  X  4)  -f  5  -  17.  Write  17.  Answer:  1,7<S6. 

Problems 

Multiply: 

1.  53  by  20.  2.  48  by  57.  3.  74  by  32.  *  4.  65  by  28 

To  cross-multiply  a  number  of  three  digits  by  a  number  of  two 
digits.  A  three-digit  number  may  be  multiplied  by  a  two-digit 
number  in  a  manner  similar  to  that  of  multiplying  a  two-digit 
number  by  a  two-digit  number. 

Example 

Multiply  346  by  28. 

Solution 

346 

28 
9688 

Explanation.  8X6  =  48.  Write  8,  carry  4.  4  (carried)  +  (8  X  4)  -f 
(6  X  2)  =  48.  Write  8,  carry  4.  4  (carried)  -f  (8  X  3)  4-  (4  X  2)  =  36.  Write 
6,  carry  3.  3  (carried)  +  (2  X  3)  -  9.  Write  9.  Answer:  9,(>XX. 

A  graphic  presentation  of  the  steps  required  appears  as  follows: 

42    3 


34il> 
2J 


Problems 


1.  324  X  28      4.  428  X  34      7.  2X9  X  85     10.  693  X  42 

2.  543  X  42      5.  51(5  X  26     8.  356  X  48     11.  384  X  56 

3.  658  X  56      6.  513  X  76      9.  7X5  X  34     12.  473  X  65 

To  cross-multiply  a  number  of  three  digits  by  another  number 
of  three  digits.  Comparison  of  the  graphic  presentation  with  that 
above  shows  that  the  first  three  steps  arc  the  same,  the  next  three 
are  new,  and  the  final  three  are  the  same. 


Example 

Multiply  428  by  356. 

Solution  Graphic  Solution 

,1  23  465  78 

428  42*          4  M  4st/S  \2  8          ±28 

356  3  5  <}          3 

152,368 


23  465  78  9. 

V  4sixS  \Z  8          428 

A,  3"^  >TM>          $56 


FUNDAMENTAL  PROCESSES  AND  SHORT  METHODS       23 

Explanation.  6  X  8  =  48.  Write  8,  carry  4.  4  (carried)  +  (6  X  2)  + 
(8X5)  =  56.  Write  6,  carry  5.  5  (carried)  +  (6  X  4)  +  (8  X  3)  +  (2  X  5) 
=  63.  Write  3,  carry  6.  6  (carried)  +  (5  X  4)  +  (2  X  3)  =  32.  Write  2, 
carry  3.  3  (carried)  +  (3  X  4)  =  15.  Write  15.  Answer:  152,368. 

Problems 

1.  124  X  251  6.  832  X  425  11.  436  X  579 

2.  262  X  158  7.  639  X  256  12.  S32  X  656 

3.  328  X  245  8.  819  X  325  13.  295  X  638 

4.  638  X  256  9.  677  X  283  14.  767  X  842 
6.  784  X  364  10.  518  X  824  15.  698  X  476 

Preparation  of  a  table  of  multiples  of  a  number.    It  is  not 

uncommon  to  have  to  use  the  same  number  many  times  in  making 
calculations,  especially  in  cost  accounting.  A  saving  of  time  and 
increased  accuracy  in  the  work  arc  achieved  if  a  table  of  multiples 
of  the  number  is  constructed.  Suppose  that  you  have  to  perform 
a  number  of  multiplications  in  which  32(5,834  is  one  of  the  factors. 
A  table  of  multiples  may  be  constructed  with  an  adding  machine 
by  locking  the  repeat  key.  Sub-total  after  each  pull  of  the  handle. 
The  sub-totals  should  check  with  the  product  column  shown  below. 
If  the  table  is  prepared  by  repeated  additions,  and  not  with  an 
adding  machine,  the  10th  product  should  be  computed,  as  it  will 
verify  all,  unless  there  are  compensating  errors  in  the  work. 

TABLE  OF  MULTIPLES 

Multiplier  Product 

1  326,834 

2  (326,834  +  326,834)       .        653,668 

3  (653,608  -f  326,834)       980,502 

4  (980,502  -f  326,834)  .       ...  1 ,307,336 

5  (1,307,336  +  326,834) . .      1 ,634,170 

6  (1,634,170  +  326,834)            .  1,961,004 

7  (1,96 1,004  +  326,834)       .               .  2,2X7,838 

8  (2,2X7,838  +  326,834)          2,614,672 

9  (2,614,672  +  326,834)  2,94 1 ,506 

Verification 
10(2,941,506  +  326,834) 3,268,340 

Example 
Multiply  326,834  by  5,249. 

Solution 

2941506  =  9  times  326,834 

1307336     =  4  times  326,834 

653668  =  2  times  326,834 
1634170  =  5  times  326,834 
1715551666  -  product 

If  the  table  is  prepared  without  toe  use  of  an  adding  machine,  proceed  as 
outlined  on  the  next  page. 


24       FUNDAMENTAL  PROCESSES  AND  SHORT  METHODS 

1.  Write  326,834  near  the  bottom  of  a  slip  of  paper  or  a  card. 

2.  Start  the  table  by  writing  326,834.    Place  the  slip  or  card  just  above  thig 
number,  thus: 

r 

326,834 


1.      326,834 

2 

3 ... 

3.  Add  the  two  numbers,   placing  the  sum,   653,668,   on  line  2.     This  is 
two  times  the  number. 

4.  Move  the  slip  or  card  down  one  line  and  add  again,  placing  the   sum, 
980,502,  on  line  3,  forming  three  times  the  number. 

5.  Continue  moving  the  slip  or  card  down  one  line  each  time  and  adding. 

6.  When  9  times  the  number  is  obtained,  check  the  accuracy  of  the  work  by 
repeating  the  process  once  more.     The  result  should  be  ten  times  the  number. 

Problems 

Set  up  a  table  of  multiples  of  245,386,  and  multiply  245,386  by  the  following 
numbers: 

1.  2,465  2.  3,542  3.  2,498  4.  5,347  6.  6,173 

Division.  Division  is  the  process  of  finding  how  many  times 
one  number  is  contained  in  another  number.  The  dividend  is  the 
number  to  be  divided,  the  divisor  is  the  number  by  which  we  divide, 
and  the  quotient  is  the  number  showing  how  many  times  the  divi- 
dend contains  the  divisor. 

The  remainder  is  a  number  less  than  the  divisor,  and  results 
when  the  dividend  does  not  contain  the  divisor  exactly.  It  is  an 
undivided  portion  of  the  dividend. 

Short  division  is  the  method  used  when  the  products  of  the 
divisor  and  the  digits  of  the  quotient  are  omitted. 

Example  Solution 

Divide  3,476  by  2.  2)3476 

1738 

Long  division  is  the  method  used  when  the  work  is  written  in  full. 
Example  Solution 

Divide  5,839  by  24.  ?4)5839(243 

48 

103 
96 
79 

72 
7 

To  divide  by  25,  60,  or  125.  The  work  of  division  can  be 
lessened  by  making  the  operation  one  of  multiplication. 


FUNDAMENTAL  PROCESSES  AND  SHORT  METHODS      25 

Example  Solution 

Divide  1,400  by  25.  14  X  4  =  56. 

Explanation.  Divide  1,400  by  100  by  dropping  the  zeros.  But,  100  ia 
4  times  the  actual  divisor,  therefore  the  quotient  14  is  £  of  the  actual  quotient, 
so  14  X  4  or  56  is  the  actual  quotient. 

In  a  similar  manner,  1,400  divided  by  50  is  2S;  and  14,000  divided  by  125 
is  112.  (Note:  Further  reference  to  this  method  is  given  under  the  subject  of 
division  by  aliquot  parts  of  100.) 

Abbreviated  division.  Instead  of  writing  the  product  and 
then  subtracting,  the  product  of  each  digit  of  the  divisor  is  sub- 
tracted mentally,  using  the  " making  change"  method,  and  only 
the  remainder  is  written. 

3285 

2347708756 
667 
1995 
1236 
66 

Use  of  tables  in  division.  If  a  number  of  divisions  are  to  be 
made  witli  the  same  divisor,  it  is  advantageous  to  set  up  a  table  of 
multiples  of  the  divisor. 

Example 

Assume  that  328  is  to  be  used  a  number  of  times  as  a  divisor,  and  that  one 
of  the  dividends  is  587,954,  a  table  of  multiples  could  be  set  up  thus: 

TABLE  OF  MULTIPLES 

Multiplier  Product 

\                                          .          .                                     ....  328 

2  .                                                             656 

3  .               .                                        984 

4  .....  1,312 

5  ...                                      ...                         ...  1,640 

6.    ...                       ..                     .                                        ..  1,968 

7        .  2,296 

8  ..          .               ..  2,624 

9 2,952 

Explanation.  Inspection  shows  the  first  digit  in  the  quotient  to  be  1 .  The 
second  partial  dividend  is  2,599.  The  table  of 

multiples  shows  the  largest  product  contained  Solution 

therein    to    be    2,296,    opposite    7.     The    third         328)5S7954(1792fft 
partial  dividend  is  3,035,  and  the  table  of  multi-  328 

pies  shows  the  largest  product  contained  therein  2599 

to   be   2,952,   opposite  9.     The  fourth   partial  2296 

dividend  is  834,  and  the  largest  product  con- 
tained  therein  is  656,  opposite  2.  The  remainder 
is  178.  The  fraction  ttf  may  be  reduced  to 

,  or  it  may  be  changed  to  a  decimal.  8<*4  178  __   89 

656          328      164 
178 


26       FUNDAMENTAL  PROCESSES  AND  SHORT  METHODS 

Division  in  this  manner  is  rapid,  as  no  time  is  lost  through 
selection  of  a  quotient  so  large  that  when  the  product  is  found  it 
exceeds  the  dividend,  necessitating  another  trial. 

Problems 

Divide  the  following  numbers  by  144  after  setting  up  a  table  of  multiples 
of  144: 

1.  374,825.  2.  628,256.  3.  496,287. 

Reciprocals  in  division.  The  reciprocal  of  any  number  is 
found  by  dividing  1  by  the  number.  The  reciprocal  of  5  is  1  -r-  5, 
or  .2,  and  the  reciprocal  of  25  is  1  -f-  25,  or  .04. 

The  quotient  in  a  division  may  be  found  by  multiplying  the 
dividend  by  the  reciprocal  of  the  divisor.  Hence,  in  instances  in 
which  it  is  necessary  to  find  what  per  cent  each  item  is  of  the  total 
of  the  items,  the  use  of  the  reciprocal  of  the  divisor  will  save  time 
and  provide  a  check  on  these  computations. 

To  find  what  per  cent  each  item  is  of  the  total  of  the  items: 

(a)  Divide  1  by  the  total  of  the  items  to  obtain  the  reciprocal 
of  the  total. 

(6)  Using  the  result  obtained  in  (a)  as  a  fixed  multiplier,  mul- 
tiply each  of  the  individual  items,  and  the  respective  results 
obtained  will  be  the  per  cents  which  the  individual  items  are  of  the 
total  sum. 

Example 

Find  the  per  cent  that  each  department's  monthly  expense  is  of  the  total 
monthly  expense. 

Department  Expense 

A $    600  00 

B 500  00 

C 1,200  00 

D 700  00 

E .  1,000  00 

Total ..  $4,000  00 

Solution 

Divide  1  by  4,000  to  obtain  the  reciprocal,  .00025.  Multiply  the  expense 
of  each  department  by  this  reciprocal,  and  the  product  will  be  the  per  cent  that 
the  department's  expense  is  of  the  total  expense. 

Department  Expense  Reciprocal        Per  Cent 

A $    600  00     X     .00025     =       15   % 


Crf 
/O 


B 500.00  X  .00025  =  12i 

C 1,20000  X  00025  =  30% 

D 70000  X  .00025  =  17i% 

E 1,000  00  X  .00025  =  25   % 

Total $4,000  00  100% 


FUNDAMENTAL  PROCESSES  AND  SHORT  METHODS       27 

The  foregoing  method  of  calculating  the  rate  per  cent  lias  a 
great  many  applications  in  an  accountant's  work.  Another  illus- 
tration is  given — that  of  calculating  the  per  cent  that  each  item  in 
a  profit  and  loss  statement  is  of  net  sales. 

QUALITY  MEAT  MARKET 
PROFIT  AND  Loss  STATEMENT  FOK  THE  YEAR 

Detail  Amount      Per  Cent 

Net  sales            $20,000  00     100  00 

Cost  of  merchandise  sold 15,712.00       78.56 

Gross  profit  $4,288 .00     ~~2~l~.  44 

Expenses  N 

Salaries  and  wages $2,266  00  11 .33 

Advertising 22  00  .11 

Wrappings 172  00  .86 

Refrigeration     210  00  1 .05 

Heat,  light,  and  power 54  00  .27 

Telephone 54  00  .27 

Rent 33S  00  1.69 

Interest                  146  00  .73 

Depreciation  of  store  equipment  152  00  .76 

Repairs  to  store  equipment  44  00  .22 

Insurance           10  00  .05 

Taxes                  42  00  .21 

Losses  from  had  debts 38  00  .19 

Other  expenses 284  00  1   42 

Total  expenses     _3»832  °°        19I1(? 

Net  profit $  _456  00      ~  2J28 

Explanation.  The  foregoing  is  a  simple  statement,  and  the  per  cents  can 
be  determined  mentally  if  each  item  is  divided  by  the  amount  of  net  sales.  For 
the  purpose  of  illustration,  however,  find  the  reciprocal  of  $20,000.00,  which  is 
.00005  (1  -T-  20,000);  then  multiply  each  item  by  this  reciprocal,  and  the  results 
will  be  as  shown  in  the  per  cent  column. 

Problems 

1.  The  floor  space  occupied  by  Z  Manufacturing  Company  was  as  follows: 

Service  Department  X 600  sq.  ft. 

Service  Department  Y .                      .              .  1,100  sq.  ft. 

Service  Department  Z . . . .                                              .  550  sq.  ft. 

Producing  Department  A .                                           .  2,000  sq.  ft. 

Producing  Department  B  1 ,568  sq.  ft. 

Producing  Department  C     .  2,234  sq.  ft. 

Sales  Department .  600  sq.  ft. 

Administrative  Offices  ....                                         .  550  sq.  ft. 

97262  sq.  ft. 

The  Building  and  Maintenance  Expense  account  shows  a  total  of  $2,982.50. 
What  amount  of  this  expense  should  be  distributed  to  each  of  the  departments? 


18       FUNDAMENTAL  PROCESSES  AND  SHORT  METHODS 

2.  In  the  following  tabulation,  find  the  per  cent  that  each  department's  floor 
space  is  of  the  total  floor  space: 

Sq.  Ft.  Per  Cent 

Floor  Space  of  Total 

Dept.  1   2,456  

Dept.  2 1,014  

Dept.  3      875  

Dept.  4      1,252 

Dept.  5 74S  _._... 

0,345  JOOJXJ 

3.  Calculate  the  per  cent  that  each  item  is  of  net  sales. 

THE  FOOD  MART 
PROFIT  AND  Loss  STATEMENT 

Net  Sales $35,00010000% 

Cost  of  Merchandise  Sold      27,909     . 

Gross  Profit $7,031     ....    .... 

Expenses 

Salaries  and  Wages                                       $4,080  ....     .... 

Advertising 28                   

Wrappings   ....  200                     .      

Refrigeration      .                     .  30S 

Heat,  Light,  and  Power  100                    

Telephone SI 

Rent .  410                   

Interest 203                        

Depreciation  of  Store  Equipment  147 

Repairs  to  Store  Equipment              .  .  45                    ....     ... 

Insurance . .  21 

Taxes 39 

Losses  from  Bad  Debts 119                   

Other  Expenses 490 

Total  Expenses .  0,349    "        T7 

Net  Profit $T,2S2    ~    ... 


CHAPTER^} 
Checking  Computations 

Methods.  Addition  may  be  checked  by  adding  the  second 
time,  adding  from  the  bottom  to  the  top  if  the  first  addition  was 
from  the  top  to  the  bottom.  This  is  preferable  to  performing  the 
work  in  the  same  way  the  second  time,  as  a  mistake  once  made  is 
likely  to  be  repeated. 

Subtraction  may  be  checked  by  adding  the  subtrahend  and  the 
remainder.  The  sum  should  equal  the  minuend. 

Multiplication  may  be  checked  by  interchanging  the  multiplier 
and  the  multiplicand  and  inuliiph  inn  again. 

Division  may  be  checked  by  multiplying  the  divisor  and  the 
quotient,  adding  to  this  product  any  remainder.  The  answer 
should  equal  the  dividend. 

Rough  check.  Rough  check  is  an  approximate  check  and  is 
often  used  to  locate  large  errors.  It  is  also  used  in  determining 
approximate  results.  It  is  especially  useful  in  checking  misplace- 
ment of  the  decimal  point  in  multiplication  and  division  of  decimal 
fractions. 

A  rough  check  of  addition  may  be  made  as  follows: 

Example  Check 

54,S92  55 

36,071  36 

53,784  54 

21,342  21 

_76,854  J77 

242,943  243 

If  the  required  result  is  thousands,  disregard  the  three  columns 
at  the  right,  except  to  increase  the  fourth  column  sum  by  one  if  the 
digit  in  the  third  column  is  5  or  more.  The  check  shows  the 
answer  to  be  approximately  243,000. 

Absolute  check.  There  is  no  such  thing  as  an  absolute  check 
because  there  are  always  possibilities  of  offsetting  errors,  but  the 
use  of  several  methods  of  checking  computations  makes  the  prob- 
ability of  error  so  slight  that  one  may  rely  on  the  result  as  correct. 

Check  numbers  obtained  by  casting  out  the  nines.  A  simple 
and  easily  remembered  check  is  that  of  casting  out  the  nines.  Add 

29 


30 


CHECKING  COMPUTATIONS 


the  digits  of  the  number,  divide  the  sum  by  nine,  and  use  the 
remainder,  which  is  called  "the  excess/'  as  the  check  number.  In 
the  number  4,875,  the  sum  of  the  digits  is  24,  and  24  divided  by  9 
equals  2  with  an  excess  of  6. 


Verification  of  addition. 

Explanation.  The  sum  of  the  digits  of  8,342  is  17  (S  +  3  -f  4  +  2). 
out  9  and  set  down  8.  If  a  number  contains  a  9,  skip 
it  in  adding  the  digits;  thus,  in  8,907,  8  +  6  +  7 
equals  21 .  Cast  out  the  nines  and  set  down  the  excess, 
3.  Find  the  check  number  of  each  line  in  the  same 
way.  Add  the  check  numbers,  and  cast  the  nines 
out  of  their  sum.  Find  the  check  number  of  the  sum 
of  the  column  being  verified.  The  final  check  number 
in  each  case  is  5. 

Problems 


Add,  and  verify  by  casting  out  the  nines: 


1. 

2487 
3156 
29S2 
4750 

8928 


2. 

7452 
8129 

5758 
2253 
70S5 


3. 

4501 
2765 
4567 
8250 
2435 


Example 
8342 
8967 

8378 
9276 
8431 
43394—5 


Cast 


8 

3 

8 

6 

_7 
32—5 


4. 

1231 
4567 
1085 
3426 
7531 


Verification  of  subtraction. 

Example 

7856  8 

213S  5 

5718  3 

Explanation.     7,856  checks  8,  and  2,138  checks  5.     8-5  =  3,  and  5,718 
i' hecks  3. 

Problems 


Subtract,  and  verify  by  casting  out  the  nines: 


1. 

7496 
2831 


2. 

7428 
1956 


3. 

4751 
3286 


4. 

8237 
5129 


Verification  of  multiplication. 

Example 

482  5 

376  J7 

181232—8  35—8 

Explanation.     482  checks  5,  and  376  checks  7. 
and  the  product,  181,232,  also  checks  8. 


7  X  5  -  35.    35  checks  8, 


CHECKING  COMPUTATIONS  31 

Problems 

Multiply,  and  verify  by  casting  out  the  nines: 

1.  2.  3.  4. 

456  412  832  765 

287  654  254  414 

Verification  of  division.  Division  may  be  verified  by  multi- 
plication; that  is,  the  product  of  the  quotient  and  the  divisor 
should  equal  the  dividend.  Apply  the  same  principle  in  verifying 
with  check  numbers. 

Example 
13)76492(5884 

65  Explanation.     76,492 

77^  checks  1.     13  checks  4. 

104  5,884  checks  7.    4X7  = 

—  28,    and    28    checks    1, 

J/*J  which  is  also  the  check 

_  number  of  the  dividend. 

52 
52 

Problems 

Divide,  and  verify  by  casting  out  the  nines: 
1.  11,550  by  42.       2.  60,882  by  73.      3.  11,049  by  127.    4.  9,854  by  26. 

Verification  of  division  where  there  is  a  remainder.  The  check 
number  of  the  remainder  added  to  the  product  of  the  check  number 
of  the  quotient  and  the  check  number  of  the  divisor  should  equal 
the  check  number  of  the  dividend. 

Example  Explanation.    Step  1 :  The 

32)75892(2371  remainder,  20,  checks  2.     The 

Q4  quotient,     2,371,     checks     4. 

—-  The  divisor,  32,  checks  5.    2  + 

1  *®  (4X5)  =  22,  and  22  checks  4. 

_  Step  2:  The  dividend, 

229  75,892,  checks  4. 

224  step  1  and  Step  2  should 

52  produce  the  same  check  num- 

32  ber. 

Problems 

Divide,  and  verify  by  casting  out  the  nines: 

1.  34,765  by  52.       2.  29,878  by  87.      3.  95,763  by  26.      4.  8,476  by  4J 

Check  numbers  obtained  by  casting  out  the  elevens.  Because 
casting  out  nines  does  not  reveal  errors  in  computations  if  two 


32  CHECKING  COMPUTATIONS 

digits  have  been  transposed,  some  persons  prefer  to  use  eleven  as  a 
check  number. 

Begin  with  the  left-hand  digit  of  the  first  number,  and  subtract 
it  from  the  digit  to  its  immediate  right.  If  the  digit  to  the  right 
is  smaller,  add  eleven  before  subtracting.  Using  the  remainder  as 
a  new  digit,  subtract  it  from  the  third  digit  from  the  left,  first 
adding  eleven  if  necessary.  Use  this  remainder  as  a  new  digit, 
and  subtract  it  from  the  fourth  digit  from  the  left,  first  adding 
eleven  if  necessary.  Continue  in  this  manner  until  all  the  digits 
in  the  number  have  been  used.  The  final  remainder  is  the  check 
number  of  the  number. 

Another  method  of  checking  results  by  means  of  the  number 
eleven  is  to  use  alternate  digits.  From  the  sum  of  the  first,  third, 
fifth,  etc.,  digits  (beginning  at  units'  place)  subtract  the  sum  of  the 
second,  fourth,  sixth,  etc.,  digits.  If  the  subtraction  cannot  be 
performed,  eleven  is  first  added  to  the  sum  of  the  odd  digits,  and 
the  sum  of  the  even  digits  is  subtracted,  the  remainder  being  the 
check  number. 

Verification  of  addition. 

Explanation.     Begin  at  the  left  with  the  number  4,324.     4  from  14  (3  +  11) 

=  10.     10  from  13  (2  +  11)  =  3.     3  from  4  =  1,  the  Example 
check  number  of  4,324. 

Take    the    second    number,    8,6cS9.     8    from    17  ,                        { 

(6  +  11)  =  9.     9  from  19  (8  +  1 1)  =  10.     10  from  20  8b89                   10 

(9  +  11)  =  10,  the  check  number  of  8,689.  6327                      2 

Check  all  the  numbers  in  the  same  manner.     Add  ^>A  7                    n 

the  check  numbers.     The  sum  of  the  check  numbers  _ 

checks  1,  and  the  sum  of  the  numbers  checks  1.  31791 — 1            23 — I 

Problems 

Add,  and  verify  by  casting  out  the  elevens: 

1.          2.          3.          4. 

3789  2450  9755  8307 

5462  1279  8256  7165 

9581  2075  3851  2693 

3998  2754  8632  2198 

5314  9287  6311  5183 

Verification  of  subtraction. 

Example 

7453  6 
1289  2 
6164  4 

Explanation.    7,453    checks    6.     1,289    checks    2.    6-2  =  4    and    6,164 
checks  4. 


CHECKING  COMPUTATIONS  33 

Problems 

Subtract,  and  verify  by  casting  out  the  elevens: 

1.  2.  3.  4. 

8795        3465        7985        3079 
1560        2134        5698        1002 

Verification  of  multiplication. 

Example 

584  1 

256  3 


149504  3 

Explanation.    584  checks  1 .    256  checks  3.    3X1=3,  and  149,504  checks  3 

Problems 

Multiply,  and  verify  by  casting  out  the  elevens: 

1.  2.  3.  4. 

346  4289  7437  287 

275  324  2856  J6 

Verification  of  division. 

Example  1  Example  2 

24)89784(374JI  31)75893(2448 

72  62 

177  138 

168  m 

98  149 

96  124 

24  253 

24  248 
~5 

Explanation  1.  89,784  checks  2.  24  checks  2.  3,741  checks  1.  2X1=2, 
the  check  number  of  the  dividend. 

Explanation  2.  75,893  checks  4.  31  checks  9.  2,448  checks  6.  The 
remainder  checks  5.  5  +  (9  X  6)  =  59.  59  checks  4,  the  same  check  number 
as  that  of  the  dividend. 

Problems 

Divide,  and  verify  by  casting  out  the  elevens: 
1.  80,925  by  83.     2.  124,392  by  142.     3.  25,874  by  49.     4.  28,769  by  135. 

Check  number  thirteen.  If  thirteen  is  used  as  a  check  number, 
transpositions  and  shif tings  of  figures  are  readily  detected.  How- 
ever, in  checking  by  13,  it  is  necessary  actually  to  divide  by  13. 


34  CHECKING  COMPUTATIONS 

TABLE  OF  MULTIPLES 


1  

13 

6  

78 

2  

26 

7  

91 

3    ... 

.  39 

8 

104 

4  

...  52 

9 

117 

5  

65 

10  

130 

All  the  dividing  is  done  mentally. 

Example 

Cast  out  13  from  247,563. 

Explanation.  Begin  with  the  two  left-hand  digits.  24  checks  11.  11,  with 
the  next  digit,  7,  is  1 1 7,  and  1 1 7  checks  0.  Use  the  next  two  digits.  56  checks  4. 
4  with  the  next  digit  is  43,  and  43  checks  4. 

The  verification  of  addition,  subtraction,  multiplication,  and 
division  is  performed  in  the  same  manner  as  with  9  and  11.  The 
difference  is  in  the  method  of  arriving  at  the  check  number,  as  has 
been  outlined. 

Problems 

1.  Add,  and  verify  by  check  number  13: 

24875 
32986 
79840 
80475 
13048 
93476 

2.  Subtract,  and  verify  by  check  number  13: 

84756 
21348 

3.  Multiply,  and  verify  by  check  number  13: 

4875 
259 

4.  Divide,  and  verify  by  check  number  13: 

975,648 
348 


CHAPTER^) 
Factors  and  Multiples 

Factors.  The  factors  of  a  number  are  the  integers  whose  prod- 
uct is  the  number.  Thus,  the  factors  of  6  are  2  and  3,  the  factors 
of  18  are  3  and  6,  or  2  and  9.  A  prime  factor  is  a  prime  number, 
that  is,  a  number  not  exactly  divisible  by  any  number  except 
itself  arid  1. 

Factoring  is  the  process  of  separating  a  number  into  its  factors. 

Example  Solution 

What  are  the  prime  factors  of  315?               3)315  The  prime  factors  of 

3)105  315  are,  therefore, 

5)  35  3X3X5X7. 

7 

Example  Solution 

What  are  the  factors  of  315?  9)315     The  factors  of  315  are, 

7)  35         therefore,  9X7X5. 

5 

Factoring  is  important  for  its  assistance  in  the  solution  of 
problems  in  fractions,  practical  measurements,  percentage,  and  all 
problems  in  which  cancellation  is  used.  One  use  of  factors  was 
given  on  page  17,  "to  multiply  by  factors  of  the  multiplier,"  and 
another  on  page  18,  "to  multiply  when  a  part  of  the  multiplier  is 
a  factor  or  multiple  of  another  part." 

Tests  of  divisibility.  To  be  able  to  factor  a  number  quickly, 
one  must  become  thoroughly  familiar  with  the  tests  of  divisibility. 

A  number  is  divisible  by : 

1.  Two,  if  is  is  an  even  number  or  if  it  ends  in  zero. 

2.  Three,  if  the  sum  of  its  digits  is  divisible  by  3.     Thus,  41754 
is  divisible  by  3  because  the  sum  of  the  digits  is  21,  and  21  is 
divisible  by  3. 

3.  Four,  if  the  two  right-hand  figures  are  zeros,  or  if  they 
express  a  number  divisible  by  4.    Thus,  13724  is  divisible  by  4 
because  24  is  divisible  by  4. 

4.  Five,  if  the  units'  figure  is  either  a  zero  or  a  5. 

5.  Six,  if  it  is  an  even  number  the  sum  of  whose  digits  is  divisible 
by  3.     Thus,  846,  918,  and  54252  are  divisible  by  6, 

35 


36  FACTORS  AND  MULTIPLES 

6.  Eight,  if  the  three  right-hand  digits  are  zeros,  or  if  they 
express  a  number  divisible  by  8.     Thus,  2000  and  5624  are  divisi- 
ble by  8. 

7.  Nine,  if  the  sum  of  its  digits  is  divisible  by  9. 

8.  Ten,  if  the  right-hand  figure  is  zero. 

(There  is  no  simple  method  of  testing  divisibility  by  7.) 
Greatest  common  divisor.     A  common  divisor  of  two  or  more 

numbers  is  a  number  that  evenly  divides  each  of  them.     Thus,  a 

common  divisor  of  16  and  24  is  4. 

The  greatest  common  divisor  of  two  or  more  numbers  is  the 

greatest  number  that  will  evenly  divide  each  of  them.     It  is  the 

product  of  all  their  common  factors. 

Example  Solution 

Find  the  greatest  common  divisor  3)36  63  54 

of  36,  63,  and  54.  3^2 ~2l~~18 

~T~T~6 
Since  4,  7,  and  6  have  no  common  factors,  the  G.  C.  D.  is  3  X  3  =  9. 

A  practical  application  of  the  principles  involved  in  finding 
the  G.  C.  D.  is  in  reducing  common  fractions  to  their  lowest  terms. 

Problems 
Find  the  G.  C.  D.  of  the  following: 

1.  64,  160,  320,  640  3.  32,  48,  12<S 

2.  36,  54,  90  4.  81,  729,  2187 

5.  X,  K,  and  Z  own  land  on  a  new  street.     X  has  600  feet  frontage,  Y  has 
720  feet,  and  Z  has  900  feet.     If  they  wish  to  cut  this  land  into  lots  of  equal 
width,  how  wide  will  the  lots  be,  and  how  many  will  each  have? 

6.  If  you  have  three  coils  of  steel  cable  measuring,  respectively,  2205,  2940, 
and  4704  feet,  and  wish  to  cut  the  whole  quantity  into  pieces  of  the  greatest  equal 
length  possible  without  waste  or  splices,  what  will  be  the  length  of  each  piece? 
How  many  lengths  will  be  cut  from  each  coil? 

Least  common  multiple.  A  common  multiple  of  two  or  more 
numbers  is  a  number  that  is  evenly  divisible  by  each  of  them. 
Thus,  24  is  a  common  multiple  of  3  and  8. 

The  least  common  multiple  of  two  or  more  numbers  is  the 
least  number  that  is  evenly  divisible  by  each  of  them.  Thus,  12 
is  the  L.  C.  M.  of  4  and  6. 

Example 
What  is  the  L.  C.  M.  of  12,  28,  30,  42,  and  64? 


FACTORS  AND  MULTIPLES  37 

Solution 

2)12  28  30  42  64 

2)  6  14  15  21  32 

3)"  3     7~15  21   16 

7)  1     7    5    7  16 

f    1     5     1  16 

2X2X3X7X5X16  =  6,720 

Explanation.  Notice  that  any  number  not  divisible  by  the  factor  is  brought 
down,  and  the  process  is  repeated  as  long  as  at  least  two  of  the  numbers  have  a 
common  factor.  Finally,  the  L.  C.  M.  is  the  product  of  the  factors  and  the 
numbers  having  no  common  factor. 

Problems 
Find  the  L.  0.  M.  of  the  following: 

1.  6,  IS,  30,  42  3.  45,  63,  72,  99 

2.  16,  24,  64,  96  4.  14,  35,  42,  28 

Cancellation.  Certain  compulations  involving  division  can  be 
shortened  by  removing  or  cancelling  equal  factors  from  both  divi- 
dend and  divisor. 

Example 

If  32  units  of  product  sell  for  $57.60,  what  will  18  units  of  the  same  product 
sell  for  at  the  same  rate? 

Solution 

3.60 
9      UM 

?  =  32.40 


w 


Problems 

Using  cancellation,  divide: 

1.  27  X  48  X  96  X  38  2.  8  X  12  X  15  X  6 


19  X  16  X    9  X    2  5  X    4  X    3  X  18 

3.  If  15  tons  of  coal  cost  $258.00,  how  much  will  25  tons  cost  at  the  same 
rate? 

4.  A  ship's  provisions  will  last  36  men  for  216  days.     How  long  will  they 
last  124  men? 


CHAPTER!^ 

Common  Fractions 

Terms  explained.  A  unit  is  a  single  quantity  by  which  another 
quantity  of  the  same  kind  is  measured:  1  foot  is  the  unit  of  5  feet; 
1  barrel  is  the  unit  of  10  barrels;  1  acre  is  the  unit  of  40  acres,  and 
so  forth. 

These  integral  units  are  often  divided  into  equal  parts  known  as 
fractional  units,  as  i  ft.,  ^  bbL,  ^  A.,  and  so  forth. 

A  fraction  is  an  expression  for  one  or  more  of  the  equal  parts  of 
a  unit,  as  •£  ft.,  f  ft.,  §  bbl.,  %  A.,  and  so  forth. 

The  number  above  the  line  in  the  expression  of  a  fraction  is 
called  the  numerator',  the  number  below  the  line  is  called  the 
denominator. 

The  denominator  indicates  the  number  (and  hence  the  size)  of 
parts  into  which  the  unit  is  divided. 

The  numerator  indicates  the  number  of  these  parts  taken. 

A  proper  fraction  expresses  less  than  a  unit,  or  its  numerator  is 
less  than  its  denominator;  as,  $,  f,  $,  and  so  forth. 

An  improper  fraction  is  a  fraction  whose  numerator  is  equal  to 
or  greater  than  its  denominator;  as,  f,  f,  f,  and  so  forth. 

A  mixed  number  is  a  number  expressed  by  a  whole  number  and 
a  fraction;  as,  2l,  3£,  16f,  and  so  forth. 

Reduction  of  fractions.  Reduction  is  the  process  of  changing 
the  numerator  and  the  denominator  of  a  fraction  without  changing 
the  value  of  the  fraction. 

A  fraction  is  reduced  to  higher  terms  when  the  numerator  and 
the  denominator  are  expressed  in  larger  numbers. 

A  fraction  is  reduced  to  lower  terms  when  the  numerator  and 
the  denominator  are  expressed  in  smaller  numbers,  and  it  is  reduced 
to  its  lowest  terms  when  there  is  no  common  divisor  of  its  numerator 
and  denominator. 

Principle.  Multiplying  or  dividing  both  numerator  and 
denominator  of  a  fraction  by  the  same  number  does  not  change 
the  value  of  the  fraction.  Thus,  £-£  may  be  reduced  to  the  equiva- 
lent fraction  %  by  dividing  both  terms  by  4.  The  fraction  iri  has 
been  reduced  to  lower  terms.  Again,  i~f  may  be  reduced  to  the 
equivalent  fraction  f  by  dividing  both  terms  by  8.  Here  the 

39 


40  COMMON  FRACTIONS 

fraction  if  has  been  reduced  to  lowest  terms,  since  2  and  3  do  not 
have  a  common  divisor. 

Conversely,  $  may  be  changed  to  an  equivalent  fraction  whose 
denominator  is  24  by  multiplying  both  terms  by  8  (obtained  by 
dividing  24  by  3),  or  £f\  Thus,  the  fraction  f  has  been  reduced 
to  a  higher  given  denominator. 

Mixed  numbers.  It  is  sometimes  desirable  to  change  a  mixed 
number  to  an  improper  fraction,  or,  conversely,  to  change  an 
improper  fraction  to  a  mixed  number. 

To  change  a  mixed  number  to  an  improper  fraction.  Multiply 
the  whole  number  by  the  denominator  of  the  fraction,  add  the 
numerator,  and  place  the  sum  over  the  denominator,  thus,  3^  is 
Y,  4|  is  V,  and  6*  is  V>. 

To  change  an  improper  fraction  to  a  whole  or  a  mixed  number, 
divide  the  numerator  by  the  denominator;  thus  V2  is  4,  f  is  l£, 
Vis  1|  or  U,  and  V-is4f. 

Problems 

1.  Reduce  to  lowest  terms:  A,  A,  A,  H,  iff,  U,  *t,  M,  if,  li 

2.  Change  to  equivalent  fractions  having  denominators  as  indicated: 

i  to  Hths  i  to  15ths  t  to  25ths 

1  to  Oths  i  to  24ths  A  to  4Sths 

$  to  20ths  |  to  24ths  |  to  32nds 

i  to  Sths  |  to  36ths  -fy  to  36ths. 

3.  Reduce  to  equivalent  fractions  whose  denominators  are  24:  TV  I,  I,  I, 
it. 

4.  Change  to  improper  fractions:  4t,  3j,  li,  7i,  8|,  6i,  3|,  5|,  5f,  9|. 

5.  Change  to  whole  or  mixed  numbers:  V,  ¥,  V,  ¥,  H,  V,  f ,  V,  ff,  ¥• 

6.  Is  the  number  of  fractional  units  increased  or  decreased  when  we  reduce 
A  to  •}?     Is  the  size  of  the  fractional  unit  increased  or  decreased  when  we  reduce 

A  to  t? 

Addition  and  subtraction  of  fractions.  Similar  fractions  are 
fractions  that  have  a  common  denominator.  Only  similar  frac- 
tions can  be  added  or  subtracted. 

To  add  fractions,  reduce  the  fractions  to  similar  fractions 
having  a  common  denominator  and  add  the  numerators. 

To  subtract  fractions,  reduce  the  fractions  to  similar  fractions 
having  a  common  denominator  and  subtract  the  numerators. 
Example  Solution 


Add:  i,  |,  and 


12 


6 

8 

JJ 

17 


COMMON  FRACTIONS  41 

Explanation.  Inspection  shows  that  12  is  the  least  common  denominator. 
i  is  A.  £  is  A,  and  i  is  A-  Adding  the^umerators  of  the  similar  fractions 
gives  17,  and  fj  is  1  A-  /, 

Example  <'   -  Solution 

Subtract:  ^  t  =  ft 

I  ~  A  (  ft  —  A  -  A 

Multiplication  of  fractions,  (a)  To  multiply  a  fraction  by  a 
whole  number,  multiply  the  numerator  or  divide  the  denominator 
of  the  fraction  by  the  whole  number. 

Example  Solution 

Multiply  6  X  A-  6  X  A  =  f  8  =  2i 

or 
12  -f-  6  =  2,  and  -J  -  2i 

(6)  To  multiply  a  whole  number  by  a  fraction,  multiply  the 
whole  number  by  the  numerator  of  the  fraction  and  write  the 
product  over  the  denominator.  Cancel  when  possible. 

Example  Solution 

•  >  Find  f  of  35.  I  X  35  =  Y  -  14 

or 
'  7 

2  X  M  =  14 

»  X  1 

(c)  To  multiply  a  fraction  by  a  fraction,  multiply  the  numer- 
ators to  obtain  the  numerator  of  the  answer,  and  multiply  the 
denominators  to  obtain  the  denominator  of  the  answer.  Cancel 
when  possible. 

Example  Solution 

Find  io,'  It.  £xi&=28  =  & 

or 
5 


__ 

3  x  n    8 

8 

(d)  To  multiply  a  mixed  number  by  a  mixed  number,  reduce 
each  mixed  number  to  an  improper  fraction  and  proceed  as  in  (c). 

Example  Solution 

Find  the  product  of:  i  X  V  =  W 

3i  X4i 

Find  the  product  of:  4 


42 


COMMON  FRACTIONS 


Problems 

Find: 

1.  9  X  A  3.  |  of  35 

2.  24  X  1  4.  A  of  16 

Division  of  fractions,  (a)  To  divide  a  fraction  by  a  whole 
number,  divide  the  numerator  or  multiply  the  denominator  by  the 
whole  number. 


6.  |  of  H 
6.  £  of  f  f 


7.  3i  X  4* 

8.  12f  X  8  i 


Example 
Divide  H  by  ,5. 


Solution 
25  -r-  5  =  5    Answer: 


5 


_ 

2X  x 


or 

5 

2S 


(fr)  To  divide  any  quantity  —  a  whole  number,  a  mixed  number, 
or  a  fraction,  by  a  fraction,  invert  the  divisor  and  multiply. 

Example  Solution 

Divide  8  by  f  .  4 


Example 
Divide  16|  by  £. 


Example 
Divide*  by*. 


1X2" 
Solution 

13       3 

W  X  0 

TxT 

2 

Solution 


30 


_        1 

"  K)2 


Problems 


Divide: 

a.  M  by  3 

b.  -H  »>y  9 


c.  8  by  | 

d.  9  by  ? 


e.  16j  by 

f.  1SJ  l)y 


g.  3i  by  li 
h.  9j  by  3i 


1.  How  many  pieces  of  wire  each  Sj  inches  long  can  be  cut  from  40  feet 
of  wire? 

2.  If  1  of  a  ton  of  coal  costs  $12.75,  what  is  the  cost  of  one  ton? 

3.  How  many  sash  weights  each  weighing  2-J-  pounds  can  be  cast  from  120 
pounds  of  pig  iron,  if  i  of  the  quantity  of  pig  iron  is  wasted  in  the  casting  oper- 
ation? 

4.  A  room  is  18f  feet  long  and  14^  feet  wide.     The  width  of  the  room  is 
what  part  of  the  length  of  the  room? 

5.  A  carpenter  has  a  board  that  is  20  feet  long,  but  it  is  -J-  longer  than  he 
needs.     How  long  a  board  does  he  need? 

6.  What  is  the  cost  of  7£  tons  of  coal  at  $14|  a  ton? 


COMMON  FRACTIONS 


43 


7.  A  house  and  lot  are  valued  at  $6,600.     If  the  lot  is  worth  f  as  much  as 
the  house,  what  is  the  value  of  each? 

8.  If  a  man  can  earn  $2f  a  day,  how  long  will  it  take  him  to  earn  $46-f  ? 

9.  A  table  is  20  feet  long.     How  many  people  can  be  seated  on  the  two 
sides  if  you  allow  if  feet  for  each  person? 

10.  Henry's  time  book  shows  that  his  working  time  for  one  week  was  as 
follows:  Monday,  7^  hours;  Tuesday,  8-J  hours;  Wednesday,  S  hours;  Thurdsay, 
9^  hours;  Friday,  8i  hours;  Saturday,  Of  hours. 

He  is  paid  straight  time  for  8  hours  or  less  and  time  and  a  half  for  hours  in 
excess  of  S  each  day  other  than  Saturday,  when  he  receives  double-time  pay  for 
hours  worked.  How  much  did  he  earn  at  $|  an  hour? 

11.  The  shipping  clerk  reported  that  he  dispatched  320  packages  averaging 
2Sj  pounds  each.     What  was  the  total  weight  of  packages  dispatched? 

12.  A  cubic  foot  of  water  weighs  62i  pounds,  and  there  are  approximately 
7^  gallons  to  the  cubic  foot.     Estimate  the  weight  of  water  that  a  10-gallon  keg 
will  contain. 

To  find  the  product  of  any  two  mixed  numbers  ending  in  £. 

(a)  Wlien  the  sum  of  the  whole  numbers  is  an  even  number.  To 
the  product  of  the  whole  numbers,  add  one-half  of  their  sum,  and 
annex  i. 


Multiply  24 £  by 


192 

16 

208 1 


Example 


Solution 


(8  X  24) 

(i  of  the  sum  of  24  and  8) 

(-J  annexed) 


Multiply: 

1.  Si  by  4i 

2.  12£  by  8i. 


Problems 

3.  28iby  12J. 

4.  H>i  by  14J. 


6.  18ihy  18i. 
6.   10i  by  34 i. 


(b)  When  the  sum  of  the  whole  numbers  is  an  odd  number.  To 
the  product  of  the  whole  numbers,  add  one-half  of  their  sum,  less 
1,  and  annex  f. 


Multiply  151  by  6l. 


Example 


Solution 


90       (6  X  15) 

_10_     (i  of  15  +  6  -  1) 

100|     (I  annexed) 


44  COMMON  FRACTIONS 

Problems 

Multiply: 

1.  18i  by  5i  3.  38£  by  5i  6.  23*  by 

2.  l4  by  7£.  4.  13i  by  8|.  6.  19£  by 

To  multiply  a  mixed  number  by  a  mixed  number. 

Example 
Multiply  524i  by  27f 


14148  6     =  common  denominator  of  fractions 

1741  4] 

13^  3  }  =  numerators  of  changed  fractions 

_  *  lj 

14336^  f     =  li 

Explanation.  Multiply  524  by  27,  obtaining  the  first  part  of  the  answer, 
14,148.  Next,  take  i  of  524,  obtaining  174|.  Then  take  £  of  27,  obtaining  13^. 
Finally,  take  -g-  of  ^,  obtaining  £.  Add  the  four  partial  products,  and  the  com- 
plete product  is  14,336^. 

Problems 

Multiply: 

1.  247|  by  39i  3.  59|  by  15|.  6.  181f  by  6f  . 

2.  849i  by  28i  4.  176f  by  34f  .  6.  56£  by  12|. 

Decimal  fractions.  A  decimal  fraction  is  a  fraction  whose 
denominator  is  some  power  of  ten,  indicated  by  a  decimal  point 
placed  just  to  the  right  of  the  units'  place.  Thus,  .1  is  TO~,  .05  is 
Tthr,  and  .25  is  tVV  or  i. 

Addition  and  subtraction.  To  add  or  to  subtract  decimals, 
write  the  numbers  so  that  the  decimal  points  fall  vertically  and 
proceed  as  in  whole  numbers. 

Example  Solution 

Add:  .01,  4.72,  78.25,  and  .005.  .01 

4.72 
78  25 
.005 

82.985 

Example  Solution 

Subtract:  47.02 

47.02  -  .92  _  92 

46.10 
Problems 

1.  Add:  25.679,  .0356,  2.78,  and  .017. 

2.  Add:  136.2,  28.348,  .004,  and  1.356. 


COMMON  FRACTIONS  45 

3.  Subtract:  13.48  from  27.049. 

4.  Subtract:  .003  from  .47. 

Multiplication.  To  multiply  decimal  fractions,  multiply  as  in 
whole  numbers  and  point  off  as  many  decimal  places  in  the  product 
as  there  are  places  in  both  multiplicand  and  multiplier. 

Example  Solution 

Multiply  3.06  X  .8.  3  06 

.8 

2.448 

Explanation.  Since  there  are  3  decimal  places  in  both  the  multiplicand  and 
the  multiplier,  point  off  three  decimal  places  in  the  product. 

Example  Solution 

Multiply:  23.8564 

23.8564  by  6.72  6  72 

477128 
1669948 
1431384 

160315008 

Explanation.  As  there  are  6  decimal  places  in  the  multiplicand  and  the 
multiplier,  point  off  six  decimal  places  in  the  product.  The  answer  is  160.315008. 
Rough  check:  24  X  7  =  168. 

Division.  Proceed  as  with  whole  numbers,  annexing  zeros  to 
the  dividend  if  necessary.  The  number  of  decimal  places  in  the 
quotient  must  equal  the  number  in  the  dividend  minus  the  number 
in  the  divisor. 

Example  Solution 

Divide :  .  24)54 . 864(228 . 6 

54.864  by  .24  6  8 

2  06 
144 
0 

Explanation.  Divide  by  writing  the  remainders  only.  The  quotient  is  2286. 
As  there  are  three  decimal  places  in  the  dividend  and  two  decimal  places  in  the 
divisor,  point  off  one  decimal  place  in  the  quotient.  The  answer  is,  therefore. 
228.6. 

Example  Solution 

Divide:  49.099 

256.7894  by  5.23  5 . 23)256.78940 

47  58 
5194 
4870 
163 


46  COMMON  FRACTIONS 

Explanation.  Predetermine  the  placing  of  the  decimal.  As  there  are  two 
decimals  in  the  divisor,  place  the  decimal  point  over  the  third  decimal  place  in 
the  dividend.  Place  the  first  figure  of  the  quotient  over  the  last  figure  of  the 
partial  dividend.  One  zero  has  been  annexed  to  the  dividend  in  order  to  obtain 
a  quotient  to  three  decimals.  Rough  check:  49  X  5  =  245. 

Problems 

Multiply:  Divide: 

1.  34.278  X  1.45  6.  5.8769  by  1.34 

2.  395.264  X  .035  7.  .0084  by  1.5 

3.  74.26  by  .00423  8.  45.87  by  .0056 

4.  .056  by  .083  9.  8.45  by  25.3 
6.  18.42  X  .045  10.  956  by  4.87 

To  abbreviate  decimal  multiplication  when  a  given  number  of 
decimal  places  is  required.  It  is  a  waste  of  time  to  carry  out 
decimal  multiplication  to  a  denomination  smaller  than  that  in 
which  the  data  are  expressed;  often  it  is  unnecessary  to  carry  it 
beyond  the  third  or  fourth  decimal. 

Example 

Multiply  4.7892  by  3.1705,  and  obtain  the  answer  correct  to  four  decimal 
places. 

Solution 


4 
5 

7892 
6713 

=  multiplicand 
=  multiplier  reversed 

14~ 

^3676 

=  4.7892  X  3. 

,4789 

2 

=4.  7892  X     .1 

,3352 

4 

i 

=  4.  7802  X      07 

,   287 

3 

% 

2      =4  7B02  X      006 

23 

9 

a 

3 

00  =  4.7302  X      0005 
%  0 

15. 

,2128 

0 

Explanation.  The  multiplier,  3.1765,  is  written  in  the  reverse  order,  56713, 
the  units'  digit  being  placed  under  the  lowest  order  of  the  multiplicand  that  is 
desired  in  the  product — ten  thousandths.  Multiply  by  each  digit  of  the  reversed 
multiplier,  beginning  with  that  digit  of  the  multiplicand  which  stands  directly 
above  the  digit  of  the  multiplier  used,  taking  care  to  include  the  digit  carried 
over  from  the  multiplication  of  the  one  (or  two)  rejected  digits  at  the  right. 

Example 

Multiply  4.7869347  by  7.25,  and  obtain  the  product  correct  to  three  decimal 
places. 

Solution 

4  786  9347 
527 


33  508  3  =4  7869  X  7 
.957  2  =4  7860  X  2 

239  0  =  4.7800  X  5 

34  704  5 


COMMON  FRACTIONS  47 

Problems 

Multiply: 

1.  5.987654  by  3.147,  obtaining  the  product  correct  to  the  4th  decimal. 

2.  3.590f  by  14.57,  obtaining  the  product  correct  to  the  3rd  decimal. 

3.  184.2Sy  by  3.145,  obtaining  the  product  correct  to  the  4th  decimal. 

4.  44.187542  by  6.2434,  obtaining  the  product  correct  to  the  3rd  decimal. 

Division  of  decimals.  Division  of  decimals  may  often  be 
abbreviated,  especially  when  the  divisor  is  given  to  a  greater 
number  of  decimal  places  than  are  contained  in  the  dividend,  and 
when  only  three  or  four  decimal  places  are  essential  in  the  quotient. 

Example 

Divide  4.39876  by  2.4871934,  and  obtain  the  quotient  correct  to  three  decimal 
places. 

Kohdion 

Ordinary  Method  Abbreviated  Method 

2  4871934)4  39S  7600       (1J76S  2.487  2W)4  398  7(1   768 

~2  487   1934  '  2_487  2 

1  911    56660  I"911   5 

1_741_0353S  L741J? 

~T70~53l220  ~T70~5 

149231604  149  2 


19  8975472  MM) 

^1  4020688  T~4 

Explanation.  Observation  of  the  ordinary  method  shows  that  the  third 
decimal  place  in  the  quotient  is  not  affected  by  the  digit  in  the  third  decimal 
place  in  the  divisor  (except  through  the  digits  carried). 

Since  the  units'  digit  of  the  divisor  is  contained  in  the  units'  digit  of  the 
dividend,  the  first  digit  in  the  quotient  is  in  the  units'  place,  and  as  three  decimal 
places  are  required,  the  quotient  will  contain  four  digits.  Therefore,  the  last 
four  digits  of  the  divisor  will  not  affect  the  quotient,  except  through  the  digits 
carried  over. 

The  first  four  digits  of  the  divisor,  2.487,  are  contained  once  in  4.398.  Multi- 
plication of  that  part  of  the  divisor  used,  by  the  quotient  digit  (including  the 
digit  carried  over  from  the  one  or  two  following  digits  —  in  this  case  considering 
the  9  as  a  unit  and  adding  it  to  the  1,  making  2)  gives  2487  2,  and  this  result 
deducted  from  the  previous  dividend  leaves  1911  5  for  the  new  dividend. 

Cancel  the  right-hand  digit,  7,  of  the  divisor,  and  divide  1911  by  248,  obtain- 
ing the  quotient  7.  Multiplying  the  divisor  by  7  (and  including  the  carrying 
digit)  gives  1741  0,  and  subtracting  leaves  a  new  dividend  of  170  5. 

Cancel  another  digit,  8,  of  the  divisor,  and  divide  by  24.  This  is  contained 
6  times  in  170.  The  product  (including  the  digit  carried  over)  is  149  2,  and 
this  product  subtracted  leaves  a  new  dividend  of  21  3. 


48  COMMON  FRACTIONS 

Cancel  another  digit,  4,  of  the  divisor.  Divide  21  by  2,  using  the  carried 
digit;  the  result  is  8.  The  new  product  is  19  9,  and  this  product  subtracted 
from  21  3  leaves  a  remainder  of  1  4. 

Example 

Divide  8.47  by  31.76983476,  and  obtain  the  quotient  correct  to  three  decimal 
places. 

Solution 
31.769  83476)8  4700  (.260 

6  3540     =".31.760     (8)  X  .2,  or  6.3539(6).     Use  6.3540. 
2  1160 

1J90G2     =     31.76     (.95)  X  .06,  or  1.9001(8).     Use  1.9062. 
"2098 

1906     =     31.7       (GO)  X  .006,  or  .1906(1).     Use  .1906. 
"192 

Problems 
Divide: 

1.  4.3954  by  37.265872,  obtaining  the  quotient  correct  to  the  3rd  decimal. 

2.  65.157  by  4.4976348,  obtaining  the  quotient  correct  to  the  4th  decimal. 

3.  1.297648  by  15.782643,  obtaining  the  quotient  correct  to  the  3rd  decimal. 

4.  3.489765  by  .28765431,  obtaining  the  quotient  correct  to  the  3rd  decimal 

To  change  a  decimal  fraction  to  an  equivalent  common  fraction. 

Write  the  denominator  of  the  decimal,  omit  the  decimal  point,  and 
reduce  to  lowest  terms.  Thus,  to  reduce  to  common  fractions  in 
lowest  terms  or  to  mixed  numbers  : 


.75  =  AV  =  *  -025  =  T?8ir  =  A 

6.25  =  6T%  =  6£        4.125  =  4^-  =  4j 

To  change  a  common  fraction  to  a  decimal.  A  common  frac- 
tion may  be  regarded  as  an  indicated  division.  Thus:  -§-  may  be 
regarded  as  2  -5-  5;  therefore,  f  expressed  as  a  decimal  is  .4;  simi- 
larly, |  is  .14$,  f  is  .375,  and  TV  is  .4375. 

Aliquot  parts.  An  aliquot  part  of  any  number  is  a  number  that 
is  contained  in  it  an  integral  number  of  times.  Thus,  5,  10,  20, 
and  50  are  aliquot  parts  of  100;  that  is,  5  =  ^V  of  100,  10  =  yV 
of  100,  and  so  forth. 

The  use  of  aliquot  parts.  As  a  means  of  saving  time  in  multi- 
plication and  in  division,  it  is  useful  to  know  the  decimal  equiva- 
lents of  common  fractions,  or,  conversely,  to  know  the  common 
fraction  equivalents  of  decimal  fractions.  Aliquot  parts  are  of 
value  in  addition  and  subtraction  if  an  adding  machine  or  a  calcu- 
lating machine  is  used,  because  machines  are  not  adapted  for 
general  work  involving  common  fractions. 


COMMON  FRACTIONS  49 

TABLE  OF  ALIQUOT  PARTS  OF  1 


Common 

Decimal 

Common 

Decimal 

Fraction 

Equivalent 

Fraction 

Equivakn 

* 

.50 

* 

.11* 

* 

.33* 

A 

.10 

i 

.66| 

A 

.09^ 

.25 

.08* 

I 

.75 

A 

.4lf 

* 

.20 

TS 

.58* 

* 

.16* 

.91* 

* 

.83* 

5 

.06* 

* 

.  14y 

A 

.06* 

2 

28-r 

A 

.18* 

I 

.42* 

A 

.31* 

4 

57* 

Tff 

.43f 

I 

71? 

IT 

.56* 

* 

.85* 

t? 

.681 

i 

.12* 

ri 

.93* 

3 

37* 

.04 

f 

.  62* 

A 

.03* 

ff 

X7i 

A 

-09| 

The  fractions  in  the  above  table  can  be  extended  as  decimals 
as  far  as  the  work  demands. 

Problems 

Express  the  following  aa  decimal  fractions;  non-terminating  fractions  should 
be  carried  to  the  sixth  decimal  place  and  the  common  fraction  annexed: 

21  1  3  5  11 

31126  2 

5  1  1  15  1  1 

•ff  TIT  T  Tff  TT  TO" 

812  13  1 

V          T8          ^  TT          T  W 

54  \\  5  5  3 

T  T  I  (T  ~S  <5  "Saf 

Multiplication  by  aliquot  parts. 

Example 
Find  16*%  of  $475.34. 

Solution 

6)$475_34 
"  $79.22 

Explanation.    Since  .16*  equals  i,  find  -g-  of  $475.34. 

Example 
Find  the  cost  of  256  units  at  37^  each. 

Solution 

256  X  |  X  $1  =  $96 
Explanation.    37i?f  is  |  of  $1.    Therefore,  256  X  f  X  $1  =  $96. 


50 


COMMON  FRACTIONS 


Problems 

Extend  the  following  items  mentally: 


1.  72 

@  .T2* 

9. 

18 

(W, 

.33* 

17. 

64  @ 

.25 

25. 

72 

@  .83* 

2.  45 

@  -lit 

10. 

39 

(r/> 

.6ft* 

18. 

27  <fo 

.22f 

26. 

32 

(«>  .87* 

3.  24 

©  .08* 

11. 

55 

($ 

.09ix 

19. 

32  © 

.18f 

27. 

36 

&  .411 

4.  36 

®  .50 

12. 

16 

© 

.75 

20. 

96  & 

•03i 

28. 

27 

^  .44f 

6.  15 

@  .06f 

13. 

49 

(n\ 

.28} 

21. 

48  (a, 

.56i 

29. 

12 

(«;   .75 

6.  75 

fe  .93* 

14. 

32 

4 

.43* 

22. 

60  (", 

.58^ 

30. 

14 

@.  .07| 

7.  48 

@  .16* 

15. 

28 

«!> 

.57| 

23. 

48  (<i, 

.37* 

31. 

18 

(tn  .16* 

8.  32 

@  .06* 

16. 

24 

<& 

.62* 

24. 

35  («, 

•1472 

32. 

16 

^  .87  ^ 

Division  by  aliquot  parts.  It  is  diffirult  to  divide  a  number  by 
a  mixed  number.  If  the  divisor  is  an  aliquot  part,  the  quotient 
may  be  found  by  multiplication,  as  follows: 

Example 
Divide  4,875  by  16*. 

Explanation.     Sinco  16*  is  Jr  of  100,  divide  4,875  by  J  of  100,  Solution 

or  ^ir2-.     This  is  the  same  as  multiplying  by  n5#.     Therefore,  divide  48  75 

by  100  by  pointing  off  two  decimal  places  from  the  right,  and  multi-  6 

ply  the  result  by  6.     The  answer  is  292.50,  or  292^.  292 ~50 


Example 

The  production  cost  of  1,250  units  is  $3,170. 
unit. 


Find  the  cost  per 


Kxplanation.  1,250  is  i  of  10,000.  Divide  S3, 170  by  10,000 
by  pointing  off  4  decimal  places  from  the  right;  then  multiply  the 
result  by  8.  The  cost  per  unit  is  found  to  be  $2.536. 


Solution 
3170 

s 

2~5360 


Divide: 

1.  1, 342  by  Hi 

2.  2,578  by  12i 


Problems 


3.  3, 126  by  33i- 

4.  384  by  25. 


5.  158  by  6i. 

6.  4,275*by  14f 


Problems 
1.  A  manufacturer  pays  dividends  amounting  to  fs- 


his  capital.     If  the 
How  many  cords 


dividends  amount  to  $37,500,  what  is  the  capital? 

2.  A  fuel  dealer  had  36  cords  of  wood  and  sold  |  of  it. 
did  he  sell? 

3.  If  a  merchant  buys  an  article  for  $12^  and  sells  it  for  $16,  the  profit  is 
what  fraction  of  the  selling  price?     What  fraction  of  the  cost  price? 

4.  A  crate  containing  10  dozen  oranges  cost  $4.50.     If  they  are  sold  at  the 
rate  of  65  cents  a  dozen,  but  i  dozen  are  spoiled,  the  profit  is  what  fraction  of  the 
selling  price? 

5.  A  man  has  $37^  and  spends  $12^.     What  fraction  of  his  money  does 
he  keep? 


FRACTIONS  51 

6.  A  factory  normally  employed  48  men.     During  a  dull  period  16  received 
temporary  lay-offs.     What  fraction  of  the  force  continued  to  work? 

7.  The  last  reading  of  a  gas  meter  was  67,324  cu.  ft.;  the  previous  reading 
was  64,815  cu.  ft.     At  $1.45  a  thousand  cubic  feet,  find  the  amount  of  the  gas  hill. 

8.  An  investment  of  $18,000  produces  an  annual  income  of  $720.     At  the 
same  rate,  what  should  an  investment  of  $25,000  produce? 

9.  Tires  costing  §18.75  were  installed  when  the  speedometer  registered 
18,985  miles.    The  four  tires  were  replaced  \\hen  the  speedometer  registered 
34,652  miles.     SI. 00  was  allowed  for  each  old  tire.     What  was  the  average  tire 
cost  per  mile,  correct  to  the  nearest  tenth  of  a  mill? 

10.  An  excavation  8  feet  in  depth  required  the  removal  of  5,328  cu.  ft.  of 
earth  and  rock.  The  average  depth  of  earth  was  5  ft.,  and  the  cost  of  earth 
removal  was  §1-}  a  cu.  yd.  The  remainder  was  lock  and  cost  $4g-  a  cu.  yd.  for 
removal.  What  was  the  cost  of  making  the  excavation? 


CHAPTER 
Percentage 

Relation  between  percentage  and  common  and  decimal  frac- 
tions. Percentage  is  a  continuation  of  the  subject  of  fractions. 
It  is  the  process  of  computing  by  hundredths,  but  instead  of  the 
term  hundredths,  the  Latin  expression  per  cent  is  used.  The  sign- 
(%)  generally  replaces  the  words  per  cent,  thus,  5%,  10%,  and  so 
forth. 

Any  per  cent  may  be  expressed  either  as  a  common  fraction  or 
as  a  decimal,  thus: 

Common  Fraction        Decimally 

1%.  -  TOO  01 

5%   .  TiU  05 


100% 

300%.. 

*%..   .  ,,2)()  or  10;)0  .00*  or  .005 

•°5- Too  "r  .0,000       °005 

Care  should  be  taken  in  writing  per  cents.  Do  not  write  both 
the  sign  and  the  decimal  point;  thus,  2%  and  .02  are  the  same, 
but  2%  and  .02 %  are  widely  different,  since  the  first  is  equivalent 
to  ro  and  the  second  to  Woo- 

Applications.  Percentage  admits  of  applications  in  many 
fields.  Business  operations  are  guided  by  carefully  prepared 
statistics,  and  the  relationships  of  items  in  statistics  are  often  more 
clearly  reflected  when  they  are  expressed  in  terms  of  percentage. 
There  are  numerous  problems  involving  percentage  besides  those 
having  to  do  with  financial  considerations,  such  as  finding  the  per 
cent  of  increase  or  decrease  in  volume;  per  cent  of  shrinkage  of 
material;  per  cent  of  waste  in  manufacturing  operations;  per  cent 
of  yield  of  crops. 

Definitions.  The  base  is  the  number  or  quantity  represented 
by  100%.  The  base  may  be,  for  example,  total  sales,  total 

53 


To  0" 

12J    125 
H)0  °l  1000 

1  00 

100 

1 

MOO 
100 

3. 

1 

100  or  1006 

TO*  (  .   5 

100  OI  10,000 

54  PERCENTAGE 

expenses,  the  face  value  of  a  note,  the  par  value  of  a  bond,  pounds 
of  material  used,  capacity,  and  so  forth. 

The  rate  is  the  number  of  hundredth*,  or  the  per  cent.  The 
rate  may  be,  for  example,  6%  or  25%,  which  are  written  decimally 
as  .06  and  .25. 

The  percentage  is  the  product  of  the  base  and  the  rate.  The 
percentage  may  be,  for  example,  the  interest  cost  of  a  sum  of  money, 
the  departmental  portions  of  an  expense  item,  the  increase  in 
pounds  of  material  used,  and  the  like. 

Fundamental  processes.  In  percentage  and  its  application, 
three  fundamental  mathematical  principles  are  involved,  namely: 
(1)  to  find  a  given  per  cent  of  a  number;  (2)  to  find  what  per  cent 
one  number  is  of  another;  and  (3)  to  find  a  number  when  a  certain 
per  cent  of  it  is  known. 

Computations.  Computations  in  percentage  are  based  on 
these  principles. 

Principle  1.  The  percentage  is  the  product  of  the  base  and  the 
rate. 

Base  X  Kate  =  Percentage 

Example 

6%  interest  on  $500  is  $30.     (500  X  .00  =  30) 

Problems 

In  the  following,  convert  the  per  cent  either  to  a  common  fraction  or  to  a 
decimal  fraction,  whichever  is  the  easier. 
Find: 

1.  25%  of  5,280  ft,  6.  2f  %  of  180  11)8. 

2.  10%  of  846  Ibs.  7.  £'  'c  of  240  pil. 

3.  16f  %  of  24  bu.  8.  -£%  of  $5,000. 

4.  37i%  of  $60.  9.  20%  of  95  yds. 

5.  80%  of  120  pp.  10.   14f ' ,',  of  42  in. 

11.  If  an  expense  item  of  $16.00  is  reduced  6-f%,  what  will  be  the  amount  of 
this  item  after  the  reduction? 

12.  A  commission  of  12-J%  was  earned  on  a  $240  sale.     What  was  the 
commission? 

13.  A  sample  of  grain  showed  2$%  weed  seed.     How  many  bushels  of  weed 
seed  are  in  600  bushels  of  this  grain? 

14.  An  item  sells  for  40  cents.     What  will  be  the  selling  price  after  a  reduction 
of  15%? 

16.  Anticipated  requirements  for  copper  will  exceed  the  manufacturer's  stock 
by  35%.  If  185  pounds  are  on  hand,  how  many  pounds  will  have  to  be 
purchased? 

Principle  2.  The  rate  may  be  found  by  dividing  the  percentage 
by  the  base. 

Percentage  -*•  Base  =  Rate. 


PERCENTAGE  55 

Example 

$30  -*•  $500  =  .06  or  6%. 

Problems 

In  the  following  find  what  per  cent  of: 

1.  72  is  24  6.     12  is  20 

2.  60  is  50  7.    64  is  S 

3.  180  is  120  8.     90  is  10 

4.  360  is  90  9.  150  is  25 
6.     50  is  20                                          10.  125  is  25 

11.  Last  year's  taxes  on  a  house  were  $520.     This  year's  taxes  were  $640 
What  per  cent  were  this  year's  taxes  of  last  year's  taxes? 

12.  A  pile  of  lumber  contained  4,500  feet,  and  3,300  feet  were  used.     What 
per  cent  remained? 

13.  Wages  are  increased  from  §1.50  an  hour  to  $1.75  an  hour.     Find  the 
per  cent  of  increase. 

14.  A  new  style  of  packaging  reduced  the  shipping  weight  from  130  Ibs.  to 
121  Ibs.     What  was  the  per  cent  of  saving  in  shipping  weight? 

15.  The  inspector  rejected  5  items  out  of  140  produocd.     What  was  the 
per  cent  of  rejects? 

Principle  3.     The  ba.se  may  be  found  by  dividing  the  percentage 
hy  the  rate. 

1'eiccntagc  4-  Rate  ==  Base. 

Example 

30  -5-  .06  =  500. 

Problems 

Find  the  number  of  which: 

1.  25  is  20%  6.     SO  is  43% 

2.  125  is  16|%  7.  374  is  17% 

3.  240  is  75%  8.  375  is  \% 

4.  48  is  i%  9.     4i  is  J% 
6.    72isl2i%                                   10.    20  is  40  % 

11.  The  fire  insurance  premium  on  a  house  was  $22.50.     The  house  was 
insured  for  80%  of  its  value  at  £%•     Find  the  value  of  the  house. 

12.  Sales  increased  each  year  over  .the  preceding  year  as  follows:  15%  the 
second  year,  20%  the  third  year,  and  25 %  the  fourth  year.     If  the  fourth  year's 
sales  were  $21,562.50,  what  were  the  first  year's  sales? 

13.  A  bankrupt  can  pay  his  creditors  72  cents  on  the  dollar.     If  his  assets 
are  $13,475.28,  what  are  his  liabilities? 

14.  The  gross  income  of  a  rental  property  is  $1,HOO  a  year.     P^xpenses  are 
$500.     If  the  net  income  is  a  return  of  6^%  on  the  investment,  find  the  value 
of  the  property. 


PERCENTAGE 

16.  One  workman  completes  a  unit  in  7^  hours.  Another  workman  com- 
nlfctes  a  similar  unit  in  5f  hours.  The  first  workman  took  what  per  cent  more 
time  than  the  second  workman  to  complete  the  unit? 

Miscellaneous  Problems 

1.  A  machine  that  cost  $50  was  marked  up  30%.     What  was  the  marked 
price? 

2.  After  a  clerk's  salary  was  increased  (>i%,  he  received  $S50  a  year.     What 
was  his  former  salary? 

3.  A  4-apartment  building  cost  $18,000.     Repairs  average  l^r%  of  the  cost; 
taxes,    2^-%;   insurance   on    90%   valuation,    f%;   other   expenses   amount   tc 
$114.25.     What  should  the  annual  rental  income  be  in  order  to  return  the 
owner  8%  on  his  investment?     What  should  be  the  average  monthly  rental  of 
each  apartment? 

4.  A  product  shrinks  10%  in  processing.     TIow  many  pounds  of  raw  material 
will  be  required  to  process  252  pounds  of  finished  product? 

5.  A  creditor  received  $037.73  from  a  bankrupt  estate  paying  68  cents  on  tlu 
dollar.     What  was  the  creditor's  loss  on  the  account? 

Daily  record  of  departmental  sales.  The  following  tabulation 
is  designed  to  show  the  total  daily  sales  by  departments,  and  the 
total  sales  for  the  week,  both  by  departments  and  for  the  business 
as  a  whole.  After  Saturday's  sales  have  been  entered,  the  total 
departmental  sales  for  the  week  may  be  found  and  also  the  per  cent 
that  each  department's  sales  is  of  total  sales.  The  per  cent  that 
each  day's  sales  is  of  total  sales  for  the  week  is  also  obtainable. 

DAILY  RKCORD  OF  DKPAUTM i:\TAL  SALES 

Dcpt.     Mon.        Tues.        \Vc<L       Thurs.        Fri.          Sat.         Total      I'crCcnt 

A     $475. SO  $275  S3  $329  SO  $424  S3  $3S7  92  $412  15 .  ... 

B       324.  IS     174  82    274  19     2S5  27     304.14     319  2S      .. 
C       456.19     259  SO     179  SO     25S  24     2SO  39     30574... 
1)       421  40     20S  75     142.50     2SO  22     17S  90     200  57 
K         175  00      125  34      150  S5     210  05      102  50      1S7   50  .  .  .. 

Total    ._         .       .    _. ~~  _--""""--         _  -  r~  ltMM)0'p 

pcr  -         ~- 

Cent 100  00% 

Problem 

Prepare  a  form  similar  to  the  above,  enter  the  sales  in  the  proper  columns, 
and  find:  (a)  the  total  sales  for  each  day  in  all  departments  (add  downward); 
(b)  the  total  sales  for  each  department  for  the  week  (add  across);  (c)  in  two 
ways,  the  total  sales  in  all  departments  for  the  entire  week;  (d)  the  per  cent  of 
grand  total  sales  made  each  day  (total  for  each  day  divided  by  the  grand  total) ; 
(e)  the  per  cent  of  grand  total  sales  made  in  each  department  (total  of  each 
department  divided  by  the  grand  total). 

Per  cent  of  returned  sales  by  departments.  In  some  lines  of 
business  it  is  important  to  keep  a  close  check  on  the  volume  of 


PERCENTAGE 


57 


returned  sales.     This  may  be  done  advantageously  by  means 
of  per  cents  derived  from  tabulated  results. 

Problem 

Prepare  a  form  similar  to  the  following,  enter  the  data,  anil  find:  (a)  the  net 
%aies  for  each  department,  and  the  net  sales  for  all  the  departments;  (6)  the 
per  cent  of  returned  sales  in  each  department,  and  the  total  per  cent  of  returned 
sales. 


SALES  AND 


SALES  BY  DKPARTMENTS 


Dept. 
A 

Saks 
$  24,Sf>3  95 

Returned             Net 
Sales                Sales 
$    756  S2     

Per  Cent 
of  Sales 
Returned 

B 
C 
D 

110,356  SO 
53,76S  21 
16,135  40 

1  ,32S  95      
975  32     
62S  74     ... 

E 
Total 

9,356  24 

256  4S     . 

Clerk's  per  cent  of  average  sales.  As  a  measure  of  efficiency, 
the  following  tabulation  may  bo  made  for  a  department,  and  each 
['lerk's  weekly  or  monthly  sales  compared  with  the  average  weekly 
or  monthly  sales. 

MONTHLY  SALES  OF  CLERKS—  DEPT.  A 

Clerk's          Monthly  l*er  Cent  of 

Number           Sales  Average 

I             $2,756  SO  ................ 

1,954  36  .............. 

2,075  S3  ............. 

2,634  S7  ......... 

2,315  62 


2 
3 

4 
5 

Total 
Average 


100  00% 


Problem 


Prepare  a  form  similar  to  the  above,  enter  the  data,  and  find:  (a)  the  total 
monthly  sales;  (b)  the  average  monthly  sales  per  clerk;  and  (c)  what  per  rent 
*ach  clerk's  sales  are  of  the  average  sales  per  clerk. 

Per  cent  of  income  by  source.  In  accounting  for  the  income 
3f  a  public  service  enterprise,  it  is  desirable  to  show  the  per  cent  of 
income  from  each  source  when  the  company's  activities  are  varied. 

Problems 

^  1.  In  the  following  tabulation  of  gross  earnings  of  a  public  utility  corporation, 
ind  what  per  cent  the  earnings  from  each  source  are  of  the  total  gross  earnings. 


58  PERCENTAGE 

Source  Gross  Earnings  Per  Cent 

Electric  light  and  power $15,817,324  00    

Electric  and  stearn  railroads        .  0,763,656  00 

City  railways  and  bus  lines                  .  4,248,824  00     

Gas 3,191,720  00        

Heat .  672,394  00    

Bridges 589,691  00     

Ice -  2,54,670  00 

Water 88,303  00 

Miscellaneous 21,816  JX)  ....__.._. 

S3 1 ,64'S,39SJ)0  JOQ  QQ  < ;, 

2.  In  the  following  tabulation  of  the  revenue  from  transportation  of  an  inter- 
urban  railway,  find  what  per  cent  each  iten:  of  revenue  is  of  the  total  revenue. 

RKVKNUU  FROM  TRANSPORTATION 

Source  Amount  1'er  Cent 

Passengers $657,855  00  . 

Baggage        550  00  ..... 

Parlor  and  chair  cars. ..         .                  .    .  9,894.00 

Special  cars .  2500 

Mail 1,500  00  

Express 21,96200  

Milk .  .  1,666  00  

Freight  .  264,214  00 

Miscellaneous.     ...                         .  26900  

^^7,935^00  H)0l)0% 

Per  cent  of  expense.  Items  of  operating  expenses  and  their 
relation  to  total  expenses  are  more  easily  compared  if  expressed  in 
terms  of  percentage. 

Problems 

1.  In  the  following  report  of  an  interurban  railway  company,  find  what  per 
cent  each  group  of  expenses  is  of  total  operating  expenses. 

OPERATING  KXPKNSKS 

Item  Amount  Per  Cent 

Way  and  structures $228,690  00  

Equipment 9S,979  00  

Power 105,890  00  

Conducting  transportation                 .      .    .  249,427  00  

Traffic 52,82300  

General  and  miscellaneous               141,560  00  

Transportation  for  investment  (credit) .    .  8,40300  

100  00% 

2.  In  the  following  statement  of  the  operating  expenses  of  a  restaurant  for 
a  period  of  one  month,  find  what  per  cent  each  item  of  expense  is  of  total  oper- 
ating expeo^es. 


PERCENTAGE  59 

OPERATING  EXPENSES 

Item  Amount      Per  Cent 

Superintendent's  labor     $      75  (X) 

General  labor 1,776  00 

Extra  labor      .  .                                    160  00 

Supplies     .           .                      200  00 

Electricity     .  58  00 

Fuel  ...                                75  00 

Laundry.               .                                        103  00 

Ice     .                                                               . .  22  00 

Repairs  and  renewals — equipment             ...  110  00 

Meals  to  employees       .  .  340  00 

Music                 \        .  75  00 

Miscellaneous         .         66  00 

Total $;V)60  00     100  00% 

Percent  of  increase  or  decrease.  Percentage  is  often  employed 
to  find  the  relation  between  numbers;  that  is,  to  find  how  much 
larger  or  smaller  one  number  is  than  another. 

Problems 

1.  In  the  following  departmental  sales  tabulation,  find:  (a)  the  increase  or 
the  decrease  in  monthly  sales  by  departments;  (h)  each  department's  per  cent  of 
increase  or  decrease  (divide  increase  or  decrease  in  each  department  by  that 
department's  monthly  sales  for  This  Month  Last  Year). 

Per  Cent      Per  Cent 
Increase       Decrease       Increase       Decrease 


This  Month 

This  Month 

Dept. 

This  Year 

Last  Year 

A 

$2,973  69 

$2,795.  84 

B 

1,426  S3 

1,S52.1S 

C 

3,752  89 

3,565  62 

D 

2,5S1   28 

2,67S  15 

E 

2,076  S2 

1,825  38 

Total 

2.  In  the  following  condensed  balance  sheet  of  a  municipal  railway,  find  the 
increase  or  decrease  for  each  item,  and  also  the  per  cent  of  increase  or  decrease. 


.4  sscts 
Capital  Assets        
Current  Assets 
Deferred  Assets 

This  Year 
.  $  7,912,526 
2,174,925 
132,124 

Laxt  Year 
37,610,139 
2,241,395 
132,125 

Increas(  ,        Per  Cent 
Decrease]     Inc.,  De,c.\ 

Total  Assets 

$10,219,575 

$9,983,659 

Liabilities,  Reserves,  and 
Surplus 
Funded  Debt 

.   $  3,992000 

$4,192000 

Current  Liabilities            .... 
Reserves   ... 
Surplus 

269,720 
1,568,469 
4,389,386 

343,126 
1,615,743 
3,832,790 

Total  Liabilities,  etc  

.   $10,219,575 

39,983,659 

60 


PERCENTAGE 


3.  In  the  following  tabulation  of  advertising  expenditures  and  direct  sales 
resulting  therefrom,  compute  the  totals,  the  increase,  and  the  per  cent  of  increase. 


Jan 

Feb.   ... 
Mar..    . 
Apr. . .  . 
May.  .  . . 
June  .    . 
July 
Aug      . 
Sept 

Get...    . 
Nov    . 
Dec 


This 

Advertising 

$2,238  00 

2,154.00 

2,435  86 

2,425  46 

2,293.12 

2,035  76 

none 

none 

none 

2,212  56 

7X5  24 

none 


Year 
Sales 

$4,251.44 
7,461  60 
8,773  84 
7,292.12 
8,709.04 
8,412  28 
7,3X3.46 
7,656  80 
8,227.84 
4,298  70 
5,260  X4 
5,6X3  96 


Last 
Advertising 


$ 


1,769  64 
1,787.96 
1,769  53 
1,840.26 
1,831  70 
1,825.49 
none 
none 
none 
1,142  04 
1,306  26 
none 


Year 
Sales 

$  3,762.00 
5,067  16 
5,232  48 
7,818  00 
4,867  20 
4,673  12 
5,0X3  20 
4,454  56 
4,650  88 
4,976  40 
2,6X2  00 
3,542  XO 


Total 

Year  ago $10,607.52     $37,650.77 

Increase 

%  Increase..     .    .     . ...  .     

4.  In  the  following  statement,  find  the  increase  or  decrease  of  revenues  and 
expenses  and  the  per  cent  of  increase  or  decrease: 

Increase, 

This  Year  Last  Year  Decrease^     Per  Cent 

Railway  operating  revenue $X66,197      $970,060  

Other  operating  revenue X/21X  7,X2()  ...    .... 

Total  operating  revenue     $X74,415     SOTTJSSO  ....     "          ..T 

Railway  operating  expense: 

Way  and  structures $  91,3X0      $  85,569  

Equipment 64,249          61 ,866 .....  

Power 108,313        114,906 .... 

Conducting  transportation 196,259        21 1,144 

Traffic 9,496          10,157 

General  and  miscellaneous     ....       12S,S49        12X,XX7 . 

Depreciation 17,324         44,645  

Taxes  (except  income  taxes)       .         26,1X5          29,840 ...     . 

Total $642,055      «GS7^014  .7.71     ~"  _  j 

Operating  income $232,360     $29Q~,X66  ^_^_^    ~" 

Non-operating  income: 

Interest  funded  securities  2,579  5,105 

Interest  unfunded  securities. ...          7,765  6,328 

Total 1^0»344     $  1],433  ~  ............... 

Gross  income $242,704     $302,299  ~ 

Deductions  from  gross  income: 

Interest $160,318     $161,402 

Miscellaneous 3,216  3,257  ................     

Total $163,534      $164^659  ^~TZ^     .~ 

Net  income $  79,170     $137,640  


PERCENTAGE 


61 


Operating  statistics.  The  operations  of  a  public  utility  engaged 
in  transportation  afford  an  excellent  opportunity  for  the  presenta- 
tion of  statistics  for  managerial  control.  The  following  problem 
has  been  derived  from  the  report  of  such  an  enterprise. 


Problem 

From  the  following  data,  ascertain  the  required  answers. 


SECTION  OF  INCOME 

Income 
Operating  revenue: 

Railway  operating  revenue 

Coach  operating  revenue 

Total  operating  revenue.    .  . . 
Non-operating  income 

Total  revenue  from  all  sources 
Operating  expenses: 

Railway  operating  expenses   . 
Coach  operating  expenses   . 

Total  operating  expenses. . 

Net  revenue  from  all  sources 


STATEMENT 

Thin  Year 

.  .   $  22,413,089 
818,328 


184,273 


Last  Year 

$  21,678,900 

5jJL282 

$  21, 730,188 
141,707 


$  23,410,290     $  21,871,955 


.   $  10,572,497 

____  780,558 

$J7T359,055 
$     0,057,235 


$   15,383,494 

41,701 

»JM_25J95 
$  73,440,700 


Railway  revenue  car-miles .52,803, 1 1 1         48,248,330 

Coach  revenue  coach-miles. .      ..  3,529,795  157,540 

Railway  revenue  car-hours 5,092, 1 90  5,207, 1 70 

Railway  revenue  passengers 357,920,108       340,1 10,298 

Railway  transfer  passengers .  .1 23,3 1 0,520       1 1 1 ,445,9 1 2 

Railway  total  passengers ...     481,230,094       457,502,210 

Coach  revenue  passengers  .  .  10,504,723  978,782 

Coach  transfer  passengers .      .  387,228     

Coach  total  passengers      10,951 ,951  978,782 

Total  revenue  and  transfer  passengers 492,188,045      458,540,992 

Railway  operating  revenue  per  car-mile  (cents)  ... 

Coach  operating  revenue  per  coach-mile  (cents) .  .      ,      .  .  ..  . 

Railway  operating  expenses  per  car-mile  (cents) .        .  .. 

Coach  operating  expenses  per  coach-mile  (cents) ..     ..........         ..     .. 

Railway  operating  revenue  per  car-hour   ($  and 

cents) - 

Railway  operating  expenses  per  car-hour   ($  and 

cents) 

Ratio  of  transfer  passengers  to  revenue  passengers 

— railway  (per  cent) 

Ratio  of  transfer  passengers  to  revenue  passengers 

— coach  (per  cent) 

Railway  revenue  passengers  per  car-mile  operated . 

Railway  transfer  passengers  per  car-mile  operated 

Total  railway  passengers  per  car-mile  operated 

Coach  revenue  passengers  per  coach-mile  operated 

Coach  transfer  passengers  per  coach-mile  operated 


62  PERCENTAGE 

Statistics  (Continued)  This  Year         Last  Year 

Total  coach  passengers  per  coach-mile  operated 

Ratio  of  railway  operating  expenses  to  railway  oper- 
ating revenue  (per  cent) 

Ratio  of  coach  operating  expenses  to  coach  operating 
revenue  (per  cent) 

Budgeting.     Percentage  is  also  applied  in  budgeting,  as  shown 
by  the  following  example  from  hotel  accounting. 

Example 

Among  the  several  items  of  the  budget  is,  China  and  Glassware,  $3,500,  to 
be  distributed  to  four  departments  on  the  basis  of  the  previous  year's  expense 
for  this  item  in  the  four  departments,  as  follows: 

Department  Per  Cent 

Rooms 1 1  29 

Restaurant 5529 

Coffee  Shop 14  86 

Beverages .  .        .      1 8  56 

Total 106~00% 

Solution 

Department  Per  Cent  Budget 

Rooms 1 1  29  $    395  00 

Restaurant 55  29  1,935.00 

Coffee  Shop 14  86  520  00 

Beverages J^L^L.  650  00 

Total WJO%  S^XTOO 

Problems 

1.  The  following  year  it  was  found  that  the  actual  disbursements  for  China 
and  Glassware  amounted  to  $2,280.74,  and  other  farts  were  as  given  in  the 
tabulation  below.  Compute  the  per  cent  for  the  distribution  of  the  budgeted 
amount  for  the  next  year,  and  the  per  cent  that  the  expense  of  China  and  Glass- 
ware is  of  the  income  for  each  department. 

China  and     Per  Cent      Per  Cent 
Gross  Glassware  of  of 

Department  Income          Expense       Expense       Income 

Rooms $141,857  50    $   269  53    .  . 

Restaurant 59,626.90      1,252  16    

Coffee  Shop 33,587.45         335  87     

Beverages 9,061 .65         423  18    


$244,133  50     $2,280.74     100  00% 
2.  The  following  budget  is  that  of  an  estimated  operating  statement. 

Per  Cent  of 
Net  sales:  Total  Sales 

Class  A $2,000,000      

Class  B 200,000      

Class  C 250,000      

Class  D 50,000 

$2,500,000      UK)  00% 


PERCENTAGE  63 

Per  Cent  of 

Sales 
Production  costs: 

Class  A $1,200,000       

Class  B  140,000      

Class  C 162,500      

Class  D _40,000      ii^iz^; 

$1,542,500      

Ptr  Cent 

Gross  margin $    957,500 

Selling: 

Sales  administration $      50,000  

General  sales  department  expense. .    .  .  12,500  

Special  promotion,  etc           12,500  

District  operating  expense             400,000  

Advertising  A 100,000  

Advertising]* 12,500  

Advertising  C 25,000  

Selling  cost        $    612,500  ....  ___.. 

Net  margin $_ 345^00  ~       

Calculate:  (a)  the  per  cent  of  net  sales  in  each  class,  as  compared  with  total  net 
sales;  (b)  the  per  cent  of  pioduction  cost  in  each  class,  based  on  sales  of  each 
class;  (c)  the  per  cent  that  selling  cost  is  of  total  net  sales;  (d)  the  per  cent  that 
the  net  margin  is  of  total  net  sales. 

3.  The  following  is  the  budget  for  the  Water  Department  of  a  municipality. 
Find  the  per  cent  that  each  budget  expenditure  is  of  the  total  for  the  department. 

Amount  Per  Cent 

Pump  station  and  filter  plant  salaries  ....  $17,300.00  

Office  salaries  and  expenses     4,600.00  

Chemicals,  filter  plant 1,000  00  

Power — pump  station  and  filter  plant 15,000  00 

Light,  heat,  and  supplies 3,000  00  

Water  service 3,000  00  

Meters  and  installation 6,000  00  

Water  main  extensions  and  fire  hydrants   .  3,000  00  . 

Motor  truck  repairs 150  00  

Interest  on  outstanding  warrants 4,246  00  

Total $^296JO  MjjjO% 

4.  Compute  the  increase  or  decrease  and  the  per  cent  of  increase  or  decrease 
in  the  following  comparative  budget. 

Public  Buildings  This  Last                                      %           % 

and  Utilities  Year  Year        Inc.       Dec.       Inc.       Dec. 
City  hall  engineers  and  jani- 

,     tors $  5,060  $  4,284  

City  hall  fuel  and  supplies ...  4,000  2,216  

City  hall  maintenance  and  re- 
pairs   1,000  850  


64 


PERCENTAGE 


Public  Buildings               This  Last 

and  Utilities                  Year  Year 

Detention  hospital  repairs.  .  .  $     300  $     400 
Detention  hospital  light  and 

fuel 900  700 

Park  light  and  fuel 1 ,200  1 ,050 

Septic  tank  electric  power. ...          600  600 

Septic  tank  repairs 100  100 

Incinerator  fuel  and  light.  .  . .       1,000  1,000 
Electric  lighting — streets, 

alleys 14,500  14,000 

Interest  on  warrants 2,2H4  2,170 

Contingent  fund  5,000.  4,036 

Detention  hospital  insurance 433 

Library  insurance ... .  281 

Park  insurance 104 

$35,1)44  $32~224 


Inc.       Dec. 


% 
Inc. 


% 
Dec. 


Profits  based  on  sales.  In  the  income  statement,  it  is  custo- 
mary to  base  all  percentage  calculations  on  sales.  With  sales 
equalling  100%,  cost  of  sales,  overhead,  and  net  profit  are  expressed 
as  per  cents  of  sales.  Overhead  expenses  are  those  incurred  in 
operating  a  business — such  as  salaries  and  wages,  rent,  heat,  light  and 
power,  depreciation,  taxes,  insurance,  advertising,  telephone,  post- 
age, and  so  forth.  In  marking  goods  bought  for  resale,  these 
expenses  must  be  taken  into  consideration.  A  few  items  of  over- 
head expense  do  not  fluctuate,  but  many  of  them  have  a  fairly 
constant  ratio  to  gross  sales.  The  merchant  determines  the  ratio 
of  overhead  expenses  to  sales  from  his  own  experience  and  that  of 
others  engaged  in  similar  businesses.  This  per  cent  of  cost  of  doing 
business  plus  the  per  cent  of  profit  decided  upon  deducted  from 
100%  determines  the  per  cent  which  the  cost  of  goods  plus  freight 
and  drayage  bears  to  the  selling  price. 


Sales 

=  100% 

Invoice  Price  plus  Freight  and 

75% 

Cartage 

Overhead        Profit 
15%              10% 

Cost  of  Sales 
75% 

Gross  Profit  on  Sales 
25% 

Example 

If  overhead  charges  amount  to  15%  of  sales,  and  a  profit  of  10%  on  sales  is 
desired,  what  is  the  selling  price  of  an  article  with  an  invoice  cost  of  $21.00  and 
freight  and  cartage  of  $1.50? 


PERCENTAGE  65 

Solution 

15%  +  10%  =  25% 
100%  -  25%  -  75% 
$21.00  +  $1.50  =  $22.50,  the  cost. 
$22.50  -T-  75%  =  $30.00,  the  selling  price. 

Verification 

25%  of  $30.00  =  $7.50,  the  overhead  and  profit. 
$30.00  -  $7.50  =  $22.50,  the  cost. 

Problems 

1.  An  article  that  cost  $15.00  was  sold  for  $20.00.     What  is  the  profit  per 
cent  on  the  selling  price? 

2.  With  an  overhead  expense  of  20%,  what  per  cent  of  profit  on  sales  is 
made  by  selling  for  $1.50  articles  that  have  an  invoice  cost  of  $1.00? 

3.  What  is  the  per  cent  of  gross  profit  on  sales  in  Problem  2? 

4.  How  much  must  the  article  in  Problem  2  be  reduced  to  sell  at  cost? 
What  per  cent  is  this  of  the  marked  price? 

5.  A  merchant  sold  an  article  for  $12.00  and  made  a  profit  of  12^%  on  the 
selling  price.     What  was  his  profit  in  dollars? 

6.  Find  the  per  cent  of  reduction  of  marked  price  to  produce  cost. 

Cost       Marked  Price    Per  Cent  Reduction 

a.  $      20          $       25 

b.  2  50  2  75  

c.  1  00  1  20  

d.  03  .05  

e.  3  00  6  00  

/.  .40  .50  

g.  09  .12  

h.  15.00  25  00  

7.  Find  the  per  cent  of  profit  on  the  selling  price. 

Per  Cent  Profit  on 
Cost        Selling  Price          Selling  Price 

a.     $     1.00        $     1  20  

6.         10.00  15.00  

c.  .60  .75 

d.  3.50  7.00 

«.  6.00  8  00  

/.  150  00  175  00  

g.  16.00  24  00 

h.  75  00  125  00  .... 

8.  The  factory  price  of  an  automobile  is  $1,300.     Freight  charges  from 
factory  to  dealer  are  $65.00.    If  the  dealer's  overhead  is  20%  and  he  expects  a 
net  profit  of  15%  on  sales,  what  should  be  the  selling  price  of  the  automobile? 

9.  A  furniture  dealer  bought  a  shipment  of  20  chairs  at  $30.00  each.    He 
marked  them  to  sell  at  a  profit  of  40%  on  cost.    The  entire  shipment  was  sold 


66  PERCENTAGE 

in  the  fall  clearance  sale  at  25%  reduction  from  marked  price.     What  was  the 
profit  or  loss? 

10.  Complete  the  following: 

%  on 
Cost          Selling  Price      %  on  Cost      Selling  Price 

a.  $    4  00          $    6  00          

b.  15.00  25  00          

c.  .16  .20  

d.  .04  .08 

e.  .OS  .10 

/.  5  00  7  00  .. 

g.  500  00  750  00 

h.  24.00  32  00  

11.  The  invoice  price  of  an  article  is  $12.00.     Freight  is  75  cents.     It  costs 
18%  to  do  business  and  you  desire  a  net  profit  of  10%  on  sales.     What  is  the 
selling  price  of  the  article? 

12.  If  the  invoice  cost  is  $28.00,  freight  $2.00,  overhead  25%,  net  profit  on 
sales  15%,  what  is  the  selling  price? 

13.  A  stock  of  merchandise  valued  at  $8,750.00  was  damaged  by  fire  and 
water.     The  loss  was  estimated  to  be  25%.     Find  the  value  of  the  damaged 
merchandise. 

14.  A  merchant's  overhead,  or  cost  of  doing  business,  is  22-f  %.     He  desires 
to  make  a  net  profit  of  7ir%.     What  will  be  the  selling  price  of  an  item  that  cost 
this  merchant  $4.90? 

16  Merchandise  is  bought  for  $3.50  less  25%  and  sold  at  $3.50  net.  What 
is  the  rate  per  cent  of  profit? 

16.  A  tea  and  coffee  merchant  blends  a  40  j£  tea  with  a  70 £  tea  in  the  ratio 
of  2  to  1.     If  the  blend  is  sold  at  65^  a  pound,  what  is  the  rate  per  cent  of  profit 
on  cost? 

17.  A  chair  manufacturer  finds  the  cost  of  material  in  a  certain  type  chair 
to  be  $7.50.     Manufacturing  cost  (labor  and  overhead)  is  $14.80.     Selling  and 
administrative  expenses  are  20%  of  sales.     What  is  the  manufacturer's  price 
for  this  chair  if  he  desires  to  net  10%  on  the  selling  price? 

18.  A  clerk  was  ordered  to  mark  a  lot  of  suits  so  as  to  make  a  profit  of  20% 
after  allowing  5%  discount  for  cash.     By  mistake  he  marked  the  suits  $24.75 
each,  which  resulted  in  a  loss  to  the  clothier  of  8%.     At  what  price  should  the 
suits  have  been  marked? 

Marking  goods.  Merchants  frequently  indicate  the  cost  price 
and  the  selling  price  on  each  article.  The  buyer  may  use  the  cost 
price  marking  for  comparison  with  current  quotations ;  slow  sellers 
may  be  checked  for  desirability  of  reducing  the  selling  price ;  inven- 
tory of  stock  may  be  taken  at  cost;  and,  under  special  systems  of 
accounting,  a  daily  record  of  cost  of  sales  is  achieved. 

In  order  to  conceal  the  cost  price  from  the  customer,  a  set  of 
symbols  is  used,  interpretation  of  which  depends  upon  a  knowledge 
of  the  key  to  the  letters  or  characters.  Any  word  or  phrase  of  ten 


PERCENTAGE  67 

letters  or  any  ten  arbitrary  characters  may  be  used  as  the  key. 
An  extra  letter  or  character  is  used  to  prevent  repetition  of  a  letter, 
and  this  extra  letter  or  character  is  called  a  repeater.  The  repeater 
makes  it  more  difficult  for  a  stranger  to  decipher  the  marks.  The 
word  or  phrase  used  must  not  contain  the  same  letter  twice. 
Otherwise  the  same  letter  will  represent  two  different  numbers. 

If  the  cost  and  the  selling  price  are  both  written  on  the  same 
tag,  the  cost  price  is  usually  written  below,  and  the  selling  price 
above,  a  horizontal  line.  If  both  cost  and  selling  price  are  marked, 
a  separate  key  is  used  for  each. 

Example 

Use  the  word  "blacksmith"  with  repeater  "w"  as  the  selling  key,  and  the 
phrase  "pay  us  often"  with  repeater  "x"  as  the  cost  key,  and  mark  an  article 
to  sell  at  $6.50  with  a  cost  of  $4.25. 

Solution 

b  1  a  c  k  s  m  i  t  h         Repeater 
1234567890  w 

pay  u  soften         Repeater 
1234567890  x 

S.kh 
U.as 

Example 

With  the  same  keys,  mark  an  article  to  sell  at  $9.55,  with  a  cost  of  $7.00. 

Solution 
T.kw 
F.nx 

The  following  are  examples  of  key  words  and  phrases: 

Blacksmith  Buy  for  cash 

Charleston  Black  horse 

Buckingham  Cash  profit 

Republican  Pay  us  often 

Authorizes  Our  last  key 

Problems 

1.  Using  "Charleston"  as  the  key  word  and  "x"  as  the  repeater,  indicate 
the  following  costs: 

a.  $5.56  d.  86.62  g.  $6.20  j.  $1.44  m.  $15.00 

b.  $6.50  e.  $7.50  h.  $5.00  k.  $26.50  n.  $2.35 

c.  $2.45  /.  $12.50          t.  $.25  I.  $12.47  o.  $1.60 

2.  Use  as  the  cost  key  "pay  us  often"  and  repeater  "w,"  as  the  selling  key 
"authorizes"  and  repeater  "x,"  and  show  markings  for  the  following: 


68  PERCENTAGE 


Coat 

Selling  Price 

a. 

'  $  2.25 

$  3.50 

b. 

1.15 

1.50 

c. 

1.25 

1  65 

d. 

23.50 

29.50 

e. 

.65 

.90 

3.  If  the  selling  key  is  "Bridgepost"  with  repeater  "w,"  and  the  cost  key 

is  "  Cumberland"  with  repeater  "x,"  write  in  figures  the  prices  given  in  the 
following: 

B.dt  Bg.ow 

a'  Oud  e'  Cu.dx 

R.pg  It.ww 

U.xd  J'  Ux.dx 
Be.ot 


^ 

'  Cb.dx  g'  libd 

I.tw  R.dt 

a'  U.ed  *'  C.rd 

Commissions.  The  commission  business  in  this  country  is 
largely  the  result  of  our  industrial  and  commercial  development. 
Economic  conditions  demand  that  there  shall  he  agents  who  shall 
represent  either  the  buyer  or  the  seller.  The  compensation  paid 
the  agent  for  his  services  is  called  a  commission.  The  principles 
of  percentage  apply  in  commission. 

The  person  who  transacts  business  for  another  is  the  agent,  and 
the  one  for  whom  the  business  is  transacted  is  the  principal.     The 
fee,  usually  a  per  cent  of  the  dollar  volume  of  the  transaction,  is 
the  commission. 
\     .  ,  V^     ^  Problems 

1.  An  agent  sells  oil  for  $3,475.00  at  3i%  commission.     What  is  the  amount 
of  the  commission? 

2.  A  merchant  buys  goods  through  an  agent  at  a  cost  of  $275.00.     The 
agent  charges  2^%  commission.     What  is  the  total  cost  of  the  goods  to  the 
merchant? 

3.  An  agent  sells  a  consignment  of  merchandise  for  $1,824,  retaining  his 
commission  of  3%.     How  much  does  he  remit  to  his  principal? 

4.  If  $302.75  was  charged  for  selling  $8,650.00  of  merchandise,  what  was 
the  rate  of  commission? 

5.  A  realtor's  fee  for  selling  a  house  and  lot  WHS  $150.00.     If  the  rate  was 
2%,  what  was  the  amount  received  by  the  principal? 

6.  An  agent's  commissions  for  one  week  were  $216.80.     If  his  sales  were 
$10,840.00,  what  rate  did  he  charge? 

7.  The  invoice  price  on  a  shipment  of  merchandise  was  $1,283.38,  i 
agent's  commission.     If  the  agent's  rate  was  3%,  what  was  the  commission? 

8.  The  proceeds  of  a  sale  received  by  the  principal  were  $828.78.     The 
commission  deducted  by  the  agent  was  $43.62.    What  was  the  rate? 

9.  Find  the  net  proceeds  of  the  following: 


PERCENTAGE  69 

ACCOUNT  SALES 

Boston,  Mass.,  Oct.  5t  19 — 
Sold  for  Account  of 

Friends  Milling  Co.,  Friendsville,  Minn. 
By  Puritan  Brokerage  Company 


19- 

Aug. 

4 

350  bbls.  Flour  (o\  $4.51                       

24 

175bbls.  Flour  @    4.4S                    

Sept. 

r> 

320  bbls.  Flour  (8l    4.50                     

19 

60  bbls.  Flour  @    4.53                    

Oct. 

1 

30  bbls.  Flour  @    4.52 

Total  Sales                                                 

Charges 

Aug. 
Oct. 

1 
4 
1 
5 

Freight                                            $420.60 
Cartage                                                 26  50 
Storage                                                  22  90 
Commission  (fll  3% 

Total  Charges 

Net  Proceeds                                              

10.  The  manufacturing  cost  of  a  certain  type  machine  is  $$0f.W.  The 
manufacturer  wishes  to  catalog  this  machine  at  a  list  price  that  will  net  a  profit 
of  25%  on  sales  after  allowing  a  dealer's  discount  of  25%  and  agent's  commission 
of  16-f  %.  Find  the  catalog  list  price. 


CHAPTER^ 
Commercial  Discounts 

Cash  discount.  Cash  or  time  discount  is  a  deduction  for 
immediate  payment,  or  for  payment  within  a  definite  time.  The 
deduction  is  a  certain  per  cent  of  the  invoice. 

The  expression  "Terms:  2/10,  1/30,  n/60"  means  that  2%  of 
the  invoice  price  may  be  deducted  by  the  purchaser  if  payment  is 
made  within  10  days  of  the  date  of  the  invoice,  that  1  %  may  be 
deducted  if  the  invoice  is  paid  within  30  days  from  the  date  of  the 
invoice,  and  that  the  invoice  is  due  in  60  days  without  discount. 
In  some  cases  notice  is  given  to  the  effect  that  interest  at  a  specified 
rate  will  be  charged  after  the  due  date. 

The  acceptance  of  a  cash  discount  is  usually  of  ndv:ml:igo  to 
the  purchaser.  The  following  table  indicates  the  annual  interest 
rates  to  which  the  usual  cash  discounts  are  equivalent: 

i%  10  days,  net  30  days     =  9  per  cent  a  year 

1%  10  days,  net  30  days     =18  per  rent  a  year 

li%  10  days,  net  30  days     =  27  per  cent  a  year 

2%  10  days,  net  30  days     =  30  per  cent  a  year 

2%  10  days,  net  60  days     —  14.4  per  cent  a  year 

2%  30  days,  net  4  months  =  8  per  cent  a  year 

The  rate  per  cent  a  year  is  calculated  by  taking  the  number  of 
days  between  the  discount  date  of  payment  and  the  end  of  the 
credit  period,  dividing  the  number  of  days  in  a  year  (360)  by  this 
number^  and  multiplying  the  quotient  by  the  rate  of  discount 
under  consideration. 

X  Rate  of  Discount  =  Equivalent  Annual  Interest  Rate 


Number  of  Days  Between 

Discount  Date  and 

End  of  Credit  Period 

Problems 
1.  Find  the  equivalent  annual  interest  rate  for  the  following  terms: 

2%  30  days,  net  60  days 

3%  10  days,  net  30  days 

3%  30  days,  net  60  days 

3%  10  days,  net  4  months 


72  COMMERCIAL  DISCOUNTS 

2.  To  pay  an  invoice  of  $1,500,  with  terms  2/10,  n/30,  the  purchaser  bor- 
rowed the  money  at  6%  in  order  to  take  advantage  of  the  2%  discount.     What 
benefit  did  he  secure  by  borrowing  the  money? 

3.  A  merchant  was  able  to  obtain  5%  discount  on  an  invoice  of  $720  by 
borrowing  the  money  at  the  bank  for  90  days  at  6%  interest.     Plow  much  was 
lie  able  to  save? 

Trade  discount.  Mercantile  or  trade  discounts  are  reductions 
from  list  prices,  or  from  the  amount  of  the  invoice  without  regard 
to  time  of  payment.  By  offering  different  rates  of  trade  discounts 
to  wholesalers  and  retailers,  the  manufacturer  can  send  the  same 
catalog  to  both  classes  of  customers.  Revised  discount  sheets  are 
issued  as  prices  fluctuate,  hut  the  same  catalog  may  be  used  a  year 
or  more  because  the  list  prices  are  fixed. 

Rules  of  percentage  are  applied  in  commercial  discounts: 

Invoice  price  =  base 

Per  cent  of  discount  =  rate 
Discount  =  percentage 

Several  discounts  are  sometimes  given.  These  are  known  as 
chain  discounts  or  a  series  of  discounts. 

The  order  in  which  the  discounts  are  deducted  will  not  affect 
the  result;  thus,  a  selling  price  stated  as  list  price  "less  10%,  20%, 
and  5%"  is  the  same  as  a  selling  price  stated  as  list  price  "less  5%, 
20%,  and  10%."  This  is  shown  in  the  following  example,  in 
which  $100.00  is  used  as  the  base: 

Example 

$100.00  X  .10      ..  .        .       $1000    $100.00  X  .05 $500 

$100.00  -  $10.00  .       $90  00    $100.00  -  $5.00 $95  00 

$  90.00  X  .20  .   SIS  00     $  95.00  X  .20       $19  00 

$  90.00  -  $1S.OO  $72  00     $  95.00  -  $19.00 $76.00 

$  72.00  X  .05      .  ..             .$3.60    $  76.00  X  .10     $7.60 

$  72.00  -  $3.60  . .  .    .       $68  40    $  76.00  -  $7.60 $68  40 

$100.00  -  $68.40 $31.60    $100.00  -  $68.40 $31.60 

The  total  discount  in  each  case  is  $31.60. 

The  dollar  amount  of  discount  determined  from  a  series  of 
rates  is  not  the  same  as  the  amount  of  discount  determined  from  a 
single  rate  equal  to  the  sum  of  the  series  of  rates.  The  sum  of  the 
series  of  rates  is  35%]  35%  of  $100.00  is  $35.00,  whereas  the  correct 
discount  is  $31.60. 

Single  discount  equivalent  to  a  series. 

First  method.  To  find  the  single  discount  that  is  equivalent  to 
a  series  of  <li>(ioimfs71TuE^^  from  100%. 

Use  the  remainder  as  a  new  base.  Multiply  it  by  the  second  dis- 
count, and  deduct  the  product.  Use  this  remainder  as  & jiew  base. 


COMMERCIAL  DISCOUNTS  73 

Compute  each  discount  successively,  ^proceeding  as  before.  The 
difference  between  100%  and  the,  last  result  will  Lii  Uic-.iiijLi^ 
discount. 

Example 

What  single  discount  is  equivalent  to  a  series  of  discounts  of  20%,  10%, 
and  8i%? 

Solution 

100% -20% 80% 

80%  X  10% .  ....     8% 

80%  -  8%..       .  ...  .    .   72% 

72%  X8i%..    .  ...     6% 

72%  -  6%..   .  .  60% 

100%  -  66%  ..  .  .  ...   34% 

Explanation.  100  %  represents  the  invoice  price.  20%,  or  I  of  100%,  equals 
20%,  which  subtracted  from  100%,  leaves  80%;  10%,  or  TV  of  80%,  equals  8%, 
which  subtracted  from  80%  leaves  72%;  S-J-%,  or  rV  of  72%,  equals  6%,  which 
subtracted  from  72%  leaves  66  %.  100%,  the  invoice  price,  less  66%,  the 
selling  price,  leaves  34%,  the  single  discount. 

Second  method.  To  find  the  single  discount  that  is  equivalent 
to  a  series  of  discounts,  subtract  each  single  discount  from  100% 
and  find  the  product  of  the  remainders.  Subtract  the  final  prod- 
uct from  100%,  and  the  remainder  is  the  single  discount  equiva- 
lent to  the  series  of  discounts. 

Example 
What  single  discount  is  equivalent  to  the  series  20%,  10%,  and  5%? 

Solution 

100%  100%  100% 

20%  v  J0%  5% 

80%  '  90%  "95% 

.80  X  .90  X  .95  =  .684,  or  OS.4% 

100%  —  68.4%  =  31.6%,  the  single  discount 

A  short  method.  To  find  the  single  discount  that  is  equivalent 
to  any  two  discounts,  subtract  the  product  of  the  discounts  from 
the  sum  of  the  discounts. 

Example 

What  single  discount  is  equivalent  to  discounts  of  20%  and  20%? 

Solution 

20%  +  20%  =  40% 

20%  X  20%  =    4% 
40%  -    4%  =  36% 

To  find  the  net  price.  To  find  the  net  price,  the  list  price  and 
discounts  being  given:  Reduce  the  discount  series  to  a  single  dis- 


74  COMMERCIAL  DISCOUNTS 

count^multiply  the  invoice  price  by  this  single  discount,  and  deduct 
the  result  from  the  invgicejjrice. 

Example 

What  is  the  net  price  of  an  invoice  of  $600.00,  less  30%,  20%,  and  10%? 

Solution 

100%  100%  100% 

30%  20%  10% 

70%  80%  90% 

.70  X  .SO  X  .90  504,  or  50.4% 

100%  -  50.4%  49.6%,  the  rate  of  discount 

$600.00X49.0%                   ..    .          .   $297.60,  the  discount 
$600.00  -  $297.60 $302.40,  the  net  price 

If  the  amount  of  discount  is  not  desired,  the  net  price  may  be 
found  as  follows: 

$600.00  X  50.4%  =  $302.40 

Problems 

1.  In  each  of  the  following,  calculate  by  the  short  method  the  single  dis- 
count that  is  equivalent  to  the  scries: 

(a)  10%  and  5%.  (c)  40%  and  5%.  (c)  35%  and  10%. 

(b)  20%  and  5%.  (d)   15%  and  10%.  (/)  30%  and  20%. 

2.  In  each  of  the  following,  find  the  net  price: 

(a)  $350.00,  less  10%,  10%,  and  5%.       (c)  $480.00,  less  20%,  10%,  and  5%. 
(6)  $500.00,  less  33£%,  5%,  and  2|%.     (d)  $1 ,200.00,  less  5%,  2i%,  and  1  %. 
(c)  $900.00,  less  50%,  20%,  and  5%. 

3.  The  list  price  of  an  invoice  is  $750.00,  with  discounts  of  10%,  5%,  and 
2t%.     The  terms  of  the  invoice  are:  2/10,  1/30,  and  n/60.     What  amount  will 
he  necessary  to  pay  the  invoice:  (a)  within  the  10-day  period;  (6)  within  the 
30-day  period? 

4.  B  purchases  merchandise  listed  at  $3,500.00,  less  20%  and  25%.     He 
sells  this  merchandise  at  the  same  list  price,  less  15%,  10%,  and  5%.     Does  he 
gain  or  lose,  and  what  amount? 

5.  A  dealer  offers  merchandise  at  a  list  price  of  $5,000.00,  less  discounts  of 
25 %,  10%,  and  10%.     Another  dealer  offers  the  same  merchandise  at  a  list  price 
of  $4,800.00,  less  discounts  of  20%,  15%,  and  5%.    Which  is  the  better  offer, 
and  by  what  amount? 

6.  Which  is  the  better  offer,  and  by  what  amount,  on  an  invoice  of  $425.00: 
(a)  30%,  20%,  and  10%;  or  (b)  a  single  discount  of  50%? 

7.  The  list  price  of  an  item  is  $24.00.     If  bought  at  that  price  less  33^%  and 
10%,  and  then  sold  at  the  same  list  price  less  20%  and  5%,  what  is  the  profit? 

8.  The  net  cost  of  an  invoice  of  merchandise  was  $1,200.00.    What  was 
the  list  price,  if  the  cost  was  25%  and  20%  off  list? 


COMMERCIAL  DISCOUNTS  75 

^-9.  If  the  list  price  is  $400.00,  and  the  net  price  is  $380.00,  \\hat  is  the  single 
rate  of  discount? 

10.  What  single  discount  is  equivalent  to  25  %,  20%,  and  12j%? 


Transportation  charges  c^j^s^^ynvoices.  In  some  cases 
transportation  charges  are  paid  by  the  seller;  in  other  cases,  by  the 
purchaser.  If  the  purchaser  is  to  pay  the  transportation  charge, 
_aiicTas"  a  matter  of  convenience  the'seTIeFpreprfys  Tf  ,  TJie~  seller  adds 
the  jcharge  to  the  invoice.  The  purchaser  is  not  entitled  to  cash 
discount  on  the  added  charge. 

If_a  shipment  is  made  "freight  allowed/'  the  discount  should 
be  figured  after  the  deduction  for  freiglit;  otherwise,  it  would  be 
equivalent  to  taking  discount  on  the  transportation  charge. 

Problems 

1.  An  invoice  of  books  amounts  to  $4.85,  and  parcel  post  charges  are  79  cents, 
a  total  of  $5.64.     If  terms  are  2/10,  what  is  the  discount  if  paid  within  the  10-day 
period? 

2.  Complete  the  following  invoice: 

6  doz.  Items  @  $6.65  .............. 

24  doz.  Articles  @      .45  .............. 

Girdoz.  Items  @      Al\  ............. 

3  only  Items  @    2.34  __....  _ 

Less  15% 

Less  freiglit  allowance  .............. 

525  Ibs.  @  .45|  cwt.  .............. 

Net    "  7ZTZI 


If  the  terms  of  the  above  invoice  are  1/10,  n/30,  what  will  be  the  discount  if 
paid  within  10  days? 

3.  An  invoice  for  paper,  freight  allowed,  amounted  to  $1,754.50.  The  freight 
bill  paid  by  the  purchaser  was  $238.54.  If  2%  discount  was  allowed  for  pay- 
ment within  10  days,  what  was  the  amount  of  the  check? 

Anticipation.  In  retail  business,  invoices  for  purchase^  often 
have  dating  terms,  The  terms  may  be  2/10,  90  days  extra.  IfHie 
merchandise  is  received  within  10  days  and  checked  by  the  receiv- 
ing department,  the  purchaser  will  deduct  2%  cash  discount,  and 
an  anticipation  discount  on  the  balance  computed  at  6%  (usually) 
for  90  days,  which  is  equivalent  to  an  additional  discount  of  l£%. 

Another  case  is  that  of  spring  purchases  of  fall  merchandis* 
invoiced  2/10,  November  1  dating.  An  invoice  with  these  terms 
may  be  discounted  2%  if  paid  before  November  11,  and  if  paid 
July  15  would  be  subject  to  anticipation  discount  for  119  days  at 
the  customary  rate,  say,  6%. 


76  COMMERCIAL  DISCOUNTS 

Example 

An  invoice  billed  April  2,  for  $3,250.75,  terms  2/10,  Nov.  1  dating,  freight 
allowed,  was  paid  April  28.  Freight  paid  by  the  purchaser  was  $132.48. 
What  was  the  amount  of  the  check? 

Solution 

Invoice $3;250.75 

Less  freight 132  48 

$3,118~27 
Less  discount,  2% 62  37 

$3,055  90 

Less  anticipation,  6%  for  197  days 100  34 

Amount  of  check ^M^Oi5 

Problems 

1.  An  invoice  for  $21.25  dated  Dec.  28,  terms  2/10,  Feb.  26,  was  paid  Jan.  7. 
What  was  the  amount  of  the  check? 

2»  What  is  the  anticipation  on  an  invoice  for  $475.50,  dated  June  10,  terms 
2/10^  90  days  extra,  if  paid  June  25? 

3.  Find  the  amount  earru-J  by  paying  an  invoice  for  $1,275.25,  dated  July  12, 
terms  2/10,  Oct.  1  dating,  on  July  2<S. 


CHAPTEI&T^J 
Simple  Interest 

Definition.  Interest,  as  commonly  defined,  is  a  payment  for 
the  use  of  borrowed  money  or  credit.  This  payment  depends  upon 
the  rate  per  cent  charged  and  upon  the  length  of  time  for  which 
interest  is  calculated.  The  sum  loaned  or  the  amount  of  credit 
used  is  the  principal.  The  number  of  hundredths  of  the  principal 
that  is  taken  is  the  rate,  which  is  usually  expressed  as  a  per  cent. 
The  principal,  with  the  interest  added,  is  called  the  amount. 

Short  method  of  calculating.  There  are  a  great  many  methods 
of  computing  interest,  each  of  them  possessing  more  or  less  merit. 
However,  with  the  accountant  the  chief  consideration  is  not  how 
many  methods  there  are,  but  rather  how  accurately  and  how 
quickly  he  can  solve  a  problem  in  interest. 

The  computation  of  the  product  of  principal,  rate,  and  time  is 
the  shortest  method  when  the  time  is  full  years  or  fractional  parts 
of  a  year,  such  as  ^-,  ^,  -J,  any  number  of  lOths,  and  so  forth;  other- 
wise, the  operation  may  be  shortened  by  taking  advantage  of 
aliquot  parts,  multiples  and  fractions,  cancellation,  and  so  forth. 

The  following  principles  and  methods  of  computing  interest  are 
quick  and  accurate  when  the  rate  is  £%. 

Sixty-day  method.     To  find  the  interest  at  6%  for: 

6  days,  point  off  3  additional  places  to  the  left  of  the  decimal  point  in  the 

principal. 
60  days,  point  off  2  additional  places  to  the  left  of  the  decimal  point  in  the 

principal. 
600  days,  point  off  1  additional  place  to  the  left  of  the  decimal  point  in  the 

principal. 
For  6,000  days,  the  interest  will  be  the  same  as  the  principal. 

Example 

Find  the  interest  on  $256.75  for  6  days  at  6%. 

Solution 

Pointing  off  3  places  to  the  left  of  the  decimal  point  in  the  principal  gives 
25675,  «r  26£. 

Example 
Find  the  interest  on  $345.65  for  36  days  at  6%. 

77 


78  SIMPLE  INTEREST 

Solution 

Point  off  3  additional  places  to  the  left  of  the  decimal  point  in  the  principal, 
and  multiply  by  6.     The  answer  is  $2.07. 

For  rates  other  than  6%,  see  adjustments  on  page  80. 

Problems 


Find  the  interest  at  6%  on: 


1.  $180.00  for  60  days.  6.  $26.50  for  18  days. 

2.  $150.00  for  54  days.  7.  $752.25  for  6  days. 

3.  $262.50  for  24  days.  8.  $15.80  for  54  days. 

4.  $32.75  for  36  days.  9.  $75.40  for  30  days. 
6.  $65.50  for  12  days.  10.  $12.85  for  24  days. 

Method  using  aliquot  parts. 

Example 
Find  the  interest  on  $275.84  for  124  days  at  6%. 

Solution 

$2  75  84  =  interest  for  60  days 

2  75  84  =  interest  for  60  days 

IS  38  =  interest  for  4  days 

$5~|70  06  =  interest  for  124  days 

Explanation.  Pointing  off  2  decimals,  as  indicated  by  the  vertical  line,  gives 
the  interest  for  60  days.  Double  this  to  find  the  interest  for  120rlays.  Four 
days'  interest  is  -fa  of  60  days'  interest.  The  sum,  $5.70,  is  the  interest  for 
124  days. 

Example 

Find  the  interest  on  $754.90  for  137  days  at  6%. 


$  7' 
7 
1 


Solution 

54  90  =  interest  for  60  days 
54  90  =  interest  for  60  days 
50  98  =  interest  for  12  days 
62  90  =  interest  for  5  days 
.68  =  interest  for  137  days 


Explanation.  Pointing  off  2  decimal  places  gives  the  interest  for  60  days. 
Double  this  to  find  the  interest  for  120  days.  Twelve  days  is  ^  of  60  days; 
therefore,  the  interest  for  12  days  is  £  of  60  days'  interest,  or  $1.5098.  Five 
days  is  T*  of  60  days,  and  the  interest  is  -£2  of  $7.5490,  or  $0.629.  The  sum, 
$17.24,  is  the  interest  for  137  days. 

After  a  little  practice,  any  number  of  days  can  be  resolved  into 
6-  or  60-day  periods  and  easy  fractions  thereof. 

Example 
Find  the  interest  on  $247.64  for  8  days  at  6%. 


SIMPLE  INTEREST  79 

Solution 

$1247.64  =  interest  for  6  days 

|§82.54  =  interest  for  2  days 

$|33§.18  =  interest  for  8  days 

Explanation.  To  find  the  interest  for  6  days,  point  off  3  decimals,  as  indi- 
cated by  the  vertical  line.  Two  days'  interest  is  4  of  6  days'  interest.  The 
inswer  is,  therefore,  33^. 

For  rates  other  than  6%,  see  adjustments  on  page  80. 

Problems 

Find  the  interest  at  6%  on: 

1.  S286.75  for  9  days.  6.  $175.82  for  34  days. 

2.  $189.22  for  8  days.  7.  $38.95  for  19  days. 

3.  $256.35  for  27  days.  8.  $47.56  for  17  days. 

4.  $178.56  for  39  days.  9.  $29.10  for  2  days. 

6.  $38.29  for  40  days.  10.  $1,286.75  for  21  days. 

The  cancellation  method.  The  cancellation  method  may  be 
ised  to  advantage  in  many  interest  calculations,  especially  in  those 
laving  fractional  rates  and  rates  other  than  6%. 

Example 

Find  the  interest  on  $750.00  for  45  days  at  5%. 

Solution 
125       15       .01 


4        0 

Explanation.    Writing  below  the  line  12  times  30,  instead  of  360  days,  facili- 
,ates  cancellation. 

Problems 

Find  the  interest,  by  the  cancellation  method,  on: 

1.  $840.00  for  12  days  at  2%.  6.  $284.00  for  34  days  at  6%. 

2.  $320.00  for  15  days  at  4%.  7.  $368.00  for  56  days  at  5%. 

3.  $160.80  for  16  days  at  5%.  8.  $775.14  for  79  days  at  5%. 

4.  $275.75  for  74  days  at  6%.  9.  $250.00  for  91  days  at  6%. 
6.  $112.50  for  85  days  at  4%.  10.  $500.00  for  102  days  at  4%. 

j^  Example 

Find  the  interest  on  $345.75  for  96  days  at  4i%. 

Solution 
Z 
345.75  X  00  X  .09      31.1175      00  oort 

12X20X2      -  -g-  =  *3'889' 


80  SIMPLE  INTEREST 

Problems 

Find  the  interest,  by  the  cancellation  method,  on: 

1.  $360.80  for  3H  days  at  4£%.  3.  $1,000.00  for  40  days  at  5j%. 

2.  $312.32  for  45  days  at  4|%.  4.  $1,600.00  for  75  days  at  4i%. 

Dollars-times-days  method,  6%.  This  method  is  rapid,  and 
is  particularly  valuable  when  a  calculating  machine  is  used.  It  is 
a  modification  of  the  cancellation  method,  where  6%  and  360  days 
anxiwD  .of  tlie  factors.  Tims: 

$  X  Days  X  .00 

300" 
6,000 

Assume  that  there  are  no  other  items  that  can  be  cancelled.  The 
number  of  dollars  is  multiplied  by  the  number  of  days,  and  the 
product  divided  by  6,000.  Any  number  may  be  divided  by  6,000 
by  pointing  off  3  decimals,  and  dividing  the  resultant  number  by  6. 

Example 

Find  the  interest  on  $256.50  for  2S  days  at  6%. 

Solution 

Multiply  the  number  of  dollars  by  the  number  of  days,  point  off  3  decimal 
places  in  addition  to  the  number  of  decimal  places  in  the  principal,  then  divide  by  6. 

$256  50 

28 

6)7  182  00 

1 . 1 97        or  $1.20,  the  interest 

This  method  may  be  used  for  any  rate  by  adding  to  or  subtract- 
ing from  the  interest  computed  at  6%,  the  fractional  part  thereof 
that  the  specified  rate  is  greater  or  less  than  the  6%  rate. 

For  8%,  increase  the  interest  by  ^  of  the  amount  computed  at  6%. 
For  7%,  increase  the  interest  by  ^  of  the  amount  computed  at  6%. 
For  5%,  decrease  the  interest  by  i  of  the  amount  computed  at  6%. 
For  4%,  decrease  the  interest  by  ^  of  the  amount  computed  at  6%. 
For  4^%,  decrease  the  interest  by  -j-  of  the  amount  computed  at  6%. 

The  above  adjustments  may  be  used  with  any  of  the  6% 
methods  in  solutions  in  which  the  rate  is  more  or  less  than  6%. 

Problems 
Find  the  interest  on  the  following: 

1.  $275.12  fop 73  days  at  5%.  4.  $138.42  for  28  days  at  4£%. 

2.  S132.S& for  28  days  at  8%.  5.  $276.95  for  17  days  at  8%. 

3.  15280.60  for  70  days  at  4%.  6.  $640.64  for  56  days  at  7%. 


SIMPLE  INTEREST  81 

Interchanging  principal  and  time.  Under  the  60-day  method, 
the  computations  may  often  be  shortened  by  interchanging  the 
principal  and  the  time. 

Example 

Find  the  interest  on  $6,000.00  for  31  days  at  6%. 

Solution 

Interchanging  the  principal  and  the  time,  the  problem  becomes  that  of 
finding  the  interest  on  $31.00  for  6,000  days.  Apply  the  6%,  60-day  method, 
and  the  interest  is  found  to  be  $31.00,  since  the  interest  is  equal  to  the  principal 
when  the  rate  is  6%  and  the  time  is  6,000  days. 

Problems 

Find  the  interest  on  the  following: 

1.  $2,400.00  for  23  days  at  6%.  4.  $3,000.00  for  193  days  at  6%. 

2.  $3,600.00  for  7  days  at  6%.  6.  $4,500.00  for  3S  days  at  Q%. 

3.  $6,000.00  for  156  days  at  6%.  6.  $4,200.00  for  41  days  at  6%. 

Exact  or  accurate  interest.  Exact  or  accurate  interest  is  that 
which  is  obtained  when  a  year  is  taken  as  365  days.  For  full 
years,  all  methods  of  computing  interest  give  the  same  result-  a 
certain  per  cent  of  the  principal;  hence  the  results  differ  only  when 
fractional  parts  of  a  year  are  used. 

Example 

Find  the  exact  interest  on  $1,200.00  for  93  days  at  6%. 

Solution 

The  cancellation  method  previously  explained  is  the  method  used,  as  it  i& 
probably  the  most  practical. 

240 
K«»X  93  XJ>6  _  1,339.20  _ 

cttltf  —  T o  "~~    'ff'AO.OtJ 

MT^f'  »  «J 

73 

Problems 

Find  the  exact  interest  on: 

1.  $750.00  for  45  days  at  6%.  2.  $1,200.00  for  68  days  at  7%. 

3.  $1,600.00  for  73  days  at  6i%. 

Accumulation  of  simple  interest.  Simple  interest  accumulates 
in  like  amount  each  period,  if  the  principal  and  rate  are  unchanged. 

Symbols.  Let  i  equal  the  rate  of  interest,  n  the  number  of 
periods,  and  P  the  principal.  Then  accumulation  of  simple 
interest  on  any  sum  of  money,  for  any  number  of  periods,  may  be 
found  as  follows: 


82  SIMPLE  INTEREST 

Example 

Find  the  simple  interest  on  $100.00  for  5  years  at  6%. 

Algebraic  Formula  Arithmetical  Substitution 

P(l  x  in)  =  Interest.  100(1  X  .06  X  5)  =  30. 

Solution 

1  X  .06  =  .06,  interest  on  1  for  1  year  at  6% 
.06  X  5  =  .30,  interest  on  1  for  5  years  at  6% 
.30  X  100  =-  $30.00,  interest  on  $100.00  for  5  years  at  6% 

TABLE  OF  SIMPLE  INTEREST 


(1) 

(2) 

(3) 

(4) 

(5) 

Total  Int. 

End  of 

Interest  Due 

at  End  of 

Sum  Due  it 

Year 

Principal 

Each  Year 

Each  Year 

End  of  Year 

1 

$100  00 

$6.00 

$  6.00 

$106  00 

2 

100  00 

6.00 

12  00 

112.00 

3 

100.00 

6.00 

18  00 

118  00 

4 

100.00 

6  00 

24  00 

124  00 

5 

100.00 

6  00 

30.00 

130  00 

Problems 

1.  A  man  borrows  $500.00  for  9  years  at  4%.     What  amount  of  interest  will 
he  pay  during  this  period?     Write  the  formula  and  solution,  as  shown  in  the 
example  above. 

2.  What  is  the  amount  of  interest  due  on  $300.00  at  the  end  of  10  years  if 
the  rate  is  7%?     Write  the  formula  and  solution,  as  shown  in  the  example 
above. 

3.  Construct  a  table  in  columnar  form,  similar  to  the  table  above  (omitting 
column  5),  for  $400.00  invested  for  5  years  at  6%. 

4.  What  is  the  interest  accumulation  on  a  debt  of  $4,270.00  for  8  years  at 
5%  simple  interest? 

Simple  amount.  The  simple  amount  is  found  by  adding  to  the 
principal  the  totaljimple  interest.  It  is  the  amount  due  at  the 
end  of  the  stated  period. 

Example 

What  is  the  amount  of  $100.00  for  5  years  at  6%? 

Algebraic  Formula  Arithmetical  Substitution 

P  +  [P(l  X  in)]  -  Amount.  100  +  [100(1  X  .06  X  5)]  =  130. 

Solution 

1  X  .06  =  .06,  interest  on  1  for  1  year  at  6% 
.06  X  5  =  .30,  interest  on  1  for  5  years  at  6% 
100  X  .30  =  30.00,  interest  on  100  for  5  years  at  6% 
100  +  30.00  =  $130.00,  amount  of  $100.00  for  5  years  at  6% 

The  simple  amount  is  shown  in  column  5  of  the  table  above; 
hence  the  construction  of  a  table  is  omitted  here. 


SIMPLE  INTEREST  83 

Problems 

Write  formulas  and  solutions  for  the  following: 

1.  The  amount  of  a  $200.00  note  due  in  6  years;  interest,  5%. 

2.  The  amount  due  in  6  years  on  $530.00  at  6%. 

3.  The  amount  due  on  a  note  for  $750.00  with  4  %  interest.     No  interest  has 
been  paid  during  a  period  of  4  years. 

Rate.  To  find  the  rate,  when  the  principal,  interest,  and  time 
are  given,  divide  the  given  interest  by  the  interest  on  the  principal 
at  1%  for  the  given  time. 

Example 
At  what  rate  will  $100.00  produce  $24.00  in  4  years? 

Algebraic  Formula  Arithmetical  Substitution 

Interest         _    .      ,  .   .  24  fl 

=  Rate  °f  lnterest' 


100(1  X  .01  X  4) 
Solution 

1  X  .01  =  .01,  interest  on  1  at  1%  for  1  year 
.01  X  4  =  .04,  interest  on  1  at  1  %  for  4  years 
100  X  .04  =  4.00,  interest  on  100  at  1  %  for  4  years 
24  -f-  4  =  6,  or  6%,  the  rate  of  interest 

Problems 

Write  formulas  and  solutions  for  each  of  the  following: 


Principal 
1.  $    400  00 

Interest 
$  48.00 

Time 
3  years 

Rate 

2.     2,000.00 

500.00 

5  years 

3.       800.00 

336  00 

6  years 

4.       300  00 

126.00 

7  years 

5.        150.00 

40.50 

6  years 

Time.  To  find  the  time,  when  the  principal,  interest,  and  rate 
of  interest  are  given,  divide  the  interest  by  the  product  of  the 
principal  and  the  given  rate  for  one  year. 

Example 
In  what  time  will  $100.00  invested  at  6%  produce  $24.00  interest? 

Algebraic  Formula  Arithmetical  Substitution 

Interest        _.  24 

Time. 


X  in)  100(1  X  .06  X  1) 

Solution 

1  X  .06  =  .06,  interest  on  1  at  6%  for  1  year 
100  X  .06  *  6,  interest  on  100  at  6%  for  1  year 
24  -r  6  =  4,  the  number  of  years 


84  SIMPLE  INTEREST 


Problems 

Principal 

Interest         Rate 

1.  $1,000  00 

$240  00         6% 

2.        750  00 

90  00        4% 

3.        300  00 

81  00        4i% 

Time 


Present  worth.  The  present  worth  of  a  debt,  due  at  some 
future  time,  without  interest,  is  the  sum  which  must  be  invested 
now  in  order  to  produce  the  specified  amount  at  the  end  of  the 
period.  Thus,  since  $1  invested  for  5  years  at  6%  will  amount  to 
$1.30,  the  amount  that  must  be  invested  now  at  6%  to  produce 
$1  at  the  end  of  5  years  is  1.00/1.30,  or  $.7692. 

To  find  the  present  worth  of  a  sum,  multiply  the  sum  by  the 
present  worth  of  $1.00  for  the  given  time. 

Example 

What  is  the  present  worth  of  a  note  for  $100.00,  due  in  5  years,  without 
interest,  money  being  worth  6%? 

Algebraic  Formula  Arithmetical  Substitution 

( ~ —  ]  =  Present  worth.        100  (--       /t     *  ^          \  =  76.92. 

\1  -f  (1  X  m)J  \1  +  (1  X  .00  X  5)/ 

Solution 

1  X  .06  =  .06,  interest  for  1  year  on  1 
.06  X  5  =  ..30,  interest  for  5  years  on  1 
1  +  .30  =  1.30,  amount  of  1  for  5  years 
1  -T-  1 .30  =  .7692,  present  worth  of  1  for  5  years 
100  X  .7692  =  $76.92,  present  worth  of  $100.00  for  5  years 

Verification 

$76.92  X  .06  =  $4.6154,  interest  for  1  year  on  present  worth 
$4.6154  X  5  =  $23.08,  interest  for  5  years  on  present  worth 
$76.92  +  $23.08  =  $100.00,  amount  due  in  5  years 

TABLE  OF  PRESENT  WORTH 


P 


(1) 

Years 
1 
2 
3 
4 
5 

(2) 

Principal 
$100  00 
100  00 
100.00 
100.00 
100.00 

(3) 

Divided  by 
Amount  of  $1 
$1.06 
1.12 
1.18 
1.24 
1  30 

(4) 

Equals 
Present  Worth 
$94  34 
89  29 
84.74 
80  65 
76  92 

(5) 
Principal  Minus 
Present  Worth 
Equals  Discount 
$  5  66 
10.71 
15  26 
19  35 
23  08 

Comparison  of  simple  amount  and  simple  present  worth.    The 

following  comparative  chart  is  presented  to  illustrate  the  accumu- 


SIMPLE  INTEREST 


85 


lation  of  simple  interest  on  a  sum  and  on  the  present  worth  of  the 
same  sum: 

130 


124 

118 

112 

106 

AMOUNT    KXH 

100 

100 

94.34 

90- 

89.28 

84.74 

80.65 

r»                    80- 

PRRSKNT 

76.92 

WORTH 
70- 

60- 

^~  ' 

"  —  —  . 

'—  —  ~__^.^ 

~~~        "^ 

r-^ 

,  -—1 

"""""'I 

0- 

BASIS  OP 
AMOUNT 


BASIS  OF 
PRESENT  WORTH 


Jan.  1,      Dec.  31.     Dec.  31.     Dec.  31.     Dec.  31.     Dec.  31. 
IstYr.      letYr.       2nd  Yr.     3rd  Yr.      4th  Yr.     6th  Yr. 

Figure  1. 

The  amount  starts  at  $100.00,  and  accumulates  to  $130.00  in 
5  years.  The  present  worth  starts  at  $76.92,  and  accumulates  to 
$100.00.  The  rate  of  interest  is  the  same  in  each  case,  6%. 

Problems 

1.  What  is  the  present  value  of  a  6-year  note  for  $650.00,  without  interest, 
if  money  is  worth  5%?  Write  the  formula  and  solution,  as  shown  in  the  example 
on/page  84. 

'  2.  A  note  for  S3, 500. 00,  without  interest,  is  due  in  5  years.  What  is  its 
present  value,  money  being  worth  6%?  Write  the  formula  and  solution,  as 
shown  in  the  example  on  page  84. 

3/ Construct  a  table  in  columnar  form,  similar  to  the  table  on  page  84, 
(omitting  column  5).  Use  $1.00  as  the  principal,  4  years  as  the  time,  and  5% 
aaoihe  interest  rate. 

4.  Construct  a  comparative  chart  showing  the  difference  in  value  of  the 
amount  and  the  present  worth  of  $400.00  due  in  8  years,  interest  at  5%. 

True  discount.  True  discount  is  the  difference  between  the 
sum  due  and  its  present  worth  computed  on  a  simple  interest 
basis.  (See  page  84.) 

Example 

Find  the  true  discount  on  a  debt  of  $100.00  due  in  5  years,  without  interest, 
money  being  worth  6%. 


86  SIMPLE  INTEREST 

Solution 

The  present  worth"of  the  debt  is  the  sum  shown  in  the  solution  on  page  84, 
or  $76.92. 

$100.00  -  $76.92  =  $23.08,  the  true  discount 

Refer  to  column  5  of  the  Table  of  Present  Worth,  page  84,  for 
the  method  of  showing  the  true  discount  in  columnar  form. 

Problems 

1.  What  is  the  true  discount  on  a  debt  of  $750.00  due  in  3  years,  money  being 
worth  4%?    Write  the  formula  and  solution. 

2.  Find  the  difference  between  the  present  value  and  the  face  value  of  a 
non-interest-bearing  note  for  $500.00,  due  in  4  years,  money  being  worth  6%. 
Write  the  formula  and  solution. 

3.  Construct  a  table  in  columnar  form,  similar  to  the  table  on  page  84,  for 
$1.00  at  4%  for  6  years. 

4.  Find  the  difference  between  the  true  discount  and  the  simple  interest  on 
$650.00  for  8  years  at  4%. 


CHAPTER  8 
Bank  Discount 

Definition.  Bank  discount  is  a  deduction  made  from  the 
amount  due  at  maturity  on  a  note  or  draft,  in  consideration  of  its 
being  converted  into  cash  before  maturity.  If  the  note  does  not 
bear  interest,  its  face  value  is  the  amount  due  at  maturity.  If  the 
note  does  bear  interest,  the  amount  due  at  maturity  is  the  face 
value  plus  interest  on  the  face  value  for  the  period  and  at  the  rate 
specified  in  the  note. 

In  bank  discount,  the  time  is  the  period  from  the  date  of  dis- 
count to  the  date  of  maturity  of  the  note.  The  date  of  maturity 
of  a  note  is  the  day  011  which  it  is  due.  Notes  due  a  given  number 
of  days  after  date  mature  after  the  exact  number  of  days  have 
elapsed.  Notes  due  a  given  number  of  months  after  date  mature 
on  the  same  date  so  many  months  hence,  except  notes  made  on  tho 
31st  and  falling  due  in  a  30-day  month,  which  mature  on  the  30th, 
and  notes  made  on  the  29th,  30th,  or  31st  of  some  month  and 
falling  due  in  February,  which  mature  on  the  last  day  of  February. 

Example 

A  note  due  30  days  after  January  31,  will  mature  on  March  2;  but  if  the  note 
is  due  in  one  month,  it  will  mature  on  the  last  day  of  the  succeeding  month,  or 
February  28.  If  the  year  should  be  a  leap  year,  the  maturity  dates  would  be 
March  1  and  February  29. 

Counting  time.  In  counting  time,  the  usual  method  is  to 
count  the  first  succeeding  day  as  one  day.  To  illustrate,  if  a  note 
is  given  on  January  15  for  10  days,  the  16th  is  counted  as  the  first, 
the  17th  as  the  second,  the  18th  as  the  third,  and  January  25  as  the 
tenth  day. 

Finding  the  difference  between  dates  by  use  of  a  table.  By 
numbering  the  days  of  the  year,  a  calendar  may  be  made  for 
determining  the  number  of  days  between  any  two  dates.  A 
portion  of  such  a  table,  and  the  use  made  of  it,  are  illustrated  on 
page  88. 

87 


2  

306 

3  

307 

4  

308 

5  

309 

6  

310 

7  

311 

8  

312 

9  

313 

10  

314 

88  BANK  DISCOUNT 

May  1   121     Nov.  1 305 

2 122 

3 123 

4 124 

5      125 

6 126 

7     127 

8     128 

9     129 

10 130 

The  number  of  days  between  May  4  and  November  9  is  found 
as  follows  : 

The  table  shows  that  November  9  is  the  313th  day  of  the  year 
The  table  shows  that  May  4  is  the  124th  clay  of  the  year 

Therefore,  the  difference  is  ISO      days,  the  time  required 

Another  form  of  table  is  one  that  shows  the  number  of  days 
from  any  day  of  any  month  to  the  corresponding  day  of  any  other 
month  not  more  than  one  year  later. 

Jan.  Feb.  Mar.  Apr.  May  June  July  Aug.  Kept.  Oct.  Nov.  Dec. 

January    ...  365  31  59  90  120  151  181  212  243  273  304  334 

February    .  334  305  28  59  89  120  150  181  212  242  273  303 

March  ..  306  337  365  31  61  92  122  153  184  214  245  275 

April 275  306  334  365  30  61  91  122  153  183  214  244 

May  ....  245  276  304  335  365  31  61  92  123  153  184  214 

June  214  245  273  304  334  365  30  61  92  122  153  183 

July 184  215  243  274  304  335  365  31  62  92  123  153 

August 153  184  212  243  273  304  334  365  31  61  92  122 

September    .  122  153  181  212  242  273  303  334  365  30  61  91 

October.     ..  92  123  151  182  212  243  273  304  335  365  31  61 

November..  61  92  120  151  181  212  242  273  304  334  365  30 

December...  31  62  90  121  151  182  212  243  274  304  335  365 

Example 

A  note  due  August  17  was  discounted  June  10.  What  was  the  term  of 
discount? 

Solution 

From  the  table,  June  10  to  August  10 61  days 

August  10  to  August  17 7  days 

Total 68  days 

Banks  do  not  compute  time  uniformly,  but  the  methods  given 
here  are  in  common  use. 

NOTE:  Tables  similar  to  those  above  may  also  be  used  to 
good  advantage  in  computing  the  unexpired  time  of  insurance 
policies. 

Proceeds.  The  proceeds  of  a  note  is  the  difference  between  the 
amount  due  at  maturity  and  the  bank  discount. 


BANK  DISCOUNT  89 

To  find  bank  discount  and  proceeds. 

Compute  the  bank  discount  as  simple  interest  on  the  amount 
due  at  maturity  for  the  unexpired  time  (term  of  discount).  Deduct 
the  bank  discount  from  the  value  of  the  note  at  maturity  to  obtain 
the  proceeds. 

Example  1 

Find  the  bank  discount  and  the  proceeds  if  a  non-interest-bearing  note  for 
$420.00  due  in  90  days  is  discounted  at  0^. 

Solution 

Face  of  note  ...   .'  ..............................  $420.00 

Bank  discount,  90  days       ......................  (5  30 

Proceeds         ..............................  1413.70 

Example  2 

A  note  for  $780.00  dated  May  5,  payable  in  6  months  with  interest  at  6%, 
is  discounted  at  6%  on  August  3.  Find  the  bank  discount  and  the  proceeds. 

Solution 
Face  of  note  ................................         $780  00 

Interest  for  6  months  at  f)%  .......................       23  40 

Value  of  note  at  maturity       .....  ........         $803  40 

Bank  discount  on  $803.40  for  94  days  ;it  6%  ...        12  59 

Proceeds  .......................................  $790~si 

To  find  the  face  of  a  note  when  the  proceeds,  time,  and  rate 
of  discount  are  given.  Divide  the  proceeds  of  the  note  by  the 
proceeds  of  $1.00  for  the  given  rate  and  time. 

Example 

For  what  sum  must  a  non-interest-bearing  note  be  drawn,  due  in  90  days, 
so  that  when  it  is  discounted  at  a  bank  at  6%  per  annum,  the  proceeds  will  bo 
$537.40? 

Solution 

$0.015  =  bank  discount  on  $1.00  for  90  days 
$1.00  -  $0.015  -  $0.985,  the  proceeds  of  $1.00 
$537.40  -T-  $0.985  =  545.58  times,  or  $545.58 


Problems 

Find  the  barik  discount  and  the  proceeds  on: 

J)  A  note  for  $750.00,  due  May  30  without  interest,  and  discounted  April  16 
at  6%. 

@  A  note  for  $1,200.00,  due  December  4  without  interest,  and  discounted 
Oct.  29  at  6%. 

(§/  A  note  for  $1,500.00,  dated  October  8  and  due  in  4  months  with  interest 
at  6%,  discounted  December  1  at  6%. 


90  BANK  DISCOUNT 


AT) A  note  for  $800.00,  dated  September  9  and  due  in  G  months  with  interest 
at  T%,  discounted  November  11  at  6%. 

{€)  A  note  for  $250.00,  dated  July  11  and  due  in  90  days  with  interest  at 
5i%,  discounted  September  1  at  6%. 

$443.03  wiis  received  a.s  the  proceeds  of  a  90-day  note  discounted  at  6%. 
the  face  of  ttie  note? 

(ft  For  what  sum  must  a  60-day  note  be  drawn  in  order  that  the  proceeds 
will  be  $600.00  when  the  note  is  discounted  at  6%? 

(K\  Find  the  date  of  maturity,  the  term  of  discount,  the  bank  discount,  and 
the  proceeds  of  a  60-day  note  for  $750.00,  dated  July  8  and  discounted  July  17 
at  5%. 

(§)  Find  the  date  of  maturity  and  the  term  of  discount  of  a  90-djty  sight 
draft,  dated  May  14,  accepted  May  17,  and  discounted  June  10. 

(iCy  Find  the  date  of  maturity,  the  term  of  discount,  the  bank  discount,  and 
the  proceeds  of  a  note  for  $050.00,  dated  Nov.  30,  due  in  3  months,  and  dis- 
counted Jan.  5  at  0%. 


CHAPTER  9 
Partial  Payments 

Part  payments  on  debts.  A  debtor  may  by  agreement  make 
equal  or  unequal  payments  on  the  principal  at  regular  or  irregular 
intervals.  Any  partial  payment  of  a  note  or  draft  should  be 
recorded  on  the  back  of  the  note  or  draft. 

Methods.  There  are  two  methods  of  applying  payments  of 
principal  and  interest  to  the  reduction  of  an  interest-bearing  debt. 
The  method  adopted  by  the  Supreme  Court  of  the  United  States 
is  termed  the  "United  States  Rule";  the  other  method,  which  has 
been  widely  adopted  by  businessmen,  is  termed  the  "Merchants' 
Rule." 

United  States  Rule.  The  United  States  Rule  is  now  a  law  in 
many  of  the  states,  having  been  made  so  either  by  statute  or  by 
court  decision. 

The  court  holds  that  when  a  part  payment  is  made  on  an 
interest-bearing  debt,  the  payment  must  first  be  used  to  discharge 
the  accumulated  interest,  and  what  remains  of  the  payment  is  then 
applied  in  cancellation  of  the  principal.  If  the  payment  is  smaller 
than  the  accumulated  interest,  no  cancellation  takes  place,  and  the 
previous  principal  continues  to  draw  interest  until  the  accumulated 
payments  exceed  the  accumulated  interest. 

Example 

An  interest-bearing  note  for  $1,800.00  dated  March  J,  1944,  had  the  following 
indorsements: 

September  27,  1944  ....  $500  00 

March  15,  1945     2500 

June  1,  1945  700  00 

How  much  was  due  September  1,  1945? 

Solution 

Yr.    Mo.     Da.  Yrs.   Moz.  Days 

Date  of  note. . .  1944—3 1 

First  payment,  $500.00  ....    1944— 9—27  6  26 

Second  payment,  $25.00    ..    .    1945-3 15  5  18 

Third  payment,  $700.00 1945—6 1  2  16 

Settlement 1945—9 1  3  0 

I         6  0 

91 


S>2  PARTIAL  PAYMENTS 

Explanation.  The  time  is  found  by  successive  subtractions  of  the  first  date 
from  the  second,  the  second  from  the  third,  and  so  on.  The  sum  of  the  different 
times  is  equal  to  the  time  between  the  date  of  the  note  and  the  date  of  settlement. 

Face  of  note,  March  1,  1944 $1,80000 

Interest  on  $1,800.00  at  6%  from  March  1  to  Sept.  27,  6  months  and 

26  days 61   SO 

Amount  due  Sept.  27,  1944 $1, SGI  SO 

Deduct  payment ,500  00 

Balance  due  Sept.  27,  1944...  ^1~36T~SO 

Interest  on  $1,361.80  at  6%  from  Sept.  27  to  March  15,  5  months  and 
18  days,  $38.13.     As  this  inteicst  is  larger  in  amount  than  the  pay- 
ment made  at  March  15,  the  interest  is  not  added  and  the  payment 
is  not  deducted. 
Interest  on  $1,361.80  at  6%  from  Sept.  27  to  June  1,  1945,  8  months 

and  4  days 55  38 

Amount  due  June  1,  1945         $1,417.18 

Deduct  sum  of  payments:  March  15  $  25  00 

June  1 700  00        725  00 

Balance  due  June  1,  1945  $    (>92~Ts 

Interest  on  $692.18  at  6%  from  June  1  to  Sept.  I,  1945,  3  months  • .    _  10  38 
Balance  due  September  1,  1945 $    702  56 

Problems 

1.  A  note  for  $1,650.00  was  dated  May  20,  1944.     The  interest  was  6%  from 
date,  and  the  following  payments  were  indorsed: 

Sept.  8,  1944 $  45  00 

Dec.  14,  1944 20  00 

Feb.  26,  1945 5000 

July  5,  1945 90  00 

Nov.  14,  1945 25000 

What  amount  was  due  December  17,  1945? 

2,  A  note  for  $1,200.00  was  dated  June  20,  1941,     The  interest  was  6%  from 
date,  and  the  following  payments  were  indorsed: 

Oct.  2,  1941 $120  40 

February  S,  1942  ..  2950 

May  23,  1942 5640 

December  11,  1942 ..  38875 

What  amount  was  due  January  23,  1943? 

3.  A  note  for  $1,000.00  was  dated  April  10,  1938.  The  interest  was  7%  from 
date,  and  the  following  payments  were  made: 

November  10,  1939 $  80  50 

July5,  1940 10000 

January  10,  1941     45080 

October  1,  1943          .                       50000 

What  amount  was  due  January  1,  1944? 


PARTIAL  PAYMENTS  93 

Merchants'  Rule.  Find  the  amount  of  the  debt  (principal  and 
interest)  to  the  date  of  final  settlement,  or  if  the  debt  runs  for  more 
than  one  year,  find  the  amount  to  the  end  of  the  first  year.  Deduct 
from  this  the  sum  of  all  the  payments  and  interest  on  same  to  the 
date  of  settlement,  or  to  the  end  of  the  year.  The  remainder  will 
be  the  amount  due  at  the  date  of  settlement,  or  at  the  beginning 
of  the  next  year. 

Example 

For  purposes  of  comparison,  the  same  problem  will  be  used  here  as  was  used 
to  illustrate  the  United  States  Kule. 

Solution 

Face  of  note,  March  1 ,  1942  $1,SOO  00 

Interest,  1  year  at  (>%  to  March  1,  1943 I  OS  00 

$I,90S  00 
Deduct: 

First  payment,  September  27,  1942 $500  00 

Interest  at  6%  to  March  1,  1943,  5  months 

and  4  days.  ^12  S3        512  S3 

Balance  due  at  begi lining  of  second  year  $1,395   17 

Interest  on  $1,395. 17  at  (>%,  March  1  to  Sept.  1, 

1943,  0  months         41   SO 

*l~437l)3 
Deduct: 

Second  payment,  March  15,  1943  $  25  00 

Interest  at  6%  from  March   15  to  Sept.   1, 

1943,  5  months  and  16  days 09 

Third  payment,  June  1,  1943 700.00 

Interest  at  (>';<  from  June  1  to  Sept.  1,  1943, 

3  months __10  50       730  19 

Balance  due ~~  $    700  ~~S4 

The  difference  of  $1.72  between  the  balance  as  computed  by 
the  Merchants'  Rule  and  the  balance  as  computed  by  the  United 
States  Rule  is  small,  but  a  much  greater  difference  will  occur  when 
the  time  is  long  and  the  amount  large. 

It  is  usual  to  compute  the  balance  due  on  notes  of  one  year  or 
less  by  the  Merchants'  Rule. 

Problems 

1.  A  note  for  $950.00  with  interest  at  6%  was  dated  Feb.  3,  1943,  and  had 
the  following  indorsements: 

March  1,  1943 $150.00    July  8,  1943 $300.00 

June  3,  1943 96 . 00     December  20,  1 943 ....     250 . 00 

What  amount  was  due  January  17,  1944? 

2.  A  note  for  $791.84  with  interest  at  6%  was  dated  December  14,  1942,  and 
bore  the  following  indorsements; 


94  PARTIAL  PAYMENTS 

January  3,  1943 $100  00    July  29,  1943 $324  00 

March  16,  1943 240.00    Augusts,  1943 20.00 

What  amount  was  due  November  14,  1943? 

3.  A  note  for  $1,200.00  dated  April  1,  1942,  bore  interest  at  7%  and  had  the 
following  indorsements: 

April  12,  1942 $161  08    July  28,  1942        J  17  90 

July  19,  1942 224  14     January  29,  1943 100  25 

What  amount  was  due  April  1,  1943? 


CHAPTER  10 
Business  Insurance 

Kinds  of  insurance.  There  are  at  least  21  kinds  of  insurance 
applicable  to  the  ordinary  business  being  done  in  big  cities  and  as 
many  as  150  kinds  of  insurance  covering  all  branches  of  human 
endeavor. 

Policy.  An  insurance  policy  is  a  written  contract.  The  con- 
sideration given  for  the  protection  promised  consists  of  a  premium 
to  be  paid  in  money  and  the  fulfillment  by  the  insured  of  acts  of 
commission  and  omission  according  to  the  terms  and  conditions 
set  forth  in  the  policy. 

Fire  insurance.  Fire  insurance  is  guaranty  of  indemnity  for 
loss  or  damage  to  property  by  fire.  Insurance  companies  are 
liable  for  loss  or  damage  resulting  from  the  use  of  water  or  chemi- 
cals used  in  extinguishing  the  fire  and  from  smoke.  A  fire  loss  is 
predicated  on  the  sound  value  at  the  time  the  loss  is  sustained  and 
not  at  the  time  the  insurance  is  written. 

Form  of  policy.  With  but  a  few  exceptions,  fire  insurance 
companies  use  a  State  standard-form  policy  made  mandatory  by 
the  State  in  which  they  operate.  The  New  York  State  standard 
form  of  policy  is  the  one  that  is  generally  used,  as  it  embraces 
nearly  all  that  is  contained  in  other  forms.  The  form  attached  to 
the  policy  is  known  as  a  rider.  £The  rider  form  directly  applies  the 
insurance  to  fit  the  facts  and  conditions  of  the  particular  risk.  It 
also  amends  the  standard  form,  which  is  not  a  contract  until  com- 
pleted by  descriptions  and  amendments, 

Rates.  Probably  no  phase  of  insurance  interests  the  business- 
man more  than  his  insurance  rate.  Independent  rating  bureaus 
operate  in  different  parts  of  the  country.  Their  business  is  to 
inspect  and  to  measure  the  hazards  in  terms  of  rates.  Rate 
schedules  are  compiled  for  this  purpose.  The  charges  are  in  the 
nature  of  penalties  for  hazards. 

Example 

A  particular  building  has  been  inspected  and  surveyed  by  the  rater.  The 
degree  of  municipal  and  local  protection  has  been  measured.  This  establishes 
the  basic  rate.  Assume  the  basic  rate  to  be  .40,  which  is  a  charge  commensurate 
with  the  degree  of  protection  and  covers  all  general  hazards  that  cannot  be 

95 


96  BUSINESS  INSURANCE 

segregated  and  measured.     The  better  the  city  protection,  the  lower  will  be  the 
basic  rate. 

Basic  rate 40 

Area:  15,SOO  sq.  ft     04 

(The  standard  unit  area  is  1 ,000  sq.  ft.,  and  an  additional 

charge  is  made  for  larger  areas.) 

Parapet  wall  deficiency 04 

Skylights  not  standard  construction 02 

Metal  stacks  through  roof .  ....       06 

Outside  wood  cornices,  loading  docks,  and  wooden  conveyor.      .06 

Gallery  decks  used  for  storage   ...        03 

Occupancy  hazard  (woodworking  mill) 92 

Shavings  allowed  to  accumulate 05 

No  cans  for  collecting  waste. .    .  05 

No  drip  pans  under  machines 05 

Floor  oil-soaked .  .         05 

Total     177 

Credit  for  open  finish  (inside  walls) .  .  ...        OS 

Building  rate  unexposed 1 .69 

Kxposuro: 

From  buildings  No.  2  and  No.  5  at  IS  ft 34 

From  building  No.  6  at  15  ft .02 

From  office  at  23  ft .05 

From  buildings  No.  9  and  No.  10 .  .     07 

Exposure  charge .4S 

Total  building  rate         217 

If  this  assured  would  have  the  parapet  wall  brought  up  to  the  standard 
requirements,  his  rate  would  be  reduced  .04.  By  having  the  shavings  removed 
daily,  and  by  installing  waste  cans  and  drip  pans  under  the  machines,  the  rate 
could  be  reduced  another  .15.  As  a  matter  of  fact,  the  owner  makes  his  own 
rate — the  rater  simply  measures  the  hazards  in  terms  of  rates. 

To  find  the  premium.  Insurance  companies  charge  a  certain 
number  of  cents  or  dollars  for  insuring  each  $100.00  worth  of 
property.  Thus,  the  insurance  rate  in  the  foregoing  example  is 
$2.17  for  each  hundred  dollars  of  insurance  carried.  If  the  build- 
ing is  valued  at  $05,000.00  and  is  insured  for  full  value,  the  amount 
of  the  premium  would  be  computed  as  follows: 

$2  17     the  rate  per  $100.00  of  insurance. 
X  6  50    the  number  of  hundred  dollars  of  insurance  purchased. 


$1,410.50    the  premium,  or  cost  of  the  insurance  for  one  year. 

Agent's  commission.  Local  agents  of  the  fire  insurance 
companies  are  located  in  almost  every  city  and  town.  They  act 
as  the  representatives  of  the  companies,  >oliciiing  the  business 
and  collecting  the  premiums.  For  this  service  they  receive  a 
certain  per  cent  of  the  premiums. 


BUSINESS  INSURANCE  97 

Example 

A  store  building  valued  at  $10,000.00  was  insured  for  80%  of  its  value,  the 
rate  being  SI. 25  a  hundred.  What  was  thf  agent's  commission  if  he  received 
15%  of  the  premium? 

Solution 

80%  of  $10,000.00  =  $8,000.00,  the  insured  value. 

SO  X  SI. 25  =  $100.00,  the  premium. 
15%  of  $100.00  =  $15.00,  the  agent's  commission. 

Cancellation  of  policies.  Both  the  insurance  company  and  the 
insured  have  the  right  to  cancel  an  insurance  policy  at  any  time. 
When  the  policy  is  canceled  by  the  insurance  company,  the  por- 
tion of  the  premium  to  be  repaid  to  the  insured  is  determined  pro 
rata. 

Example 

On  April  10,  the  owner  of  a  building  insured  his  property  for  one  year.  The 
premium  was  $36.00.  On  October  20,  the  policy  was  canceled  by  the  insurance 
company.  What  rebate  did  the  insured  receive  for  the  unexpircd  term? 

Solution 

The  time  from  April  10  to  October  20  is  103  days,  expired  term  of  the  policy. 
(See  page  88  for  table  of  number  of  days  between  dates.) 

ill  of  $30.00  is  $19.04,  amount  of  premium  earned. 
$36.00  -  $19.04  =  $16.96,  amount  of  premium  returned. 

When  the  policy  is  canceled  by  the  insured,  the  amount  of 
premium  returned  is  determined  by  the  "short  rate/'  The  short 
rate  is  an  arbitrary  per  cent  fixed  by  the  insurance  companies,  and 
is  shown  by  a  table  like  the  one  at  the  top  of  page  98. 

Example 

On  May  2,  a  one-year  policy  was  written  on  a  shop.  The  premium  was 
$38.75.  On  September  26,  the  policy  was  canceled  at  the  request  of  the  insured. 
What  rebate  did  the  insured  receive? 

Solution 

From  May  2  to  September  26  is  147  days.  The  table  shows  that  60%  of 
the  premium  is  to  be  retained  when  the  policy  has  been  in  force  150  days,  which 
is  the  number  of  days  next  higher  than  147.  Then  40%  of  the  premium  will  be 
returned 

$38.75  X  .40  =  $15.50,  the  return  premium. 

Coinsurance.  This  is  a  form  of  insurance  in  which  the  person 
who  insures  his  property  agrees  to  carry  insurance  equal  to  a 
certain  percentage  of  the  valuation  of  the  property.  If  he  fails 
to  carry  that  percentage  with  an  insurance  company,  he  (the 


98  BUSINESS  INSURANCE 

SHORT  RATE  CANCELLATION  TABLE 

Period  exceeding  20  days  and  not  exceeding  25  days,  to  be  the  rate  of  25  days, 
and  so  on  up  to  one  year. 


Policy  in  Force 

Per  Cent 
of  A  nnual 
J'rem. 

Pi 

rticy 

in  Force 

Per  Cent 
of  Annual 
Prem. 

1  day     

...       2% 

55 

days 

..     29% 

2  days  

.-*       4% 

60 

days 

or  2 

months.  . 

..     30% 

3  days  

...       5% 

65 

days 

..     33% 

4  days  

...       6% 

70 

days 

..     36% 

5  days  

...       7% 

75 

days 

.     37% 

6  days  

...     s% 

SO 

days 

.  .     3S% 

7  days  

.  ..       9% 

85 

days 

..     39% 

S  days  

...       9% 

90 

days 

or  3 

months 

40% 

9  days  

.  ..      10% 

105 

days 

..     46% 

lOdays     

,  ..    10% 

120 

days 

or  4 

months 

.       50% 

1  1  days       

...    1  1  % 

135 

days 

..     56% 

12  days     

,  .  .  .    1  1  % 

150 

days 

or  5 

months   

.       60% 

13  days      

..      12% 

165 

days 

..     66% 

14  days     

....      13% 

ISO 

davs 

or  6 

months  .  . 

.     70% 

1  5  days  

.  ...      13% 

195 

days 

..     73% 

1  6  days  , 

....      H% 

210 

days 

or  7 

months 

75% 

17  days    

....      15% 

225 

davs 

..     7S% 

IS  days  

....      16% 

240 

davs 

or  S 

months   . 

..     so% 

19  days  

.  ...      16% 

255 

davs 

..      S3% 

20  days  

....      17% 

270 

days 

or  9 

months  .... 

...     S5% 

25  days        

.  ...      19% 

2S5 

days 

..    ss% 

30  days  or  1  month  , 

20% 

300 

days 

or  1< 

I)  months 

00% 

35  days       

...     23% 

315 

<lays 

,  ..     93% 

40  days     

25% 

330 

days 

or  1 

1  months 

.     95% 

45  days       

•    •     27% 

345 

days 

..     9S% 

50  days       

.      -      2S% 

360 

days 

or  1 

2  months 

.       100% 

insured)  becomes  a  coinsurer  on  the  loss,  in  the  ratio  which  his 
lack  of  insurance  bears  to  the  amount  he  should  have  carried. 

Illustration  of  SOC/0  coinsurance  clause: 

Case  1.     Value  of  building  and  contents $75,000 

Assured  should  carry  S0%  of  value  or 60,000 

Insurance  actually  carried  ,  . .  45,000 

Loss  by  (ire        10,000 

Paid  by  insurance  company,  75%  of  loss,  or.    .  7,500 

Assured  must  bear  25%  of  loss,  or 2,500 

Insurance  carried  was  only  75%  of  what  assured  should  have 
carried  to  comply  with  the  80  %  clause. 

Case  2.     Value  of  property $10,000 

Insurance  required S,000 

Insurance  carried 9,000 

Losses  up  to  $9,000 Paid  in  full 


BUSINESS  INSURANCE  99 

Case  3.     Value  of  property $10,000 

Insurance  required  8,000 

Insurance  carried  S,000 

Losses  exceeding  $S,000 

Face  of  policy,  $8,000,  is  paid. 

Case  4.     Value  of  property  ..      .  $10,000 

Insurance  required 8,000 

Insurance  carried  ...  5,000 

Losses  exceeding  $8,000 

Face  of  policy,  $5,000,  is  paid. 
Ixjsses  under  $8,000 

Paid  in  the  proportion  that  $5,000  bears  to  $8,000,  or 
f  of  the  loss. 

Problems 

1.  \Vh;it  premium  must  be  paid  on  a  policy  for  $3,7(10  at  $1.50  a  hundred? 

2.  A  house  worth  $12,000  is  insured  for  J  of  its  value  for  three  years  at  $2.35 
a  hundred.     How  much  is  the  premium? 

3.  An  agent  wrote  a  policy  of  $4,500  on  a  store  building  at  a,  rate  of  85  cents 
If  the  agent's  commission  >\as  15%,  what  was  the  amount  of  his  commission? 

4.  Find  the  amount  paid  by  the  insurance  company  under  the  80%  coinsur- 
ance clause  in  the  following: 

(«)  (&)  M  (d) 

Value  of  property  $50,000  $75,000  $100,000  $200,000 

Insuiance  carried   40,000     00,000       80,000       80,000 

Lossbyhrc     J  0,000     45,000       40,000       40,000 

Paid  by  insurance  company       .. . 

5.  You  are  presented  the  following  tornado  insurance  plan  and  arc  asked  to 
select  one  of  the  four  policies  and  to  decide  upon  whether  to  insure  for  one  or 
three  years.     In  your  opinion,  what  policy  should  be  taken  and  for  how  long 
a  term?     The  sound  value  of  the  property  to  be  insured  is  $1,242,000. 


(^insurance 
(I)   None 
(2)    50% 
(3)    80% 
(4)    90% 

Amount  of     ' 
Insurance 
$    200,000 
021,000 
993,600 
1.117.800 

One-  Year                          Three-  Year 
Rate          Premium           /fate          Premium 
20                                         50 

.102           

255 

.0749         .    .. 

.  1872 

.0678         

1695         

Compute  the  premiums  at  the  one-year  rate  and  at  the  three-year  rate. 
Find  the  average  yearly  premium  on  each  policy  at  the  three-year  rate,  and 
make  comparisons  in  order  to  determine  which  policy  to  accept. 

6.  A  one-year  policy  on  a  dwelling  was  dated  June  5.  The  premium  was 
$42.50.  On  October  1,  the  policy  was  canceled  at  the  request  of  the  insured. 
Find  the  amount  of  return  premium. 

Use  and  occupancy  insurance.  This  kind  of  insurance  is  pro- 
tection against  loss  due  to  interruption  of  business  by  fire  or 
tornado.  It  is  insurance  against  a  loss  that  is  suffered  on  account 
of  destruction  of  the  property. 


100  BUSINESS  INSURANCE 

The  insurance  recovery  or  indemnity  is  the  profit  that  would 
have  been  made  if  business  had  not  been  interrupted  and,  in 
addition,  the  total  of  expenses  that  must  continue  during  suspen- 
sion of  business.  A  business  that  is  not  profitable  may  be  so 
insured  in  order  to  recover  the  continuing  expenses. 

Generally  speaking,  use  and  occupancy  insurance  insures  gross 
profits  plus  the  salaries  of  key  employees  kept  on  the  payroll 
account.  The  policy  excepts  payroll  (other  than  that  of  the  key 
employees),  heat,  light,  power,  and  expenses  of  maintaining  prop- 
erties not  destroyed  (such  as  taxes,  depreciation,  and  maintenance 
thereof).  These  items  can  be  picked  up  by  analyzing  the  running 
expense  accounts.  In  no  event  does  the  policy  pay  expenses 
required  to  be  insured  unless  it  is  proved  that  they  continue  after 
the  fire. 

Coinsurance  clauses  are  also  applicable  in  use  and  occupancy 
insurance. 

A  simple  procedure  to  arrive  at  use  and  occupancy  value  for 
the  past  twelve  months  is  as  follows: 

Total  Sales 

Deduct: 

Cost  of  Merchandise 

(Opening  Inventory  +  Purchases  —  Closing 

Inventory)     

Ordinary  Labor  Payroll ... 

Light,  Heat,  and  Power .    .  .    . 

Total  deductions 

Actual  100%  use  and  occupancy  value  for  the  period   


The  foregoing  procedure  is  predicated  on  the  assumption  that 
all  expenses  other  than  ordinary  payroll  and  light,  heat,  and  power 
will  continue  at  the  same  cost  as  if  the  business  were  operating. 

A  more  exact  method  is  one  considered  in  the  light  of  a  problem 
in  arithmetic  or  algebra,  as  follows: 

Let  x  =  Use  and  Occupancy  Insurable  Interest  Each  Day 

a  =  Expenses  That  Do  Not  Continue  During  Suspension  of  Business 

6  =  Selling  Price  of  Merchandise 

c  =  Cost  of  Merchandise 

d  —  Number  of  Working  Days  in  the  Month 

Then: 


Example 
The  expenses  of  a  business  for  a  given  month  were  determined  as  follows: 


BUSINESS  INSURANCE  101 

Part  of  Expense     Part  of  Expense 
That  Must  That  Will  Xot 

Continue  During    Continue  During 
Item  Total         Suspension  Suspension 

Payroll $45,000 

Salaries  and  Wages  of  Key  Em- 
ployees Who  Must  Be  Retained.  $20,000 
Salaries  and  Wages  of  Employees 
Not  Retained         ...            *    ..  $25,000 

Power .          750  —  750 

Heat  and  Light  525  225  300 

Leasehold        .  .       1,200  1,200 

Advertising  1 ,725  725  1 ,000 

Taxes  ....  .  950  050 

Insurance  .  1,375  500  S75 

Interest       ...  .  .          525  525 

Other  Expenses  1,950  S75  1,075 

#54,000          $25,000  $29,000 

Find  the  estimated  amount  of  insurance  to  he  carried  for  each  day  of  the 
month  if  sales  are  estimated  to  be  $1  SI  ,500  and  cost  of  merchandise  sold  $109,500. 
Average  number  of  working  days  each  month  is  25. 

Solution 

b  -  c  -  a  _     181,500  -  109,500  -  29,000  _ 

d          ~X  25  -1,7-0 

Therefore,  on  the  basis  of  estimates,  $1,720  is  the  amount  of  insurance  to  be 
carried  for  each  day  in  the  month. 

The  same  result  may  be  obtained  in  the  following  manner,  using  the  esti- 
mates given: 

Sales  for  the  Month  $1  SI  ,500 

Less:  Cost  of  Sales 109,500 

dross  Profit 72,000 

Total  Expenses .  54,000 

Net  Profit 1S,000 

Add:  Expenses  That  Must  Continue  During  Suspension  25,000 

Use  and  Occupancy  Value  for  the  Month  .  $  43,000 

43,000  4-  25  =  1,720  ~~ ' 

Problems 

1.  The  gross  profit  of  a  business  was  $200,000  after  charging  raw  materials 
and  payroll  into  manufacturing  cost,  but  excluding  light,  heat,  and  power. 
Continuing  payroll  of  key  men  was  fixed  at  $20,000.     If  the  policy  contained 
the  80%  clause,  what  was  the  required  amount  of  insurance? 

2.  An  audit  of  the  expense  accounts  of  the  company  insured  in  Problem  I 
showed  that  items  that  would  not  have  to  be  continued  after  the  fire  totaled 
$60,000.     What  amount  would  be  collectible  for  a  twelve-month  period,  other 
facts  being  as  stated  in  Problem  1? 

3.  Assume  that  it  takes  15  months  to  rebuild  the  plant.     How  much  insur- 
ance would  be  collectible? 


102  BUSINESS  INSURANCE 

4.  If  a  manufacturer  on  a  Sept.  30  fiscal-year  basis  had  a  fire  on  April  1 
and  it  is  shown  by  previous  experience  that  the  following  six  months  are  the 
most  profitable — in  fact,  that  66$%  of  the  net  earnings  are  made  in  that  period — 
would  the  adjustment  take  this  into  consideration,  or  would  it  be  made  on  an 
average  for  the  year? 

6.  If  the  conditions  in  Problem  4  were  reversed,  what  earnings  would  the 
adjustment  reflect? 

6.  Compute  the  use  and  occupancy  value  from  the  following  data:  Beginning 
inventory,  $  1 70,4X2.00;  ending  inventory,  $171 ,72 1 .77 ,  manufacturing — including 
raw  materials,  labor,  light,  heat,  and  power,  maintenance,  depreciation,  adminis- 
tration, insurance,  tuxes,  interest,  advertising,  and  all  other  expenses,  $3,409,- 
058.42.  Fixed  charges  that  arc  included  in  the  foregoing  and  that  are  expected 
to  continue  are:  administrative  salaries,  $35,200;  interest,  $4,X(K);  taxes,  89,901 .22; 
dues  and  pledges,  $0,150;  credit  information,  $235,  insurance,  $7, 91 X. 49;  salaries 
of  office,  supervisors,  and  foremen  that  will  have  to  be  retained,  $237,075; 
miscellaneous  expenses,  $42,395.02.  Sales  were  $3, 551, 70S. SI. 

Group  life  insurance.  While  this  type  of  insurance  is  a  part  of 
the  subject  of  life  insurance,  it  is  presented  in  this  chapter  because 
it  is  a  common  form  of  business  insurance.  The  principles  of  life 
insurance  are  presented  in  another  chapter. 

Group  life  insurance  affords  employees  of  a  business  with 
ordinary  life  insurance  at  low  cost  so  long  as  they  are  employed  by 
the  particular  employer,  as  the  employer  pays  a  part  of  the  pre- 
mium. The  operation  of  this  type  of  insurance  is  best  explained 
by  an  example  of  an  actual  plan. 

Group  Life  Plan 

1.  Eligibility.     The  following  plan  of  group  life  insurance  is 
offered  to  all  present  employees  of  the  company  who  will  have 
completed  six  months  or  more  of  continuous  service  on  November 
11,  19 — ,  and  to  all  new  employees  after  they  have  been  with  the 
company  for  six  months, 

2.  Amounts  of  insurance.     The  amount  of  insurance  available 
to  each  employee  under  age  65,  nearest  birthday,  will  be  based  on 
annual  earnings  as  follows: 

Class  Annual  Earnings  Life  Insurance 

1.  Less  than  $1 ,200 \ $1,000 

2.  $1 ,200  but  loss  than  $2,200 1 ,500 

3.  $2,200,  but  less  than  $2,800  2,000 

4.  $2,SOO,  but  less  than  $3,200  2,500 

5.  $3,200,  but  less  than  $3,800 3,000 

6.  $3,800,  but  less  than  $4,200 3,500 

7.  $4,200,  but  less  than  $4,800  .  ...  4,000 

8.  $4,800,  but  less  than  $5,200..    .  ...  4,500 

9.  $5,200  and  over 5,000 


BUSINESS  INSURANCE  103 

3.  Cost  of  insurance.  The  monthly  cost  of  the  insurance  will 
be  based  on  the  employee's  insurance  age  on  each  anniversary  date 
of  the  plan,  as  shown  in  the  following  schedule : 

Em  ploycc  's  Mon  thly 

Attained  Age  on  Policy  Contribution  per 

Anniversary  Each  Year  $1,000  of  Insurance 

Age  44  and  under     $0 . 70 

Ages  45  to  54,  inclusive 1 . 00 

Ages  55  to  59,  inclusive 1 . 50 

Age  GO  and  over 1  .SO 

Problems 

1.  Employee  Y  is  42  years  of  age  and  his  earning  classification  is  Class  5. 
What  is  the  monthly  deduction  for  his  insurance? 

2.  If  Y  were  14  years  older,  what  would  be  the  monthly  deduction? 

3.  B  is  40  years  of  age  and  earns  $3,000  a  year.     How  much  insurance  is 
available  to  him,  and  what  will  be  his  monthly  contribution? 

4.  Company  A"  insures  each  of  its  employees  for  $1,000.     Under  age  50  the 
cost  to  the  employee  is  60  cents  a  month;  at  age  50  or  over,  the  cost  is  $1.00  a 
month.     There  are  54  employees,  classified  as  follows: 

Age  Number 

18 .                      1 

22     .                                   .  .    .                             6 

25     .  10 

29          ....  4 

30 7 

45           .  ...    12 

47            .                                  .  X 

52                ..  ...     2 

56       .  .                3 

5S        i 

What  is  the  amount  of  the  monthly  payroll  deduction? 

5.  The  manual  shoWvS  the  cost  of  group  insurance  on  a  monthly  basis  to  be 
is  follows: 

Age  Premium 

18  ..                   .    .                                                           .     $     51 

22  ...               53 

25  .         54 

29  .      .  .    .  .  55 

30  .          .55 

45  ...  .    .          .80 

47  90 

52 1  26 

56 1.71 

58  • 2.00 

With  the  number  in  each  age  group  being  that  given  in  Problem  4,  what  is 
the  amount  of  insurance  premium  that  is  borne  by  Company  XI 


104  BUSINESS  INSURANCE 

Health  insurance.  Some  plans  are  contributory  and  others 
non-contributory.  In  either  case,  the  benefits  are  much  the  same; 
but  in  contributory  plans  the  employee  pays  a  part  of  the  cost  in 
the  form  of  a  monthly  premium  deducted  from  wages,  while  in  the 
non-contributory  plans  the  cost  is  borne  by  the  employer.  Few 
businesses  have  their  own  insurance  departments,  most  of  the 
plans  being  handled  by  insurance  companies  under  a  group  plan. 

Incapacities  include  sickness  and  non-occupational  accidents 
(occupational  accidents  being  covered  by  Workmen's  Compensa- 
tion Insurance),  but  the  employer  usually  reserves  the  right  to 
withhold  benefits  if  the  incapacity  is  the  result  of  the  employee's 
misconduct  or  negligence. 

The  following  examples  are  illustrative  of  the  many  ways  in 
which  the  factor  of  service  is  employed  to  favor  the  veteran 

worker. 

Example  1 

Amount  and  Duration  of 
Length  of  Service  Disability  Itenejits 

Under  2  years Such  practice  as  the  company  may  establish 

2  but  less  than  5  years  .    .            .      Full  pay  4  weeks,  half-pay  9  weeks 
f>  but  less  than  10  years             .          Full  pay  13  weeks,  half-pay  13  weeks 
10  years  and  over Full  pay  13  weeks,  half-pay  39  weeks 

Example  2 

Amount  and  Duration  of 

Length  of  Serriee  Disability  Benefits 

1  but  less  than  10  years  50%  of  wages 

10  but  less  than  30  years  75%  of  wages 

30  years  and  over 100%  of  wages 

Maximum:  26  weeks  in  3  years 

Example  3 

Amount  and  Duration  of 

Length  of  Xermce  Disability  Benefits 

6  months  but  less  than  1  year       35%  of  wages;  maximum:  $14.00 

per  week,  for  0  weeks 

1  but  less  than  2  years  .   50'  'v  of  wages;  maximum:  $20.00 

per  week,  for  13  weeks 

2  but  less  than  3  years. .  00%  of  wages;  maximum:  $24.00 

per  week,  for  13  weeks 

3  but  less  than  4  years 70%  of  wages;  maximum:  $28.00 

per  week,  for  26  weeks 

4  years  and  over 75%  of  wages;  maximum:  $30.00 

per  week  for  26  weeks. 

Problems 

1.  A  was  insured  under  the  plan  in  Example  1.  He  was  employed  for  3  years 
and  became  incapacitated  for  a  period  of  6  weeks.  His  average  weekly  wage 
was  $35.80.  What  amount  of  disability  benefit,  was  he  entitled  to  receive? 


BUSINESS  INSURANCE  105 

2.  B  was  insured  under  the  plan  in  Example  2.     He  had  been  with  the  same 
employer  for  12  years.     Two  years  ago  he  drew  compensation  for  8  weeks,  and 
last  year  for  12  weeks.     This  year  he  was  again  incapacitated  for  a  period  of 
8  weeks.     If  his  average  weekly  wage  was  $45.00,  what  amount  of  disability 
benefit  was  he  entitled  to  this  year? 

3.  C  was  employed  by  an  employer  using  the  plan  in  Example  3,  and  had 
worked  for  this  employer  for  a  period  of  6  years.     He  became  incapacitated 
when  receiving  a  weekly  salary  of  $60.00,  and  was  unemployed  for  10  weeks. 
What  was  the  amount  of  compensation  paid? 

Workmen's  compensation  insurance.  This  type  of  insurance 
is  financial  protection  against  loss  of  time  for  the  wage-earning 
group,  resulting  from  accident  and  occupational  sickness  while  on 
duty.  The  cost  is  levied  on  the  employer  in  the  form  of  a  premium 
on  the  payroll  classified  according  to  the  hazard  of  occupation.  A 
few  states  have  their  own  Workmen's  Compensation  Insurance 
Departments,  but  in  most  states  the  insurance  is  carried  by  the 
insurance  companies  specializing  in  this  type  of  insurance,  gener- 
ally referred  to  as  casualty  insurance  companies. 

Problems 

Find  the  cost  of  workmen's  compensation  insurance  on  payrolls  divided  into 
four  classifications  with  respective  rates  as  follows: 

1.  $239,530.39  (m  .(HI  per  0 

75,535.62  ©  .519  per  C 

241,327.85  ©  .081  per  C 

99,791.48  ©  .586  per  C 

2.  $272,584.07  ®  .611  per  C 

91,856.68  ©  .5 19  per  C 
292,258.87  ©  .081  per  C 
148,735.42  ©  .586  per  C 

3.  $254,248.83  ©  .581  per  C 

79,950.31  ©  .548  per  C 
272,368.08  ©  .085  per  C 
105,553.36  @  .564  per  C 

4.  A  deposit  of  $100.00  was  made  on  a  public  liability  policy.     The  payroll 
audit  was  as  follows: 

$381,839.77  ©  .052  per  C 
294,212.15  ©  .026  per  C 
138,631.05  ©  .026  per  C 

What  amount  of  additional  premium  was  due  on  completion  of  the  payroll 
audit? 


CHAPTER  11 
Payroll  Records  and  Procedure 

Requirements.  The  term  payroll  records  has  gained  new 
significance  since  enactment  of  the  Social  Security  Act  and 
more  recently  the  Current  Tax  Payment  Act  of  1943.  Formerly 
each  business  handled  its  payroll  system  in  accordance  with  its 
own  particular  needs.  Now  payroll  systems  are  becoming  more  or 
less  standardized  in  so  far  as  certain  information  must  be  provided 
in  order  to  meet  the  tax  requirements. 

Requirements  at  the  time  of  wage  payments  are  that  the 
employer  must  deduct  the  taxes,  both  Federal  Old  Age  Benefit  Tax 
and  Withholding  Tax.  Tax  legislation  has  not  dictated  the  form 
of  records  to  be  kept,  but  regulations  have  stipulated  that  certain 
information  must  be  available,  and  that  a  statement  shall  be  fur- 
nished the  employee  on  or  before  January  31  of  the  succeeding 
calendar  year,  or,  on  the  day  on  which  the  last  payment  of  wages 
is  made  where  employment  is  terminated  before  the  close  of  the 
calendar  year.  Records  needed  are  best  determined  from  the 
reports  required. 

For  operational  purposes  many  employers  give  a  pay  statement 
with  each  wage  payment.  The  pay  detail  can  be  shown  on  a  stub 
attached  to  the  pay  check,  on  a  duplicate  of  the  pay  check,  on  the 
cash  pay  envelope,  or  on  a  separate  slip. 

Payroll  procedure.  Payroll  procedure  involves,  first  of  all,  the 
production  of  the  time  .card,  which  the  individual  employee  either 
fills  out  or  else  inserts  in  the  time  recorder  at  stated  times  and 
which  therefore  contains  the  basic  information  for  other  records. 

The  following  forms  contain  information  transferred  from  the 
time  card:  the  payroll  summary  sheet,  the  pay  check  or  the  pay 
envelope  (if  a  pay  envelope  is  used,  a  pay  receipt  is  also  required), 
and  the  individual  employee's  historical  earning  record. 

Tjmebooks.  Another  method  that  is  still  employed  to  quite 
an  extent  entails  the  work  of  timekeepers  who  keep  time  books. 
The  pencil  or  pen  records  that  these  timekeepers  turn  in  show  that 
individual  employees  work  a  certain  number  of  hours  and  fractions 
of  hours  on  particular  work. 

Time-clock  cards.  Time-clock  cards  provide  a  written  record 
of  the  time  each  employee  is  on  duty.  The  employee's  time  card 

107 


108 


PAYROLL  RECORDS  AND  PROCEDURE 


may  be  either  a  job  ticket  covering  a  single  job,  or  a  daily  time 
report  recording  all  jobs  worked  upon,  during  the  day.  Attendance 
records  or  the  In-and-Out  clock  cards  are  the  controls  on  total 
time.  A  time-clock  card  must  be  prepared  for  each  individual 
employee  for  each  day  or  each  pay  period,  according  to  the  system 


•ifoppinif 
time,    anc 
when  coi 
work  reco 
number  oi 
At  the  er 
elapsed   ti 
from  the 
and  check( 
on  the  att 

NO.    138                    PAY 
NAME     Ed  Wolper-3 

ENDING    8/28/46 

DEDUCTIONS 

r  o  A  B  .'J 

INC    TAX             2  Of 

imprinted  with  tl 
the  pay  period  d 
number,   employe 
and  possibly  othei 
ing  data.     It  show 
identification  of  tl 
the  employee,   sta 

ie  day  or 
ate,  clock 
e's   name, 
"  identify- 
s  the  date, 
ie  job  and 
rting  and 

"j^to 

2/- 

TOTAL  t       4  ^ 

12— 

1 

»755 

2759 

*7s? 

5749 

2 

5803 

3 

4 

21130 

5 

a  12oi 

*12°3 

51202 

£  12oo 

No. 

NAME 

DAILY  COST  CA 

138                            R; 
I    Ed  Wolper 

RD 

kTF     /  '*• 

6 

s!228 

21224 

*1229 

51227 

21226 

7 

8 

TIME  IMPRINTS 

ii'-Mrn 

QUANTITY 

JOB  NO 

COST 

9 

^ 

10 

*432 

2435 

*432 

5438 

£433 

CO 

11 

AUG  23    164 

4- 

u. 

2 

27S- 

+* 

12 

AUG  23    160 

CO 

13 

AUG  23    160 

1 

to 

+ 

+31 

ff 

14 

AUG  23    15  1 

IS 

16 

AUG  23    149 

/ 

u. 

<. 

TO, 

ff 

?JM 

g 

7* 

ft 

Jf 

ef 

AUG  23    140 

to 



TO 
OTAL  DCC 
Nt 

AUG.  23    139 

'3 

u. 

t 

S9, 

/O 

UCTK 

AUG  23    126 

to 

PAY 

AUG  23    120 

7 

*• 

+ 

t// 

77 

:ime,  rate,  elapsed 
I    amount    earned 
npleted.     Piece- 
rds  also  show  the 
f  pieces  produced, 
id  of  the  day  the 
Line    is    computed 
clock  registration 
ed  with  that  shown  j 
endance  records. 

AUG  23    113 

CO 

AUG  23    112 

<* 

u. 

,. 

7^2 

/  32 

AUG.  23    100 

to 

AUG  23    100 

+ 

«*. 

j 

AUG  23 

96 

v> 

AUG  23 

95 

7 

u. 

,-r 

216 

77 

AUG  23 

88 

w 

AUG  23 

87 

7 

u. 

<* 

,17 

77 

AUG  23 

80 

v> 

TOTALS 

72 

792 

DATE  r/**     FOREMAN  0  K&&*~~~/ 

i  and  Out  Clock  Card  and  Daily  Cost  Card 

The  illustration  is  that  of  the  weekly  In-and-Out  clock  card 
from  which  the  payroll  is  prepared  and  the  daily  cost  card  used 
for  cost  accounting  purposes.  The  In-and-Out  clock  card  illus- 
trated shows  the  exact  minute  of  entering  and  leaving.  Some 
clocks  register  this  time  in  tenths  of  an  hour  instead  of  the  exact 
time.  The  daily  cost  card  shows  hours  and  tenths,  beginning  at 
the  bottom  and  reading  toward  the  tog,  so  arranged  to  facilitate 
computation  of  elapsed .timA.  The  closing  hour,  16.4,  is  24  minutes 
past  4  o'clock. 


PAYROLL  RECORDS  AND  PROCEDURE  109 

The  In-and-Out  card  shows  8  hours'  elapsed  time  on  Monday, 
the  factory  hours  being  from  8  A.  M.  to  12  M.,  and  12:30  P.  M.  to 
4:30  P.  M.  Of  course,  it  is  impossible  to  check  in  and  out  on  the 
specified  hour,  and  a  certain  tolerance  is  allowed.  Different  com- 
panies have  varying  rules  regarding  tardy  registrations.  Wage 
and  Hour  inspectors  object  to  too  early  registration,  and  more 
than  15  or  20  minutes  early  is  likely  to  be  counted  as  overtime. 

The  daily  cost  card  shows  7.2  hours'  productive  time;  therefore, 
the  difference  of  .8  of  an  hour  is  nonproductive  time.  A  recon- 
ciliation can  be  made  showing  where  the  .8  of  an  hour  was  not 
utilized  on  productive  work. 

.  1  Between  Jobs  837  and  266 

.  1  Between  Jobs  266  and  490 

.  1  Between  Jobs  722  and  61 1 

.  1  At  noon  (Starting  time  being  12,6  instead  of  12,5.) 

.  1  Between  Jobs  598  and  701 

.2  Between  Jobs  701  and  431 

.  1  At  close  of  day  (Finishing  time  being  16.4  instead  of  16.5.) 

.8  Total  lost  or  nonproductive  time 

Deductions.  Fixed  or  standard  deductions,  such  as  group 
insurance,  employees'  benefit  association  dues,  hospital  service 
dues,  and  so  forth,  can  be  entered  at  the  time  the  caid  is  made  up. 
Federal  Old  Age  Benefit  Tax  and  Withholding  Tax  cannot  be 
entered  until  earnings  are  computed.  ^ 

At  the  present  time,  F.  0.  A.  B.  Tax  is  t%  of  earnings.  A 
portion  of  the  Withholding  Tax  schedule  effective  January  1,  1946, 
that  for  a  weekly  payroll,  is  presented  for  use  with  the  problems. 

Withholding  exemptions.  For  income  tax  computations,  the 
personal  exemption  is  on  a  per  capita  basis;  therefore,  withholding 
exemptions  are  on  a  pcr-ciapita-basis  as  follows: 

A  single  person  is  allowed  one  exemption. 

Husband  and  wife  have  two  exemptions:  if  both  are  working,  either  spouse 

may  take  both  exemptions  or  each  may  take  one;  if  one  is  not  working,  the 

other  may  take  both  exemptions. 
One  exemption  may  be  taken  for  each  dependent  (a  person  whose  income  is 

less  than  3#00  a  year,  who  is  closely  related  to  the  taxpayer,  and  for  whom 

the  taxpayer  provides  more  than  one-half  the  support). 

The  number  of  exemptions  claimed  determines  the  proper 
column  to  be  used  in  the  wage-bracket  tables  in  determining  the 
tax  to  be  withheld. 

Employees'  names  on  time  cards  and  payrolls  are  marked  to 
indicate  the  number  of  withholding  exemptions  claimed.  Find 
the  employees'  earnings  in  the  two  columns  at  the  left;  where  this 
line  intersects  the  exemption  column,  the  amount  of  tax  to  be  with- 
held is  shown. 


If  the  payroll  period  with  respect  Co  an  employee  in  WEEKLY 


At  g* 

••**  I   tZ-Z 


And  me  number  of  wttMioldIn0  exemption*  claimed  t»— 


il'II 

12  ____ 

13  .  . 
$14  ____ 


$11. 

12 

13- 

15 


%  of,  1 

»~«  $0  $0 

$2.001  .10  o 

2.1Of  3Q  O 

2  3O  .50  O 

2.5O  ,7O|  O 


$0 
O 
O 


$0 

o 
o 
o 
o 


$0 
O 
O 

8 


if;:: 


1  2O 
1  30 


t20 
21 


... 

8::: 


$21 
$22 


170 
1.80 
20O 
220 
2.40 


.20 
.40 
.50 


$25 
26 
27 
28 


(29 
$30 


25O 
27O 
29O 
300 
320 


.70 
.90 
1  OO 
1  20 
140 


33 

$34 


$31 
$3? 
$33 
H4 
$35 


60 
5  70 
590 


340 
3  60 
370 
390 
4.10 


2  20 


.,„ 

.20 
.40 


$35 
f  36 
$37 


$40 


6  10 

62O 

t4O 
60 
680 


420 
4  4O 
4  60 
480 
4  90 


§4O 
60 
280 
290 
3  10 


1-30 


$4O 
$41 
f  42 

$43 
$44 


$41 
$42 
143 

$44 
$45 


5  1O 
5  3O 

5  40 


330 
3  40 
3  60 
380 
400 


1  40 
1  60 
1  8O 
200 
2.10 


X| 


$47 
$48 
$49 


$46 
$47 
$48 
$49 
$5O 


780 
8OO 
820 


23O 


.50 

.60 

.80 

1  OO 

1.2O 


1! 

$54 


$51 

$52 
$53 
$54 
$55 


18 


6  BO 
7OO 
720 

7  30 
7  50 


SCO 
6  2O 
5  30 
550 
5  TO 


§20 
30 

IPS 

380 


1.30 

1  bO 
1  7O 
1  80 
20O 


$55 
$56 
$57 

ill 


$56 
$57 
$58 
$59 
$60 


9VO 
90O 
10  1O 
10  3O 
1O5O 


$60 
$62 


(66 

$68 


$62 
$64 
$66 
168 
*/0 


1O7O 
11  1O 
11  5O 

Ulg 


770 
79O 
8  10 


580 

too 
70 
6  3O 
6.50 


400 
4  20 
4  40 
450 
4.70 


2  ?0 
2  40 
250 
~  TO 
2.90 


.40 
.50 
.70 

.38 


1 

2 
2  3O 
27O 


.50 
.80 


13 

(74 
(76 
$78 


12  GO 

13  OO 
13  40 

13  TO 

14  10 


1060 
11  OC 
11  4O 
11  TO 
12.1O 


1O.1C 


48O 
52O 

5  50 
590 
€20 


300 
330 
3  TO 
4.OO 
4.40 


1  20 

1  5O 
1.9O 

2  2O 
25O 


$«O 
$82 

(84 


14  5O 

138 

1560 
16  OO 


1050 
1O9O 
11  20 

J1 


50 


«•  - 


47O 

118 

57O 
6-10 


*    . 


290 
3  2O 
3  6O 


»1OO 
*105 
$UO. 
$115 
$120. 


!$105 

*110. 

$115. 

.'$130 

,$125. 


18  SO 
1950 

20  4O 

21  4O 


165O 
1750 

18  4O 

19  4O 

20  JO 


12  4O 
1330 
14.30 


290 
380 

SJ8 

6-30 


$;  t 
$1  . 
•  • 


Si  ft        $• 
:«,)  .     $- 


21  2O 

22  2O 

23  1O 

24  10 
2500 


1920 

20  2O 

21  1O 
2200 
23.00 


IS* 

£i§ 

2100 


ns 


18  __ 
1900 


17X» 


11  2O 

12  1O 

13  1O 

14  OO 
15-OO 


9-20 

10  10 

11  1O 

12  CO 
12^90 


is 


11? 

7JOL 


. 

$••« 
s.  •«) 

SirtO 

$:  w 


$20O  and  over. 


28  4O 
30  3O 
32  2O 


26  4Ol 

28  3Ol 
-yi  ?n! 


24  4O| 
26  30 
9*  **)' 

'Ki  >' 
- 


22.40 
24  30 

26  2O 
28  OO 
2990 


18  4O 

2O  3O 
22  1O 
24  .OO 

25.90 


16,40  14  4Oi 

18  2O  16  2O 

2O  1O  18  lO 

22.00  20.00 

23-90'  21  SO] 


1O3O 
12  2O 
14  1O 


12,  1C 

14.0C 

*5  ac 


1»  percent  off  tt»* 


over  S2OO 


72.801 


18-80!     16.8C 


PAYROLL  RECORDS  AND  PROCEDURE 


111 


Example 

Brown's  earnings  are  $29.00  for  the  week,  and  his  number  of  withholding 
exemptions  is  2.  What  is  the  amount  of  Withholding  Tax? 

Solution 

In  the  columns  at  the  left  find  the  bracket  $29.00  to  $30.00,  and  follow  across 
to  the  intersection  of  the  column  headed  "2."  The  tax  is  found  to  be  $1.40. 

Problems 

1-2.  Complete  the  following  weekly  time  cards.  Notice  that  the  daily 
recordings  are  made  across  the  card  instead  of  vertically,  as  in  the  illustration  on 
page  10S.  Make  the  extension  of  elapsed  time  at  the  extreme  right  of  the  card. 


1. 


PAY  ENDING     10/25 
Nt.    24 

NAME                 Garret  Knlgbt  -  3 

IN 

OUT 

IN 

OUT 

IN 

OUT 

M  7  57 

M1201 

M1259 

M4  05 

TUSOI 

TU1202 

TU1258 

TU402 

W  7  53 

W1202 

W1257 

W  3  59 

TH?59 

TH1201 

TH  1  01 

TH404 

FR75« 

FR1203 

FR1256 

FR402 

'  ' 

SA758 

SA1210 

FOA. 
WHD 
O    £" 

DEDUCTIONS 

r  •  , 

TAX 

MP<:      ^'J/«       ^       >UT   J'>    "/) 

Z~J  Q~    2S- 

TOTALS 

I            ' 

t 

TOTAL  DFt>iJCTlON«»  

TOTA 

L  * 

NET  PAY 

$ 

PAY 
NO.    25                         ENDING    Sept.   24 

NAME                     Alice  Vharton  -  1 

IN 

OUT 

IN 

OUT 

IN 

OUT 

M   8  2» 

M  1203 

M1259 

M  4   4C 

TU  8  25 

TU1205 

TU1287 

TU4  60 

W   8  28 

WH  M 

W12S5 

W4    46 

TH  832 

TH1201 

TH  1255 

TH4  55 

FR  8  17 

FRl2oz 

FR    1  01 

FR  449 

SA  830 

SA1210 

roj 

WHC 
INS 
TOT 

DEDUCTIONS 

i  a 

MRS                     •        -*~*  •  AMT 

TV 

TOTAL  PAY  
TOTAI  OfOUCT'GMt  

/(. 

M  * 

NET  MY 

ft 

112 


PAYROLL  RECORDS  AND  PROCEDURE 


3-4.  Complete  the  following  semi-monthly  time  cards.  A  section  of 
Withholding  Tax  schedule  for  semi-monthly  pay  periods  is  given  to  enable 
to  compute  the  tax: 


A  section  of  the 
you 


$54  to  $56 SI .  50 

56  to    58      .       . .     1  80 

68  to    60 2  20 


3. 


PAY 

NO.       16                        ENDING     Oct.    31 

NAME                    H«rry   Burr  -  2 

DATE 

IN 

OUT 

IN 

OUT 

IN 

OUT 

16 

^ 

M    7  32 

M1204 

M    12^9 

M    4  02 

*/> 

TU  749 

TUl2ot 

TU1253 

TU  4  03 

y» 

W    7«9 

W1204 

w!2». 

w  4  oo 

*/*> 

TM7  SB 

TH1203 

M12M 

T«4  04 

^ 

PR  8  oo 

r*12oo 

rnl23» 

r*  401 

% 

3A79* 

M 

«     105 

M 

& 

% 

M    7  3* 

Ml2ot 

M      lOO 

M    4<»* 

% 

TU  745 

T012C3 

TU  1259 

TU  4  04 

% 

w  7M 

w  12o. 

w  12  IT 

w  4o« 

% 

TH  7   54 

«12o< 

Wl24« 

Tn4  OX 

% 

rn  7  49 

r«  12oa 

rni2M 

fA4°l 

% 

SA  7  59 

W     1.3 

% 

% 

W7W 

M  12oi 

M    1257 

M    4  03 

M    4'« 

M     9  34 

FO/ 
WHC 
0 

DEDUCTIONS 

HRS                  «  .—  ^*r-      AMT 

TAX 

TOTAL  P 

TOTAL  otoucTior 

NET  PAY 

H 

c 

j^ 

TOT 

IL  f 

$ 

PAYROLL  RECORDS  AND  PROCEDURE 


113 


The  following  problem  contains  overtime,  which  is  to  be  computed  at 
time  and  one-half.  At  the  bottom  of  the  time  card,  enter  the  regular  hours  on 
the  first  line  and  the  overtime  hours  on  the  second  line.  You  will  notice  that 
f;he  rate  has  already  been  adjusted  to  one  and  one-half  times  the  regular  rate. 


PAY 
NO.       14                         ENDING       Sept.   30 

NAME                   Richard  Knight  -  2 

DATE 

IN 

OUT 

IN 

OUT 

IN 

OUT 

16 

FR  7  50 

FR1205 

FR1230 

FR  4  35 

X 

X 

x> 

M    7   58 

M1202 

M1229 

M  4    33 

>20 

TU  7  57 

TU1200 

TU1229 

TU4  30 

TU  4  59 

TU  7  02 

X 

W  7  49 

W1203 

W  1228 

W  4  38 

X 

TH  8  13 

TH1207 

TH1229 

TH  4  37 

% 

FR  749 

FR1201 

FR1230 

FR  4  39 

% 

x> 

,% 

M    7   59 

M  12  00 

M1230 

M  4    34 

X 

TU  8  00 

TU1202 

TU1230 

TU4  30 

^ 

W  7  58 

W1203 

W1228 

W  4    37 

% 

TH748 

TH1205 

TH1230 

TH4  38 

% 

FR  7  50 

FR1200 

FR  1232 

FR  4  22 

% 

rot 

WHC 
INS 
TOT 

SEDUCTIONS  

A 

HRS  9  — 

f// 



TAX 

TOTAL  PA 
TOTAL  DEDUCTION 
NIT  PAY 

>—  - 

ILt 

Payroll  sheets.  Whether  payment  is  made  by  cash  or  check, 
listings  of  payments  to  employees  are  made  on  payroll  sheets  for 
record  purposes,  and  the  hours  worked,  rate  of  pay,  gross  earnings, 
deductions,  and  net  pay  are  shown  for  each  employee. 

Piecework  system.  Quantity  of  work  produced  rather  than 
number  of  hours  worked  is  the  basis  of  earnings  under  a  piecework 
system.  Under  this  system  there  is  an  incentive  for  the  skilled 
worker  to  produce  more,  thereby  increasing  his  earnings.  The 
payroll  is  designed  to  record  the  number  of  pieces  produced  rather 
than  the  number  of  hours  worked;  hence,  no  provision  need  be 
made  for  overtime. 


114 


PAYROLL  RECORDS  AND  PROCEDURE 


1 

if 


gl, 


o> 

3 
o 


0 


a  I" 

Q>    O 

"E  ° 
S  ° 
o^ 
Oo 


c 


"8 


a 

I 


CQ 

•< 
d 


Thu 


v 

Exemption 


Name 


fe|**1 


G^CJ 


^ 

.  0) 

d 
PC 


0 


c;S 


O 

•£^«s 
£-S9 

<»?  CO 


PAYROLL  RECORDS  AND  PROCEDURE 


115 


£ 

03 
P, 


•o   « 

-g     % 

•S  2 


a; 


3  S 

O 


d 


'a  ^ 


00 
d^ 


pq 
6 


i 

H 


W 
o 

aj 


COrH 


!2  CO 


CSJ 


S 


(E 
'h 

^•r 


S 


116 


PAYROLL  RECORDS  AND  PROCEDURE 


O  T3 

£"0 


Ss"* 
-.2 

O  +•* 


<D  ' 

2 

o3 


^  o 

pt 

0)  ^ 

•*  2 

(D     >> 

s  s. 


.S3 

SO) 
O 

2  * 
03 .2 

"-P  x 
o5  ^ 

o  ^ 

73    fl 

S2 


So 

.s 

'5 


art  g- 


C5      QJ      CO 


"Id 


No. 


TIME  SH 
and 
PAYROLL  SU 


Q 


PQ 


«  s 

' 


H 


tl 


vo 


CO 


^  M 


H   K 


CO 


CM 


CM 


rH^ 


-P 
0-H 
O  6 
&  CO 

PQPn 
<H; 


to 


00 
IOLO 


CM  CM 
rH  rH 


$$ 


£* 


PAYROLL  RECORDS  AND  PROCEDURE 


117 


tt 

c 
^c 

s 

o 


c 
O 


CO 
OT 


ss 
•^ 


o 
H 


I- 


JM 

Z    X 


j^oh 
<OS^K 


S 


00 


l«3 


*S? 
*>£ 


<* 
^01 


^14 


fclfclsl 


(Slw-V 


ci 


118 


PAYROLL  RECORDS  AND  PROCEDURE 


Pay  checks.  After  the  payroll  has  been  figured,  the  next  opera- 
tion is  the  writing  of  checks.  Two  types  of  payroll  checks  are 
illustrated,  the  stub  portions  on  this  page,  and  the  check  por- 
tions on  the  following  page. 


Form  A 


BOONE  COMPANY, 
BOONEVILLE,  MO. 


Statement  of  Employee's  Earnings  and  Payroll  Deductions 


RATE 

TIME  WORKhD 

AMOUNT 
EARNED 

OTHFR 
COMPEN- 
SATION 

TOTAL 
AMOUNT 
TAXABLE 

DEDUCTIONS 

Days 

Hours 

Standard 

Hours 

Tax 

Federal 
Old  AKC 
Tax 

Miscel 

1.00 

5 

40 

40 

40.00 

4.00 

44.00 

2.10 

.44 

1.51 

This  is  your  statement  of  earnings  and  receipt  for  deductions  as  required  by  law. 
Save  it  carefully  as  it  is  the  basis  of  any  claim  for  Unemployment  Insurance  or 
Old  Age  Pension. 


Form  A  is  the  typo  produced  on  a  bookkeeping  machine 
writing  from  the  time  cards  and  making  the  payroll  chock,  pay- 
roll summary  sheet,  and  earnings  record  in  a  single  operation. 


Form  B 


Pay-Roll  R«milUoc«  Voucher 


Employ**  S.  S. 
Acct  No  . 


341 


Herman  Shultz  -  3 

SEP     15     19 


Hours  nr\ 

Worked  Reg   fc*U    Overtime 


Amount  Earned 

90.00 

••*«*». 

Fed.  Old-Age  Tax 

.90 

Inc  Tax 

3.70 

Gr.  Ins 

1.00 

Misc 

5.30 

TXHIltuBlll. 

10.90 

•rttaeiMhtt 

79.10 

DETACH  AND  RETAIN  THIS  VOUCHER 
Marter  Industries,  Inc. 


PAYROLL  RECORDS  AND  PROCEDURE 


119 


Form  A  (Continued) 


BOONE  COMPANY, 
BOONEVILLE,  MO. 


PAYROLL  CHECK 


i 

I     DATE 


AUG19 


CHECK  NO 


56793 


PAY  TO  THE  ORDER  OF 

D.C.  DANBURY       364  09  1898  EXACTLY  $     39.95 

Not  Good  for  Over  Two  Hundred  Dollar* 


\          FIRST  NATIONAL  BANK 
|  BOONEVILLE,  MO. 

89-11 


Thit  chick  not  valid  ttnle$*  prettnted  for  payment  within  60  day*  from  date  ofiuue. 


Treasurer 


Form  B  (Continued) 


:-~ •  •  •  us-" '    *  .  • !~  ' :..  It  :  •:  'i.    • '    "  :«:  .iTrrnSn'S'i?    .I:.::'  ui-  -jtii:!"  i  it     "  . ' 

DATE    SEP  15  19 

Master  Industries,  Inc.       No.  2317 
JlwnujaflwiviA  JloUonaf  JKan£  Lecenter,  Minn. 

Lecenter,  Minn. 

PAY      Seventy-nine  and  10/100 $  79.10 


TO  THE 

OF      Herman  Shultz  -  3 


PAY    CHECK 


Master  Industries,  Inc. 


BY. 


120 


PAYROLL  RECORDS  AND  PROCEDURE 


Form  B  is  the  type  of  check  produced  on  a  typewriter  from  data 
on  the  payroll  summary  sheet.  Three  operations  are  required; 
preparation  of  the  payroll  summary  sheet,  writing  of  checks, 
and,  finally,  posting  to  employees'  earning  records. 

Pay  envelopes  and  receipts.  Some  factories  pay  their  employ- 
ees by  cash  instead  of  by  check.  In  such  cases,  pay  envelopes  and 
pay  receipts  are  used. 


Employ**'* 
Nam* 


PAY-ROLL  RECEIPT 

Lee  Spence    —  2 


Employ**'*       .Q1  c7flnet  Company        . 
S  S.  Acd.  N«.191-57-8055  rwi  No.l^f 


R.<*iv.d  From    Ace  Sales  Company 

Earning,  to Au«U9t  20 19 

Hour. 

Worked.  Regular *U Ov*rtim* 


Amount  Earned 

Commission 


$  21.00 


$  15.00 


Total 


36.00 


DEDUCTIONS: 
F.d.ral  Old-Ag* 
Ann'ty  Tax  @  ^%  *  - 


Inc.  T«  _ 


$  2.60 


Group  Int.. 


$     .50 


Total  Deduction*  *      3*46 

N*k  Amount  H*r*with         $    3g>54 


The  flap,  printed  with  the  remittance  data,  when  signed  by 
the  employee  becomes  a  receipt  for  wages.  The  face  of  the  enve- 
lope, printed  the  same  as  the  flap,  contains  a  carbon  copy  of  the 
data  and  is  the  employee's  permanent  record. 

Coin  sheet  and  currency  memorandum.  Where  employees  are 
paid  in  cash,  each  employee  receiving  an  envelope  containing  the 
exact  amount  of  his  net  earnings,  it  is  necessary  to  prepare  a 
currency  and  coin  sheet  in  order  to  have  the  correct  number  of 
units  of  each  denomination. 


PAYROLL  RECORDS  AND  PROCEDURE 


121 


^ 
J/ 


Name 


Curiency  and  Coin  Sheet 


Net 
Earnings 


Hi 


21 


2/3 


S+ 


Currency 


20 


10 


Coin 


50 


25 


10 


05 


01 


A  currency  memorandum, 
made  up  from  the  foregoing,  is 
taken  to  the  bank  to  enable 
the  paying  teller  to  make  up 
the  amount  of  money  required 
in  different  denominations. 


PAY  ROLL 

Pitt                                                 " 

FOR        _.                                                    -_ 

Currency 

•tUAM 

"«i^""'" 

«v. 

?"•                             ^ 

/<r^ 

00 

I0*i                             * 

/o 

o  o 

JLS 

5  •                             *^ 

JC 

0  0 

i». 

fiil^^ 

so 

O  * 

H.1v^                                     -/ 

X 

oc 

X 

/ 

CO 

nimM                      4r 

f* 

Nickel.                           V^ 

/o 

Pennle*                           </ 

*/ 

TOTAL 

X/J 

r*/ 

Problems 

Rule  currency  and  coin  sheets  and  complete  them  for  Problems  1,  2,  and 
3,  pages  114,  115,  and  116. 


CHAPTER/12/ 
Average 

Simple  average.  The  simple  average  of  a  group  of  items  is 
determined  by  adding  the  items  to  be  averaged  and  dividing  the 
sum  by  the  number  of  items  added. 

Example 

From  the  following  statistics,  find  the  average  rate  per  kilowatt  hour  for 
electrical  energy: 

New  England  States  2  «SSf* 

South  Atlantic  States  2  77^ 

Atlantic  States  2   I  Of4 

North  Central  States  .                                              1   88? 

Pacific  Northwest  .    .                                                181^ 

Solution 

2.88  +  2.77  +  2.19  +  1.88  -f  1.81  =  11.53 
11.53  -r-  5  =    2.306 

Explanation.  The  number  of  items  to  be  added  is  5,  and  the  sum  is  11.53$£. 
1 1.53  divided  by  5  equals  2.306,  therefore,  2.306^  is  the  average  rate  per  kilowatt 
hour. 

Problems 

1.  The  following  delivery  record  shows  the  number  of  deliveries  made  each 
day  by  the  live  trucks  of  the  delivery  department: 

TRUCKS 

DAY  No.  1  No.  2  No.  3  No.  4  No.  5        TOTAL       AVKUAUK 

Monday 242  320  271  141  243 

Tuesday     217  328  393  182  218 J  /  L 

Wednesday  ..    256  290  296  120  325        ,„.    ..  /.. 

Thursday     .      .  302  289  344  149  297 

Friday 293  306  301  216  218 

Saturday  .  .     317  365  423  227  303 

Total  ....  I ' 

Average.  .  .       «2.7/ 


(a)  Calculate  the  total  number  of  deliveries  made  by  each  truck,  and  the 
daily  averages  for  the  week  (vertical  columns). 

(6)  Calculate  the  total  number  of  deliveries  made  each  day,  and  the  average 
number  of  deliveries  per  truck  (horizontal  lines). 

123 


124  AVERAGE 

2.  The  monthly  output  of  motor  cars  and  trucks  for  one  year  was  as  follows: 

January     231,728 

February 323,796 

March 413,327 

April 410,104 

May 425,783 

June 396,796 

July 392,076 

August 461,298 

September 415,285 

October .        .  397,096 

November 256,936 

December       ...                    .    .  233,135 

Total ....  .        ..: 2% 

What  was  the  average  monthly  output  for  the  year? 

3.  The  sales  of  five  clerks  on  a  certain  day  were  as  follows: 

A $356  80 

It 438.90 

C 395  60 

1) 410.85 

E .  440.90 

Total        ...  

(a)  Find  the  average  sales. 

(6)  Which  clerks  sold  above  the  average? 

(c)  Which  clerks  sold  below  the  average? 

Moving  averages.  Moving  averages  are  a  series  of  simple 
averages  of  statistics  applicable  to  groups  of  an  equal  number 
of  time  units,  each  successive  group  excluding  the  data  for  the 
first  time  unit  of  the  preceding  group  and  including  the  data  for  the 
time  unit  immediately  following  those  of  the  preceding  group. 
For  example,  a  yearly  moving  average,  by  months,  may  begin  with 
an  initial  group  including  the  data  applicable  to  the  twelve  months 
of  1943.  The  next  group  would  omit  the  data  applicable  to  Janu- 
ary, 1943,  and  include  the  data  applicable  to  the  remaining 
eleven  months  of  1943  and  that  applicable  to  the  month  of  January, 
1944. 

Example 

The  labor  costs  in  a  certain  manufacturing  plant  for  the  first  six  months  of 
1943  were  as  follows: 

January $3,363. 17 

February 3,644. 15 

March 4,472.90 

April 3,209.20 

May 3,415.40 

June 4,152.05 


AVERAGE  125 

The  labor  costs  for  the  next  two  months  were: 

July  .............................................  $3,824.06 

August  ..........................................     4,015.25 

What  has  been  the  average  labor  cost  for  each  six  months  since  January  I, 
1943? 

Solution 

The  labor  cost  for  the  period  from  January  1  to  June  30  is  the  sum  of  the 
labor  costs  for  each  of  the  six  months,  or  $22,256.87.  The  average  for  the  period 
is  $22,256.87  -^  6,  or  $3,709.48. 

The  average  for  the  period  from  February  1  to  July  31  is  computed  as  follows: 

Total:  January  1  to  June  30  .......................   $22,256.87 

Deduct:  January  labor  cost  ........................       3,363  17 

$T8~893770 
Add:  July  labor  cost  ..............................       3>8?4_L()6 

122,717  76 
$22,717.76  -^  6  =  $3,786.29 

The  average  for  the  period  from  March  1  to  August  31  is  calculated  in  the 
same  manner: 

Total:  February  1  to  July  31  .......................   $22,717.76 

Deduct:  February  labor  cost  .......................       3,644  15 

$19,0731)7 
Add:  August  labor  cost  ..........................   _  4,015  25 


$23,088.86  -T-  6  =  $3,848.14  -  -____- 

Comparison  of  these  averages,  $3,709.48,  $3,786.29,  and  $3,848.14,  shows  an 
increase  for  each  period. 

In  permanent  records,  these  averages  should  be  tabulated. 

Moving  Increase  or 

194S                       Labor  Cost      Average  Decrease^ 

January-  June  ..............  $22,256  87     $3,709  48  $  .......... 

February-  July  .............     22,717  76      3,786  29  76.81 

March-August  .............     23,088  .  86      3,848  .14  61  .  85 

t  Indicate  decreases  by  means  of  daggers. 

Further  comparisons,  based  on  the  figures  of  prior  periods, 
may  be  made  in  succeeding  years.  A  column  may  be  annexed 
to  show  the  increase  or  decrease  of  the  average  of  each  six  months' 
period  compared  with  the  simple  average  for  the  preceding  year. 
Another  column  may  be  used  to  show  the  increase  or  decrease  in 
the  moving  average  for  the  current  six  months'  period  compared 
with  the  moving  average  for  the  same  period  of  the  preceding 
year. 


126  AVERAGE 

Problems 

1.  Below  are  stated  the  labor  costs,  for  the  succeeding  months,  of  the  com- 
pany in  the  preceding  example;  compute  the  moving  averages. 

September $4,275. 60 

October 3,981  28 

November     4,013  75 

December 4,010  80 

2.  Using  the  averages  obtained  in  Problem  1,  complete  the  tabular  record 
for  the  year  1943. 

3.  Find  the  simple  average  for  the  year  1943. 

Progressive  average.  The  method  of  progressive  average  is 
cumulative.  The  results  of  the  latest  period  are  added  to  the 
total  previously  computed,  and  the  amount  is  divided  by  the 
previous  divisor  plus  1. 

Example 

Department  A  sales  were:  January,  $5,364.00;  February,  $4,872.00;  March, 
$5,024.00.  Department  B  sales  were:  January,  $2,561.00;  February,  $2,325.00; 
March,  $2,753.00.  Find  the  progressive  monthly  averages. 

Solution 
SALES  RECORD 

Dept.       Jan.          Feb.         Total         Aver.      March        Total         Aver.      April 
A        5,364       4,872        10,236       5,118       5,024        15,260       5,087 
B         2,561        2,325         4,886        2,443        2,753          7,639        2,546       etc. 

Explanation.  Department  A  sales  for  January  and  February  total  $10,236.00. 
$10,236.00  -h  2  =  $5,118.00,  the  average  for  the  two  months.  $10,236.00  + 
$5,024.00  =»  $15,260.00,  the  total  sales  for  the  three  months.  $15,260.00  -5-  3 
=  $5,087.00,  the  average  for  the  three  months.  The  record  for  the  year  would 
be  completed  in  this  manner. 

The  totals  and  averages  of  Department  B  are  computed  in  the  same  way. 

Problem 

Using  the  above  record  and  the  following  information,  complete  the  record 
('or  the  six  months'  period. 

Department  A  sales: 

April $5,986.00 

May 6,125.00 

June 6,398.00 

Department  B  sales: 

April 2,482.00 

May 2,593.00 

June 2,715.00 

Periodic  average.  Periodic  average  is  simple  average  applied 
for  several  periods  to  statistics  applicable  to  the  same  unit  of  time. 


AVERAGE  1*7 

It  may  be  used  to  show  a  variation  in  expenses,  earnings,  sales, 
and  so  forth. 

Example 
EXPENSES 

Month              1943          1942          1941          1940  Total  Average 

January $478  60  $392  cS5  $429  65  $356  00  $1,657. 10  $414  28 

February 462  37  529  83  531 .33       535  35  2,058.88  514  72 

March 347  92  629  89  432  45      567  89  1,978  15  494  54 

Explanation.  The  expenses  for  January  for  the  four  years  are  totaled;  the 
total,  $1,657.10,  divided  by  4,  the  number  of  years  shown,  equals  $414.28,  the 
average  monthly  expense  for  January.  The  other  averages  are  calculated  in 
the  same  manner. 

Problem 

The  output  of  a  factory  for  the  first  quarter  of  the  years  1943,  1942,  1941, 
and  1940  is  shown  in  the  following  table: 

Month                1943          1942  1941          1,940  Total         Average 

January   231,728  238,908  309,544  240,592     

February 323,796  304,735  364,180  283,577 

March      ...      .  413,327  394,513  434,470  374,425    


Compute  the  periodic  average. 

Weighted  average.  Weighted  averages  take  into  account  not 
merely  the  number  of  units  to  be  averaged,  but  also  the  value  of 
each  unit. 

The  average-price  method  of  pricing  requisitions  in  cost 
accounting  is  illustrated  in  the  following  example. 

Example 

A  stock  record  shows  the  following  receipts: 

4,800  Ibs.  ©  20^ 
3,000  Ibs.  @  18£ 
4,000  Ibs.  @2\i 

What  is  the  average  price  per  pound  for  the  month? 

Solution 

4,800  Ibs.  @  20^  =  $    960  00 

3,000  Ibs.  @  18£  =       540.00 

4,000  Ibs.  @  21  £  =       840  00 

11^800  Ibs.  =  $2,340"  00 

2,340  -r  11,800  =  19.83,  or  19.83^  per  pound,  the  average  price. 

Example 

A  manufactured  product  is  composed  of  four  ingredients,  the  relative  pro- 
portions and  costs  per  pound  being  as  follows: 


128  AVERAGE 

Material  Pounds  Price  per  Pound 

A  I  *l  50 

B"..  .  3  75 

C  4  1  25 

D  2  2  00 

It  was  found  in  the  second  year  that  owing  to  price  fluctuations,  the  raw 
material  costs  had  increased  as  follows: 

Material  Per  Cent 

A  .  50 

B  .....  100 

C  10 

D  .  25 

What  was  the  average  per  cent  of  increase  in  the  cost  of  raw  material  com- 
posing the  finished  product? 

Solution 

Per  Cent 
Cost        Total 
Material 

A  

B     .. 

C  .    . 

D. 

$12.75  $4  50 

4.50  -I-  12.75  =  35.29%,  the  weighted  average  per  cent 

Problems 

1.  A  stock  card  shows  the  following  receipts: 

3,000  Ibs.  ©  2S^ 
2,000  Ibs.  <$  27ff 
1,500  Ibs.  (a)  29jS 
4,000  Ibs.  fe  30^ 

What  is  the  average  price  per  pound? 

2.  X  owns  the  following  securities: 

$3,000  Power  Corp.  Si's 
$1,000  Alabama  Company  6's 

10  shares  Northern  Power  Co., 

7%  Preferred  Stock,  Par  Value, 

$100.00  a  share 
$2,000  Union  Depot  Co.  5's 

What  is  the  average  rate  of  interest  earned  on  X's  investment,  assuming;  that 
the  securities  were  bought  at  par? 

3.  A  product  is  manufactured  from  the  following  materials  used  in  the  relative 
oroportions  given: 


Cost 

Total 

Price 

Increased 

Pounds 

per  Lb. 

Cost 

Increase 

Cost 

1 

$1.50 

$  1  50 

50 

$  .75 

3 

.75 

2  25 

100 

2.25 

4 

1.25 

5.00 

10 

.50 

2 

2.00 

4  00 

25 

1  00 

AVERAGE  129 


Material 
A  . 
B. 
C  . 
D 

If  the  raw  materials  used  advance  in  price  at  the  following  rates,  what  will 
be  the  average  per  cent  of  increase  in  the  cost  of  the  finished  product? 

A  ..  ..   25%     C  .  33i% 

B 30%    D 50% 


Pounds 
4 

Price 
per  Lb. 
30ff 

...      .        6 
.3 

.                           2 

25^ 

75^ 

CHAPTER  13 
Averaging  Dates  of  Invoices 

Definition.  Averaging  dates  of  invoices  is  the  process  of 
finding  the  date  when  several  invoices  due  at  different  daces  may 
he  paid  in  one  amount,  without  loss  of  interest  to  either  debtor  or 
creditor.  This  date  is  called  the  equated  date  of  payment. 

Use.  The  process  of  averaging  the  dates  of  invoices  is  most 
frequently  used  in  bankruptcy  settlements,  where  claims  when 
filed  with  a  trustee  must  show  the  average  due  date  of  the  items 
if  interest  is  to  be  obtained  on  overdue  amounts.  In  general,  the 
equated  date  is  important  in  the  settlement  of  bills  of  long  stand- 
ing, and  in  the  fixing  of  the  date  of  a  note  in  settlement  of  invoices. 

Term  of  credit.  A  term  of  credit  is  the  time  elapsing  between 
the  date  of  a  bill  and  the  date  on  which  it  becomes  due;  as,  "Bill 
purchased  January  10,  Term  of  Credit  10  days."  The  due  date 
would  be  January  20. 

Average  due  date.  The  average  due  date  is  the  date  on 
which  settlement  of  the  complete  account  should  be  made  by  pay- 
ment of  the  amount  of  the  invoices,  without  charge  for  interest 
on  overdue  items  or  allowance  for  discount  on  prepaid  items. 

Focal  date.  The  focal  date  is  an  assumed  date  of  settlement 
with  which  the  due  dates  of  the  several  items  may  be  compared,  to 
determine  the  equated  date  of  payment. 

Any  elate  ma}'  be  used  as  the  focal  date,  and  the  final  result 
will  be  the  same.  In  the  interest  method,  any  rate  per  cent  may 
be  used,  and  the  result  will  be  the  same.  However,  6%  is  usually 
used,  as  the  computations  are  then  less  complicated. 

In  all  calculations,  use  the  nearest  dollar.  For  example,  for 
S115.29,  use  SI  15.00;  and  for  $101.84,  use  $162.00. 

When  several  bills  are  sold,  some  of  which  have  a  term  of  credit, 
first  find  the  due  date  of  those  with  a  term  of  credit,  arid  then  find 
the  equated  date  of  the  several  bills. 

With  respect  to  bills  with  terms  of  credit,  the  due  date  of  such 
bills,  rather  than  the  invoice  date,  is  used  in  computing  the  equated 
date. 

Do  not  use  fractions  of  a  day  in  determining  the  average  date. 

Methods.  There  are  two  methods  in  common  use:  the  Prod- 
uct Method,  and  the  Interest  Method. 

131 


132  AVERAGING  DATES  OF  INVOICES 

Rule  for  product  method.  Use  as  the  focal  date  the  last 
of  the  month  preceding  the  first  item.  Multiply  each  item  by  the 
number  of  days  intervening  between  the  assumed  date  and  the 
due  date  of  the  item,  and  divide  the  sum  of  the  several  products 
by  the  sum  of  the  account.  Count  forward  from  the  assumed  date 
the  number  of  days  obtained  in  the  quotient.  The  result  will  be 
the  average  due  date. 

Example 

Find  the  date  at  which  the  following  bills  of  merchandise  may  be  paid  in  one 
amount  without  loss  to  either  party:  Due  January  1,  $150.00;  February  14, 
$200.00;  April  20,  $155.00;  June  15,  $200.00. 

Solution  by  Product  Method 

(Focal  date,  Dec.  31) 

Due  January  1 $150  X      1  =       150 

Due  February  14        200  X    45  =    9,000 

Due  April  20      155X110  =  17,050 

Due  June  15 J200  X  166  =  33,200 

705  59,400 

59,400  4-  705  =  84  days. 
84  days  after  December  31  is  March  25. 

Explanation.  For  convenience,  assume  December  31  as  the  date  of  settle- 
ment. On  the  first  bill,  which  is  due  January  1,  there  would  be  interest  for 
1  day.  On  the  second  bill  there  would  be  interest  from  December  31  to  Febru- 
ary 14,  or  45  days,  which  is  equivalent  to  interest  on  $9,000.00  for  1  day.  On 
the  third  bill  there  would  be  interest  from  December  31  to  April  20,  or  110  days, 
which  is  equivalent  to  interest  on  $17,050.00  for  1  day.  On  the  fourth  bill 
there  would  be  interest  from  December  31  to  June  15,  or  166  days,  which  is 
equivalent  to  interest  on  $33,200.00  for  1  day. 

If  all  the  bills  were  paid  December  31,  the  debtor  would  be  entitled  to  interest 
on  $59,400.00  for  1  day,  or  interest  on  $705.00,  the  amount  of  the  account,  for 
84  days.  It  is  evident  that  the  bills  could  be  paid  at  a  time  84  days  later  than 
December  31,  or  March  25,  without  loss  to  either  party. 

Verification 

The  interest  on  $150.00  for  83  days  is $2.08 

The  interest  on  $200.00  for  39  days  is 1.30 

Total  gain  of  interest  to  debtor $3J38 

The  interest  on  $155.00  for  26  days  is ITT67 

The  interest  on  $200.00  for  82  days  is 2.72 

Total  gain  of  interest  to  creditor $3.39 

The  gain  of  interest  to  the  debtor  is  on  all  bills  paid  after  they  are  due. 

The  gain  of  interest  to  the  creditor  is  on  all  bills  paid  before  they  are  due. 
These  two  results  should  be  equal,  or  within  a  few  cents  of  the  same  amount. 
The  reason  for  a  little  discrepancy  is  the  fraction  of  a  day  which  is  disregarded 
in  determining  the  due  date  of  the  account. 


AVERAGING  DATES  OF  INVOICES  133 

Solution  by  Interest  Method 

January  1 $150  00  for      1  day    =  $0  03,  interest 

February  14 200  00  for    45  days  =     1 . 50,  interest 

April  20 155  00  for  110  days  =     2  84,  interest 

June  15 200.00  for  166  days  =    5  53,  interest 

Total  interest $9.90 

Interest  on  $705.00  for  1  day  is  $0.1175. 
$9.90  •*-  $0.1175  =  84,  or  84  days. 

Explanation.  Assume  December  31,  the  last  day  of  the  month  preceding 
the  first  item,  to  be  the  date  of  settlement.  If  the  amount  of  the  account, 
$705.00,  is  paid  December  31 ,  there  will  be  a  loss  of  interest  to  the  debtor  of  $9.90. 
The  interest  on  $705.00  for  1  day  at  6%  is  $0.1175.  It  will  take  a  principal  of 
$705.00  as  many  days  to  produce  $9.90  as  the  number  of  times  that  $0.1175  is 
contained  in  $9.90,  or  84  days,  the  same  result  as  was  obtained  by  the  product 
method. 

Problems 

1.  Several  invoices  mature  as  follows: 

April  12 $260  00     August  18  $120  00 

May  25 500.00     September  2 300  00 

At  what  date  may  the  foregoing  invoices  be  paid  in  one  amount  without  loss 
to  either  party? 

2.  Calculate  the  average  due  date  of  the  following  invoices: 

June  10 $400  00     August  15     $250  00 

July  27 100  00     September  22  300 . 00 

3.  Calculate  the  average  due  date  of  the  following  invoices: 

May  8     $275  00  on  60  days'  credit 

May  24  150  00  on  2  months'  credit 

June  10 300 . 00  on  90  days'  credit 

July  1   250  00  on  30  days'  credit 


CHAPTER  14 
Equation  of  Accounts,  or  Compound  Average 

Definition.  Equation  of  accounts,  or  compound  average,  is 
the  process  of  finding  the  date  when  the  balance  of  an  account 
having  both  debits  and  credits  can  be  paid  without  loss  to  either 
debtor  or  creditor.  With  respect  to  bills  with  terms  of  credit,  the 
due  date  of  the  bill  rather  than  the  invoice  date  is  used  in  comput- 
ing the  equated  date.  Credits  other  than  for  cash  (such  as  non- 
interest-bearing  notes)  are  extended  to  the  due  date  thereof. 
Summarizing  briefly,  the  date  to  be  used  for  each  debit  and  credit 
is  the  date  when  the  item  has  a  cash  value  of  the  amount  shown  in 
the  entry. 

Rule  for  the  product  method.  After  finding  the  date  that  each 
item  has  a  cash  value,  use  the  last  day  of  the  month  preceding  the 
earliest  date  as  the  focal  date  for  both  sides  of  the  account.  Find 
the  number  of  days  between  the  focal  date  and  the  due  date  of 
each  item;  multiply  each  item  by  the  number  of  days  intervening 
between  the  focal  date  and  the  due  date  of  the  item.  Find  the 
sum  of  the  products  on  both  the  debit  and  the  credit  sides  of  the 
account.  Divide  the  difference  between  the  sums  of  the  debit 
and  the  credit  products  by  the  balance  of  the  account.  The 
quotient  will  be  the  number  of  days  between  the  focal  date  and 
the  average  date  of  the  account. 

When  to  date  forward  or  backward.  The  average  date  is 
forward  from  the  focal  date  when  the  balance  of  the  account  and 
the  excess  of  the  products  are  on  the  same  side  (both  debits  or  both 
credits) ;  if  they  are  on  opposite  sides,  the  average  date  is  backward 
from  the  focal  date. 

Example 

At  what  date  may  the  balance  of  the  following  account  be  paid  without  loss 
of  interest  to  either  party? 

Debits  Credits 

July     1,  Mdse.  30  days $250.00    Aug.  15,  Cash $400.00 

July  26,  Mdse.  30  days 425  00    Sept.  10,  Cash  300.00 

Aug.  15,  Mdse.  60  days 320.00    Sept.  20,  Cash 150.00 

Aug.  30,  Mdse.  60  days 500 . 00 

135 


136    EQUATION  OF  ACCOUNTS,  OR  COMPOUND  AVERAGE 

Solution  by  Product  Method 

Since  the  earliest  date  is  July  31,  the  assumed  focal  date  would  be  June  30. 
However,  since  July  31  is  an  end-of-month  date,  this  date  is  used,  as  each  multi- 
plier is  31  less  than  it  would  be  if  June  30  were  used. 

My  31,  $    250  00  X    0  =  00,000  Aug.  15,  $400  00  X  15  =    6,000 

Aug.  25,       425  00  X  25  =  10,625  Sept.  10,    300  00  X  41  =  12,300 

Oct.  14,       320  00  X  75  =  24,000  Sept.  20,    150.00  X  51  =    7,650 
Oct.  29,       500  00  X  90  =  45,000 

$1,495.00                79^625  $850  00                25^950 

Debit  side $1,495  00     $79,625.00 

Credit  side 850.00      25,950  00 

$~645.00     $53~675.00 
$53,675.00  4-  $645.00  =  83. 

The  equated  date  is,  therefore,  83  days  after  July  31,  or  October  22. 

Explanation.  First  find  the  due  date  of  each  item.  For  convenience,  assume 
July  31,  the  earliest  due  date,  as  the  day  of  settlement  for  ail  the  items  on  each 
side  of  the  account.  Proceed  as  in  the  process  of  averaging  dates,  which  was 
described  in  the  preceding  chapter.  With  July  31  used  as  the  focal  date,  there 
is  a  loss  of  interest  on  the  total  debits  equivalent  to  the  interest  on  $79,625.00  for 
1  day,  and  a  gain  of  interest  on  the  total  credits  equivalent  to  the  interest  on 
$25,950.00  for  1  day;  or  a  net  loss  of  interest  equivalent  to  the  interest  on 
$53,675.00  for  I  day,  which  is  equal  to  the  interest  on  $645.00  for  83  days.  It  is 
evident  that  the  date  when  there  would  be  no  loss  of  interest  to  either  party 
must  be  83  days  after  July  31,  or  October  22. 

Solution  by  Interest  Method 

Debits 

July  31,    0  days'  interest  on  $  250  00  =  $  .00 

Aug.  25,  25  days'  interest  on  425  00  =  1   77 

Oct.    14,  75  days'  interest  on  320  00  =  4  00 

Oct.   29,  90  days'  interest  on  500  00  =  7  50 

$1,495766       $13727 
Credits 

Aug.  15,  15  days'  interest  on  $  400.00  =  $  1  00 
Sept.  10,  41  days'  interest  on  300  00  -  2  05 
Sept.  20,  51  days'  interest  on 150  00  =  1  28 

$~850.00       $"4.33 

Dr.  $1,495  00        Interest,  $13  27 
Cr.       850_00         Interest,      4  33 
6000)_  645^00  $  8.94 

$      1075  interest  for  one  day. 
$8.94  +  .1075  =  83. 

The  equated  date  of  payment  is,  therefore,  83  days  after  July  31,  or 
October  22. 

Explanation.  The  explanation  of  the  product  method  is  applicable  to  the 
interest  method.  The  variance  is  in  finding  interest  on  each  item  and  dividing 
the  interest  balance  by  the  interest  for  1  day  on  the  balance  of  the  account. 


EQUATION  OF  ACCOUNTS,  OR  COMPOUND  AVERAGE    137 


Problems 

Find  the  equated  date  in  each  of  the  following: 
1. 


Debits 

June  10,  Mdse 

Aug.  20,  Mdse 

Oct.  30,  Mdse 

2. 

Debits 

May  3,  Mdse.  60  days. 
June  15,  Mdse.  60  days. 
July  20,  Mdse.  30  days 
Aug.  27,  Mdse.  60  days 

3. 

Debits 

Mar.    1,  Mdse.  30  days 
Mar.  20,  Mdse.  2  mos 
Apr.     5,  Mdse.  60  days 


Credits 

$500  00  July  5,  Cash $300  00 

100  00  Aug.  10,  Cash 150  00 

250  00  Sept.  25,  Cash 200  00 

Credits 

$300  00    June  20,  Cash $150.00 

250  00    July     1,  Note,  30  days  with- 

175.00        out  int 200.00 

225.00    Aug.  10,  Note,  60  days,  int., 

6% 300.00 


Credits 


$225.00  Mar.  31,  Cash 
300  00  Apr.  15,  Cash 
150.00  May  10,  Casli 


$150.00 
100  00 
200.00 


CHAPTER  15 
Account  Current 

Definition.  An  account  current  is  a  transcript  of  the  ledger 
account.  It  should  show  the  dates  on  which  sales  were  made, 
the  term  of  credit  for  each  item,  cash  payments,  and,  if  settlements 
were  made  by  note,  the  date  and  other  details  of  each  note. 

Methods.  Two  methods  are  used  in  finding  the  amount  due: 
the  Interest  Method,  and  the  Product  Method. 

Example 

Find  the  balance  due  January  1  on  the  following  ledger  account,  which  bears 
interest  at  6%. 

J.  B.  JOHNSON 
Dr.  Cr. 

Sept.    1,  Balance $1,200  00    Oct.     1,  Cash $1,000  00 

Sept.  20,  Mdse.  30  days  ...        400  00     Nov.  10,  Cash 200.00 

Oct.    30,  Mdse.  30  days   ...         520  00     Dec.     3,  Cash 400.00 

Nov.  25,  Mdse.  30  days 350.00     Dec.  15,  Note  10  days 300.00 

Solution  by  Interest  Method 

J.  B.  JOHNSON 

Dr.  Cr. 

Date  Due      Amount    Days  Interest  Date  Amount    Days    Interest 

Sept.    1     $1,200  00    122  $24  40  Oct.      1  $1,000  00     92       $15  33 

Oct.    20          400  00      72  4.87  Nov.  10  200  00     52           1  73 

Nov.  29         52000      33  2.86  Dec.     3  40000    29          1.93 

Dec.  25 350_00       7     .41  Dec.  25     300  00      7            .35 

$2~470  00  $32.54  $1,900.00  $19.34 

1,900  00  19  34 

$    570  00      -f-  $13.20  =  $583.20 

Explanation.  The  number  of  days  opposite  each  entry  is  the  actual  number 
of  days  from  the  date  of  the  item  to  January  1,  the  date  which  is  taken  as  the 
focal  date. 

Solution  by  Product  Method 


Dr. 

Cr. 

Date  Due 

Amount 

Days        Product 

Date 

Amount 

Days       Product 

Sept. 

1 

$1,200 

X 

122 

=   $146,400 

Oct. 

1 

$1,000 

X 

92 

=  $  92,000 

Oct. 

20 

400 

X 

73 

=       29,200 

Nov. 

10 

200 

X 

52 

10,400 

Nov. 

29 

520 

X 

33 

17,161 

Dec. 

3 

400 

X 

29 

=       11,600 

Dec. 

25 

350 

X 

7 

2,450 

Dec. 

25 

300 

X 

7 

2,100 

$2,470 

$195,210 

$1,900 

$116,100 

1,900 

116,100 

$    570  $  79,110 

The  intent  on  $79,110  for  1  day  =  $  13.20 
$570.00  +  $13.20  =  $583.20 
139 


140  ACCOUNT  CURRENT 

In  some  instances  it  is  more  convenient  to  find  the  equated  due 
date,  and  then  calculate  the  interest  on  the  balance  of  the  account 
from  that  date  to  the  date  of  settlement. 

Problems 

1.  Find  the  amount  that  will  settle  the  following  account  Sept.  10,  interest 
at  6%. 

Dr.  Cr. 

Mar.  15,  Mdse.  4  mos $450  00  July     5,  Cash $400  00 

Mar.  30,  Mdse.  60  days     ....  375  00  July  30,  Cash      375  00 

Apr.  18,  Mdse.  30  days     ....  700  00  Aug.  15,  Cash      690.00 

May  15,  Mdse.  4  mos 62000  Sept.    5,  Cash 61500 

May  30,  Mdse.  4  mos 410  00 

2.  Find  the  amount  that  will  settle  the  following  account  on  June  1,  interest 
at  6%. 

Dr.  Cr. 

Jan.     4,  Mdse.  30  days $500  00  Feb.  20,  Cash $300 . 00 

Jan.  30,  Mdse.  30  days 200  00  Feb.  28,  Note,  60  days  with 

Feb.     5,  Mdse.  30  days 600  00                     interest  at  6%  ...     300  00 

Mar.    1,  Mdse.  30  days 400.00  Mar.  20,  Cash 15000 

3.  A  borrowed  $10,000.00  from  a  bank  on  January  2,  giving  a  note  secured 
by  a  mortgage  for  building  a  home,  due  in  one  year,  with  interest  at  6%.     From 
time  to  time  the  bank  advanced  him  money  to  pay  contractors'  estimates. 
Before  maturity  the  bank  had  actually  advanced  $9,000.00,  as  follows: 

January  31 $3,000  00 

March  15 3,00000 

April  15 1,500.00 

May  15 1,500.00 

On  June  1,  the  following  year,  the  maker  of  the  note  desires  to  pay  it.     (a) 
How  should  interest  be  computed?     (6)  What  amount  is  due  June  1? 


CHAPTER  16 
Storage 

Definition.  Storage  is  the  charge  made  by  a  warehouse  or 
depositary  for  the  storing  of  goods  until  they  are  required  for  use 
or  for  transportation  to  some  other  point. 

Running  account.  When  goods  are  being  received  and 
delivered,  the  storage  company  keeps  a  running  account,  showing 
the  dates  at  which  goods  are  received  and  delivered,  together  with 
details  of  the  number  of  packages,  barrels,  and  so  forth.  Storage 
is  charged  for  the  average  number  of  days  for  which  one  package, 
barrel,  or  box  has  remained  in  storage.  The  average  number  of 
days  is  divided  by  30  to  reduce  the  average  number  of  days  to 
months,  or  by  7  to  reduce  the  average  number  of  days  to  weeks,  as 
the  case  may  be ;  then  the  number  of  months  or  weeks  is  multiplied 
by  the  price  per  month  or  per  week. 

Example 

The  following  is  a  memorandum  of  the  quantity  of  salt  stored  with  a  storage 
company  at  4^  per  barrel  per  term  of  30  days'  average  storage. 


Time  in 

Equivalent 

Date 

Receipts 

Deliveries 

Balance 

Storage 

for  1  Day 

June    4 

120  bbi. 

120  bbl. 

28  days 

3,360  bbl. 

July    2 

20  bbl. 

100  bbl. 

18  days 

1,800  bbl. 

July  20 

100  bbl. 

200  bbl. 

10  days 

2,000  bbl. 

July  30 

50  bbl. 

150  bbl. 

11  days 

1,650  bbl. 

Aug.  10 

100  bbl. 

50  bbl. 

15  days 

750  bbl. 

Aug.  25 

50  bbl. 

Obbl. 

9,560  bbl. 

Explanation.  9,560  bbl.  for  1  day  are  equivalent  to  1  barrel  for  9,560  days, 
and  9,560  divided  by  30  (the  number  of  days  per  term)  equals  318-f  terms.  In 
some  cases  a  full  month's  storage  is  charged  for  any  part  of  a  month  that  goods 
remain  in  storage;  in  other  cases,  15  days  or  less  are  called  one-half  of  a  month, 
and  any  period  of  over  15  days  is  counted  as  a  whole  month.  318f  terms  would 
be  charged  for  as  319  terms,  and  319  X  .04  =  12.76.  Therefore,  $12.76  is  the 
storage  charge. 

141 


142  STORAGE 

Problems 


the  following  memoranda,  compute  storage  at  4ff  per  barrel  per  term 
of  30  days'  average  storage: 

Received  Delivered 

Feb.  10     ...........  300  bbl.  Feb.  20                  ..      .  150  bbl. 

Feb.  19  ..........  150  bbl.  Mar.  5  .......  200  bbl. 

Mar.  12     .........  500  bbl.  Mar.  15  .......  400  bbl. 

v          Mar.  30        .       .        .  .  300  bbi.  Apr.  14            .....  300  bbl. 

;  2/)A  grower  stored  5,000  bushels  of  potatoes  at  5^  per  cwt.,  the  term  being 
30  days'  average  storage.  The  following  is  a  memorandum  of  the  transactions 
that  occurred.  Compute  the  amount  of  storage. 

Received  Delivered 

Sept.  1  ..........   2,500  bushels     Nov.  4  ..........  500  bushels 

Sept.  10  .........   1,500  bushels     Dec.  10  .........  600  bushels 

Oct.  5  ...........   1,000  bushels    Jan.  15  ..........  750  bushels 

Feb.  1  ........  1,500  bushels 

Mar.  18.              .  750  bushels 

Apr.  2  ........  000  bushels 


CHAPTER  17 
Inventories 

Valuation  of  inventories.  The  bases  of  inventory  valuation 
most  commonly  used  by  business  concerns  are:  (a)  cost;  and  (b) 
cost  or  market,  whichever  is  lower.  However,  the  average  cost 
method  is  used  in  some  instances  —  the  tobacco  industry,  for 
example  —  and  market  value  as  a  basis  is  used  in  grain  and  cotton 
inventories  and  in  inventories  of  dealers  in  securities. 

Cost  or  market,  whichever  is  lower.  In  valuing  inventories 
at  cost  or  market,  whichever  is  lower,  a  comparison  of  inventory 
totals  at  the  two  values  is  not  sufficient.  It  is  necessary  to  consider 
each  item  or  group  of  similar  items  purchased  at  the  same  price, 
and  to  make  the  extension  at  the  cost  or  market  price,  whichever 
is  lower.  *^f\ 

&      f 
Example 

One  hundred  tons  of  sugar  (200,000  Ibs.)  were  purchased  at  7|4  a  pound,  and 
later,  50  tons  (100,000  Ibs.)  were  purchased  at  6^  a  pound.  The  entire  150  tons 
were  on  hand  at  the  close  of  the  year,  at  which  time  the  market  value  of  sugar 
was  6^^  a  pound.  Compute  the  inventory  at:  (a)  cost;  and  (b)  at  cost  or  market, 
whichever  is  lower. 

Solution 

(a)  200,000  Ibs.  @  .07  ..............................  $14,000 

100,000  Ibs.  @  .06  ...............................   6,000 

Inventory  at  cost  ..................   §?0,000 

(b)  200,000  Ibs.  @  .065 
100,000  Ibs.  @  .06 


_ 
Inventory  at  cost  or  market,  whichever  is  lower   .  .   $19,000 

Problems 

A 

(jj.*  Given  the  following  inventory  of  a  retail  shop  for  children's  clothing,  toys, 
and  so  forth  (correct  as  to  quantities  and  values),  state  the  amount  which  should 
be  shown  on  a  balance  sheet  as  merchandise  inventory,  adopting  the  method  of 
valuing  inventory  at  cost  or  market,  whichever  is  lower. 


*  C.  P.  A.,  Maryland. 

143 


144 


INVENTORIES 


Value  Per  Unit 


Item 


150  knit  towels $ 

16  crepe  cle  chine  carriage  sets 10  00 

125  lingerie  and  pongee  hats 2 .00 

85  rubber  bibs  with  sleeves 

240  creepers 2 

200  spring  coats 9 

50  spring  coats 17 

8  play  yards     6 

8  desks  used  in  office 55 

140  shirts 

200  boys'  wash  suits   0 

125  bloomers 1 

5  cribs 21 

12  electric  trains 1 

Total 


)St 

Market 

38 

$  0  35 

00 

12.50 

.00 

1.75 

.50 

.50 

05 

1  98 

50 

10  00 

50 

18  50 

00 

5.75 

.00 

60  00 

.75 

.79 

20 

5  98 

85 

1   cSO 

00 

19  98 

50 

1.50 

Total 

Value 

Cost 

Market 

$      57.00 

$      52  50 

160  00 

200  00 

250  00 

218.75 

42  50 

42.50 

492  00 

475  20 

1,900  00 

2,000.00 

875  00 

925  00 

48  00 

46.00 

440  00 

480  00 

105  00 

110.60 

1,240  00 

1,196  00 

231  25 

225  00 

105  00 

99  90 

18  00 

18  00 

$5,963  75     $6,089  45 


(  2.*/  You  are  called  in  by  the  X.  Y.  Z.  Clothing  Company  to  advise  them  on 
the-mlculation  of  their  inventory.  They  have  always  followed  the  policy  of 
cost  or  market,  whichever  is  lower.  You  are  informed  that  the  inventory  will 
be  used  for  the  tax  return,  as  well  as  for  the  annual  report  to  stockholders. 

Value  per  Unit 

Item  Cost  Market 

13  suits,  grade  A $60  00  $55  00 

12  suits,  grade  B 40  00  37  50 

77  suits,  grade  C 30  00  30  00 

7  suits,  grade  D 20.00  22  00 

24  overcoats,  grade  1 75  00  80  00 

5  overcoats,  grade  2 40  00  45  00 

10  overcoats,  grade  3 30  00  25  00 

6  topcoats,  grade  X 20  00       17 . 50 

9  topcoats,  grade  Y 15  00       12  50 

18  topcoats,  grade  Z 10.00       11 .00 

Total 


Total 
Cost 

I  780  00 
480  00 
510  00 
140  00 
1,800  00 
200  00 
300  00 
120  00 
135  00 
180  00 


Value 

Market 

$    715  00 

450  00 

510  00 

154  00 

1,920  00 
225  00 
250  00 
105  00 
112.50 
198.00 


$4,645  00     $4,639  50 


Which  total  figure  would  you  advise  the  company  to  use  for:  (a)  tax  reports; 
(b)  annual  report  to  stockholders? 

Average  cost  method.  The  general  rule  that  the  average  cost- 
method  of  valuing  inventories  will  not  be  accepted  for  income  tax 
purposes  is  subject  to  certain  exceptions.  In  the  tobacco  industry, 
for  example,  tobacco  is  bought  from  the  producer,  usually  in  small 
quantities  and  at  greatly  varying  prices.  Different  grades  of 
tobacco  are  mixed  and  stored  in  hogsheads,  and  it  is  practically 
impossible  to  determine  the  exact  cost  of  any  particular  hogshead. 
The  inventory  is  therefore  averaged  monthly,  according  to  grades, 
as  follows: 

First  method.  From  the  inventory  of  each  grade  at  the  begin- 
ning of  the  month  is  subtracted  the  amount  of  tobacco  of  that 

*C.  P.  A.,  Wisconsin. 


INVENTORIES 


14* 


grade  used,  leaving  so  many  pounds  costing  so  many  dollars;  to 
this  is  added  the  tobacco  of  that  grade  purchased  during  the 
month,  and  a  new  average  is  determined.  This  is  the  inventory 
for  the  close  of  the  month,  and  is  consequently  the  opening  inven- 
tory of  the  next  month. 

Example 
STSCK  CARD 


RECEIVED 

ISSUED 

BALANCE 

Date 

Quan- 
tity 

Rate 

Amount 

Date 

Quan- 
tity 

Rate 

Quan- 
tity 

Rate 

Amount 

6-29 
9-30 
12-10 

100,000 
80,000 
125,000 

$1  00 
1.10 
.95 

$100,000 
88,000 
118,750 

9-1 
12-5 
12-18/ 

80,000 
30,000 
20,000 

$1  00 
1  08 
1  08 

100,000 
20,000 
100,000 
70,000 
195,000 
175,000 

$1.00 
1  00 
1  08 
1.08 

$100,000 
20,000 
108,000 
75,600 
194,350 
172,750 

Inv't 
12-31 

175,000 

$0  987 

$172,750 

Explanation.  It  will  be  noticed  that  the  receipt  of  125,000  at  .95  on  Dec.  10 
was  extended  into  the  balance  column  in  quantity  and  amount  only,  and  that 
the  issuance  on  Dec.  18  was  made  at  the  rate  established  on  the  first  of  the 
month.  This  is  the  method  that  is  used  when  receipts  are  frequent,  as  it  saves 
the  time  that  would  be  required  to  compute  a  new  rate  after  each  receipt,  and 
establishes  a  standard  rate  of  issuance  for  the  month. 

Second  method.  When  receipts  are  not  frequent  and  are  large 
in  amount,  a  new  average  price  is  computed  upon  the  entry  of 
each  receipt. 

Example 
STOCK  CARD 


RECEIVED 

ISSUED 

BALANCE 

Date 

Quan- 
tity 

Rate 

Amount 

Date 

Quan- 
tity 

Rate 

Quan- 
tity 

Rate 

Amount 

6-29 
9-30 
12-10 

100,000 
80,000 
125,000 

$1.00 
1.10 
.95 

$100,000 
88,000 
118,750 

9-1 
12-5 
12-18 

80,000 
30,000 
20,000 

$1.00 
1.08 
99i 

100,000 
20,000 
100,000 
70,000 
195,000 
175,000 

$1.00 
1  00 
1.08 
1.08 
.99^ 
.99i 

$100,000 
20,000 
108,000 
75,600 
194,350 
174,430 

Inv't 
12-31 

175,000 

$0.99^ 

$174,430 

146 


INVENTORIES 


^~\  Problems 

fy.  Rule  two  stock  cards  as  in  the  preceding  example,  and  enter  the  following 
data.  Compute  the  balances:  (a)  by  the  first  method;  and  (6)  by  the  second 
method. 

Received 

July  5 80,000  units  @  $0.90 

Aug.  15. ...  20,000  units  @    1 .00 

Sept.  1 30,000  units  @    1.10 

.—   Dec.  8 20,000  units  @    1 .20 


Issued 

Aug.  1 50,000  units 

Dec.  2 20,000  units 

Dec.  20 30,000  units 


2.  Complete  the  following  stock  ledger  card,  using  average-price  method. 


Stores  Ledger 

Actual  Receipt  Price 

./£" 

.20 

Average  Price 

J5 

/7o<f 

Name      {ffludshs^A 

'  5      7/7&t&                          part  No.  5&  "A  7& 

Minimum        /#?     " 

Maximum    3O0               Location      jL3 

Drawing  No. 

Unit 

REFERENCE 

QUANTITY 

ON  HAND 

Date               Number 

Remarks 

Received 

Issued 

Quantity 

Value 

I 

Z 

OCT  4            4523 
OCT  10          34567 
OCT  12         35$54 
OCT  13         38765 
OCT  15         -39458 
OCT  16           4587 
OCT  19         40156 

100 
50 

5 
10 
3 
12 

10 

1 

2 

3 

— 





J^ 
5 

6 

F 





7 
8 

6 

9 

01 

10 

U 
Zl 
81 





11 
12 
13 

"First-in,  first-out"  method  of  inventory.  Where  the  same 
merchandise  has  been  purchased  at  various  prices  during  the  year, 
and  the  goods  on  hand  cannot  be  identified  with  specific  invoices, 
the  amount  on  hand  at  the  end  of  the  year  may  be  inventoried 
at  the  latest  purchase  price.  If,  however,  the  quantity  on  hand  is 
greater  than  the  amount  purchased  at  the  last  price,  the  balance 
may  be  inventoried  at  the  next  to  the  last  purchase  price,  and  so  on. 
This  method  is  termed  "first-in,  first-out  method"  of  inventory. 

Example 

Inventory,  December  31 275,000  units 

Invoices : 

November  10 125,000  units  @  $0.95  per  C 

September  5 80,000  units  @    1.10  per  G 

June  10 70,000  units  @    1.00  per  G 

How  should  the  foregoing  inventory  be  valued? 


INVENTORIES  147 

Solution 

125,000  units  @  $0.95  per  C  ........................  $1,187.50 

80,000  units  @    1.10  per  C  ........................  880.00 

70,000  units  @  1.00  per  C  ........................  700.00 

275,000  units  inventoried  ...........................  $2,767  .  50 

"Last-in,  first-out"  method  of  inventory.  Under  the  "last-in, 
first-out"  method  inventories  are  valued  at  the  cost  of  goods 
earliest  acquired,  and  in  computing  profits  from  sales  the  cost  of 
goods  last  acquired  is  used.  This  method  will  show  smaller 
profits  when  prices  are  rising  and  larger  profits  when  prices  are 
falling  than  the  afirst-in,  first-out"  method.  Businesses  which 
use  raw  materials  or  other  goods  includable  in  inventory,  which  are 
subject  to  sharp  price  fluctuations;  businesses  in  which  the  value  of 
inventory  is  large  compared  with  other  assets  and  sales;  and 
businesses  in  which  production  consumes  an  extended  period  are 
most  likely  to  benefit  from  the  use  of  this  method.  (Consult  the 
Internal  Revenue  Code  relative  to  the  requirements  incident  to 
adoption  and  use  of  this  method.) 

Example 

A  has  an  opening  inventory  of  10  units  at  10  cents  a  unit,  and  during  the 
year  he  makes  purchases  of  10  units  as  follows: 


January  

1  @  .11  =« 

11 

April       

2  @  .12  = 

?4 

July           

3  @  .13  = 

39 

October 

4  @  .14  = 

56 

10                  f 

730 

His  closing  inventory  shows  15  units.    What  is  the  value  of  the  closing 
inventory? 

Solution 

10  @  .10  =  1.00 

1  @  .11  (Jan.)  =     .11 

2  @  .12  (Apr.)  =     .24 
_2  @  .13  (July)  =     .26 

Totals  15  1.61 

Problem 
Value  the  closing  inventory,  using  the  "  last-in,  first-out'*  method. 

Opening  inventory:  50  units  at  $1.00 
Production: 

First  quarter:       50  units  at  $1.50 

Second  quarter:  100  units  at  $1.75 

Third  quarter:      50  units  at  $2.00 

Fourth  quarter:  100  units  at  $2.25 
Closing  inventory:  150  units 


148  INVENTORIES 

Merchandise  turnover.  The  number  of  times  that  the  valu« 
of  the  inventory  is  contained  in  the  cost  of  sales  is  the  merchandise 
turnover. 

The  final  inventory  should  not  be  used  in  computing  turnover, 
unless  it  represents  a  normal  inventory  for  the  fiscal  period,  or  is 
the  first  inventory  that  has  been  taken. 

If  a  perpetual  inventory  system  is  in  use,  the  monthly  inven- 
tories should  be  added  to  the  inventory  at  the  beginning  of  the 
period,  and  the  sum  divided  by  the  number  of  months  in  the  fiscal 
period  plus  one.  In  a  year  there  would  thus  be  thirteen  inven- 
tories— the  one  at  January  1,  and  the  twelve  inventories  at  the 
ends  of  the  months.  When  a  perpetual  inventory  is  not  used, 
add  the  inventory  at  the  beginning  of  the  fiscal  period  to  the  one 
at  the  close  of  the  period;  then  divide  by  two.  The  quotient  will 
be  the  estimated  average  inventory  for  the  period.  If  semiannual 
inventories  are  taken,  use  three  inventories  and  divide  by  three. 
If  quarterly  inventories  are  taken,  use  five  inventories  and  divide 
by  five. 

FORMULA 
Cost  of  Sales  -f-  A  verage  Inventory  at  Cost  —  Rate  of  Turnover 

Example 

A  department  store  found  the  average  inventory  of  Department  A  for  the 
fiscal  period  to  be  $30,000.  The  cost  of  sales  for  the  same  period  was  $120,000. 

$120,000  -^-  $30,000  =  4,  the  rate  of  turnover. 

An  estimated  inventory  at  the  end  of  any  period  may  be 
obtained  by  dividing  the  sales  for  the  period  by  100%  plus  the 
per  cent  of  gross  profit  based  on  cost,  and  deducting  the  quotient 
from  the  total  of  purchases  and  first  of  period  inventory.  A  more 
complete  discussion  of  the  gross  profit  test  is  given  in  Chapter  18. 

Example 

In  the  above  example,  assume  that  in  Department  A  the  total  cost  of  mer> 
chandise  was  $150,000,  that  the  sales  were  $144,000,  and  that  the  average  profits 
were  20%.  Using  the  per  cent  of  gross  profits  to  determine  the  average  inven 
tory,  the  solution  would  be  as  follows: 

$144,000  (sales)  -i-  120%  =  $120,000,  the  cost  of  sales. 
$150,000  (total  cost  of  goods)  -  $120,000 

(cost  of  sales)  =  $30,000,  the  estimated  inventory. 
$120,000  (cost  of  sales)  -f-  $30,000 

(inventory)  =  4,  the  rate  of  turnover. 

Number  of  turnovers.  The  number  of  turnovers  varies  in 
different  lines  of  business.  Records  show  turnovers  varying  from 
1  to  more  than  20,  depending  on  the  kind  of  business.  It  is  possi- 


INVENTORIES  149 

blc  to  make  a  larger  profit  by  several  turnovers  with  a  small 
mark-up*  than  by  1  or  2  turnovers  with  a  large  mark-up.  Limited 
capital  and  frequent  turnovers  can  produce  a  profit  equal  to  that 
produced  by  a  greater  capital  turned  fewer  times  a  year.  If  a 
merchant  turns  $1  eight  times  in  the  course  of  a  year,  he  has  used 
i  of  the  capital  that  would  be  required  if  the  rate  of  turnover 
were  1. 

Example  1    . 

A  merchant  had  a  rate  of  mark-up  of  50%,  with  a  turnover  of  1.  He  found 
that  by  using  a  rate  of  mark-up  of  30%  he  had  a  turnover  of  2.  If  his  former 
sales  were  $300,000  annually,  how  much  were  his  gross  profits  increased,  pro- 
vided he  continued  to  use  the  same  investment  in  merchandise? 

$300,000  +  150%  =  $200,000,  cost  of  sales. 
$300,000  -  $200,000  =  $100,000,  gross  profits. 

Under  the  new  policy  he  turns  the  $200,000  twice,  the  equivalent  of  $400,000 
annually. 

$400,000  at  30%  =  $120,000,  profits. 

$120,000  -  $100,000  =  $20,000,  increased  profits  due  to  lowering 

the  rate  of  mark-up  and  increasing 
the  rate  of  turnover. 

Example  2 

What  investment  in  merchandise  would  he  required  under  the  new  policy 
to  make  the  same  amount  of  profits  that  was  made  under  the  old  policy? 

$100,000  -h  30%  =  $333,333.33,  cost  of  goods  sold  to  make  profits  of  $100,000. 

Since  there  were  2  turnovers,  the  cost  of  goods  sold  was  twice  the  amount  of 
the  a.verage  inventory.  Therefore: 

$333,333.33  -T-  2  =  $166,666.67,  the  average  inventory. 

Hence,  the  merchant  could  make  the  same  amount  of  gross  profits  with  an 
investment  $33,333.33  smaller  than  that  required  under  his  old  policy. 

Problems 

1.  A  rate  of  mark-up  of  30%  results  in  2  turnovers  of  an  average  inventory 
of  $30,000.     If  the  expenses  of  conducting  the  business  are  $8,000,  what  is  the 
net  profit? 

2.  A  rate  of  mark-up  of  20%  results  in  3  turnovers  of  an  average  inventory 
of  $30,000.     If  expenses  remain  at  $8,000,  what  is  the  net  profit? 

3.  The  Cost  of  sales  in  Department  B  was  $42,000.     The  average  inventory 
was  $12,000.     What  was  the  number  of  turnovers? 

4.  A  merchant's  sales  amounted  to  $42.000.     His  average  inventory  was 
$10,000,  and  the  average  rate  of  mark-up  was  40%.     Find  the  number  of 
turnovers. 


*  "Mark-up,"  as  used  in  this  text,  refers  to  the  addition  made  to  the  cost   jf 
merchandise  to  produce  the  selling  price. 


150  INVENTORIES 

6.  Commodity  X,  with  a  rate  of  mark-up  of  40%,  had  a  turnover  of  2.  With 
a  rate  of  mark-up  of  30%,  it  had  a  turnover  of  3.  If  prior  sales  were  $56,000, 
find  the  sales  and  the  increase  in  gross  profit  with  the  30%  rate  of  mark-up. 

6.  A  rate  of  mark-up  of  35  %  results  in  a  turnover  of  2  and  in  sales  amounting 
to  $540,000.  A  rate  of  mark-up  of  20  %  results  in  a  turnover  of  4.  How  muck 
less  capital  under  the  latter  plan  is  required  to  make  as  much  profit  as  under  the 
former  plan? 

7.*  On  January  1,  a  concern  dealing  in  a  single  commodity  had  an  inventory 
of  merchandise  which  cost  $20,000.  The  goods  were  marked  to  sell  at  125% 
of  cost,  and  all  subsequent  purchases  during  the  six  months  ending  June  30 
were  marked  at  the  same  rate.  The  selling  price  of  the  inventory  at  June  30 
was  $24,000.  Purchases  and  sales  by  months  were: 

Purchases  Sales 
(Cost)        (Selling  Price) 

January     $  8,000  $  9,000 

February             9,000  9,500 

March 14,000  12,000 

April 16,000  18,000 

May....                  .              ..                 .        13,000  22,000 

June 10,000  18,000 

(a)  Compute  estimated  inventories  at  cost  price  at  the  end  of  each  of  the 
six  months. 

(b)  Compute  the  rate  of  turnover  for  the  six  months'  period,  using  (1)  the 
January  1  and  June  30  inventories;  (2)  ail  the  inventories. 

(c)  State  which  method  gives  the  more  accurate  results. 

Per  cent  of  mark-down  to  net  cost.  If  an  item  costs  $1  and 
is  marked  $1.25,  in  order  to  sell  the  item  for  cost  the  price  must  be 
reduced  25 i.  The  marked  price  is  the  base  when  prices  are 
reduced.  25  jS  is  £  of  $1.25.  £  =  20%. 

An  item  costs  $1  and  is  marked  $1.50.     50^  reduction  is  -V  oJ 

$1.50,  or  33£%. 

Problems 

Calculate  the  per  cent  of  mark-down  for  each  of  the  following  items: 

Marked    Per  Cent  of  Mark-down 

Item                                          Cost  Price             to  Produce  Cost 

A $  2  00  $  2  50                 ... 

H                                             1  00  1.25 

(y                               .                  .35  .40 

7) 80  1  00 

E 15.00  25  00 . 

F 3.50  4  00                ....... 

6'     20  30                

H 5.00  7  00 

/ 08  .10 

J 2.00  4  00                

K 4000  7500 

L 12.00  18.00                


1  American  Institute  Examination. 


INVENTORIES 


151 


Computation  of  inventory  by  the  retail  method.  The  need  for 
frequent  inventories  has  led  many  department  stores  to  adopt  the 
"retail  method "  of  computing  inventories.  The  accuracy  of  the 
inventory  by  this  method  depends  upon  the  care  exercised  in 
recording  the  mark-ups  and  the  mark-downs  of  merchandise 
prices,  and  the  classification  of  merchandise  into  departments 
and  groups  and  sub-classes  within  the  departments.  In  addition 
to  the  usual  records  showing  sales  (at  selling  price  only),  records 
are  kept  which  show  the  opening  inventory  and  purchases  at 
cost  and  at  retail  (or  selling)  prices.  An  estimated  inventory  may 
be  prepared  from  such  records  in  the  following  manner. 

INVENTORY  COMPUTATION 

Cost          Retail 

Inventory,  beginning  of  period $  6,000     $     8,000 

Purchases  during  the  period,  including  freight 

and  cartage 74,000       1 1 1 ,200 

Totals $<XO,000     $Tl9,200 

%  Mark-on  =  $39,000  ~  $119,200  or  32.XS59%.)~ 


.        ..  104,200 

Inventory  at  retail     .  .  $   157)00 

Estimated  inventory  =  $15,000  -  ($15,000  X  32.XX59%)  =  $10,007". 

The  foregoing  illustration  does  not  take  into  consideration 
changes  in  selling  price  after  the  original  mark-up.  Price  changes 
must  be  dealt  with,  and  the  retail  mercantile  business  has  terms 
for  these  changes  that  are  not  generally  understood;  therefore,  to 
prevent  any  •  '•  .•  :•  •  '  •  •'•  j  the  following  diagram  is  presented 
and  the  terms  explained. 


Cost  Plus  Original  Mark-Up 


Original  Retail  (Selling  Price) 


Original  mark-up.     The  amount  by  which  the  original  retail 
Erice  of  an  article  exceeds  the  cost  is  the  original  mark-up. 


152  INVENTORIES 

Additional  mark-up.  An  amount  that  increases  the  original 
retail  price  is  an  additional  mark-up. 

Additional  mark-up  cancellation.  A  reduction  in  the  additional 
mark-up  is  an  additional  mark-up  cancellation,  and  the  amount 
cannot  exceed  the  amount  of  the  additional  mark-up. 

Net  mark-up.  The  sum  of  additional  mark-ups  minus  the  sum 
of  additional  mark-up  cancellations  is  the  net  mark-up. 

Mark-downs.  Deductions  from  the  original  retail  price  to 
establish  a  new  but  lower  retail  price  are  mark-downs. 

Mark-down  cancellations.  A  reduction  of  the  amount  of  a 
mark-down  is  a  mark-down  cancellation.  Mark-down  cancella- 
tions cannot  exceed  the  total  mark-clowns.  It  is  evident  that  the 
retail  price  of  merchandise  is  increased  when  the  mark-down  is 
reduced,  but  such  an  increase  is  not  to  be  considered  as  an  addi- 
tional mark-up. 

Net  mark-down.  The  difference  between  the  sum  of  the  mark- 
downs  and  the  sum  of  the  mark-down  cancellations  is  the  net 
mark-down. 

Mark-on.  The  difference  between  cost  and  the  original  retail 
plus  the  net  mark-up  is  the  mark-on. 

To  illustrate  the  terms,  let  the  following  transactions  be 
assumed. 


INVENTORIES 


153 


Cost  of  Article 
$1.00 


Original 
Mark-Up 


Original  Retail  (or  Selling)  Price 
$1.00  -f  .50  =  $1. 50     _ 

$1.50 


Addn'l 
Mark-Up 


1st  Adjusted  Retail  Price 

$1.50  -f  -25  =  $1.75 


2nd  Adjusted  Retail  Price 

$1.75  -  .10  -  $1.05 


3rd  Adjusted  Retail  Price 

$1.65  -  .15  =  $1.50 


4th  Adjusted  Retail  Price 
_  $1^.50^-^.1^=  $1.35 

5th  Adjusted  Retail  Price 
$1.35  -  .35  =  $1.00 


$1.00 


e 
25* 


6th  Adjusted  Retail  Price 
$1.00  +  .25  =  $1.25 


a.  Additional  mark-up 
cancellation 
Net  mark-up  =  15^ 
Mark-on          =  65^ 


b.  Additional  mark-up 
cancellation 

Net  mark-up  =    0 
Mark-on         =50^ 

c.  Mark-down 


d.  Mark-down 


e.  Mark-down  cancellation 

Net  mark-down  = 
.15  +  .35  -  .25  =  .25 


Determining  the  ratio  of  cost  to  retail.  In  determining  the  ratio 
of  cost  to  retail,  it  is  customary  to  include  additional  mark-ups, 
and  additional  mark-up  cancellations  hut  to  exclude  mark-downs 
and  mark-down  cancellations.  To  illustrate,  let  us  assume  the 
following  facts : 

Inventory  at  beginning  of  month: 

Cost $30,000.00 

Retail 43,000  00 

Purchases : 

Cost 46,000  00 

Retail 55,000.00 

Returned  purchases: 


154 


INVENTORIES 


Cost 1,000  00 

Retail 1,50000 

Additional  mark-ups       5,500  00 

Additional  mark-up  cancellations 2,000  00 

Mark-downs         6,000  00 

Mark-down  cancellations     1,000  00 

Sales  at  retail 71,000.00 

Compute  the  inventory  by  the  retail  method. 

Solution 

Cost  Retail 

Inventory $30,000  00  $  43,000  00 

Purchases 46,000  00  55,000  00 

$76,000 "(JO  $  98,000  00 

Deduct:  Returned  purchases  1,000  00 1,500  00 

$75,000.00  $"90,500*00 

Additional  mark-ups  less  cancellations  thereof 3,500  00 

$100,000~00 

$100,000  -  $75,000  =  $25,000. 
$25,000  ~  $100,000  =  25%. 

Mark-downs  less  mark-down  cancellations 5,000  00 

$  95,00(fOO 

Sales  at  retail __71_,000_  00 

End-of-month  inventory  at  retail  value $  24,000 . 00 

$24,000  X  25%  =  $6,000. 
$24,000  -  $6,000  =  $18,000,  the  cost  value  of  the  inventory. 

Problems 

1.  From  the  records  kept  for  Department  B,  the  following  information 


obtained: 


Cost 


Inventory  at  Beginning  of  Month $15,000  00 

Purchases 36,00000 

Returned  Purchases       500.00 

Additional  Mark-Dps 

Additional  Mark-Up  Cancellations 

Sales '. 


Retail 

$25,000  00 

54,000  00 

700  00 

2,000  00 

1,000  00 

60,000  00 


Calculate  by  the  retail  method  of  inventory  the  cost  of  the  book  inventory 
at  the  end  of  the  month. 

2.*  In  a  certain  department  of  a  large  dry-goods  house,  the  purchases  for 
one  year  were  $30,000.  They  were  in  the  first  place  marked  up  for  selling  pur- 
poses to  $45,000.  Later,  additional  mark-ups  amounting  to  $2,000  were  made, 
and  mark-downs  aggregating  $5,000  were  also  recorded.  At  the  end  of  the 
fiscal  period  there  were  found  to  be  on  hand  goods  of  a  marked  selling  value  of 
$10,000.  State  how  you  would  arrive  at  their  inventory  value  for  the  purpose 
of  closing  the  books,  and  calculate  the  amount.  Explain  fully. 

*  American  Institute  Examination. 


CHAPTER  18 
Gross  Profit  Computations 

Gross  profit.  The  gross  profit  represents  the  margin  between 
the  sales  and  the  cost  of  goods  sold,  and  when  expressed  as  a  per 
cent  of  sales  indicates  to  one  who  is  familiar  with  trade  practice 
whether  a  sufficient  margin  of  profit  is  being  made.  Use  of  the  per 
cent  of  gross  profit  to  check  the  correctness  of  the  value  set  upon 
the  inventory  is  called  the  gross  profit  test  of  inventory. 

Rate  per  cent  of  gross  profit.  The  gross  profit  test  is  based 
on  the  supposition  that  in  normal  times  and  under  normal  condi- 
tions, any  business  will  produce  approximately  the  same  per  cent 
of  gross  profit  on  sales  in  any  one  period  of  time  as  in  any  other 
corresponding  period  of  time. 

Procedure.  Statements  of  the  gross  profit  and  sales  for  each 
of  several  prior  periods  should  be  obtained.  The  gross  profit  for 
any  one  period  divided  by  the  sales  for  the  same  period  gives  the 
rate  of  gross  profit  for  that  period,  based  on  sales.  Disregard  any 
per  cent  that  is  abnormal.  Add  the  remaining  per  cents,  and 
divide  by  the  number  added.  The  quotient  is  the  average  per 
cent  of  gross  profit  in  prior  periods. 

Uses.  The  per  cent  of  gross  profit  may  be  used  in  two  ways: 
first,  to  prove  inventories;  and  second,  to  compute  the  estimated 
inventory  when  it  is  impossible  or  impracticable  to  take  a  physical 
inventory. 

Example 

Assume  that  the  average  gross  profit  for  the  past  five  years  has  been  40% 
of  sales,  and  that  an  audit  of  the  books  shows  that  the  inventory,  taken  prior 
to  the  beginning  of  the  audit,  and  valued  at  $100,000,  seems  smaller  than  it 
should  be,  while  the  previous  inventory  and  purchases  amounted  to  $400,000. 
The  sales  for  the  period  are  $400,000.  Show  by  comparative  statement  the 
possibility  of  error. 

Solution 

In  the  following  set-up,  both  the  average  and  the  current  per  cents  and 
results  are  shown.  As  the  profit  in  prior  years  has  been  40%  of  sales,  the  cost 
of  goods  sold  has  been  60%  of  sales.  60%  of  $400,000  (sales)  =  $240,000,  cost 
of  sales. 

155 


156 


GROSS  PROFIT  COMPUTATIONS 


Sales 

Cost  of  Sales 
First  of  year  inven- 
tory and  purchases  $400,000 
Less:  Current  inv't. .    J00,000 

Gross  profit 


CURRENT  YEAR 
Actual       Currtn* 
Amounts    Per  Cent 
$400,000       100 


CURRENT  YEAR  IN  TERMS 
OF  AN  AVERAGE  YEAR 
Test  Average 

Amounts         Per  Cent 
$400,000  100 


300,000 


__ 

25% 


J40,000 
$160,000 


60 

40  < 


If  the  sales  are  correct,  the  cost  of  sales  is  $60,000  too  high,  unless  the  rate 
has  really  changed.  This  discrepancy  may  be  caused  by  any  of  the  following:  the 
volume  of  sales  may  be  incorrectly  stated;  the  current  inventory  may  be  errone- 
ous, and  the  cost  of  sales  affected  thereby;  or  there  may  be  an  abnormal  increase 
in  the  cost  of  merchandise  purchased,  when  compared  with  the  5-year  average. 
The  accountant  should  determine  the  reason  for  the  discrepancy. 

Cost  of  goods  sold.  The  average  rate  per  cent  of  gross  profit, 
applied  to  the  sales  for  the  current  period,  will  give  the  estimated 
gross  profit  for  the  current  period.  Deduction  of  the  estimated 
gross  profit  from  the  sales  gives  the  estimated  cost  of  goods  sold. 
This  procedure  may  be  reduced  to  a  formula  as  follows : 

AVERAGE  FOR  PRIOR  PERIODS 

1.  Sales  —  Cost  of  sales  =  Gross  profit. 

2.  Gross  profit  -5-  Sales  =  Per  cent  of  gross  profit  (based  on  sales). 

APPLICATION  TO  CURRENT  PERIOD 

3.  Sales  X  Per  cent  of  gross  profit  (prior  periods)  =  Estimated  gross  profit. 

4.  Sales  —  Estimated  gross  profit  (current  period)  =  Estimated  cost  of  sales. 


Example 

Sales 

First  period $400,000 

Second  period 450,000 

Third  period 350,000 

Fourth  period 100,000 


Cost  Gross 

of  Sales  Profit 

$300,000  $100,000 

340,000  110,DOO 

260,000  90,000 


What  was  the  cost  of  sales  during  the  fourth  period? 

S'olution 
AVERAGE  FOR  PRIOR  PERIODS 

First  period,  $100,000  ^  $400,000  -  25.00% 
Second  period,  110,000  +  450,000  =  24  44% 
Third  period,  90,000  -J-  350,000  =  25.71% 

75.15% 

75%  "5-  3  =  25%,  the  average  rate  of  gross  profit. 


GROSS  PROFIT  COMPUTATIONS  157 

APPLICATION  TO  CURRENT  PERIOD 

$100,000  X  25%  =  $25,000,  estimated  gross  profit. 
$100,000  -  $25,000  =  $75,000,  estimated  cost  of  sales. 

Rate  per  cent  of  cost  of  sales.  If  the  rate  of  profit  lias  been 
based  on  cost  price  instead  of  on  selling  price,  the  cost  of  sales  may 
be  tested  by  the  following  computations : 

AVERAGE  FOR  PRIOR  PERIODS 

1 .  Sales  —  Cost  of  sales  -  Gross  profit. 

2.  dross  profit  -f-  Cost  of  sates  —  Per  cent  of  gross  profit  (based  on  cost  of  sates) 

APPLICATION  TO  CURRENT  PERIOD 

3.  Sales  -r-  (100%  -f-  Per  cent  of  gross  profit)  =  Cost  of  sales. 

Example 

Sales  Cost  of  Sales 

First  period $400,000  $300,000 

Second  period 450,000  340,000 

Third  period 350,000  260,000 

Fourth  period 100,000 

What  was  the  cost  of  saleh  for  the  last  period? 

Solution 
AVERAGE  FOR  PRIOR  PERIODS 

First  period,  $100,000  4-  $300,000  =  33  33% 
Second  period,  110,000-5-  340,000=  3235% 
Third  period,  90,000  -r  260,000  =  _34_61_% 

100729% 

100%  4-  3  =  33^%,  average  per  cent  of  gross  profit. 
APPLICATION  TO  CURRENT  PERIOD 

$100,000  -f-  1.33i  (1  +  .33i)  -  $75,000,  cost  of  sales. 
$100,000  -  $75,000  =  $25,000,  gross  profit. 

It  follows  that  if  the  cost  of  sales  can  be  found,  any  element 
(inventory  at  beginning  of  period,  purchases,  closing  inventory, 
and  so  forth)  which  goes  to  make  up  the  cost  of  sales  can  be  found, 
provided  the  other  elements  of  the  costs  are  given. 

Fire  losses.  Insurance  companies  are  generally  willing  to 
settle  inventory  losses  resulting  from  fire  on  the  basis  of  values 
determined  by  the  gross  profit  method. 

Example 

The  insurance  company  agrees  that  the  following  facts  are  to  be  the  basis 
of  its  reimbursement  to  the  insured  for  his  fire  losses: 

Average  gross  profit  for  4  years,  40%  of  sales. 
Sales  for  this  period  to  date  of  fire,  $50,000. 
Cost  of  goods  available  for  sale,  $300,000. 


158  GROSS  PROFIT  COMPUTATIONS 

Solution 

$50,000  (sales)  X  40%  (rate  of  gross  profit)  =  $20,000,  gross  profit. 

$50,000  (sales)  -  $20,000  (gross  profit)  =  $30,000,  cost  of  goods  sold. 
$300,000  (goods  available  for  sale)  -  $30,000 

(cost  of  goods  sold)  =  $270,000,  estimated  inventory 

at  date  of  fire. 

Use  of  gross  profit  test  in  verification  of  taxpayer's  inventory. 

Assessors  make  use  of  the  gross  profit  test  to  determine  the  approxi- 
mate inventory  and  to  check  the  item  of  inventory  in  the  schedule 
filed  by  the  taxpayer,  since  assessment  dates  seldom  coincide  with 
closing  dates.  The  following  forms  have  been  given  to  the  tax- 
payer to  fill  out,  the  date  of  assessment  being  May  1. 

FOR  MERCHANTS 

1.  Book  value  of  last  inventory  of  stock  of  merchandise 

2.  Add  purchases  since  last  inventory  to  May  1 

3.  Add  in-freight  and  cartage  paid  since  last  inventory  to  May  1 

4.  Total  of  above  three  items 

Deduct  from  above  total  net  result  of  following  two  items: 

5.  Amount  of  net  sales  from  date  of  last  inventory  to 

May  1    

6.  Less.  Gross  profit  on  sales  estimated  at % 

(Previous  near  %  may  be  used  where  actual  %  is 
unknown.) 

7.  Net  inventory  of  merchandise  on  May  1  (Item  4  less  Item  6)   


FOR  MANUFACTURERS 

1.  Book  value  of  raw  materials,  finished  goods,  and  work-in- 

process  at  last  inventory.     Date 

2.  Acid  purchases  of  raw  materials  and  finished  goods  since  last 

inventory  to  May  1 

3.  Add  amount  paid  for  in-freight  and  cartage  from  last  inventory 

to  May  1    

4.  Add  amount  paid  for  labor  and  manufacturing  expenses  from 

last  inventory  to  May  1 

5.  Total  of  above  four  items 

Deduct  from  above  total  the  net  result  of  the  following  two  items: 

6.  Amount  of  net  sales  from  date  of  last  inventory  to 

May  1 

7.  Less:  Gross  profit  on  sales  estimated  at % 

(Previous  year  %  may  be  used  where  actual  %  is  not  known.) 

8.  Net  value  of  raw  materials,  goods-in-process,   and  finished 

goods  on  May  1  (Item  5  —  Item  7) 


Problems 

1.  From  the  figures  in  the  following  tabulation,  calculate  the  per  cent  of  gross 
profit  for  each  year,  and  by  means  of  the  average  per  cent  of  gross  profit  calculate 
the  inventory  at  the  end  of  the  first  half  of  the  fifth  year. 


GROSS  PROFIT  COMPUTATIONS 


159 


Sales 

First  year $120,000 

Second  year  .  .  .      150,000 

Third  year 165,000 

Fourth  year  .         ...      180,000 
Fifth  year  (6  mo.)...       95,000 


Opening 

Purchases  Inventory 
$  90,000 

100,000  10,000 

110,000  12,000 

122,000  10,000 

62,000  11,000 


Closing 

Inventory 

$10,000 

12,000 

10,000 

11,000 


Per  Cent  of 
Gross  Profit 


2.  From  the  following  facts,  find  the  inventory  as  of  December  31: 

Inventory,  January  15  following,  $16,578.50. 

Sales,  December  31  to  January  15,  $2,890.00. 

$765  of  the  above  sales  shipped  and  invoiced  before  December  31. 

Purchases,  December  31  to  January  15,  at  cost,  $1,256.50. 

Average  gross  profit,  25  %  of  cost. 

3.  The  average  gross  profit  of  the  X.  Company  for  the  past  three  years  has 
been  45%  of  the  sales.     During  the  fourth  year  the  sales  amounted  to  $159.500. 
Goods  were  purchased  to  the  amount  of  $105,000.     Returned  purchases  totaled 
$5,000  for  the  period.     Freight  paid  on  purchases  was  $6,000.     The  inventory 
at  the  beginning  of  the  period  was  $40.000.     Current  market  prices  are  10% 
above  the  purchase  prices  for  the  year.     Find  the  cost  of  replacing  the  goods  at 
the  end  of  the  year. 

4.  On  April  30,  the  board  of  managers  of  the  Ames  Mercantile  Company 
removed  the  superintendent  on  the  general  suspicion  that  his  books  misrepre- 
sented the  true  financial  condition  of  the  business.     Prepare  a  statement  showing 
the  nature  and  the  probable  extent  of  the  misrepresentations;  also  an  approxi- 
mate statement  of  income  and  profit  and  loss  for  the  four  months  ending  April  30. 

The  following  is  a  trial  balance  taken  from  the  books,  April  30: 


Capital  Stock     

Furniture  and  Fixtures       

Inventory,  January  1         

Cash  

Accounts  Payable        

Accounts  Receivable 

Loans  Payable     .  

Sales         

Purchases        .  .        .  .... 

Salaries,  Salesmen  

Advertising       .         .  .  .... 

Salaries,  Office 

Rent 

Interest     

Insurance,  January  1  to  December  31 

Stationery  and  Printing 

Reserve  for  Depreciation  of  Furniture  &  Fix- 
tures   

Surplus,  January  1 


$  10,000 

128,600 

15,450 

24,600 


40,700 

2,200 

1,650 

1,100 

400 

200 

999 

105 


$  75,000 


39,000 

10,000 
51,000 


2,710 

48,294 

S226J004  $226,004 


160  GROSS  PROFIT  COMPUTATIONS 

An  analysis  of  the  Purchases,  Sales,  and  Inventory  accounts  revealed  the 
following: 

Opening  Closing 

Purchases  Hales           Inv't           Inv't 

First  year $122,000  $153,750     $101,000  $100,000 

Second  year      123,000  153,170       100,000       102,000 

Third  year     121,000  154,722       102,000       128,600 

5.*  The  hooks  of  a  concern  recently  burned  out  contained  evidence  of  pur- 
chases, including  inventory,  to  the  amount  of  $200,000,  and  sales  of  $40,800, 
since  the  last  closing.  Upon  investigation,  however,  the  auditor  ascertained 
that  a  sale  of  merchandise  had  been  made  just  prior  to  the  fire,  and  not  recorded 
in  the  books,  at  an  advance  of  two-fifths  over  cost  less  a  10%  cash  discount;  the 
profit  on  the  transaction  was  $31,928.  The  past  history  of  the  business  indi- 
cated an  average  gross  profit  of  50%  on  cost  of  goods  sold. 

(a)  What  amount  should  be  claimed  as  fire  loss? 

(6)  What  rate  of  gross  profit  do  the  transactions  finally  yield? 

6.f  The  store  and  stock  of  the  Diamond  Jewelry  Company  was  destroyed  by 
fire  on  November  1.  The  safe  was  opened,  and  the  books  were  recovered  intact. 
The  trial  balance  taken  off  was  as  follows: 

Cash  in  Bank .  ....   $     1,000 

Accounts  Receivable                  .        .  10,000 

Accounts  Payable  ......  $  30,000 

Merchandise  Purchases 90,000 

Furniture  and  Fixtures 7,500 

Sales 110,000 

General  Expense 18,000 

Insurance 1 ,500 

Salaries 5,500 

Real  Estate— Store  Lot 50,000 

Store  Building 35,000 

Capital  Stock 50,000 

Surplus 28,500 

$218,500  $218,500 

The  average  gross  profit  as  shown  by  the  books  and  accepted  by  the  insurance 
companies  was  40%  of  sales.  The  insurance  adjuster  agreed  to  pay  75%  of 
the  book  value  of  furniture  and  fixtures,  90%  of  the  book  value  of  the  store 
building,  and  the  entire  loss  on  merchandise  stock. 

Draft  journal  entries  to  include  the  account  against  the  insurance  companies. 

Installment  sales  of  personal  property.  The  large  increase  in 
sales  of  personal  property  on  the  installment  plan,  and  the  option 
that  the  government  allows  a  taxpayer  coming  within  the  defini- 
tion of  an  installment  dealer  to  return  his  gross  income  from  sales 
on  the  installment  basis,  are  indications  of  the  growing  importance 
of  this  subject. 

The  installment  plan  of  selling  was  devised  for  the  purpose  of 

*  American  Institute  Examination, 
t  C.  P.  A.,  Oklahoma. 


GROSS  PROFIT  COMPUTATIONS 


161 


stimulating  sales,  whereas  the  installment  basis  of  reporting  income 
was  devised  for  the  purpose  of  deferring  from  year  to  year  the 
income  to  be  realized  from  installment  sales,  with  a  view  to  the 
possible  effect  that  this  deferment  might  have  upon  the  amount  of 
federal  income  tax  to  be  paid.  It  is,  of  course,  essential  that  the 
latest  Federal  Income  Tax  Law  be  observed. 

Computation  of  gross  profit.  The  gross  profit  to  be  reported 
may  be  ascertained  by  taking  that  proportion  of  the  total  cash 
collections  received  in  the  taxable  year  from  installment  sales 
(such  collections  being  allocated  to  the  year  against  whose  sales 
they  apply)  which  the  annual  gross  profit  to  be  realized  on  the 
total  installment  sales  made  during  each  year  bears  to  the  gross 
contract  price  of  all  such  sales  made  during  that  particular  year. 

Example 

The  books  of  the  Model  Credit  Company,  selling  merchandise  on  the  install' 
ment  plan,  show  the  following: 

First 
Year 

Sales $  80,000 

Cost  of  Sales 

Inventory  (old) $  45,000 

Purchases _55>000 

$100,000 
Less:  Inventory  (new)  . .       40,000 

Cost  of  sales $150,000 

Gross  profit $  20,000     ^          ^ 

Collections  were  made  in  the  fourth  year  on  each  year's  contracts  as  follows: 

First       Second       Third        Fourth 

Year         Year         Year  Year 

$1,600      $4,800      $25,000      $70,000 

What  was  the  gross  profit  to  be  reported  for  the  fourth  year? 


Second 

Third 

Fourth 

Year 

Year 

Year 

$110,000 

$130,000 

$90,000 

$  40,000 
75,000 

$  50,000 
90,000 

$48,000 
50,000 

$fl  5,000 
50,000 

$140,000 
48,000 

$98,000 
40,000 

$"G57000 

$  92,000 

$58,000 

$45,000 

$  38,000 

$32,000 

Solution 

Per  Cent  of  Gross  Profit 

First  year,      $20,000  (gross  profit)  •*-  $  80,000  (sales) 
Second  year,    45,000       "         "       -4-    110,000      " 
Third  year,      38,000       "        "       +    130,000      " 
Fourth  year,    32,000       "        "       ^      90,000      " 

Profit  on  Collections  in  Fourth  Year 
Collected  on  first-year  contracts,  $  1,600  X  25.00% 


4,800  X  40.91% 
25,000  X  29.23% 


"  second-year   " 
"          "  third-year 

"  fourth-year    "            70,000  X  35.55%  = 
Gross  profit  realized  in  the  fourth  year. . , ,    


=  25.00% 
=  40  91% 
=  29.23% 
=  35.55% 


$     400  00 

1,963  68 

7,307.50 

24,855^.00 

$34,526718 


162  GROSS  PROFIT  COMPUTATIONS 

Reserve  lor  unearned  gross  profit.  The  gross  income  to  be 
realized  on  installment  sales  is  credited  to  "  Reserve  for  Unearned 
Gross  Profit/7  and  at  this  time  this  account  is  debited  with  the 
gross  profit  on  collections.  The  balance  of  the  account  represents 
gross  profit  on  installment  sales  contracts  remaining  unpaid  at  the 
date  of  closing. 

Example 

The  books  of  the  X.Y.Z.  Company,  selling  merchandise  on  the  installment 
plan,  show  the  following: 

First  Second  Third  Fourth 

Year  Year  Year  Year 

Sales $89,257  99     $111,825  86    $137,012  32     $97,912  26 

Gross  profits 29,962~89        48,068  37    ~38j28~63 39,168~71 

Collections  during  the 
fourth  year  on  each 
year's  accts 1,635.35  4,832  00  25,182. 14  69,927  92 

What  amount  should  be  ci  edited  to  Reserve  for  Unearned  Gross  Profit  to 
represent  deferred  income  for  the  fourth  year?  What  amount  should  be  debited 
to  Reserve  for  Unearned  Gross  Profit  to  represent  income  realized  from  the 
first,  second,  third,  and  fourth  years'  collections  received  in  the  fourth  year? 

Solution 
(a)  Per  Cent  of  Gross  Profit 

First  year $29,962.89  ^  $  89,257.99  =  33  57% 

Second  year 48,068.37  -*•     11 1,825.86  -  42  98% 

Third  year 38,128.63  -;-    137,012.32  =  27  83% 

Fourth  year 39,168.71  4-      97,912.26  =  40  00% 

Profit  on  Collections 

First-year  accounts $  1,635.35  X  33.57%         =  $      548  99 

Second-year  accounts. . . .       4,832.00  X  42.98%         =      2,076  79 

Third-year  accounts 25,182.14  X  27.83%          =       7,008  19 

Fourth-year  accounts. . . .     69,927.92  X  40.00%         =    27,971 . 17 

Gross  profit  realized  in  4th  yr $37,605^14 

Journal  entries 
Installment  Sales  Contracts $  97,912.26 

Cost  of  Sales $  58,743  55 

Reserve  for  Unearned  Gross  Profit. . .  39,168  71 

Cash $101,577.41 

Installment  Sales  Contracts $101,577.41 

Reserve  for  Unearned  Gross  Profit.*. ...  $  37,605. 14 

Realized  Gross  Profit  on  Installment 
Sales $  37,605. 14 

Bad  debts.  The  bad  debts  written  off  during  the  year  should 
be  allocated  by  years,  and  a  charge  should  be  made  to  Reserve 
for  Unearned  Gross  Profit  for  the  percentage  of  gross  profit  in 
each  year's  write-off,  and  to  Profit  and  Loss  (Bad  Debts)  for  the 
remainder,  the  entire  credit^  being  made  to  Installment  Sales 
Contracts. 


GROSS  PROFIT  COMPUTATIONS  163 

Example 

During  the  fourth  year,  bad  accounts  were  written  off  as  follows: 

First-Yr.  Accts.       Second-Yr.  Accts.        Third-Yr.  Accis.       Fourth-Yr.  Accts. 
$67  65  $141  05  $65  62  $126  25 

What  amount  should  be  charged  to  these  accounts:  Profit  and  Loss  (Bad 
Debts),  and  Reserve  for  Unearned  Gross  Profit? 

Solution 

Unrealized 
Profit         Remainder 

$  67.65  X  33.57%  $  22  71         $  44  94 

141.05X42.98%  60.62  cSO  43 

65.62X27.83%  1826  47.36 

126.25  X  40.00%  .  .       50  50  75  75 

$152.09         $248  48 

Reserve  for  Unearned  Gross  Profit  $152  00 

Profit  and  Loss  (Had  Debts)  .      248  48 

Installment  Sales  Contracts       $400  57 

Problems 

1.  The  X.Y.Z.  Company's  books  for  the  5th  year  showed: 

Sales                                     ....                  .  $128,642  60 

Gross  profit  42,975   12 

Collections  were  made  in  the  fifth  year  on  each  year's  contracts  as  follows: 

1st  Yr.  2nd  Yr.  3rd  Yr.  4th  Yr.  5th  Yr. 

$230  60  $1,590  31  $9,326  80          $21,256  30         $82,327  58 

Calculate:  (a)  the  per  cent  of  gross  profit  for  the  fifth  year;  (b)  the  amount 
to  be  credited  to  Reserve  for  Unearned  Gross  Profit;  (c)  the  amount  to  be  debited 
to  Reserve  for  Unearned  Gross  Profit.  Use  the  rates  given  in  the  solution  on 
page  162  for  the  first  four  years. 

2.  The  analysis  of  bad  debts  written  off  during  tire  5th  year  was: 

Ist-Yr.Acct.      2nd-Yr.Acct.      3rd- Yr.  Acct.      4th-Yr.  Acct.      5th-Yr.  Acct. 
$8  35  $209  75  $910.40  $150  80  $470  62 

What  amounts  should  be  charged  to  Reserve  for  Unearned  Gross  Profit  and 
to  Profit  and  Loss,  respectively? 

3.  Results  for  the  6th  year: 

Sales $140,695  39 

Gross  profit          54,541   07 

Collections  were  made  in  the  sixth  year  on  each  year's  contracts  as  follows: 

1st  Yr.          2nd  Yr.          3rd  Yr.  4th  Yr.  5th  Yr.  6th  Yr. 

$62.70          $492.54        $2,798.30        $4,689.30        $2,657  80       $90,275.89 

(a)  Calculate  the  per  cent  of  gross  profit  for  the  6th  year. 

(b)  Calculate  for  the  6th  year  the  gross  profit  on  collections  made. 

4.  Accounts  receivable  were  written  off  as  follows: 

1st  Yr.         2nd  Yr.          3rd  Yr.  4th  Yr.  5th  Yr.  6th  Yr. 

$52.83          $31.50          $51.10  $150.00          $163.82          $108.28 


164  GROSS  PROFIT  COMPUTATIONS 

Compute  the  charges  to  be  made  to  Profit  and  Loss  (Bad  Debts)  and  to 
Reserve  for  Unearned  Gross  Profit. 

5.*  The  "A  &  B"  Company  is  engaged  in  the  business  of  retailing  musical 
merchandise.  The  majority  of  the  sales  consist  of  installment  sales  of  pianos 
and  talking  machines,  on  which  the  initial  payment  is  less  than  25%  of  the  sales 
price  and  the  balance  is  payable  in  monthly  installments  over  a  period  of  three 
to  five  years.  The  company  was  incorporated  and  began  business  on  January  1. 
The  following  schedules  are  submitted  on  the  various  classes  of  merchandise: 

SALES 

Piano  Install-  Machine  Install-  Other 

ment  Sales             ment  Sales  Mdse  Sales 

First  year     $148,650  00           $92,475  00  $38,337  60 

Second  year     163,520  00             88,535  00  39,543  50 

Third  year 180,400  00              94,256.00  40,731 . 15 

PURCHASES 

First  year 106,322  37  67,432. 18  27,108  88 

Second  year 120,987  41  55,116  92  27,224  35 

Third  year 140,125  25  60,013.22  27,469  33 

INVENTORIES 

First  year 20,103  14  10,248  31  8,323  64 

Second  year 32,105  86  15,012  83  15,299  41 

Third  year 39,29444  18,14477  13,52131 

Attention  is  called  to  the  fact  that  "Other  Merchandise  Sales"  are  sales  for 
cash,  or  credit  sales  other  than  installment  sales. 

No  adjustments  to  Deferred  Income  account  are  made  until  the  end  of  the 
year.  Additions  to  this  income  are  made  at  the  end  of  the  year  on  the  basis 
of  the  balance  due  on  the  current  year's  installment  sales,  and  deductions  are 
made  on  the  basis  of  cash  received  during  the  current  year  on  installment  sales 
of  previous  years.  On  December  31  of  the  third  year,  the  unpaid  balances  on 
third-year  piano  installment  sales  amount  to  $110,425.50,  and  on  third-year 
machine  installment  sales  to  $60,475.00 — exclusive  of  accrued  interest.  The 
following  amounts  were  received  during  the  third  year  on  installment  sales  of 
previous  years: 

On  first-year  piano  installment  sales $30,285.00 

On  second-year  piano  installment  sales     42,413  00 

On  first-year  machine  installment  sales 25,386 . 00 

On  second-year  machine  installment  sales 26,285.00 

The  above  amounts  are  exclusive  of  interest,  which  is  credited  direct  to 
interest  revenue. 

Fractional  percentages  may  be  disregarded  in  the  computation  of  ratios — 
over  -J-  of  1  %  should  be  added,  and  less  than  \  of  1  %  should  be  dropped. 

On  first-year  machine  installment  sales,  uncollectible  balances  amounting  to 
$399.00  were  charged  on  the  books  of  the  company  to  expense,  and  credited  to 
installment  sales  contracts. 

Federal  income  taxes  paid  in  the  third  year  were  charged  to  surplus. 


*  C.  P.  A.,  Michigan. 


GROSS  PROFIT  COMPUTATIONS 


165 


Depreciation  is  calculated  at  the  following  rates: 

Buildings 2%    Furniture  and  Fixtures ...   10% 

Auto  Trucks 25% 

The  following  is  a  copy  of  the  trial  balance  as  of  December  31,  end  of  thiid 
year,  before  closing  and  before  apportionment  of  deferred  income  on  installment 
sales : 

Cash $  15,327.48 

Notes  Receivable .  2,000.00 

Accounts  Receivable                         .  20,842 . 1 1 

Installment  Sales  Contracts     .    .        ..  205,418.50 

Inventories ..  62,418.10 

War  Bonds 5,000.00 

Real  Estate 10,00000 

Buildings .  40,00000 

Furniture  and  Fixtures     .      .  4,500  00 

Auto  Trucks 3,000 . 00 

Notes  Payable $  50,000 . 00 

Accounts  Payable 13,458  25 

Deferred  Income  on  Installment  Sales  83,245  70 

Reserve  for  Depreciation,  Buildings  1,600  00 

Reserve  for  Depreciation,  Fur.  <fc  Fix  900  00 

Reserve  for  Depreciation,  Trucks     ...  1 ,500  00 

Capital  Stock .      ...  150,000 . 00 

Surplus 75,556.21 

Sales 315,387.15 

Piano  Rentals 1 ,785  00 

Interest  on  Installment  Sales                 . .  2,035  23 

Interest  on  War  Bonds 237  50 

Cash  Discounts  on  Purchases              .  . .  2,452.07 

Purchases 227,607.80 

Salaries,  Officers 14,000  00 

Salaries,  Store 8,10146 

Light  and  Heat 717.68 

Advertising 4,01571 

Truck  Expense ...  508.53 

Sundry  Store  Expense       2,239  17 

Salaries,  Office 2,020  00 

Traveling  Expense  ....            648  50 

Postage 472  30 

Telephone  and  Telegraph  441  40 

Insurance 1 ,309  06 

Real  Estate  and  Personal  Property  Taxes  2,029  69 

Bad  Debts,  Accounts  Receivable  709  66 

Bad  Debts  (first  year  machine  install- 
ment sales) 399  00 

Repairs,  Sundry 365. 68 

Donations 20000 

Cash  Discounts  on  Sales 444  48 

State  Franchise  Tax     187.80 

Capital  Stock  Tax 233.00 

Interest  Paid 3,000  00 

$698,157.11  $698,157.11 


166  GROSS  PROFIT  COMPUTATIONS 

You  are  asked  to  give:  (a)  the  net  taxable  income  (for  federal  tax  purposes) 
for  the  third  year;  (6)  a  balance  sheet  of  the  "A  &  B"  Company  as  of  January  1, 
beginning  of  fourth  year. 

Deferring  income;  its  effect  on  tax.  The  statement  was  made 
in  the  second  paragraph  of  this  subject  that  the  installment  basis 
of  accounting  defers  income  with  a  view  to  the  possible  effect  that 
deferment  may  have  on  the  amount  of  federal  income  tax  to  be 
paid.  Since  the  income  is  deferred,  the  tax  is  deferred  (not  saved). 

The  amount  of  profit  realized  and  to  be  realized  from  the  sales 
of  a  particular  year,  if  not  taxed  in  that  particular  year,  will  be 
taxed  eventually,  and  the  saving  of  tax  results  from  a  possible 
reduction  in  the  rate  of  tax  or  from  the  spreading  of  taxable  income 
over  several  years.  If  it  is  anticipated  that  the  rate  of  tax  will  be 
increased,  it  may  not  be  wise  to  defer  the  income. 

Second,  a  change  from  the  accrual  to  the  installment  basis 
results  in  double  taxation,  for  Section  44  (c)  of  the  Internal 
Revenue  Code  provides  as  follows:  "If  a  taxpayer  entitled  to  the 
benefits  of  subsection  (a)  elects  for  any  taxable  year  to  report 
his  net  income  on  the  installment  basis,  then  in  computing  his 
income  for  the  year  of  change  or  any  subsequent  year,  amounts 
actually  received  during  any  such  year  on  account  of  sales  or  other 
dispositions  of  property  made  in  any  prior  year  shall  not  be 
excluded/' 

The  amount  of  gross  income  which  may  be  deferred  on  install- 
ment sales  is  governed  by : 

(1)  The  terms  of  sale; 

(2)  Annual  increase,  if  any,  in  sales; 

(3)  Per  cent  of  year's  sales  collected  in  the  current  year;  and 

(4)  Fluctuation  of  gross  profits. 

Example 

Assume  the  terms  of  sale  to  be  10%  down,  and  10%  a  month;  the  annual 
increase  in  sales  to  be  $10,000;  the  per  cent  of  year's  sales  collected,  and  the 
sales  throughout  the  year,  to  be  uniform;  and  the  per  cent  of  gross  profit  to  be 
fixed. 

Gross  Profit 
Year  Sales        on  Sales 

First $50,000          30% 

Second 60,000          30% 

Third     70,000  30% 

Fourth 80,000          30% 

Fifth  90,000          30% 

Since  it  has  been  assumed  that  the  sales  are  uniform  throughout  the  year 
and  that  collections  are  met  promptly,  the  second  year's  business  may  be  analyzed 
as  follows: 


GROSS  PROFIT  COMPUTATIONS 


167 


Down  Payments 
Jan 10%  of  $  5,000 


Feb 10%  of 

Mar 10%  of 

Apr 10%  of 

May 10%  of 


of 
of 


June 10' 

July     10< 

Aug  10%  of 

Sept   10%  of 

Get     10%  of 

Nov 10%  of 

Dec  10%  of  _ 

Year's  sales $60,000 

J  )own  payments 

Install,  payments 

Total  payments 

Ratio  of  payments  to  sales:  $37,500  -r 


5,000 
5,000 
5,000 
5,000 
5,000 
5,000 
5,000 
5,000 
5,000 
5,000 
5,000 


Installment  Payments 
500 

500  10%  of  $  5,000  =  $     500 

500  10%  of  10,000  =  1,000 

500  10%  of  15,000  =  1,500 

500  10%  of  20,000  =  2,000 

500  10%  of  25,000  =  2,500 

500  10%  of  30,000  =  3,000 

500  10%  of  35,000  =  3,500 

500  10%  of  40,000=  4,000 

500  10%  of  45,000=  4,500 

500  10%  of  45,000  =  4,500 

500  10%  of  45,000=  4,500 


.  $  6,000 
.  31,500 
.  $37^00 
$60,000  =  62.5%. 


$31,500 


A  comparison  of  the  income  to  be  reported  on  the  accrual  basis 
and  on  the  installment  basis  may  be  made  as  follows: 

ACCRUAL  BASIS 


Sales  

Second 
Year 
.   $60,000 
30% 
.  .      1S.OOO 

Third 
Year 
$70,000 
30% 
21,000 

BASIS 

$22,500 
43,750 

Fourth 
Year 
$SO,()0() 
30% 
24,000 

$26,250 
50,000 

Fifth 
Year 
$90,000 
30 
27,000 

$30,000 
56,250 

Gross  profit  (  %)         
Gross  profit  ($)    

INSTALLMENT 

Collections  : 
Ist-year  accounts       $18,750 
2nd-year  accounts          37.500 

3rd-year  accounts 

4th-year  accounts 
5th-year  accounts     .    .  . 
Gross  income  to  be  reported: 
30  %  of  Ist-year  coll 
30  %  of  2nd-year  coll 

$  5,625 
.      11,250 

$  6,750 
13,125 

$  7,875 
15,000 

$  9,000 
16,875 

30  %  of  3rd-year  coll  

30  %  of  4th-year  coll  .  .  . 

30  %  of  5th-year  coll  

Total  income  reported  

.  .  $16,875 

$19,875 

$22,875 

$25,875 

Income  deferred                .    .    . 

.  .   $  1,125 

$  1,125 

$  1,125 

$  1,125 

It  may  be  observed  from  the  foregoing  analysis  that  with  an 
annual  increase  of  $10,000  in  sales,  and  with  a  constant  gross  profit 
ratio  of  30%,  the  amount  of  income  deferred  from  year  to  year  is 
$1,125. 

With  an  annual  increase  of  $20,000  in  sales,  and  other  cond> 


168  GROSS  PROFIT  COMPUTATIONS 

tions  the  same,  the  amount  of  income  deferred  would  be  $2,250 
(2  X  $1,125). 

Problems 

1.  Assume  the  terms  of  sale  to  be  10%  down  and  5%  a  month,  the  annual 
increase  in  sales  $10,000,  the  per  cent  of  year's  sales  collected  and  the  sales 
throughout  the  year  uniform,  and  the  per  cent  of  gross  profit  fixed. 

Gross  Profit 

Year  Sales  on  Sales 

First $50,000  30% 

Second 60,000          30% 

Third 70,000          30% 

Fourth       80,000  30% 

Fifth          90,000  30% 

Show  the  amount  of  income  deferred  when  the  installment  basis  is  used. 

2.  If  the  terms  of  payment  were  5%  down  and  5%  a  month,  and  other  con- 
ditions were  the  same  as  in  Problem  1,  what  would  be  the  amount  of  income 
deferred  each  year? 


CHAPTER  19 
Analysis  of  Statements 

Financial  and  operating  ratios.  An  analysis  of  the  financial 
and  the  operating  ratios  of  a  business  means  a  study  of  the  relation- 
ships that  are  expressed  in  the  statistics  presented.  Well-known 
and  commonly  used  ratios  are  those  of  expenses  and  earnings  to 
sales,  and  of  earnings  on  capital  employed.  Other  ratios,  relation- 
ships, and  turnovers  that  are  indicators  of  the  condition  of  a  busi- 
ness should  also  be  considered. 

A  summary  of  financial  and  operating  ratios,  relationships,  and 
turnovers  would  include  the  following: 

(1)  Ratio  of  costs  and  expenses  to  net  sales. 

(2)  Ratio  of  gross  profit  to  net  sales. 

(3)  Ratio  of  operating  profit  to  net  sales. 

(4)  Ratio  of  net  profit  to  net  sales. 

(5)  Ratio  of  operating  profit  to  total  capital  employed. 

(6)  Ratio  of  net  profit  to  net  worth. 

(7)  Earnings  on  common  stockholders'  investments. 

(8)  Working  capital  ratio. 

(9)  Sources  of  capital. 

(10)  Manner  in  which  capital  is  invested. 

(11)  Turnover  of  total  capital  employed. 

(12)  Turnover  of  inventories. 

(13)  Turnover  of  accounts  receivable. 

(14)  Turnover  of  fixed  property  investment. 

There«are  many  other  ratios  which  are  important  measures  of 
efficiency,  but  of  which  only  brief  mention  can  be  made  in  this 
chapter.  Depending  on  the  type  of  business  being  analyzed, 
these  other  ratios  might  include  the  labor  turnover,  the  unit  of 
output  per  operative,  the  average  wage  per  man,  the  average  wage 
per  hour,  and  other  statistics. 

Costs,  expenses,  and  profits.  Costs,  expenses,  and  profits 
should  be  expressed  as  per  cents  of  money  values  and,  where  possi- 
ble, should  be  expressed  in  terms  of  dollars  per  production  unit, 
such  as  the  ton,  pound,  yard,  or  gallon.  The  per  cents,  compared 
with  those  of  previous  years,  show  whether  sales  prices  have  been 

169 


170  ANALYSIS  OF  STATEMENTS 

adjusted  proportionately  to  cOvSts  of  production  and  distribution. 
The  unit  prices  supplement  the  per  cents  and  afford  a  direct 
comparison. 

Ratio  of  gross  profit  to  net  sales.  The  ratio  of  gross  profit  to 
net  sales  is  an  indication  of  the  spread  between  the  cost  of  pro- 
duction and  the  selling  price.  The  gross  profit  must  be  as  large 
as  possible,  for  out  of  it  must  come  the  expenses  of  selling,  adminis- 
tration, finance,  and  other  charges,  before  a  net  return  is  realized 
on  capital. 

Ratio  of  operating  profit  to  net  sales.  The  ratio  of  operating 
profit  to  net  sales  expresses  the  basic  relationship  between  profits 
and  sales.  Operating  profits  represent  the  gain  before  the  deduc- 
tion of  federal  taxes,  interest  on  borrowed  money,  and  extra- 
ordinary losses,  but  do  not  include  miscellaneous  income  not 
attributable  to  ordinary  operations. 

Ratio  of  net  profit  to  net  sales.  The  ratio  of  net  profit  to  net 
sales  indicates  the  margin  of  profit  on  the  selling  price.  The 
rapidity  of  stock  turnover,  and  the  capital  invested  in  accounts 
receivable,  in  inventory,  and  in  plant,  should  be  considered  with 
this  ratio. 

Ratio  of  operating  profit  to  total  capital  employed.  The  ratio 
of  operating  profit  to  total  capital  employed  forms  a  ready  basis  for 
a  comparison  of  the  operating  results  of  a  business  or  of  several 
plants  under  a  single  control.  Capital  employed  includes  plant, 
inventories,  accounts  receivable,  cash  balances,  and  so  forth, 
regardless  of  the  source  of  such  capital,  and  is  readily  determined 
by  referring  to  the  asset  side  of  the  balance  sheet. 

Ratio  of  net  profit  to  net  worth.  The  ratio  of  net  profit  to  net 
worth  expresses  the  measure  of  earnings  available  to  the  stock- 
holders or  proprietors,  and  is  the  final  indicator  of  the  success  or 
failure  of  any  business. 

Earnings  on  common  stockholders'  investments.  The  earn- 
ings on  common  stockholders'  investments  are  based  on  the 
stockholders'  share  of  the  net  profit,  in  relation  to  their  interest  in 
the  net  worth  of  the  business.  There  are  two  ways  in  which  these 
earnings  may  be  stated:  (a)  as  a  per  cent  of  the  amount  of  such 
investments;  and  (6)  in  dollars  earned  per  share  outstanding. 

Example 

The  following  profit  and  loss  statement,  together  with  certain  other  facts, 
is  presented  to  illustrate  items  1-7  in  the  summary  on  page  169.  The  numbers 
in  parentheses  refer  to  the  numbered  ratios  in  the  summary. 


ANALYSIS  OF  STATEMENTS  171 

BLANK  MERCANTILE  COMPANY 

PROFIT  AND  Loss  STATEMENT 

FOR  THE  TWELVE  MONTHS'  PERIOD  ENDED  DECEMBER  31,  19  —  . 
Sales: 

Gross  Sales  ...........  $693,004  .  10 

Less:  Sales  Rebates  and 

Allowances     ...  $        870.64 
Prepaid  Freight  .  .   ___  200.25 

^   1,070^89 

--    -   - 


^ 

Net  Sales  ...........  --    -   -      $691j933  21   1C0.00% 

Cost  of  Sales: 

Inventory,  beginning  of 

year  ..........  $107,278  46 

Purchases  ..........       $624,225  28 

Freight  ............       16,271  98 

$640,497"26 
Less:  Purchase  Rebates 

and  Allowances  630  81 

639,866  45 

$747,144  91 

Inventory,  end  of  year.  124,814  04 

Cost  of  Sales     ........  ~~~  622,330^87     89  94       (1) 

Gross  Profit     ..........  $  69,602~34     10"~06%  (2) 

Delivery  Expenses: 

Salaries  of  Drivers  .....   $     3,414  34 

Dep'n  on  Equipment  .  .         2,839  57 

Auto  Repairs       .......         1,562  53 

Gasoline  and  Oil    .....         1,479  27 

Drivers'  Expenses  .....  1  19  40 

Drayage  ..............  66  84 

Total  ...............  $    9,481.95  1.37       (1) 

Selling  Expenses: 

Salesmen's  Salaries  .....  $  11,812  50 

Salesmen's  Expenses.  .  .          1,942.06 

Advertising  ...........  844  .  32 

Telephone   and   Tele- 

graph ..........  642.57 

Total  ...............  $  15,241.45  2.20      (1) 


172 


ANALYSIS  OF  STATEMENTS 


General  Expenses: 

Salaries $    8,722  33 

Expenses 613  36 

Executive  Salaries 3,600  00 

Taxes  (other  than  fed- 
eral)          1,906  23 

Insurance 1,723  46 

Depreciation 1,259  54 

Light,  Heat,  and  Water  829  49 

Printing  and  Stationery  444.50 

Postage 408.52 

Collections 219  76 

Repairs 115  91 

Storage 22  69 

Miscellaneous 380  50 

Total "      20,246  29  2  93       (1) 

Total  Expense "  44,969  69      6.50%  (1) 

Net  Operating  Profit $  24,632  65       3  56%  (8} 

Additions  to  Income: 

Discount  on  Purchases.  $     9,565  86 

Interest  Earned 563  32 

Bad  Debts  Recovered   .  102  53 

Total _10,23JL71       1.48 

$  34,864.36       5.04% 
Deductions  from  Income: 

Discount  on  Sales $    4,771 . 92 

Interest  Paid  for  Money 

Borrowed 4,373  16 

Interest  Paid  on  Build- 
ing Contract 3,010 . 00 

Bad  Debts  Reserve ....  1,283 . 91 

Donations 162  00 

Total "  13,600.99      1  97 

Net  Profit $  21,263.37       3  07%  (4) 

Supplemental 

Total  Capital  Used  (see  Balance  Sheet,  below)  $276,317.34 

Ratio  of  Profit  to  Capital  7 . 69%  (5) 

Net  Worth  (beginning  of  year) 124,252.36 

Ratio  of  Profit  to  Net  Worth 17 . 11       (6) 

Common  Stock  Outstanding 1 13,400 .00 

Number  of  Shares  ($50.00  par  value) 2,268 

Per  cent  earned 18 . 75       (7) 

Dollars  earned  per  share 9 . 38 

Working  capital  ratio.  This  ratio  is  probably  the  best-known 
measure  applied  to  financial  statements,  because  more  than  any 
other  it  has  been  stressed  by  bankers  and  businessmen.  It  is 
computed  by  dividing  the  amount  of  the  current  assets  by  the 
amount  of  the  current  liabilities.  If  the  quotient  is  2,  the  current 
assets  are  said  to  be  in  a  "2  to  1 "  ratio;  that  is,  in  a  ratio  of  $2  of 
current  assets  to  each  $1  of  current  liabilities. 


ANALYSIS  OF  STATEMENTS  173 

What  the  working  capital  ratio  should  be  depends  upon  differ- 
ences in  types  of  business,  location,  and  other  factors,  the  effect  of 
which  is  to  vary  somewhat  the  proportions  involved.  While  some 
lines  of  trade  may  be  expected  to  maintain  a  2-to-l  ratio,  others 
may  necessitate  a  proportion  as  high  as  10  to  1. 

The  rapidity  with  which  receivables  and  inventory  are  turned  is 
a  factor  bearing  on  the  adequacy  of  the  working  capital  ratio. 
With  respect  to  accounts  receivable,  there  is  a  range  of  turnover 
from  3  days  in  some  of  the  retail  chain  stores  to  80  or  90  days  in 
coal  and  heavy  manufacturing  industries.  The  turnover  of 
inventories  is  most  rapid  in  such  industries  as  slaughtering  and 
meat  packing,  retail  chain  stores,  chemical  products,  and  iron  and 
steel,  while  the  turnover  of  inventories  is  found  to  be  slow  in  such 
industries  as  tobacco  products,  machinery  manufacturing,  leather 
products,  and  rubber  goods. 

Example 

The  following  balance  sheet  is  presented  to  illustrate  the  working  capital 
ratio.  It  will  also  be  referred  to  in  later  paragraphs,  where  the  computation  of 
other  ratios  is  discussed. 

BLANK  MERCANTILE  COMPANY 

BALANCE  SHEET 
DECEMBER  31,  19 — . 

Assets 
Current: 

Cash  in  Banks $  13,598.85 

Cash  on  Hand 4,113.24  $  17,712  09 

Accounts  Receivable — Customers  $  64,832  57 

Accounts  Receivable — Others     .        .  .  647  92 

Notes  Receivable — Customers  5,329  91 

Notes  Receivable— Others 227.31 

Securities 1,274  34 

Accrued  Interest 32  98 

Railroad  Claims i3_J6. 

$"72^438779 

Less:  Reserve  for  Bad  Debts        .  1,890.06       70,548  73 

Merchandise  Inventory  .  124,814  04 

Total $213,074.86 

Fixed: 

Land $     3,450.00 

Warehouse  Building $  50,373.48 

Warehouse  Equipment 545 . 77 

Delivery  Equipment 14,090 . 39 

Furniture  and  Fixtures 2,488 . 85 

$  67,498  49 

Less:  Accumulated  Depreciation 9,152.48       58,346.01 

Total $  61,796.01 


174  ANALYSIS  OF  STATEMENTS 

Deferred  Charges: 

Prepaid  Insurance $     1,298. 13 

Prepaid  Interest        148  34 

Total 1,446  47 

$276,317  34 
Liabilities 
Current: 

Payroll $     1,131.77 

Accounts  Payable 16,177.08 

Notes  Payable— Banks  50,000  00 

Notes  Payable— Others       17,600  00 

Notes  Payable— Stockholders 11,700  00 

Accrued  Taxes     1,575  17 

Accrued  Interest — Notes     1 ,393  92 

Accrued  Interest— Contracts 3,010.00 

Total $102,587,94 

Fixed: 

Warehouse  Contract  for  Deed       .  43,000  00 

Net  Worth: 

Capital  Stock— Common $11 3,400  00 

Surplus 17,329  40     130,729  40  

$276,317  34 

In  the  foregoing  balance  sheet,  the  current  assets  are  stated  at  $213,074.86, 
and  the  current  liabilities  are  stated  at  $102,587.94. 

213,074.86  -T-  102,587.94  =  2.077. 
The  ratio  of  working  capital  is,  therefore,  2.077. 

Sources  of  capital.  The  sources  of  capital  may  be  stated  in  a 
general  way  under  four  heading's,  as  follows: 

(1)  Short-term  borrowings  and  credits. 

(2)  Long-term  borrowings  and  credits. 

(3)  Stockholders'  investments. 

(4)  Surplus  (earnings  left  in  the  business). 

Summarizing  the  liability  section  of  the  foregoing  balance 
sheet  and  dividing  each  section  total  by  the  total  of  all  sections, 
the  ratio  of  capital  supplied  by  each  source  is  as  shown  in  the 
right-hand  column  of  the  following  tabulation : 

Amount      Per  Cent 

Current  Liabilities $102,587.94      37. 13 

Fixed  Liabilities 43,000.00       15.56 

Capital  Stock— Common 113,400.00      41 .04 

Surplus 17,329.40        6.27 

1276,317.34    100. 00 


ANALYSIS  OF  STATEMENTS  175 

Manner  in  which  capital  is  invested.  The  manner  in  which 
the  capital  is  employed  in  the  business  is  shown  by  a  summary 
of  the  asset  sections. 

Amount      Per  Cent 

Current  Assets $213,074.86      77. 12 

Fixed  Assets 61,796.01       22.36 

Deferred  Charges 1,446.47     .£2 

$276,317.34     100.00 

Turnover  of  total  capital  employed.  This  item  expresses  the 
relation  of  the  net  sales  to  the  total  capital  employed.  The  aver- 
age capital  employed  throughout  the  year  should  be  used,  but,  in 
the  absence  of  monthly  statements,  the  capital  at  the  beginning 
of  the  year  and  the  capital  at  the  end  of  the  year  should  be  added 
and  divided  by  two  to  give  an  estimate  of  the  average  capital 
employed.  In  arriving  at  this  average,  investments  not  employed 
in  operations  should  be  eliminated  from  the  total  assets,  for,  as  a 
rule,  they  represent  a  surplus  not  required  in  the  conduct  of  the 
business.  Income  from  such  investments  should  be  eliminated 
from  the  statement  of  earnings  before  the  ratio  is  computed. 

Total  assets  at  beginning  of  year     $246,351 . 89 

Total  assets  at  end  of  year 276,317 . 34 

2)$522,669~23 
Average  capital  employed  (securities  not  eliminated, 

as  the  amount  was  negligible) $261,334.61 

The  turnover  of  total  capital  employed  is,  therefore : 
$691,933.21  (net  sales)  -f-  $261,334.61  (average  capital)  =  2.64. 

Turnover  of  inventories.  The  subject  of  inventory  turn- 
over was  presented  in  Chapter  17. 

The  rate  of  turnover  is  computed  as  follows: 

$622,330.87  (cost  of  sales)  •*-  $112,131.69  (average  inventory)  =  5.55. 

Turnover  of  accounts  receivable.  The  normal  credit  period, 
whether  it  be  30,  60,  or  90  days,  is  compared  with  the  average 
number  of  days'  sales  uncollected  obtained  from  the  following 
formula,  as  a  means  of  judging  the  efficiency  of  the  collection 
department : 

A  Counts  receivable  at  end  of  fiscal  period        _.         .     .      _        .    , 

— — - — - — : r-; X  Days  in  fiscal  period 

Sales  for  fiscal  period 

—  Average  number  of  days'  sales  uncollected. 

The  Accounts  Receivable  account  showed  $64,832.57  of  out- 
standing accounts  at  the  close  of  the  fiscal  period.  The  sales  for 
the  fiscal  period  of  12  months  amounted  to  $691,933.21,  and  the 


176 


ANALYSIS  OF  STATEMENTS 


average  term  of  credit  granted  at  time  of  sale  was  30  days.  The 
average  number  of  days'  sales  represented  in  standing  accounts  is 
computed  as  follows: 

84,832.57 


691,933.21 


X  365  =  34. 


If  the  average  number  of  days'  sales  uncollected  is  greater  than 
the  average  term  of  credit,  the  presence  of  overdue  accounts  is 
indicated.  This  is  true  of  the  example  just  given. 

Turnover  of  fixed  property  investment.  This  turnover  ex- 
presses the  relationship  between  the  volume  of  business  done  and 
the  capital  invested  in  plant  and  equipment.  Large  investments 
in  property  and  equipment  increase  the  expense  burden  through 
charges  for  depreciation,  insurance,  taxes,  and  so  forth,  and  may 
make  a  favorable  or  an  unfavorable  operating  statement,  depend- 
ing on  the  volume  of  business  handled. 

The  number  of  dollars  of  sales  for  each  dollar  of  fixed  property 
investment  is  calculated  as  follows : 

$691,933.21  (net  sales)  -r-  $58,346.01  (net  fixed  property  investment)  =  11.86. 

Problems 

1.  From  the  balance  sheets  and  supplemental  information,  determine  the 
ratios  named,  following  the  balance  sheets. 


This  Year  Last  Year 

Current  Assets $215,003  48  $213,074.86 

Fixed  Assets— Net 57,535  04  61,796.01 

Deferred  Charges _M93.59  1,446.47 

Total $2737732.11  $276,317734 

Liabilities 

Current  Liabilities $  86,229 . 30  $102,587 . 94 

Fixed  Liabilities 38,000  00  43,000  00 

Total  Liabilities $124,229730  $145,587794 

Net  Worth 

Capital  Stock $124,300.00  $114,300.00 

Surplus 25,202.81  16,429.40 

Total  Net  Worth $149,502.81  $130,729.40 

Total $273/732. 11  $276,317.34' 

Annual  Sales i688,f67798  $691,933^21 

Annual  Expense 47,340.74  44,969.69 

Ratios 

Current  Ratio 

Worth  to  Debt 

Worth  to  Fixed  Assets 

Sales  to  Fixed  Assets 

Sales  to  Current  Debt 

Sales  to  Worth 

Expense  to  Sales  (%) 


ANALYSIS  OF  STATEMENTS 


177 


2.  The  United  Manufacturing  Company's  card  in  the  credit  file  of  the  Second 
National  Bank  contained  the  data  for  the  year  ended  January  31,  1944,  and 
from  their  balance  sheet  and  profit  and  loss  statement  you  have  entered  the 
comparative  figures  for  the  year  ended  January  31,  1945.  Compute  the  com- 
parative ratios  for  1945. 


COMPARATIVE  RATIOS 

COMPARATIVE  FINANCIAL  STATEMENTS 

1/31        1/31 

_                                  1944      1945    19       19       19 

ASSE 

TQ                    I/Si          t/31 
TS                1944        1945        19            19            19 

FIXED  ASSETS  TO 
TANGIBLE  NET  WORTH 

24.8 

CASH 

3,206 

1,862 

ACCOUNTS  RECEIVABLE 

45,199 

42,267 

CURRENT  DEBT  TO 
TANGIBLE  NET  WORTH 

48.0 

NOTES  TRADE  AO 

EPT  RECV. 

INVENTORIES 

89,342 

83,218 

NET  WORKING  CAPITAL  REP 
BY  FUNDED  DEBTS 

NET  SALES  TO 
INVENTORY 

4.3 

NET  WORKING  CAPITAL  REP 
BY  INVENTORY 

107.7 

TOTA 

CURRENT    f 

168,709 

127^349 

DUE  FROM  AFFILIA 

FE  OR  SUBS'Y 

INVENTORY  COVERED  BY 
CURRENT  DEBT 

61.3 

LAND  BUILDINGS 

MACHINERY.flXTl 

t£S 

L_  28,244 

49,248 

AVERAGE  COLLECTION  PERIOD 

42.5 

NOTES  ACC'TS  (Of  TORS.  PARTNERS) 

2L716 

2,156 

TURNOVER  OF 
TANGIBLE  VET  WORTH 

3.4 

TURNOVER  OF 
NET  WORKING  CAPITAL 

4.7 

TO' 

U.  ASSETS 

168,709 

179.754 

NET  PROFITS  ON  NET  SALES 

.52 

LIABILITIES 

NET  PROFITS  ON 
TANGIBLE  NET  WORTH 

1.8 

ACCEPT  ,  NOTES  PAYABLE 

NET  PROFITS  ON 
NET  WORKING  CAPITAL 

2.4 

PAYABLE  AFFILIATE  OR  SUBS'Y 
—  AtCnilALS  —  

'        ' 

' 

CURRENT  ASSETS 
TO  CURRENT  DEBT 

2    5 

Due  Officers 

_8,130 

.5,321 

TOTAL  DEBT  INCLUDING  N  W 
TANGIBLE  NET  WORTH 

148.0 

TOTAL  CURRENT 

54,725 

55,825 

SALES 

388,553 

394,774 

'MORTGAGES 

EXPENSES 

Deferred  Bank  Loan 

8,280 

NET  PROFIT 

2,048 

1,664 

WORKING  CAPITAL 

83,022 

71,523 

TANGIBLE  NET  WORTH 

113,963 

114,649 

-~ 

FIXED  ASSETS 

26,244 

49,244 

CAPITAL  STOCK 

103,^00 

los'.Voo 

FUNDED  DEBT 

8,280 

SURPLUS 

10,883 

12.548 

,    ,_i, 

3.  From  the  data  given  in  the  following  balance  sheet  and  profit  and  loss 
statement,  together  with  the  supplemental  data,  compute  the  fourteen  financial 
and  operating  ratios  relationships,  and  turnovers  outlined  in  the  preceding 
sections  of  this  chapter. 


178  ANAL/SIS  OF  STATEMENTS 

BLANK  MERCANTILE  COMPANY 

BALANCE  SHEET 
DECEMBER  31,  19 — 

Assets 
Current: 

Cash  in  Banks $  13,771  58 

Cash  on  Hand 3,616  34  $  17,387.92 

Accounts  Receivable — Customers  . .  $  59,424  48 

Accounts  Receivable— Others 704  30 

Notes  Receivable— Customers.      ..  3,746  76 

Notes  Receivable— Others 272. 19 

Securities 994  64 

Accrued  Interest 52  30 

Railroad  Claims 50J)5 

$~65,245~62 

Less:  Reserve  for  Bad  Debts 3,852.57      61,393  05 

Merchandise  Inventory 136,222  51 

Total $215,003  48 

Fixed: 

Land $    3,450.00 

Warehouse  Building $  50,180  55 

Warehouse  Equipment 545  77 

Delivery  Equipment        .         ...  14,090  39 

Furniture  and  Fixtures 2,503  85 

$  67,320  56 

Less:  Accumulated  Depreciation    .  13,235  52      54,085  04      57,53504 
Deferred  Charges: 

Prepaid  Insurance 1,193  59 

Total $273J32"1T 

Liabilities 
Current: 

Payroll $    1,116.17 

Accounts  Payable 13,325 . 73 

Notes  Payable— Banks 24,000  00 

Notes  Payable— Others 14,500  00 

Notes  Payable— Stockholders.     .       .  26,70000 

Accrued  Taxes 1,641  97 

Accrued  Interest— Notes 2,285  33 

Accrued  Interest — Contracts..       .  2,660.00 

Total $86,229.20 

Fixed: 

Warehouse  Contract  for  Deed .  38,000 . 00 

Net  Worth: 

Capital  Stock— Common $123,400.00 

Surplus 26,102.91    149,502.91 

Total $273,732.11 


ANALYSIS  OF  STATEMENTS 

BLANK  MERCANTILE  COMPANY 


179 


PROFIT  AND  Loss  STATEMENT 
FOR  THE  YEAR  ENDED  DECEMBER  31,  19 — . 
Sales: 

Gross  Sales $689,361.43 

Less:  Sales  Rebates  and  Al- 
lowances    $     1,059  89 

Prepaid  Freight 133  56 

__1,193.45 

Net  Sales $688,167.98  100.00% 

Cost  of  Sales: 
Inventory,  beginning  of  year  $124,814.04 

Purchases.... $611,332.45 

Freight __iy  84_6^ 

$626,517713 

Less:  Pur.  Rebates  and  Al- 
lowances          1,392  74 

~  625,124.39 

$749,938.43 
Inventory,  end  of  year 136,222  51 

Cost  of  Sales '  613,715  92  % 

Gross  Profit $  74,452.06  % 

Delivery  Expenses: 

Salaries  of  Drivers $    3,874 . 27 

Dep'n  on  Equipment 2,818 . 08 

Auto  Repairs  1,430.61 

Gasoline  and  Oil 1,231  29 

Drivers'  Expenses 125  35 

Drayage 52  91 

Total "  $    9,532.51  % 

Selling  Expenses: 

Salesmen's  Salaries $  12,300  00 

Salesmen's  Expenses 2,015  78 

Advertising 1,357 . 83 

Telephone  and  Telegraph ...  536 . 2 1 

Total 16,209.82 % 


180 


ANALYSIS  OF  STATEMENTS 


General  Expenses : 

Salaries 

Expenses 

Executive  Salaries 

Taxes  (other  than  federal) . 

Insurance 

Depreciation 

Light,  Heat,  and  Water . .  . 
Printing  and  Stationery. . . 

Postage 

Collections 

Repairs 

Storage 

Miscellaneous 


Total 

Total  Expense 

Net  Operating  Profit 

Additions  to  Income: 
Discount  on  Purchases . . 

Interest  Earned 

Bad  Debts  Recovered . . 
Total 


Deductions  from  Income: 

Discount  on  Sales 

Interest    Paid    for    Money 

Borrowed 

Interest    Paid    on   Building 

Contract 

Bad  Debts  Reserve 

Donations 

Total 

Net  Profit 


8,797  50 

265  43 

4,175  00 

2,069  17 

1,937  82 

1,264  96 

826  33 

516  70 

486  85 

238  65 

106  47 

18  29 

895  24 


$  21,598.41 


47,340  74 
$  27,111  32 


$    9,759  20 

1,348  60 

10  65 


11,118.45 
$  38,229.77 


$    4,523.98 
4,443.87 

2,660  00 

3,446  80 

269  20 


15,343  85  ... 
$  22,885.92  ... 


Supplemental 

Total  Capital  Employed  (see  Balance  Sheet) $ 

Ratio  o^f  Profit  to  Capital 

Net  Worth  (beginning  of  year) 130,729 . 40 

Ratio  of  Profit  to  Net  Worth 

Common  Stock  Outstanding  (see  Balance  Sheet)  

Number  of  Shares  ($50.00  per  value) . .... 

Per  Cent  Earned 

Dollars  Earned  Per  Share 


CHAPTER  20 
Partnership 

Definition.  A  partnership  association  is  defined  by  Chancellor 
Kent  as  follows:  "  A  contract  of  two  or  more  competent  persons  to 
place  their  money,  effects,  labor,  and  skill,  or  some  or  all  of  them, 
in  lawful  commerce  and  business,  and  to  divide  tiie  profits  and 
bear  the  losses  in  certain  proportions." 

Mathematical  calculations.  The  most  important  mathemati- 
cal calculations  in  partnership  accounting  are  concerned  with : 

(1)  Division  of  profits. 

(2)  Division  of  assets  upon  liquidation. 

(3)  Calculation  of  goodwill. 

Goodwill.     The  calculation  of  goodwill  also  has  to  be  considered 
in  connection  with  the  other  types  of  business  organizations- 
namely,  individual  proprietorship  and  corporation — -when  changes 
in  ownership,  reorganizations,  consolidations,  and  so  forth,  are 
made;  see  Chapter  21. 

Profit-sharing  agreements.  Profits  may  be  shared  in  many 
ways.  A  few  of  the  most  common  methods  of  profit  distribution 
are: 

(1)  Arbitrary  ratios. 

(2)  In  the  ratio  of  capital  invested  at  organization  of  business. 

(3)  In  the  ratio  of  capital  accounts  at  the  beginning  or  at  the 
end  of  each  period. 

(4)  In  the  ratio  of  average  investments. 

(5)  Part  of  the  profits  may  be  distributed  as  salaries  or  as 
interest  on  capital  invested,   and  the  remainder  in  some  other 
ratio. 

(6)  If  the  investment  is  less  than  the  amount  agreed  upon, 
interest  is  charged  o™  the  shortage ;  and  if  the  investment  is  more 
than  the  amount  agreed  upon,  interest  is  credited  on  the  excess; 
the  resulting  profit  or  loss  is  then  distributed  in  a  ratio  agreed  upon. 

Lack  of  agreement.  If  the  partners  have  failed  to  include 
in  their  articles  of  co-partnership  an  agreement  as  to  the  method 
by  which  profits  are  to  be  distributed,  the  law  provides  that  the 

181 


T82  PARTNERSHIP 

profits  shall  be  divided  equally,  regardless  of  the  ratio  of  the  part- 
ners7 respective  investments. 

Losses.  If  losses  are  incurred  and  no  provision  has  been  made 
for  their  distribution,  the  profit-sharing  ratio  governs. 

Arbitrary  ratio. 

Example 

A  and  B  are  partners.  A  has  $3,000.00  invested,  while  B  has  $2,500.00 
invested.  A  is  to  receive  f  of  the  profits,  and  B  is  to  receive  £.  The  profits 
for  the  year  are  $2,400.00.  What  is  each  partner's  share? 

Solution 

Net  profits  $2,400  00 

A's  share,  f  of  $2,400.00 1,600  00 

B's  share,  i  of  $2,400.00 800.00 

Problems 

A  and  B  were  partners.  Gain  or  loss  was  to  be  divided  f  and  f ,  respectively. 
A  invested  $3,500.00,  and  B  invested  $2,400.00.  During  the  year,  A  withdrew 
$500.00,  and  B  withdrew  $700.00.  At  the  end  of  the  year  the  books  showed  the 
following  assets  and  liabilities: 

Cash  on  Hand  and  in  Bank $8,000  00 

Inventory  of  Merchandise 7,500  00 

Notes  Receivable      790  00 

Accounts  Receivable 840 . 00 

Notes  Payable .               .  4,70000 

Accounts  Payable 7,24000 

(a)  What  has  been  the  gain  or  loss?  (6)  What  is  each  partner's  net  capital 
at  the  end  of  the  year? 

Ratio  of  investment. 

Example 

January  1 ,  A's  investment $10,000.00 

January  1 ,  #'s  investment 6,000  00 

January  1,  C's  investment 4,000  00 

Total '  $20,000700 

December  31,  Profits $  4,000  00 

Profits  are  to  be  shared  in  the  ratio  of  investments  at  the  beginning  of  the 
year. 

Solution 

Investment  Ratio  Profits  Shares 

A $10,000          i£  $4,000  $2,000 

B 6,000           A  4,000  1,200 

C 4,000          A  4,000  800 

$20,000          |S  $4,000 

Explanation.  Add  the  beginning-of-year  investments  of  each  of  the  partners, 
and  take  for  the  numerator  of  the  fraction  representing  each  partner's  share  his 
investment  at  the  beginning  of  the  year,  and  for  the  denominator  the  total 


PARTNERSHIP  183 

capital.    Using  these  fractions,  calculate  the  fractional  parts  of  the  net  profit 
or  lose,  and  these  will  be  the  partners'  shares. 

Problems 

In  each  of  the  following,  show  the  division  of  net  profit  or  net  loss,  which 
is  to  be  calculated  in  the  ratio  of  investments: 

INVESTMENTS 
A  B  C      NET  PROFIT     NET  Loss 

1.  $4,000     $4,000    $2,000       $2,500 

2.  5,000  3,000  1,500  $1,200 

3.  6,000  7,500  2,500  2,000 

4.  2,000  3,500  1,500  1,400 

5.  3,500  2,500  1,000  750 

Division  of  profits  by  first  deducting  interest  on  capital. 

Example 

January       1,  A's  investment $10,000 

January       1,  B's  investment 6,000 

January       1 ,  f"s  investment  4,000 

December  31,  Net  profits 4,000 

By  agreement,  each  partner  is  to  receive  5%  interest  on  his  investment 
(this  interest  to  be  deducted  from  total  profits),  and  the  balance  of  the  profits  is 
to  be  distributed  equally. 

Solution 

A's  investment,  $10,000,  X  .05 $    500,  interest 

ZTs  investment,      6,000,  X  .05 300,  interest 

C's  investment,      4,000,  X  .05 200,  interest 

Total  $l7XXJ 

Net  profits,  $4,000  -  $1,000  -  $3,000,  to  be  divided  equally.  $3,000  -r-  3 
=  $1,000,  each  partner's  share  after  interest  is  deducted. 

Interest  Profit  Total  Credit 

A $500  $1,000          $1,500 

B 300  1,000            1,300 

C 200  1,000            1,200 

Total $4,000 

Problems 
Show  the  division  of  profits  in  each  of  the  following: 

RATE  OF  INT. 

INVESTMENTS                 NET                  ON  BALANCE  TO 

ABC        PROFITS       INVESTMENT  BE  DIVIDED 

1.  $  8,000     $  4,250     $  3,700     $4,000                 5%  Equally 

2.  9,750        3,500       10,000       6,000  6%  Equally 

3.  4,725        5,300        5,250       5,300  6%  Equally 

4.  12,000        6,000        4,000       4,500  4%  *,  T,  I 

5.  20,000       10,000        5,000       5,000  6%  i,  i,  i 

Profits  insufficient  to  cover  interest  on  investment.  If  it  is 
agreed  that  each  partner  is  to  be  credited  with  interest  on  his 


184  PARTNERSHIP 

invevStment,  the  interest  must  be  credited  to  each  partner,  even 
though  the  total  profits  are  not  large  enough  to  cover  the  credit. 
Any  over-distribution  incurred  by  the  distribution  of  the  interest 
should  be  divided  among  the  partners  in  accordance  with  the  agree- 
ment as  to  the  division  of  profits.  The  same  rule  applies  where 
there  is  a  loss  before  interest  is  credited. 

Example 

January       1,  A's  investment $10,000 

January        1,  B's  investment 6,000 

January       1^  C"s  investment 4,000 

December  31,  Business  profits 700 

By  agreement,  each  partner  is  to  receive  5%  interest  on  his  investment,  and 
the  profits  are  to  be  shared  equally. 

Solution 

A's  investment,  $10,000,  X  .05 $    500,  interest 

£'s  investment,      6,000,  X  .05 300,  interest 

C"s  investment,      4,000,  X  .05 200,  interest 

Total  interest  to  be  credited  $  1 ,000 

Profits  earned 700 

Net  loss $    300 

Since  the  loss  is  to  be  shared  equally,  each  partner's  loss  is  $100. 

Credit  Debit  Net 

Interest  Loss  Credit 

A $500  $100  $400 

II 300  100  200 

(J 200  100  100 

Total $700 

Problems 

Find  the  net  credit  or  debit  to  each  partner  in  each  of  the  following: 

INVESTMENTS  PROFIT  OR  Loss     INTEREST    BALANCE  TO 

ABC        BEFORE  INTEREST       RATE        BE  DIVIDED 

1.  $  8,000     $  8,000     $  4,000       Profit,  $    800  6%  Equally 

2.  5,000        7,000        2,000       Profit,       140  6%  A,  A,  A 

3.  3,800        4,200        5,000      Loss,         200  6%  Equally 

4.  10,000        7,500        5,000       Profit,    2,150  6%  f ,  $,  i 
6.     15,000       15,000       10,000       Profit,    2,000            6%             Equally 

Adjustments  of  capital  contribution.  If  the  partners  do  not 
invest  the  agreed  amounts,  adjustments  may  be  made,  provided  the 
contract  so  states.  Partners  may  be  charged  with  interest  en 
the  amount  of  the  shortage  of  their  investment  from  the  agreed 
amount,  and  may  be  credited  with  interest  on  the  excess  of  their 
investment  over  the  agreed  amount.  These  adjustments  should 
be  mad(t  before  the  profits  for  the  period  are  prorated.  If  interest 
adjustments  result  in  an  over-distribution  of  profits,  the  amount 


PARTNERSHIP  185 

over-distributed  is  divided  in  the  ratio  of  the  division  of  profits, 
unless  otherwise  agreed. 

Example 

Agreed  to  Invest  Invested 

January       1,  A $10,000  $12,000 

January       1,  B 6,000  5,000 

January       1,  C 4,000  2,000 

December  31,  Profits  for  the  year. ...  3,100 

By  agreement,  A  is  to  be  allowed  5%  interest  on  his  excess  investment,  and 
B  and  C  are  to  be  charged  5%  interest  on  their  shortages.  After  these  adjust- 
ments have  been  made,  profits  are  to  be  divided  equally. 

Solution 

A's  excess,  $2,000,  X  .05 $100,  interest 

7?'s  shortage,  $1,000,  X  .05 50,  interest 

C"s  shortage,  $2,000,  X  .05 100,  interest 

Charge  to  #'s  account $  50 

Charge  to  C's  account TOO    $150 

Credit  to  A's  account 100 

Net  amount  of  interest $  50 

The  net  amount  of  interest,  $50,  is  added  to  net  profits. 
Profits  before  distribution: 

Net  profits $3,100 

Add  net  interest 50 

Total $3,150 

$3,150  -T-  3  =  $1,050,  each  partner's  share  after  interest  adjustment. 

Afs*  profits $1,050 

Add  interest 100     $1,150,  total  credit  of  A 

B's  -J-  profits $1,050 

Less  interest 50       1,000,  net  credit  of  B 

C's  -J  profits $1,050 

Less  interest 100          950,  net  credit  of  C 

Total  profits $3,100 

Problems 

1.  Prepare  a  statement  of  profit  distribution  from  the  following  facts: 

A  B  C  D 

Agreed  investment $6,000  $6,000  $8,000  $4,000 

Investment 7,000  6,000  6,000  2,500 

Profit  ratio  after  adjustment  of 
6%  interest  on  excess  or  de- 
ficiency of  investment 25%  25%  33J%  16|% 

Net  profits  before  adjustments  for  interest,  $6,500. 

2.  Prepare  a  statement  of  profit  distribution  from  the  following  facts: 

X  Y  Z 

Agreed  investment $5,000    $4,500    $4,500 

Investment 4,000      4,000      6,000 


186  PARTNERSHIP 

Profits  to  be  shared  equally  after  adjustments  for  6%  interest.  Profits  before 
adjustments  for  interest,  $600. 

3.  The  capital  of  a  certain  organization  was  to  be  $40,000.00,  of  which  A  and  R 
were  to  contribute  one-half  each,  A  to  receive  55%  of  the  profits  and  B  to  receive 
45%.  A,  being  short  of  funds,  invested  only  $15,000.00,  and,  the  firm  being 
short  of  capital,  11  put  in  the  balance  until  A  could  make  up  his  shortage,  with 
the  provision  that  he  be  allowed  6%  interest  on  the  excess  of  his  investment 
over  the  agreed  amount.  The  profits  for  the  year  were  $12,000.00.  Show 
distribution  of  profits. 

Profit  sharing  in  ratio  of  average  investment. 

First  method.  Multiply  the  original  investment  by  the  number 
of  days  or  months  during  which  the  amount  was  in  the  business 
without  change.  The  product  may  be  termed  Day-Dollars  or, 
Month-Dollars.  The  ratio  of  any  product  to  the  total  of  the  prod- 
ucts is  the  average  capital  ratio  for  that  partner. 

When  the  capital  is  changed,  either  by  additional  investment 
or  by  withdrawal,  the  changed  capital  is  multiplied  by  the  number 
of  days  or  months  to  find  its  value  in  day-dollars  or  month-dollars, 
and  for  each  change  a  new  calculation  is  made.  The  ratio  of  the 
total  of  the  day-dollars  or  month-dollars  for  each  partner  to  the 
sum  of  the  day-dollars  or  month-dollars  for  all  the  partners  gives 
the  ratio  of  each  partner's  investment  to  the  total  investment. 

Example 

A 
Debit  ('red  it 

Feb.  1 $1 ,000     Jan.  1                $10,000 

June  1 1,500     May  1       4,000 

Nov.  1 500     July  1        1,000 

H 

July  1 $1,000     Jan.  1 $  6,000 

Dec.  1 1,000     Aug.  1 4,000 

Oct.  1          2,000 

Net  profits  of  the  business  for  the  year  were  $4,530. 

Solution 

A 

MONTHS  IN  MONTH- 

INVESTED     BUSINESS  DOLLARS 

Jan.   1,  $10,000  X  1  month $10,000 

Feb.  1,      9,000  X  3  months 27,000 

May  1,    13,000  X  1  month 13,000 

June  1,    11,500  X  1  month 11,500 

July  I,    12,500  X  4  months 50,000 

Nov.  1,    12,000  X  2  months 24,000 

A's  month-dollars  investment $135,500 


PARlNtRSHIP  187 

R 

Jan.   1,  S  6,000  X  6  months $36,000 

July  1,      5,000  X  1  month  5,000 

Aug.  1,      9,000  X  2  months  18,000 

Oct.    1,     1 1,000  X  2  months     22,000 

Dec.  1,    10,000  X  1  month     10,000 

B's  month-dollars  investment  $  01 ,000 

Total  month-dollars  investment $226,500 

A's  share  of  profits,  ty~~y,  <>f  $4,530 $  2,710 

^^O,OUU 

01  000 
#'s  share  of  profits,  ~>,.--rrnn  of  $4,530.  .        1,820 

22b"3°°  $  4,530 

If  the  average  investment  is  desired,  it  can  be  found  by  dividing  the  month- 
dollars  by  12,  as: 

A's  month-dollars,  S135,500  -~  12  .          $1  1,201   67 

B's  month-dollars,  SOI, 000  4-   12  7,583  33 

Total  average  monthly  investment  $1X,S75  00 

The  ratios  of  the  average  monthly  investments  are  the  same  as  the  ratioti 
of  the  month-dollars  investments. 

Second  method.  Multiply  each  investment  by  the  number 
of  months  from  the  date  made  until  the  end  of  the  period;  find 
the  sum  of  the  products  obtained.  Likewise,  multiply  each  with- 
drawal by  the  number  of  months  from  the  date  withdrawn  until 
the  end  of  the  period;  find  the  sum  of  the  products  obtained. 
Deduct  the  sum  of  the  withdrawal  products  from  the  sum  of  the 
investment  products;  the  result  for  each  partner  should  be  the  same 
as  the  month-dollars  obtained  by  the  first  method. 

The  example  under  the  first  method  is  used  in  the  following 
solution. 

Solution 
A 

TIME  TO 

INVESTMENTS  END  OF         MONTH- 

Date      Amount          YEAR          DOLLARS 

Jan.    1     $10,000  X  12  months  =  $120,000 

May  1         4,000  X    8  months  =      32,000 

July   1         1,000  X    6  months  =        6,000 

~  $158,000 

WITHDRAWALS 

Feb.  1  $  1,000  X  11  months  =  $  11,000 
June  1  1,500  X  7  months  =  10,500 
Nov.  1  500  X  2  months  =  1,000 

22,500 
A's  month-dollars  $135,500 


188  PARTNERSHIP 

B 

INVESTMENTS 

Jan.  1  $  6,000  X  12  months  =*  $  72,000 
Aug.  1  4,000  X  5  months  =  20,000 
Oct.  1  2,000  X  3  months  =  6,000 

*  98,000 
WITHDRAWALS 

July  1  $  1,000  X  6  months  =  $  6,000 
Dec.  1  1,000  X  1  month  =  1 ,000 

~~  7,000 

#'s  month-dollars  $  91,000 

The  distribution  of  the  profits  is  the  same  as  in  the  preceding  example. 

Problems 

1.  A,  #,  and  C  began  business  January  1.     Their  accounts  for  the  year 
appear  as  follows: 

A 

Jan.  1 $  7,500 

July  1   2,500 

n 

Mayl $4,000     Jan.  1       $10,000 

C 

Oct.   1 $7,000     Jan.   I $10,000 

Aug.  I 3,000 

Their  profits  for  the  year  were  $3,310.     Determine  the  share  of  each  partner, 
if  profits  were  divided  on  the  basis  of  average  investment. 

2.  Ames  and  Brown  engaged  in  the  hardware  business,  and  at  the  end  of 
the  first  year  their  books  showed  a  profit  of  $2,35701.     They  had  agreed  to 
share  profits  and  losses  equally,  after  allowing  6%  interest  on  average  invest- 
ment.    Their  investments  and  withdrawals  for  the  year  were: 

Ames 

July      1 $    500     Jan.      1 $3,000 

Sept.  15 1,500 

Brown 

Sept.  15 $1,000     Jan.      1 $2,500 

July     1 250 

Determine  the  net  capital  of  each  partner  at  the  end  of  the  year. 

3.  C.  H.  John  and  C.  B.  Arthur  formed  a  partnership.    John  invested 
$15,000,  but  four  months  later  withdrew  $3,000.     Arthur  invested  $10,000,  and 
eight  months  later  withdrew  $2,000.     Interest  at  6%  was  to  be  credited  on 
average  investment;  the  remainder  of  the  profits  was  to  be  distributed  in  pro- 
portion to  original  investments.    The  first  year's  profits,  before  interest  adjust- 
ment, were  $2,500.     What  was  the  net  capital  of  each  partner  at  the  beginning 
of  the  second  year? 

Liquidation  of  partnership.     Because  of  the  nature  of  the 
association,  a  partnership  must  necessarily  be  terminated  on  or 


PARTNERSHIP  189 

before  the  death  of  any  one  of  the  partners.  It  is  not  necessary  to 
discuss  here  the  various  causes  of  dissolution,  but  only  the  problems 
met  with  at  the  time  of  settlement.  The  purpose  of  the  formation 
of  a  partnership  is  the  making  of  profits,  and  the  division  of  losses 
is  governed  by  the  same  general  rule  as  the  division  of  profits. 
Profits  should  be  credited  and  losses  should  be  charged  before  any 
division  of  assets  is  made.  If  this  rule  were  not  followed,  an  unfair 
distribution  of  capital  would  result. 

When  dissolution  is  accompanied  by  liquidation,  each  of 
the  partners  has  an  equal  obligation  to  share  in  the  work.  But 
since  it  does  not  usually  require  the  time  of  all  the  partners,  any 
one  of  the  partners,  or  an  outsider,  may  liquidate  the  business. 

In  liquidation,  profits  or  losses  must  first  be  divided  in  the  profit 
or  loss  ratio,  and  the  remaining  capital  should  then  be  shared  by 
the  partners  in  the  capital  ratio. 

In  insolvency,  partners  must  share  losses  in  the  profit  and  loss 
ratio,  and  not  in  the  capital  ratio.  This  may  at  times  result  in  a 
deficit  in  capital  for  some  one  or  more  of  the  partners.  Each 
partner  with  a  deficit  should  contribute  to  the  firm  the  amount  of 
his  deficit.  But  if  he  is  totally  unable  to  pay  into  the  firm  ai)> 
portion  of  his  deficit,  the  remaining  partners  must  bear  this  loss  in 
the  profit  and  loss  ratio. 

The  governing  profit  and  loss  ratio,  when  a  partner  is  unable  to 
pay,  should  be  stated  in  fractions,  of  which  the  numerators  are 
the  profit-and-loss-sharing  per  cents  of  the  partners  with  credit 
balances,  and  the  denominators  are  the  sums  thereof.  It  is  evident 
that  it  is  incorrect  to  compute  the  test  loss  division  by  multiplying 
the  loss  by  the  profit  and  loss  per  cents,  since  the  full  amount  of  the 
test  loss  would  not  be  distributed. 

Methods.    Liquidation  may  be  accomplished  in  two  ways: 

(1)  All  the  assets  may  be  converted  ,  all  the  liabilities  paid,  the 
profits  or  losses  distributed,  and  all  the  capital  divided  at  one  time. 

(2)  A  periodic  distribution  of  the  capital  may  be  made  before 
all  the  assets  are  converted. 

Total  distribution.  The  first  method  of  liquidation  does  not 
involve  any  very  difficult  calculations. 

Example 

From  the  following  figures,  show  the  amount  of  capital  distributed  to  each 
partner  at  dissolution: 

ABC 
Capital    balances    before    conversion    of 

assets         $10,000     $6,000     $4,000 

Profit  ratio 40%        40%        20% 

Assets  converted  into  cash $30,000 

Liabilities  to  be  paid 14,000 


190 


PARTNERSHIP 


Solution 

Assets  Liabilities        Net  Assets 

$30,000     -     $14,000     =     $16,000 

Total  Investment,  $20,000,  less  Net  Assets,  $16,000  =  Loss,  $4,000 


Capital  balances  before  conversion  of  assets.  $10,000 

Distribution  of  loss _L>600 

Balances $~S7,400 

Cash  distributed <S,400 


B 

(40%) 
$6,000 

1,600 
$4;400 

4,400 


C 

(20%} 
$4,000 
_J400 
$3^00 
3,200 


Total 
(100%) 
$20,000 


16,000 


Periodic  distribution.  Periodic  distribution  may  result  from 
either  of  two  causes: 

(1)  The  desire  of  the  partners  to  reduce  the  capital  of  the  firm, 
or  to  completely  dissolve  the  firm,  even  though  it  is  still  solvent. 

(2)  Forced  liquidation. 

As  the  assets  are  converted  into  cash,  and  the  debts  are  paid, 
the  balance  of  cash  should  be  distributed  periodically  to  the  part- 
ners. This  should  be  done  in  such  a  way  as  to  reduce  the  accounts 
to  the  profit  and  loss  ratio  existing  among  the  partners.  The 
distribution  is  made  on  the  assumption  that  all  book  assets  may  be 
a  total  loss  until  converted  into  cash. 

The  following  example  illustrates  the  adjustment  of  capital 
ratios  to  profit  and  loss  ratios. 

Example 
From  the  following  data,  show  the  periodic  distribution  of  the  cash  collected: 

A  B  C 

Capital  balances  before  conver- 
sion of  assets $10,000     $6,000     $4,000 

Profit  ratio 40%       40%       20% 

First  period: 

Net  loss $  1 ,000 

Cash  collected  .  .        .  9,000 

Assets  unrealized  .  .  10,000 

Second  period: 

Net  loss..  ....  1,000 

Cash  collected  . .  .  5,000 

Assets  unrealized     .  .  4,000 

Third  period: 

Cash  collected 2,000 

All  other  assets  uncollectible. 


PARTNERSHIP 


191 


Solution 


A 


1.  Capital  balances  before  conversion  of 

assets       $10,000 

2.  Distribution  of  loss  .  400 

3.  Balance  after  distiibution  of  loss      .         $  9,600 
For  the  purpose  of  making  a  test,  it  will 

be  assumed  that  the  unrealized  assets 
will  never  be  realized. 

4.  Test  loss  in  profit  and  loss  ratio     .  .  . 

5.  After  the  test  loss  has  been  deducted, 

the  remaining  amounts  will  show  the 
proper  distribution  of  the  cash  bal- 
ance (3-4) 

6.  Balance  at  the  end  of  the  first  period  (3-5)  $  4,000 

7.  Net  loss  for  second  period .  .          ....  400 

8.  Balance  after  distribution  of  loss  $  3,600 
The  balances  of  the  accounts  are  now  in 

the  profit  and  loss  ratio. 

9.  Distribution  of  cash 

10.  Balance  at  end  of  second  period 

1 1.  Net  loss  for  third  period 

12.  Balance  after  distribution  of  loss 

13.  Cash  distribution 


$6,000 

400 

$5,600 


$4,000 

200 

$3,800 


Total 

$20,000 

1,000 

$19,000 


(4,000)    (4,000)    (2,000)    (10,000) 


5,600 


1,600 

$4,000 

400 

$3^600 


1^800 

$27)00 

J200 

$1^800 


9,000 

$io,ooo 

1,000 
$~9,6o6 


2,000 

2,000 

1,000 

1  period 

$"1,600 
SOO 

$1,600 
800 

$    800 
400 

n  of  loss 

$  "  SOO 
SOO 

$    SOO 
SOO 

$    400 
400 

5,000 
47000 
2,000 
2,000 

__      2££2 

The  following  example  illustrates  the  adjustment  of  capital 
to  the  profit  and  loss  ratio,  where  a  deficiency  of  one  partner  is 
involved. 

Example 

Show  how  each  period's  cash  should  be  distributed  in  the  following: 

A  B  C 

Capital  balances  before  conver- 
sion of  assets  .     $10,000    $8,000     $2,000 

Profits  to  be  shaied  equally. 

First  period: 

Net  loss 

Cash  to  be  distributed 

Second  period: 

Net  loss 

Cash  to  be  distributed          .    . 

Third  period: 

Remaining  assets  sold  for 

Solution 

A 
Capital  balances  before  conversion  of  assets  $10,000 

First  period's  loss  distributed 500 

Balance  after  distribution  of  loss 

Test  loss  of  amount  of  the  remaining  assets 
It  will  be  observed  from  the  test  loss  that 
C's  possible  loss  is  $2,000  greater  than  his 
capital.     If  the  test  loss  should  become 


$1,500 
8,000 

1,500 
3,000 


4,000 


B 

$8,000 
500 


C 

$2,000 
500 


Total 
$20,000 
1,500 


$  9,500    $7,500     $1,500    $18,500 
(3,500)    (3,500)    (3,500)    (10,500) 


192 


PARTNERSHIP 


(1,000)    (1,000)     2,000 


an  actual  loss,  C  will  owe  the  firm  $2,000, 
and  if  C  should  be  unable  to  pay  in  this 
$2,000,  A  and  B  would  be  required  to 
bear  this  additional  loss.  To  provide 
against  this  contingency,  a  further  test 
loss  charge  of  $2,000  is  made  against  A 
and# 

When  the  sum  of  the  two  test  losses,  $4,500 
($3,500  +  $1,000),  is  deducted  from  A's 
investment  of  $9,500,  it  can  be  seen  that 
A  should  receive  $5,000;  it  can  also  be 
seen  that  the  sum  of  B's  test  losses 
deducted  from  his  investment  gives  the 
amount  of  cash  which  is  payable  to  him. 

Cash  distribution 

Balance  of  capital  undistributed          

Second  period's  loss  distributed 

Balance  after  distribution  of  loss 

Test  loss  of  unrealized  assets  

What  applied  above  applies  again  here.  C"s 
account  shows  a  possible  loss,  and  the 
amount  must  be  distributed  as  a  test  loss 
to  be  taken  up  by  the  other  partners. 

Test  loss  for  C's  s 

Cash  distribution 

Balance  undistril 

Third  period's  ne 

Cash  on  hand . . . 

Cash  distributed 


The  following  example  illustrates  a  return  of  investments  and 
loans  of  partners  which  is  complicated  by  the  accounting  principle 
that  "  loans  must  be  paid  before  capital  is  returned  to  partners. " 


$J5,qoq 

$~4,500 
,500 

$"  4,000 
(2,000) 


$3,000 

$4,500 

500 

$4,000 

(2,000) 


$1,500 

500 

$1,000 

(2,000) 


$  8,000 

$107566 
1,500 
$  9,000 
(6,000) 


7's  account 

(500)       (500)     1  000 

tion 

1  ,500       1  ,500 

3  000 

tributed  

$~~2,566     $2^500    $1,000 

$  6660 

j  net  loss 

....          667          667          666 

2  000 

ted  

$  1,833     $1,833     $    334 
1.833       1.833         334 

$  4,066 
4.000 

Partners 


A. 
B. 
C. 
D. 


Example 

Capital  Accounts     Loan  Accounts     Profit  Ratio 


$33,000 
28,500 
18,000 
10,500 

$90,000 


$10,500 
10,000 
21,000 
|8,500 

$60,000 


The  partners  have  decided  upon  a  dissolution,  and  after  paying  all  their 
liabilities,  they  have: 

Cash $  30,000 

Other  assets 110,000 

Net  loss 10,000 

How  should  the  cash  be  distributed  among  the  partners,  assuming  that  no 
member  of  the  firm  has  private  property  with  which  to  repay  a  capital  account 
that  has  been  reduced  by  losses  to  a  debit  balance? 


PARTNERSHIP 


195 


Profit  and  loss  ratio .... 
Capital  balances  before  conver- 
sion of  assets 

Loss  distributed  

Balance  of  capital 

Test  loss  of  assets 

Possible  deficiency  of  capital 


Solution 

A 
40% 


B 
30% 


$33,000 
4,000 


$28,500 
3,000 


C 

20% 

$18,000 
2,000 


D 

10% 

$10,500 
1,000 


Total 
100% 

$90,000 
10,000 


$29,000    $25,500    $16,000 
(44,000)    (33,000)    (22,000) 


$  9,500    $80,000 
(11, 000)  (110,000) 


$15,000     $  7,500     $  6,000    $  1,500     $30,000 


By  applying  the  possible  loss  of  unrealized  assets  against  capital  accounts, 
it  is  found  that  the  possible  loss  is  greater  than  the  capital  invested. 

In  practice,  as  long  as  the  firm  has  assets  and  owes  each  partner  money  on  a 
loan  account,  a  partner  will  generally  not  pay  cash  into  the  firm,  since  the  firm 
would  have  to  pay  it  back  immediately.  The  problem  states  that  none  of  the 
members  of  the  firm  has  money  other  than  that  invested  in  the  firm.  As  the 
shortages  are  debts  due  the  firm,  and  as  the  partners  have  loan  accounts,  these 
loan  accounts  will  undoubtedly  be  used  as  a  set-off.  Therefore,  the  shortages 
will  be  deducted  as  follows: 

A  B  C  D  Total 

Loans  by  partners $10,500     $10,000     $21,000     $18,500     $60,000 

Less  possible  shortages J  5,000        7,500        6,000         1^500      30,000 

Kach  partner's  standing  in  the 

business  after  distribution  of 

the  test  loss ($4,500)   $2,500    $15,000    $17,000    $30,000 

As  A's  loan  is  not  enough  to  take  up  the  possible  shortage,  and  as  the  problem 
states  that  A  has  no  other  property,  it  is  necessary  to  distribute  A's  test  shortage 
to  the  other  partners. 

A  B  C  D  Total 

Ratio  of  distribution £ J  $#  H 

Balances  after  distribution  of  test 

loss ($4,500)   $2,500    $15,000    $17,000     $30,000 

A's  test  shortage  distributed     .  4,500     02,250)      (U>00)       J750) 

Balances  .  0     $~250     $13^>o6"   $16,250     $30"^00 

Cash  distribution 0         250      13,500       16,250      30,000 


The  accounts  of  the  partnership  now  stand: 


Net  assets $110,000 

A's  capital 

^'s  capital 

^"s  capital 

/)'s  capital 

A's  loan 

B'sloan $10,000 

Less  payment 250 

C'a  loan $21,000 

Less  payment 13,500 

D'sloan $18^500 

Less  payment 16,250 


29,000 
25,500 
16,000 
9,500 
10,500 

9,750 
7,500 
2,250 


$110,000  $110,000 


194 


PARTNERSHIP 

Problems 


1.  Ay  B,  and  C  decided  to  dissolve  partnership.     On  the  basis  of  the  following 
facts,  show  the  proper  distribution  for  each  period: 

ABC 

Capital $8,000    $4,000    $6,000 

Ratios 444%     22f%     33i% 

First  period: 

Net  loss $4,000 

Assets  uncollected 9,000 

Cash.  5,000 

Final  period: 

Assets  converted     7,000 

Losses 2,000 

2.  Show  the  periodic  casli  distribution  based  on  the  following  facts: 

Capital     Loans     Ratio 

X $22,000     $2,000       50% 

Y 8,000       3,000    33^% 

Z     4,000       1,000     16f% 

First  period: 

Cash $  4,000 

Assets  uncoliected     36,000 

Second  period: 

Cash 10,000 

Assets  uncollected     24,000 

Loss 2,000 

Third  period: 

Cash 21,000 

Loss 3,000 

C.  P.  A.  Problems 

1.*  The  capital  of  a  partnership  is  contributed  as  follows: 

A $90,000 

B . .        .  .     45,000 

C    15,000 

The  partnership  agreement  provides  for  profit  sharing  in  the  following  ratios: 

A 50% 

B 30% 

C 20% 

The  partners'  salaries  are  as  follows: 

A .     $5,000 

B 3,000 

C 2,000 

At  the  end  of  the  first  year's  business,  C  dies.  The  books  are  closed,  and 
the  net  assets  of  the  business  are  shown  to  be  $152,500.  A  and  B  liquidate  the 
affairs  of  the  partnership,  and  distribute  the  surplus  as  follows: 

First  distribution $42,410.20 

Second  distribution 74,622 . 30 

Final  distribution 31,967.50 


*  C.  P.  A.,  Maryland. 


PARTNERSHIP 


195 


Prepare  a  statement  of  the  partners'  accounts,  showing  how  the  distribution 
of  assets  should  be  made  and  how  the  losses  should  be  apportioned. 

2.*  A,  B,  C,  and  D  enter  into  partnership  with  a  capital  of  $100,000.  A 
invests  $40,000;  B,  $30,000;  f ,  $20,000;  and  /),  $10,000.  They  are  to  share 
profits  or  losses  in  the  following  proportions:  .1,  35%;  B,  28%;  C,  22%;  and 
/),  15%.  They  are  also  to  receive  stipulated  salaries  chargeable  to  the  business. 

At  the  end  of  six  months,  there  is  a  loss  of  $S,000,  and  meantime  the  partners 
have  drawn  against  prospective  profits  as  follows:  .1,  $400;  B,  $600;  C,  $600; 
and  /),  $400. 

They  dissolve  partnership,  and  agree  to  distribute  the  proceeds  of  firm  assetvS 
monthly  as  realized.  C  and  D  enter  other  businesses,  and  A  and  B  remain  to 
wind  up  the  firm's  affairs,  it  being  stipulated  that  from  all  moneys  collected 
and  paid  over  to  C  and  /),  a  commission  of  5<  '0  be  deducted  and  divided  equally 
between  A  and  B  for  their  services  in  liquidating  the  partnership. 

The  realization  anil  liquidation  lasts  four  months,  and  the  transactions  are 
as  follows: 

Kxpwscs  and 


First  month 
Second  month 
Third  month 
Fourth  month 


Prepare  partners'  accounts,  showing  the  amount  payable  monthly  to  each 
partner. 

3.f  A,  B,  (',  and  D  formed  a  personal-service  partnership,  the  clientele  of 
the  firm  being  personal  clients  of  the  respective  partners. 

All  fees  received  and  all  expenses  were  pooled  by  the  firm,  and  the  partnership 
agreement  stated  that  the  net  earnings  for  the  year  were  to  be  shared  as  follows: 


Losses  on 

Idealization, 

A  .wets 

Liabilities 

Exclusive  of 

Realized 

Liquidated 

Commissions 

$  30,190 

$  7,900 

$    400 

50,300 

0,100 

750 

20,010 

3,SO() 

340 

9,500 

2,200 

110 

$110,000 

$20,000 

$1,600 

B 
C 
D 


40% 

16*% 
10% 


On  August  31,  as  a  result  of  a  dispute,  a  supplementary  agreement  covering 
the  remainder  of  the  year  was  made  between  the  partners.  This  agreement 
provided  that  the  distribution  of  net  earnings  was  to  be  made  on  the  basis  of 
the  above  percentages,  except  that  in  the  distribution  of  the  net  earnings  for 
the  last  four  months  of  the  year,  so  far  as  C  and  D  were  concerned,  a  net  earning 
was  to  be  assumed  on  the  basis  of  payment  by  the  clients  of  A  and  B  of  gross 
fees  of  $175,000  and  $250,000,  respectively,  instead  of  the  amounts  actually 
received  from  those  clients. 

The  deficiency  in  A's  gross  fees  was  to  be  charged  to  him,  and  the  excess  in 
Z?'s  gross  fees  credited  to  him. 


*  C.  P.  A.,  New  York. 

t  American  Institute  Examination. 


196  PARTNERSHIP 

No  adjustment  for  expenses  was  to  be  applicable  to  either  the  deficiency  or 
the  excess. 

The  net  income  from  January  1  to  August  31  was  $75,000. 

From  September  1  to  December  31,  the  following  gross  fees  were  received: 

From  clients  of  A $110,000 

From  clients  of  B 290,000 

From  clients  of  C 15,000 

From  clients  of  D 25,000 

The  operating  expenses  for  the  last  four  months  were  $55,000. 
Determine  the  total  net  income  of  each  partner  for  the  year,  taking  into 
account  the  supplementary  agreement. 

4.*  On  January  1,  19 — ,  Adams,  Burk,  and  Oline  became  partners  in  the 
operation  of  a  dry  goods  business  in  Scranton,  Pa. 

At  December  31  of  the  same  year,  the  trial  balance  of  the  partnership,  before 
any  adjustments  were  made,  was  as  follows: 

Adams,  capital $  50,000 

Burk,  capital 30,000 

Cline,  capital 20,000 

Inventory  of  merchandise,  January  1             .    .  $125,000 

Accounts  receivable,  customers.      .                     .  75,000 

Accounts  receivable,  employees     .  .  3,000 

Cash 6,000 

Notes  payable 60,000 

Accounts  payable 15,000 

Sales 500,000 

Purchases,  including  freight 323,000 

Salaries  and  store  expenses.     ...            .  125,000 

Bad  debts  written  off .  2,500 

Interest  paid  on  notes  payable .  6,000 

Salary  to  Mr.  Adams 2,500 

Salary  to  Mr.  Burk 4,000 

Salary  to  Mr.  Cline 3,000 

$675,000  $675,000 

Prepare  a  balance  sheet  as  of  December  31,  a  profit  and  loss  statement  for 
the  year  ended  the  same  date,  and  a  statement  of  the  partners'  accounts  after 
the  following  adjustments  have  been  made: 

Interest  to  be  credited  on  partners'  capital  at  6  %  per  annum. 

Mr.  Adams  owns  the  store,  which  the  partnership  occupies  under  an  agree- 
ment providing  for  an  annual  rent  of  $10,000  payable  in  monthly  installments 
in  advance.  No  rent  has  been  paid  during  the  year.  The  year's  rent  should 
therefore  be  credited  to  Adams,  together  with  $325  interest  on  unpaid  monthly 
installments. 

Of  the  interest  paid  on  notes  payable,  $2,000  applies  to  the  period  subsequent 
to  December  31;  accrued  taxes,  $1,000;  accrued  wages,  $1,500.  A  reserve  of 
$1,500  is  required  to  cover  possible  losses  from  doubtful  accounts. 

Ten  per  cent  of  the  profits,  if  any,  after  the  foregoing  adjustments  have 
been  made,  is  to  be  credited  to  "  Bonuses  to  department  managers  and  salesmen." 


*  C.  P.  A.,  Pennsylvania. 


PARTNERSHIP  197 

The  remaining  profits  or  losses  are  to  be  apportioned  to  the  partners  as 
follows: 

Mr.  Adams  40% 

Mr.  Burk 33i% 

Mr.  Ciine 26$% 

5.*  A  partnership  composed  of  two  members  divides  its  profits  equally,  after 
all  items  of  income  and  expense  for  each  calendar  year  have  been  determined. 
One  of  the  items  of  income  is  interest  on  partners'  withdrawals,  which  is  calcu- 
lated and  charged  to  each  partner  at  the  end  of  the  year.  By  agreement,  the 
interest  calculation  is  made  on  the  partners'  average  monthly  balances  as  shown 
by  the  books.  Partner  A's  account  for  the  calendar  year  19—,  before  interest 
is  charged  to  him,  is  found  to  be  as  follows: 

Debits  Credits 

January  1,  19—,  Balance $  1,080  21     $ 

January  account 6,000  00          550.00 

February  account 2,500  00          550  00 

March  account 3,052  74          550.00 

April  account 13,009  81       9,550  00 

May  account       5 . 45          550  00 

June  account       1,15420          55000 

July  account   1,500  00         550  00 

August  account        1 ,500  00          550  00 

September  account   500  00          550  00 

October  account     1 ,000  00       4,050  00 

November  account       1 ,014  10          550.00 

December  account 1,000  00         550  00 

Show  a  statement  of  the  interest  which  partner  A  should  be  charged  at 
December  31,  19 — ;  simple  interest,  6%  per  annum. 

6.f  A  and  B  are  in  partnership.  A  receives  two-thirds  and  B  one-third  of 
the  profits.  On  November  30,  1933,  the  Profit  and  Loss  account  (after  interest 
on  capital  has  been  charged  at  5%),  shows  a  profit  of  $6,000.  On  December  1, 
1932,  the  start  of  the  year  under  audit,  A  had  a  capital  of  $10,000.00  in  the 
business,  and  during  the  year  he  has  drawn  out  $4,500.00.  B  on  the  same  date 
had  a  capital  of  $8,000.00,  and  during  the  year  has  drawn  out  $1,000.00. 

Make  up  the  two  capital  accounts  as  they  should  appear  on  November  30, 
1933. 

7.f  A,  By  and  C  formed  a  partnership.  A  agreed  to  furnish  $5,000,  B  and  C 
each  $3,500.  A  was  to  manage  the  business,  and  was  to  receive  one-half  of 
the  profits;  B  and  C  were  each  to  receive  one-quarter.  A  supplied  merchandise 
valued  at  $4,250,  but  no  additional  cash.  B  turned  over  to  A,  as  manager, 
$4,500  cash,  and  C  turned  over  $2,750.  The  business  was  conducted  by  A  for 
some  time,  but  exact  books  were  not  kept.  While  manager,  A  purchased  addi- 
tional merchandise  amounting  in  all  to  $37,500,  and  made  sales  amounting  to 
$50,000.  The  cash  received  and  paid  out  for  the  partnership  was  not  kept 
separate  from  A's  personal  cash.  B  took  over  the  management  to  straighten 
out  the  affairs.  He  found  accounts  receivable  amounting  to  $10,000.  Of  these 
he  collected  $2,250.  The  remaining  merchandise  he  sold  for  $250.  These 


*  C.  P.  A.,  North  Carolina. 
fC.  P.  A.,  Indiana. 


198  PARTNERSHIP 

receipts  he  deposited  to  the  firm's  credit  in  the  bank.  The  balance  of  accounts 
receivable  proved  worthless.  The  outstanding  accounts  payable  amounted  to 
$1,000,  of  which  $750  had  been  incurred  in  purchasing  merchandise,  while  $250 
represented  expenses.  B  paid  these  accounts. 

A  presented  receipted  claims,  showing  that  during  his  management  he  had 
paid  other  expenses  of  $1,200.  By  mutual  agreement,  B  was  held  to  be  entitled 
to  $50  on  account  of  interest  on  excess  capital  contributed,  and  A  and  C  were 
each  charged  $37.50  for  shortage  of  contributed  capital. 

(a)  Prepare  the  Trading  and  Profit  and  Loss  accounts  and  the  accounts  of 
each  of  the  partners,  including  the  final  adjustments  to  be  made  at  the  close  of 
the  partnership. 

(6)  Show  how  the  above  final  adjustments  would  be  modified  if  A  proved  to 
have  no  assets  or  obligations  other  than  those  of  the  partnership. 

8.*  A  and  B,  who  are  partners  in  a  trading  firm,  decide  to  admit  C  as  from 
January  1,  1934. 

They  make  an  agreement  with  (\  as  follows: 

C  is  unable  to  contribute  any  tangible  assets  as  his  capital  investment,  but 
agrees  to  allow  his  share  of  the  profits  to  be  credited  to  his  capital  account  until 
lie  shall  have  one-fifth  interest.  (i  is  to  share  profits  and  losses  to  the  extent 
of  one-fifth. 

C  is  to  receive  a  salary  of  $30,000  per  annum,  payable  monthly,  in  addition 
to  his  share  of  the  profits. 

The  balance  sheet  of  A  and  B  at  December  31,  1933,  is  as  follows: 

A  sse  ts  Liabilities 

Cash     $   1,500     Accounts  Payable $  8,000 

Accounts  Receivable  10,000  Capital  Accounts: 

Merchandise 7,500         .1         $10,000 

Furniture  and  Fixtures 1,500         B _5»?99 

Goodwill 2,500  ~~             15,000 

$23,000  $23,000 

During  the  six  months  ended  June  30,  1934,  the  business  has  sustained  unusual 
losses,  and  it  is  decided  to  dissolve  the  partnership. 
The  balance  sheet  at  that  date  is  as  follows: 

A  ssets  Lia  bilitws 

Cash $      500     Accounts  Payable       $12,500 

Accounts  Receivable 12,500  Capital  Accounts: 

Merchandise 5,000        A $10,000 

Furniture  and  Fixtures 1 ,500        B 5,000 

Goodwill 2,500  15,000 

Deficit:  Being  loss  on  trading 

for  6  mos 5,500 

$27,500  $27^00 

Accounts  receivable  were  sold  for  $9,000,  the  buyer  assuming  all  responsi- 
bility for  collection  and  loss,  if  any. 

Merchandise  realized  $6,500,  and  furniture  and  fixtures  $500. 

You  are  asked  to  make  an  examination  of  the  accounts  from  January  1, 
and  to  prepare  statements  showing  the  realization  of  assets,  the  adjustment  of 
the  partnership  accounts,  and  the  distribution  of  funds, 

*  American  Institute  Examination, 


PARTNERSHIP  199 

In  your  examination,  you  find  that  C  has  not  drawn  his  salary  for  four 
months,  and  that  B  has  advanced  to  the  partnership  $2,500  as  a  temporary  loan. 
You  find  that  these  liabilities  are  included  in  the  sum  of  $12,500  shown  as  accounts 
payable. 

C  is  ascertained  to  have  no  assets. 

9.*  A,  B,  and  C  were  in  partnership,  A's  capital  being  $90,000,  7?'s  $50,000 
and  C's  $50,000.  By  agreement,  the  profits  were  to  be  shared  in  the  following 
ratio:  A,  60%;  B,  15%;  (7,  25%.  During  the  year,  C  withdrew  $10,000.  Net 
losses  on  the  business  during  the  year  were  $1 5,000,  and  it  was  decided  to  liquidate. 
It  is  uncertain  how  much  the  assets  will  ultimately  yield,  although  none  of  them 
is  known  to  be  bad.  The  partners  therefore  mutually  agree  that  as  the  assets 
are  liquidated,  distribution  of  cash  on  hand  shall  be  made  monthly  in  such  a 
manner  as  to  avoid,  so  far  as  feasible,  the  possibility  of  one  partner's  being  paid 
cash  which  he  might  later  have  to  repay  to  another.  Collections  are  made  as 
follows:  May,  $15,000,  June,  §13,000;  July,  $52,000.  After  this  no  more  can 
be  collected.  Show  the  partners1  accounts,  indicating  how  the  cash  is  distributed 
in  each  installment;  the  essential  feature  in  the  distribution  is  the  observance 
of  the  agreement  given  above. 

10. f  Brown,  Green,  and  Black  engage  in  a  soliciting  business  under  an 
agreement  that  Brown  is  to  receive  a  salary  of  $200  per  month,  (Jreen  a  salary 
of  $150  per  month,  and  Black  a  salary  of  $100  per  month;  that  the  earnings  are 
to  be  determined  at  any  time  at  the  request  of  any  partner;  and  that  the  profits 
of  the  business  are  to  be  divided  on  the  basis  of  the  amount  of  business  secured 
by  each. 

The  partnership  is  in  business  nine  (9)  months,  and  the  business  record  for 
that  period  is  as  follows: 

Brown's  business          $4,500.00 

Green's  business 2,SOO.OO 

Slack's  business       3,000  00 

Net  profits  of  the  business  amount  to  $5,026.50. 

The  partners  then  decide  to  rescind  the  agreement  as  to  salaries,  and  to 
divide  the  profits  on  the  basis  of  business  secured  individually,  treating  all 
salaries  drawn  as  advances. 

Drawings: 

Brown     $1,000  00 

Green         .    .                    .                                                  ...  1,200  00 

Black 900  00 

You  find  that  the  following  errors  have  occurred  during  the  nine  months: 

Office  furniture  charged  to  expense $  65  00 

Accts.  rec.  (Green's  business)  worthless 210  00 

Cash  advanced  by  Black — credited  to  his  account  as  busi- 
ness secured 400  00 

Items  not  paid  nor  entered  in  the  books: 

Brown's  salary        $200.00 

Green's  salary 150.00 

Advertising      27  50 

Clerk  hire 130.00 


*  American  Institute  Examination, 
t  C.  P.  A.,  Indiana. 


200  PARTNERSHIP 

Telephone 6  00 

Rent 50  00 

Stationery  and  supplies — exp  15  00 

Show  the  journal  entries  necessary  to  readjust  the  accounts.  Make  up  a 
statement  of  the  Profit  and  Loss  account,  showing  all  corrections  and  the  dis- 
tribution of  the  profits. 

11.*  Brown  and  Green  entered  into  a  joint  venture. 

On  May  1,  19—,  they  purchased  5,000  tons  of  coal  in  Philadelphia  at  $4  per 
ton,  f.o.b.,  for  which  they  gave  notes  on  May  10  for  one-half  at  3  months  and 
for  the  other  half  at  6  months.  The  coal  was  shipped  to  Mexico  City  on  May  15, 
the  freight,  and  so  forth,  amounting  to  $5,000. 

A  joint  banking  account  was  opened  on  May  10,  each  party  contributing 
$G,000. 

The  freight  was  paid  by  check  on  May  20,  and  on  May  25  a  check  was  drawn 
for  $1,000  for  charges  at  Mexico  City. 

The  coal  was  sold  at  $7  per  ton,  and  the  proceeds  used  to  purchase  a  cargo 
of  timber,  which  was  shipped  to  Philadelphia.  Freight  and  other  charges 
thereon,  amounting  to  $3,750,  were  paid  by  check  June  30. 

During  July,  four-fifths  of  the  timber  was  sold  for  $32,000.  This  amount 
was  received  and  paid  into  the  joint  account  August  2. 

In  order  to  close  the  transaction,  Brown  agreed  to  take  over  the  remaining 
one-fifth  at  cost  price,  including  freight  and  charges,  and  he  paid  a  check  for 
this  into  the  joint  account  August  10. 

The  first  note  fell  due  and  was  paid  August  13,  and  on  the  same  day  the 
other  note  was  paid  under  discount  at  the  rate  of  4%  per  annum. 

Prepare  accounts  showing  the  results  of  the  foregoing  transactions;  disregard 
interest  on  capital  contributions. 

*  American  Institute  Kxamination. 


CHAPTER  21 
Goodwill 

Definition.  Goodwill  is  an  intangible  asset,  and  may  be 
defined  in  general  terms  as  the  value  of  any  benefits  or  advantages 
which  may  accrue  to  a  business  from  its  being  soundly  established, 
bearing  a  good  reputation,  having  a  favorable  location,  and  so 
forth.  It  results  in  the  earning  of  a  higher  rate  of  net  income  than 
that  of  less  fortunate  concerns  in  the  same  line  of  business. 

Basis  of  valuation.  When  two  or  more  businesses  are  consoli- 
dated or  merged,  the  payment  made  for  each  business  depends 
upon : 

(1)  The  value  of  the  net  assets  of  each  business. 

(2)  The  earning  power  of  each  business. 

A  committee  should  be  formed,  consisting  of  members  from 
each  of  the  businesses  being  consolidated  or  merged  (proprietor- 
ship, firm,  or  corporation);  this  committee  should  have  the  assist- 
ance of  an  appraiser  and  an  accountant  in  the  preparation  of  a 
report  dealing  with  the  net  assets  and  the  earning  power. 

The  report  should  contain  a  balance  sheet  of  each  business, 
stating  the  values  at  which  it  is  proposed  to  take  over  the  assets, 
and  stating  the  liabilities  to  be  assumed. 

The  value  of  the  fixed  assets  and  of  the  inventory  should  be 
determined  by  the  appraiser.  The  accountant,  after  making 
an  audit,  should  submit  the  other  balance  sheet  items. 

Earning  power  determined  from  profit  and  loss  statements. 
The  following  points  should  receive  consideration  when  earning 
power  is  being  determined  from  profit  and  loss  statements : 

(1)  Number  of  years  included.     The  value  of  goodwill  depends 
to  some  extent  on  whether  profits  have  been  uniform  year  after 
year,  or  have  steadily  increased  or  decreased,  or  have  fluctuated 
from  year  to  year.     Therefore,  in  order  to  show  the  trend  of  pro- 
fits, it  is  necessary  to  have  profit  and  loss  statements  for  several 
years.     A  statement  of  average  profits  is  insufficient,  as  it  does 
not  show  the  trend. 

(2)  Adjustments  to  correct  profits.     Adjustments  may  be  neces- 
sary to  correct  errors,  such  as: 

201 


202  GOODWILL 

(a)  Wrong  classification  of  capital  and  revenue  expenditures. 
(6)  Omission  of  provision  for  depreciation,  bad  debts,  and  so 
forth. 

(c)  Inadequate  provision  for  repairs. 

(d)  Anticipation  of  profits  on  consignments  and  sales  for 
future  delivery. 

(3)  Uniformity  of  methods. 

(a)  If  the  methods  of  computing  the  manufacturing  costs 
are  not  uniform,  the  cost  statements  should  be  revissd 
and  put  on  a  uniform  basis. 

(6)  The  depreciation  charges  should  be  analyzed  as  to 
method  and  rate.  If  different  methods  and  rates  have 
been  used,  adjustments  should  be  made  so  that  the 
charges  will  have  been  calculated  on  a  uniform  basis. 

(c)  There  may  be  a  wide  difference  in  the  management 
salaries  paid  by  the  consolidating  companies  for  the  same 
services.     The  salaries  should  be  adjusted.     In  a  single 
proprietorship  or  partnership,  salaries  may  not  have  been 
paid  or  credited;  in  that  case  they  should  be  included  at 
an  arbitrary  figure. 

(d)  If,  iii  a  partnership,  interest  on  capital  has  been  charged 
as  an  expense,  the  entries  should  be  reversed  and  the  item 
of  interest  on  capital  thus  eliminated. 

(4)  Eliminations.     Eliminations    may    have    to  be  made   for 
extraordinary  and  non-operating  profits  or  losses. 

Methods  of  valuing  goodwill.  Goodwill  may  be  valued  on  the 
basis  of: 

(1)  An  appraisal  of  goodwill. 

(2)  A  number  of  years'  purchase  price  of  the  net  profits. 

(3)  A  number  of  years1  purchase  price  of  excess  profits  over 
interest  on  net  assets. 

Capitalization  of  profits  in  excess  of  interest  on  net  assets  is 
usually  calculated  as  follows: 

Net  assets $100,000.00 

Profits 10,000  00 

Interest  on  net  assets  ©  6% 6,000  00 

Excess  of  profits  over  interest 4,000  00 

Excess  capitalized  at  20%  (4,000  +  .20) 20,000  00 

Case  illustrations.  The  following  four  cases  of  goodwill  valua- 
tion, taken  from  reports  of  consolidations,  show  how  goodwill  has 
been  valued  in  practice. 


GOODWILL  203 

Case  1.  The  goodwill  of  the  consolidating  units  was  fixed  at  the 
sum  of  the  profits  for  the  two  preceding  years,  plus  an  additional 
10%. 

Case  2.  The  goodwill  was  based  on  the  total  profits  for  the  five 
years  preceding,  less  five  years'  interest  on  the  net  worth. 

Case  3.  The  goodwill  was  the  average  annual  earnings  for  the 
four  years  preceding  consolidation,  less  the  following  deductions: 

(a)  Profits  on  favorable  contracts  about  to  expire. 

(6)  $100,000  for  the  estimated  value  of  services  rendered  by 

the  retiring  president. 
(c)  6%  interest  on  actual  capital  invested. 

The  remainder  was  capitalized  on  a  10%  basis. 

Case  4.  From  the  net  profits  of  each  company  the  following 
items  were  deducted: 

(a)  7%  on  capital  actually  employed. 

(fr)  li%  on  sales. 

(f)  2%  depreciation  on  brick  buildings, 

(rf)  4%  depreciation  on  frame  buildings. 

(e)  8%  depreciation  on  machinery. 

The  remainder  was  capitalized  at  20%,  or  5  times  the  amount  of 
such  earnings  in  excess  of  7%  on  capital  and  other  deductions 
agreed  upon. 

Valuation  by  appraisal.  There  is  no  particular  problem  in  the 
calculation  of  the  value  of  goodwill  by  appraisal.  It  may  be 
appraised  by  a  disinterested  party;  or,  more  often,  it  is  the  amount 
on  which  the  vendor  and  the  vendee  agree.  They  usually  appraise 
the  net  assets,  and  agree  that  the  purchase  price  shall  be  a  certain 
amount  in  excess  of  the  value  of  the  net  assets.  This  excess  is  the 
payment  for  goodwill. 

Valuation  by  number  of  years'  purchase  price  of  net  profits. 
The  goodwill  may  be  estimated  at  so  many  years'  purchase  price 
of  the  net  or  gross  profits  of  any  one  year,  or  at  so  many  years' 
purchase  price  of  the  average  profits  of  a  number  of  years. 

Example 

The  consideration  of  the  sale  of  a  business,  as  agreed  to  between  the  parties, 
is  four  years'  purchase  price  of  the  average  profits  for  the  preceding  three  years, 
plus  the  net  value  of  the  assets. 

Net  value  of  assets          $100,000 

Profits  of  preceding  three  years: 

1st  year $20,000 

2nd  year 15,000 

3rd  year 28,000 

What  is  the  selling  price  of  the  business? 


204  GOODWILL 

Solution 

Net  value  of  assets $100,000 

Profits  of  preceding  three  years: 

1st  year          $20,000 

2nd  year 15,000 

3rd  year 28,000 

$63,000 

$63,000  -s-  3  =  $21,000,  average  profits  for  three 
years. 

$21,000  X  4  (goodwill) 84,000 

Selling  price     $184,000 

Valuation  on  basis  of  excess  of  profits  over  interest  on  net 
assets.  The  value  of  goodwill  is  calculated  under  this  method  by, 
first,  deducting  from  the  average  profits  a  fair  return  of  interest  on 
the  capital  invested,  and,  second,  by  multiplying  the  remainder  of 
the  profits,  or  the  excess,  by  an  agreed  number  of  years'  purchase 

price. 

Example 

A  agrees  to  buy  a  certain  business,  and  to  pay  for  it  in  cash.  He  agrees  to 
give  dollar  for  dollar  of  the  value  of  the  net  assets,  plus  a  six  years7  purchase 
price  of  the  excess  of  the  profits  over  the  interest  on  capital  at  6%.  Net  assets 
are  valued  at  $100,000,  and  average  profits  are  $18,000.  What  is  the  purchase 
price  of  the  business,  including  goodwill? 

Solution 

Net  assets $100,000 

Profits,  average $18,000 

$100,000  X  .06 _°j9°0 

Excess  profits «T2JOOO 

$12,000  X  6  (goodwill) 72,000 

Purchase  price     $172^)00 

It  would  be  more  favorable  to  the  seller  to  determine  the  value  by  using  a 
higher  rate  of  interest  and  capitalizing  the  excess  profits  at  this  rate;  thus: 

Net  assets $100,000 

Profits,  average $18,000 

$100,000  X  .08 8,000 

Excess  profits $10,000 

$10,000  -T-  .08  (goodwill) 125,000 

Purchase  price $225,000 

$225,000  -  $172,000  «  $53,000,  advantage  to  the  seller. 

In  the  foregoing  example,  the  goodwill  represents  the  capitali- 
zation of  that  portion  of  the  profits  which  is  not  attributable  to  the 
net  tangible  assets.  The  rate  to  be  used  depends  largely  on  the 
kind  of  business  under  consideration.  In  some  lines  of  business 
the  per  cent  may  be  as  low  as  6%  or  8%;  in  others  it  may  be  10%; 
and  in  still  others  15%,  or  even  20%. 


GOODWILL 


205 


Basis  of  stock  allotment.  Since  most  phases  of  the  calculation 
of  the  value  of  goodwill  are  found  in  consolidations,  an  example  of 
consolidation  is  given.  The  matter  of  stock  allotment  is  included, 
because  when  an  agreement  has  been  reached  as  to  the  valuation 
of  the  assets  and  as  to  the  earning  power  of  each  of  the  businesses, 
the  next  question  to  decide  is  the  method  of  making  payment. 

The  following  three  typical  methods  will  be  presented: 

(1)  Payment  entirely  in  common  stock. 

(2)  Payment  in  preferred  stock  for  the  net  assets;  payment  in 
common  stock  for  the  goodwill. 

(3)  Payment  in  bonds  for  the  fixed  assets,  or  for  an  agreed 
percentage  thereof;  payment  in  preferred  stock  for  the  balance  of 
the  net  assets;  payment  in  common  stock  for  the  goodwill. 

In  the  allotment  of  securities,  the  fundamental  rule  is  to  dis- 
tribute them  in  such  a  manner  that,  if  the  income  of  the  consolida- 
tion is  the  same  as  the  combined  income  of  the  several  businesses, 
oach  of  the  old  businesses,  or  the  former  owners  or  stockholders 
thereof,  will  receive  the  same  net  income  as  before  the  consolidation. 

To  illustrate  how  this  principle  would  operate  under  each  of  the 
three  methods  outlined,  assume  that  three  companies  are  to  be 
consolidated  on  the  basis  of  the  following  statements : 


Net  assets $40,000 

Average  earnings 4,000 

Rate    of    income    on    net 

assets  10% 


H 

$60,000 
12,000 


20 


Vo 


C 

$120,000 
20,000 

10-1% 


Total 
$220,000 
30,000 


Common  stock  only.  When  only  common  stock  is  to  be  issued, 
it  must  be  issued  in  the  ratio  of  the  net  earnings  if  the  income  of  the 
consolidation  is  to  be  distributed  in  the  ratio  in  which  the  com- 
panies contributed  earnings.  To  determine  the  amount  of  stock 
which  is  to  be  issued,  capitalize  the  earnings  by  dividing  the  income 
of  each  company  by  a  rate  of  income  agreed  upon.  Thus,  if  it  is 
agreed  that  the  rate  be  10%,  the  distribution  of  common  stock  is 
made  as  follows : 

A  B  C  Total 

Stock  to  he  issued: 

A:  $4,000  -T-  .10 $40,000 

B:  $12,000  -5-  .10 $120,000 

C:  $20,000  -T-  .10 $200,000 

Total $360,000 

Less  net  assets  trans- 
ferred      40,000        60,000     ^20,000    _220,000 

Goodwill 0    $  60,000    $~80^KK)    $140,000 


206 


GOODWILL 


Ten  per  cent  was  chosen  as  the  basic  rate,  because  it  was  the 
lowest  rate  earned  by  any  one  of  the  three  companies. 

If  the  profits  of  the  consolidation  amount  to  $36,000,  it  will  be 
possible  to  pay  a  10%  dividend,  which  would  be  distributed  as 
follows  : 

A:  10%  of  $40,000  ..........................  $  4,000 

B:  10%  of  $120,000     ...............................      12,000 

C:  10%  of  $200,000  ........................    __20i0f)0 

$36jKJO 

This  is  an  equitable  division,  so  far  as  profits  are  concerned. 
However,  it  is  objectionable  because  it  gives  each  old  company  an 
interest  in  the  assets  which  is  proportionate  to  the  profits  earned 
before  the  consolidation,  instead  of  an  interest  proportionate  to  the 
assets    contributed.     This   might   work   a   hardship   in   case    of 
liquidation. 

Net  Assets 
A  .....................  $  40,000 

B  .....................       60,000 

C  .....................      120,000 

$220,000 


Goodwill 
0 

$60,000 
^  80,000 
$140,000 


Total 

$  40,000 

120,000 

200,000 

$360,000 


Fraction 


Assume  that  after  a  number  of  years  it  is  decided  to  liquidate 
the  consolidated  company,  and  that  in  the  meantime,  all  of  the 
profits  have  been  paid  out  as  dividends.  The  goodwill  lias  no 
realizable  value,  so  there  is  $220,000  to  be  distributed  as  follows : 


Former  stockholders  of  A : 
Former  stockholders  of  B: 
Former  stockholders  of  C: 


of  $220,000 
of    220,000 
fSHrof    220,000 


$  24,444.45 

....       73,333  33 
.      122,222  22 

$220,000  00 
lose,   and  the  former 


The  former  stockholders  of  A   would 
stockholders  of  B  and  C  would  profit. 

Former 

Stockholders  Assets 

of  Company  Contributed 

A $  40,000 

B 60,000 

C J  20,000 

$220,000 

Preferred  stock  for  net  assets.  In  order  to  avoid  giving  an 
advantage  to  one  or  more  companies  at  the  expense  of  the  others, 
it  is  advisable  to  issue  preferred  stock  for  the  net  assets,  and 
common  stock  for  the  goodwill.  The  goodwill  should  be  allotted 
to  the  several  companies  in  the  ratio  of  the  excess  of  the  profits 
contributed  over  the  dividends  on  the  preferred  stock. 


Liquidating 

Dividend 

Gain 

Loss 

$  24,444.45 

$15,555.55 

73,333  33 

$13,333  33 

122,222  22 

2,222  22 

$220,000  00 

$15,555  55 

$15,555  55 

GOODWILL  207 

Assume  that  in  the  above  illustration  6%  stock,  preferred  as  to 
assets,  is  to  be  issued  for  the  net  assets,  and  that  common  stock  is 
to  be  issued  for  the  goodwill. 

ABC          Total 

Earnings  $4,000     $12,000     $20,000     $3l>,000 

Less   dividends  on   preferred 
stock: 

A:    6 9;  of  $  40,000 2,400 

B.    6%  of      60,000 3,600 

C:    6%  of    120,000 7,200 

Excess  earnings $1,600     $  S,400     $12,SOO 

Common  stock  should  be  issued  in  the  ratio  of  the  excess 
earnings.  If  five  years'  purchase  of  the  excess  profits  were  agreed 
upon,  the  distribution  of  stock  would  be: 

.4  B  r  Total 

Preferred  stock $40,000    $60,000     $120,000    $220,000 

Common  stock S,()00       42,000         64,000       1 14,000 

Assuming  profits  of  $,36,000  as  before,  the  distribution  of 
dividends  would  be: 

Profits $36,000 

Preferred  dividends:  6%  of  $220,000 13,200 

Balance  available  for  common  stock  dividends $22, SCO 

Then,  $22,800  4-  $114,000  -  20%,  the  rate  per  cent  which 
could  be  paid  on  the  common  stock. 

A  B  C 

Preferred  dividends: 

6%  of  $  40,000  $2,400 

6%  of      60,000  $  3,600 

6%  of     120,000  ...  $  7,200 

Common  dividends: 

20%  of  $  X,000  .  .      1,600 

20%  of    42,000  8,400 

20%  of    64,000  _12'800 

Total  dividends       .  $4,000     JJM2JOOO     $20,000 

These  dividends  are  in  each  case  equal  to  the  profits  contrib- 
uted to  the  consolidation  by  the  several  companies. 

It  is  important  to  note  that  goodwill  should  be  based  on  the 
profits  contributed  minus  the  profits  to  be  returned  as  preferred 
dividends,  and  not  on  the  total  profits. 

Bonds,  preferred  stock,  and  common  stock.  If  bonds  are 
issued  for  a  percentage  of  the  net  assets,  preferred  stock  for  the 
remaining  net  assets,  and  common  stock  for  the  goodwill,  the  good- 
will should  be  based  on  the  profits  turned  in  minus  the  bond 
interest  and  the  preferred  dividends. 


208  GOODWILL 

Assume  that  5%  bonds  are  to  be  issued  for  80%  of  the  net 
assets,  6%  preferred  stock  for  the  remaining  net  assets,  and 
common  stock  for  the  goodwill,  which  is  to  be  computed  by  capita- 
lizing at  20%  the  earnings  of  each  company  in  excess  of  bond 
interest  and  preferred  dividends  to  be  paid  to  former  stockholders. 
The  issues  of  the  three  classes  of  securities  would  be  computed  as 
follows : 

.4  B  C          Total 

Bonds: 

A:  80%  of  $  40,000 $32,000 

J3:80%of      60,000     ....  $48,000 

C:  80%  of    120,000  ....  $96,000 

Total  bonds $176,000 

Preferred  Stock: 

yl:20%of  $  40,000  8,000 

B:  20%  of      60,000  ....  12,000 

C:20%of    120,000.     ..  24,000 

Total  preferred  stock..    ..  44,000 

Common  Stock: 

A:  Earnings $  4,000 

B:  Bond  interest $1 ,600 

Pfd.  dividend 480       2,080 

Excess '  $  1,920 

$1,920  -T-  .20 "  ~      ^  $  9,600 

B:  Earnings $12,000 

Bond  interest $2,400 

Pfd.  dividend 720  _3, 1 20 

Excess $  8,880 

$8,880  4-  .20 '  $44,400 

C:  Earnings $20,000 

Bond  interest $4,800 

Pfd.  dividend 1 ,440       6,240 

Excess "  *^f760 

$13,760  -*-  .20 ""  $68,800 

Total  common  stock. ..    .  $122,800 

With  profits  of  $36,000  before  allowance  for  bond  interest  and 

preferred  dividends,  the  former  stockholders  would  receive  interest 

and  dividends  as  follows: 

A  B  C          Total 

Bond  interest: 

5%  of  $32,000 $1,600 

5%  of    48,000 .  $  2,400 

5%  of    96,000 .  $4,800 

Total $  8,800 

Preferred  dividends: 

6%  of  $  8,000 480 

6%  of    12,000 720 

6%  of    24,000 1,440 

Total 2,640 


GOODWILL  209 

Common  dividends: 

20%  of  $  9,600 $1,920 

20%  of    44,400 $  8,880 

20%  of    68,800 $13,760 

Total $24,560 

Total  distribution f4^     IH£22     $20,000     $36,000 

Conclusion.  The  illustrations  given  are  merely  indicative  of 
the  principles  to  be  borne  in  mind  in  the  distribution  of  stock  and 
other  securities;  they  cannot  be  accepted  as  procedures  to  be 
invariably  followed,  for  several  reasons. 

First,  in  the  illustrations,  the  profits  of  the  consolidation  are 
assumed  to  be  the  same  as  the  combined  profits  of  the  separate 
companies  before  they  were  consolidated.  However,  consolidations 
are  usually  made  with  the  object  of  increasing  profits;  hence  the 
question  is  raised  as  to  how  the  additional  profits  should  be  divided. 
Should  the  preferred  stock  be  participating  or  non-participating? 

Second,  the  question  of  control  involves  the  matter  of  the 
voting  rights  of  the  several  classes  of  stock. 

These  and  other  considerations  would  tend  to  cause  modifica- 
tions in  the  methods  described,  but  the  illustrations  serve  to 
indicate  the  basic  principles  which  must  be  followed  in  security 
allotment  in  order  that  the  stockholders  of  the  several  consolidating 
companies  may  preserve  their  interests  in  the  assets  and  earnings 
of  the  consolidation. 

Problems 

1.  A,  B,  and  C  are  about  to  consolidate.     The  following  data  are  presented; 

A  B  (J  Total 

Net  Assets $250,000  $150,000  $600,000  $1,000,000 

Average  Profits 50,000  15,000  150,000  215,000 

Interest  Hate 10% 

Profit  Rate      20%  10%  25% 

Prepare  tabulations  showing  the  stock  distribution: 

(a)  Preferred  stock  for  the  net  assets,  and  common  stock  for  the  goodwill. 

(6)  Show  possible  disadvantage  of  issuing  only  common  stock. 

2.  Using  the  data  in  Problem  1,  show  the  security  allotment  if  5%  bonds 
are  issued  for  80%  of  the  net  assets,  6%  preferred  stock  for  the  remainder,  and 
common  stock  for  the  goodwill,  which  is  to  be  based  on  excess  earnings  capitalized 
at  15%. 

3.  A,  B,  and  C  call  upon  you  to  draw  up  plans  for  their  consolidation.     They 
submit  the  following  information: 

Assets  A  B  C 

Plants $3.50,000  $200,000  $180,000 

Materials 100,000  20,000  20,000 

Accounts  Receivable 80,000  60,000  40,000 

Cash 20,000  10,000  10,000 


210 


GOODWILL 


Liabilities  ABC 

Accounts  Payable $  70,000    $  30,000     $  20,000 

Capital 350,000       200,000       100,000 

Surplus 130,000         60,000       130,000 

Average  income 30,000        35,000        40,000 

Upon  your  recommendation,  the  consolidation  will  issue:  (a)  6%  bonds  for 
the  fixed  assets;  (b)  7%  preferred  stock  for  the  remaining  net  assets;  (c)  common 
stock  for  the  goodwill,  which  is  to  be  based  on  excess  profits  capitalized  at  10%. 

Assuming  that  the  consolidation  will  have  net  profits  amounting  to  $105,000, 
prepare  statements  showing  the  allotment  of  securities  and  the  distribution  of 
profits. 

4.*  The  net  worth  and  profits  of  three  companies  are  as  follows: 

X 

Capital $100,000 

Profits 50,000 

r 

Capital          200,000 

Profits 50,000 

Z 

Capital     250,000 

Profits 50,000 

(a)  Give  your  theory  of  how  a  consolidation  should  be  made. 

(b)  Show  the  respective  interests  of  A',  of  Y,  and  of  Z  in  the  consolidated 
company,  using  a  factor  of  6%  to  represent  the  normal  value  of  money. 

5.f  A  has  agreed  to  sell  to  B  the  goodwill  of  the  X.  Y.  Company  on  the  basis 
of  three  years'  profits  of  the  business,  which  arc  to  be  determined  by  you,  on 
sound  principles  of  accounting  and  as  accurately  as  possible,  from  the  follow1  ng 
statement  handed  you  by  A.  You  are  required  to  compute  the  value  of  the 
goodwill,  but  are  not  expected  to  take  into  account  any  considerations  except 
those  presented  by  the  statement. 


Credits  1st  Year 

Sales  (selling  prices  substantially  uni- 
form throughout  period) $038,400 

Estimated  value  of  construction  work 
performed  arid  charged  to  property       110,000 

Appreciation  of  real  estate  upon  re- 
valuation by  experts 

Profit  on  sale  of  Bethlehem  Steel  Co. 
stock 

Inventory  at  end  of  period: 

Production  material  at  cost  72,000 

Finished  goods  at  selling  prices 76,500 

$8967900 


2nd  Year 

$602,500 

77,600 

80,000 


103,100 
114,000 


3rd  Year 
$    564,000 
154,000 

85,000 

106,600 

150,000 

$977~200         $1,059,600 


*  C.  P.  A.,  Michigan. 

t  American  Institute  Examination. 


GOODWILL 


211 


Debits 

Production  materials  purchased $233,000 

Production  labor 50,850 

Production    expense     (including     de- 
preciation)    66,750 

Helling  expenses 52,500 

Interest     96,000 

Cost  of  construction  work       74,600 

Inventory  at  beginning  of  period: 

Production  material  at  cost 51,400 

Finished  goods  at  selling  prices 54,900 

ioso^dod 

Balance,  being  profit  claimed  by  A. ..     $216,900 

6.*  In  the  preceding  problem,  does  the  basis  used  for  arriving  at  the  value 
of  the  goodwill— three  years'  piofits- -appear  to  you  to  be  reasonable  in  view 
of  the  facts  disclosed  to  you?  If  not,  what  advice  would  you  offer  upon  the 
question  if  A  or  It  were  your  client? 

7.*  .1  and  H  are  partners  in  business,  and  have  the  following  statement: 


$252,400 
61,400 

$  220,300 
60,900 

69,300 
55,650 
94,000 
49,000 

70,300 
62,800 
98,500 
86,000 

72,000 
76,500 
$730,250 
$246,950 

103,100 
114,000 
$  815,900 
$  243,700 

Store     ...  

Accounts  Receivable   .    .  . 

Cash  

Furniture  and  Fixtures     . 
Merchandise 
Miscellaneous  Equipment 


$15,000 

12,000 

9,000 

2,S()0 

37,000 

4,200 

$80,000 


Accounts  Payable $10,000 

Bills  Payable 5,000 

-t's  Capital   30,000 

#'s  Capital 35,000 


$80,000 

C  is  admitted  as  a  special  partner,  under  the  following  arrangement:  C  is  to 
contribute  $30,000,  and  is  to  be  entitled  to  one-third  of  the  profits  for  1  year. 
Before  the  contribution  is  made,  the  following  changes  are  to  be  made  in  the 
books:  store  to  be  marked  down  5%;  allowance  for  doubtful  accounts  to  be 
created,  amounting  to  2 ',<',;  merchandise  to  be  revalued  at  $35,000;  furniture 
and  fixtures  to  be  revalued  at  $2,500.  At  the  end  of  the  year,  the  goodwill  is 
to  be  fixed  at  3  times  the  net  profits  for  the  year  in  excess  of  $20,000,  this  good- 
will to  be  set  up  on  the  books  and  the  corresponding  credit  to  be  to  A  and  II 
equally.  A,  B,  and  C  are  each  to  draw  $3,000  in  cash,  and  the  remaining  profits 
are  to  be  carried  to  their  capital  accounts. 

During  the  year,  the  following  transactions  took  place: 

Merchandise  bought  on  credit $240,000 

Cash  purchases 25,000 

Cash  sales 125,000 

Sales  on  credit 175,000 

Accounts  payable  paid  (face,  $245,000;  discount,  2%) . . .  240,100 
Accounts  receivable  collected  (face,  $170,000;  all  net  ex- 
cept $50,000,  on  which  2%  was  allowed)             169,000 

Buying  expenses,  paid  cash   ....          1,500 

Selling  expenses,  paid  cash 21,000 

Delivery  expenses,  paid  cash      9,000 

Management  expenses,  paid  cash     4,500 

Miscellaneous  expenses,  paid  cash 3,000 

Interest  on  notes  payable,  paid  cash 250 

*  American  Institute  Examination. 


212  GOODWILL 

The  partners  each  withdrew  $3,000  cash,  as  agreed. 

When  the  books  were  closed  for  the  purpose  of  determining  the  profits  and 
goodwill,  the  following  were  agreed  upon: 

Value  of  merchandise  on  hand $60,000 

Depreciation  on  store 285 

Additional  allowance  for  doubtful  debts 165 

Furniture  and  fixtures  written  down 200 

The  goodwill  having  been  estimated  and  duly  entered,  C  then  contributes 
enough  cash  to  make  his  capital  account  equal  one-third  of  the  total  capital. 

Prepare  statements  showing  how  the  accounts  are  to  be  adjusted,  and  pre- 
pare the  balance  sheet  after  the  final  adjustment. 


CHAPTER  22 
Business  Finance 

Stock  rights.  Corporations,  in  undertaking  to  secure  addi- 
tional capital,  not  infrequently  offer  additional  stock  to  their 
stockholders  at  a  price  below  the  prevailing  market  quotation  of 
the  outstanding  shares. 

'Phis  privilege  of  subscribing  has  value  as  long  as  the  market 
price  of  the  old  stock  remains  higher  than  the  offering  price  of  the 
new  stock;  and  if  the  stockholders  prefer  not  to  exercise  the  rights, 
they  may  sell  them  in  the  market  for  whatever  they  will  bring. 

Example 

A  corporation  has  a  capital  stock  of  $100,000,  divided  into  1,000  common 
shares  of  $100  par  value.  The  entire  amount  is  outstanding,  and  the  market 
quotation  is  $150.  Finding  that  $50,000  additional  capital  is  needed,  the 
directors  decide  to  offer  to  the  stockholders  500  shares  of  new  common  stock 
at  $125.  They  accordingly  announce  on  August  1  that  stockholders  of  record 
as  of  September  1  will  have  the  privilege  of  subscribing  for  the  new  issue  in  the 
proportion  of  one  share  of  the  new  stock  for  every  two  shares  of  the  old  stock 
held  on  the  latter  date.  The  subscriptions  are  payable  on  or  before  October  1 
following,  and  transferable  warrants  for  the  rights  are  to  be  issued  as  soon  as 
practicable  after  September  1.  What  is  the  value  of  a  right? 

Explanation.  According  to  the  conditions  of  this  offer,  every  holder  of  two 
of  the  old  shares  at  the  close  of  business  on  September  1  will  be  entitled  to 
subscribe  to  one  of  the  new  shares.  He  will  therefore  come  into  possession  of 
two  "  rights/'  as  that  term  is  used  on  the  New  York  Stock  Exchange,  or  one 
right  for  every  old  share  held.  (On  some  stock  exchanges  the  term  right  indi- 
cates the  privilege  of  subscribing  to  one  share  of  the  new  issue.) 

Trading  in  the  rights  will  begin  following  the  declaration  of  the  directors 
on  August  1,  and  will  continue  until  October  1.  Until  the  warrants  are  in  the 
hands  of  the  stockholders — during  the  period  from  August  1  to  September  1 — 
the  trading  will  be  on  a  "when  issued"  basis;  that  is,  delivery  and  payment  for 
the  rights  will  be  made  when  the  warrants  are  available.  During  this  time  the 
stock  will  sell  "rights-on";  that  is,  the  market  value  of  the  shares  will  include 
the  value  of  the  rights. 

With  the  delivery  of  the  warrants  on  September  1,  the  stock  will  sell 
"ex-rights,"  its  price  no  longer  including  the  value  of  the  rights.  With  the 
issuance  of  the  warrants,  and  until  October  1,  trading  in  the  rights  will  be  for 
immediate  delivery  and  payment;  that  is,  delivery  and  payment  the  day  after 
the  sale  is  made. 

Should  a  holder  of  two  shares  exercise  his  privilege  of  subscription,  he  would 
own: 

213 


214  BUSINESS  FINANCE 

2  shares  @  $150 $300 

!_  share  ©  125 125 

3  shares  @    141.67  $425 

It  will  be  noticed  that  the  difference  between  the  market  price  of  the  old 
stock  and  the  average  price  of  the  three  shares  is  $8.33,  the  value  of  a  right;  also 
that  the  difference  between  the  market  price  of  the  old  stock  and  the  offering 
price  of  the  new  stock  is  $25.00,  or  three  times  the  value  of  a  right. 

Therefore,  the  following  formula  may  be  used: 


Formula 
Market  price  —  Offering  price 


-  =  Value  of  a  right. 


Number  of  rights  to  purchase  1  share  -f-  1 

Substitution 
150-125      25 
-2+-l-=8->or*U3- 

During  the  second  period — that  is,  while  the  stock  is  quoted  ex-rights — the 
value  of  the  rights  may  be  ascertained  as  follows: 

Formula 

Market  price  —  Offering  price 

' 1,1 

Number  of  rights  to  purchase  I  share 

It  will  be  found  that  the  market  price  of  the  rights  during  the  second  period 
will  tend  to  coincide  with  this  value.  Any  appreciable  difference  opens  an 
opportunity  for  a  profit. 

The  foregoing  discussion  and  example  apply  only  to  values  on 
the  market.  The  profit  or  loss  resulting  from  the  sale  of  rights, 
and  the  profit  or  loss  from  the  sale  of  stock  acquired  by  the  exercise 
of  rights,  are  governed  by  Section  29.22(a)-8,  Regulations  111. 

Sale  of  stock  and  rights,  federal  income  tax.  Ordinarily, 
a  stockholder  derives  no  taxable  income  from  the  receipt  of  rights 
to  subscribe  for  stock,  nor  from  the  exercise  of  such  rights,  but  if  he 
sells  the  rights  instead  of  exercising  them,  he  may  derive  taxable 
income,  or  sustain  a  loss. 

The  following  rule  is  stated  in  Sec.  29.22(a)-8,  Regulations  111. 

"(1)  If  the  shareholder  does  not  exercise,  but  sells,  his  rights 
to  subscribe,  the  cost  or  other  basis,  properly  adjusted,  of  the  stock 
in  respect  of  which  the  rights  are  acquired  shall  be  apportioned 
between  the  rights  and  the  stock  in  proportion  to  the  respective 
values  thereof  at  the  time  the  rights  are  issued,  and  the  basis  for 
determining  gain  or  loss  from  the  sale  of  a  right  on  one  hand  or  a 
share  of  stock  on  the  other  will  be  the  quotient  of  the  cost  or  other 
basis,  properly  adjusted,  assigned  to  the  rights  or  the  stock, 
divided,  as  the  case  may  be,  by  the  number  of  rights  acquired  or 
by  the  number  of  shares  held." 


BUSINESS  FINANCE  215 

Example 

A  purchased  100  shares  of  stock  at  $125.00  a  share,  and  in  the  following 
year  the  corporation  increased  its  capital  by  20%.  A,  therefore,  received  100 
rights,  entitling  him  to  subscribe  to  20  additional  shares  of  stock;  the  subscription 
price  was  $100.00  a  share.  Assume  that  at  the  time  that  the  rights  were  issued 
the  stock  had  a  fair  market  value  of  $120.00  a  share,  and  that  the  rights  had  a 
fair  market  value  of  $3.00  each.  If,  instead  of  subscribing  for  the  additional 
shares,  A  sold  the  rights  at  $4.00  each,  his  taxable  gain  would  be  computed  as 
follows: 

100  shares®  $125.00...  .  $12,500.00,  cost  of  stock  in  respect  of 

which  rights  were  issued 

100  shares  ®  $120.00.      .  $12,000.00,  market  value  of  old  stock 

100  rights  @  $3.00  .        .  $300.00,  market  value  of  rights 
12  000 

T9^nri  °^  12>^ $12,195.12,  cost  of  old  stock  apportioned 

onn  ^°  surn  stock  after  issuance  of  rights 

3  °-  of  12,500 $304.88,  cost  of  old  stock  apportioned 

12,300  to  rights 

100  rights  @  $4.00     .  .    .  $400.00,  sales  price  of  rights 

$400.00  -  $304.88  $95.12,  profit  on  sale  of  rights 

For  the  purpose  of  determining  the  gain  or  loss  from  the  subsequent  sale  of 
the  stock  in  respect  of  which  the  rights  were  issued,  the  adjusted  cost  of  each 
share  is  $121.95— that  is,  $12,195.12  -f-  100. 

Rule  2  of  Sec.  29.22(a)-8,  Regulations  111,  states: 

"(2)  If  the  shareholder  exercises  his  rights  to  subscribe,  the 
basis  for  determining  gain  or  loss  from  a  subsequent  sale  of  a  share 
of  the  stock  in  respect  of  which  the  rights  were  acquired  shall  be 
determined  as  in  paragraph  (1).  The  basis  for  determining  gain 
or  loss  from  a  subsequent  sale  of  a  share  of  the  stock  obtained 
through  exercising  the  rights  shall  be  determined  by  dividing  the 
part  of  the  cost  or  oth^r  basis,  properly  adjusted,  of  the  old  shares 
assigned  to  the  rights,  plus  the  subscription  price  of  the  new  shares, 
by  the  number  of  new  shares  acquired." 

Example 

A  purchased  100  shares  of  stock  at  $125.00  a  share,  and  in  the  following  year 
the  corporation  increased  its  capital  by  20%.  A,  therefore,  received  100  rights 
entitling  him  to  subscribe  to  20  additional  shares  of  stock;  the  subscription  price 
was  $100.00  a  share.  Assume  that  at  the  time  that  the  rights  were  issued  the 
stock  had  a  fair  market  value  of  $120.00  a  share,  and  that  the  rights  had  a  fair 
market  value  of  $3.00  each.  A  exercised  his  rights  to  subscribe,  and  later  sold 
for  $140.00  a  share  10  of  the  20  shares  thus  acquired.  The  profit  is  computed 
as  follows: 

Cost  of  old  stock  apportioned  to  rights  in  accordance 
with  the  computation  in  the  example  under  Rule  1 ...  $    304  88 

Subscription  price  of  20  shares  at  $100.00  a  share 2,000.00 

Basis  for  determining  gain  or  loss  from  sale  of  shares 
acquired  by  exercise  of  rights $2,304.88 


216  BUSINESS  FINANCE 

$2,304.88  -T-  20  =  $115.24,  basis  for  determining  gain  or  loss  from  sale  of 
each  share  of  stock  acquired  by  exercise  of  rights. 

Proceeds  of  sale: 

10  shares  <&  $140.00  $1,400  00 

Cost  of  stock  sold : 

10  shares  @  $115.24  1,152  40 

Profit $  247  "60 

The  basis  for  determining  the  gain  or  loss  from  the  subsequent  sale  of  the 
remaining  10  shares  of  stock  acquired  on  subscription  is  $115.24  a  share;  and  the 
basis  for  determining  the  gain  or  loss  on  the  stock  in  respect  of  which  the  rights 
were  issued  is  $121.95  a  share — that  is,  $12,195.12  -T-  100,  as  in  the  example 
under  Rule  1. 

Problems 

1.  A  company  has  a  capital  stock  of  $1,000,000,  divided  into  10,000  common 
shares  of  $100  par  value.     The  entire  amount  is  outstanding,  and  the  market 
quotation  is  $150  a  share.     Finding  that  $500,000  of  additional  capital  is  needed, 
the  directors  decide  to  offer  to  the  stockholders  5,000  shares  of  new  common 
stock  at  par.     What  is  the  approximate  market  value  of  a  right  if  each  stock- 
holder may  subscribe  for  one  share  of  new  stock  for  every  two  shares  of  the  old 
stock  held? 

2.  A  corporation  offered,  at  $100  a  share,  one  share  of  its  new  stock  for  each 
six  shares  held.     The  stock  was  selling  at  $185  a  share  when  the  offer  was 
announced.     What  was  the  approximate  market  value  of  a  right? 

3.  W  owned  100  shares  of  Purity  Baking  Common  that  cost  him  $1.3,400. 
Later,  he  received  rights  to  subscribe  to  additional  stock,  but  since  he  did  not 
care  to  increase  his  investment,  he  sold  the  rights  at  3f  less  commission,  receiving 
therefor  $357.30.     At  the  date  when  the  stock  was  quoted  ex-rights,  the  average 
market  values  were: 

Stock 124i 

Rights 3f 

(a)  What  was  W's  loss  on  the  sale  of  the  rights? 
(6)  What  was  the  carrying  value  of  the  stock? 

4.  Smith  owned  100  shares  of  common  stock  in  the  W.  Corporation,  which 
offered  rights  to  subscribe  to  new  common  stock  at  $100  a  share,  the  basis  of 
the  offering  being  one  share  for  each  five  shares  held.     The  average  market 
values  on  the  date  when  the  stock  sold  ex-rights  were: 

Stock 150.50 

Rights 11.8125 

Smith  later  sold  his  rights  at  $14.50. 

(a)  if  Smith  paid  $120  a  share  for  the  original  100  shares,  what  is  his  profit 
on  the  sale  of  the  rights? 

(6)  What  is  the  carrying  value  of  the  100  shares? 

Working  capital.  One  of  the  most  difficult  problems  for  any- 
one entering  a  new  business  is  to  know  how  much  money  will  be 
required  to  finance  the  enterprise  until  the  receipts  will  equal  or 


BUSINESS  FINANCE 


217 


exceed  the  disbursements.     While  this  is  strictly  a  question  of 
finance,  the  accountant  is  often  called  upon  to  deal  with  it. 

Example 

A  manufacturer  gives  you  the  following  data,  and  requests  that  you  estimate 
the  amount  of  working  capital  required  to  finance  the  making  and  selling  of  an 
article: 

Selling  price,  each $100 

Cost  to  make,  each 60 

Selling  expenses,  each 20 

Overhead,  each 10 

Net  profit,  each 10 

Sales,  first  month . .  50  articles 

"      second  month     100 

"      third  month     150 

"      fourth  month 200 

"      each  month  thereafter          200       " 

All  the  sales  are  installment  sales,  the  payments  being  $10  per  month. 
Assume  arbitrarily  that  the  complete  cost  of  $90  on  each  article  is  incurred  at 
the  time  that  the  sale  is  made. 

What  will  be  the  largest  amount  of  capital  required,  and  in  which  month 
will  it  be  required? 

Solution 

Working 

Total  Receipts      Deficiency          Capital 
Each  Month      Each  Month       Required 
$      500 
1,500 
3,000 
5,000 
7,000 
9,000 
11,000 
13,000 
15,000 
17,000 


Months 
First  ... 
Second  . 
Third  . 
Fourth 
Fifth... 
Sixth  .. 
Seventh 
Eighth 
Ninth 
Tenth . 
Eleventh 
Twelfth. 


Total  Costs 
Each  Month 
$  4,500 
9,000 
13,500 
18,000 
18,000 
18,000 
18,000 
18,000 
18,000 
18,000 
18,000 
18,000 


18,500 
19,500 


Deficiency 
Each  Month 
$  4,000 
7,500 
10,500 
13,000 
11,000 
9,000 
7,000 
5,000 
3,000 
1,000 
500f 
l,500f 


f  Receipts  from  collections  are  more  than  the  costs  for  the  month. 


$  4,000 
11,500 
22,000 
35,000 
46,000 
55,000 
62,000 
67,000 
70,000 
71,000 
70,500 
69,000 


The  above  table  shows  in  the  last  column  the  amount  of  working  capital 
required  to  finance  the  business  by  months.  The  greatest  amount  required  is 
found  to  be  $71,000  in  the  tenth  month.  Thereafter,  the  collections  are  greater 
than  the  costs. 

Problems 

1.  A  company  is  about  to  be  formed  for  the  purpose  of  manufacturing  a 
specialty.  After  careful  investigation,  the  following  estimates  have  been  made: 

Selling  price,  each $75 

Cost  to  make,  each  35 

Selling  and  administration  expense 14 

Net  profit 26 


218  BUSINESS  FINANCE 


First  month 30  machine*, 

Second   "     70 

Third      "     180         " 

Fourth    "     200         " 

Each  month  thereafter 225         " 

The  terms  of  payment  are  $15  down,  and  $5  per  month.  What  is  the  greatest 
amount  of  working  capital  that  will  be  required,  and  in  which  month  will  this 
amount  be  needed? 

2.  The  X.  Company  plans  to  sell  on  the  installment  basis,  direct  from  factory 
to  consumer.  Their  product  is  a  specialty  retailing  at  $100,  payable  $10  with 
order  and  balance  in  nine  equal  installments. 

Cost  to  manufacture: 

Material 40% 

Labor 35% 

Burden 25% 

Helling  expense  15%  of  sale? 

Administration  expense     4%  of  sales-1 

Other  expense 1  %  of  sales 

Labor  cost  is  expected  to  increase  14y%,  which  will  decrease  the  profit  35%. 

Estimated  sales: 

First  month 50  machines 

Second    "     .  100 

Third      "     150 

Fourth   "     200 

Each  month  thereafter 200         " 

Assuming  that  all  the  expenses  of  a  sale  are  paid  during  the  month  in  which 
the  sale  is  made,  prepare  a  schedule  showing  the  essential  facts,  and  the  amount 
of  working  capital  needed  monthly.  ' 

3.*  On  the  basis  of  the  following  facts,  determine,  by  months,  the  cash 
requirements  of  an  installment  dealer  for  the  first  year's  operations: 

1.  Cost  of  article $50  00 

2.  Sales  price 90  00 

3.  Selling  expense 15  00 

4.  Overhead 15  00 

5.  Profit 10.00 

6.  Sales  for  the  first  month  were  100  articles 

7.  Sales  for  the  second  month  were  200  articles 

8.  Sales  for  subsequent  months  were  300  articles  per  month 

9.  Merchandise  paid  for  on  the  month  following  the  sale 

10.  Expenses  paid  during  the  month  of  sale 

11.  Payments  are  received  at  the  rate  of  $10  down  and  $10  per 

month;  assume  that  no  irregularities  are  experienced 

4.t  The  A.  B.  Company  acquired  the  right  to  sell  musical  instruments  in  a 
given  territory.  They  request  you: 

*  C.  P.  A.,  Wisconsin, 
t  C.  P.  A.,  Pennsylvania. 


BUSINESS  FINANCE  219 

(a)  To  state  how  much  capital  will  be  required  to  carry  on  the  business 
during  the  first  year. 

(6)  To  demonstrate  by  computation  how  your  estimates  would  work  out 
during  the  first  six  months. 

Assume  that  the  sales  for  the  first  year  will  total  $180,000  from  the  sale  of 
instruments,  and  $24,000  from  service  work  (respectively,  $15,000  and  $2,000 
monthly).  The  overhead  and  direct  selling  costs  are  estimated  at  $30,000  for 
the  year.  This  amount  includes  all  expenses  except  the  cost  of  instruments 
sold  and  parts  used  in  service,  the  latter  being  estimated  at  $12,000. 

The  instruments  are  purchased  on  60  days'  credit,  at  a  discount  of  30%  from 
the  price  at  which  they  are  sold  to  the  customer.  Twenty  per  cent  of  the  instru- 
ments are  sold  for  cash,  and  80%  on  lease  contracts.  When  they  are  sold  on 
lease  contracts,  25  %  is  required  as  a  down  payment,  and  the  balance  in  12 
months.  All  payments  are  made  to  the  A.  B.  Company.  The  leases  are  dis- 
counted at  the  bank,  the  charges  by  the  bank  being  added  to  the  price  charged 
the  customer.  The  service  charges  are  billed  and  payable  in  30  days. 

Cumulative  voting.  Cumulative  voting  is  a  method  whereby 
each  shareholder  is  entitled  to  cast  a  number  of  votes  equal  to  the 
product  of  the  number  of  shares  which  he  holds  and  the  number  of 
directors  to  be  elected.  The  shareholder  may  cast  all  his  votes  for 
any  one  or  more  of  the  directors  to  be  elected,  or  may  distribute  his 
votes  in  any  way  that  he  desires.  Thus,  the  minority  stock- 
holders, by  combining  their  votes,  may  elect  a  representative  on 
the  board  of  directors. 

Example 

A  corporation  has  an  outstanding  capital  stock  of  $100,000,  composed  of 
1,000  shares  of  common  stock  with  a  par  value  of  $100  each.  The  stockholders 
are  to  elect  seven  directors  at  the  annual  meeting.  Calculate  the  least  number 
of  shares  required  to  elect  three  of  the  directors,  provided  that  the  cumulative 
method  of  voting  is  used. 

Formula  Substitution 


Explanation. 

a  =  Number  of  shares  outstanding 
6  =  Number  of  directors  to  be  elected 
c  —  Number  of  directors  minority  desires  to  elect 
x  =  Required  number  of  shares 
1,000  X  7  =  7,000,  total  votes  of  1,000  shares 

376  x  7  =  2,632,  total  votes  of  376  shares 
2,632  -r-  3  =  877,  the  number  of  votes  each  of  the  three 
directors  would  receive  if  the  holders  of  the 
376  shares  of  stock  cast  all  their  votes  for 
three  directors 
7,000  -  2,632  =  4,368,  balance  of  votes 

4,368  -T-  5  =  873,  the  largest  number  of  votes  the  remaining 
stockholders  could  cast  for  five  directors 


220  BUSINESS  FINANCE 

By  the  same  method  of  calculation,  it  will  be  found  that  the  owners  of  375 
shares  would  have  875  votes  for  each  of  three  directors,  while  the  remaining 
stockholders  would  have  875  votes  for  each  of  five  directors.  This  would  give 
a  tie  vote,  and  neither  side  could  elect  the  desired  number. 

Problems 

1.  In  a  corporation  which  uses  the  cumulative  method  of  voting,  how  many 
of  the  seven  directors  can  you  safely  seek  to  elect,  if  you  own  1,501  out  of  the 
4,000  voting  shares? 

2.  Seven  directors  are  to  be  elected  by  the  X.  Company,  which  has  a  voting 
capital  of  5,000  shares.     How  many  shares  are  necessary  to  elect  four  directors 
under  the  cumulative  plan? 

Book  value  of  shares  of  stock.  A  corporation  is  owned  by  the 
stockholders,  whose  evidences  of  ownership  are  shares  of  stock. 
The  hook  value  of  a  share  of  stock  is  equal  to  the  quotient  of  the 
net  worth  divided  by  the  number  of  shares  of  stock  outstanding. 

Formula 

Assets  —  Liabilities 

-vr—  -     — --      =  Book  value  per  share 

Number  of  shares 

Example 

A  corporation  has  assets  of  $340,000  and  liabilities  of  $120,000.  It  has  a 
capital  stock  of  $200,000.  If  the  shares  have  a  par  value  of  $100  each,  what 
is  their  book  value? 

Solution 

$340,000  -  $120,000 $220,00000 

$220,000  +  2,000  shares 110.00 

Problems 

1.  Find  the  book  value  of  a  share  of  stock  in  each  of  the  following  companies 

Par  Value 

Assets  Liabilities  Capital  of  Shares 

(a)      $350,829.75        $134,082.47  $100,000  $100 

(6)      $575,850.00        $190,260.75  25,000  shares      No  Par 

(c)  $1,322,080  35        $110,809.20        $1,000,000  $50 

2.  A  company  has  assets  of  $385,915.28  and  liabilities  of  $158,910.75.     If 
there  are  5,000  shares  outstanding,  each  with  a  par  value  of  $50,  what  is  the 
book  value  of  each  share? 

Profits  distribution.  The  distribution  of  profits  in  the  part- 
nership type  of  business  organization  was  discussed  under  the 
subject  of  partnership  (Chapter  20).  The  distribution  of  the 
profits  of  corporations  is  illustrated  in  the  following  examples. 

Example  1 

At  the  end  of  a  certain  year,  a  corporation  had  outstanding  2,250  shares 
of  common  stock,  par  value  $100.  A  2%  dividend  was  declared.  Net  earnings 


BUSINESS  FINANCE  221 

were  $53,320.84.     What  was  the  amount  of  the  dividend,  and  what  amount 
of  the  year's  profits,  after  the  declaration  of  the  dividend,  remained  in  surplus? 

Solution 

2,250  shares  @  $100 $225,000,  capital  stock 

2%  of  $225,000  $4,500,  dividend 

$53,320.84  -  $4,500 $48,820.84,  credit  to  surplus 

Example  2 

A  company  had  the  following  number  of  outstanding  shares,  each  with  a 
par  value  of  $50:  common,  858,860  shares;  6%  cumulative  preferred,  291,047 
shares;  5%  non-cumulative  and  non-participating  preferred,  28,849  shares. 

The  company  declared  a  6%  dividend  on  the  common  stock.  What  amount 
of  profits  was  distributed  to  each  class  of  stock?  What  amount  of  profits  was 
necessary  to  cover  the  dividends? 

Solution 

Capital  Dividends 

858,860  shares  common  @  $50 $42,943,000  00 

6  %  of  $42,943,000 $2,576,580  00 

291,047  shares  6%  cumulative  preferred  (m  $50.      14,552,350.00 

6%  of  $14,552,350     873,141 .00 

28,849  shares  5%  non-cumulative  preferred  (a\ 

$50 1,442,450.00 

ri%  of  $1,442,450 _  72,122.50 

Profits  necessary  to  cover  dividends $3,521,843.50 

Problems 

1.  Calculate  the  amount  of  dividends  on  the  following  stocks;  each  class  has 
a  par  value  of  $100: 

182,260  shares  common  @  6%; 
57,633  shares  7%  cumulative  preferred. 

2.  A  company  earned  $2,850,460.03.     It  paid  from  this  8%  dividends  on 
$12,379,850   preferred   stock   outstanding.     If  the   preferred   stock   was   non- 
participating,  what  per  cent  was  earned  for  the  common  stock,  of  which  there 
was  $10,600,000  outstanding?     Par  value  in  each  case  was  $50  a  share. 

3.  The  outstanding  stock  of  a  corporation  consists  of: 

Preferred  A  stock,  without  par  value,  series  1,  cumulative  divi- 
dends $7  per  share 10,000  shares 

Participating  preference  stock,  without  par  value,  cumulative 

dividends  $8  per  share 16,301  shares 

Preferred  R  stock,  without  par  value,  non-cumulative  dividends 

$3.50  per  share 6,659  shares 

Common  stock  without  par  value 198, 145  shares 

The  participating  preference  stock  entitles  the  holder  to  receive,  among  other 
things,  participating  dividends  equal  share  for  share  to  any  dividends  paid  from 
time  to  time  on  the  common  stock. 

If,  in  the  course  of  a  year,  50^  is  paid  on  each  share  of  common  stock,  what 
will  be  the  total  amount  of  dividends  paid? 


CHAPTER  23 
Public  Finance  and  Taxation 

Governmental  functions.  Governmental  functions  are  divided 
among  three  major  classes  of  governmental  units — the  federal 
government,  the  forty-eight  state  governments,  and  thousands  of 
local  government  bodies,  including  counties,  towns,  townships, 
cities,  villages,  and  such  special  units  as  drainage,  levee,  irrigation, 
and  park  districts. 

Purposes  of  taxes.  Federal  government  taxes  are  used  to  pay 
the  army  and  navy,  the  salaries  of  governmental  personnel,  pen- 
sions, and  other  goverment  expenses  such  as  education,  highways, 
economic  development,  social  welfare,  and  so  forth.  Expenses  of 
the  federal  government  in  1943  amounted  to  more  than  78  billion 
dollars;  most  of  this  was  for  national  defense,  which  cost  72  billions. 
The  budget  included  a  billion  dollars  for  aid  to  agriculture  and 
almost  2  billions  for  interest  on  the  public  debt. 

State  government  taxes  are  used  to  pay  the  salaries  of  state 
government  personnel,  the  support  of  schools,  universities,  and 
asylums,  and  for  sundry  other  state  expenses.  The  largest  single 
item  in  state  expenditures  for  the  48  states  during  the  year  1942 
was  that  for  operation  and  maintenance,  which  in  that  year 
amounted  to  $4,083,877,000;  of  this  amount,  $1,030,117,000  was 
for  operation  and  maintenance  of  schools. 

County  taxes  are  used  to  pay  the  salaries  of  county  employees, 
the  cost  of  roads,  charities,  and  miscellaneous  other  county 
expenses. 

City  taxes  are  used  to  pay  the  salaries  of  city  employees,  police 
and  fire  protection,  support  of  schools,  and  other  city  expenses. 

Town  taxes  are  used  to  pay  the  salaries  of  town  employees, 
support  of  schools,  and  other  town  expenses. 

Taxes  levied  by  special  units  are  for  the  purpose  of  paying  for 
and  maintaining  the  special  units. 

Appropriations.  Funds  deemed  to  be  necessary  for  the  conduct 
of  government  are  set  up  by  the  respective  governing  bodies  as 
appropriations.  The  size  of  the  appropriations  may  result  from 
modifications  in  the  quantity  and/or  quality  of  governmental 
activities  and  from  changes  in  commodity  prices  and  wage  and 

223 


224 


PUBLIC  FINANCE  AND  TAXATION 


^  TAX  REVENUES  | 
|      M  THE  U.  S.      | 


W/////////M^^^ 


Courtesy  of  Minnesota  Taxpayers'  Assn. 

salary  levels.    To  meet  these  appropriations,  taxes  are  levied  upon 
persons  and  property. 

Kinds  of  taxes.  Commodity  taxes  may  be  limited  in  their 
scope,  applying  to  particular  commodities,  such  as  taxes  on 
tobacco,  liquors,  and  so  forth;  or  they  may  be  general,  applying  to 
the  manufacture  and  sale  of  all  or  most  commodities,  such  as  sales 
taxes,  manufacturers'  excise  taxes,  and  so  forth. 


PUBLIC  FINANCE  AND  TAXATION  825 

Highway  taxes  are  best  known  as  the  motor  vehicle  tax  or 
license,  and  the  motor  vehicle  fuel  tax  known  as  the  gasoline  tax. 

The  general  property  tax  and  the  special  property  taxes  are  the 
major  sources  of  revenue  for  state  and  local  governments.  The 
tax  rate  imposed  on  the  assessed  value  of  a  piece  of  property  is  a 
total  of  a  series  of  separate  tax  rates  imposed  by  local  and  state 
governments.  Real  property  is  listed  and  valued  by  the  assessor, 
while  personal  property  is  usually  listed  by  the  taxpayer,  who  sets 
his  own  value  on  the  articles  he  lists,  although  the  assessor  may 
change  the  values  if  investigation  proves  them  to  be  incorrect. 

Taxes  on  business  consist  of  state  bank  taxes  and  state  taxes  on 
insurance  companies,  railroads,  and  public  service  enterprises  such 
as  telephone  and  light  and  power  companies.  There  are  also  state 
taxes  on  business  in  general  in  the  form  of  license  fees  and  franchise 
taxes. 

Income  taxes  are  levied  on  the  income  of  both  persons  and 
business.  The  first  income  tax  was  a  federal  tax,  but  now  most  of 
the  states  have  income  tax  laws.  The  excess  profits  tax  (repealed 
in  1946)  is  a  tax  on  the  excess  profits  of  a  corporation  and  is  a 
federal  tax. 

Death  taxes  on  transfer  of  property  of  a  deceased  person  to  his 
beneficiaries  or  heirs  are  levied  by  the  federal  government  and  by 
most  of  the  state  governments.  One  form  is  the  federal  estate 
tax,  another  is  the  state  inheritance  tax.  To  prevent  distribution 
of  large  holdings  of  property  in  order  to  escape  estate  and  inherit- 
ance taxes,  the  gift  tax  was  enacted  by  the  federal  government. 
Several  of  the  states  also  have  gift  taxes. 

The  chart  on  page  224  summarizes  the  general  sources  of  tax 
revenues. 

Income,  inheritance,  estate,  and  gift  taxes  are  complex  and 
constitute  complete  studies  in  themselves;  also,  rates  and  regula- 
tions are  frequently  undergoing  changes.  Therefore,  these  sub- 
jects will  not  be  presented  in  this  text. 


Property  Tax 

Determination  of  tax  rate.  The  amount  of  money  needed 
divided  by  the  assessed  valuation  of  the  property  determines  the 
rate  to  be  levied.  For  example : 

State  tax: 

Budget S       3,750,000 

Assessed  value  of  property  in  state 1 ,250,000,000 

3,750,000  -v-  1,250,000,000  =  .30%,  State  rate 


226  PUBLIC  FINANCE  AND  TAXATION 

County  tax: 

Budget $     90,000 

Assessed  value  of  property  in  county     .  .          4,500,000 

90,000  +  4,500,000         =  2  00%,  County  rate 

City  tax: 

Budget 75,000 

Assessed  value  of  property  in  city 2,500,000 

75,000  -T-  2,500,000         =  <LOO%,  Citv rate 

5.30%,  Total  rate 

To  find  the  amount  of  tax.  Multiply  the  assessed  valuation  by 
the  rate.  Tax  rates  may  be  expressed:  as  so  many  mills  on  the 
dollar;  as  a  certain  rate  per  cent;  or  as  dollars  on  the  thousand. 

Example 

Property  is  assessed  at  $8,500;  the  tax  rate  is  $40.60  a  thousand.  Find 
the  tax. 

Solution 

$  40  60,  rate  on  each  thousand 

<S  5  ,  number  of  thousands  assessed 
$345.10,  the  tax. 

Problems 

1.  What  is  the  total  tax  rate  on  the  following? 

State: 

Budget $        5,500,000 

Assessed  valuation 2,500,000,000 

County : 

Budget 72,000 

Assessed  valuation 6,000,000 

City: 

Budget 125,000 

Assessed  valuation 3,750,000 

School  District: 

Budget 35,000 

Assessed  valuation 1,750,000 

2.  In  a  certain  town  the  tax  rate  on  $1,000  was  as  follows: 

State $  1 .20 

County 30  00 

Town: 

Library $1 . 24 

Revenue 91 

Road  and  Bridge 2  74 

Road  Drag 91 

5.80 

School  District 1 .00 

Total  rate 

If  the  assessed  value  of  X  Company's  property  in  this  town  was  $93,960, 
what  was  the  amount  of  property  tax? 


PUBLIC  FINANCE  AND  TAXATION  227 

3.  If  property  is  assessed  at  $6,880  and  the  tax  rate  is  $100,30  a  thousand, 
what  is  the  tax? 

4.  If  the  tax  rate  is  $100.30  a  thousand,  what  is  the  rate  per  cent?     What  is 
this  rate  in  mills  on  the  dollar? 

5.  A  tax  rate  of  $99.20  a  thousand  is  made  up  of  the  following  rates: 

State: 

Debt $  7  33 

Road,  Bridge,  and  Soldiers'  Relief 1 . 10 

School 1 .23 

Teachers'  Retirement .04 

County : 

Revenue 27.09 

City: 

Revenue 61.41 

School  District 1 .00 

Total ...~.H 

Property  valued  at  $75,000  is  assessed  at  $  of  its  value.  What  per  cent  of 
the  total  tax  applies  to  each  division? 

6.  In  a  certain  city  taxes  are  due  January  1,  hut  may  be  paid  in  two  install- 
ments, the  first  on  or  before  May  31  and  the  second  on  or  before  Oct.  31. 

If  the  first  half  is  not  paid  on  or  before  May  31,  the  following  penalties  will 
attach:  During  June,  3%;  July,  4%;  August,  5%;  September,  6%;  October,  7%. 

During  November  and  December,  the  penalty  is  8%  computed  on  any 
amount  unpaid. 

The  second  one-half  cannot  be  paid  until  the  first  one-half  has  been  paid. 

If  the  tax  rate  is  $96.50  a  thousand,  find  the  total  tax  paid  on  the  following: 

(a)  Property  with  assessed  value  of  $35,000,   both  installments  paid  on 
August  10. 

(b)  Property  with  assessed  value  of  $7,500,  the  iirst  installment  paid  July  15 
and  the  second  installment  paid  December  15. 

7.*  A  city  with  an  assessed  valuation  of  $1,000,000  and  estimated  receipts 
for  current  expenses  from  miscellaneous  sources  of  $50,000  and  of  $2,000  from 
sinking  fund  investments  has  submitted  to  you  the  following  budget  of  expendi- 
tures for  the  year: 

Mayor  and  other  Commissioners $20,000 

Water  Department 15,000 

Bond  Interest 5,000 

Fire  Department 20,000 

Police  Department 21,000 

Health  Department 15,000 

Retirement  of  Bonds 10,000 

Street  Department 18,000 

General  Government 25,000 

What  tax  levy  must  be  made  to  provide  the  necessary  revenue? 

8.f  The  assessed  valuation  of  the  taxable  property  of  the  State  of  W.,  as 


*  C.  P.  A.,  North  Carolina. 

t  Adapted  from  C.  P.  A.  pjxamination. 


PUBLIC  FINANCE  AND  TAXATION 

determined  by  the  Tax  Commission  for  a  certain  year,  was  $4,068,268,534. 

What  would  a  citizen  whose  property  was  assessed  at  $507,374  have  to  pay 
for  each  of  the  following  purposes,  and  what  would  be  the  amount  of  his  total 
tax  bill? 

Purposes — Total  Amount  to  Be  Raised 

Interest  on  Certificates  of  Indebtedness $    199,339.42 

Free  High  Schools 175,000.00 

State  Graded  Schools 200,000.00 

Highway  Improvements 1,700,000.00 

General  Purposes 100 . 00 

In  addition  to  the  above,  the  following  mill  taxes  are  assessed: 

University f  mill 

Normal  Schools .  .     \  mill 

Common  Schools ^  mill 


CHAPTER  24 
Fundamentals  of  Algebra 

Explanation.  The  work  of  an  accountant  is  complicated  in 
many  particulars  and  requires  technical  calculations.  It  is 
extremely  difficult  to  make  some  of  these  calculations  by  arithme- 
tic. On  the  other  hand,  if  the  fundamentals  of  algebra  are  under- 
stood, the  calculations  may  be  made  with  comparative  ease.  Only 
the  more  common  and  more  useful  principles  of  algebra  will  be 
discussed  in  this  chapter. 

Symbols  and  terms.  In  algebra,  the  letters  of  the  alphabet 
are  usually  employed  as  symbols  to  represent  numbers. 

The  following  signs  have  the  same  meaning  in  algebra  as  in 
arithmetic : 

-f  is  read   'plus" 

—  is  read   'minus" 

X  is  read   'times"  or  "multiplied  by" 

-f-  is  read    'divided  by" 

=  is  read   'equals" 

An  exponent  is  a  number  or  symbol  written  at  the  right  of 
another  number  or  symbol  and  a  little  above  it,  to  show  how  many 
times  the  latter  is  to  be  used  as  a  factor  and  to  indicate  its  power. 
For  example,  "25*"  =  25  X  25  X  25  =  15,625.  When  no  expo- 
nent is  indicated,  1  is  understood  to  be  the  exponent. 

An  equation  is  an  expression  of  equality  between  two  numbers. 

An  axiom  is  a  statement  which  is  admitted  to  be  true  without 
any  proof.  Algebraic  operations  make  use  of  the  following  axioms : 

(1)  The  equality  of  both  sides  of  an  equation  is  not  destroyed 
by  the  addition  of  the  same  number  to  both  sides. 

(2)  The  equality  of  both  sides  of  an  equation  is  not  destroyed 
by  the  subtraction  of  the  same  number  from  both  sides. 

(3)  The  equality  of  both  sides  of  an  equation  is  not  destroyed 
by  the  multiplication  of  both  sides  by  the  same  number. 

(4)  The  equality  of  both  sides  of  an  equation  is  not  destroyed 
by  the  division  of  both  sides  by  the  same  number. 

Positive  and  negative  numbers.  A  positive  or  negative  state 
of  any  concrete  magnitude  may  be  expressed  without  reference  to 
the  unit;  thus,  numbers  that  are  greater  than  zero  are  positive,  and 
numbers  that  are  less  than  zero  are  negative. 

229 


230  FUNDAMENTALS  OF  ALGEBRA 

A  number  which  has  a  "  +,"  or  positive,  sign  prefixed  to  it  is 
called  a  positive  number;  thus,  +5.  If  a  "  —  "  sign  precedes  the 
unit,  it  is  called  a  negative  number,  and  is  written  thus,  —5. 

Addition  of  positive  and  negative  numbers.  When  two  or 
more  positive  and  negative  numbers  are  combined  into  a  single 
number,  the  result  is  called  the  sum  of  the  numbers. 

Example 

The  sum  of  +6  and  -4  is  +2 

1    +4a  and  -2a  is  +2a 
'    +2  and  -6  is  -4 
*    +3a  and  —la  is  —  4a 
'    -4  and  -3  is  -7 
1    —  3a  and  —  4a  is  —  7a 

From  the  foregoing,  the  following  rules  may  be  derived  : 

(1)  To  add  two  numbers  or  terms  of  different  signs,  subtract 
the  smaller  number  or  term  from  the  larger,  and  prefix  the  sign  of 
the  larger. 

(2)  To  add  two  negative  numbers,  add  their  absolute  values 
and  prefix  the  negative  sign. 

Example 

Add  +4a,  —  3a,  +6a.    The  problem  may  be  restated  thus:  (+4a)  +  (—  3a) 

Solution 

+  4a 
+  6a 


+  10a 
-  3a 


The  addition  of  positive  quantities  is  made  in  the  same  way  as 
in  ordinary  addition. 

The  addition  of  a  positive  and  a  negative  number  is  equivalent 
to  deducting  the  smaller  number  from  the  larger,  and  retaining  the 
sign  of  the  larger. 

Problems 

Find  the  sum  of  each  of  the  following: 

1.  +4,  -3,  +7,  -2,  +6,  +4,  -8,  -6. 

2.  -4,  -8,  +6,  +5,  -4,  -3,  -2,  -7. 

3.  +1,  -4,  -6,  +2,  +7,  +6,  +4,  +§. 

The  coefficient.  The  number  or  letter  put  before  a  mathemati- 
cal quantity,  known  or  unknown,  to  show  how  often  it  is  to  be 
taken,  is  called  the  coefficient.  In  adding  terms  which  are  multiples 


FUNDAMENTALS  OF  ALGEBRA  231 

of  the  same  letter,  add  the  coefficients  of  these  terms,  and  prefix 
the  proper  sign.     Thus,  +6a,  +76,  —  5a,  +66  becomes: 

+6a         +  76 

-5a        +  66 

Added        +  a        +136 

It  is  convenient  to  arrange  the  terms  in  columns,  so  that  like 
terms  stand  in  the  same  column. 

Problems 

Find  the  sum  of  each  of  the  following: 

1.  +4a,  +36,  -4a,  -26. 

2.  +6a,  +46,  -6c,  +3a,  -46,  +4r. 

3.  +4*,  -4y,  +4*,  -3*,  +3//,  +2*. 


6.  - 

6.  +4z,  +3z,  -Go;,  +2z,  -12*, 

7.  +4a,  +36,  +3c,  -4c,  +46,  -a. 

8.  —a,  —6,  +c,  +6,  —  c,  +a,  —  r. 

9.  +c,  +z,  -z,  +y,  -2c,  +  6x,  - 
10.  + 


Hereafter,  when  a  term  is  not  preceded  by  a  positive  or  a  nega- 
tive sign,  it  is  to  be  understood  as  a  positive  term. 

Parentheses,  brackets,  and  braces.  Parentheses,  brackets, 
and  braces  are  used  to  indicate  that  the  part  inclosed  is  to  be 
employed  as  a  single  term  or  as  a  single  unit. 

Since  the  same  rules  apply  to  all  signs  of  aggregation,  in  the 
explanation  given  hereafter,  only  parentheses  will  be  mentioned. 

RULES,  (a)  When  a  term  in  parentheses  is  preceded  by  a 
"+,"  or  positive,  sign,  the  parentheses  may  be  removed  without 
any  change  in  the  signs  of  the  inclosed  terms. 

(6)  If  a  term  in  parentheses  is  preceded  by  a  "  —,"  or 
negative,  sign,  when  the  parentheses  are  removed  it  is  necessary  to 
change  each  of  the  positive  and  the  negative  signs  of  the  terms 
inclosed. 

Examples 


(a  -  6)  +  (c  -  d)  -  (e  -/)  =  a  -  6  +  c  -  d  -  e  +f 
(4a  +  36  -  c)  -  (d  +  4*  -  /)  =  4a  +  36  -  c  -  d  -  4e  +  / 

Wherever  possible,  like  terms  should  be  combined  ;  as  : 
(3a  -  36  +  4c)  -  (2a  -  3c)  =  a  -  36  +  7c 

If  several  algebraic  expressions  are  inclosed  one  within  the 
other  by  inclosure  signs,  such  as  parentheses  or  brackets,  eliminate 
the  innermost  pair  of  inclosure  signs  first* 


232  FUNDAMENTALS  OF  ALGEBRA 

Example  1 

a  +  [b  -  (c  -  d)]  =  a  +  [b  -  c  +  d] 
-a  +  6-c  +  d. 

Example  2 

a  +  b  -  [-  (b  +  c)  +  (c  -  d)]  =  a  +  b  -  [-b  -  c  +  c  -  d\ 

=a+6+6+c-c+d 
=  a  -f  26  -f  d. 

Problems 

Simplify  the  following  by  removing  all  signs  of  aggregation: 

1.  x  4-  y  +  (x  +  y)  -  (2x  +  2y). 

2.  3a  +  (6  -  4c)  -  (4a  -  6  -  c). 

3.  42  +  (37  +  6)  -  (40  +  20). 

4.  30  -  [20  +  (3  +  4)  -  (4  +  2)]. 

6.  (3x  -  4y)  -  (6*  -f  2t/)  +  (4x  -  3*/). 

6.  -3a  +  [46  -  (6c  +  la)  -  56  +  6c]. 

7.  -362  -f  4a2  -  (262  -  4a2)  +  4a. 

8.  a  +  6  +  c  +  d  -  (2a  +  26  -  2c  +  2c). 

9.  -x  -  y  -  (z  -  x)  -  y  -  z(x  +  y  +  z). 
10.  (8a  -  46)  +  (3c  +  d)  -  (3o  -f  6  4-  c). 

Subtraction.  Subtraction  is  the  process  of  determining  one  ot 
two  numbers  when  their  sum  and  one  of  the  numbers  are  given. 

The  minuend  is  the  sum  of  the  two  numbers. 

The  subtrahend  is  the  number  to  be  deducted. 

The  remainder  is  the  required  number. 

To  subtract,  change  the  sign  of  the  subtrahend  and  add  the 
subtrahend  and  the  minuend. 

Example  1 

Subtract  8x  from  4:r. 

Solution 


&x  from  4r  =  (+4s)  -  (+8&) 
Removing  parentheses:  =  4x  —  Sx 

=  -4s 

Example  2 

Deduct  —  8#  from  —  4#. 

Solution 

-Sx  from  -4z  =  (-4z)  -  (-Sx) 
Removing  parentheses:  =  —  4x  +  Sx 


Example  3 

Deduct  —  Sx  from  4x. 


-8x  from  4x  =  (+4x)  -  (-8x) 
Removing  parentheses:  =  4x  +  Sx 

=  +12a? 


FUNDAMENTALS  OF  ALGEBRA  233 

To  subtract  algebraic  expressions  having  two  or  more  terms, 
change  the  sign  of  each  term  of  the  subtrahend  and  proceed  as  in 
addition. 

Example 

Subtract  3a  -  5c  -  3d  from  7c  -  16a  -  2d. 

Solution 
The  changing  of  the  signs  of  the  subtrahend  is  usually  done  mentally;  thus. 

-16a  +    7c  -  2d 
3a  -    5c  -  3d 


Oa-h  12c  +    d 
Problems 


Subtract: 


1.  MX  from  28z. 

2.  I3x  from  3x. 

3.  IGafrom  -18a. 

4.  —  6a  from  18a. 

5.  a  -f  2a  from  8a  —  3a. 

6.  4a  -  36  from  3a  -f-  46. 

7.  16a  -  146  +  3c  from  -13a  -f  26  -  4c. 

8.  -  16a  +  46  +  2c  from  18a  -  146  -f-  3c. 

9.  3cs  -  562;  from  -6cs  +  76z. 
10.  3p  +  4g  from  4p  —  4g. 

Multiplication.  When  two  or  more  numbers  are  multiplied, 
the  result  is  called  the  product  of  the  numbers. 

When  two  numbers  with  like  signs,  either  positive  or  negative, 
are  multiplied,  the  product  is  positive. 

Examples 

(+a)  X  (+6)  -  +ab 
(-a)  X  (-6)  =  +ab 
(-4)  X(-4)  =  4-16 

When  two  numbers  with  unlike  signs  are  multiplied,  the 
product  is  negative. 

Examples 

(+a)  X  (-6)  =  -a6 
(+4)  X(-3)  =  -12 

The  exponent  to  be  used  in  the  product  is  equal  to  the  sum  of 
the  exponents  appearing  in  the  multiplier  and  the  multiplicand. 

Examples 

(0)  *  X  (a)1  =  a2 
(a4)  X  (-a3)  =  -a7 
(-3a2)  X  C-2a3)  =  +6a6 


234  FUNDAMENTALS  OF  ALGEBRA 

Problems 

Multiply  the  following: 

1.  a2  by  a2.  7.  3a  +  100  by  6. 

2.  a3  by  a6.  8.  3(a  +  b). 

3.  -a3  by  a4.  9.  1,000  +  s  -  6  by  6. 

4.  ab  by  db.  10.  (100  -  c)  -  6  by  4. 

6.  a2  by  fc2.  11.  [20,000  -  (2,000  +  z)]  by  12£. 

6.  a  +  6  +  c  by  5.  12.  [40,000  -  (T  -  B  -  2,000)]  by  12. 

Division.  Division  is  the  process  of  finding  one  of  two  numbers 
when  the  other  number  and  the  product  of  the  two  numbers  are 
given.  When  the  dividend  and  the  divisor  have  like  signs,  either 
positive  or  negative,  the  quotient  is  a  positive  number. 

Examples 

ab  -f-  a  —  b 

—  ab  -. 6  =  a 

-14  ^--7  =  2 

When  the  dividend  and  the  divisor  have  unlike  signs,  the 
quotient  is  a  negative  number. 

Examples 

—ab  -T-  a  =  —  b 

ab  -T-  —b=  —a 

-10  ^  5  =  -2 

10  -T-  5  =  2 

The  exponent  to  be  used  in  the  quotient  is  equal  to  the  differ- 
ence between  the  exponent  in  the  dividend  and  the  exponent  in  the 
divisor. 

Examples 

a7  -7-  a2  =  a8 

—  ab2  -f-  b  =  -ab 
8a664  -r-  -2ab3  =  -4a66 

206  -f-  202  =  20' 

Problems 

Divide  the  following: 

1.  63  by  9.  7.  5a2  -  606  by  5. 

2.  -40  by  8.  8.  50a10  -  25a15  by  5. 

3.  -125  by  -5.  9.  20,000  -  (2,000  -  5x)  by  5. 

4.  48a  by  12a.  10.   1,600  -  (200  +  4x)  by  4. 

6.  a26  by  a.  11.  GOa28  -  30a24  +  15a20  by  15a6. 

6.  9a2  by  3a.  12.  72a8  -  18a10  -  54a6  by  9a2. 


CHAPTER  25 
Equations 

Simple  equations.  An  equation  is  an  expression  of  equality 
between  two  magnitudes  or  operations.  The  members  of  the 
equation  are  separated  by  the  sign  of  equality,  "  =,"  which  means 
"is  equal  to."  Either  member  of  an  equation  may  contain 
numerals,  letters,  or  both.  A  simple  equation  is  an  equation  of 
the  first  degree,  and  contains  but  one  unknown. 

Example 

Simple  equation:  x  =  50 

Or:  10*  =  500 

If  the  same  number  is  added  to  or  subtracted  from  both  sides 
of  an  equation,  the  equality  is  not  destroyed. 

Examples 

Simple  equation:  x  =  50 

Adding  10  to  each  side:  x  +  10  =  60 

Simple  equation :  lOx  =  500 

Subtracting  10  from  each  side:  lOz  —  10  =  490 

If  both  sides  of  an  equation  are  multiplied  by  or  divided  by  f  he 
same  number,  the  equality  is  not  destroyed. 

Example  1 

3z  =  15 

Multiplied  by:  5 

15*  =  75 

Simplifying,  or  dividing  by  15:  x  =  5 

Example  2 

4)20s  «  40 

Divided  by  4:  5*  =  10 

Simplifying,  or  dividing  by  5:  x  =  2 

Any  term  of  either  member  of  an  equation  may  be  transposed 
from  one  side  to  the  other  by  changing  the  sign  of  the  term,  and  the 

235 


236 


EQUATIONS 


equality  of  both  sides  of  the  equation  is  not  destroyed.  This 
operation  is  equivalent  to  either  adding  the  same  quantity  to,  or 
subtracting  the  same  quantity  from,  both  sides  of  the  equation. 


Simple  equation: 

Transposing  the  "  —  10"  to 
the  right  side  of  the  equa- 
tion, and  changing  the 
sign: 

Or: 


Example  1 
lOz  -  10  =  490 


lOz 
lOx 


490  +  10 
500 


Simple  equation: 

Transposing: 

Or: 


Example  2 

x  +  5  =  15 


x  =  15  —  5 
x  =  10 


Example  3 

Solve  the  equation,  lOx  —  14  =  6x  -\-  2. 


The  equation: 

Transposing  "§x"  to  the 
left  side,  and  changing 
the  sign  to  "-": 

Transposing  "  —  14"  to 
the  right  side,  and 
changing  the  sign: 

Uniting  similar  terms: 

Dividing  by  4: 


Adding: 


Left  side 
lOx  =40 

-14  =  -14 

26 


Solution 
lOz  -  14  =  6z  +  2 

-  6z  -  14  =  +2 


lOz  —  6z  =  14  +  2 
4z  =  16 
x  =  4 

Verification 


Adding: 


Right  side 

6z  =  24 

2=2 

26 


Therefore  the  two  sides  of  the  equation  are  equal,  and  x  =  4. 

Problems 

In  the  following  equations,  solve  for  the  unknown  quantities,  and  check  your 
results: 


1.  40a;  -  20  =  10s  +  10. 

2.  10z  +  5  =  25. 

3.  8a  +  8  =  2a  +  32. 

4.  16  +  5a  =  8a  +  1. 

5.  496  -  4  =  376  +  8. 


6.  2x  +  6(4z  -  1)  =  98. 

7.  39  +  4(a  +  6)  =  8a  -  1. 

8.  2,000  -  (5*  +  500)  =  2,500. 

9.  86  -  5(46  +  3)  =  1  -  4(26  -  7). 
10.  18y  -  (lOy  -  8)  =  20y  -  (6y  +  4). 


EQUATIONS  237 

Example 

How  can  90  be  divided  into  two  parts  in  such  a  way  that  one  part  will  be 
four  times  the  other? 

Solution 

Let:  x  —  the  smaller  part 

Then:  4z  =  the  larger  part 

Adding:  5x  =  90 

Dividing  each  member  of 

the  equation  by  5:  x  =  18,  the  smaller  part 

4x  =  72,  the  larger  part 

Example 

How  many  dimes  and  cents  are  there  in  $2.40,  if  there  are  60  coins  in  all? 

Solution 

Let:  x  =  the  number  of  dimes 

Then:  60  —  x  —  the  number  of  cents 

Therefore:  10z  =  the  value  of  the  dimes 

But:  60  —  x  —  the  value  of  the  cents 

Simplifying:  lOz  -f  60  -  x  =  240 

Transposing:  9z  =  180 

x  =  20,  or  there  are  20  dimes 
60  —  x  =  40,  or  there  are  40  cents 

Verification 

20  dimes  =  $2.00 
40  cents  =  .40 
60  coins  =  $2.40 

Example 

The  P.  Q.  Company  wrote  off  depreciation  on  its  building,  which  cost  $50,000, 
at  the  rate  of  2%  of  the  original  cost  per  annum.  This  amount  was  included  in 
General  Administration  Expense  account,  and  constituted  Y$  of  that  account. 

The  purchases  cost  two  and  one-half  times  the  old  inventory.  The  value 
of  the  old  inventory  was  twice  the  amount  of  the  selling  expense.  The  Belling 
and  General  Administration  Expense  accounts  each  equalled  10%  of  the  sales. 
The  new  inventory  was  valued  at  an  amount  equal  to  the  selling  expense. 

The  interest  and  discount  costs  were  £  of  the  selling  expense.  Set  up  a  profit 
and  loss  statement  showing  the  net  profit  from  operations. 

Solution 

$50,000  X  2%  =  $1,000,  Depreciation 
$1,000,  Depreciation  =  -j^  of  General  Adm.  Expense 
Multiplying  by  10:    $1,000  X  10  =  $10,000  =  General  Adm.  Expense 

$10,000  =  Selling  Expense 
Selling  Expense  =  ytr  of  Sales 
Multiplying  by  10:  $10,000  X  10  =  $100,000,  or  Sales 

2  X  $10,000  =  $20,000,  or  Old  Inventory 
2^  X  $20,000  =  $50,000,  Purchases 
i  of  $10,000  «  $2,000,  Interest  and  Discount 


238  EQUATIONS 

THE  P.  Q.  COMPANY 

PROFIT  AND  Loss  STATEMENT 

DECEMBER  31,  19 — . 

Sales $100,000 

Cost  of  Sales: 

Old  Inventory $20,000 

Purchases 50,000 

Total $70,000 

Less  New  Inventory 10,000      60,000 

Gross  Profit $"40^000 

Selling  Expense     $10,000 

General  Administrative  Expense 10,000      20,000 

Operating  Profit  $  20,000 

Interest  and  Discount 2,000 

Net  Profit $  18,000 


Problems 

1.  How  can  640  he  divided  into  two  parts,  in  such  a  way  that  one  part  will 
be  seven  times  the  other? 

2.  How  many  quarters  and  cents  are  there  in  $4.00,  if  there  are  40  coins 
in  all? 

3.  A  sum  of  money,  $10.00,  is  made  up  of  dimes  and  quarters,  the  total 
number  of  coins  being  88.     How  many  dimes  and  quarters  are  there? 

4.  In  a  certain  dairy,  the  ice  cream  contains  14%  cream.     If  the  mixture 
is  to  be  made  from  coffee  cream  of  20%  butter  fat  and  milk  which  tests  4%, 
what  portion  of  each  will  have  to  be  used  in  a  mixture  of  100  Ibs.? 

5.  How  many  pounds  of  coffee  worth  25^  per  pound  must  a  grocer  mix 
with  other  coffee  worth  42^  per  pound  to  make  a  mixture  worth  34^  per  pound? 
The  total  quantity  desired  is  50  Ibs. 

6.  Barnes  has  $6,000  invested  in  5%  bonds.     How  much  must  he  invest 
in  8%  stock  to  make  his  average  net  income  6%? 

7.  An  estate  of  $33,120  was  to  be  divided  among  the  mother,  three  sons, 
and  three  daughters.     The  mother  was  to  receive  four  times  as  much  as  each 
son,  and  each  son  three  times  as  much  as  each  daughter.    How  much  was  each 
to  receive? 

8.  In  the  making  of  candy,  a  mixture  of  75%  sugar  at  5jz£  per  pound  and  25% 
corn  syrup  at  2£  per  pound  is  used.    If  the  price  of  sugar  advances  to  7£  per 
pound,  what  must  be  the  ratio  of  sugar  and  corn  syrup  to  be  used,  if  the  cost  of 
production  is  to  remain  the  same? 

9.  A  vinegar  manufacturer  makes  various  grades  of  vinegar,  ranging  from 
50%  to  100%  pure  cider  vinegar.     How  much  100%  pure  cider  vinegar  must 
be  mixed  with  63%  pure  to  make  100  gallons  of  75%  pure? 

10.  How  much  milk  with  a  butter  fat  test  of  3.5%  must  be  mixed  with  milk 
testing  4.65%  to  fill  75  quart  bottles  which  are  to  test  3.95%? 


EQUATIONS  239 

Fractions.     In  many  accounting  problems  it  is  necessary  to  use 

simple  fractions  in  algebraic  form,  such  as  " yV  of  x"  or  "^  times 

o 

x        4ot 

a,"  which  when  simplified  are  written  T^  or  -=-• 

10        o 

Example 

Divide  129  into  two  parts,  in  such  a  way  that  |  of  one  part  will  equal  |-  of 
the  other. 

Solution 

Let:  x  —  one  part 

Then:  129  —  x  =  the  other  part 

2  v       3 

The  problem  states  that:  ~  =  -  (129  —  x) 

9       8 

The  common  denominator  of  the 
fractions,  as  seen  by  inspec- 
tion, is  72,  and  by  the  process 
of  changing  the  fractions  to  a 
common  denominator,  the  re- 

suit  is:  ^  =  |  (129  -  x) 

Multiplying    each    side    of    the 

equation  by  72,  to  clear  the 

equation  of  the  fractions,  the 

result  is:  HKT  =  27(129  -  x) 

Eliminating  the  parentheses:  Hu;  =  3,483  —  27 x 

Transposing:  43.r  =  3,483 

Dividing  by  43:  x  =  81 

129  -  x  =  48 

Verification 
|  of  81  =  18 
f  of  48  =  18 

Example 

The  superintendent  of  a  certain  plant  was  hired  under  a  contract  which 
provided  that  he  was  to  receive  10%  of  the  net  profits  of  the  business  as  a  salary, 
after  his  salary  had  been  deducted  as  an  expense.  The  profits  for  the  year 
were  $13,200.  Compute  the  superintendent's  salary. 

Solution 

Let:  s  =  the  amount  of  salary 

The  problem  is  stated:  *  =  TV($  13,200  -  s) 

Clearing  of  fractions  by  multiplication:     10s  =  $13,200  —  s 
Transposing:  11s  =  $13,200 

Dividing  by  11:  s  =  $1,200 

Verification 

Net  Profits $13,200 

Less  Salary 1,200 

$12,000 
Dividing  by  10 * 1,200 


240  EQUATIONS 

Clearing  of  complex  fractions.  To  clear  an  equation  of  a 
complex  fraction,  multiply  the  opposite  term  of  the  equation  by 
the  denominator  of  the  complex  fraction,  and  solve  the  resulting 
equation  by  the  usual  methods. 

Example 
Find  the  value  of  (1.06)10  in  the  following  equation: 

i__l_ 

(1.06) 10  =  $7  3600871 

Solution 

STEPS 
1 


1  - 


10 
'-    =  $7.3600871 


.06 
Multiplying  right-hand 

term  by  .06:  1  -  7j-^yr0  =  7.3600871  X  .06  (1) 

Clearing:  1  -       * '        =  .4416052  (£) 

Transposing  and  changing 
signs:  •  =  1  -  .4416052  (8) 

Clearing:  ~~  =  .5583947  (4) 

Changing  and  dividing:          (1.06)10  =  1  -f-  .5583947  (5) 

Clearing:  (1.06) I0  =  1.790847  (6) 

Problems 

1.  One-half  of  a  certain  number  exceeds  one-sixth  of  the  same  number  by  8. 
What  is  the  number? 

2.  Find  the  value  of  (1.05)10  in  the  following  complex  fraction: 

I 


1  - 


(L°5)1°  =  7.72173. 


.05 

3.  Find  the  value  of  (1.03)15  in  the  following  complex  fraction: 

— ^^—^  =  18.598913. 

.Uo 

4.  Find  the  value  of  (1.04)20  in  the  following: 

^ —  =  .0735818 

"        .04 


EQUATIONS  S41 

6.  Find  the  value  of  (1.005)48  in  the  following: 

=  42.5803178. 


Simultaneous  equations  with  two  or  more  unknowns.  When 
each  of  two  equations  contains  two  or  more  unknown  quantities, 
and  every  equation  containing  those  unknowns  may  be  satisfied  by 
the  same  set  of  values  for  the  unknown  quantities,  the  equations 
are  said  to  be  simultaneous. 

The  value  of  the  unknown  quantities  in  two  or  more  simul- 
taneous equations  may  sometimes  be  found  by  combining  the 
equations  into  a  single  equation  containing  only  one  unknown 
quantity. 

This  combining  may  be  done  in  several  different  ways,  and  is 
known  as  elimination. 

The  method  of  elimination  by  addition  and  subtraction  is 
probably  the  most  simple,  and  will  therefore  be  the  one  used  here. 

It  is  often  necessary  to  multiply  one,  or  sometimes  both,  of  the 
equations  by  a  number  that  will  make  the  terms  that  contain  one 
of  the  unknowns  in  each  equation  of  equal  absolute  value.  Substi- 
tute known  values  where  possible. 

Add  or  subtract  the  resulting  equations,  and  the  sum  or 
remainder  will  be  an  equation  containing  one  unknown  less  than 
the  previous  equations. 

The  chief  difficulty  in  the  practical  application  of  these  rules  is 
the  expression  of  the  unknowns  in  the  form  of  equations.  It  seems 
advisable  to  make  a  written  statement  of  each  condition,  equation, 
or  unknown,  and  also  a  similar  statement  of  each  of  the  knowns. 

After  each  statement,  a  symbol  or  letter  should  be  used  to 
represent  each  unknown  or  known.  In  algebra,  the  letters  "x" 
"y,"  and  "z"  are  commonly  used,  but  it  seems  to  be  more  practical 
to  use  the  initial  letter  of  the  name  of  the  thing  whose  value  is  to 
be  found. 

Example  1 

Carol  has  five  times  as  much  money  as  Mary.     Together  they  have  $60. 
How  much  money  has  each? 
Statement  of  equations: 

M  =  the  number  of  dollars  belonging  to  Mary 
C  =  5M,  or  five  times  as  much  as  belongs  to  Mary 
M  +  C  =  $60 

Solution 

Substitution  of  5M  for  C:  M  +  5M  =  60 
Combining:  6M  =  60 

Dividing  by  6:  M  =  10 

C  =  5M,  or  $50 


242  EQUATIONS 

Example  2 

It  cost  $98.50  to  manufacture  and  sell  a  certain  article.  The  cost  of  the 

labor  was  equal  to  the  cost  of  the  material  used.     The  cost  of  the  overhead  was 
$9.50  more  than  the  expenses.     The  overhead  and  expenses  totaled  $2.50  more 
than  the  material.     Find  the  cost  of  each  item. 
Statement  of  equations: 

STEPS 

M  =  Cost  of  Material  (1) 

L  =  Cost  of  Labor  (2) 

O  =  Cost  of  Overhead  (5) 

K  =  Cost  of  Expenses  (4) 

M  -f-  L  +  O  -f-  E  =  $98.50  (5) 

Solution 

M  =  L  (6) 

()  =  E  +  $9.50  (7) 

M  =  0  +  E  -  $2.50  (8) 

M  =  E  +  $9.50  -f  E  -  $2.50  (9) 

M  =  2E  -f  $7.00  (10) 

M  =  2E  -f  $7.00  in  terms  of  E  (11) 

L  =  2K  -f  $7.00  "      "       "  "  (12) 

O  =    E  +  $9.50  "      "       "   "  (13) 

E  =    E                  "      "       "  "  (14) 


Adding  (11),  (12),  (IS),  and  (14):  M-\-L  +  0  +  E  =  6E  +  $23.50 

Substituting:  $98.50  =  6#  +  $23.50            (16) 

Transposing:  QE  =  $98.50  -  $23.50      (J7) 

Dividing  by  6:  E  =  $12.50 


Substituting  in  (/I),  (^),  (/3),  and  (^),  the  value  of  each  item  may  be 
found. 

Example  3 

In  the  following  simultaneous  equations,  solve  for  the  values  of  a  and  b: 

STEPS 

10a  -  66  =  38  (1) 

14a  +  86  =    4  (£) 


Multiplying  (1)  by  7:                     70a  -  426  =  266  (3) 

Multiplying  (2)  by  5?                       70a  -f  406  =  ^0  (4) 

Subtracting  (4)  from  (3)  :                         ^826  =  246  (6) 

Dividing  by  82:                                             —6  =  3  (6) 

Or:                                                                 6  =  -3  (7) 

Substituting  the  value  of  6  in  (1)  :     lOo  +  18  =  38  (8) 

Transposing  18  in  step  (8):                          10a  =  20  (9) 

Dividing  by  10:                                             a  =  2  (10) 

Example  4 

The  following  are  the  condensed  balance  sheets  of  three  companies  who  wish 
to  know  their  true  worth  as  of  December  31  : 


EQUATIONS 


243 


Company 

A 

Assets,  exclusive  of  intercompany  in- 
vestments   $200,000  00 

investment  in  Company  B  (50%)     .  .     350,000  00 
"  Company  C  (20%) ....     250,000  00 
"  Company  C  (20%).    .. 
"  Company  A  (10%)   ... 

S800JOOO  00~ 


Company 
B 


Company 
C 


$500,000  00    $300,000.00 

100,000  00 

____ 100,000  00 

$600,000  00     $400~000  00 


Capital  Stock $500,000  00 

Surplus     300,000  00 


$400,000  00     $300,000  00 
200,000  00       100,000.00 


As  each  company  owns  stock  in  each  of  the  others,  there  are  three  unknown 
quantities.     The  true  worth  may  be  found  as  follows: 


Company  A  owns  . . 

Company  B  owns 

Company  C  owns . . 

Let: 

Let: 

Let: 


Solution 
SUMMARY  OF  OWNKRSIIII' 


Company  A 


Net  Assets 

$200,000  00 

500,000  00 

..     300,000  00  10% 

A  —  Net  Worth  of  Company  A 
B  =  Net  Worth  of  Company  B 
C  =  Net  Worth  of  Company  C 


Company  B 
50  % 


Statement  of  equations : 


A  =  200,000  + 
B  =  500,000  + 
C  =  300,000  + 


+  1C 


Company  C 
20% 
20% 


Solution 


Adding: 

Multiplying  (4)  by  10: 

Multiplying  (2)  by  5: 

Subtracting  (6)  from  (5): 

Multiplying  (3)  by  7: 

Subtracting : 

Multiplying  (9)  by  10: 

Dividing  by  97: 

Using  (5)  and  substituting 

the  value  of  —  rg-A : 
Transposing: 
Using  (2)  and  substituting 

the  value  of  —  £0: 
Transposing: 


STEPB 

A  -£#  - 

ir  -     200,000.00 

(1) 

/y  - 

1C  =       500,000  00 

(£) 

AA         + 

C  =       300,000  00 

(3) 

xVA  4-  ir#  + 

1C  -    1,000,000.00 

(4) 

9A  +  5£  + 

6CY  =  10,000,000.00 

(*) 

+  SB  — 

C  =    2,500,000  00 

(6) 

9A             + 

76'  =    7,500,000.00 

(7) 

xV^            + 

7Cy  =    2,100,000.00 

(8) 

=    5,400,000  00 

(9^ 

97A 

=  54,000,000  00 

(10) 

A 

=       556,701  03 

(ID 

-55,670.10  +  C  =       300,000  .  00 

(12) 

C=       355,670.10 

(IS) 

J5  -  71,134 

.02  =       500,000.00 

(14) 

B  -       571,134.02 

U6) 

244  EQUATIONS 

Verification 

ABC  Total 

Assets,  exclusive  of  inter- 
company investments ..  $200,000.00  $500,000.00  $300,000.00 

A  owns:  £  of  B 285,567.01 

iofC 71,134.02 

Total  worth  of  A $556,701703  $    556,701 . 03 

B  owns:  i  of  C 71jL134^02 

Total  worth  of  B $571,134  02  571,134.02 

C  owns:  A  of  A 55>670J° 

Total  worth  of  C $355,67010       355,670J  0 

Total  value  of  the  three 

companies $1,483,505  15 


Problems 

1.  For  a  certain  piece  of  advertising,  the  total  cost  of  printing,  envelopes, 
and  postage  is  $14.50.     The  envelopes  cost  $1.50  more  than  the  postage.     The 
cost  of  printing  is  $0.50  more  than  the  combined  costs  of  postage  and  envelopes. 
Find  the  separate  costs. 

2.  At  the  end  of  the  year,  the  books  of  the  Blank  Company  showed  a  net 
profit  of  $12,247.50.     The  treasurer  was  to  receive  12^-%  of  the  net  profits  as  a 
bonus.     The  treasurer  defaulted,  and  it  was  found  that  his  account  was  short 
$4,847.55.     Show:  (a)  the  true  profit;  and  (b)  the  treasurer's  account  balance. 

3.  A  man  has  $7,000  invested  at  5%.     How  much  must  he  invest  at  65-%  to 
make  his  total  income  equal  to  6%  on  his  total  investment? 

4.  In  a  gallon  of  a  certain  kind  of  paint  there  are  equal  parts  of  pigment 
and  oil.     How  much  oil  must  be  added  to  a  gallon  of  this  paint  to  make  a  paint 
of  $  pigment  and  %  oil? 

6.  A  merchant  made  25%  on  his  capital  the  first  year,  excluding  his  salary. 
He  withdrew  $1,800  for  his  personal  expenses,  and  had  $9,153.13  left.  Find 
the  amount  of  his  investment. 

6.  A  certain  candy  contains  40%  corn  syrup  and  60%  sugar.     The  syrup 
costs  2ff  per  pound  and  the  sugar  5^  per  pound.     If  sugar  advances  to  6^  and  the 
cost  of  the  syrup  is  unchanged,  in  what  proportions  must  the  ingredients  of 
the  mixture  be  used  in  order  to  keep  the  cost  the  same? 

7.  A  and  B  were  partners,  and  agreed  to  share  profits  in  proportion  to  the 
capital  invested.     The  profits  were  $2,000.     A  owned  a  f  interest  plus  $400, 
and  his  share  of  the  profits  was  $900.     What  was  the  value  of  the  business  and 
the  capital  of  each  partner? 

8.  The  audited  statements  of  a  company  show  the  following:  The  cash  is 
$2,400  more  than  the  expenses.     The  accounts  receivable  equal  twice  the  amount 
of  cash  less  $8,000.     The  cash  and  the  accounts  receivable  together  exceed  the 
expenses  by  $10,000.     Find  the  amount   of   cash,   accounts   receivable,   and 
expenses. 

9.  The  following  is  the  balance  sheet  of  the  B.  Company: 


EQUATIONS 


245 


Assets 

Cash $      200 

Accounts  Receivable 2,500 

Plant  and  Equipment 18,000 

Officers'  Accounts: 

President  Smith 3,000 

Treasurer  Brown 4,500 

$28,200 


Liabilities 

Accounts  Payable $  5,000 

Capital  Stock 10,000 

Surplus  or  Net  Profits 13,200 


$28,200 


By  agreement,  the  president  was  to  receive,  in  lieu  of  salary,  15%  of  the  net 
profits,  and  the  treasurer  was  to  receive  10%  of  the  net  profits.  Both  the 
deductions  were  to  be  included  in  expenses. 

The  treasurer,  who  was  not  bonded,  has  disappeared,  and  you  are  now 
requested  to  state  the  amount  of  Smith's  and  of  Brown's  accounts  and  also  the 
true  amount  to  be  credited  to  surplus. 

10.  You  are  requested  to  find  the  value,  for  consolidated  purposes,  of  the 
following  balance  sheets,  all  as  of  the  same  date: 


Assets,  other  than  stock 
Stock  in  B  Company 

A 
.    ..  $400,000 
60,000 

B 
$200,000 

C 
$200,000 
20,000 

Stock  in  C  Company          .  .  .  . 

60,000 

20,000 

Deficit 

40000 

$520,000 

$260,000 

$220,000 

Liabilities 

$200  000 

$160000 

$  40000 

Capital  Stock 

300,000 

100,000 

100  000 

Surplus     

20,000 

80,000 

$520,000 

$260,000 

$220,000 

The  investments  in  stock  were  at  par  and  cost,  there  being  neither  surplus  nor 
deficit  at  the  date  of  purchase. 

11.  Three  companies  agree  to  consolidate,  and  each  agrees  to  accept  its 
pro  rata  share  in  the  capital  stock  of  the  new  corporation,  D.  Corporation  D  i» 
formed  with  500,000  shares  of  no  par  value  stock. 


Total  Assets  ..............  $    750,000 


Total  Liabilities 
Capital  Stock 
Surplus 


125,000 
500,000 
125,000 


Total  Assets  ..............  $1,000,000    Total  Liabilities  ..........  $   375,000 

Capital  Stock  ............        375,000 

Surplus  ..................        250,000 


$1,000,000 


$1,000,000 


Total  Assets $1,750,000     Total  Liabilities $    550,000 

Capital  Stock 1,000,000 

Surplus 200,000 

$1775(^000  $1,750,000 


246  EQUATIONS 

STOCK  OWNERSHIP 

ABC 

A  owned  15%          15% 

Current  at $50,000    $150,000 

B  owned 15%  10% 

Current  at $75,000  $  37,500 

C  owned 5  %  5  % 

Current  at $25,000     $30,000 

What  percentage  of  the  capital  stock  of  Corporation  D  will  each  incorporalor 
receive,  arid  what  will  be  the  book  value  of  each  company's  interest  in  Corpo- 
ration D? 

Arithmetical  solution  of  problems  containing  unknown  quanti- 
ties. Home  accountants  prefer  to  solve  problems  containing 
unknown  quantities  by  an  arithmetical  rather  than  an  algebraic 
method.  The  arithmetical  method  consists  of  making  estimates 
of  the  unknown  quantities  on  the  basis  of  quantities  which  are 
known.  A  second  test  is  made,  based  upon  the  results  of  the  first 
estimate.  Succeeding  tests  or  approximations  are  then  made  until 
the  correct  value  is  ascertained. 

A  peculiarity  of  this  method  is  that  mistakes  made  in  the 
computations  will  always  be  eliminated,  and  the  final  computation 
will  always  be  correct.  An  error  in  computation  may  necessitate 
a,  greater  number  of  approximations,  but  in  the  end  it  will  be 
eliminated  in  the  process  of  solution. 

Example 

You  are  requested  to  find  the  value,  for  consolidated  balance  sheet  purposes, 
of  the  following  corporations.  The  separate  balance  sheets  show  the  value  of 
each  corporation  as  of  the  same  date. 

AMES  COMPANY 

Assets  $200,000  Liabilities  . .         .  .  $100,000 

Stock  of  Brown  Co.  (par)       30,000  Capital  Stock     .  .  .  150,000 

Stock  of  Coulter  Co.  (par)       30,000  Surplus 10,000 

$260,000  $2fyyJOQ 

BROWN  COMPANY 

Assets  $100,000     Liabilities  $  80,000 

Stock  of  Coulter  Co.  (par)       10,000     Capital  Stock 50,000 

Deficit 20,000  

$130,000  jiU30,QOQ 

COULTER  COMPANY 

AssetvS $100,000    Liabilities      .  $  20,000 

Stock  of  Brown  Co.  (par)       10,000     Capital  Stock 50,000 

Surplus 40,000 

$110,000  sno,ojo 

The  holding  company  purchased  the  stock  of  the  subsidiaries,  paying  book 
value  therefor.  Book  value  was  par,  for  the  subsidiaries  had  neither  surplus 
nor  deficit  at  the  date  of  acquisition  by  the  holding  company. 


£ 


J  -Q 

*<  K 

S  8 
HJg 

Q 
W 

H  — 


88 

O  O  I 


EQUATIONS 

S'8J   ££•§<=• 


SI 


O  C^  iJN 

O  Oi  '  O5 

O  Oi  '  O 


>  co   oo 
1  *'   cc 


(M   ^H     CO 
CO  00 


.0 
:o 


§0000 
O  (O  O  'O  , 

Q  Q  '  Q  Q  '  Q  ! 


^  o   w  —i  M  ^-<  w  f5 

0||      §  5f'~  ^  g  ^  -  S 

O5  .">.   +3  (r,  <-G  Q  O 

i       ^  <y-     y1  ..  x,x  O  <+-> 


c^»i 


y:    ^  rS    ;5  _r> 

^-•-__^o5^ 


CO 


88i8 


§0000 
o  o  o  ,0 

O  O  ,O  O  'O 


o  o  o  o  o        o  o  '  o  o  ID  'i    <x>  ~c  -Xs  r/l 


c3  f      I-H  <^|     r/    ^U     1)     O 

^  "  .s  f  .2  i  r°  £  « 


247 


§" o"  (>r  <>f  i 
j  O  CN  10  i  t 


08  • 

•T!    : 


a 
£ 
2' 


o 

CM 


•     CD 

.'  o 

•5    •  *® 

|s  ^^ 

s?     O    S- 


O 

^, 


c0^So2^^2'5    . .  t5  is      oo^ 


3  . 

o 

u 


s: 

PQi 


O    (1)    > 

S.5.S 


248  EQUATIONS 

Problems 

1.  A  corporation  wishes  to  create  an  insurance  fund  equal  to  25%  of  the 
net  profits  after  deduction  of  the  insurance  fund  and  of  the  manager's  bonus  of 
10%  of  the  profits,  the  bonus  and  the  insurance  fund  both  to  be  considered  as 
expenses.     The  profits  are  -147,250.     Find  the  amount  of  the  bonus,  the  insur- 
ance fund,  arid  the  balance  to  be  carried  to  surplus. 

2.  The  assets  and  liabilities  of  two  companies  are  as  follows: 

Smith  Jones 

Company  Company 

Assets,  exclusive  of  intercompany  investments  $360,000  $320,000 
Due  from  Jones  Company                  .  .        .              20,000 

Due  from  Smith  Company  SO, 000 

Deficit                    .      .                    .    .                        100,000  160,000 

i$4SO,000  $560,000 

Liabilities,  exclusive  of  intercompany  invest- 
ments  ...                .      .                   \    .  .           *400,000  $540,000 
Due  to  Jones  Company                                                SO, 000 
Due  to  Smith  Company  .                  ...  20,000 

84X0,000     #560,000 

Both  companies  having  failed,  you  are  required  to  state  how  many  cents  on 
the  dollar  each  firm  can  pay. 

3.  Three  companies,  M,  Ar,  and  O,  agree  to  consolidate.     Their  balance  sheets 
show  assets  as  follows: 

M:  Other  assets,  $  42,5,000  +  10%.V  -h  15%0 
N:  Other  assets,  462,500  +  15',',J/  +  I5%0 
O:  Other  assets,  1,145,000  +  V2%</0M  +  5%N 

What  are  the  assets  of  each  company? 


CHAPTER  26 
Logarithms 

Uses  of  logarithms.  It  is  not  the  purpose  of  this  chapter  to 
explain  how  logarithms  are  derived,  but  to  make  as  clear  as  possible 
the  simple  use  of  logarithms  by  the  accountant.  The  accountant 
desires  to  know  how  to  make  a  particular  calculation  accurately 
and  in  the  least  possible  time.  Logarithms  are  an  exceedingly 
valuable  tool  for  this  purpose  and  have  many  practical  applications. 

The  use  of  logarithms  greatly  simplifies  the  multiplying  and 
dividing  of  numbers,  the  raising  of  numbers  to  powers,  and  the  find- 
ing of  the  roots  of  numbers.  Logarithms  reduce  the  multiplying 
and  dividing  of  numbers  to  problems  of  addition  and  subtraction; 
the  finding  of  the  power  of  a  number  to  a  problem,  in  multipli- 
cation; and  the  extraction  of  a  root  to  a  problem  in  division. 

Exponents.  Logarithms  are  exponents;  that  is,  the  logarithm 
of  a  number  is  the  exponent  indicating  the  power  to  which  a  base 
must  be  raised  to  produce  the  number.  The  base  of  the  common 
system  of  logarithms  is  10.  Therefore: 

The  logarithm  of  100  is  2,  because  10  must  be  raised  to  the  2nd  power  to 
produce  100. 

The  logarithm  of  1,000  is  3,  because  10  must  be  raised  to  the  3rd  power  to 
produce  1,000. 

The  logarithm  of  10,000  is  4,  because  10  must  be  raised  to  the  4th  power  to 
produce  10,000. 

The  logarithm  of  100,000  is  5,  because  10  must  be  raised  to  the  5th  power  to 
produce  100,000. 

Obviously,  the  log  of  a  number  between  10  and  100  will  be 
something  between  1  and  2;  for  instance,  the  log  of  50  is  1. (59897. 
And  obviously,  the  log  of  a  number  between  100  and  1000  will  bo 
something  between  2  and  3;  for  instance,  the  log  of  625  is  2.79588. 
A  table  may  now  be  made  as  follows : 

Number  Log 

10  ...  I. 00000 

50  .  1  69897 

100  .      .  2  00000 

625 .  2  79588 

1000      3  00000 

10000          4  00000 

100000     5.00000 

249 


250  LOGARITHMS 

Parts  of  a  logarithm.  The  part  of  a  logarithm  at  the  left  of  the 
decimal  point  is  called  the  characteristic;  the  part  at  the  right  of  the 
decimal  point  is  called  the  mantissa. 

If  the  number  is  10  or  more  hut  less  than  100,  the  characteristic  is  1. 
If  the  number  is  100  or  more  but  less  than  1000,  the  characteristic  is  2. 
If  the  number  is  1000  or  moic  but  less  than  10000,  the  characteristic  is  3. 

Numbers  which  are  less  than  1  have  negative  characteristics. 
The  nature  of  positive  and  negative  characteristics  is  indicated 
below: 

Characteristic 

Number  of  Loq 

100,000  5 

10,000  .                                                        4 

1 ,000     .  3 

100     .  2 

10   .  1 

1.        .  0 

1      .  -1 

01..  -2 

.001  -3 

.0001  ...                                                    -4 

From  the  foregoing,  the  following  rules  for  the  determination 
of  the  characteristic  may  be  developed: 

Logs  for  the  number  1  and  all  numbers  in  excess  thereof  have 
positive  characteristics;  the  characteristic  is  one  less  than  the 
number  of  figures  at  the  left  of  the  decimal  point  in  the  number. 

Logs  for  numbers  less  than  1  have  negative  characteristics;  the 
characteristic  is  one  more  than  the  number  of  zeros  between  the 
decimal  point  and  the  first  significant  figure  at  the  right  thereof. 

The  usual  way  of  writing  the  logarithm  of  a  number  is  as 
follows: 

log  US  =  2.07 1SS2 

Sometimes  a  sign,  ";//,"  is  used  to  separate  a  number  from  its 
logarithm;  the  number  is  then  read:  118  nl  (the  number  whose 
logarithm  is)  2.071882;  or  reversed,  it  is  read:  2.071882  In 
(the  logarithm  of  the  number)  118. 

Characteristic.  The  characteristic,  as  has  been  stated,  is  the 
part  of  the  number  at  the  left  of  the  decimal  point  of  the  logarithm. 
The  characteristic  of  the  logarithm  of  each  number  from  1  to  9 
inclusive  is  0.;  from  10  to  99  inclusive,  it  is  1.;  from  100  to  999 
inclusive,  2. ;  and  so  forth.  Also,  the  characteristic  of  each  number 
from  .1  to  .9  is  —  1.;  from  .01  to  .09  inclusive,  —  2.;  and  so  forth. 


LOGARITHMS  251 

Examples 

The  logarithm  of      5.  has  a  characteristic  of  0. 
......     25.     "    "  "  *'    1. 

"  490.     "    "  "  "  2. 

The  logarithm  of  370.       has  a  characteri.stic  of      2. 

41       4.         "    "  "  "       0. 

«       0          "    "  "  "       0. 


. 


"  "  "         .3       "    "  "  "    -1.  or  9.  (mantissa)  -10 

.49     "    "  "  "    -1.  or  9.  "  -10 

Positive  characteristic.  From  the  foregoing  list  of  numbers 
and  of  the  characteristics  of  their  logarithms,  it  will  be  seen  that 
the  characteristic  for  the  log  of  41)0.  is  the  same  as  that  for  the  log 
of  370.  Also,  it  is  the  same  for  the,  log  of  4.  as  for  (he  log  of  2.7. 
It  is  not  the  value  of  the  digits  in  the  number,  but  the  number  of 
digits  at  the  left  of  the  decimal  point  in  the  number,  that  gives  the 
value  to  the  positive  characteristic. 

Negative  characteristic.  If  a  number  is  less  than  1.,  its  log  has 
a  negative  characteristic;  that  is,  the  log  of  .3  or  of  .49  has  a  nega- 
tive characteristic  of  —1.,  written  as  1.,  and  the  log  of  .01  or  of 
.0245  has  a  negative  characteristic!  of  —  2. 

Examples 

The  log  of  .().-,  2  (>99() 

The  lojr  of  .(MM).")  4   0990 

The  log  of  .000005  0  0990 

If  a  given  quantity  is  added  to  any  number,  and  from  the  sum 
is  subtracted  the  same  quantity,  the  result  is  the  same  as  the, 
original  number;  that  is,  if  10  be  added  to  4,  making  14,  and  from 
that  sum  10  be  subtracted,  the  result,  is  still  4,  although  the  form 
in  which  it  is  written  is  different;  as,  4  —  4  +  10  —  10.  Instead 
of  writing  a  logarithm  with  a  negative  characteristic  as,  1.1950 
in  some  calculations  it  is  found  to  be  more  convenient  to  indicate 
the  negative  characteristic;  in  the  following  manner:  9.1959  —  10. 
Or  it  may  be  convenient  in  some  cases  to  use  a  larger  number; 
thus,  1.1959  may  be  represented  as  29.1959  -  30,  or  as  0.1959  -  7. 
Any  number  may  be  added,  provided  the  same  number  is  used  as  a 
negative  quantity.  The  change  in  form  does  not  change  the  value. 

A  characteristic  may  be  either  positive  or  negative,  but  a 
mantissa  is  always  positive.  The  small  dash  over  the  characteristic 
is  intended  to  serve  as  a  reminder  that  only  the  characteristic  is  a 
minus  quantity,  while  the  mantissa  is  invariably  a  plus  quantity. 

Mantissa.  The  mantissa  of  the  log  of  a  number  is  a  group  of 
figures  which  stands  for  or  represents  the  sequence  of  the  digits  of 
the  number. 


252  LOGARITHMS 

The  mantissa  of  the  log  of  125.  is  shown  by  the  table  to  be 
.096910,  which  is  also  the  mantissa  of  the  logs  of  12.5,  1.25,  and 
125,000.  The  mantissa  is  determined  by  the  sequence  of  the  digits 
of  a  number,  and  the,  characteristic  is  used  to  show  the  correct 
placing  of  the  decimal  point. 

Examples  of  Numbers  and  the  Mantissas  of  Their  Logarithms 

Tho  sequence  of  dibits  in  125.  is  indicated  by  the  numti^a,  .090910 

"  "  "  1250.  "  "  <:  "  li  .090910 

"  "  "  "  "  125000.  "  "  "  "  <k  .090910 

"  "  "  12.5 "  4k  .090910 

"  "  "  "  "  1.25  4l  <4  "  '4  <v  .()9r>910 

«  n  „  ,2-  „  u  .,  t<  ..  .()()(;()!() 

«  u  n  i.  i,  4--  u  n  ,4  4.  „  .(J70(i94 

t-  a  u  ^25  *»  .•  <k  «<  -  .795SSO 

u  u  4*  ()9()<  «  .«  «<  -  «  .0(i;j7,ss 

"             "          "  "  1.  "          "            "  "  *'  .000000 

Examples  of  Numbers  and  Their  Complete  Logarithms 

iatic  and  ,\> 


loK  1235.  -  3.09107 

"      123.5  ----  2.09107 

"         123.")  -  1.09107 

1. 23.')  0  09107 

"            .123.")  -  1.09107 

.0123.')  -  2.09107 

"             .001235  -.-.  3.09 107 

The  table  given  in  Appendix  III  of  this  book  is  called  a  ,sv.r- 
pluce  table.  'This  means  that  the  mantissas  as  given  are  accurate, 
to  the  sixth  place.  However,  it  does  not  necessarily  follow  that  by 
the  use  of  a  six-place  table  calculations  can  be  performed  accurately 
to  the  sixth  place.  Tables  of  logarithms  accurate  to  six,  eight,  or 
even  ten  places  are  sometimes  used,  but  the  six-place  table  is 
sufficient  for  ordinary  purposes.  If  the  accountant  ha>  many 
computations  involving  large  numbers,  he  should  procure  a  more 
extended  table. 

How  to  use  a  table  of  logarithms. 

For  numbers  of  one  siynijicant  figure.  The  table  shows  only  the 
mantissa  of  each  logarithm.  If  it  is  desired  to  find  the  mantissa  of 
a  number  such  as  2,  20,  200,  2,000,  or  of  any  number  whose  only 
significant  figure  is  2,  it  is  necessary  to  turn  to  the  table  and  in  the 
column  at  the  left,  headed  "Ay  run  down  the  line  until  the  number 
2(K)  is  reached.  To  the  right  of  this  number,  in  the  column  headed 
'()/'  the  mantissa  .301030  is  found.  This  is  the  mantissa  for  the, 
logs  of  2,  20,  200,  or  .2,  .02,  .002,  and  so  forth.  The  mantissa  for 


LOGARITHMS  253 

any  other  number  of  one  significant  figure  may  be  found  in  a 
similar  manner.  In  order  to  obtain  the  complete  logarithm  of  a 
number  it  is  necessary  to  supply  the  characteristic.  The  charac- 
teristic of  the  log  of  2  is  ().  Hence,  the  complete  logarithm  of  2  is 
0.30 1030. 

For  numbers  of  two  significant  fit/nres.  If  it  is  desired  to  find 
the  logarithm  of  a  number  containing  two  significant  figures  as, 
17,  170,  or  1.7  it  is  necosary  to  look  in  the  column  at  the  left  of 
the  table,  headed  "A'/'  and  run  down  the  column  until  170  is 
reached.  In  the  column  to  the  rin;ht  of  170,  headed  "0,"  the 
mantissa  .230441)  is  found.  Keep  in  mind  that  this  is  the  mantissa, 
for  the  logs  of  17,  170,  1,700,  and  so  forth. 

For  numbeis  of  three  snjnijicnnt  figures.  Assume  that  the 
logarithm  of  I IX  i>  desired.  In  the  left-hand  column  of  the  table, 
headed  lk  A,"  iind  t  he  number  1 1 S.  To  t  he  riirht  of  t  his  number,  in 
the  column  headed  "0,"  the  mantissa  .071 SS2  is  given.  This,  of 
course,  is  the  mantissa  for  the  logs  of  1  IS,  1.1S,  1 1  ,<SOO,  and  so 
forth.  In  the  foregoing  illustrations,  the  mantissa  only  was  found. 
By  the  rules  previously  given,  the  characteristic  of  the  log  of  IIS 
is  ascertained  to  be  2.  Therefore,  t  he  logarithm  of  the  number 
I  IS  is  2.071SS2. 

For  numbers  of  four  sn/nijieont  Jujures.  To  illustrate:  It/  is 
required  to  find  the  logarithm  of  1,(>4S.  Find  the  number  1(>4  at 
the  left  of  the  table,  in  the  column  headed  "  A  V  On  the  horizontal 
line  to  the  right  of  104,  in  the  column  headed  "S,"  the  mantissa 
found  is  .2K>Of>7.  The  characteristic  for  the  log  of  1,04X  is  3. 
The  complete  logarithm  of  1,048  is  3.210957. 

Interpolation  for  numbers  of  Jive  or  more  si(/nijie(int  Ji(/ure,s. 
The  logarithm  of  a  number  of  five  or  more  significant,  figures  may 
be  found  by  the  process  of  interpolation.  This  method  is  based 
upon  the  assumption  that  the  differences  of  the  mantissas  are 
proportional  to  the  differences  of  the  numbers  given.  This  pro- 
portion is  not  strictly  exact,  for  the  difference's  really  grow  smaller 
as  the  mantissas  themselves  grow  larger.  However,  the  results 
obtained  deviate  only  slightly  from  the  true  results,  and  are 
sufficiently  accurate  for  most  purposes. 

Example 

Find  the  logarithm  of  131,525. 

Solution 

In  the  column  headed  "/)/*  on  the  line  horizontal  to  the  number  131  in  the 
column  headed  ''A',"  the  difference  between  the  mantissa  of  the  log  of  1,315  and 
the  mantissa  of  the  log  of  the  next  higher  number,  1,310,  is  given  as  330;  this  is 
more  correctly  stated  as  .000330.  The  excess  of  131,525  over  131, .500  is  T2A  of 
the  difference  between  131. ,500  and  131,600.  Therefore,  multiply  .000330  by 


254  LOGARITHMS 


)  obtain  the  fractional  part  of  the  difference  in  the  mantissa;  it  is  .0000825. 
The  mantissa  for  131,500  is  .1 1X926.  To  this  add  the  .OOOO.S25,  which  will  give 
the  mantissa  for  131,525,  .1 1900S5.  Stated  again: 

The  log  of  131,600.  has  a  mantissa  of  .1  19256 
"      "    "    131,500.     u    "          "         "   .11S926 
The  difTerence  of          100  =  the  difTerence  of  .000330 
131,525  -  131,500  =  25 

The  difTerence  of  25  in  the  numbers  requires  that  a  proportional  part  of  the 
difference  in  the  mantissas,  .000330,  be  added  to  the  mantissa  oi  the  log  of  the 
smaller  number. 

iVo  of  .000330  =  .OOOOS25 
.1  1X920   1-  .0000X25  -  .1  1900X5 

.11900X5  is  approximately  the  correct  mantissa.  The  characteristic  for  the 
log  of  131,525  is  5.  Therefore,  the  complete  log  of  131,525  is  5.1  1900X5. 

Problems 

Find  the  logarithms  of  the  following  (express  your  answers  in  the  form: 
"log  1  IX  -  2.071XX2"). 


1.  4 

6.  5 

11.  1.127 

16.  .S237X 

2.  20 

7.  X2 

12.  1,275 

17.  .03264 

3.  30 

S.  775 

13.  1,4S2 

18.  .0003X2 

4.  .00 

9.  S27 

14.  739.S2 

19.  1.00375 

5.  25 

10.  S.37 

15.  6X,439 

20.  2  4S7(>5 

To  find  a  number  when  the  logarithm  is  given.  If  the  above 
process  of  finding  the  logarithm  from  a  number  is  reversed,  the 
number  can  easily  be  found  from  the  logarithm.  This  process  is 
called  finding  the  antilogarithm.  It  is  necessary  first  to  find  the 
digits  of  the  number,  and  this  must  be  done  from  the  mantissa. 
If  the  mantissa  can  be  found  in  the  table,  take  the  digits  corre- 
sponding to  the  mantissa,  and  point  off  these  digits  decimally  as 
indicated  by  the  characteristic. 

Example 

Find  the  number  whose  logarithm  is  0.2S1033. 

Solution 

In  the  table,  the  mantissa,  .2S1033,  is  found  in  the  vertical  column  headed 
"0,"  opposite  the  number  191.  This  indicates  that  the  sequence  of  digits, 
ignoring  possible  initial  or  final  zeros,  is  191.  Using  the  characteristic  for  the 
correct  placing  of  the  decimal,  the  number  is  found  to  be  1.91. 

To  find  a  number  whose  mantissa  is  not  in  the  table.     If  the 

mantissa  is  not  given  in  the  table,  it  is  necessary  to  reverse  the 
process  of  interpolation. 


LOGARITHMS  255 


Example 

Find  the  number  \vho>e  logarithm  is  5.11900S5. 


In  the  table,  the  mantissa  for  the  number  nearest  the  one  given,  and  less 
than  it,  is  found  to  be  .11S92(>.  Subtracting  this  from  the  mantissa  given  in 
the  problem,  the  difference  is  .OOOOS25.  Use  this  as  the  numerator,  and  the 
diffeience,  .000330  (indicated  in  the  table  in  the  column  at  the  right,  headed  "  /)") 
as  the  denominator.  Reduce  this  fraction  .OOOOS25  .0003300;  the  result  is  .25, 
or  25  100  of  the  difference  between  1,315  and  1,31(>.  Ar.ncx  this  amount  to 
1,31*5)  and  the  sequence  of  digits  is  found  to  be  131,525,  which,  when  pointed 
off  as  indicated  by  the  characteiistic,  ghes  the  result,  131,525. 

Problems 

Find  the  numbers  lepresented  by  the1  tollo\\ing  logarithms: 

1.  l.()5X011  6.    1J)9M)70  11.  S.I  720  19-  10  16.  4.037540 

2.  2.71IS07  7.  2.<»M7.s4  12.  19.003S91   -20  17.  2.0S2()S5 

3.  0.0X1241  8.  3.C>2107(>  13.  3.17(i()<)l  -4  18.  <>.  403794  -  7 

4.  9.67209S  -  10       9.  5.(>3,s9XS  14.  (U321XS  -  9  19.  3.390791 

5.  2.707570  10.    I.S()()23<>  15.  7.30304S  20.    1.S44334 

Rules  for  computation  by  logarithms. 

RULE  1.  To  multiply  numbers,  add  their  logarithms;  the  sum 
is  the  logarithm  of  tin*  product. 

RrLK  2.  To  divide  numbers,  subtract  the  logarithm  of  the 
divisor  from  the  logarithm  of  the  dividend;  the  remainder  is  the 
logarithm  of  the  quotient. 

Rt'LK  3.  To  obtain  a  power  of  a  number,  multiply  the  loga- 
rithm of  the  number  by  the  exponent  of  the  power  sought;  the 
product  is  the  logarithm  of  the  power  of  the  number. 

RTLK  4.  To  obtain  a  root  of  a  number,  divide  the  logarithm 
of  the  number  by  the  index  of  the  root  sought;  the  quotient  is  the 
logarithm  of  the  root  of  the  number. 

Multiplication  by  logarithms. 

Example 

Multiply  H35  by  22,  using  the  method  of  Rule  I. 

Solution 

log  635  2  802774 

log  22  1   342423 

log  of  product         .  ...   4.145197 

In  the  table,  the  mantissa  .145190  is  found  to  correspond  to  the  digits  1,397. 
The  characteristic  4  indicates  that  the  product  has  five  digits  to  the  left  of  the 
decimal  point,  hence,  the  product  is  13,970. 


256  LOGARITHMS 

Problems 
Multiply: 

1.  25  by  25  8.  1.43  by  .032  15.   145.3  by  6.296 

2.  42  by  37  9.  1,480  by  .138  16.  .003  by  .002 

3.  240  by  381  10.  92.7  by  8.75  17.  100.05  by  100.25 

4.  762  by  431  11.  3.39  by  8.92  18.  47.2  by  200 

5.  42.5  by  49.2  12.  9.293  by  48.67  19.  .999  by  647.2 

6.  34.7  by  1.42  13.  143.9  by  1.478  20.  3.8  by  4.9 

7.  1,430  by  .249  14.  1.278  by  3.84 

Division  by  logarithms.     Rule  2  gives  the  procedure  for  divi- 
sion by  means  of  logarithms. 

Division  of  a  greater  number  by  a  lesser  number. 

Example 
Divide  S75  by  37. 

Solution 

log  875                            ...  2  94200S 

log  37  I  56X202 

log  quotient  1  373X06 

In  the  table,  the  mantissa,  .373X06,  coi  respond-;  to  the  dibits  23,648.  The 
characteristic  indicates  that  the  quotient  has  t\\v  digits  to  the  left  of  the  decimal 
point.  Pointed  oil,  the  an>\ver  is  23.648. 

Division  of  a  lesser  number  by  a  greater  number.  At  times  it  is 
necessary  to  divide  a  smaller  number  by  a  larger  one;  this,  of 
course,  produces  a  quotient  which  is  less  than  1,  and  requires  the 
operation  of  subtraction  of  a  greater  logarithm  from  a  lesser  one. 
To  do  this,  change  the  form  of  the  logarithm  of  the  minuend;  that 
is,  add  10  (or  a  multiple  of  10)  to  the  characteristic  of  the  minuend, 
and  write  -10  (or  a  multiple  of  -U),  the  same  multiple  to  be  used  in 
each  case)  after  the  mantissa. 

Example 
Divide  269  by  239,000. 

Solution 

log  269  -  2.429752,  or  12  429752  -  10 

Deduct  log  239,000,  or  ...  5  378398 

log  quotient.  ...  7  0.51354  —  10 

Changing  7.051354  —  10  to  its  simple  form,  it  becomes  3.051354.  By 
reference  to  the  logarithm  table  of  mantissas,  and  by  applying  the  rule  for 
pointing  off  numbers  by  the  characteristic,  the  quotient  is  found  to  be  .0011255 
(correct  to  five  significant  figures). 

The  above  method  is  also  applicable  to  the  subtraction  of  a 
negative  logarithm  from  a  positive  one. 


LOGARITHMS 


257 


Example 

Divide  14,200  by  .000191. 

Solution 

log  14.200  =  4.152288,  or 14. 152288  -  10 

log  .000191  =  4.281033,  or 6  281033  -  10 

log  quotient 7.871255  -    0 

By  finding  the  antilog  of  7.871255,  the  resulting  quotient  is  ascertained  to 
be  74,345,500  (correct  to  five  significant  places). 

Problems 

Divide: 

1.  128  by  64  8.  2.486  by  3.45 

2.  2,160  by  150  9.  .6843  by  89 

3.  344  by  8  10.  9.278  by  12.43 

11.  6  by  2  * 

12.  89  by  47 

13.  2.1  by4S 

14.  3.875  by  23S.7 

Rule 


4.  93  by  .31 

5.  649.4  by  24.3 

6.  8.42  by  2.48 

7.  .3472  by  124 

Powers  of  numbers. 

the  powers  of  numbers. 


15.  1.425  by  892.7 

16.  147.25  by  9,276 

17.  .03  by  6,000 

18.  .01 25  by  3,427 

19.  1 .005  by  927.8 

20.  2.4255  by  384.275 

21.  6,497.S  by  2.S74 


gives  the  procedure  for  finding 

Example 

Find  the  fourth  power  of  26. 

Solution 

log  26  .  1    414973 

Multiply  by  the  exponent  of  the  power        .    .  .  .  4 

log  power  .    5  659892 

In  the  table,  the  mantissa  .659892  represents  the  sequence  of  digits  456,975. 
The  chin acteristic  5  indicates  that  the  product  has  six  digits  to  the  left  of  the 
decimal  point.  Hence,  the  fourth  power  of  26  is  456,975  (correct  to  five  signifi- 
cant places). 

Process  with  a  negative  characteristic. 

Example 

Find  the  fifth  power  of  .025. 

Solution 

log  .025  =  2.397940,  or  

Multiply  by  the  exponent  of  the  power. . 


41.9S9700  -  50,  changed 

The  mantissa  .9897 

Pointed  off  bv  the  characteristic 


Find  the  value  of: 

1.  5th  power  of  25 

2.  4th  power  of  35 

3.  5th  power  of  2 

4.  7th  power  of  1 .25 
6.  4th  power  of  2.47 

6.  3rd  power  of  575 

7.  9th  power  of  4 


Problems 

8.  12th  power  of  7 

9.  121 

10.  142 

11.  9,200' 

12.  18* 

13.  1472 

14.  .01 52 


8  397940  -  10 

—       J? 
4 1   989700  -  50 

9  9897 
976562 

00000000976562 


16.  8.921 

16.  1461 

17.  .07* 

18.  .97* 

19.  303 

20.  274 

21.  9B 


258  LOGARITHMS 

Roots  of  numbers.  Rule  4  gives  the  procedure  for  finding 
the  roots  of  numbers. 

Example 

Find  the  cube  root  of  875. 

Solution 

log  875 2.942008 

Divide  by  index  of  the  root 3)2_M200^ 

log  quotient 0.980669 

In  the  table,  the  mantissa  .980669  corresponds  to  the  sequence  of  digits 
95,646,  and  the  characteristic  indicates  that  the  number  is  9.5646. 

Process  with  negative  characteristics.  In  stating  the  equiva- 
lent of  the  negative  characteristic  of  a  logarithm,  care  must  be 
taken  to  see  that  the  right-hand  number,  or  minus  quantity,  is 
exactly  divisible  by  the  index  of  the  root  with  a  quotient  of  10  or  a 
multiple  thereof. 

Example 

Find  the  4th  root  of  .125. 

The  negative  characteristic  of  the  complete  logarithm  of  .125  may  be  stated 
in  several  different  ways:  as  1.096910;  as  3.096910  -  4,  as  7.096910  -  8,  as 
19.096910  -  20;  or  as  39.096910  -  40. 

In  this  problem,  in  order  to  avoid  any  complication,  it  is  best  to  have  the 
right-hand  number  of  the  characteristic  exactly  divisible  by  4  (the  index  of  the 
required  root),  with  a  quotient  of  10. 

Solution 

log  .125 I  09691C 

Or,  log  .125     39  096910  -  40 

Divide  by  index  of  root 4)_39J^969i°J7  i9 

log  quotient 9  774228  -  10 

Or 1.774228 

The  mantissa  .774228  corresponds  to  the  succession  of  digits  594,604  (accurate 
fo  five  places).  Pointed  off  by  the  characteristic,  —1,  the  4th  root  of  .125  is 
594604. 

Problems 

Find  the  value  of  the  following  to  five  significant  places: 

1.  Square  root  of  64.  8.  10th  root  of  10. 

2.  Square  root  of  97.  9.  \Xl25. 

3.  Square  root  of  .64.  10.  tyl.  _ 

4.  Cube  root  of  81.  11.  ^89. 

6.  Cube  root  of  .081.        12.  ^/89~27.  19.  506  X 

6.  6th  root  of  49.  13.  V-9643.  20.  3.872  X 

7.  7th  root  of  750.  14.  ^2980.  21.  -v/225  X 

The  slide  rule.  This  is  an  old  device,  yet  it  is  used  to  only  a 
limited  extent  by  accountants  in  general.  It  has  been  described  as 
"logarithms  on  a  stick."  In  its  simple  form,  this  mechanical 


LOGARITHMS  259 

device  consists  of  a  grooved  base  rule  into  which  a  slide  rule  is 
fitted.  A  third  part  is  the  runner.  The  graduations  on  the  upper 
part  of  the  base  and  slide  are  called  "upper  scale/'  while  those  on 
the  lower  part  of  the  base  and  slide  are  called  "lower  scale. " 
These  graduations  are  for  different  purposes,  and  because  of  the 
different  requirements  they  are  graduated  differently. 

Use  of  slide  rule.  The  slide  rule  is  used  either  to  check  figures 
or  for  original  calculations.  It  may  be  used  to  check  or  compute 
any  operation  involving  multiplication,  division,  raising  to  powers, 
or  extracting  of  roots.  It  has  a  great  many  applications  in  busi- 
ness, although  it  is  generally  considered  as  a  device  used  only  by 
engineers. 

The  slide  rule  should  appeal  to  the  accountant  as  a  device 
which  may  be  carried  in  the  pocket  or  the  brief  case;  its  weight 
is  negligible. 

Accuracy  of  calculations  made  by  the  slide  rule.  It  is  possible, 
after  having  attained  proficiency  in  the  handling  of  the  slide  rule, 
to  obtain  results  in  which  the  margin  of  error  will  not  be  more  than 
one-quarter  of  1%.  This  is  satisfactory  for  most  business  prob- 
lems. As  in  logarithms,  when  the  slide  rule  is  used  the  digits  are 
taken  from  the  left  to  the  right  of  a  number,  regardless  of  the  value 
of  the  number.  The  slide  rule  is  as  nearly  accurate  in  the  calcula- 
tion of  decimal  numbers  as  it  is  in  the  calculation  of  whole  numbers 
of  large  denominations.  It  will  give  correct  answers  of  two  places, 
and  if  careful  computations  have  been  made,  a  three-place  solution 
of  a  good  degree  of  accuracy  may  be  expected.  If  extreme  care  is 
used  in  the  computation  of  a  problem,  an  answer  of  four  places  may 
be  had  with  a  fair  degree  of  accuracy.  Of  course,  a  long  and  care- 
fully graded  slide  rule  will  give  better  results  than  either  a  short  or 
a  carelessly  graded  rule. 

Theory  of  the  slide  rule.  The  theory  of  the  slide  rule  is  indi- 
cated very  roughly  in  the  following  simple  example  and  illustration. 

Example 

Find  the  sum  of  4  and  6. 


RULE1 

01234       5678      9      10     11  12 

01234       56789       10     11  12 

RULE  2 

Figure  2. 

Solution 

Above  are  two  ordinary  rulers,  set  opposite  each  other.     To  find  the  sum  of 
4  and  6,  perform  the  following  steps: 


260  LOGARITHMS 

(1)  Set  the  "0"  on  Rule  1  over  the  "4"  on  Rule  2. 

(2)  Observe  the  figure  "6"  on  Rule  1. 

(3)  On  Rule  2,  immediately  below  the  "6"  on  Rule  1,  will  be  found  "10"— 
the  sum  of  4  and  6. 

The  process  of  subtraction  may  be  shown  as  follows. 

Example 
Find  the  difference  between  9  and  4. 


RULE  1 

0 

1 

2 

3 

4 

5 

678 

9 

10     11  12 

0     1 

2345 

6 

7 

8 

9 

10 

11  12 

RULE 

2 

Figure  3. 

Solution 

(1)  Locate  the  subtrahend  "9"  on  Rule  2. 

(2)  On  Rule  1,  locate  the  minuend  "4,"  and  place  it  immediately  over  bhe 
subtrahend  "9"  on  Rule  2. 

(3)  On  Rule  2,  the  number  immediately  below  the  "0"  on  Rule  1  will  be  the 
remainder,  which  in  this  case  is  5. 

The  foregoing  examples  show  that  addition  and  subtraction 
of  small  numbers  can  be  performed  on  two  ordinary  rulers.  The 
principle  of  the  slide  rule  is  similar. 

On  the  common  slide  rule,  the  graduations  are  made  according 
to  logarithms.  Hence,  if  any  two  numbers  on  the  slide  rule  are 
added,  the  result  obtained  is  the  sum  of  two  logarithms. 

The  addition  of  the  logarithms  of  numbers  results  in  multipli- 
cation of  the  numbers;  and  the  subtraction  of  the  logarithms  of 
numbers  results  in  division  of  the  numbers. 

How  to  learn  to  use  the  slide  rule.  Practice  is  without  doubt 
the  only  efficient  method  of  learning  how  to  use  the  slide  rule.  If 
possible,  a  slide  rule  should  be  obtained  for  use  in  this  chapter. 
If,  however,  a  slide  rule  is  not  available,  a  cardboard  model  may  be 
made  for  practice.  Care  must  be  used  to  have  the  markings  as 
accurate  as  possible,  in  order  to  obtain  fair  results.  A  model  such 
as  the  following  can  be  used  very  conveniently  for  the  practice 
material. 


i                                 ra                   13 
i     i    i    i    i    i  i  i  i  1                     1 

4 

5           |6 

7        |8 

9 

i         i        I       I       I       1      1      1     1      I                                  "    j 

1                                             !2                          !3 

4 

5            6 

7        8 

9 

Figure  4. 


LOGARITHMS  261 

Reading  the  slide  rule.  In  reading  the  numbers,  go  over  the 
rule  from  left  to  right,  as  follows:  1,  2,  3  ...  10;  then,  beginning 
at  1  again  and  calling  it  10,  read  10,  20,  30  ...  100;  then,  again 
beginning  at  1  and  calling  it  100,  read  100,  200,  300  ...  1,000. 
It  is  possible  to  do  this  because  the  mantissa  for  10  is  the  same  as 
that  for  100,  1,000,  etc. 

It  will  be  noticed  in  Figure  4  that  the  spaces  decrease  from  left 
to  right.  These  decreases  should  correspond  exactly  to  the  differ- 
ences between  the  logarithms  from  1  to  10.  Assume  that  you 
divide  your  model  rule  into  1,000  equal  parts.  Then,  since  log 

2  =  .301,  the  2  would  be  placed  at  the  301st  graduation.     Log 

3  =  .477;  therefore  3  would  be  placed  at  the  477th  graduation. 
Log  4  =  .602;  therefore  4  would  be  placed  at  the  602nd  gradua- 
tion.    Similarly,  5  would  be  placed  at  the  698th;  6  at  the  778th; 
7  at  the  845th;  8  at  the  903rd;  and  9  at  the  954th. 

Marks  can  be  put  in  to  show  the  mantissas  for  the  logarithms 
of  1.5,  2.5,  3.5,  and  so  on,  but  they  will  not  be  half  the  distance 
between  the  previous  graduations,  because  log  1.5  is  0.176,  and  this 
is  not  half  the  difference  between  log  1  and  log  2. 

It  will  be  noticed  that  the  distance  from  1  to  2  is  divided  into 
10  divisions.  These  are  read  from  left  to  right,  like  telephone 
numbers,  thus:  one-one,  one-two,  one-three,  and  so  forth,  to 
one-nine;  then  2.  These  are  understood  as  1.1,  1.2,  1.3,  and  so 
forth.  Consulting  the  table  of  logarithms  for  the  logs  of  110, 
120,  130,  and  so  forth,  we  find  that  the  marks  will  be  placed  at  the 
following  graduations:  41,  79,  113,  146,  176,  204,  230,  255,  and 
278. 

Construction  of  model  slide  rule.  Obtain  a  piece  of  cardboard 
12^  inches  long  and  1  inch  in  width.  Rule  a  line  midway  along  the 
full  length,  and  mark  it  off  in  graduations  of  one-eighth  inch.  This 
will  make  a  measure  with  100  graduations  instead  of  1,000,  but  for 
the  purpose  of  this  work  it  will  be  satisfactory. 

Since  log  2  =  .301,  mark  2  at  30.1 
log  3  =  .477,  mark  3  at  47.7 
log  4  =  .602,  mark  4  at  00.2 
log  5  =  .698,  mark  5  at  69.8 
log  6  =  .778,  mark  6  at  77.8 
log  7  =  .845,  mark  7  at  84.5 
log  8  =  .903,  mark  8  at  90.3 
log  9  =  .954,  mark  9  at  95.4 

Between  1  and  2  are  1.1,  1.2,  1.3,  ..,  1.9,  as  previously 
explained. 


262 


LOGARITHMS 


Since  log  1.10  =  .041,  mark  at  4.1 
log  1 .20  =  .079,  mark  at  7.9 
log  1.30  =  .113,  mark  at  11.3 
log  1.40  =  .146,  mark  at  14.6 
log  1.50  =  .176,  mark  at  17.6 
log  1 .60  =  .204,  mark  at  20.4 
log  1.70  =  .230,  mark  at  23,0 
log  l.SO  =  .255,  mark  at  25.5 
log  1.90  =  .278,  mark  at  27.8 

For  closer  graduations,  you  will  find  that  you  can  make  10 
Indentations  with  a  sharp  pin  in  one-eighth  inch  space.  By  care- 
fully counting  the  points  placed,  you  can  use  the  full  logarithm 
and  make  a  fairly  accurate  slide  rule. 

Having  completed  the  graduations,  cut  the  cardboard  length- 
wise on  the  medial  line.  This  will  give  two  pieces  with  measure- 
ments exactly  the  same.  These  two  measures  may  now  be  used  as 
Rule  1  and  Rule  2  in  the  following  simple  problems. 

Multiplication  on  the  slide  rule.  Add  the  logarithms  of  the 
numbers  to  be  multiplied. 

Example 

Multiply  2  by  3. 


RULE  1 

1                            2 

3 

4 

5       6 

7 

8 

91 

1                            2                 3           4 

5       6 

7     8 

9  1 

RULE 

2 

Figure  6. 

Solution 

(1)  Locate  "2"  on  Rule  2. 

(2)  Place  "1"  on  Rule  1  over  "2"  on  Rule  2. 

(3)  Locate  "3"  on  Rule  1. 

(4)  Read  the  number  on  Rule  2  immediately  below,  which  is  "6." 

Problems 

Multiply: 

1.  2  by  4.      2.  3  by  4.     3.  2  by  5.     4.  3  by  2.     6.  4  by  2.     6.  3  by  3. 

If  the  problem  is  of  such  a  nature  that  the  rules  cannot  be 
operated  by  placing  the  left-hand  "1"  on  Rule  1  over  the  number 
on  Rule  2,  it  is  necessary  to  use  the  right-hand  "I"  on  Rule  1. 

Example 
Multiply  4  by  5. 


1 

2               345 

678 

91 

RULE  1 

BULE2 

1                          2 

3 

456 

7    8    91 

Figure  6, 


LOGARITHMS  263 

Solution 

(1)  Locate  "4"  on  Rule  2. 

(2)  Place  "  1  "  on  Rule  1  over  "4"  on  Rule  2,  so  that  the  "5"  on  Rule  I  is 
over  Rule  2.     In  this  case  it  is  necessary  to  use  the  "  1 "  on  the  right-hand  side 
of  Rule  1. 

(3)  Read  the  number  immediately  under  "5"  on  Rule   1,  which  is  "2." 
"2"  may  be  either  2.,  .2,  .02,  20,  200,  or  any  other  number  in  which  the  left-hand 
digit  is  "2."     In  this  case  the  number  can  be  determined  by  inspection  to  be  20. 

Problems 

Multiply: 

1.  5  by  S.     2.  4  by  5.     3.  3  by  10.     4.  20  by  20.     5.  40  by  5.     6.  2  by  30. 

Division  on  the  slide  rule.     Subtraction  of  logarithms  results 
in  the  division  of  the  numbers. 

Example 

Divide  9  by  (>. 


RULE  1 

1                                 2 

i 

3 

4 

5        G      7     8    0  1 

1  '                  '  '  '  '  2                  3 

4          5 

6 

7     8    U  1 

RULE  2 

Figure  7. 

Solution 

(1)  Locate  "9"  on  Rule  2. 

(2)  Place  "6"  on  Rule  1  over  "9"  on  Rule  2. 

(3)  Read  on  Rule  2  immediately  below  "  1  "  on  Rule  1.     As  the  "  1  "  is  over 
the  5th  graduation  on  Rule  2,  the  digits  will  be  15,  and  by  inspection  the  answer 
is  determined  to  be  1.5. 

Problems 
Divide: 

1.  6  by  2.  4.  6  by  4.  7.  35  by  7.  10.  95  by  5. 

2.  X  by  4.  5.  9  by  4.  8.  25  by  5.  11.  IS  by  12. 

3.  5  by  2.  6.  64  by  8.  9.   12  by  4.  12.  30  by  IS. 

Another  type  of  problem  is  that  of  multiplying  two  or  more 
numbers  and  dividing  their  product  by  another  number. 

Example 
8X9-5-4  =  ? 

Solution 

(1)  Set  Rule  1  so  that  the  "  1  "  on  the  right  is  over  "8"  on  Rule  2. 

(2)  Read  the  number  immediately  under  "9"  on  Rule  1,  which  is  "72." 

(3)  Set  "4"  on  Rule  1  over  "72"  on  Rule  2. 

(4)  Read  the  number  on  Rule  2  immediately  under  "1"  on  Rule  1,  and  the 
answer  is  found  to  be  18. 

Problems 
Solve  the  following  by  the  use  of  the  slide  rule: 

1.  16  X  6  -r-  32.      3.  35  X  35  -f-  5.      6.  37  X  19  -i-  13.      7.  8  X  14  -h  12. 

2.  20  X  40  +  8.      4.  45  X  12  -i-  8.      6.  44  X  34  ~  27.      8.  4  X  7  -^  200. 

The  following  problems  illustrate  some  of  the  practical  appli- 
cations of  the  slide  rule. 


264  LOGARITHMS 

Problems 

1.  Payroll  calculation.     "Brown's  time  rani  for  a  particular  clay  showed  the 
following: 

Time 


Job  No. 

II 

M 

12 

3 

30 

21 

2 

10 

32 

50 

45 

1 

30 

S~ 

00 

If  Brown  was  paid  54^  an  hour,  what  was  the  labor  cost  chargeable  to  each 
job? 

2.  Prorating  expend .     The  power  cost  of  a  small  plant  is  to  be  distributed 
to  the  departments  on  the  basis  of  horsepower  hours,  as  follows: 

Dept.  A  .                                 45  horsepower 

Dept.  B..  35  horsepower 

Dept.  C     .  .  .  .  90  horsepower 

Dept.  1)   ....  .                                        20  horsepower 

Dept.  10. ..                              .  .5  horsepower 

Dept.  K  ...            5  horsepower 

Total 200  horsepower 

The  total  power  cost  was  $450.     What  was  the  cost  of  power  in  each  depart- 
ment ? 

3.  The  air  fare  from  .V  to  }"  is  $40.     The  traveler  goes  over  three  divisions 
of  airway,  respectively  380,  230,  and  190  miles  in  length.     Find  the  amount  of 
the  (are  to  be  apportioned  to  each  division. 

4.  An  article  that  cost  $25  is  sold  for  $50  less  20%-     Find  the  per  cent  of 
gain  on  the  cost.     Find  the  per  cent  of  gain  on  the  selling  price. 

6.  (liven: 

Sales .    .    $500 

Cost  of  (ioods  Sold  300 

Selling  Expenses                     .                          .  75 

(ieneral  Expenses                     .      .  50 

Profit 

What  per  cent  of  the  sales  is  each  item? 

6.  Work  the  following: 

%  on  Selling 

(^ost  Selling  Price,  Price 

(a)         $  5  00  20                    ...     

(6)              8  00  40  .      ... ... 

(c)  12  00  25  ..      ... 

(d)  20  00  20 

(e)  10  00  30                   

7.  The  list  price  of  an  article  is  $25,  less  10%  and  5%.     Find  the  net  cost 

8.  Find  the  interest  on  $600  at  5%  for  3  years  6  months. 

9.  A  field  is  40  rods  wide  and  80  rods  long.     How  many  acres  does  it  contain? 
10.  Find  the  cost  of  SO  items  at  $1.50  a  dozen. 


CHAPTER  27 
Graphs  and  Index  Numbers 

Charts  and  graphs.  Charts  and  graphs  arc  becoming  increas- 
ingly popular  as  a  means  of  presenting  the  results  of  accounting 
and  mathematical  computations.  Accountants,  credit  men,  pro- 
duction managers,  sales  managers,  advertising  men,  and  general 
business  executives  are  realizing  more  and  more  how  greatly 
graphic  charts  may  help  them  in  their  work.  The  reason  is 
obvious.  Long  rows  of  figures  must  be  thoroughly  studied  if  the 
relations  between  quantities  are  to  be  grasped.  This  is  a  tedious 
task.  On  the  other  hand,  pages  of  valuable  data  may  be  presented 
on  a  simple  chart  that  will  convey  more  real  information  than  the 
most  elaborately  written  report.  It  is  necessary,  however,  to 
distinguish  between  important  and  unimportant  data.  Fur- 
thermore, a  method  of  presentation  must  be  chosen  that  will 
convey  a  correct  impression,  for  it  is  quite  possible  to  prepare 
misleading  charts  from  correct  data.  Two  important  points 
must,  therefore,  be  borne  in  mind: 

(1)  The  selection  of  the  data; 

(2)  The  selection  of  the  design. 

Circle  chart.  This  type  of  chart  is  used  extensively  for  popular 
presentation,  and  is  designed  to  exhibit  the  true  proportions  of  the 
component  parts  of  a  group  total.  It  is  adapted  to  such  purposes 
as  exhibiting  the  distribution  of  disbursements,  the  sources  of 
receipts,  and  the  allocation  of  appropriations  in  government 
finance. 

The  circle  with  sectors,  however,  is  not  so  desirable  a  form  of 
presentation  as  the  bar  chart,  described  in  later  paragraphs,  since  it 
does  not  possess  the  same  degree  of  flexibility.  It  is  impossible, 
for  instance,  in  a  profit  and  loss  analysis,  to  exhibit  a  loss.  More- 
over, it  does  not  always  permit  a  convenient  arrangement  of  cap- 
tions, which  must  sometimes  be  written  in  at  an  angle.  Another 
disadvantage  is  that  the  figures  are  not  easily  compared.  For 
these  reasons,  it  is  probably  best  to  limit  the  circle  chart  to  the 
illustration  of  facts  which  are  not  intended  to  be  compared  from 


266 


GRAPHS  AND  INDEX  NUMBERS 


period  to  period.     However,  the  sector  method  is  so  widely  used 
that  it  is  perhaps  better  understood  generally  than  any  other. 

The  circle  is  segmented  on  the  basis  of  100°,  not  360°,  as  geo- 
graphic circles  are  segmented.  This  is  because  the  chart  circle  is 
designed  to  exhibit  a  percentage  scale. 

Example 

Distribution  of  the  expense  dollar. 


Figure  8.     Circle  Chart. 

Comparison  of  circles.  Comparisons  in  magnitude  are  some- 
times made  by  presenting  circles  of  different  sizes.  The  objection 
to  this  method  is  the  resulting  confusion  in  the  mind  of  the  reader 
as  to  whether  area  or  diameter  is  used  as  the  basis  of  measurement. 

It  is  impossible  to  estimate  accurately  the  difference  between 
the  diameters  of  two  circles  by  merely  looking  at  them.  When 
comparative  diameters  are  being  estimated,  the  circles  themselves 
have  to  be  subordinated  in  the  mind  of  the  reader  while  the  diam- 
eters are  visualized.  Charts  were  devised  because  of  their  ease  of 
comprehension,  and  their  purpose  is  defeated  when  conflicting 


GRAPHS  AND  INDEX  NUMBERS  267 

metal  processes  are  involved.     For  this  reason,  circles  of  different 
sizes  should  never  be  used  for  comparative  purposes. 

The  same  criticism  applies  to  squares  and  cubes.  A  cube 
whose  edge  is  twice  that  of  another  will  possess  eight  times  the 
cubic  content  and  four  times  the  outside  area  of  the  smaller 
one,  and  unless  the  basis  of  measurement  is  carefully  explained,  the 
comparison  may  easily  be  misleading. 

Problems 

1.  Using  the  figures  in  the  following  condensed  operating  statement,  prepare 
a  circle  chart  showing  the  distribution  of  the  "sales  dollar." 

Sales        .                        .  $66,734.49 

Plant  Operating  F,xpenses  ..                        ....          .  21,464.91 

Payroll .  ..                          ...  18,055  22 

Taxes .                       1,055.07 

Depreciation                14,252.63 

Depletion  .  4,667.20 

Net  Profit         .    .        .                      .              7,239  46 

2.  Prepare   a   circle   chart   to   illustrate   the   following   accounts   receivable 
analysis. 

Current  to  60  days  old          ...  47  08% 

90  days  old .    .  11.69% 

120  days  old 6  62% 

4  to  6  months  old .            .  10.65% 

6  months  to  1  year  old  8  54% 

Over  1  year  old 15.42% 

3.  Prepare  a  circle  chart  to  illustrate  the  distribution  of  the  sales  dollar. 

Raw  materials ,          .  .   $  55 

Wages  and  salaries .      .  25 

Direct  taxes .  .07 

Selling,  advertising,  and  miscellaneous  expense  05 

Reinvestment  in  the  business .  .      .03 

Wear  and  tear  on  equipment ...      .02| 

Dividends 02£ 

Bar  chart.  The  bar  chart,  like  the  circle  chart,  is  designed  to 
exhibit  the  true  proportions  of  the  component  parts  of  a  group 
total. 

Bars  used  in  charting  may  consist  of  single  heavy  lines,  or  they 
may  be  widened  into  rectangles.  Three  ways  of  presenting  data 
by  means  of  bars  are  in  common  use : 

(1)  Comparisons  are  made  by  presenting  a  series  of  bars  of 
different  lengths,  each  bar  representing  a  different  magnitude  (see 
Figure  9). 

(2)  A  single  bar  is  subdivided  into   component  parts    (see 
Figure  10). 


268  GRAPHS  AND  INDEX  NUMBERS 

(3)  A  combination  of  (1)  and  (2)  may  be  employed  (see  Figure 
11). 

Where  color  is  not  used,  bars  or  parts  of  a  bar  may  be  differenti- 
ated by  crosshatching,  and  also  by  the  use  of  solid  black  and  white, 
1st  Year  -  $6,803,407  mmmmmmmmmmmmmmmmmm 


2nd  Year  -  7,008,564 
3rd  Year  -  7,602,939 
4th  Year  -  8,411,776 
5th  Year  -  10,122,473 
6th  Year  -  12,089,857 


Figure  9.     Bar  Chart  Showing  Sales  for  6-year  Period. 

Crosshatching  consists  of  the  fine  parallel  lines  drawn  across  the 
face  of  the  bar  at  various  angles,  and  sometimes  crossed  into  small 
rectangles. 

In  making  comparisons  by  means  of  parallel  bars,  it  is  essential 
that  the  bars  be  of  the  same  width,  in  order  that  measurement  by 
length  be  not  confused  with  that  of  area. 


^       s. 

AL.r,br~$J.<5u,uuu  

Cost  of  Sales 

$135,000 

Expenses, 

$35,000 

Profit, 

$  iu.ua 

Figure  10.     Bar  Chart  Showing  Net  Profit  on  Sales. 

Bars  may  be  placed  in  either  horizontal  or  vertical  position. 

In  the  bar  method  of  charting,  figures  may  be  placed  at  the 
*ide  or  in  the  bat  and  decimal  points  kept  in  line;  it  is  thus  easy  to 
foot  the  figures  representing  the  various  components  and  to  verify 
the  total  (Figure  11). 

Because  of  its  decimal  divisions,  a  millimeter  scale  is 
convenient  in  constructing  these  charts. 

Problems 

i.  Chart  the  following  data,  using  vertical  bars. 

Period  Amount 

t  40,000 

2  50,000 

3  60,000 

4  75,000 

5  80,000 

6  90,000 

7  95,000 

8  100,000 

9  105,000 
10       110,000 


GRAPHS  AND  INDEX  NUMBERS 


269 


2.  Chart  the  following  data,  using  the  single 
bar  subdivided  into  component  parts. 


Sales  

81,000,000 

Cost  of  Sales 

550,000 

General  Expenses 

200,000 

Selling  Expenses 

150,000 

Net  Profit 

100,000 

3.  Chart  the  following, 

using  horizontal  bars. 

Period 

Amount 

] 

2,931 

2 

9,052 

3 

13,541 

4 

16,403 

5 

20,919 

6 

28,063 

7 

36,365 

s 

41,393 

9 

49,404 

10 

56,615 

4.  Chait  the  following  similarly  to  Figure  12. 

CiUOWTH    IN    NUMliKH    OF    EMPLOYEES 
A'O. 

Year  Employees 


1st 
2nd 
3rd 
4th 
5th 
6th 


6,587 
6,893 
7,205 
7,581 
7,810 
8,104 


No. 

No. 

Male 

Female 

1,859 

4,728 

2,OS  1 

4,812 

2,270 

4,935 

2,317 

5,264 

2,168 

5,642 

2,278 

5,826 

Line  or  curve  chart.     The  Hue  or 

curve  chart  is  probably  adaptable  to 
a  greater  variety  of  uses  than  any  other 
type  of  graphic  presentation.  It  is  used 
particularly  to  exhibit  trends  and  fluc- 
tuations in  data,  the  abnormal  condi- 
tions being  shown  by  unusual  " peaks7' 
and  "valleys." 

On  line  charts,  the  scale  is  indicated 
by  vertical  and/or  horizontal  rulings. 
If  the  coordinate  type  of  ruling  is  used, 
the  lines  in  each  direction  are  spaced 
an  equal  distance  apart,  horizontal  lines 
marking  the  vertical  scale,  and  vertical 
lines  marking  the  horizontal  scale. 


EARNINGS  RETAINED  AND 

INVESTED  IN  BUSINESS 

STOCK  DIVIDEND  (RETAINED 

IN  THE  BUSINESS) 

GASH  DIVIDENDS 


TAXES 

DEPRECIATION 
INTEREST 


MATERIAL  AND  ALL 
OTHER  EXPENSES 


WAGES,  SALARIES 
AND  COMMISSIONS 


$580,252 

fOTiwj 

fMMifttf'OWtfw 

:%2W$& 

S?.  102,456  !: 

S  894  168 

^840.618^ 

115,440,450 

< 

$2W 

48,458 

^$50,035,903 

Figure  11. 


270 


GRAPHS  AND  INDEX  NUMBERS 


All  rectilinear  charts  have  two  axes — the  horizontal,  called 
the  z-axis,  and  the  vertical,  called  the  ?/-axis. 

Rules  for  coordinate  charts.  Custom  has  established  certain 
rules  governing  the  construction  of  coordinate  charts,  which  must 
he  observed  if  they  are  to  be  plotted  in  accordance  with  good 
usage. 

(1)  The  zero  line  should  always  appear,  or  attention  should  be 
specifically  called  to  its  omission. 


IstYr.        2nclYr.         3rd  Yr.        4th  Yr.         6th  Yr.        6th  Yr. 
Figure  12.     Bar  Chart  Showing  Annual  Gross  and  Net  Income. 

(2)  The  time  element  should  always  be  expressed  by  the  hori- 
zontal scale,  and  magnitude  by  the  vertical  scale. 

(3)  The  curves  should  be  sharply  distinguished  from  the  ruling. 

(4)  Figures  and  lettering  should  be  so  placed  that  they  are 
read  from  the  bottom  or  from  the  right-hand  side. 

(5)  Exact  data  should  be  inserted  at  the  top  of  the  chart,  the 
figures  in  each  case  appearing  immediately  above  the  correspond- 
ing point  on  the  curve. 

(6)  The  figures  of  the  vertical  scale  should  be  placed  on  the 
left.     In  wide  charts  they  may  be  repeated  on  the  right. 

(7)  The  horizontal  scale  should  read  from  left  to  right,  and  the 
vertical  scale  from  bottom  to  top. 


GRAPHS  AND  INDEX  NUMBERS 


271 


It  is  considered  good  practice  to  make  the  zero  line  heavier 
than  the  other  coordinate  rulings.  In  percentage  charts  the  100% 
line  is  also  accentuated  by  heavier  ruling. 

Example 

The  earnings  of  a  corporation  over  a  period  of  years  were  as  follows: 


1st  yea 
2nd  ye 
3rd  yet 
4th  yei 
5th  ye; 
6th  yei 
7th  yei 
8th  yet 
9th  yei 
10th  y< 

50 
45 
40 
86 
80 

1 

325 
20 
15 
10 
5 
n 

r     .         
ar  .     . 
ir 
ir  . 
vr 
ir 
ir 
ir 
ir 
^ar   

$39,202,000 
37,555,000 
28,621,000 
30,438,000 
28,693,000 
27,319,000 
28,358,000 
35,941,000 
31,772,000 
34,249,000 

— 

— 

— 

~ 





— 

^ 

^ 

— 

— 

\ 
\ 

\ 

/ 
/ 

\ 

X 

^^" 

\ 
\ 

A 

rera 

£C 

/ 
/ 

"• 

" 

\ 

-"' 

"-^ 

^ 

— 

— 

^, 

' 

_ 

— 



— 





— 









| 

YEARS 
Figure  13. 

NOTE:  Limited  space  does  not  permit  insertion  of  exact  figures  (Rule  5). 

Problems 

1.  Using  the  following  data,  prepare  a  line  chart  showing  the  corporation's 
earnings  and  the  dividends  paid  over  a  period  of  years. 


272  GRAPHS  AND  INDEX  NUMBERS 

Year  Net  Income  Dividends  Paid 

1st $28,154,431  $16,354,000 

2nd .                 35,422,514  10,360,632 

3rd  .  .                     40,129,417  16,369,400 

4th  .      2S,6S4,916  16,404,509 

5th  31,54S,606  17,47S,459 

6th  32,070,274  1S,209,281 

7th  30,61S,77X  20,639,196 

Sth  32,600, 1 50  20,662,854 

9th  44,552,482  20,662,854 

I  Oth  35,419,903  20,943,094 

11th  .      .                35,657,410  22,609,650 

2.*  From  the  information  given  in  the  following  table,  prepare  a  line  graph 
of  the  Bonded  Debt  Limit  and  the  Net  Bonded  Indebtedness  of  the  City  of  X 
for  the  15-year  period.  (Scale,  1  in.  =  $2,000,000.) 


Year 

1st 

2nd 

3rd 

4th 

5th 

6th 

7th 

8th 

Oth 

10th 

llth 

12th 

13th 

14th 

15th 

Logarithmic  chart.  The  common  logarithms  of  the 
table,  being  the  expressions  of  numbers  in  terms  of  the  power  of  10, 
are  particularly  adapted  to  the  presentation  of  percentage  relation- 
ships. This  fact  has  led  to  the  construction  of  a  chart  in  which 
percentage  relationships  are  revealed  by  a  comparison  of  different 
sets  of  data  plotted  in  terms  of  numerical  magnitude. 

The  logarithmic  chart  is  a  variation  of  the  rectilinear  type, 
The  ruling  differs  from  that  of  the  customary  coordinate  chart 
in  that  the  data  lines  representing  the  scale  are  not  evenly  spaced, 
but  conform  to  certain  proportions  expressed  by  the  first  ten 
numbers  in  a  table  of  common  logarithms. 

Figure  14  illustrates  the  method  of  laying  out  such  a  scale. 
The  first  column  of  figures  is  purely  for  drafting  purposes,  and 


Assessed 

Bonded 

Net  Bonded 

Valuation 

Debt  Limit 

Indebtedness 

$442,932,255 

$\  5,887,062 

•1:510,698,500 

460,548,763 

18,272,142 

9,321,050 

486,424,005 

20,804,104 

10,577,500 

496,342,170 

23,291,794 

11,101,500 

505,713,510 

23,919,607 

11,921,000 

521,239,125 

24,702,675 

14,730,750 

539,457,120 

25,491,759 

16,566,000 

574,020,559 

20,307,724 

16,534,750 

588,556,266 

27,289,865 

18,254,800 

075,011,540 

28,988,846 

22,030,250 

681,198,160 

30,588,436 

23,965,500 

677,070,755 

27,750,500 

24,800,000 

725,603,037 

29,033,300 

27,403,300 

755,229,851 

30,773,800 

25,023,500 

810,509,504 

33,974,550 

25,744,500 

*  O.  P.  A.,  Wisconsin. 


GRAPHS  AND  INDEX  NUMBERS 


273 


consists  of  the  first  ten  figures  of  a  logarithmic  table,  only  the  first 
two  decimal  digits  being  used  in  each  case. 

The  condensed  table  of  logarithms  of  numbers,  page  274,  will 
be  of  assistance  in  preparing  a  logarithmic  chart. 

In  the  vertical  scale,  each  horizontal  line  is  spaced  the  distance 
from  the  base  of  the  scale  which  represents  the  proportion  that 
its  scale  number  bears  to  100;  that  is,  the  first  line  above  the  base 
line  is  thirty  one-hundredths  of  the  total  height  of  the  scale. 


100          10 

05           9 

30           8 

S4            7 

77           C 

ea          5 

€0          4 

/' 

.2 

X                                                  0 

03*7                 d 

/ 

X 

1 

^ 

30            ^ 

/ 

/ 

/ 

X 

igita 

LO«8.                         ( 

P              ^ 

1 

.8 

2 

0              .4 

Y.  a 

3 

7          .6 

via 

4 
0      .6 

5 
9    .1 

6 
7   .J 

7 
4.f 

8     9 
0.951. 

Figure  14.     Logarithmic  Chart. 

the  next  line  is  forty-seven  one-hundredths,  and  so  on,  the  top 
line  representing  100.  A  ruler  having  a  100-millimeter  scale 
may  be  conveniently  used  for  making  these  horizontal  lines. 

The  second  vertical  column  represents  the  numerical  magnitude 
wale,  and  is  numbered  from  1  to  10,  or  some  multiple  thereof,  the 
ruled  lines  being  spaced  according  to  the  logarithms  of  these 
numbers.  The  result  is  that  when  data  are  plotted  on  such  a  chart 
in  terms  of  numerical  magnitude,  the  relationships  shown  between 
the  various  groups  of  data  plotted  will  be  correct  percentage 
relationships.  This  does  not  hold  true  where  data  are  plotted 
numerically  on  an  ordinary  coordinate  chart;  where  the  curves 


274  GRAPHS  AND  INDEX  NUMBERS 

represent  widely  differing  magnitudes,  an  attempted  comparison 
of  the  fluctuations  in  the  data  will  be  misleading.  In  order  that 
data  plotted  on  a  coordinate  chart  may  present  correct  percentage 
relationships,  the  lines  must  represent  a  percentage,  not  a  numeri- 
cal, scale. 

LOGARITHMS  OF  NUMBERS 

Num-      Loga-      Num-      Loga-      Num-      Loga-      Num-      Loga- 
ber         rithrn         bcr         rithrn         her         rithm         ber         rithm 


1 

0.00 

10 

1  00 

100 

2  00 

1,000 

3  00 

2 

0.30 

20 

1.30 

200 

2.30 

2,000 

3  30 

3 

0.47 

30 

1.47 

300 

2  47 

3,000 

3  47 

4 

0.60 

40 

1.60 

400 

2  60 

4,000 

3  60 

6 

0.69 

50 

1.69 

500 

2  69 

5,000 

3  69 

6 

0.77 

60 

1.77 

600 

2  77 

6,000 

3  77 

7 

0.84 

70 

1  84 

700 

2.84 

7,000 

3  84 

8 

0  90 

80 

1  90 

800 

2  90 

8,000 

3  90 

9 

0.95 

90 

1  95 

900 

2  95 

9,000 

3  95 

10,000       4.00 

In  comparing  on  a  logarithmic  scale  data  of  widely  differing 
magnitudes,  it  is  necessary  to  use  inoia  than  one  grouping  of 
logarithmic  rulings.  Each  such  grouping  is  called  a  cycle,  because 
it  represents  10,  or  some  multiple  thereof.  For  instance,  if  data 
represented  by  figures  in  the  hundreds  group  and  data  running 
into  the  thousands  were  to  be  compared,  it  would  be  necessary  to 
use  two  cycles;  if  the  figures  ran  into  the  ten-thousands  group,  it 
would  be  necessary  to  use  three  cycles. 

It  will  be  noticed  that  the  base  line  in  a  logarithmic  chart  is 
numbered  1,  instead  of  0.  This  is  because  in  the  tables  the  log  of 
1  is  .0.  Therefore,  in  a  logarithmic  chart  there  is  no  zero  line. 

To  illustrate  the  use  of  a  full  logarithmic  chart,  a  simple 
example  in  multiplication  may  be  cited.  Applying  the  principle 
that  in  multiplication  the  logarithm  of  the  product  of  two  numbers 
is  the  sum  of  the  logarithms  of  the  numbers,  the  addition  may  be 
performed  graphically  on  a  logarithmic  chart.  By  doubling  the 
distance  of  any  number  represented  by  a  digit,  the  square  of  the 
digit  is  obtained.  Thus,  the  distance  from  1  to  9  is  twice  the  dis- 
tance from  1  to  3,  because  9  is  the  square  of  3. 

Since  the  scales  of  both  axes  are  the  same,  lines  projected  at 
right  angles  from  identically  numbered  points  on  both  axes  will 
complete  a  square,  the  diagonal  of  which  is  45°,  to  the  right  of  any 
point  on  the  base  of  the  rr-axis;  for  instance,  any  horizontal  line 
which  intersects  this  diagonal  will,  if  projected  downward  from 
the  intersecting  point  to  the  base  line  of  the  x-axis,  and  at  right 


GRAPHS  AND  INDEX  NUMBERS 


275 


angles  thereto,  record  on  the  .r-axis  digit  scale  the  product  of  the 
two  numbers  representing  the  starting  points  of  the  diagonal  and 
the  horizontal  lines. 

Figure  14  illustrates  a  computation  of  this  kind,  and  affords 
a  mechanical  demonstration'of  the  logarithmic  principle  referred  to. 
A  diagonal  line  is  projected  upward  at  an  angle  of  45°  from  the 


10,000 
9,000 
8,000 
7.000 

6.000 
6,000 
4,000 

8,000 
2.000 


tooo 

900 
800 
700 
600 
600 

400 
800 

200 


100 


Jan. 


Z 


Z 


z 


Feb.  Mar. 

Figure  15. 


Apr. 
Ratio  Chart. 


May 


June 


digit  2  on  the  x-axis.  The  point  at  which  it  intersects  the  hori- 
zontal line  numbered  4  on  the  y-scale  is  directly  above  digit  8  on 
the  x-scale,  and  2  times  4  is  8.  Likewise,  the  sum  of  the  distances  1 
to  2  on  the  z-axis,  and  1  to  4  on  the  7/-axis,  will,  if  laid  off  by  a 
pair  of  dividers,  arrive  at  point  8  on  either  scale. 


276 


GRAPHS  AND  INDEX  NUMBERS 


Ratio  charts.  Figure  15  illustrates  a  two-cycle  chart.  Only 
the  horizontal  lines  are  ruled  in  accordance  with  the  logarithmic 
scale.  The  vertical  lines  are  evenly  spaced.  This  is  called  a 
semi-logarithmic  or  ratio-ruled  chart,  and  is  the  kind  most  com- 
monly used. 

The  use  of  the  full  logarithmic  chart,  with  the  ratio  ruling 
in  both  directions,  is  rare,  and  is  confined  chiefly  to  mathematical 
demonstrations. 

The  ratio  chart  has  its  limitations.  While  it  exhibits  correct 
percentage  relationships,  it  does  not  record  correct  numerical 
magnitudes,  and  it  should  bo  used  only  where  percentage  relation- 
ships are  desired.  But  because  relationships  between  fluctuating 
data  are  more  easily  grasped  in  terms  of  percentages  than  in  purely 
numerical  terms,  the  semi-logarithmic,  chart  has  a  wide  field  of 
usefulness. 

Where  the  percentage  of  increase  or  decrease  of  one  item,  such 
as  sales,  is  to  be  compared  with  the  percentage  of  increase  or 
decrease  of  some  other  item,  such  as  expenses,  the  result  is  best 
shown  in  the  ratio  or  semi-logarithmic;  chart. 


January 
February 
March 
.  April 

May 

Juno 


Example* 


dross  1'ro  fit 


Sales 

Cost 

(40',) 

Expense 

Net  Profit 

$  f>,400 

$  3,240 

*  2,100 

«  2,000 

S    160 

4,500 

2,700 

1,SOO 

1,500 

300 

8,000 

4.SOO 

3,200 

2,200 

1,000 

10,000 

('),()()() 

4,000 

2,500 

1,500 

7,000 

4,200 

2,SO() 

2,100 

700 

0,000 

3,600 

2,400 

1,500 

900 

$40,000 

$24,540 

SI  0,360 

$11,800 

$4,560 

Problems 

1.  Prepare  a  ratio  chart  to  illustrate  the  following 
statements : 


condensed   operating 


Sales  

1st  Year     2nd  Year     3rd  Year     4th  Year     5th  Year     6th  Year 

.   $61,960.29  $74,401.38  $80  598  00  $72  887  60  $79  647  14  $65  315  48 

Tost  of  Sales.  , 
Gross  Profit.  . 
Expenses  

.     27,745.59     32,967.89     48,935.10     41,660.83     48,945.54     33,414.53 
.     34,214.70     41,433.49     31,662.90     31,226.77     30,701.60     31,900.95 
.     31,152.64    33,30886    2864073    3058032    3034226    29  736  O9 

Net  Profit  .  . 

.       3,062.06      8,124.63      3,022.17         646.45         359.34      2,164.93 

*  Charted  in  Figure  15. 


GRAPHS  AND  INDEX  NUMBERS 


27T 


2.  Prepare   a    ratio   chart   to   illustrate   the   following   comparative   income 
accounts  of  a  public  utility  company: 


Power-Opr., 
Conducting; 

Mainte- 

Transporta- 

Gross 

nance  and 

tion,  and 

Fixed 

Net 

Yoar 

Earnings 

Renewals 

General 

Tnxos 

Charges 

Income 

1st  ^ 

$22,147^)74 

$37001,198 

$  8,027,973 

$17591,523 

$  8,827,988 

$    5(0,7081 

2nd 

23,282,408 

3,492,301 

9,097,001 

1,059,518 

8,901,120 

72,312 

3rd 

24,210,592 

3,030,088 

9,081,213 

1,000,230 

9,321,559 

538,490 

4th 

23,901.398 

3,r>9  1,209 

8,825,005 

1,808,951 

9,531,232 

201,311 

5th 

21,315,455 

3,017,318 

8,077,405 

1,783,510 

9,022,031 

581,501 

<>th. 

27,279,517 

4,091,928 

9,382,587 

1,812,511 

9,01  1,908 

2,377,553 

7th 

29,720,927 

4,459,039 

10,723,912 

2,100,709 

9,573,522 

2,803,085 

8th 

31,704,127 

4,755,004 

13,355,575 

2,128,8)9 

9,029,553 

1,534,810 

9th 

30,039,520 

4,955,124 

17,287,117 

2,315,750 

9,735,052 

1,715,877 

10th 

39,400,341 

5,905,409 

20,028,504 

2,00  1,253 

9,823,110 

382,005 

llth 

12,911,010 

8,500,400 

19,874,309 

2,798,821 

9,870,158 

1,807,292 

12th 

43,235,972 

8,500,  100 

20,407,117 

2,580.001 

9,853,177 

1,829,277 

131h 

45,552,031 

8,500,400 

22,  179,553 

2,095,708 

10,010,370 

1,800,000 

14th 

40,215,488 

8,500,  100 

22,078,890 

2,700,903 

10,104,921 

1,810,305 

f  Loss. 


3.  Prepare  a  ratio  chart  to  show  the  following  expenses: 

Salaiies 

Kent  ..... 

Ciedit  Losses  . 

Heat          ...  . 

Light  .    . 

Taxes  .    . 

Shipping  and  Receiving  . 

Depreciation       . 

Miscellaneous  Expenses  . 

Power  .....  .  .. 

Freight  . 

Delivery  Expense  .  , 

Insurance  .  .    . 

HINT.     Kearrangc  items  fioiu  highest  to  lowest  per  cent. 


3 
5 

25 

1  0 

5 

5 

1 


Index  Numbers 

Uses  of  index  numbers.  Current  newspapers,  magazines, 
bulletins,  and  books  contain  much  economic  information  expressed 
in  terms  of  index  numbers.  Following  are  examples  from  the 
Monthly  Review  of  Agricultural  and  Business  Conditions  issued  by 
the  Federal  Reserve  Bank: 

"From  June  1939  to  March  1942  prices  rose  30  per  cent,  according  to  the 
index  of  wholesale  prices  compiled  by  the  U.  8.  Bureau  of  Labor 


x/o 


GRAPHS  AND  INDEX  NUMBERS 


"  Industrial  production  reached  a  peak  in  February  (1945)  when  the  index 
stood  at  236  per  cent  of  the  1935-1939  average." 

"The  index  of  prices  paid  by  farmers  was  unchanged  at  173  for  the  fourth 
consecutive  month  according  to  data  from  the  Department  of  Agriculture." 
(July  30,  1945) 

Index  numbers  may  be  used  in  the  form  of  a  chart  similar  to 
the  following,  which  appeared  in  a  recent  issue  of  the  Chicago 
Tribune: 


INDEX 
1939-1 
225 

200 
175 
150 
125 

100 
75 

DO                   latest  Figure  for  August,  1945 

^. 

Pric 
b 

es  Recei 
y  Farmer 

^d 

/ 

s  — 

/ 

y. 

..-«••* 



••••* 

^^1 

..*"' 

N 

rices  Pai 
y  Farmer 

d 

b 

s 

1939    1940 


1942    1943    1944 


1945 


Index  numbers.  An  index  number  is  a  number  that  is  used  for 
measuring  trends  in  prices  or  in  other  movements  which  can  be 
quantitatively  expressed  by  means  of  statistical  data.  Among 
those  more  frequently  used  in  business  are  the  following: 

Production  indexes.  Automobile  production,  building  con- 
struction, electric  power  output,  steel  production  fponrningr  of 
capacity). 

Trade  indexes.  Carloadings,  department  store  sales,  check 
clearings,  inventories  (of  manufacturers,  wholesalers,  and  retailers), 
value  of  imports  and  exports. 

Financial  indexes.  Prices  of  stocks  and  bonds,  business  fail- 
ures, new  capital  issues,  commercial  bank  loans,  prices  of  basic 
commodities,  prices  of  agricultural  products,  foreign  exchange 
rates. 

Miscellaneous.  Wage  rates,  employment,  financial  situation  of 
the  government,  foreign  affairs. 

Most  of  the  indexes  listed  in  the  foregoing  are  expressed 
statistically ;  therefore,  their  trends  can  be  measured  mathematically. 

The  nature  of  the  particular  business  primarily  determines  the 
indexes  most  useful  to  it.  To  answer  the  question,  "How  is 
business?"  recourse  may  be  had  for  purposes  of  comparison  to 
some  index  of  general  business  conditions,  for  example,  to  the 
Federal  Reserve  index  of  industrial  production,  a  national  sum- 
mary charted  for  a  period  of  years :  / 


GRAPHS  AND  INDEX  NUMBERS 


279 


INDUSTRIAL  PRODUCTION 

PER  PHYSICAL  VOLUME  SEASONALLY  ADJUSTED,  PER 

CENT  1935-39 -loo  CENT 


IDU 

240 
220 
200 
180 
160 
140 
120 
100 
80 
60 

/\ 

i^ 

260 
240 
220 
200 
180 
160 
140 
120 
100 
80 
60 

r 

^  - 

- 

/ 

- 

- 

s 

- 

- 

s 

- 

- 

f 

- 

- 

y 

'  \ 

f 

** 

- 

\ 

,/ 

- 

1937  1938  1939  1940  1941  1942  1943 

1944 

or  possibly  to  indexes  somewhat  more  restrictive  as  to  locality  and 
type  of  business,  such  as  the  following  from  the  Ninth  Federal 
Reserve  District  Monthly  Review  of  July  30,  1945: 


/riONS-  1935  -1939  =  100 

,/  uric 

May 

,/  unc 

J  'unc 

1,94» 

1945 

1944 

1V43 

237 

201 

20S 

176 

224 

207 

201 

173 

I  S3 

172 

153 

146 

1  73 

163 

152 

137 

1  02 

150 

149 

143 

HO 

100 

102 

12S 

114 

120 

114 

109 

1  39 

203 

143 

145 

1S5 

ISO 

177 

17S 

134 

143 

151 

229 

233 

228 

Bank  Debits— 93  Cities  .     .      . 
Hank  Debits — Fanning  Centers 

City  Dept.  Store  Sales       

City  Dept.  Store  Stocks 
Country  Dept.  Store  Sales 
Country  Lumber  Sales        ... 
Miscellaneous  Carloadings 
Total  Carloadings  (excl.  Misc.) 
Farm  Prices — Minn,  (unadj.) 
Employment — Minn,  (nnadj.  1936  =  100) 
Minnesota  Payrolls  (unadj.  1936  -  100) 

Economic  position  of  agriculture.  Agriculture  creates  a  stream 
of  income  that  exceeds  that  of  any  other  industry.  This  income 
flows  through  various  channels  into  the  industrial  and  commercial 
life  of  the  nation.  Probably  as  many  or  more  people  are  engaged 
in  the  processing  and  handling  of  agricultural  commodities  from 
the  producer  to  the  consumer  than  are  engaged  in  the  actual 
production  processes.  Since  agriculture  and  other  industries  are 
customers  of  one  another,  any  enhancement  of  the  buying  power' 
of  one  operates  to  the  advantage  of  the  other.  With  this  back- 
ground, the  industry  of  agriculture  is  chosen  to  furnish  much  of  the 
material  in  this  chapter. 

Suppose  we  desire  to  compare  the  price  of  wheat  for  different 
years.  The  July  15,  1937-1941  average  price  of  wheat  was 


280  GRAPHS  AND  INDEX  NUMBERS 

73  cents  a  bushel,  and  on  July  15,  1945,  the  price  was  $1.48  a 
bushel.  We  can  say  that  wheat  has  gone  up  75  cents  a  bushel, 
but  a  better  comparison  is  to  show  what  per  cent  the  1945  price  is 
of  the  1937-1941  average. 

1.48  -1-  .73  =  2.03,  approximately 
2.03  X  100  -  203% 

Thus  the  1945  price  of  wheat  is  203%  of  the  1937  1941  average. 
Expressed  as  an  index  number,  it  is  203  when  the  base  is  t41937- 
1941  average  price  equals  100." 

Construction  of  index  numbers.  In  the  study  of  price  fluctua- 
tions, the  first  thing  to  determine  is  the  original  base  price.  This 
may  be:  (a)  the  price  of  a  commodity  on  a  certain  date;  (/>)  the 
average  price  of  a  commodity  during  a  certain  year;  or  (r)  the 
average  price  of  a  commodity  during  several  years.  Whichever 
is  used,  that  price  is  assigned  a  numerical  value  of  100. 

Other  index  numbers  for  following  years  or  periods  are  obtained 
by  multiplying  the  price  for  the  years  or  periods  by  100  and  divid- 
ing the  product  by  the  original  base  price.  Thus,  the  index 
numbers  of  the  prices  paid  to  Minnesota  wheat  growers  from  June 
15,  1939  to  June  15,  1944,  were  as  follows: 


PRICE   RECEIVED  BY  MINNESOTA  FARMERS 
(Fifteenth-of-Month  Comparison) 

Arrr.  Price         Index  \  'umber  or 

All  Wheat  (hu.)  per  Bushel  Jtelutire  Price 

1930-1939  average  .........  S     79  100 

1  940     .....  .      .  75  95 

J941  .......  .  S5  10S 

1942...  1   01  12S 

1943  .....  I   27  161 

1944  (0  mos.).  1   48  1X7 

Index  numbers  or  relative  prices  may  be  found  for  all  farm 
products  for  which  average  prices  are  known.  The  base  period 
taken  for  making  calculations  is  arbitrarily  selected,  depending  on 
the  period  for  which  comparisons  are  wanted. 

Problems 

Given  the  following  data,  compute  the  index  numbers: 
1.  Corn  (bu.): 

1930-1939  average          ..............................   $    .52 

1940  ......................  .....  48 

1941  .............  ..............         53 

1942  .........................          68 

1943  .........................................  90 

1944  (6  mos.)   ...................................     1.01 


GRAPHS  AND  INDEX  NUMBERS  281 

2.  Potatoes  (bu.): 

1930-1939  avenge     $     50 

1940     ...  ...      .         50 

1941 .  4S 

1942 ....         93 

1943     ...  .  ..1.27 

1944(6  mos.)  ..      1.12 

3.  Hogs  (lOOllxs.): 

1930-1939  average  $  6  78 

1940  .  5  20 

1941  .  9  04 

1942  13   10 

1943  13  70 

1944  (6  mos.)  .  .    .  ...      13  25 

4.  If  the  average  farm  price  of  A\lieat  used  as  the  base  was  S,5  cents  a  bushel 
(as  in  1941),  what  is  the  index  number  for  1944,  when  the  price  was  S1.4S  cents 
a  bushel? 

Composite  price  indexes.  It  is  commonly  desired  to  express 
the  price  of  a  group  of  commodities  or  of  all  commodities  in  a  single 
number  or  series.  Thus,  we  say  that  the  index  of  prices  received 
by  farmers  in  mid- April  (1945)  was  203,  calculated  from  the 
average  base  price  of  1910  1914.  The  index  203  was  obtained  by 
taking  the  average  price  for  all  farm  products  for  each  of  the  two 
periods  of  comparison.  At  the  same  time,  the  prices  paid  by 
farmers  for  things  used  in  farm  production  and  family  maintenance, 
including  interest  and  taxes,  was  173,  calculated  from  the  average 
base  price  of  1910  1914.  The  ratio  of  the  indexes  of  prices 
received  to  prices  paid  is  the  so-called  parity  ratio  established  by 
law  which  designates  the  years  1910  1914  as  the  base  period.  This 
ratio  averaged  100  in  the  base  years.  It  was  117  in  mid-April 
(1945):  203  -r  173  -  1.17,  and  1.17  X  100  -  117.  At  the  bottom 
of  the  depression  in  1932,  the  parity  ratio  was  01.  The  term 
parity,  as  applied  to  the  price  of  an  agricultural  commodity,  is  that 
price  which  will  give  to  the  commodity  a  purchasing  power  equiva- 
lent to  the  average  purchasing  power  of  the  commodity  in  the  base 
period,  1910-1914. 

Weighted  index  numbers.  An  index  number  of  prices  which 
will  satisfactorily  measure  changes  in  the  price  level  must  include  a 
considerable  number  of  representative  commodities,  and  these 
commodities  must  be  weighted  in  accordance  with  their  importance 
in  trade  and  industry.  The  number  of  items  may  be  30,  or  the 
number  may  be  500  or  even  more.  For  purposes  of  illustration,  we 
shall  use  three  principal  agricultural  commodities:  wheat,  corn- 
and  cotton. 


282  GRAPHS  AND  INDEX  NUMBERS 

Suppose  we  desire  to  compute  the  average  rise  in  the  price  of  ths 
three  commodities  from  1939  to  1942,  making  due  allowance  for 
the  total  quantity  of  each  commodity  produced,  so  that  the  index 
of  prices  found  will  give  us  information  on  the  increase  in  buying 
power  accruing  to  the  producers  as  a  result  of  the  rise  in  prices. 
We  obtain  the  production  for  the  base  period  (1939)  and  the 
average  prices  for  the  years  1939  and  1942  from  the  Yearbook  of  the 
Department  of  Agriculture  for  1943  and  set  up  the  material  as 
follows: 

Wheat  (lorn  Cotton 

Total 


Wheat 

(bu.) 

(lorn 
(bu.) 

Cotton 

(500-lb.  bale  a) 

Total  production  in 

1939  .    . 

741,180 

,000 

2,5X0,912 

,000 

1 

1 

,817 

,000 

Average     price 

in 

1939 

7S/- 

a 

bu. 

50 

*t 

al 

>u. 

S45 

4 

5  a  1 

>ale 

Average     price 

in 

1942. 

SI  26 

a 

bu. 

85 

5<£  a  bu. 

$94 

0 

5  a  bale 

Value  at  1939  prices  $57N,  120,400  $1,405,958,010  $    537,082,050  $2,581,101,000 
Value  at  1942  prices  $933,880,800  $2,206,079,760  $1,118,509,050  $4,259,075,610 

Price  index:  $4,259,075,010  -r-  $2,581,101,000  =  1.05 

1.05  X  10()'(',  =  105% 
Average  rise  in  price:  165%  —  100%  =  65% 

Explanation.  The  values  of  the  commodities  produced  in  1939  were  calcu- 
lated at  the  average  prices  in  both  1939  and  1942,  and  the  total  value  of  all  three 
for  each  of  the  two  years  was  found.  The  aggregate  value  for  1942  was  then 
divided  by  the  aggregate  value  for  1939.  This  method  of  constructing  an  index 
number  of  prices  is  recommended  because  it  does  not  require  the  statistics  of 
current  production.  (It  is  frequently  impossible  to  obtain  such  statistics 
promptly.) 

The  calculation  procedure  of  the  foregoing  example  may  he 
stated  in  terms  of  a  formula  for  weighted  aggregative  price  index 
as: 


where  : 

p  represents  the  price  of  a  commodity; 

q  represents  the  quantity  of  a  commodity; 

o  represents  the  base  period,  the  period  from  which  the  price  changes  are 

measured  ; 

n  represents  the  given  period,  the  period  being  compared  with  the  base; 
2)  is  the  symbol  of  summation,  or  addition. 

In  the  illustration  for  1942,  the  index  number  formula  is: 


P    —  .    ^'^21939 


There  is  no  short  cut.     The  q0  in  the  numerator  and  the  q0  in  the 
denominator  may  not  be  canceled.     The  quantity  of  each  com- 


GRAPHS  AND  INDEX  NUMBERS  283 

modity  included  in  the  index  must  be  multiplied  by  its  respective 
price  in  both  the  base  year  and  the  given  year  and  the  several 
products  added,  as  indicated  in  the  formula  and  illustrated  in  the 
example. 

.    Problems 

1.  Corn  and  barley  are  two  principal  feed  grains  used  in  fattening  hogs  for 
market.     From  the  following  figures  calculate  the  weighted  index  for  1942,  using 
1939  as  the  base  year. 

Hogs  (Ibs.)  Corn  (bu.)  Burlcy  (bu.) 

Total  production  in  1939   17,081,824,000  2,580,912,000  278,163,000 

Average  price  in  1939  (cents) 7  74  56  8                  40  5 

Average  price  in  1942  (cents) 13  04  85  5                  59  4 

2.  Calculate  the  individual  price  indexes  for  hogs,  corn,  and  barley.     Did  the 
price  of  hogs  keep  pace  with  the  price  of  corn?     How  about  the  price  of  hogs 
and  the  price  of  barley?     If  both  feeds  were  available,  which  would  be  the  more 
profitable  to  use,  assuming  that  each  has  the  same  feed  value  for  pork  production? 

3.  Find  the  weighted  index  for  the  three  following  fresh  fruits  sold  in  large 
quantities  for  home  canning. 

Apples  (bu.)  Peaches  (bu.)  Pears  (bu.) 

Total  production  in  1939  .    139,379,000  64,222,000    29,279,000 

Average  price  in  1939  ....         $   .64  $     S2             $     70 

Average  price  in  1 942  ....        $  I  38  $1  49            $  1 . 57 

4.  In  the  following  tabulation  are  given  the  index  numbers  for  the  average 
price  paid  by  farmers  and  the  average  price  received  by  farmers  for  the  years 
1932  to    1942,   inclusive,   based  on    1910-1914  averages,  the  years  chosen   for 
computation  of  parity  prices,  at  which  time  parity  was  100. 

Prices  Paid      Prices  Received 

1932  .    124          62 

1933  120          81 

1934  .     .    129          103 

1935  130  107 

1936  128  125 

1937  134  105 

1938  127  93 

1939  125  96 

1940  126  103 

1941  133  J42 

1942  .  .    151         175 

Calculate  the  parity  price  indexes  for  the  years  1935,  1940,  and  1942. 

Farm  evaluation  on  the  basis  of  crop  production  index.     The 

farm  owner,  the  banker  or  financial  agent  making  farm  loans,  arid 
other  interested  persons  may  partially  evaluate  a  farm  in  terms  of 
crop  production.  Other  factors,  such  as  building  improvements, 
fencing,  drainage,  location  relative  to  roads,  rural  electrification 
line,  and  so  forth,  are  also  to  be  considered  in  fixing  the  total  value. 
The  crop  production  index  is  a  comparison  of  the  yield  per  acre 


284  GRAPHS  AND  INDEX  NUMBERS 

of  all  crops  on  a  given  farm  with  the  average  yield  per  acre  of  all 
crops  on  a  number  of  farms  in  the  same  locality  —  community, 
township,  or  county.  A  crop  production  index  of  92  indicates  that. 
the  yield  of  crops  on  a  farm  with  this  index  is  92%  of  the  average 
for  the  locality;  a  farm  with  a  crop  production  index  of  105  has  a 
yield  5%  greater  than  the  average. 

Computation  of  the  crop  production  index.     Assume  that  a  farm 
has  a  cropping  system  as  follows  : 

50  acres  of  corn  yielding  40  bushels  per  acre 
50  acres  of  oats  yielding  35  bushels  per  acre 
50  acres  of  alfalfa  yielding  2^  tons  per  acre 

and  that  the  average  yields  for  the  county  in  which  this  farm  is 
located  are  : 

Corn  .  .          ...........  30  bushels  per  acre 

Oats         .  .  .  .......  .  25  bushels  per  acre 

Alfalfa     ............................     3  tons  per  acre 

The  crop  production  index  is  computed  as  follows: 

50  X  40  =  2,000,  the  number  of  bushels  of  corn  produced 
50  X  35  =  1,750,  the  number  of  bushels  of  oats  produced 
50  X  2  J  =      125,  the  number  of  tons  of  alfalfa  produced 
150  acres 

Next,  find  how  many  acres  would  be  required  to  produce  each 
crop  using  the  county  averages. 

2,000  bu.  of  corn  -r  30  1m.  per  acre  would  require         66^  acres  for  corn 
1,750  bu.  of  oats  -f-  25  bu.  per  acre  would  require          70    acres  for  oats 
125  tons  of  alfalfa  -r-  3  tons  per  acre  would  require    41  £  acres 

acres  required 


Since  178^  acres  would  be  needed  to  produce  what  this  farm  produced,  on  the 
basis  of  county  averages,  the  crop  production  index  is  118.9. 

178i 

Irn    X  10°  =   118'9 
loO 

Problem 

Ten  acres  of  a  160-acre  farm  were  used  for  farmstead  and  pasture,  and  the 
150  acres  in  cultivation  produced  the  following  crops,  as  compared  with  the 
county  average: 

Number  of       Yield  per        County  Average 
Crop  Acres  Acre  Yield  per  Acre 

Corn   ..............  40  45  bu.  42^  bu. 

Oats  ...............          40  50  bu.  48  bu. 

Alfalfa  .............  30  2  tons  2.^  tons 

Wheat  ..............  40  25  bu.  20  bu. 

150 
Compute  the  crop  production  index  for  this  farm. 


CHAPTER  28 
Progression 

Definition.  A  progression  is  a  series  of  numbers,  each  term  of 
which  is  obtained  from  the  preceding  or  following  term  by  a  fixed 
law;  as,  2,  4,  6,  8,  and  so  forth;  or,  2,  4,  8,  16,  and  so  forth. 

Increasing  series.  A  progression,  each  term  of  which  is 
greater  than  the  preceding  term,  is  known  as  an  increasing  or 
ascending  series. 

Decreasing  series.  A  progression,  each  term  of  which  is  less 
than  the  preceding  term,  is  known  as  a  decreasing  or  descending 
series. 

Arithmetical  progression.  When  each  term  of  a  progression 
differs  from  the  preceding  or  following  term  by  a  common  differ- 
ence, the  progression  is  said  to  be  arithmetical. 

Symbols.  The  five  elements  of  an  arithmetical  progression 
are  represented  by  certain  well-established  symbols: 

Number  of  terms  ...                      .  .  .  .  n 

First  term  .           .  .  a 

Last  term  .                                               .  I 

Common  difference  .  .  d 

Sum  of  the  terms  .  s 

Relation  of  elements.  The  five  elements  whose  symbols  are 
indicated  above  are  so  related  that  if  any  three  of  them  are  given, 
the  remaining  two  may  be  found. 

Increasing  Series 

The  formulas  used  in  connection  with  increasing  series,  and  the 
methods  of  solution,  are  shown  below;  in  each  case  the  values  of  the 
terms  have  been  taken  as  follows: 

Number  of  terms                                 .  12 

First  term                                                     ...                            .2 
Last  term  35 

Common  difference  3 

Sum 222 

To  find  the  number  of  terms. 

Algebraic  Formula  Arithmetical  Substitution 

I  —  a  35  —  2 

h  1  =  H.  '-    0 h  1  =  number  of  terms. 

d  .5 

985 


286  PROGRESSION 

Solution 

Simplifying  the  numerator:  35  —  2  =  33 
Dividing  by  denominator:  33  -r-  3  =  11 
Adding:  11  +  1  =  12,  the  number  of  terms 

To  find  the  first  term. 

Algebraic  Formula  Arithmetical  Substitution 

I  -  (n  -  \)d  =  a.  35  -  (12  -  1)3  =  first  term. 

Solution 

Removing  parentheses:  12  —  1  =  11 
Multiplying  by  3:  1 1  X  3  =  33 

Subtracting:  35  —  33  =  2,  the  first  term 

To  find  the  last  term. 

Algebraic  Formula  Arithmetical  Substitution 

a  +  (n  -  l)d  =  1.  2  +  (12  -  1)3  =  last  term, 

Solution 

Removing  parentheses:     12  —  1  =  11 
Multiplying  by  3:  11  X  3  =  33 

Adding:  2  +  33  =  35,  the  last  term 

To  find  the  common  difference. 

Algebraic  Formula  Arithmetical  Substitution 

I-  a  35  -  2  .... 
=  a.                                     -— —  -   =  common  difference. 

Solution 

Simplifying  the  numerator:  35  —  2  =  33 
Simplifying  the  denominator:  12  —  1  =  11 
Dividing:  33  -7-11  =3,  the  common  difference 

To  find  the  sum. 

Algebraic  Formula  Arithmetical  Substitution 

\  (a  +  1)  -  8.  ~  (2  +  35)  =  sum. 

Solution 
Adding  the  terms  in  parentheses:  2  +  35  =  37 

12 
Multiplying  by  the  fraction:        37  X  —  =  222,  the  sum 

A 

Decreasing  Series 

The  decreasing  series  will  be  illustrated  in  the  same  manner  a*» 
the  increasing  series,  with  the  values  of  the  terms  taken  as  follows: 


PROGRESSION  287 

Number  of  terms 8 

First  term           7 

Last  term                    —21 

Common  difference          .      .  .             —4 

Sum     -56 

To  find  the  number  of  terms. 

Algebraic  Formula  Arithmetical  Substitution 

J-a,!  (-21)  -7,    ,  .         f. 

1_  i  =  n<  h  1  =  number  of  terms. 

a  —4 

Solution 

Simplifying  the  numerator:         (  —  21)  —  7  =  —28 

Dividing  the  denominator:   (—28)  -j-  (—4)  =  7 

Adding:  7+1=8,  the  number  of  terms 

To  find  the  first  term. 

Algebraic  Formula  Arithmetical  Substitution 

I  -  (n  -  })d  =  a.  -21  -  [(8  -  l)(-4)]  =  first  term. 

Solution 

Removing  the  parentheses:  8  —  1=7 

Multiplying:  7  X  (-4)  =  -28 

Simplifying:  (-21)  -  (-28)  =  (-21)  +28 

And:  -21  +  28  =  7 

To  find  the  last  term. 

Algebraic  Formula  Arithmetical  Substitution 

a  +  (n  -  \)d  =  I.  7  +  1(8  -  l)(-4)]  =  last  term. 

Solution 

Removing  parentheses:  8—1=7 

Multiplying  by  -4:  7  X  (-4)  =  -28 

Subtracting:  7  -  28  =  -21 

To  find  the  common  difference. 

Algebraic  Formula  Arithmetical  Substitution 

I- a  (-21)  -  7 

— — -  =  a.  — - — — —  =  common  difference, 

n  —  1  o  —  1 

Solution 

Simplifying  the  numerator:       (-21)  -  7  =  -28 
Simplifying  the  denominator:  8  —  1  =  7 

Dividing:  -28  ^  +7  =  -4 

To  find  the  sum. 

Algebraic  Formula  Arithmetical  Substitution 

I  (a  +  I)  =  s.  I    7  +  (-21)     =  sum. 

6  A  \  \ 


288  PROGRESSION 

Solution 

o 

Simplifying  the  fraction:  9  =  ^ 

Adding:  7  +  (-21)  =  -14 

Multiplying:  4  X  (-14)  -  -56 

Problems 

1.  Given  a  —  2,  n  =  6,  /  =  12;  find  d  and  s. 

2.  Find  the  sum  of  all  the  even  numbers  from  10  to  SO  inclusive. 

3.  The  first  term  of  a  progression  is  6,  the  number  ot  terms  is  15,  the  common 
difference  is  7;  find  the  last  term  and  the  sum. 

4.  Given  n  =  12,  /  =  — 17,  s  =     -72,  find  d  and  a. 

5.  The  first  term  is  6,  the  last  term  is  1S1,  and  the  common  difference  is  7; 
find  the  number  of  terms. 

6.  Given  I  =  57,  n  =  23,  a  =   —9;  find  d  and  s. 

7.  A  building  is  to  be  leased  for  a  term  of  21  years.     The  first  yeai's  rental 
is  to  be  $10,000.00,  equal  increases  in  rent  are  to  be  made  each  year,  and  the 
rental  for  the  twenty-first  year  is  to  be  $30,000.00.     Find:  (n)  the  difference  in 
each  year's  rental;  (b)  the  total  rental  that  will  be  paid  during  the  period  of 
21  years. 

8.  A  deposited  $25.00  in  his  savings  account  on  January  1,  and  on  the  first 
of  each  month  thereafter  deposited  $5.00  more  than  the  previous  month.     How 
many  dollars  did  he  deposit  December   1,  and  what  was  the  amount  of  the 
accumulated  deposits?     Do  not  take  interest  into  consideration  in  solving  this 
problem. 

9.  A  punch  board  has  50  numbers  in  each  section  (numbers  1  to  50).     A 
person  pays  the  amount  of  the  number  punched.     If  the  board  has  four  sections, 
what  will  be  the  amount  of  revenue  derived  from  the  board? 

10.  A  man  invests  his  savings  in  the  shares  of  a  building  and  loan  association, 
depositing  $240.00  the  first  year.     At  the  beginning  of  the  second  year  he  is 
credited  with  $16.80  interest,  and  deposits  $223.20.     At  the  beginning  of  the 
third  year  he  is  credited  with  $33.00  interest,  and  deposits  $206.40.     What  is 
his  credit  at  the  end  of  10  years,  and  how  much  cash  has  he  paid  in? 

11.  A  bond  issue  of  $40,000.00  bearing  interest  at  4%  is  to  be  retired  in 
10  equal  annual  installments.     What  amount  of  interest  will  be  paid  during  the 
life  of  the  issue? 

12.  An  employee  started  work  for  a  company  at  $1,200.00  for  the  first  year, 
with  a  guaranteed  yearly  increase  of  $100.00.     What  was  his  salary  12  years 
later?     How  much  had  the  company  paid  him  in  the  course  of  the  12  years? 

Geometrical  Progression 

A  geometrical  progression  is  one  in  which  any  term  after  the 
first  is  obtained  by  multiplying  the  preceding  term  by  a  fixed 
number  known  as  the  ratio. 


PROGRESSION  289 

Elements.  In  a  geometrical  progression,  there  are  five  ele- 
ments so  related  that  any  three  being  given,  the  others  may  be 
found.  These  five  elements,  together  with  their  symbols,  are: 

Number  of  terms n 

First  term      a 

Last  term      I 

Sum  of  the  series s 

Ratio        r 

Increasing  series.  As  in  an  arithmetical  progression,  the 
formulas  and  solutions  in  a  geometrical  progression  are  based  on 
one  set  of  terms,  and  the  solutions  are  given  as  though  the  required 
term  in  the  example  in  question  were  lacking.  In  some  cases,  two 
different  formulas  may  be  used. 

Number  of  terms     .  .    .  6 

First  term ...                              3 

Last  term             .  .             96 

Sum  of  terms         .  .                                         .                                    189 

Ratio  of  increase.  . .                    ...                                                  2 

To  find  the  first  term. 

Algebraic  Formula  Arithmetical  Substitution 

1  %         r    ff 

(r)«-i  =  «•  (2)°  1  =  firsttmn- 

Solution 

Simplifying  the  exponent:      (2)6"1  =  (2Vr> 

Finding  the  power:  (2)5  =  32 

Dividing:  90  -r-  32  =  3,  the  first  term 

To  find  the  last  term. 

Algebraic  Formula  Arithmetical  Substitution 

a(r)"-1  -  1.  3(2)c-1  =  last  term. 

Solution 

Simplifying  the  exponent:     (2)6"1  =  (2^ 

Finding  the  power:  (2)5  =  32 

Simplifying:  3  X  32  =  96,  the  last  term 

To  find  the  sum. 

Algebraic  Formula  Arithmetical  Substitution 

IT  -  a  (96  X  2)  -  3 

—l    =s.  -2-—- 

Solution 

Simplifying:  (96  X  2)  =  192 

Subtracting:  192  -  3  =  ISO 

Simplifying  the  denominator:  2  —  1  =  1 
Dividing:  189  -M  =  189,  the  sum 


290 


PROGRESSION 


To  find  the  ratio. 

Algebraic  Formula 
'l 


A  rithmetical  Substitution 


. 
=  ratio. 


Simplifying  the  exponent: 
Simplifying  the  fraction: 
Extracting  the  5th  root: 

Decreasing  series. 

Number  of  terms. 

First  term 

Last  term 

Ratio  of  decrease 
Sum  of  terms 


6  - 

96  -r- 

5 


Solution 

1  =  5 

3  =  32 

>2  =  2,  the  ratio 


6 

96 
3 

-        i 

.    189 


To  find  the  first  term. 

Algebraic  Formula 
I 


A  rithmetical  Substitution 
3 


(i)6"1 


.   =  first  term. 


Simplifying  the  denominator:  (i)6~ 
Dividing:  3  -f-  ^ 

To  find  the  last  term. 

Algebraic  Formula 
a(r)»-'  =  L 

Simplifying  the  exponents:     6  —  1 
Finding  the  power:  (£)5 

Multiplying:  96  X  A 

To  find  the  sum. 

Algebraic  Formula 
Ir  -  a 


Solution 

i  =  -Jr 


r  -  1 


=  s. 


=  96,  the  first  term 


A  rith  metical  Substitution 

96(i)b~l  =  last  term. 
Solution 

=  5 

=  A 

=  3,  the  last  term 


Arithmetical  Substitution 
(3  X  i)  -  96 


-  1 


=  sum. 


Multiplying: 

Subtracting: 

Simplifying  the  denominator: 

Dividing: 

To  find  the  ratio. 

Algebraic  Formula 


3  X  i  =  f 
f-  96  =   - 

-J-  —  l  =   —  ^ 
-j  --  ^  =  189,  the  sum 


Arithmetical  Substitution 

'"'/I 

"V96  " 


PROGRESSION  291 

Solution 

Simplifying  the  fraction:  <nr  =  ^2" 

Simplifying  the  radical:  \/~&?  =  i>  the  ratio 

Progression  problems  solved  by  the  use  of  logarithms.     The 

use  of  logarithms  replaces  laborious  calculations  in  solving  prob- 
lems in  progression. 

Example 

What  is  the  average  yearly  rate  of  increase  if  $100.00  placed  at  interest  for 
10  years  produces  $179.08? 

Algebraic  Formula  Arithmetical  Substitution 

n\    ~Vd^^nd~  __  _  10/ 179.08  _ 

K  ~  \Vaiue  at  beginning       *'  Kate  "  \  lOCUX) 

Solution 

179.0S  -4-  100.00  -  1.790S 

log  1.790S  =  0.253047 
log  0.253047  -r-  10  =  log  0.025305 

In  the  table  of  logarithms,  it  is  found  that  log  0.025305  stands  for  1.00. 

1.06  -  1.00  =  .06,  or  6% 

Problems 

1.  A  machine  costing  $27,500.00  is  found  to  be  worth  only  $2,750.00  at  the 
end  of  12  years.     Find  the  fixed  percentage  of  diminishing  value. 

2.  A  building  cost  $80,000.00.     At  the  end  of  each  year  the  owners  deducted 
10  fo  from  its  carrying  value  as  estimated  at  the  beginning  of  the  year.     What 
is  the  estimated  value  at  the  end  of  10  years?     NOTE:     The  value  at  the  end 
of  the   10th  year  is  the  value  at  the  beginning  of  the   llth  year;  therefore: 
Value  -  $80,000  X  (.9)11-1. 

3.  An  asset  that  cost  $15,000.00  has  been  written  down  3%  of  the  decreasing 
balance  each  year  for  10  years.     At  the  end  of  the  10th  year,  what  is  its  value 
as  shown  by  the  books? 

4.  If  the  population  of  a  city  increases  in  5  years  from  150,000  to  175,000, 
find  the  average  rate  of  yearly  increase. 

5.  If  a  savings  bank  pays  5%  compounded  annually,  what  will  be  the  amount 
of  $200.00  at  the  end  of  6  years? 

6.  There  are  seven  terms  in  a  geometric  progression  of  which  the  first  term 
is  2  and  the  last  term  is  1.458.     Find  r  and  5. 

7.  Find  a  and  s  in  a  geometric  progression  where  r  =  2,  a  =  9,  and  I  =  256. 

8.  Find  r  and  s  in  a  geometric  progression  where  a  =  1 7,  I  =  459,  and  n  =  4. 

9.  If  the  bacteria  in  milk  double  every  two  hours,  how  many  times  will  the 
number  be  multiplied  in  24  hours? 


CHAPTER  29 
Foreign  Exchange 

Foreign  trade.  The  foreign  trade  connections  of  American 
concerns  make  it  necessary  for  the  accountant  to  be  acquainted 
with  the  basic  principles  of  exchange  values. 

Rate  of  exchange.  Theoretically,  the  rate  of  exchange  between 
any  two  countries  is  the  ratio  between  the  values  of  the  amounts 
of  metal  in  their  standard  monetary  units.  This  theoretical  rate 
is  commonly  known  as  "par  of  exchange/'  but  because  of  the  many 
economic  factors  in  foreign  business,  another  rate,  the  "current 
rate/'  is  usually  used. 

Par  of  exchange.  A  country  that  stands  ready  to  redeem  all  of 
its  obligations  in  gold,  upon  demand  and  without  restriction,  is  said 
to  be  "on  the  gold  standard."  Conditions  at  this  writing  (April 
1946)  are  chaotic,  and  all  countries  are  "off  the  gold  standard." 
However,  coinages  of  world  monetary  systems  are  based  on  actual 
or  theoretical  monetary  units  containing  gold  or  silver  of  a  legally 
established  weight  and  fineness.  The  mint  par  rate  of  exchange 
between  any  two  countries  is  the  ratio  between  the  amounts  of 
gold,  if  the  countries  are  on  a  gold  basis,  or  of  silver,  if  they  are  on  a 
silver  basis,  contained  in  their  standard  monetary  units.  If  one 
country  is  on  a  gold  basis  and  another  is  on  a  silver  basis,  reference 
must  be  made  to  the  market  prices  of  the  two  kinds  of  bullion. 
If  a  foreign  coin  contains  516.4058  grains  of  fine  silver,  and  the 
market  price  of  silver  in  terms  of  gold  in  this  country  is  60  cents  for 
5  ounces  (480  grains),  the  rate  of  exchange  is:  (516.4058  -v-  480) 
X  .60  =  .646.  The  silver  par  of  exchange  is  not  fixed  because  the 
market  price  of  silver  in  terms  of  gold  money  is  continually  chang- 
ing. Calculation  of  the  par  rate  of  exchange  of  the  English  sover- 
eign and  the  United  States  gold  dollar  is  shown  in  the  following 
example. 

Example 

Find  the  par  of  exchange  of  the  Knglish  sovereign  and  the  United  States  gold 
dollar. 

293 


294  FOREIGN  EXCHANGE 

Solution 

Weight  of  English  sovereign  piece         123  2744700  grains 

Deduct  alloy,  Sj-  %  10  272H725 

Net  weight  of  gold        .  1 13  001,5975 

*Weight  of  United  States  gold  dollar  15  23X0952  grains 

Deduct  alloy,  10%       .  .  1  523S095 

Net  weight  of  gold  .    .  .        .13  7142857 

The  par  of  exchange  is  1 13.0015975  -5-  13.7142S57  -  S  239099S 

A  comparative  table  showing  the  values  of  foreign  coins 
is  issued  at  frequent  intervals  by  the  United  States  Treasury 
Department. 

Current  'rate  of  exchange.  The  current  rate  of  exchange, 
under  normal  conditions,  will  fluctuate  between  the  gold  import 
and  the  gold  export  point,  slightly  below  or  above  par.  This 
fluctuation  is  caused  chiefly  by  demand  and  supply. 

Under  abnormal  conditions,  the  rates  are  quite  different  from 
par,  owing  to  a  number  of  causes,  some  of  which  are: 

(1)  Suspension  of  the  gold  standard  in  the  country  where  the 
rate  is  quoted,  or  in  the  country  whose  currency  is  quoted. 

(2)  The  extent  to  which  the  currency  of  either  country  is  depre- 
ciated as  a  result  of  management  of  the  currency  or  inflation. 

(3)  Lack  of  confidence  in  the  stability  of  foreign  government. 

(4)  Issuance  of  a  large  amount  of  paper  money  by  the  foreign 
country. 

(5)  Shrinkage  of  gold  and  other  legal  reserves. 

Sterling  quotations  are  usually  made  on  " demand,"  but  may 
be  made  on  30-,  GO-,  or  90-day  drafts.  Where  time  is  a  factor  in 
calculations,  three  days  of  grace  are  allowed,  interest  being  calcu- 
lated on  the  basis  of  3(>5  days  to  the  year  for  sterling  drafts. 

Six  classes  of  problems.  The  mathematics  of  foreign  exchange 
may  be  resolved  into  six  classes  of  problems: 

(1)  Conversion  of  one  monetary  unit  into  terms  of  another. 

(2)  Interest  on  foreign  exchange. 

(3)  Bankers'   and  brokers'   problems   of  valuing  time  bills  of 
exchange. 

(4)  Finding  the  value  of  an  account  as  a  whole. 

(5)  Averaging  accounts  of  foreign  exchange  bearing  interest. 

(6)  Foreign  branch  house  accounting. 

Conversion  of  one  monetary  unit  into  terms  of  another.     The 

necessity  of  converting  one  monetary  unit  into  terms  of  another 

*  By  proclamation  of  the  President,  the  weight  of  the  gold  dollar  \ras  fixed  at 
15<i5f  grains  nine-tenths  fine  on  January  31,  1934. 


FOREIGN  EXCHANGE  295 

results  from  the  simple  purchase  of  letters  of  credit,  travelers* 
checks,  and  cable  transfers  on  a  foreign  country.  These  transfers 
are  calculated  at  the  quoted  or  current  rate  of  exchange.  This 
current  rate  is  the  rate  prevailing  for  that  particular  class  of 
transfer  on  the  date  of  purchase. 

To  make  the  calculations,  reduce  the  amount  of  foreign  money 
to  the  term  of  the  monetary  unit  to  which  the  exchange  rate  is 
applicable,  that  is,  in  units  and  a  decimal  fraction  thereof,  as, 
1*150  7s.  6d.  =  £150.375.  Multiply  by  the  conversion  rate  and 
add  the  charges  for  the  transfer. 

Example 

What  is  the  cost  of  a  letter  of  credit  on  London  for  £150  7s.  6d.,  if  the  bank 
charges  \(  'v  for  its  service?  Assume  that  the  current  rate  of  exchange  is  $4.0"). 

Solution 

C150  000 

7s.  -  -Jo-  of  £1  :*50 

<>d.  =  2To  ()f  M  025 

£150  375 

Multiply  l»y  rate  of  exchange  4  05 

Dollar  value  at  current  rate  .    .           .      .      (109  02 

Service  charge  of  2  '  (  •*  05 

Total  cost  .       .                                               (>12  07 

Conversion  of  decimals  of  one  monetary  unit  into  monetary 
units  of  a  smaller  denomination.  This  is  simply  a  matter  of 
reduction  of  decimals  of  denominate  numbers. 

Example 

What  value  of  London  exchange  may  be  purchased  with  $1,000,  if  the  rate 
is  S4.40i? 

Solution 

$1,000  ~  S4  40375  -  224  02(>9,  the  number  of  pounds  sterling 
.02o9  X  20  =  .53Ss.;  therefore,  no  shillings 
.r>3S  X  12  =  (5.450d. 

Answer:  €224  Os.  6d. 

Problems 

Find  the  cost  in  dollars  of  each  of  the  following: 

1.  €1,200  16s.  at  S3.7.H. 

2.  £8,240  5s.  Sd.  at  $3.90;  commission,  1%. 

3.  Calculate  how  much  exchange  can  be  purchased  with  the  following.  $750 
on  England,  rate,  S3. 845. 

Interest  on  foreign  exchange.  To  find  the  interest  on  foreign 
exchange : 

(a)  Reduce  the  amount  of  foreign  money  to  the  term  of  the 
monetary  unit  to  which  the  exchange  rate  is  applicable. 


296  FOREIGN  EXCHANGE 

(b)  Compute    the   interest    in    the    terms   of   that    monetary 
unit. 

(c)  Reduce  the  interest  to  its  proper  exchange  units. 

Example 

Find  the  interest  on  £200  5s.  fxl.  for  63  days  at  6%. 

Solution 

£  ....  .  £200  00 

5s.  -  |  of  £1  (decimally)  .  25 

(3d.  -  Vo  of  €1  (decimally)  .  025 

£200  275 

Interest  for  GO  days  (point  ofT  2  places)  .          C     2  0027500 

Interest  for  3  days  (2*0  of  GO  days'  int.)  .  1001375 

t     2   102SS75 

The    above    amount    is    ordinary    interest    and    is 

changed  to  exact  interest  by  deducting  ^  02SS066 

C     2  0740S09 

Reducing  the  decimal  of  a  pound  to  shillings,  .0740S09  X  20  =  1.4X1G1S 
shillings.  Reducing  the  decimal  of  a  shilling  in  pence,  .4S101S  X  12  =  5.779410, 
or  practically  0  pence.  Answer:  £2  Is.  (kl. 

Problems 

1.  Find  the  interest  on  £250  10s.  (kl.  for  93  days  at  6%. 

2.  Find  the  interest  on  I' 1,000  15s.  lOd.  for  03  day;  at  5<;)t 

To  find  the  value  of  a  time  bill  of  exchange.  The  four  elements 
entering  into  the  determination  of  the  value  of  a  time  hill  of 
exchange  are: 

(1)  Current  rate  of  demand  exchange. 

(2)  Interest  on  money  (exact  time). 

(3)  Stamp  charges. 

(4)  Commission  of  home  or  foreign  banks. 

Example 

If  the  demand  rate  of  exchange  is  $4.05000,  discount  rate  4ro,  stamps  $$(  [,, 
and  commission  \%,  what  is  the  60-day  rate? 

Solution 

Demand  rate .         .  S4  05000 

Deduct: 

Interest  at  4%  for  ^W  of  a  year  02790 

Stumps,  ?V%  -.-.  00203 

Commission,  -\-%         .       .  .  01013 

04012 

SO-day  rate     S4  00988 


FOREIGN  EXCHANGE  297 

Problem 

Determine  the  proceeds  of  a  60-day  draft  on  London  for  £200,  if  the  following 
conditions  prevail : 

Demand  rate  on  London       $3.90 

Interest  rate             .                   .           7% 

Stamp  charges          .                             -^rc 

Commission  charged  by  Knglish  hank              }-% 

Commission  charged  by  American  bank j  % 

Foreign  exchange  accounts.  Kates  of  exchange  fluctuate; 
therefore,  it  is  necessary  to  establish  a  method  of  keeping  records 
of  transactions  to  show  current  values. 

Procedure:  (a)  Extend  the  items  of  debits  and  credits  in  foreign 
currency  at  the  rates  of  exchange  stated  for  each  item.  (In 
practice,  this  would  be  performed  when  the  entry  is  recorded.) 

(b)  Strike  a  balance  of  the  foreign  currency,  and  convert  this 
balance  at  the  current  rate  of  exchange.  Insert  this  result  among 
the  dollar  items. 

(c}  Find  the  difference  between  the  total  debit  dollars  and  the 
total  credit  dollars.  A  debit  difference  indicates  a  loss  on  exchange, 
while  a  credit  difference  indicates  a  profit  on  exchange. 

Example 

State  the  balance  of  the  following  account  in  both  foreign  and  domestic 
Currency,  and  show  the  profit  or  loss  on  exchange,  exclusive  of  interest  charges 
and  credits.  The  current  price  of  exchange  is  quoted  at  .$4.51  on  the  last  day 
of  the  month. 

DAVIS  &  COMPANY- -BANKKKS,  LONDON 

Debits: 

Jan.     1  Remittance,  sight  bill,   C  1,000  at  $4.4(>j[ 

Jan.  10  Remittance,  sight  bill,  £500  10s.  at  $4.49i 
Credits: 

Jan.    S  Drafts  drawn,  Cl 00  at  $4.47 

Jan.  24  Cable,  £1,100  10s.  (id.  at  $4.52$ 

Solution 

Foreign  Rale        Domestic 
Debits: 

Jan.     1      Remittance                      .                C  1,000    Os.  $4  4(>g     $4,463  75 

Jan.  10     Remittance                                        500  10s.  4  495      2,249  75 

Profit  on  exchange  60  27 

£1,500  10s.  $6,779  77 

Credits: 

Jan.    S     Drafts        C    100    Os.  Od.  $4  47       $    447  00 

Jan.  24     Cable                       .  .                        1,100  10s.  6<i.  4  525      4,979  88 

Balance              299  19s.  6d.  4  51          1,352  89 

£1,500  10s.  Od.  $6,779  77 


298  FOREIGN  EXCHANGE 

Explanation.  The  balance  of  the  account  is  found  by  first  deducting  the 
smaller  side  of  the  account  in  foreign  money  from  the  larger  side.  In  the  above 
case,  the  smaller  side  amounts  to  £1,200  10s.  6d.,  and  the  larger  side  to  £1,500 
10s.;  the  balance  is  therefore  £299  19s.  6d.  Second,  convert  this  balance  into 
domestic  money  at  the  current  rate  of  exchange,  $4.51;  the  balance  is  found  to 
be  $1,352.89. 

The  profit  or  loss  on  exchange  is  the  difference  between  the  total  debit  and 
the  total  credit  of  the  domestic  money  columns. 

There  might  be  a  question,  in  determining  the  correct  valuation  of  the  balance 
on  hand,  as  to  whether  the  £299  1 9s.  6d.  should  have  been  valued  at  the  last 
buying  price  of  $4.49i,  or  at  $4.51,  the  current  market  price.  This  is  purely  a 
question  of  accounting  and  finance.  The  usual  practice  is  to  take  the  current 
rate  on  the  last  day  of  the  month,  when  given;  otherwise,  the  cost  of  the  exchange 
on  hand,  if  that  can  be  determined  by  inspection,  will  be  acceptable. 

Problems 

1.  Find  the  balance  of  the  following  account  in  both  foreign  and  domestic 
currency,  and  the  profit  or  loss  on  exchange,  exclusive  of  interest: 

Debits: 

Aug.    1  Balance      £     500  10s.  4d.  ©  $4 . 85 

Aug.  10  Remittance             11,000    Os.  Od.  @    3  90 

Aug.  20  Collection              200    Os.  6d.  («}    3  875 

Aug.  25  Discounts 196    5s.  6il.  (a)    3  88 

Credits: 

Aug.    3  Cable          £      200    Os.  Od.  (m  $3  88 

Aug.    7  Sight  draft            400  10s.  Od.  (m    3  S75 

Aug.    9  Sight  draft        300    Os.  Od.  ©    3  89 

Aug.  15  Cable 2,000  15s.  6d.  (m    3  87i 

Aug.  20  Cable 400    Os.  Od.  (5)    3  87i 

Demand  rate  on  Aug.  31,  $3.90. 

2.*  A  dealer  in  foreign  exchange  finds  from  his  books  that  he  has  had  tht 
following  transactions  in  London  exchange  during  a  particular  month: 

Exchange  bought  in  local  market : 

Jan.    1  30-day  bill,  payable  in  London,  £300  («),  $4.75 

Jan.  15  Hill  due  in  London,  at  sight,  £2,500  @  $4.76 
Exchange  sold  in  local  market: 

Jan.    5  Bill  due  in  London,  at  sight,  £1,000  @  $4.77 

Jan.  20  Cable  transfer,  £2,000  @  $4.78 
Foreign  correspondent's  draft  honored  and  paid: 

Jan.  20  Bill  at  30  days  after  sight,  accepted  December  21,  £500  @  $4.78 

State  how  the  balance  of  the  account  stands  at  the  close  of  the  month,  and 
how  much  profit  or  loss  has  been  derived  from  the  transactions.  (At  January  31, 
the  rate  for  cable  transfers  is  $4. SO.) 

Is  the  profit  or  loss  so  seated  final? 

Averaging  accounts  in  foreign  exchange.     In  general,  trans- 
actions in  foreign  exchange  accounts  are  large,  and  involve  the 


*  American  Institute  Examination. 


FOREIGN  EXCHANGE 


299 


holding  of  considerable  sums  of  money;  because  of  this,  interest 
is  a  very  important  factor.  If  the  interest  has  not  been  taken  care 
of  at  the  time  of  the  transaction,  it  must  necessarily  be  considered 
later. 

The  principle  of  averaging  accounts,  as  explained  previously  in 
this  text,  will  be  applied  here. 


Example 

Find  the  average  due  date  of  the  following  account;  also  the  amount  due 
June  1  following,  including  interest  at  6%: 


Debits 


Jan.  2 
Jan.  31 
Mar.  16  . 


Credits 

£600    Feb.  15  .  £500 

300     Apr.     1  200 

100 

Solution 

Debits        Days        Day-£         Total 


Jan.     2 

£    600 

2 

£  I  200 

Jan.  31  

300 

31 

9300 

Mar.  16 

100 

75 

7500 

Feb.  15  

£1,000 

Credits 
C    500 

Days 
46 

Day-£ 
£23  000 

£18,000 
Total 

Apr.     1  

200 

91 

18,200 

Balance.  £  

£    700 
300 

£41,200 
£23.200 

Using  the  last  day  of  the  month  preceding  the  first  item  as  the  focal  date, 
23,200  -f-  300  =  77  days.  As  the  balances  of  the  £'s  and  the  Day-£'s  are  on 
opposite  sides,  the  due  date  will  be  counted  backward  from  the  focal  date; 
that  is,  £300  should  have  been  paid  77  days  before  December  31,  or  on  October  15, 
in  order  that  neither  party  should  lose  interest. 

From  October  15  of  one  year  to  June  1  of  the  next  year  is  229  days.  £300  for 
229  days  at  6%  is: 

229 
300  X  .06  X  -    I  =  £11.29315 

ouO 


.29315  X  20  =     5.8630s. 
.863  X  12  =  10.356d. 


Answer: 


Principal  due  October  15  last 
Interest  to  June  1 
Total  due  June  1 


£3000s.    Od. 

1 1  5s.  IQd. 

£311  5s7"iOcT 


Problems 

1.  Find  the  average  due  date  of  the  following  account,  arid  the  amount  due 
on  July  1,  including  interest  at  5%: 


300  FOREIGN  EXCHANGE 

Del/its  Credits 


June  1 

£200  Os.  Od. 

June  4  .  . 

£400  15s  9d 

June  4  

300  10s.  6d. 

June  12  . 

400  Os.  Od. 

June  6 

.   .  .   400  Os.  Od. 

June  24   .... 

500  1  5s.  Od. 

June  12 

.  .   600  15s.  3d. 

June  30  

600  Os  Od 

June  25   

700  Os.  Od. 

June  28 500    Os.  Od. 

2.  Bond,  who  is  located  in  New  York,  has  an  account  with  Waite  in  London. 
Waite  engages  an  accountant  to  prepare  from  the  following  data  a  statement  to 
be  mailed  to  Bond: 

Debits  Credits 

May  12                 .  £  650  June  10   £  400 

May  30.     .  .        .217  June  30 400 

June  12                  .              .                240  July     1,  Balance        557 

July     1  .      J250  

£1,357  £1,357 

Find  the  average  due  date  of  the  account,  and  the  interest  at  5%  to  July  1, 
using  365  days  to  the  year. 

Conversion  of  foreign  branch  accounts.*  The  conversion  of 
foreign  branch  accounts  requires  the  application  of  different  rates 
of  exchange. 

Current  asset  and  current  liability  values  should  be  converted 
at  the  rate  prevailing  as  of  the  date  of  the  balance  sheet. 

Fixed  asset  values  should  be  converted  at  the  rate  prevailing 
at  the  time  of  purchase,  or  at  the  average  rate  for  purchases  during 
a  fiscal  period.  If  there  have  been  no  changes  in  fixed  assets 
during  the  fiscal  period,  the  fixed  assets  should  be  valued  at  the 
same  rate  as  in  the  preceding  period.  Differences  in  values  of 
fixed  assets  from  one  period  to  another,  due  to  fluctuations  in 
exchange,  should  not  be  allowed  to  affect  the  results  of  the  fiscal 
period. 

Remittances  should  be  converted  at  the  actual  rate  prevailing 
on  the  day  of  the  transmittal  of  the  money. 

Revenue  items  should  be  converted  at  the  average  rate  for  the 
period. 

The  Controlling  or  Adjustment  account  and  the  Old  Inventory 
account  should  be  converted  at  the  rate  established  on  the  head 
office  books  at  the  end  of  the  preceding  fiscal  period. 

The  following  example  illustrates  the  above  principles  of 
conversion. 

Example 

A  corporation  having  its  head  office  in  Boston  had  a  branch  office  in  London. 
The  trial  balances  of  the  head  office  and  of  the  branch  office  on  December  31  were* 


*  See  page  294, "  Current  rate  of  exchange. " 


FOREIGN  EXCHANGE 


301 


BOSTON  OFFICE 
TRIAL  BALANCE 


Cash     ..      .    . 
Branch  Account 
Remittances 
Expenses  . 
Income 
Capital  Stock 
Surplus 


$    100,000 
876,000 

$    130,500 
40,000 

60,000 

800,000 

25,500 

$f,0l6,006  $1,016,000 


LONDON  BRANCH  OFFICE 

TRIAL  BALANCE 

Cash 

Remittances 

Customers     . 

Inventory,  December  31 

Expenses 

Income 

Creditors 

Boston  OiTicc  Control 


£   15,000 
30,000 
130,000 
50,000 
10,000 

£  25,000 
10,000 

_   200,000 
£235,000  £235^000 


An  analysis  of  the  Remittance  account  showc 

June  1,  from  London  Branch 
Sept.  1,  from  London  Branch 
Dec.  1,  from  London  Branch  . 

Current  rate 

Average  rate 


£10,000  ©  $4  42 
10,000  &  4  28 
10,000  ©  4  35 
$4  40 

4  30 


From  the  foregoing  data,  prepare: 

(a)  Statement  of  conversion. 

(h)  Consolidated  trial  balance  working  paper. 

(c)  Consolidated  profit  and  loss  statement. 

(d)  Consolidated  balance  sheet. 

Solution 
BRANCH  OFFICE 


Cash      . 
Remittances 
Customers 
Inventory 

Expenses 

Income     .    .        . 
Creditors 
Boston  Control 
Profit  on  Exchange 


STATEMENT  OF  CONVERSION 

]  Bounds 

Pounds 

Rate 

Dollars 

Dollars 

15,000 

$4  40 

66,000 

30,000 

4  35 

130,500 

130,000 

4  40 

572,000 

.     50,000 

4  40 

220,000 

10,000 

4  30 

43,000 

25,000 

4  30 

107,500 

10,000 

4  40 

44,000 

200,000 

4  38 

876^000 

4.000 

2357000 

235,000 

F,03  1,500 

1^031,500 

302 


FOREIGN  EXCHANGE 


cv 

j-< 

;43 

o 

fl 

8    ' 

O 

g 

&  S 

o  sfi 

d 

r< 

cp  O 

o 

O 

CJ 

d 

a  ^ 

W 

fi 

o 

a 

C5 
3 

£ 

0    g 

W  o 

^ 

3 

fci 

PQ 

S 

S 

o 

O 

o 

o 

^ 

^3 

o 
5    fl 

i  2 

)« 

Remittances 

Expenses  at 
Income  at  B 

a 
O 

W 

31 
'§•£ 

da 

Remittances 

Customers  . 

> 

1 

c 

£ 

c 

HH 

Expenses  at 

w 

+0 

c3 
<D 

1 

HH 

Creditors  .  .  . 

G 
O 

0 

1 

a 

0 

1 

OH 

FOREIGN  EXCHANGE  303 

CONSOLIDATED  PROFIT  AND  Loss  STATEMENT 
Income: 

Boston  Office  .......................  $  60,000 

London  Office  .........  107,500 

Total  ............  .  $167,500 

Expenses  : 

Boston  Office  $  40,000 

London  Office  ....       43,000 

Total  .  _  83,000 

Operating  Income  .  $  84,500 

Profit  on  Exchange  t  .  ___  4,000 

Net  Profit  to  Surplus         .......  $jS8j500 

f  Some  may  question  the  advisability  of  including  the  Profit  on  Exchange  in  the 
Not  Profits.  The  sotting  up  of  an  account  "Reserve  for  Fluctuation  of  Exchange" 
is  sometimes  advooatod.  This  is  purely  an  accounting  problem,  however,  and  will 
not  bo  discussed  here. 

CONSOLIDATED  BALANCE  SHEET 

.4  ssvts 
Cash: 

Boston  .  .  ...   $100,000 

London  ....  .  .  66,000  $166,000 

Customers  .  572,000 

Inventory          .................  220,000 

$958,000 

Liabilities  and  Capitol 
Creditors        .  .  .    .  .    .  $  44,000 

Capital: 

Capital  Stock  ....  $800,000 

Surplus: 

Balance,  Jan.  1         .  ...      $25,500 

Net  Profit       ...........     88,500     114,000     914,000 


In  the  above  example,  no  fixed  assets  were  stated.  When,  as 
sometimes  happens,  this  item  appears  in  the  Branch  Trial  Balance, 
other  difficulties  are  encountered. 

To  overcome  these  difficulties,  it  is  necessary,  in  making  a 
statement  of  conversion,  to  divide  the  account  on  the  Head  Office 
books,  "Branch  Control,"  into  "Branch  Control  —  Fixed  Assets," 
and  "Branch  Control—  Current  Assets."  The  rate  at  the  time  of 
purchase  should  be  used  for  the  conversion  of  the  Branch  Control  — 
Fixed  Assets  account.  By  converting  at  this  rate,  the  value  of  the 
fixed  assets  is  found  in  terms  of  the  monetary  unit  of  the  country 
in  which  the  Head  Office  is  located.  By  deducting  the  value  of  the 
fixed  assets,  or  the  Branch  Control  —  Fixed  Assets,  from  the 
Branch  Control  account,  the  value  of  the  Branch  Control  — 
Current  Assets  is  found. 

The    following   example    illustrates    this   point: 


304 


FOREIGN  EXCHANGE 


Example 

A.  United  States  company  doing  business  in  London  through  its  London 
Branch,  received  a  trial  balance  of  the  branch  on  December  31,  as  follows: 

LONDON  BRANCH 

Cash 

Remittances       

Customers 

Inventory  (new) 

Expenses 

Plant 

Creditors  . 

Income  from  Sales 

New  York  Control         ... 


£  5,000 

25,000 

50,000 

25,000 

25,000 

100,000 

£  50,000 

50,000 

130,000 

£230,000 

€230,000 

NEW  YORK  OFFICE 


Cash 

London  Brand 
Remittances 
Expenses     . 
Capital  Stock 
Surplus . . 


*  100,000 

617,500 

$118,875 
50,125 

500,000 

148,750 

$767,625   $7077625 


£5,000  ©  $4.75i 
£5,000  ©  $4.75f 


The  following  remittances  were  received: 
£5,000  (m  $4.75 
£5,000  ©  $4.75i 

£5,000  ©  $4.76 

The  current  rate  of  exchange  was  $4.76^. 
The  average  rate  of  exchange  was  $4.75^. 
The  fixed  rate  of  exchange  for  the  plant  was  $4.74^. 

From  the  information  given,  show:  (a)  a  statement  of  conversion;  (b)  a 
balance  sheet  of  the  London  Branch;  and  (c)  a  consolidated  balance  sheet  of  the 
New  York  Office.  Show  also:  (d)  the  accounts  on  the  books  of  the  New  York 
Office  in  which  the  profit  is  taken  up. 

Solution 


STATEMENT  OF  CONVERSION 

Cash  

£     5,000 

@  $4  7  6  3-  $ 

23,825  00 

Remittances   

25,000 

©      4    75|r 

118,875  00 

Customers  

50,000 

©    4  76^ 

238,250  00 

Inventory.   . 

25,000 

©    4  76£ 

119,125  00 

Expenses.  . 

25,000 

©    4  75| 

118,812  50 

Plant      .  . 

100,000 

©    4  74£ 

474,500  00 

Creditors 

£  50,000  ©    4  76^ 

<1 

i    238,250  00 

Income  from  Mdse 

50,000  ©    4  75| 

237,625  00 

New  York  Control 

(Plant  Acct.)  .... 

100,000  (a\    4  74£ 

474,500  00 

New  York  Control 

(Current  Acct.)  . 

30,000  ©    4.  70f 

143,000  00 

Profit  on  Exchange 

12  50 

£230,000  £230,000 


$1,093,387  50  $1,093,387  50 


FOREIGN  EXCHANGE 


305 


CONVERSION  OF  BRANCH  CONTROL  ACCOUNT 

Plant  and  Fixed  Assets 

London  Branch  Control     

Deduct: 

Value  of  Plant  £100,000 

Rate  of  exchange  for  fixed  assets 4.74^ 

Value  in  dollars     

New  York  Control     £130,000 

Less  Plant  Account J^i000 

Current  Assets  at  Branch         £  30^000 

The  rate  is  found  to  be  143,000  -^  30,000  =  4.76f . 

BALANCE  SHEET  OF  LONDON  BRANCH 
Assets 


$617,500 


474,500 
$143,000 


143,000 


Cash 

Customers 
Inventory 
Plant 

Liabilities 
Creditors 
Capital 
Plant 
Current  (Schedule  A) 


( 1r  edits 


Balance 
Income 


Profit  on  Kxchangc 


€     5,000  Ca\  $4  76i  $  23,825 

50,000  (a,    4.704-  238,250 

25,000  (a\    4.764  1 1 9, 1 25 

100,000  (n>    4.74i  474,500 

£180,000  $855,700 

50,000  (m  $4.7(>i  238,250 

£130,000  $617,450 

100,000  Cm  $4  74£  $474,500 

30,000  @    4  76i  142,950     617,450 

SCHEDULE  A 
Current  Account 

£30,000  (m  $4  76|  $143,000  00 
.  .      50,000  <&    4  75i     237,625.00 
£80,000 

12.50 


$380.637.50 

Debits 
Expenses 

£25000  (< 

7*  $4  75j- 

$118812  50 

Remittances        

25,000  (j 

l\    4  .  754- 

118,875  00 

£50,000 

237,687  50 

Balance 

....    £30,000 

$142,950  00 

The  rate  is  found  to  be  142,950  -r-  30,000  =  4.76^. 

NEW  YORK  OFFICE 
LONDON  CONTROL  ACCOUNT 

$474,500  00     Remittances 

143,000  00    Expenses     

237,625  00     Ii§il  I  Plant  Acct 

'  I  Current  Acct 


p  i  f  Plant  Acct 

'  I  Current  Acct 
Income  from  Mdse 
Profit  on  Exchange 


12  50 
$855,137  50 


$118,875.00 
118,812.50 
474,500.00 
142,950.00 

$855,137.50 


306  FOREIGN  EXCHANGE 

PROFIT  AND  Loss  ACCOUNT 

Expenses  of  Branch $1 18,812  50     Income  of  Branch  .  .   $237,625.00 

Expenses  of  Home  Office.       50,125  00     Profit    on    Exchange    of 

Profit  to  Surplus 68,700  00        Branch 12  50 

$237L637^5Q  $237^637  50 

NEW  YORK  BALANCE  SHEET 
Assets 
Current: 
Cash: 

New  York   $100,000 

London 23,825  $123,825 

Customers — London 238,250 

Inventory— London 119,125  $481 ,200 

Fixed: 

Plant— London     474,500  $955,700 

Liabilities 
Current : 

Creditors—London 238,250 

$717,450 
Capital: 

Capital  Stock        $500,000 

Surplus         $148,750 

Net  Profits 68,700   217,450 

"jm  7,450 

A  balance  sheet  of  the  London  Branch  is  next  set  up,  but  this  balance  sheet 
needs  no  explanation. 

In  the  above  solution,  note  particularly  the  schedule  of  the  current  account 
and  the  method  of  finding  the  rate. 

Various  methods  of  taking  up  the  profits  of  the  branch  on  the  Head  Office 
books  are  used.  Probably  no  explanation  need  be  made,  except  as  to  the  credit 
to  the  Profit  and  Loss  account  of  the  $12.50  profit  on  exchange.  This  might 
have  been  credited  to  an  account  called  "Reserve  for  Fluctuation  of  Exchange." 

Problems 

1.*  On  December  31,  the  trial  balance  on  the  books  of  the  London.  Office  of 
the  A.  Rubber  Company  is  as  follows: 

£       s.  d.         £       s.  d. 

Estate  Purchase  3,000     0  0 

Estate  Development       8,000     0  0 

Estate  Produce  Stock,  Mar.  1 . .     .   .          600    0  0 

Cash  at  Bank,  London     800     0  0 

Estate  Manager,  Jan.  1 928  12  8 

Remittance  to  Estate  Manager     .    .  .        1,000     0  0 

London  Office  Expenses 400    0  0 

Share  Capital 12,000    00 

Creditors      1,900     00 

Profit  and  Loss  Balance 828  12  8 

£14,728"! 2^  £l4^728"l2~8 


*  C.  P.  A.  Examination. 


FOREIGN  EXCHANGE  307 

After  the  above  balances  have  been  taken  out,  the  accounts  to  December  31 
are  received  from  the  estate  manager,  as  follows  (the  dollar  to  be  taken  at  4s.  4d.)  : 

Balance,  Jan.  1  .................       $  4,280 

Remittances  from  London  ..........       8,600 

Rebates  .  .  ...  131 

Sale  of  Produce  .      .  ..       2,000 

Profit  on  Rice  ...  .  .          249 

Expenditures  on  Development  .  .  .  .  $  9,000 

Expenditures  on  Purchase  of  New  Land.  .  2,800 

Expenditures  on  Upkeep  of  Estate     ...  .  1,640 

Balance  Carried  Forward  ...............  1,820 

115,266 


The  produce  unsold  at  December  31  was  valued  by  the  manager  at  $5,500. 
You  are  required  to  construct  the  Revenue  account  and  the  balance  sheet  for 
presentation  to  the  shareholders. 

2.  Change  into  dollars  and  cents  the  following  items  of  a  London  Brunch, 
and  show  the  new  value  of  the  Head  Office  account: 

Fixed  assets  .....                  .  £6,000  Rates  of  exchange: 

Inventory  (new)   .  500  Current    .............  $4  78-g- 

Cash                   ...  1,000  Average  remittance  ......  4  62fV 

Profit  of  period   ...  1,500  Opening  rate  ..........  4  86fV 

Head  Office  account     .    .      .  5,000  Average  rate  of  year  4  60T7<5 

Remittance  from  Head  Office.  .  1,000  Balance    on    Head    Office 

books  ..................  $27,200 


PART  IB 


CHAPTER  30 
Compound  Interest 

Compound  interest.  In  computing  simple  interest  (Chapter 
7),  it  has  been  seen  that  the  principal  remains  constant.  In  com- 
puting compound  interest,  the  principal  is  increased  by  the 
additions  of  interest  at  stated  intervals.  The  total  amount 
accumulated  at  the  end  of  some  given  time  is  the  compound 
amount,  and  the  difference  between  the  compound  amount  and 
the  original  principal  is  the  compound  interest.  The  principal  of 
compound  interest  is  logical,  for  if  the  periodic  interest  were  paid 
to  the  lender,  he  would  have  this  additional  principal  available 
for  investment  during  the  following  period,  and  so  on  to  the  end  of 
the  last  period. 

Compound  interest  method.  The  compound  interest  method 
is  the  most  accurate  and  scientific  means  of  finding  the  true  value 
of  an  investment.  For  this  reason  it  is  essential  that  accountants, 
and  also  investors  in  general,  be  familiar  with  it.  The  method  is 
based  on  the  foregoing  assumption  that  all  accumulations  of 
interest  become  a  part  of  the  investment  at  the  end  of  each  interest 
period. 

Actuarial  science.  Actuarial  science  is  the  mathematical  sci- 
ence based  upon  compound  interest  and  upon  insurance  probabili- 
ties (see  Chapter  38).  It  deals  with  the  investment  of  funds,  and 
also  with  the  mortality  tables  used  by  insurance  companies  (see 
Table  7,  page  536,  in  the  Appendix).  The  actuary  uses  tables  for 
the  greater  part  of  his  work :  yet  he  must  also  have  a  knowledge  of 
the  fundamentals  of  his  science.  The  accountant's  interest  in 
actuarial  science  is  to  give  the  best  service  to  his  clients  by  being 
able  to  compute  investment  values,  prepare  schedules  of  amortiza- 
tion, set  up  sinking  fund  accounts,  and  so  forth. 

Symbols.  In  the  choice  of  the  symbols  used  in  this  text,  the 
attempt  has  been  made  to  select  those  which  are  most  commonly 
accepted. 

The  following  symbols  are  given  at  this  time  for  reference : 

1  A  unit  of  value,  as  $1,  or  the  basis  of  any  unit  of  value. 

t  The  rate  of  interest  for  a  single  period. 

j  The  nominal  annual  rate,  if  interest  is  compounded  more  often 
than  once  each  year. 

311 


COMPOUND  INTEREST 

n  The  number  of  periods. 

r  The  periodic  ratio  of  increase,  or  (1  +  t). 
m  Frequency  of  periods  during  year. 

s  The  compound  amount  of  1 . 

v  The  present  worth  of  1. 
D  The  compound  discount  on  1  or  the  quantity  of  discount. 

7  The  compound  interest  on  1  or  the  quantity  of  interest. 
an  The  present  value  of  an  annuity  of  1 .     (A  is  also  used.) 
R  Rent,  or  periodic  payment  of  an  annuity. 
sn  Amount  of  an  annuity  of  1. 
P  The  principal. 
S  Any  amount. 

Principal.  The  principal  is  a  sum  of  money  or  element  of 
value  for  which  interest  is  paid,  or  on  which  interest  computations 
are  based;  1  or  $1  will  be  used  in  most  of  the  calculations  in  this 
work. 

Time.  The  time  which  an  investment  has  to  run  is  commonly 
stated  in  years  and  months,  but  in  compound  interest  computa- 
tions it  is  better  to  state  the  time  as  a  number  of  periods  of  equal 
duration.  Thus,  if  a  problem  calls  for  "four  years  and  six  months, 
interest  compounded  semiannually,"  the  time  should  be  stated  as 
nine  periods  of  six  months  each. 

Example 

Number  of 

Frequency  of  Periods,  or 

Time                                                        Compoundijig  Value  of  n 

1  year Annually  1 

1  year  .      ...            .          Semiannually  2 

1  year .   Quarterly  4 

1  year. .  .                                  .      .              Monthly  12 

3  years,  6  months         .          .          .  .       Semiannually  7 

3  years,  6  months                              .       Quarterly  14 

5  years                                           .      .    .   Annually  5 

5  years Monthly  60 

Rate.  The  rate  is  the  measure  of  interest  on  the  investment 
or  principal.  It  may  be  indicated  in  different  ways;  for  example, 
as  .06,  6%,  or  yjj-g-.  The  rate  is  generally  stated  as  so  much  a 
year,  but  when  the  period  of  compounding  is  of  any  length  other 
than  a  year,  it  is  necessary  to  restate  the  rate  as  so  much  per 
period. 

If  the  interest  period  is  a  half-year,  it  is  necessary  to  divide  the 
stated  annual  rate  by  two,  and  multiply  the  time  in  years  by 
two;  if  the  interest  is  compounded  quarterly,  it  is  necessary  to 
divide  the  annual  rate  by  four,  and  multiply  the  time  in  years  by 
four. 


COMPOUND  INTEREST 

Example 


313 


Time 
1  year            

Rate 
6% 

1 

Frequency  of      1 
Compounding 
Annually 

Number  of 
°eriods,  or 
Value  of  n 
1 

Rate, 
or  Value 
ofi 
6% 

1  year   .... 
1  year.    . 

6% 
6% 

Semiannually 
Quarterly 

2 

4 

v  /O 

3% 

l4r% 

1  year  
2  years 
2  years     

.  ..        .6% 
.      .    .   6% 

6% 

Monthly 
Semiannually 
Monthly 

12 
4 
24 

1  2   /O 
*% 

3% 
i% 

3  years,  6  months 
3  years,  6  months     . 
3  years,  6  months.    .  . 

.  .       8% 
.        •    •   4% 
.      ...  4% 

Semiannually 
Semiannually 
Quarterly 

7 
7 
14 

4    /C/ 

4% 
2% 
1% 

Ratio  of  increase.  From  every  investment,  the  investor 
expects  to  receive  the  amount  of  his  investment  plus  interest.  He 
buys  bonds,  stocks,  or  other  investments  with  the  expectation  of 
receiving  an  increased  amount  in  return.  If  the  total  investment 
be  multiplied  by  1  plus  the  interest  rate  expressed  decimally — as, 
for  example,  1.06- — the  result  will  be  the  amount  of  his  investment 
at  the  end  of  one  interest  period.  The  1  plus  the  interest  rate  is 
called  the  ratio  of  increase. 

Again,  if  the  principal,  $1,  is  placed  at  interest  at  6%  for  1 
year,  it  will  be  worth  $1.06  at  the  end  of  the  year.  The  amount  to 
be  added  to  the  $1  is  $0.06.  The  ratio  of  increase  is  expressed  as 
(1  +  .06),  or  1.06;  if  the  symbols  previously  given  were  used,  the 
symbol  would  be  (1  +  2),  or  r. 

Compound  amount  tables.  A  compound  amount  table  is  a 
compilation  of  the  value  of  1  for  various  numbers  of  periods  at 
various  rates  per  cent.  It  is  constructed  by  making  the  successive 
multiplications  (1.06) l,  (1.06)2,  (1.06)3,  (1.06)4,  and  so  forth. 

A  compound  amount  table  is  given  in  Table  2,  Appendix  III, 
page  512. 

When  a  compound  amount  table  is  available,  many  calcula- 
tions may  be  eliminated.  However,  if  one  is  not  available,  or  if 
the  factors  to  be  used  are  not  given  in  the  table,  the  desired  amount 
may  be  obtained  by  one  of  the  methods  described  below. 

Calculation  of  compound  amount.  The  following  methods  of 
calculation  are  given  to  show  the  different  means  of  arriving  at  the 
value  of  $1  for  a  given  time. 

First  method.  The  first  method  shows  each  step  taken  to  find 
the  value  at  the  end  of  each  period. 


Example 
find  the  value  of  $1  at  6%  compound  interest  for  8  years. 


314  COMPOUND  INTEREST 


Solution 

1 

.00 

X 

1.06 

=  1.06 

for 

1 

period 

1 

.06 

X 

1.06 

= 

.1236 

for 

2 

periods 

1 

.1236 

X 

1.06 

= 

.191016 

for 

3 

periods 

1 

.191016 

X 

1.06 

= 

.262477 

for 

4 

periods 

1 

.262477 

X 

1.06 

= 

.338225 

for 

5 

periods 

1 

.338225 

X 

1.06 

= 

.418519 

for 

6 

periods 

1 

.418519 

X 

1.06 

= 

.503630 

for 

7 

periods 

1.503630 

X 

1.06 

= 

.593848 

for 

X 

periods 

It  can  be  seen  that  the  compound  amount,  1.593848,  is  the  sum 
of  the  investment  and  the  compound  interest. 

The  above  method  of  arriving  at  the  compound  amount 
becomes  very  laborious  when  there  are  many  periods. 

Second  method.  *In  this  method,  multiplication  is  performed  by 
using  powers  of  numbers. 

Example 
Find  the  value  of  (1  +  i)12  or  (1.06)12. 

Solution 

ALGEBRAIC  INCREASE 

(1  +  i)  X  (I  +  i)    =  (1  +  i)2 

(1  +  i)2  X  (1  +  i)2  =  (1  +  i)4 

(1  +  i)4  X  (1  +  i)4  =  (1  +  i)8 

(1  +  i)8  X  (1  +  i)4  =  (1  +  *)12 

ARITHMETICAL  INCREASE 

Exponents  Powers  of  Katio  of 
Added  Increase  Multiplied 
(1.06)  =  1.06  for  1  period 
(1.06)  =  1.06 

(1.06)2    =1.1236      for  2  periods 
(1.Q6)2    =  1.1236 

(1.06)4    =  1 .262477  for  4  periods 

(1.06)4    =  1.262477 

(1.06)8    -  1.593848  for  8  periods 

(1.06)4   =  1.262477 

(1.06)12  =  2.012196  for  12  periods 

The  above  principle  may  be  applied  for  any  power  of  a  number. 
If  the  compound  interest  table  does  not  extend  to  a  sufficient 
number  of  periods,  the  required  value  may  be  found  by  this 
method,  as  illustrated  in  the  following  example: 

Example 
Find  the  compound  amount  of  $1  at  6%  for  80  years. 


COMPOUND  INTEREST  315 

Solution 

Assume  that  we  referred  to  the  compound  interest  table,  and  found  that  the 
highest  value  shown  at  6%  was  for  20  years,  and  was  3.2071355.  The  calcu- 
lation for  80  years  could  be  made  as  follows: 

Powers  of 
Ratio  of 

Exponents        Increase 
Added         Multiplied 
(LOG)20  =      3.2071355 
(LOG)20  =      3.2071355 
(LOG)40  =     10.285718 
0_.  OG^°  =     10.285718 
(LOG)80  =  105.795994 

Third  method.  The  calculation  by  the  third  method  is  made  by 
the  use  of  logarithms. 

Example 

Find  the  value  of  (LOG)80. 

Solution 
log  LOG...    .  ...  .0  0253059 

Multiply  by  exponent  80 

log  of  80th  power  of  1 .06  ~~~2  0244720 

Antilog  2.024472       .  .105  80 

Problems 

Work  the  following  problems  by  the  methods  indicated,  and  check  your 
results  by  referring  to  the  compound  interest  table.  Show  the  complete  work, 
as  in  the  foregoing  solutions. 

Find  the  compound  amount  of: 

1.  (1.04)4  by  Method  1.  7.  (LOG)24  by  Method  2. 

2.  (L03)6  by  Method  1.  8.  (L02)40  by  Method  3. 

3.  (LOG)8  by  Method  1.  9.  (1.005)60  by  Method  3. 

4.  (1.02)6  by  Method  2.  10.  (1.04)75  by  Method  3. 
6.  (1.005)30  by  Method  2.  11.  (1.04) 60  by  Method  3. 
6.  (1.03)20  by  Method  2.  12.  (1.05)80  by  Method  3. 

Compound  amount  of  given  principal.  To  compute  the  com- 
pound amount  of  any  principal,  apply  the  following  procedure. 

Procedure:  (a)  Compute  the  compound  amount  of  1  for  the 
number  of  periods  at  the  given  rate,  (1  +  i}n,  or  s. 

(b)  Multiply  the  compound  amount  of  1  by  the  number  of 
dollars  in  the  investment,  P(l  +  i)n  =  S. 

Example 

What  will  be  the  compound  amount  of  $100  placed  at  interest  at  6%  for 
4  years,  interest  compounded  annually? 

Formula  Arithmetical  Substitution 

P(l  +  i>  =  S  lOO(LOG)4  =  $126.25 


316  COMPOUND  INTEREST 

Extended  Solution 

1.00  X  1.06      =    1.06;  or,  amount  of  1  for  1  year  =  (1.06)1 
1.06  X  1.06      =    1.1236;  or,  amount  of  1  for  2  years  =  (1.06)2 
1.1236  X  1.06      =     1.1910;  or,  amount  of  1  for  3  years  =  (1.06)* 
1.1910  X  1.06      =     1.2625;  or,  amount  of  1  for  4  years  =  (1.06)4 
$100  X  1.2625  =  $126.25;  or,  compound  amount  of  $100  for  4  years 

Hereafter,  instead  of  the  compound  amount  of  1  at  the  end  of 
each  year  being  found  as  above,  the  solutions  will  be  shortened  by 
the  use  of  the  value  of  the  expression  (1  +  i)n,  or  s,  as  in  the  follow- 
ing. Find  the  value  of  s4  at  6%. 

Contracted  Solution 

(1.06)4  =  1.2625,  compound  amount  of  1  for  4  years 
$100  X  1.2625  =  $126.25,  compound  amount  of  $100  for  4  years 

Use  the  compound  interest  table  given  in  Table  2,  Appendix 
III,  page  512. 

Problems 

Construct  formulas  and  write  contracted  solutions  for  the  following: 
Principal     Rate      Compounded     Years 


1. 

$    600 

.00 

6% 

Annually 

4 

2. 

$    400 

.00 

3% 

Annually 

5 

3. 

$    600 

.00 

5% 

Annually 

3 

4. 

$1,000 

00 

4% 

Annually 

12 

6. 

$    256 

25 

6% 

Annually 

10 

6. 

$1,247 

.50 

3% 

Annually 

6 

7. 

$3,847 

.50 

4% 

Annually 

8 

8. 

$1,472 

25 

3i% 

Annually 

4 

9. 

$2,442 

.50 

7% 

Annually 

10 

10. 

$8,247 

.50 

9% 

Annually 

20 

Compound  interest.  Since  an  investment  placed  at  interest 
for  a  definite  time  at  a  fixed  rate  will  produce  a  given  amount,  the 
difference  between  this  amount  and  the  original  investment  will  bo 
the  increase,  or  compound  interest.  To  compute  the  compound 
interest,  it  is  therefore  necessary  to  find  the  amount  at  the  end  of 
the  time  and  to  deduct  the  principal  from  it. 

Procedure:  (a)  Determine  the  compound  amount  of  1  for  the 
number  of  periods  at  the  given  rate,  (1  +  i)n  =  s. 

(6)  Find  the  compound  interest  on  1  by  deducting  1  from  the 
compound  amount  of  1,  s  —  1  =  I. 

(c)  Multiply  the  compound  interest  on  1  by  the  number  of 
dollars  in  the  investment,  P(s  —  1)  =  /. 

Example 

Find  the  compound  interest  on  $100.00  for  4  years  at  6%. 

Formula  Arithmetical  Substitution 

P(s  -  1)  =  /  100(1.2625  -  1)  =  $26.25 


COMPOUND  INTEREST  317 

Solution 

(1.06)4  =  1.2625,  compound  amount  of  1  for  4  years 
1.2625  —  1  =  .2625,  compound  interest  on  1  for  4  years 
$100  X  .2625  =  $26.25,  compound  interest  on  $100  for  4  years 

TABLE  OF  ANALYSIS  OF  COMPOUND  INTEREST 


(1) 

End  of 
Period 
1 

(2) 

Principal 
$1  00 

(3) 
Simple 
Interest 
$  06 

(4) 
Interest 
on  Interest 

$ 

(5) 
Compound 
Interest 
$  06 

(6) 
Compound 
Amount 

$1  06 

2 
3 

4 

1  00 
1  00 
1.00 

.12 
.18 
.24 

.0036 
.011016 
.022477 

.1236 
.191016 
.262477 

1   1236 
1   191016 
1.262477 

Problems 

1.  Find  the  compound  interest  on  $500  for  5  years  at  3%. 

2.  Find  the  compound  interest  on  $650  for  6  years  at  4%. 

3.  Find  the  compound  interest  on  $2,560  for  4  years  at  6%. 

4.  Construct  a  table  of  analysis  of  the  compound  interest  on  $800  at  4% 
for  4  years. 

5.  Construct  a  table  showing  the  complete  analysis  of  $447.20  at  5%  com- 
pound interest  for  4  years. 

Results  of  frequent  conversions  of  interest.  Compound  inter- 
est is  usually  stated  as  a  certain  rate  per  annum,  but  if  the  interest 
is  to  be  compounded  more  often  than  once  each  year,  the  total 
accumulation  will  be  greater  than  the  accumulation  of  (1  +  i)n 
times  the  principal,  where  i  is  used  as  the  annual  rate  and  n  as  the 
number  of  years. 

A  study  of  the  following  will  show  the  difference  between  the 
amount  of  $100  at  6%  interest,  compounded  monthly  for  10 
years,  and  the  amount  of  $100  at  6%  interest  compounded  annually 
for  10  years. 

100  X  (1.005)120  (compounded  monthly  for  10  years). .  .  .  $181  94 
100  X  (1.06)10  (compounded  annually  for  10  years). . .  .  179.08 
Difference  caused  by  frequent  conversions  $  2  86 

Nominal  and  effective  rates.  Nominal,  as  the  word  implies,  is 
defined  as  "in  name  only."  In  illustration  (a),  above,  6%  is  the 
nominal  rate.  Effective  interest  is  the  interest  actually  received 
by  the  investor,  and  is  based  upon  the  amount  invested  and  upon 
1  year  as  the  period  of  time.  In  illustration  (a),  $6.17  is  the 
amount  of  interest  received  in  1  year  on  an  investment  of  $100;  in 
effect,  this  is  6.17%  on  the  investment,  or  an  effective  rate  of 
6.17%. 

In  order  to  distinguish  between  nominal  and  effective  rates,  the 
following  symbols  are  used: 


318  COMPOUND  INTEREST 

i  =  the  effective  annual  rate 
j  =  the  nominal  annual  rate 
m  =  the  number  of  conversions  each  year 

The  procedure  in  calculating  the  effective  rate,  the  nominal 
rate  being  given,  is  as  follows: 

Procedure:  (a)  Find  the  number  of  conversion  periods  each 
year;  (6)  find  the  rate  per  period;  (c)  determine  the  compound 
amount  of  1  for  the  number  of  periods  found  in  (a)  and  at  the  rate 
per  period  found  in  (6) ;  (d)  deduct  1  from  the  compound  amount 
of  1. 

Example 

What  is  the  annual  effective  rate  if  the  nominal  rate  is  6%,  compounded 
quarterly? 

Formula  A  rithmctical  Substitution 


Solution 

1X4  =  4,  number  of  periods 
.06  -r-  4  ==  .015,  rate  per  period 
1  X  1.015  =  1.015,    compound   amount   of    1    for 

1  period  at  1.5% 

1.015  X  1.015  =  1.030225,  compound  amount  of  1  for 

2  periods  at  1.5% 

1.030225  X  1.030225  =  1.061364,  compound  amount  of  1  for 

4  periods  at  1.5% 

1.061364  -  1  =  .061364,  or  6.1364%,  annual  effective 
rate 

The  procedure  in  calculating  the  nominal  rate,  the  effective 
rate  being  given,  is  as  follows. 

Procedure:  (a)  Determine  the  log  of  1  plus  the  effective  rate, 
or  the  ratio  of  increase. 

(6)  Divide  the  log  found  in  (a)  by  the  number  of  periods  of 
compounding. 

(c)  Find  the  antilog  of  the  quotient  of  (b),  to  determine  1  plus 
the  nominal  rate. 

(d)  Deduct  1  to  determine  the  nominal  rate. 

Example 

An  insurance  cormoany  receives  6%  effective  interest  on  a  certain  investment* 
what  is  the  nominal  rale  per  annum,  if  interest  is  compounded  quarterly? 

Formula  Arithmetical  Substitution 

KX(1  +  t)  -  l]mn  =  Nominal  rate  [^(1.06)  -  1]4  =  .058788 


COMPOUND  INTEREST  319 

Solution 

log  (1.06)  =  0.0253059 

0.0253059  -4-  4  =  0.0063265,  log  of  ratio  of  increase 
antilog  0.0063265  =  1.014672,  ratio  of  increase 

1.014672  -  1  =  .014672,  rate  of  increase  for  1  quarter 
.014672  X  4  =  .058788,  or  5.8788%,  nominal  rate  per  annum 

NOTE.     v^T-OG)  is  sometimes  written  (1.06)*. 

Effective  interest.  While  calculations  of  the  effective  rate  and 
of  the  nominal  rate  are  always  based  on  a  unit  period  of  1  year, 
most  investments  are  for  periods  of  more  than  1  year.  The  added 
feature  of  a  number  of  years  may  be  included  in  the  calculation  by 
either  of  two  methods : 

(1)  Find  the  effective  rate  for  the  year,  and  use  the  actual 
number  of  years  for  the  period  of  investment. 

(2)  Find  the  rate  for  one  period,  and  also  the  number  of 
periods,  and  use  these  results  as  the  values  of  i  and  n.     See  page 
312  under  "rate." 

The  use  of  the  first,  or  effective  interest  method,  is  advanta- 
geous in  some  annuity  computations.  However,  because  of  its 
simplicity  the  second  method  is  the  one  most  commonly  used;  it 
will  be  employed  hereafter  unless  it  is  necessary  to  use  the  first 
method  for  explanatory  purposes. 

First  method. 

Procedure :  (a)  Calculate  the  effective  rate  per  year. 

(6)  Find  the  compound  amount  for  the  number  of  years. 

(c)  Multiply  the  compound  amount  by  the  principal  in  dollars. 

Example 

What  amount  will  be  due  in  5  years  if  $200  is  placed  at  interest  at  5%,  com- 
pounded quarterly? 

Solution 
( !  +  V  )    -  1  =  .050945,  effective  rate  for  1  year 

(1. 050945) B  =  1.282037,  compound  amount  for  5  years 
1.282037  X  $200  =  $256.41,  compound  amount  of  $200  for  5  years 

Second  method. 

Procedure :  (a)  Find  the  total  number  of  periods. 
(6)  Find  the  rate  per  period. 

(c)  Determine  the  compound  amount  of  1  for  the  number  of 
periods  at  the  rate  found  in  (a). 

This  method  is  preferable  when  interest  tables  are  available. 


320  COMPOUND  INTEREST 

Example 

What  amount  will  be  due  in  5  years  if  $200  is  placed  at  interest  at  5%,  com- 
pounded quarterly? 

Solution 

The  number  of  interest  periods  is  4  X  5,  or  20.  The  rate  of  interest  per 
period  is  .05  -f-  4,  or  .0125. 

(1.0125)20  =  1.282037,  compound  amount  for  20  periods  at 

1.25% 

1.282037  X  $200  =  $256.41,  compound  amount  of  $200  for  5  years 
at  5%,  compounded  quarterly 

Problems 

1.  Find  the  compound  amounts  of  the  following: 

(a)  $1,500  at  2%  compounded  quarterly  for  5  years. 

(6)  $450.25  at  4%  compounded  semiannually  for  8  years. 

(c)  $1,250  at  3%  compounded  quarterly  for  10  years. 

2.  Find  the  effective  rate  equivalent  to : 

(a)  4%  compounded  quarterly.  (c)  8%  compounded  quarterly. 

(b)  7%  compounded  semiannually.        (d)  6%  compounded  monthly. 

3.  Construct  a  table,  similar  to  the  one  on  page  317,  for  4  years  at  3  %,  interest 
to  be  compounded  semiannually. 

Compound  present  worth.  Sometimes  it  is  desired  to  find 
what  principal  placed  at  interest  now  will  amount  to  a  certain  sum 
at  a  definite  future  time. 

The  present  worth  of  a  sum  which  is  due  at  the  end  of  a  certain 
number  of  periods  is  a  smaller  sum  which,  if  put  at  compound 
interest  at  a  given  rate,  will  amount  to  the  known  sum  in  the  given 
time. 

The  ratio  of  increase  employed  in  accumulating  1  to  a  com- 
pound amount  is  the  same  as  the  ratio  of  increase  employed  in 
accumulating  a  present  worth  to  1.  To  illustrate: 

Compound  Amount  Present  Worth 

Basis  of  calculation:  $1 .00  Present  worth  of  $1 :  $  .  792094 

Multiplying:                1.06  Multiplying:  1  06 

$1.06                                                       $  .839619 

Multiplying:                1.06  Multiplying:  1  06 

$1 . 1236                                                     $  .889996 

Multiplying:                   1.06  Multiplying:  1.06 

$1.191016                                                 $  .943396 

Multiplying:                       1.06  Multiplying:  1.06 

Compound  amount:  $1 .262477  Basis  of  calculation:  $1.000000 

1  +  1.262477  =    .792094,  present  worth 

or 
1  -T-    .792094  =  1.262477,  compound  amount 


COMPOUND  INTEREST  321 

Procedure:  (a)  Compute  the  present  worth  by  dividing  1  by  the 
compound  amount  of  1,  1  -5-  s  =  t»n. 

(6)  Multiply  the  present  worth  of  1  by  the  number  of  dollars 
to  be  produced,  S  X  vn  =  P. 

Example 

What  amount  of  money,  invested  at  compound  interest  at  6%  for  4  years, 
will  produce  $100? 

Formula  Arithmetical  Substitution 

S  X  vn  =  P  100  X  .7921  =  $79.21 

Solution 

(1.06)4  =  1.262477,  compound  amount  of  1  for  4  years  at  6% 
1  -7-  1.262477  =  .7921,  compound  present  worth  of  1  for  4  years 

at  6% 
$100  X  .7921  =  $79.21,    compound    present    worth    of    $100    for 

4  years  at  6% 

Verification 

79.21  X  1.06  =  $  83.96,  compound  amount  for  1  year 
83.96  X  1.06  =  $  89.00,  compound  amount  for  2  years 
89.00  X  1.06  =  $  94.34,  compound  amount  for  3  years 
94.34  X  1.06  =  $100.00,  compound  amount  for  4  years 


(1)            (2) 

(3) 

(4) 

End  of 

Compound  Amount 

Present 

Period  Principal 

(Inverted  Order) 

Worth  of  1 

$1.00     -f- 

$1.262477 

$   .792094 

1             1  00     ^ 

1.191016 

.839619 

2            1.00-7- 

1  .  1236 

.889996 

3             1.00     -T- 

1.06 

943396 

4            1.00     -^ 

1  00 

1.000000 

Problems 

Set  up  formulas,  solutions,  and  verifications  for  the  following: 

1.  What  amount  placed  in  the  bank  at  4%,  interest  compounded  semi- 
annually,  will  accumulate  to  $2,000  in  5  years? 

2.  What  principal  will  have  to  be  placed  at  interest  at  3i%,  compounded 
semiannually,  to  accumulate  to  $3,000  in  3  years? 

3.  What  amount  of  money  will  have  to  be  placed  on  deposit  to  cancel  a  debt 
of  $2,375.50  due  in  5  years  without  interest,  if  the  amount  deposited  is  to  be 
credited  with  interest  at  4%,  compounded  quarterly? 

4.  Construct  a  table,  similar  to  the  one  above,  for  $1  afe  4%  for  5  years, 
interest  compounded  semiannually. 

Compound  discount.  The  compound  discount  is  the  difference 
between  1  and  the  present  worth  of  1. 

Procedure  :  (a)  Calculate  the  compound  discount  by  deducting 
from  1  the  present  worth  of  1,  (1  —  vn)  =  D. 


322  COMPOUND  INTEREST 

(6)  Multiply  the  compound  discount  on  1  by  the  number  of 
dollars,  S(l  -  vn)  =  D. 

Example 

What  is  the  compound  discount  on  $100  due  in  4  years,  if  money  can  be 
invested  at  6%,  interest  compounded  annually? 

Formula  Arithmetical  Substitution 

S(l  -  vn)  =  D  100(1  -  .792094)  =  $20.79 

Solution 

(1.06)4  =  1.262477,  compound  amount  of  1  for  4  years 

at  6% 
1  -f-  1.262477  =  .792094,  compound  present  worth  of  1  due  at 

the  end  of  4  years  at  6% 
1  —  .792094  =  .207906,  compound  discount  on  1   due  at  the 

end  of  4  years  at  6  % 

$100  X  .207906  =  $20.79,  compound  discount  on  $100  due  at  the 
end  of  4  years  at  6  % 

Problems 

Construct  formulas  and  write  solutions  for  the  following: 

1.  Find  the  compound  discount  on  $500  due  in  4  years,  money  being  worth 

5%. 

2.  Compute  the  compound  discount  on  $600  due  in  5  years,  money  being 
worth 


3.  Required,  the  compound  discount  on  $800  due  in  10  years,  money  being 
worth  4%. 

Rate.  The  rate  of  interest  may  be  computed  if  the  principal, 
the  amount,  and  the  time  are  known.  The  computation  invoh  es 
the  use  of  a  principle  illustrated  in  the  chapter  on  logarithms  (see 
page  256). 

Procedure  :  (a)  Calculate  the  compound  amount  of  1  by  dividing 
the  given  compound  amount  by  the  principal. 

(6)  Determine  the  log  of  the  compound  amount  of  1. 

(c)  Divide  the  log  of  the  compound  amount  of  1  by  the  number 
of  periods. 

(d)  Determine  the  ratio  of  increase  by  finding  the  antilog  of  (c). 

(e)  Deduct  1  from  the  ratio  of  increase. 

Example 
If  $100  amounts  to  $126  25  in  4  years,  what  is  the  rate  of  interest? 

Formula  Arithmetical  Substitution 

/Amount       ,       .  4/126.25       , 


COMPOUND  INTEREST  323 

Solution 

126.25  -T-  100  —  1.2625,  compound  amount  of  1  for  4  years 

at  the  unknown  rate 
log  1.2625  =  0.101231 
0.101231  -T-  4  =  0.025307 
antiiog  of  0.025307  =  1.06,  ratio  of  increase 
1.06  -  1  =  .06,  or  6%,  the  rate 

Verification 

1.00  X  1.06      =  1.06 

1.06  X  1.06  =  1.1236 
1.1236  X  1.06  =  1.1910 
1.1910  X  1.06  -=  1.2625 
$100.00  X  1.2625  -  $126.25 

Problems 

1.  If  $100  amounts  to  $130.70  in  5  years,  what  is  the  rate  of  interest? 

2.  If  $1,000  amounts  to  $1,127.16  in  2  years,  what  is  the  rate  of  interest,  com- 
pounded monthly?     Set  up  the  formula,  the  solution,  and  the  verification. 

3.  Compute  the  annual  rate  of  interest  for  each  of  the  following: 


(a) 

$    100 

$    133.82 

5  years 

(W 

200 

310  59 

10  years 

(c) 

80 

212.26 

20  years 

(d) 

1,000 

2,830.75 

25  years 

(e) 

40 

68.10 

18  years 

4.  At  what  nominal  rate  of  interest  per  annum  will  $200  amount  to  $268.78 
in  5  years,  if  interest  is  converted  semiannually? 

6.  At  what  rate  of  interest  will  any  principal  double  itself  in  10  years? 

Time.  By  applying  the  principles  of  logarithms,  the  time  may 
l)e  computed  if  the  principal,  the  amount,  and  the  rate  are  given. 

Procedure :  (a)  Determine  the  compound  amount  of  1  by  divid- 
ing the  compound  amount  by  the  principal. 

(6)  Determine  the  log  of  the  compound  amount  of  1. 

(c)  Determine  the  log  of  the  ratio  of  increase. 

(d}  Divide  (6)  by  (c),  to  determine  the  time  in  periods. 

Example 

If  $100  placed  at  interest  at  6%,  compounded  annually,  amounts  to  $126.25, 
what  is  the  time  of  the  investment? 

Formula  Arithmetical  Substitution 

.        ,  Amount  .        f  126.25 


log  of  (1  +  t)  log  of  (1.06) 


(b) 

1,000 

3,207.14 

6% 

(c) 

200 

533.17 

4% 

(d) 

40 

62.32 

3% 

(e) 

500 

1,621.70 

4% 

CO 

300 

609.84 

6% 

(g) 

100 

200.00 

5% 

324  COMPOUND  INTEREST 

Solution 

126.25  -4-  100  =  1.2625,  compound  amount  of  1  at  6%  for 

n  periods 

log  1.2625  =  0.101231 
log  1.06  =  0.025306 
0.101231  -*-  0.025306  =  4,  time  in  periods,  or  4  years 

Problems 
Compute  the  time  in  each  of  the  following: 

Principal      Amount        Rate        Convertible 
(a)       $    100       $    240.66       5%          Annually 


Semiannually 


Compound  amount  for  fractional  part  of  conversion  period. 

In  the  problems  thus  far,  the  time  contained  an  exact  number  of 
conversion  intervals.  How  shall  compound  interest  be  computed 
when  there  is  a  fractional  part  of  an  interest  period,  for  example,  if 
the  time  is  4  years,  2  months,  interest  at  6%  convertible  semi- 
annually? 

In  actual  practice,  simple  interest  is  customarily  used  for 
fractions  of  an  interest  period.  In  the  example  above:  (a)  com- 
pound interest  would  be  computed  for  8  years  at  3%;  (6)  simple 
interest  would  be  computed  on  the  amount  found  in  (a)  at  3  %  for 
2  months;  and  (c)  the  sum  of  the  answers  found  in  (a)  and  (6) 
would  be  the  amount  due. 

Problems 

1.  Find  the  compound  amount  of  $250  for  3  years  and  3  months,  interest  at 
5%  converted  annually. 

2.  Find  the  compound  amount  of  $1,575  for  4  years  and  3  months,  interest  at 
5%  converted  semiannually. 

3.  Find  the  present  value  of  $2,750  due  in  2  years,  8  months,  if  the  interest 
rate  is  6%,  compounded  semiannually. 

Review  Problems 

1.  How  much  money  will  have  accumulated  after  8  years  if  $500  is  invested 
now  at  4%  converted  quarterly? 

2.  Calculate  the  value  of  vb  at  3%  by  dividing  1  by  (1.03)5  as  shown  by  the 
compound  amount  table.     Compute  the  value  of  t;6  at  3%  by  logarithms. 

3.  If  the  annual  interest  rate  is  5%,  what  is  the  corresponding  rate  of 
discount?     f  HINT:  d  =*      l    .•  J 


COMPOUND  INTEREST  325 

4.  How  long  will  it  take  money  to  double  itself  when  invested  at  3  %  con- 
verted annually? 

5.  What  nominal  rate,  convertible  monthly,  is  equivalent  to  4%  a  year 
effective? 

6.  Find  the  present  value  of  a  debt  of  $750  due  in  5  years,  if  the  current 
rate  of  interest  is  6%  convertible  monthly. 

7.  If  you  can  get  4%  converted  annually,  how  much  will  you  need  to  invest 
to  accumulate  $7,500  in  15  years? 

8.  Find  the  present  value  of  $1,250  due  in  4  years,  6  months,  with  interest 
at  4%  convertible  semiannually. 

9.  At  age  42,  a  man  has  $6,725  to  invest.     What  interest  rate,  converted 
annually,  must  he  receive  in  order  for  the  investment  to  accumulate  to  $12,500 
at  age  60? 

10.  Calculate  by  means  of  logarithms  the  present  value  of  $382.45  due  in 
5  years  without  interest,  if  money  is  worth  4%  effective. 


CHAPTER  31 
Ordinary  Annuities 

Definition.  An  annuity  is  a  series  of  equal  payments  made  at 
equal  intervals  of  time.  Examples  of  annuities  are:  premiums  on 
life  insurance,  interest  payments  on  bonds  or  mortgages,  rentals  of 
property,  pensions,  sinking  fund  payments,  regular  preferred  stock 
dividends,  and  so  forth. 

The  word  annuity  suggests  annual  payments,  but  the  broad 
meaning  of  the  term  is  a  series  of  equal  payments  made  at  equal 
stated  intervals,  whether  these  intervals  are  one  year,  six  months, 
three  months,  or  any  other  period  of  time. 

Kinds  of  annuities.     Annuities  are  of  two  kinds : 

(1)  Ordinary  annuity.     This  is  a  series  of  payments  where  each 
periodical  payment  is  made  at  the  end  of  a  period. 

(2)  Annuity  due.     This  is  a  series  of  payments  where   each 
periodical  payment  is  made  at  the  begining  of  the  period. 

Ordinary  annuities  will  be  discussed  in  this  chapter  and  annui- 
ties due  in  the  next  chapter. 

Rent  of  an  annuity.  The  periodic  payments  are  known  as 
"rents,"  arid  the  single  periodic  payment  is  represented  by  the 
symbol  R. 

Amount  of  an  ordinary  annuity.  The  amount  of  an  annuity  of 
1  for  any  number  of  periods  (n)  is  represented  by  sn  (read  "s  sub  n  "). 
This  symbol  used  in  conduction  with  the  rate  of  interest  becomes 
sn|t ;  for  example,  s^5%  represents  the  amount  of  an  annuity  of  1  a 
period,  for  10  periods,  at  5%.  The  amount  of  an  ordinary  annuity 
is  the  sum  at  the  end  of  the  term  of  all  the  periodic  payments,  plus 
the  interest  on  all  payments. 

Analysis  of  compound  interest.  To  understand  annuities,  it  is 
necessary  to  know  how  compound  interest  tables  are  built  up. 
Take  (1  +  i)n  =  (LOG)4.  This  amount  can  be  found  in  the  6% 
column  of  a  compound  interest  table,  the  fourth  number  from  the 
top.  An  analysis  shows  that  it  is  composed  of  three  elements:  a 
principal  of  1;  an  annual  addition  of  .06  simple  interest;  and 
"interest  on  interest"  of  .02247696.  A  further  analysis  may  be 
made  as  shown  on  the  next  page. 

327 


328 


ORDINARY  ANNUITIES 


End  of     End  of     End  of       End  of 
Initial        1st          2nd          3rd  4th 

Payment   Period     Period     Period        Period  Total 

Invested $1.00  $1.00 

1st  annual  interest 

on  1 .06 

Interest  on  .06....  .0036  .0036    0036 

Interest  on  1st 

.0036  .000216  000216 

Interest  on  2nd 

.0036 .000216 

Interest  on  .000216  .00001296       .07146096 

2nd  annual  interest 

on  1 .06 

Interest  on  06  .    .  .0036         0036 

Interest  on  .0036  .  .000216  .067416 

3rd  annual  interest 

on  1 .06 

Interest  on  .06 ....  .  0036  . 0636 

4th  annual  interest 

on  1 .06  .06 

Total 1  26247696 

When  the  above  tabulation  is  summarized,  three  distinct  parts 
appear: 

1 .  The  principal $1 . 00 

2.  The  four  equal  annual  amounts  of  simple  interest, 

$.06 24 

3.  The  accumulations  of  " interest  on  interest" 02247696 

Total $1  26247696 

This  may  be  further  reduced  to : 

1.  Principal $1  00 

2.  Compound  interest 26247696 

3.  Compound  amount $f  26247696 

Relation  of  compound  interest  and  annuities.  In  the  above 
table  of  analysis,  it  can  be  seen  that  there  is  a  series  of  payments  of 
$.06  each  at  regular  stated  intervals  of  1  year,  and  that  compound 
interest  is  calculated  on  each  of  these  $.06  payments  until  the  end 
of  the  fourth  year.  Therefore,  .26247696  is  the  amount  of  an 
annuity  of  .06  for  4  years  at  6%. 

An  annuity  of  1  may  be  computed  from  this  result  as  follows: 

.26247696  -r-  6  =  .04374616,  amount  of  an  annuity  of  .01  for 

4  years  at  6% 

.04374616  X  100  =  4.374616,  amount  of  an  annuity  of  1  for  4  years 
at  6% 

Procedure  in  computing  the  amount  of  an  annuity.  From  the 
foregoing  discussion  may  be  derived  the  following  general  pro- 


ORDINARY  ANNUITIES  3*9 

cedure  for  computing  the  amount  of  an  annuity  for  any  given 
number  of  periods  at  any  stated  interest  rate : 

Procedure :  (a)  Calculate  the  compound  amount  at  the  periodic 
rate  and  for  the  number  of  periods  given,  or  obtain  the  compound 
amount  from  a  compound  amount  table,  (1  +  i)n  =  s. 

(b)  Deduct  1  from  the  compound  amount,  5  —  1  =  7. 

(c)  Divide  the  compound  interest  on  1  by  the  rate  per  cent 
expressed  decimally,  to  obtain  the  amount  of  the  annuity  of  1, 

(d)  Multiply  the  amount  of  the  annuity  of  1  by  the  number  of 
dollars  of  each  annuity  rent,  R  X  s-^  —  S. 

Example 

It  is  desired  to  find  the  amount  of  an  ordinary  annuity  of  $100  for  5  years 
at  6%. 

Solution 

1.338226  =  compound  amount  of  1  at  6%,  or  (1.06)5 
1.338226  -  1  =  .338226,  compound  interest  on  1  at  6% 
.338226  -s-  .06  =  5.6371,  amount  of  annuity  of  1 
$100  X  5.6371  =  $563.71,  amount  of  annuity  of  $100  for  5  years 

at  6% 

From  the  above,  the  following  may  be  derived: 

(1  +  i)»,  or  (1.06)5  =  1.338226,  compound  amount,  or  s. 
(1  +  i)n  -  1,  or  (1.06)5  -  1  =  .338226,  compound  interest,  or  7. 

-. >  or     -'- — — =  5.6371,  amount  of  an  annuity  of  1,  or  s-\ . 

Rsn^  =  S        or        $100  X  5.6371  =  $563.71 

Verification 

Rent  paid  at  end  of  first  year $100.00 

Interest  at  6%  on  $100     $     6. 00 

Rent  paid  at  end  of  second  year 1 00 . 00     106 . 00 

Amount  of  annuity  for  2  years $206.00 

Interest  at  6%  on  $206 $  12.36 

Rent  paid  at  end  of  third  year  ...      .    .    100 . 00     1 12  36 

Amount  of  annuity  for  3  years .          $318.36 

Interest  at  6%  on  $318.36. ...                    .      ...  $  19. 10 

Rent  paid  at  end  of  fourth  year                        .    .  100.00     11910 

Amount  of  annuity  for  4  years             $437 . 46 

Interest  at  6%  on  $437.46     ..             $2625 

Rent  paid  at  end  of  fifth  year. .                    ....  100.00     126.25 

Total $563.71 

Semiannual  or  quarterly  basis.  If  the  rents  are  payable  every 
six  months,  or  every  three  months,  and  the  interest  is  to  be  com- 
pounded on  the  same  dates,  the  method  of  using  the  rate  per  period 


330 


ORDINARY  ANNUITIES 


and  the  time  in  periods  is  preferable  to  the  method  of  finding  the 
effective  interest  for  1  year  and  using  the  time  in  years.  Only  the 
former  method  will  be  illustrated. 

Procedure:  (a)  Find  the  nominal  rate  per  period,  — 
(6)  Find  the  number  of  periods,  mn. 

(j\mn 
1  H — j    . 

(d)  Determine  the  compound  interest  on  1  for  the  number  of 

(j  \mn 
1  H — )      —  1  =  /. 

(e)  Divide  the  compound  interest  found  in  (d)  by  the  nominal 

j 
rate  per  cent  per  period,  /  -f-  —  =  s-|t. 

(/)  Multiply  the  amount  of  the  annuity  of  1  by  the  number  of 
dollars  of  each  periodic  rent,  Ksr^. 

Example 

What  will  be  the  amount  of  an  ordinary  annuity  of  8  rents  of  $50  each, 
payable  every  6  months,  interest  at  6%  per  year,  compounded  semiannually? 

Formula  Arithmetical  Substitution 


Rs-t   =  ,Sf 


Solution 

4X2  =  8,  number  of  periods 
.06  4-  2  =  .03,  rate  per  cent  per  period 
(1.03)8  =  1.26677,  1  at  compound  interest  for  8  periods 
1.26677  -  1  =  .26677,  compound  interest  on  1  for  8  periods 
.26677  -T-  .03  =  8.8923,  amount  of  annuity  of  1  for  8  periods,  or  s^3%.* 
$50  X  8.8923  =  $444.62,  amount  of  annuity  of  $50  for  8  periods 

Problems 

1.  Find  the  amount  of  an  ordinary  annuity  of: 

(a)  $200  at  5%  for    4  years,  rents  and  interest  payable  annually. 

(b)  $120  "  4%          6       ' 

(c)  $250  "  3%        20  "        semiannually. 

(d)  $250  "  6%        10       '          '        '  u  tt 

(e)  $500  "  4%        30       '  '  "       annually. 
(/)  $100  "  6%        40       '           ' 

2.  Construct  a  table  of  analysis  of  the  amount  of  an  annuity  of  1  for  5  years 

at  4%. 

3.  If  for  5  years  $250  is  deposited  at  the  end  of  every  six  months  in  a  bank 
paying  3^%,  interest  converted  semiannually,  what  will  be  the  amount  credited 
to  the  account  at  the  end  of  the  term?     Set  up  a  schedule  showing:  (a)  number 


The  value  of  «^J%  may  be  found  in  Table  4,  page  527. 


ORDINARY  ANNUITIES  331 

of  periods;  (6)  amount  deposited;  (c)  interest  each  period;  (d)  amount  to  be 
added  to  the  account;  (e)  balance  of  the  account  each  period. 

4.  A  purchases  a  house,  and  agrees  to  pay  $60  each  month  for  1  year.  If 
money  is  worth  6%,  interest  compounded  monthly,  what  sum  paid  in  one 
amount  at  the  end  of  the  year  would  be  the  equivalent  of  A's  total  monthly 
payments? 

Rent  of  an  ordinary  annuity.  Frequently  the  amount  of  an 
ordinary  annuity  is  known  and  it  is  desired  to  find  the  periodic 
rent,  as  in  problems  of  sinking  funds. 

Procedure :  (a)  Compute  the  amount  of  the  annuity  of  1  for  the 
given  number  of  periods  at  the  given  rate  per  period,  s  . 

(6)  Divide  the  number  of  dollars  of  the  required  amount  by  the 
amount  of  an  annuity  of  1.  The  result  will  be  the  rent,  or  periodic 
payment,  P  -r-  sn-{  =  R. 

Example 

What  should  be  the  amoant  of  each  equal  annual  payment  into  a  fund  which, 
in  4  years  at  6%,  interest  compounded  annually,  is  to  amount  to  $1,000? 

Solution 
P  •*-  s-,.  =  R        1000  +  slv  =  R 

n  \  416/0 

In  Table  4  it  will  be  found  that 

SV|6%  =  4-374616,  amount  of  annuity  of  1  for  4  years 

at  6% 

$1,000  -T-  4.374616  =  $228.59,  rent  required  to  accumulate  $1,000 
at  the  end  of  4  years 

Verification 

Rent  at  end  of  first  year $  228  59 

Interest  on  $228.59  for  1  year  at  6%.  13  72 

Rent  at  end  of  second  year ....  228  59 

Amount  of  annuity  at  the  end  of  2  years $  470  90 

Interest  on  $470.90  for  1  year  at  6%  *  28.25 

Rent  at  end  of  third  year 228  59 

Amount  of  annuity  at  the  end  of  3  years $  727  74 

Interest  on  $727.74  for  1  year  at  6%  43  66 

Rent  at  end  of  fourth  year 228  60 

Amount  of  annuity  at  the  end  of  4  years  at  6  %  ....  $1 ,000  00 

TABLE  OF  AMOUNT  OF  ANNUITY 


(1) 

End  of 
Period 
1 

(2) 

Rent 
$228  59 

(3) 
Interest 
Accumulation 

$ 

(4) 
Addition  to 
Principal 
$228  59 

(5) 
Total 
Amount 
$    228  59 

2 
3 
4 

228  59 
228  59 
228.60 

13  72 
28.25 
43.66 

242  31 
256.84 
272.26 

470.90 
727.74 
1,000.00 

332  ORDINARY  ANNUITIES 

Problems 

1.  A  savings  bank  pays  2%,  interest  compounded  quarterly.     How  much 
must  be  deposited  at  the  end  of  each  quarter  in  order  to  accumulate  $400  at  the 
end  of  2  years?    Prepare  a  table  for  verification. 

2.  A  company  owes  $600,  due  in  4  years.     How  much  must  be  set  aside 
semiannually  at  4%,  interest  compounded  semiannually,  to  accumulate  to  the 
amount  of  the  debt  at  maturity?     Prepare  formula,  solution,  and  table. 

3.  A  company  issued  bonds  for  $30,000,  due  in  10  years.     Interest  is  at  5%, 
compounded  quarterly.     How  much  must  the  company  set  aside  every  three 
months  in  order  to  be  able  to  meet  the  payments  on  the  bonds  when  they  become 
due? 

4.  A  company  has  a  debt  of  $20,000,  due  at  the  end  of  10  years.     Money  is 
worth  5  %,  interest  compounded  annually.     How  much  must  be  set  aside  annually , 
to  accumulate  to  the  amount  of  the  debt? 

5.  At  the  age  of  30,  Y  decides  that  he  ought  to  deposit  in  the  bank,  every 
three  months,  an  amount  which  will  have  accumulated  to  $25,000  by  the  time 
he  is  55.     The  bank  allows  him  4%,  interest  compounded  quarterly.     What  is 
the  amount  of  F's  quarterly  deposits? 

Use  of  effective  interest  in  annuities.  Very  often  the  rents  are 
paid  annually,  and  the  interest  is  compounded  semiannually  or 
quarterly;  when  such  is  the  case,  the  effective  late  of  interest  must 
be  used. 

If  the  interest  is  compounded  more  or  less  frequently  than  the 
rents  are  paid,  it  is  necessary  to  convert  the  nominal  interest  rate 
to  the  effective  interest  rate  applicable  to  the  rent  periods. 

Procedure:  (a)  Calculate  the  effective  periodic  interest  on  the 

(j\m 
1  +  —  I     —  1. 
m) 

(fe)  Calculate  the  amount  of  an  annuity  of  1,  using  the  effective 
rate  per  period;  the  number  of  periods  corresponds  to  the  number 
01  rents,  s~,.. 

7      nil 

(c)  Multiply  the  amount  of  an  annuity  of  1,  found  in  (6),  by 
the  number  of  dollars  of  each  rent,  Rs^.  =  8. 

;          nit 


Example 

What  will  be  the  amount  of  an  annuity,  the  annual  payments  of  which  are 
$100  for  4  years  at  6%,  interest  compounded  quarterly? 


Solution 
The  solution  to  this  example  will  be  stated  in  two  parts 

(1)  Calculation  of  the  effective  rate  per  year. 

(2)  Calculation  of  the  amount  of  the  annuity  of  $100. 


ORDINARY  ANNUITIES  333 

PART  1 
Formula  Arithmetical  Substitution 

I1  +  ")     ~  l  =  Effective  rate  (  1  +  ^J    -  1  =  .0613635 

.06  -f-  4  =  .015,  rate  per  period 

1  H-  .015  =  1.015,  ratio  of  increase 

(1.015)4  =  1.0613635,  compound  amount  of  1  for  1  year 

1.0613635  -  1  =  .0613635,  effective  rate  per  year 

PART  2 

Use  the  effective  rate  found  in  Part  1,  and  proceed  as  in  the  examples  previ- 
ously given. 

Formula  Arithmetical  Substitution 


(1.0613635)4  =  1.2689855,  compound  amount  of  1  for  4 

periods  at  0  1  363,")  '  [  * 
1.2689855  -  1  =  .2689855,  compound  interest  on  1  for  4 

periods  at  6.13635% 
.2689855  -f-  .0613635  =  4.383477,  amount  of  annuity  of  1  for  4 

periods  at  •    .•>•  ••>•"> 

$100  X  4.383477  =  $438.35,  amount  of  annuity  of  $100  for 
4  years 

Verification 

First  year: 

Rent  at  end  of  year  .......................  $100  00 

Second  year: 

$100  X  .0613635  (effective  rate)  .............   $     614 

Rent  ....................................  100  00  106  14 

Amount  of  annuity  for  2  years  .............  $206.  14 

Third  year: 

$206.14  X  .0613635  ..................   $1265 

Rent  ................................    JOO  00     H2^  65 

Amount  of  annuity  for  3  years  ......  .  $318.79 

Fourth  year: 

$318.79  X  .0613635  .........     $  19  56 

Rent  .........................    100  00  119  56 

Amount  of  annuity  for  4  years       .          ......  $438  35 

Problems 

1.  Barlow  has  a  5-year  annuity  for  which  the  payments,  made  at  the  end 
of  each  year,  are  $300  each.     Interest  is  at  4%,  compounded  semiannually. 
What  is  the  amount  of  the  annuity?    Prepare  proof  of  answer. 

2.  Ware  desires  to  know  how  much  he  will  have  in  the  savings  bank  at  the 
end  of  25  years  if  he  deposits  $150  at  the  end  of  each  six  months.     The  bank  pays 
4%,  and  the  interest  is  compounded  at  the  end  of  each  quarterly  period. 

*  The  compound  amount  of  1  for  4  periods  at  6.13635%  is  the  same  as  the  com- 
pound amount  of  1  for  16  periods  at  1£%,  and  (1.015)16  is  readily  found  in  the  com- 
pound amount  table  given  in  Appendix  III. 


334  ORDINARY  ANNUITIES 

3.  Deposits  of  $500  are  made  at  the  end  of  each  year  for  20  years.  If  the 
bank  credits  the  account  with  quarterly  interest  at  4%  (nominal  rate),  what 
will  be  the  amount  of  the  accumulation? 

Sinking  fund  contributions.  A  sinking  fund  produced  by 
equal  periodic  payments  accumulating  at  compound  interest  is  one 
type  of  annuity.  The  principles  applicable  to  annuities  will  be 
further  illustrated  with  special  reference  to  sinking  funds. 

To  find  the  rent  of  a  sinking  fund,  divide  the  number  of  dollars 
required  in  the  total  fund  by  the  amount  of  the  annuity  of  1  for  the 
specified  number  of  periods  at  the  given  rate,  P  -f-  s^.  =  R. 

Example 

A  company  borrowed  $2,500  for  5  years,  and  established  a  sinking  fund  to 
provide  for  the  payment  of  the  debt.  The  contributions  to  the  funcl  were  to 
be  made  at  the  end  of  each  year.  If  money  is  worth  6%,  what  should  be  the 
amount  of  each  annual  contribution? 

Solution 
P  +  s  .   =  R        $2,500  -5-  s,~  =  $443.49 

nit  0,6/0 

In  Table  4  it  will  be  found  that 

s -,  „  =  5.63709,  amount  of  ordinary  annuity  of  1  for 

6  |fi  /o 

5  periods  at  6% 
$2,500  -5-  5.63709  =  $443.49,  contribution  to  sinking  fund. 

TABLE  OF  ACCUMULATION  OF  SINKING  FUND  CONTRIBUTIONS 


(1) 

(2) 

(3) 

(4) 

(5) 

End  of 

Yearly 

Total 

Period 

Contribution 

Interest 

Increase 

Fund 

1 

$    443  49 

$...-  

$    443  49 

$    443  49 

2 

443  49 

26  61 

470  10 

913  59 

3 

443  49 

54  82 

498  31 

1,411  90 

4 

443  49 

84  71 

528  20 

1,940  10 

5 

443  49 

116  41 

559  90 

2,500  00 

$2,217.45      $282  55     $2,500  00 
Problems 

1.  A  company  establishes  a  sinking  fund  to  provide  for  the  payment  of  a 
debt  of  $8,000  maturing  in  4  years.     The  contributions  to  the  fund  are  to  be 
made  at  the  end  of  each  six  months.     Interest  at  4%  is  to  be  compounded 
semiannually.     What  must  be  the  amount  of  each  semiannual  contribution? 
Construct  a  table,  as  in  the  example  above. 

2.  A  debt  of  $30,000  is  due  in  4  years.     A  sinking  fund  is  to  be  established, 
and  contributions  are  to  be  made  at  the  end  of  each  six  months.     What  must 
be  the  amount  of  each  semiannual  contribution,  if  interest  at  4  %  is  compounded 
semiannually?    Construct  a  table,  as  in  the  example  above. 

3.  A  has  an  obligation  of  $8,000  maturing  in  3  years.     How  much  must  he 
set  aside  each  month  at  6%,  interest  compounded  monthly,  in  order  to  be  able 
to  pay  the  debt  when  due? 


ORDINARY  ANNUITIES  335 

Present  value  of  an  ordinary  annuity.  The  present  value  of  an 
ordinary  annuity  ^presented  by  the  symbol  a-,  is  the  sum  which, 
if  put  at  compound  interest,  will  produce  the  periodic  rents  of  the 
annuity  contract  as  they  become  due. 

First  method.  Procedure:  Find  the  present  value  of  each  peri- 
odic rent  separately ;  add  the  present  values  of  these  periodic  rents ; 
their  sum  is  the  present  value  of  the  annuity. 

Example 

Assume  that  it  is  desired  to  find  the  value,  at  the  beginning  of  the  first  period, 
of  a  series  of  four  annual  periodic  payments  of  Off  each.  Respective  payments 
are  to  be  made  at  the  end  of  each  year. 

PRESENT  VALUE  AT  DATE  OF  CONTRACT 
1st  rent,  payable  at  end  of  1st  year  ...   .06  X  -  =  .00  X  .94339  =    056003 

2nd  rent,  payable  at  end  of  2nd  year...    .00  X  77-7^,  =  -06  X  .SS999  =   .053399 
3rd  rent,  payable  at  end  of  3rd  year. . .   .00  X  7^7.77,  =  -00  X  .83962  =  .050377 

4th  rent,  payable  at  end  of  4th  year. . .    .00  X  7777,77-4  =  -OG  X  .79209  =  .047525 

( 1  .Uo)  —    - 

Present  value  of  an  annuity  of  .00  =     207004 

Second  method.  It  will  he  noted  that  the  present  value  at  6% 
of  an  annuity  of  4  rents  of  6^  each  is  the  same  as  the  compound 
discount  on  1  for  4  periods  at  6%.  The  computation  of  the  com- 
pound discount  on  1  for  4  periods  at  the  rate  of  6  %  is  as  follows : 

(l.OO)4  =  1.202477,  compound  amount  of  1  for  4  periods  at  0% 
1  -r-  1.202477  =     .792094,  present  value  of  1  for  4  periods  at  0% 
1  -    .792094  =    .207904,  compound  discount  on  1  for  4  periods  at  6% 

Substituting  symbols  for  figures,  it  is  apparent  that  the  present 
value  of  an  annuity  for  n  periods,  the  rents  of  which  when  stated 
in  cents  are  the  same  as  z,  is  equal  to  the  compound  discount  on  1 
for  n  periods  at  the  rate  of  i. 

In  the  example  just  given,  the  basis  of  calculation  is  the  periodic 
payment  of  6^,  and  this  is  stated  as  "an  annuity  of  6^.M  How- 
ever, the  basis  most  frequently  used,  or  the  common  basis,  is  1, 
and  is  expressed  as  "an  annuity  of  1."  (Annuity  tables  are  built 
on  this  basis.)  Therefore,  in  order  to  find  the  present  value  of  an 
annuity  of  1,  divide  the  compound  discount  by  the  rate  per  cent 
expressed  decimally.  Using  the  figures  given  above,  the  calcula- 
tion would  be:  .207904  -f-  .06  =  3.465105,  or  the  amount  of  an 
annuity  of  1  for  4  periods  at  6%. 


336  ORDINARY  ANNUITIES 

Procedure:  (a)  Calculate  the  compound  discount  on  1  for  the 
required  number  of  periods  and  at  the  required  rate  per  cent, 
1  -  t;n  =  D. 

(b)  Divide  the  compound  discount  by  the  given  rate  per  cent, 
expressed  decimally.     The  quotient  will  be  the  present  value  of  an 
annuity  of  1,  D  -5-  i  =  a~}.. 

(c)  Multiply  the  present  value  of  the  annuity  of  1  by  the  num- 
ber of  dollars  of  each  rent,  R  X  a-^  =  A. 

Example 

The  terms  of  an  annuity  contract  call  for  the  payment  of  $100  at  the  end 
of  each  year  for  4  years.  If  money  is  worth  6%,  interest  compounded  annually, 
what  is  the  present  value  of  the  annuity  contract  at  the  beginning  of  the  first 


year? 


Formula  Arithmetical  Substitution 

1 


Ra  ,    =  A  100 


(LOG)4 


=  $346.51 


.06 
Solution 
(1.06)4  =  1.262477,  compound  amount  of  1  for  4  years  at 

<)% 

1  -r-  1.262477  =  .792094,  present  value  of  1  for  4  years  at  6% 
1  -  .792094  =  .207906,   compound  discount  on   1   for  4  years 

at  6% 
.207906  4-  .06  =  3.4651,  present  value  of  an  annuity  of  1   for  4 

years  at  6%,  or  the  value  of  «^6^.* 
3.4651  X  $100  =  $346.51,  present  value  of  an  annuity  of  $100 

Verification 
Beginning  of  first  year: 

Present  value  of  contract $346  51 

End  of  first  year: 
Deduct: 

Rent $100.00 

Interest  on  $346.51  at  6%       20  79 

Reduction  in  value  of  annuity  contract  79  21 

Present  value $267~30 

End  of  second  year: 
Deduct: 

Rent $100  00 

Interest  on  $267.30 __  16  04 

Reduction  in  value  of  annuity  contract ....  83 . 96 

Present  value $183  34 


*  The  value  of  a4{e%  may  be  obtained  directly  from  Table  5,  page  532. 


ORDINARY  ANNUITIES  337 

End  of  third  year: 
Deduct: 

Rent $100.00 

Interest  on  $183.34 11.00 

Reduction  in  value  of  annuity  contract.  89.00 

Present  value $  94  34 

End  of  fourth  year: 
Deduct: 

Rent $100  00 

Interest  on  $94.34 5  66 

Reduction  in  value  of  annuity  contract   ...  94  34 

$~(TOO 

Amortization.  Payments  made  on  the  principal,  as  shown  in 
the  above  example,  are  known  as  amortization  payments.  Amor- 
tization is  the  gradual  repayment  of  the  principal  through  the 
operation  of  the  two  opposing  forces  of  compound  interest  and 
periodic  payments.  Compound  interest  increases  the  principal, 
while  the  payments  reduce  it.  In  the  verification  above,  it  can 
be  seen  that  the  excess  of  each  payment  over  the  interest  for 
the  period  is  the  amount  by  which  the  principal  is  reduced.  The 
amount  of  this  reduction  is  the  amortization. 

Problems 

1.  Find  the  present  value  of  each  of  the  following  ordinary  annuities: 

Rents             Paid  Interest  Years 

(a)       $400  Annually                 3%  6 

(6)       $225  Annually                 47o  10 

(c)  $350  Annually                4%  10 

(d)  $255  Semiannually         6%  3 

(e)  $340  Semiannually         7%  3 

2.  By  the  terms  of  an  annuity  contract,  $500  is  to  be  paid  at  the  end  of  each 
six  months  for  4  years.     Money  is  worth  6%,  interest  compounded  Semiannually. 
(a)  Find  the  present  value  of  the  annuity.     (6)  Submit  solution  and  verification 
showing  the  applications  each  year  of  the  payments  as  to  interest,  amortization 
of  principal,  and  new  principal  against  the  present  value  of  the  annuity. 

3.  What  is  the  value  at  the  beginning  of  the  first  period  of  an  annuity  of  $300 
payable  at  the  end  of  each  year  for  10  years,  money  being  worth  4%,  interest 
compounded  Semiannually?     Prepare  columnar  table. 

4.  A  man  purchases  a  house  for  $1,800  cash  and  sixteen  notes  of  $400  each, 
without  interest,  one  due  at  the  end  of  each  six  months  until  all  the  notes  are 
paid.     If  money  is  worth  4%,  interest  compounded  serniannuaily,  what  is  the 
cash  value  of  the  property? 

5.  What  is  the  cash  value  of  a  contract  which  calls  for  the  payment  of  $50  at 
the  end  of  each  month  for  5  years,  if  money  is  worth  6%,  interest  convertible 
monthly? 

6.  A  disability  insurance  contract  provides  that  the  insured  may  choose 
one  of  the  following  options:  (a)  $50  a  month,  payable  at  the  end  of  each  month. 


338  ORDINARY  ANNUITIES 

for  48  months;  (6)  $500  cash,  and  $50  a  month  for  36  months;  (c)  $100  a  month 
for  12  months,  and  $50  a  month  for  the  ensuing  26  months.  If  money  is  worth 
6%,  interest  compounded  semiannually,  which  is  the  best  option? 

Computation  of  the  rents  or  periodic  payments  of  the  present 
value  of  an  ordinary  annuity.  If  the  present  value  of  an  annuity, 
the  rate  per  cent,  and  the  time  are  given,  the  rents  or  periodic 
payments  may  be  calculated  as  follows: 

Procedure:  (a)  Determine  the  present  value  of  an  annuity  of  1 
for  the  required  number  of  periods  at  the  given  rate  per  period, 
or  a  .. 

n  i 

(b)  Divide  the  given  present  value  of  the  annuity  by  the  present 
value  of  the  annuity  of  1  found  in  (a),  A  -f-  a  —  R. 

Example 

What  annual  rent  will  be  produced  by  an  ordinary  annuity  the  present 
value  of  which  is  $346,51,  if  there  are  four  rents,  and  money  is  worth  6 %? 

Formula  Arithmetical  Substitution 

-i-fi.  ^^-=$100.00. 

1  -      1_ 
(1.06)_4 

.06     ~~ 
Solution 

(LOG)4  =  1.262477,  compound  amount  of  1  for  4  years 

at  6% 

1  -f-  1.2G2477  =  .792004,  present  value  of  1  for  4  years  at  6% 
1  —  .792094  =  .207900,  compound  discount  on  1  for  4  years 

at  6% 
.207906  -T-  .06  =  3.4651,  present  value  of  an  annuity  of  1  for 

4  years  at  6%,  or  value  of  a       .* 
$346.51  -T-  3.4651  =  $100,  rent  of  annuity 

See  verification  shown  on  pages  336  337. 

Problems 

1.  If  the  present  value  of  an  annuity  contract  is  $6,000,  what  amount  must 
be  paid  at  the  end  of  each  year  for  10  years  to  cancel  the  obligation,  money  being 
worth  4%,  interest  compounded  annually? 

2.  An  annuity  contract  is  worth  $12,000  at  the  present  date.     If  the  time  to 
maturity  is  10  years,  and  money  is  worth  3%,  interest  compounded  semiannually, 
what  periodic  payment  must  be  made  at  the  end  of  each  six  months  to  cancel 
the  contract  in  10  years? 

3.  The  present  value  of  a  12-year  annuity  contract  is  $8,000.     If  money  is 
worth  4%,  interest  convertible  quarterly,  what  amount  must  be  paid  at  the 
end  of  each  quarter  to  cancel  the  contract  in  12  years? 


*  This  value  is  readily  found  in  Table  5,  page  532. 


ORDINARY  ANNUITIES  339 

Payment  of  debt  by  installments.  In  many  contracts  it  is 
agreed  that  the  principal  of  the  debt  together  with  the  interest  are 
to  be  paid  in  equal  periodic  payments;  each  payment  is  to  cancel 
the  interest  due  to  date,  and  the  balance  is  to  be  applied  toward  the 
repayment  of  the  principal. 

The  procedure,  formula,  and  solution  are  similar  to  those  given 
on  page  338. 

Example 

If  Smith  borrows  $1,000  from  Jones  at  0%,  and  agrees  to  cancel  the  debt 
(principal  and  interest)  in  five  equal  annual  payments,  what  will  be  the  amount 
of  each  payment? 

The  formula  for  "rent  of  present  value  of  annuity,"  given  on  page  338,  is 
applicable. 

TABLE  OF  INSTALLMENT  PAYMENTS 


(1) 

End  of 
Period 
1 

(2) 
Payment 
Made 
$    237  30 

(3) 
To  Cancel 
Interest 
$  (>0  00 

(4) 
Amortization 
of  Principal 

$177  39 

(5) 
Balance  of 
Principal 
$1,000  00 

2 

237  39 

49  3(5 

18S  03 

822  61 

3 

237  39 

3S  07 

199  32 

634  58 

4 

237  39 

2tt  11 

211   28 

435  26 

5 

Totals 

237  39 

13  44 

223  95 

223  98 

$1,1  SO  95 

SING  98 

$999  97 

$            03 

A  slight  error,  such  as  that  in  the  above  table,  is  likely  to  occur 
in  many  problems  in  installment  payments;  it  is  caused  by  the  fact 
that  the  computations  are  not  carried  to  a  sufficient  number  of 
decimal  places.  In  such  cases  the  difference  should  be  corrected  in 
the  last  payment.  In  the  table,  the  fifth  payment  should  be 
$237.42,  instead  of  $237.39.  This  would  leave  column  (5)  with  no 
remainder,  and  column  (4)  would  have  a  total  of  $1,000. 

Problems 

1.  A  contracts  for  the  purchase  of  a  house,  and  agrees  to  pay  for  it  in  install- 
ments of  equal  amounts  over  a  period  of  10  years.     The  cash  value  of  the  house 
is  $10,000.     If  money  is  worth  5%,  interest  convertible  quarterly,  what  should 
he  the  amount  of  each  payment? 

2.  What  annuity,  payable  quarterly  for  20  years,  would  be  required  to  repay 
a  loan  of  $12,840,  the  nominal  rate  of  interest  being  4%  per  annum? 

3.  A  debt  of  $3,500,  with  interest  at  5%,  compounded  semiannually,  will  be 
discharged,  principal  and  interest,  by  equal  payments  at  the  end  of  each  six 
months  for  10  years.     Determine  the  amount  of  each  payment. 

4.  A  house  cost  $15,000.    The  purchaser  paid  $3,000  cash,  and  agreed  to 
pay  the  balance  in  equal  quarterly  payments,  principal  and  interest,  over  a 
period  of  8^r  years.     If  money  is  worth  6%.  interest  convertible  quarterly,  what 
is  the  amount  of  each  payment? 


340  ORDINARY  ANNUITIES 

6.*  A  company  purchased  machinery  on  December  1,  at  a  cost  of  $40,000. 
Twenty-five  per  cent  of  the  cost  was  paid  in  cash,  and  the  balance  is  to  be  paid 
in  60  monthly  installments  of  equal  amount.  The  monthly  payments  are  to 
be  represented  by  notes,  payable  on  the  first  day  of  each  month  and  secured 
by  chattel  mortgage.  Interest  at  6%  per  annum,  or  £  of  1%  per  month,  is  to 
be  included  in  the  notes.  Compute  the  amount  of  each  note,  taking  the  com- 
pound interest  on  $1  as  .348850  for  the  given  time  and  rate. 

Computation  of  the  term  of  an  annuity.  When  the  rate  per 
cent,  the  amount  of  the  annuity,  and  the  size  of  each  periodic  pay- 
ment are  stated,  it  is  possible  to  calculate  the  number  of  periodic 
payments  to  be  made. 

In  calculating  the  time,  it  is  necessary  to  resort  to  equations 
and  logarithms. 

Procedure:  (a)  Divide  the  amount  of  the  annuity  by  the  num- 
ber of  dollars  in  one  of  the  periodic  payments,  to  find  the  amount 
of  the  annuity  of  1  at  the  given  rate. 

(b)  Multiply  the  amount  of  the  annuity  of  1  by  the  rate  per 
period  expressed  decimally,  to  find  the  compound  interest  on  1  for 
the  unknown  time. 

(c)  Add  1  to  the  compound  interest  on  1  to  find  the  compound 
amount  of  1. 

(d)  Determine  the  log  of  the  compound  amount  of  1  found 
in  (c). 

(e)  Determine  the  log  of  1  plus  the  rate  per  cent  for  the  period 
expressed  decimally;  that  is,  the  ratio  of  increase. 

(/)  Divide  the  log  of  the  compound  amount  of  1,  (d),  by  the  log 
of  the  ratio  of  increase,  (e),  to  find  the  number  of  periods,  or  the 
number  of  periodic  payments. 

Example 

The  amount  of  an  annuity  is  $1,318.08,  the  rent  is  $100  each  year,  and  the 
rate  is  6%.  Find  the  term. 

Formula 

/Amount  of  annuity  vy          \1 

XJ  _  Term 


log  (Ratio  of  increase) 

Arithmetical  Substitution 


log  (1.06) 
Solution 


10 


Dividing  by  rent:  1,318.08  -s-  100  =  13.1808 

Multiplying  by  rate:  13.1808  X  .06  =  .790848 

*  C.  P.  A.,  Ohio. 


ORDINARY  ANNUITIES  341 

Adding  1:  1  +  .790848  =  1.790848 

log  of:  1.790848  =  0.253059 

log  of:  1.06  =  0.025306 

Dividing:  0.253059  -r-  0.025306  =  10 

As  the  rents  are  to  be  paid  annually,  the  annuity  term  will  be  10  years. 

If  the  payments  are  to  be  made  more  often  than  once  each  year, 
the  rate  should  be  reduced  to  a  rate  per  period. 

Example 

A  purchases  a  house  for  $2,629.02,  and  agrees  to  pay  $500  down  and  $50  at 
the  end  of  each  month.  Each  payment  is  to  cancel  the  interest  to  date,  and 
the  balance  is  to  apply  against  the  principal.  The  debt  bears  6%  interest. 
How  many  months  will  it  take  A  to  pay  off  the  debt? 

Reducing  the  rate  to  the  rate  per  period,  .06  -f-  12  =  .005. 


Formula  Arithmetical  Substitution 


log 


"* 


-: =  Term 


X  .005 
50 


48 


log  (l  +  i)  log  1.005 

Solution 

Dividing:  2,129.02  ~  50  =  42.5S04 

Multiplying:  42.5804  X  .005  -  .212902 

Subtracting:  I  -  .212902  =  .787098 

Dividing:  1  -f-  .787098  =  1.270489 

log  of:  1.270489  =  0.1039709 

log  of:  1.005  =  0.0021661 

Dividing:  0.1039709  ~  0.0021661  =  48,  approx. 

Hence  48  monthly  payments  will  be  necessary  to  pay  off  the  debt. 

Problems 

1.  L.  Miller  purchased  a  house  and  lot  for  $7,800.     He  agreed  to  pay  $2,800 
cash,  and  $50  at  the  end  of  each  month  until  the  debt  should  be  paid.     How 
many  months  will  it  take  Miller  to  pay  the  debt,  if  each  payment  is  to  cancel 
the  interest  due  and  the  balance  is  to  apply  against  the  principal?     The  interest 
rate  is  6%. 

2.  B  sells  his  residence  for  $10,500.     lie  receives  a  down  payment  of  $2,500. 
The  balance  is  to  be  paid  on  the  basis  of  $75  each  month.     Each  monthly  pay- 
ment is  to  cancel  the  interest  first,  and  the  balance  is  to  apply  against  the  princi- 
pal.    The  contract  states  that  interest  is  at  the  rate  of  6%  per  annum.     How 
long  will  it  take  the  buyer  to  pay  off  the  debt? 

3.  Cole  buys  a  farm  for  $18,000,  and  agrees  to  pay  $6,000  down,  the  balance 
to  draw  interest  at  6%  until  paid.     Any  payments  made  are  to  apply  on  interest 
due  to  date,  and  if  the  payments  exceed  the  interest  due,  the  balance  is  to  be 
applied  to  the  reduction  of  the  principal.     Cole  desires  to  know  how  long  it 
will  take  him  to  pay  for  the  farm  if  he  makes  equal  quarterly  payments  of  $400. 

Use  of  effective  rate  in  annuities.     If  interest  is  compounded 
more  frequently  than  the  rents  are  paid,  or  vice  versa,  it  ie  neces- 


342  ORDINARY  ANNUITIES 

sary  to  reduce  the  rate  of  interest  to  an  effective  rate  for  a  perioc 
corresponding  to  the  periods  of  the  rent  payments. 

Procedure:  (a)  Calculate  the  effective  rate  of  interest  for  OIK 
rent  period. 

(6.-1)  I  f  the  amount  of  the  annuity  is  known,  use  the  procedun 
previously  given. 

(6.-2)  If  the  present  value  of  the  annuity  is  known,  use  the 
procedure  previously  ^iven. 

Example 

The  rents  of  $100  each  are  to  he  paid  annually,  the  amount  is  $318.64,  anc 
the  interest  rate  is  6%,  compounded  semiannually.     Find  the  term. 

PART  1 
Formula  A  rith  me  tic  a  I  Su  bstit  at  ion 

f  1  +  -M     -  1  =  KfTective  late  M  +  '^j    -  1  =  .0609 

Solution 

(  ^  ~^~    9   /    ~  1-0609,  effective  ratio  of  increase  for  1  year 
1  .0009  -  I   -  .0609,  effective  rate 
PART  2 

As  the  problem  states  the  amount  of  the  annuity,  it  may  be  solved  by  pro 
cedure  (6.-  1). 

Form  ula  +  1  rith  mctical  Substit  ution 


/_-!     ___    o 


. 

log  (Ratio  of  increase)  log  1.0609 

Solution 

Dividing:  31S.64  -5-  100  =  3.1864 

Multiplying:  3.1864  X  .0609  =  .1940517 

Adding  1:  1  +  .1940517  =  1.1940517 

log  of:  1.1940517  =  0.077022 

log  of:  1.0609  =  0.025674 

Dividing:  log  0.077022  -*-  log  0.025674  =  3 

The  result,  3,  indicates  that  there  are  three  annual  payments  of  $100  each. 

I  Aerification 

First  year: 

Rent  .......................  $100  00 

Second  year: 

$100  X  .0609     ..........................  $    6  09 

Rent  ..................................     100  00     106  09 

$206  09 

Third  year: 

$206.09  X  .0609  ................  .       $  1  2  55 

Rent   ............  100  00     112  55 

Amount  .................  ~~~     ~~   $318  64 


ORDINARY  ANNUITIES  343 

Problems 

1.  Compute  the  number  of  periods  in  each  of  the  following: 

Amount  of  an 

Ordinary  Annuity  Rents  Rate 

(a)         $        cS  0191  $        1  00  44% 

(6)             663.29  100  00  4% 

(c)  9,549.11  1,00000  5% 

(d)  1,061.82  200  00  3% 

2.  Smith  desires  to  repay  his  debt  of  $5,000  by  paying  $500  at  the  end  ot 
each  year.     If  money  is  worth  5%,  interest  convertible  semiannually,  how  many 
payments  will  be  have  to  make? 

3.  A  has  a  debt  of  $10,570.     lie  makes  a  payment  of  $1,000  at  the  end  of 
each  year.     Money  is  worth  6%,  interest  convertible  quarterly.     How  many 
full  periodic  payments  will  A  have  to  make  in  order  to  cancel  the  debt? 

4.  To  repay  a  loan  of  $1,000  bearing  interest  at  4%,  convertible  quarterly, 
Adams  makes  a  payment  of  $100  at  the  end  of  each  six  months.     How  long 
will  it  take  him  to  cancel  the  debt? 

5.  A  building  is  puicluised  for  $15,000.     Payment  is  to  be  made  in  install- 
ments of  $1,500  at  the  end  of  each  six  months'  period.     Interest  is  at  0%,  com- 
pounded quarterly.     How  many  payments  will  be  required  to  cancel  the  debt? 

Computation  of  the  rate  of  an  annuity.  The  mathematical 
theory  of  an  annuity  deals  with  five  elements  term,  rent,  rate  of 
interest,  present  value  of  the  annuity,  and  amount  of  the  annuity. 
Tf  the  rate  and  any  other  three  elements  are  given,  the  missing 
element  may  be  found.  However,  as  the  rate  is  used  twice  in 
annuity  calculations,  it  can  be  only  approximated  if  it  is  not  given. 

To  obtain  an  approximate  rate  by  inspection,  find  a  trial  rate, 
and  test  it  to  see  whether,  when  it  is  used  with  the  given  component 
parts,  it  produces  an  amount  equivalent  to  or  nearly  equivalent  to 
the  amount  of  the  annuity.  If  the  trial  rate  proves  to  be  near  the 
required  rate,  use  it  as  a  basis,  and  select  another  rate  such  that  of 
the  two  rates  chosen  one  will  be  more  and  the  other  less  than  the 
required  rate.  Proceed  to  select  the  approximate  rate  by  the  proc- 
ess of  interpolation. 

The  test  rates  chosen  may  be  found  by  means  of  an  annuity 
table,  or  by  the  calculation  of  the  amount  of  an  annuity  of  1. 

Selection  of  rates  by  use  of  an  annuity  table. 

Procedure:  (a)  Divide  the  given  amount  of  the  annuity  by  the 
annuity  rent,  to  find  the  amount  of  an  annuity  of  1. 

(b)  Choose  from  the  annuity  table  the  two  amounts  nearest  to 
the  amount  found  in  (a). 

(c)  Proceed  by  interpolation  to  find  the  approximate  required 
rate. 


344 


ORDINARY  ANNUITIES 

Example 


4>*Uff(f/lC 

An  ordinary  annuity  the  amount  of  which  is  $1,099.62  has  fhe  annual  rents 
of  $200  each.     What  is  the  rate  of  the  annuity? 


SECTION  OF  ANNUITY  TABLE 


Periods   3?  % 

4(/o 

4i% 

5f/o 

0% 

1 

1 

.00000 

1 

00000 

1 

00000 

1 

00000 

1 

00000 

2 

2 

.03500 

2 

.04000 

2 

04500 

2 

05000 

2 

06000 

3 

3 

.10622 

3 

.12160 

3 

13702 

3 

15250 

3 

1  S360 

4 

4 

.21494 

4 

24646 

4 

.27819 

4 

31012 

4 

37461 

5 

5 

.36246 

5 

.41632 

5.47071 

5 

.52563 

5 

63709 

$1,099.62  -f-  $200 


Solution 
$5.4981,  amount  of  an  annuity  of  $1  for  5  periods 


The  second  step  is  to  choose  from  the  annuity  table,  horizontally  to  the  right 
of  the  fifth  period,  the  two  amounts  nearest  to  $5.49X1.  By  inspection,  the 
amount  $5.47071,  in  the  4-J-%  column,  is  found  to  he  the  nearest  one  under 
$5.4981,  and  $5.52563,  in  the  5%  column,  is  found  to  be  the  nearest  one  over 
$5.4981.  Using  these  rates  and  amounts  as  a  basis,  the  following  may  be 
derived : 

Amount  of  a  $1  annuity  at  5% $5  52563 

Amount  of  a  $1  annuity  at  4-j%  5  47071 

Difference  in  amount  caused  by  \r/0  difference  in  interest 

rate     $     05492 

Amount  of  a  $1  annuity  at  unknown  rate $5  49S1 

Less  amount  at  \\  % 5  4707 1 

Difference $    02739 

of  i%  —  .2493%,  or  approximately  1  % 
4i%  +  •£%  =  4f  %,  the  approximate  rate 

Selection  of  rate  by  calculation  of  amounts  of  annuities.     If 

no  annuity  table  is  at  hand,  it  is  necessary  to  obtain  the  two  basic 
rates  by  the  calculation  of  the  amounts  of  the  annuities,  using 
estimated  rates. 

Procedure:  (a)  As  this  is  a  five-payment  annuity  and  each  pay- 
ment is  $200,  the  total  paid  in  will  be  $1,000.  Deducting  this 
$1,000  from  the  amount,  $1,099.62,  the  interest  is  found  to  be 
$99.62. 

(6)  The  payments  will  draw  interest  thus : 

$200  for  4  years 
200  for  3  years 
200  for  2  years 
200  for  1  year 
200  for  0  years 
10  years 


ORDINARY  ANNUITIES  34$ 

Hence  we  have  the  equivalent  of  $200  for  10  years. 

Simple  interest  on  $200  for  10  years  at  5%  is  $100,  or  a  little 
more  than  the  interest  in  the  problem  when  compound  interest  is 
disregarded.  Thus,  it  can  be  seen  that  the  interest  is  probably  less 
than  5%,  but  as  the  compounding  of  interest  is  infrequent,  the 
variation  will  be  small.  Hence  it  is  advisable  to  try  the  rate  of  5%. 

First  trial  rate.     The  solution  by  the  5%  mte  is  as  follows: 


Solution 

(1.05)6  =  1.276281,  compound  amount  of  1  at  5%  for  5 

periods 
1.27G2S1   —  1  =  .270281,  compound  interest  on  1  at  5%  for  5 

periods 
.2702X1  -r-  .05  =  5.52503,  amount  of  annuity  of  1  at  5%  for  5 

periods 

$200  X  5.52503  =  $1,105.13,  amount  of  annuity  of  $200  at  5%  for 
5  periods 

This  is  found  to  be  a  little  more  than  the  required  amount;  therefore  the  next 
trial  rate  should  be  less  than  5%. 

Second  trial  rate.     A  trial  rate  of  4^%  will  be  used. 


Solution 

(1.045)5  =  1.2401819,  compound  amount  of  1  at  4.5%  for 

5  years 
1.2401X19  -  1  =  .2401X19,  compound  interest  on  1  at  4.5%  for 

5  years 
2401X19  -r-  .045  =  5.470709,  amount  of  annuity  of  1  at  4.5%  for 

5  years 

$200  X  5.470709  =  $1,094.14,  amount  of  annuity  of  $200  at  4.5% 
for  5  years 

This  is  found  to  be  smaller  than  the  amount  in  the  problem,  $1,099.02. 
Interpolate  between  the  rates  used  in  the  first  and  second  trials,  as  follows* 

Interpolation  of  A  mounts 

Amount  of  annuity  at  5%   $1,105. 13 

Amount  of  annuity  at  4^% 1,094  14 

Difference  in  amount  caused  by  \%  difference  in  rate. .  $      10  99 

Amount  of  annuity  at  unknown  rate $1,099  62 

Amount  of  annuity  at  4^% 1,094. 14 

Difference  in  amount. . .  $       5  48 

of  i%  =  approximately  -J-% 
+  i%  =  4|-%,  the  required  rate 


346  ORDINARY  ANNUITIES 

The  approximate  rate  is  found  from  the  present  value  of  annui- 
ties in  exactly  the  same  manner  as  from  the  amounts  of  annuities, 
except  that  the  formula  for  the  present  value  is  used  instead  of  the 
formula  for  the  amounts. 

Problems 

1.  Calculate  the  rates  in  each  of  the  following: 


Amount  of 

No. 

Annuity 

Rents 

Periods 

(a) 

$        5  58 

$     1  00 

5 

(b) 

232  76 

10  00 

15 

(c) 

4,486  52 

100  00 

21 

(d) 

4,358  54 

125  00 

20 

(e) 

1,729  46 

137  50 

10 

(/) 

347  50 

20  00 

12 

(g)        6,684  00       200  00        20 
2.  Calculate  the  rates  in  each  of  the  following: 

] ^resent  Value 
No.      of  Annuity         Rents       Periods 

(a)  $        4  45       $     1  00  5 

(b)  77  22  10  00         10 

(c)  196  90  25  50         10 

(d)  5,018  75         500  00         15 

(e )  657 . 42  20  00         36  monthly  payments. 
(/)          1,200  00         100  00        20 

(g)         7,500  00         500  00        25 

Solution  of  annuity  problem  with  limited  data.     It  is  not 

uncommon  to  find  in  examinations  in  accounting  aproblem  followed 
by  a  list  of  numerical  values  of  certain  terms,  from  which  the 
candidate  must  calculate  certain  other  values  needed  for  solution 
of  the  problem.  The  purpose  of  this  is  to  test  the  candidate's 
knowledge  of  the  relationships  between  actuarial  terms. 

Problems 

1.*  A  company  is  issuing  $100,000  of  4%,  20-year  bonds,  which  are  to  be 
paid  at  maturity  by  means  of  a  sinking  fund  into  which  annual  deposits  are  to 
be  made.  The  board  of  directors  wishes  to  assume  that  this  fund  will  earn 
5ri%  f°r  the  first  5  years,  5%  for  the  next  5  years,  and  4%  for  the  last  10  years, 
What  is  the  annual  deposit  required? 
Given: 

£*%         5%          4% 

tf, 5  581       5  526      5  416 

»S10 12.875  12.578  12006 

(1  -M)6 1.307   1.276   1  217 

(l+O10 1.708       1.629       1.480 


1  American  Institute  Examination. 


ORDINARY  ANNUITIES  347 

2.*  A  city,  with  its  fiscal  year  ending  April  30,  prepares  its  budget  and 
makes  its  tax  levy  for  the  subsequent  fiscal  year  during  March,  taxes  being 
payable  on  or  after  November  1. 

A  bond  election  was  held  in  June,  1942,  and  bonds  of  $1,000,000  were  issued 
dated  August  1,  1942,  due  in  20  years.  A  sinking  fund  was  to  be  provided,  calcu- 
lated on  a  basis  of  4%,  interest  compounded  annually. 

An  audit  having  been  made  as  of  April  30,  1943,  the  balance  of  $409,588.25 
in  the  sinking  fund  is  found  to  differ  from  the  actuarial  requirements. 

Calculate  the  correct  amount  which  should  have  been  in  the  fund,  and 
ascertain  the  annual  adjustment  that  the  city  must  thereafter  make  to  be  able 
to  meet  the  bonds  at  maturity;  the  difference  is  to  be  spread  over  the  subsequent 
levies,  and  not  provided  for  in  the  next  levy  only. 

Assume  that  4%  interest  will  be  earned  in  the  future,  that  all  taxes  will  be 
collected  in  full  by  the  end  of  the  fiscal  year,  and  that  a  deposit  of  the  correct 
amount  is  to  be  made  in  the  sinking  fund  annually  on  April  30. 
Given,  at  4%: 

r8    =  .7306902         (1  +  i)8    =  1.3685690 

r9    =  .7205867         (1  -ft)9    =  1.4233118 

v10  =  .6755642         (1  +  i)10  =  1.4802443 

(1  -f  r)10  =  2.1068492 

(1  +  i)20  =  2.1911231 

(1  +  i)21  =  2.2787681 

Review  Problems 

1.  Amos  Brown  sets  aside  $400  at  the  end  of  each  year  to  provide  a  fund 
for  his  daughter's  college  expenses.     If  he  invests  the  money  at  3%  effective, 
compounded  annually,  what  will  be  the  amount  at  the  end  of  10  years? 

2.  Ben  Told  invests  $200  at  the  end  of  each  year.     At  the  end  of  the  fifth 
year  he  has  accumulated  $1,040.81.     Write  the  equation  whose  solution  will 
give  the  rate  of  interest.     Solve  and  check  your  answer  as  nearly  us  possible 
from  the  s  1  table.     Interest  convertible  annually. 

3.  If  in  Problem  1  the  interest  realized  had  been  2^%,  what  would  have 
been  the  amount? 

4.  An,  annuity  of  $1.00  a  year  amounted  to  $8.00  in  7  years.     What  was 
the  effective  rate  of  interest? 

5.  If  money  can  be  invested  at  3%  effective,  how  many  full  years  will  be 
necessary  to  accumulate  a  fund  of  at  least  $2,000  from  $100  set  aside  at  the 
end  of  each  year? 

6.  Find  the  amount  of  an  annuity  of  $1,000  a  year  paid  in  four  quaiterly 
installments  of  $250  for  6  years  if  the  interest  rate  is  4%  effective. 

7.  How  long  will  it  take  to  accumulate  $1,500  by  depositing  $20  at  the 
end  of  each  month  if  the  bank  pays  2%  effective?     Give  your  answer  to  the 
nearest  month. 

8.  If  you  pay  a  paving  tax  of  $52.17  at  the  end  of  each  year  for  10  years  and 
the  rate  of  interest  is  5%,  what  is  the  actual  tax  for  the  paving? 

9.  What  is  the  present  value  of  an  annuity  of  $1,800  a  year  in  monthly 
installments  for  10  years  if  money  is  worth  4%  effective? 


'  American  Institute  Examination. 


348  ORDINARY  ANNUITIES 

10.  How  long  will  it  take  to  pay  for  a  house  and  lot  priced  at  $6,000  if  you 
pay  $1,000  down  and  $800  at  the  end  of  each  year  until  full  payment  is  made, 
assuming  interest  to  be  6%  effective? 

11.  If  $750  invested  at  the  end  of  each  year  for  6  years  amounts  to  $4,912.62, 
what  is  the  rate  of  interest? 

12.  An  annuity  of  $100  a  year  for  H  years  amounts  to  $900.     Find  the  effective 
rate  of  interest,  convertible  annually. 

13.  Find  the  amount  of  an  annuity  of  $500  for  10  years:  (a)  with  effective 
rate  4%;  (b)  with  a  nominal  rate  of  4%,  converted  quarterly. 

14.  George  Smith  deposits  $50  in  a  savings  bank  at  the  end  of  each  throe 
months.     The   bank   pays   2%   convertible   semiannually.     What   will    be   the 
amount  to  Smith's  credit  at  the  end  of  5  years? 

16.  Find  the  present  value  of  an  annuity  of  $500  a  year  for  10  years  if  money 
is  worth  4%  effective. 

16.  A  realtor  offers  a  house  for  $4,000  cash  and  $1,000  a  year  for  six  years 
without  interest.     A  buyer  desires  to  pay  cash.     If  money  is  worth  6%  effective, 
what  should  be  the  cash  price  of  the  house? 

17.  Find  the  present  value  of  an  annuity  of  $2,000  a  year  for  5  years  if  money 
is  worth  4%,  converted  quarterly. 

18.  A  realtor  offers  a  house  for  $1,500  cash  and  $50  a  month  for  10  years, 
without  interest.     If  money  is  worth  6%  effective,  what  is  the  equivalent  cash 
price? 

19.  What  is  the  present  value  of  an  annuity  of  $840  a  year  in  quarterly 
installments  for  six  years:  (a)  if  money  is  worth  4%  nominal,  convertible  quar- 
terly; (6)  if  money  is  worth  4%  nominal,  convertible  semiannually? 

20.  How  many  years  will  it  take  to  accumulate  $785  if  $100  is  invested  at 
the  end  of  each  year  at  4^%? 


CHAPTER  32 
Special  Annuities 

Annuity  due.  An  annuity  due  is  an  annuity  the  periodic  pay- 
ments of  which  are  made  at  the  begining  of  each  period.  A  com- 
parison of  the  following  tables  will  show  the  difference  between  an 
annuity  due  and  an  ordinary  annuity;  it  will  be  remembered  that 
the  payments  of  an  ordinary  annuity  are  made  at  the  end  of  eacli 
period. 


(1) 


(2) 


Hogi lining  of  1st  >r.         Contract  made 


2nd 
3rd 
4th 
5th 


Contract  ends 


(3) 

Rents  Payable 

in  Ordinary 

Annuity 

None 

1st  payment 
2nd 
3rd 
4th 


(4) 

Rents  Payable 
in  Annuity 

Due 

1st  payment 
2nd 

3rd        " 

4th        " 

None 


(D 

End  of 
Period 

1 

2 

3 
4 
5 


(2)_ 
4-  Period 
impound 
Amount 
00 

(3)                       (4)^ 
Amount  of            5-Period 
Ordinary            (Compound 
Annuiti/              Amount 
1   00                        00 

(«r>) 
A  mount  of 
Annuiti/ 
Due  ' 
1   06 

06 

2  06 

06 

2   1S36 

.1236 

3  1836 

.1236 

3  374616 

191016 

4  374616 

191016 

4  637093 

262477 

5  637093 

262477 

5  637093 

Amount  of  an  ordinary  annuity  for  5  periods. .  .  5  637093 

Deduct  1   000000 

Amount  of  an  annuity  due  for  4  periods  .  4  637093 


To  find  the  amount  of  an  annuity  due.  From  the  above  tabu- 
lations it  can  be  seen  that  the  amount  of  an  annuity  due  of  1  for  4 
periods  is  1  less  than  the  amount  of  an  ordinary  annuity  for  5 
periods. 


349 


350 


SPECIAL  ANNUITIES 


Totfd  Value, 

First 
Period 
1 

Second      Third 
Period      Period 

Fourth 
Period 

End  of  Last 
Period 
1   262477 

1 

1    191016 

1 

1    1236 

1 

1   06 

ANALYSIS  OF  THE  AMOUNT  OF  AN  ANNUITY  DUE 


Assumed  Periods 
Rent,  beginning  of  1st  yr 
"  2nd  " 

"              "          "  3rd  " 
"  4th  " 
Knd  of  4th  year,  annuity  due 4  637093 

As  shown  above,  each  periodic  rent  draws  interest  from  the 
begining  of  the  year  in  which  it  is  deposited,  and  continues  to 
draw  interest  until  the  due  date.  The  sum  of  these  periodic 
amounts  is  the  amount  of  the  annuity  due. 

Since  each  payment  of  an  annuity  due  is  made  at  the  beginning 
of  a  period,  the  final  result  is  that  the  amount  of  an  annuity  due 
exceeds  the  amount  of  an  ordinary  annuity  by  the  interest  on  the 
Amount  of  the  ordinary  annuity  for  one  period. 

The  amount  of  an  annuity  due,  represented  by  the  symbol  S', 
*iay  be  found  in  either  of  two  ways: 

(1)  By  adding  the  interest  for  one  period  to  the  amount  of  an 
ordinary  annuity  for  the  number  of  periods  specified. 

(2)  By  finding  the  amount  of  an  ordinary  annuity  for  one  pay- 
ment more  than  the  number  of  payments  specified,   and  then 
deducting  one  payment  or  rent  from  the  total. 

First  method.  Procedure:  (a)  Compute  the  amount  of  an 
ordinary  annuity  at  the  given  rate  per  cent  and  for  the  given  rents. 

(6)  Multiply  the  amount  found  in  («)  by  1  plus  the  rate  of 
interest  per  period  expressed  decimally. 

Example 

Annuity  payments  of  $100  are  to  be  made  at  the  beginning  of  each  year  for 
4  years.  Money  is  worth  6%.  What  is  the  amount  of  the  annuity? 

Formula 


A rith mctical  Substitution 


Solution 
(1.06)4  =  1.262477,  compound  amount  of  1  at  6%  for 

4  periods 
1.262477  -  1  =  .262477,  compound  interest  on  1  at  6%  for 

4  periods 

.262477  -T-  .06  =  4.374616,  amount  of  an  ordinary  annuity  of 
1  nt  6%  for  4  periods,  or  $4  6%.* 


The  value  of  «4  fi%  may  be  found  in  Table  4,  page  529. 


SPECIAL  ANNUITIES  351 

100  X  4.374616  =  437.4616,  amount  of  an  ordinary  annuity  of 

100  at  6Vo  for  4  periods 
$437.4616  X  1.06  =  §463.71,  amount  of  an  annuity  due  of  $100 

}  'erijicatwn 
Beginning  of  1st  period: 

Contract  made. 

Rent   ...  ...................   $100.00  $100  00 

Beginning  of  2nd  period: 

Rent         .  ...............      100  00 

Interest  on  $100  at  6  %     ............         6  00     106.00 

New  principal     .................  $206  00 

Beginning  of  3rd  period: 

Rent   .      .        .          .  ................   $100  00 

Interest  on  $206  at  (>%  ...          .        12  36     112  36 

New  principal    .......  ....  $318.36 

Beginning  of  4th  period: 

Rent  ........  .         $100  00 

Interest  on  $31S.36  at  6%  J9   10    JJO  10 

New  principal  .  .  $437  46 

End  of  4th  period: 

Interest  on  $437.46  .  .  26  25 

Amount  due  .  ....  $463  71 

Second  method.  Procedure:  (a)  Determine  the  amount  of  an 
ordinary  annuity  of  1  for  the  required  number  of  periods  plus  1. 

(ft)   Deduct  1  from  the  result  found  in  (a). 

(r)  Multiply  the  difference  found  in  (ft)  by  the  number  of 
dollars  in  each  periodic  rent,  and  this  product  will  be  the  amount 
of  the  annuity  due. 

Formula 


A  rithmctical  Substitution 


Solution 

(1.06)*  =  1.338225,  compound  amount  of  1   for  5  periods 

at  6% 
1.338225  -  1  =  .338225,  compound  interest  on  1  for  5  periods 

at  6% 

.338225  -v-  .06  =  5.63709,  amount  of  an  ordinary  annuity  of  1  for 

5  periods  at  6  %,  or  s^%  from  Table  4,  page  529. 

5.63709  -  1  =  4.63709,  amount  of  an  annuity  due  of  1  for  4 

periods  at  6% 

$100  X  4.63709  =  $463.71,  amount  of  an  annuity  due  of  $100  for 
4  years  at  6% 


352  SPECIAL  ANNUITIES 

Problems 

Prepare  the  formula,  solution,  and  verification  for  each  of  the  following: 

1.  An  annuity  contract  calls  for  the  payment  of  $1,000  at  the  beginning  of 
each  year  for  5  years.     Money  is  worth  6%,  interest  compounded  annually. 
What  is  the  amount  of  the  annuity  at  the  end  of  the  fifth  year? 

2.  For  5  years  a  man  deposits  in  the  bank  $150  on  the  first  of  each  quarter. 
The  bank  allows  him  4%  interest,  compounded  quarterly.     What  will  be  the 
amount  of  his  savings  in  the  bank  at  the  end  of  the  5-year  period? 

3.  The  X.Y.Z.  Company  deeds  a  house  and  lot  to  Smith.     In  return,  Smith, 
is  to  deposit  with  the  company,  over  a  period  of  10  years,  $200  at  the  beginning 
of  each  six  months.     The  rate  of  interest  is  to  be  6%,  compounded  semiannually. 
What  will  be  the  amount  of  the  accumulation  at  the  end  of  the  10-year  period? 

Present  value  of  an  annuity  due.  The  present  value  of  an 
annuity  due,  represented  by  the  symbol  A',  is  the  present  or  actual 
cash  value,  at  the  date  of  the  first  payment,  of  all  the  payments  to 
be  made  under  the  annuity  contract.  The  following  table  shows 
the  present  value  of  an  annuity  due  for  which  payments  of  1  are  to 
be  made  for  4  years;  interest  is  calculated  at  6%,  compounded 
annually : 


Rent,  beginning  of  first  period. 


Total. 


Value  at 

Beginning 
of  Period 
1   000000 

First 
Period 
I 

Second 
Period 

Third 
Period 

Fourth 
Period 

043390 

1. 

8X9996 

1. 

.839619 

1 

3  673011 

Comparison  of  present  value  of  an  ordinary  annuity  and  that 


of  an  annuity  due. 

Example 

the  terms  of  an  annuity 
be  made. 


Example 

Under  the  terms  of  an  annuity  due,  four  annual  payments  of  $1  each  aie  to 
made.     Interest  is  at  6%.     Find  the  present  value  of  the  annuity. 


TABLE  OF  COMPARISON  OF  PRESENT  VALUE  OF  AN  ORDINARY 
ANNUITY  AND  AN  ANNUITY  DUE 


(I) 

(2) 

(3) 

(4) 

Four  Rents, 

Three  Rents, 

Number  of 

Ordinary 

Ordinary 

Annuity  Due 

Rents 

Annuity 

Annuity 

of  Four  Rents 

1 

.943396 

943396 

1  000000 

2 

1  S33392 

1   833392 

1  943396 

3 

2  673011 

2  673011 

2  833392 

4 

3  465105 

3  673011 

SPECIAL  ANNUITIES  353 

To  find  the  present  value  of  an  annuity  due.  This  may  be 
done  in  two  ways: 

(1)  If  a  number  in  column  2  of  the  above  table  is  multiplied  by 
1  plus  the  rate  per  cent,  (1.06),  the  product  will  be  the  number  in 
column   4   corresponding   to   the   same   number   of   rents;   thus: 
3.465105  X  1.06  =  3.673011,  the  present  value  of  an  annuity  duo 
at  6%  for  4  periods. 

(2)  If  1  is  added  to  the  present  value  of  an  ordinary  annuity, 
the  sum  will  be  the  present  value  of  an  annuity  due  of  one  morv 
rent  than  the  ordinary  annuity ;  thus : 

Present  value  of  an  ordinary  annuity  of  3  rents 2.673011 

Adding  1     1  000000 

Present  value  of  an  annuity  due  for  4  periods 3  67301 1 

First  method.  Procedure :  (a)  Compute  the  present  value  of  an 
ordinary  annuity,  using  the  given  number  of  dollars  in  the  rents, 
the  given  rate  per  cent,  and  the  given  number  of  periods. 

(6)  Multiply  the  present  value  found  in  (a)  by  the  rate  per  cent 
per  period  plus  1. 

Example 

Under  the  terms  of  an  annuity  due,  four  annual  payments  of  $100  each  are 
to  be  made.  Money  is  worth  6%,  interest  compounded  annually.  Find  the 
present  value  of  the  annuity. 

Formula 

(Ran])(\  +  i)  =  A' 

Arithmetical  Substitution 

L        i    \ 


100 1 ^6-J(1.06)  =  $367.30. 

Solution 

(1.06)4  =  1.262477,  compound  amount  of  1  at  6%  for 

4  periods 
1  -f-  1.262477  =  .7920937,   present  value  of   1   at  6%   for  4 

periods 
1  -  .7920937  =  .2079063,  compound  discount  on  1  at  6%  for 

4  periods 

.2079063  -f-  .06  =  3.465105,  present  value  of  an  ordinary  annuity 
of  1,  or  o^,      from  Table  5,  page  532. 

100  X  3.465105  =  346.5105,  present  value  of  an  ordinary  annuity 

of  100 

$346.5105  X  1,00  -  $367.30,  present  value  of  an  annuity  due  of 
$100 


354  SPECIAL  ANNUITIES 

Verification 

Beginning  of  first  period: 

Present  value     $367 . 30 

Rent  deducted  $100.00     100.00 

New  principal $267 . 30 

Beginning  of  second  period: 

Rent 100.00 

Less  interest  on  $267.30 16  04 

Balance  to  apply  on  principal 83 . 96 

New  principal $183734 

Beginning  of  third  period: 

Rent     $100.00 

Less  interest  on  $183.34      11  00 

Balance  to  apply  on  principal 89  00 

New  principal    $94 . 34 

Beginning  of  fourth  period: 

Rent $100  00 

Less  interest  on  $94.34 5  66 

Balance  to  apply  on  principal 94  34 

Second  method.  Procedure:  (a)  Compute  the  present  value  of 
an  ordinary  annuity  of  1  at  the  required  rate  and  for  one  less  than 
the  required  number  of  periods. 

(6)  To  the  present  value  of  the  annuity  found  in  (a)-  add  1,  and 
the  result  is  the  present  value  of  an  annuity  due  of  1  for  the 
required  number  of  periods. 

(c)  Multiply  the  present  value  of  an  annuity  due  of  1  by  the 
number  of  dollars  in  each  rent. 

Formula 
R(a--\.  +  1)  =  A' 

Arithmetical  Substitution 

A L_    \ 

100  \     y"  +  7  =  $367-30- 

Solution 
(1.06)4-1,  or  (1.06)3  =  1.191016,    compound   amount   of    1    for   3 

periods  at  6% 
1  ^  1.191016  =  .839619,   present  value  of   1   for  3  periods 

at  6% 
1  -  .839619  =  .160381,    compound   discount   on    1    for   3 

periods  at  6% 

.160381  -4-  .06  =  2.6730,  present  value  of  an  ordinary  annuity 
of  1  for  3  periods  at  6%,  or  a^|ft%  from 
Table  5,  page  532. 
2.6730  +  1  =  3.6730,  present  value  of  an  annuity  due  of 

1  for  4  periods  at  6% 

$100  X  3.6730  =  $367.30,  present  value  of  an  annuity  due 
of  $100  for  4  years  at  6%     ' 


SPECIAL  ANNUITIES  355 

Problems 

1.  What  is  the  present  value  of  an  annuity  due  in  which  $100  payments 
are  to  be  made  on  the  first  day  of  each  six  months  for  10  years,  if  money  is  worth 
5%,    interest    compounded    semiannually?     Prepare    formula,    solution,    and 
verification. 

2.  The  rents  of  an  annuity  due  are  $500  each,  and  are  payable  semiannually 
for  10  years.     If  money  is  worth  6%,  interest  compounded  semiannually,  what 
is  the  value  of  the  annuity  at  the  date  of  the  payment  of  the  first  rent?     Prepare 
formula,  solution,  and  verification. 

3.  A  contract  provides  for  the  payment  of  $150  on  the  first  of  each  quarter 
for  a  period  of  10  years.     Interest  is  4%,  compounded  quarterly.     What  is  the 
present  value  of  the  contract? 

4.  What  is  the  present  value  of  a  contract  which  calls  for  the  payment  of 
$50  on  the  first  of  each  month  for  a  period  of  10  years,  interest  to  be  computed 
monthly  at  6%  per  annum? 

5.*  A  is  considering  two  propositions  for  the  investment  of  $75,000  belonging 
to  an  estate.  The  first  proposition  offers  him  six  7%  notes  maturing  as  follows: 

On  July  1 ,  1943  $  5,000 

On  July  1,  1945 .       5,000 

On  July  1,  1947 .  5,000 

On  July  1,  1949 5,000 

On  July  1,  1950..  .    .  .  5,000 

On  July  1,  1951        ....  ....             .  .      50,000 

Total $75,000 

The  second  proposition  offeis  him  two  5%  notes,  maturing  as  follows: 

On  July  1,  1946 $25,000 

On  July  1,1951 60,000 

Total $85,000 

In  each  case  the  loan  is  adequately  secured,  and  the  interest  is  payable 
semiannually;  each  proposition  is  offered  to  A  for  $75,000  in  cash  on  July  1, 
1941. 

A  requests  you  to  determine  which  proposition  is  the  better  one  for  him  to 
accept.  State  your  findings,  and  demonstrate  the  correctness  of  your  answer. 

Given:  The  present  value  of  1,  ten  periods  hence  at  3^%,  is  .708919. 

Rents  of  the  amount  of  an  annuity  due.  The  rents  may  be 
found  by  the  following  procedure : 

Procedure:  (a)  Use  the  Second  Method  (see  page  351)  to  com- 
pute the  amount  of  an  annuity  due  of  1  for  the  required  number  of 
periods  and  at  the  required  rate  per  period. 

(6)  Divide  the  given  amount  of  the  annuity  due  by  the  amount 
of  an  annuity  due  of  1,  as  computed  in  (a),  to  find  the  rent. 


*  Adapted  from  0.  P.  A.,  Illinois. 


356  SPECIAL  ANNUITIES 

Example 

The  rents  of  an  annuity  due  are  paid  at  the  beginning  of  each  yeai  for  4  years. 
At  the  end  of  the  fourth  year  the  amount  of  the  annuity  is  $100.  Interest  is 
at  6%,  compounded  annually.  Compute  the  rents. 


Formula 
P 


=  R' 


A rith  mrtica  I  tiuhstit  u  tion 
100 


.06 

Solution 
(1.06)6  =  1.338225,  compound  amount  of  1  for  5  period'* 

at  6% 
1.338225  —  1  =  .338225,  compound  interest  on  1  for  5  periods 

at  6% 

,338225  -T-  .06  =  5.6371,  amount  of  an  ordinary  annuity  of  1  for  5 
periods  at  6%,  or  s^6%  from  Table  4,  page  529 
5.6371  —  1  =  4.6371,  amount  of  an  annuity  due  of  1  for  4  periods 

at  6% 

$100  -r-  4.6371  =  $21.57,  rent  of  the  amount  of  an  annuity  due  of 
$100  for  4  years  at  6% 

Verification 

Beginning  of  first  year: 

Rent $  21  57 

Beginning  of  second  year: 

Rent $21.57 

Interest  on  $21.57  at  6% L29       22  86 

Amount $  44  43 

Beginning  of  third  year: 

Rent $21.57 

Interest  on  $44.43  at  6% 2.67 24J24 

Amount $~68767 

Beginning  of  fourth  year: 

Rent $21   57 

Interest  on  $68.67  at  6% 4. 12       25  69 

Amount $  94  36 

End  of  fourth  year: 

Interest  on  $94.36  at  6% 5  66 

$100  02 
Problems 

1.  What  are  the  rents  of  an  annuity  due  which  amount  to  $4,762.40  in 
10  years,  if  money  is  worth  6%  per  annum?     Prepare  formula,  solution,  and 
verification. 

2.  Mr.  Ames  desires  to  know  how  much  he  must  deposit  in  the  bank  on  the 
first  day  of  each  six  months'  period  for  10  years,  in  order  that  at  the  end  of  the 


SPECIAL  ANNUITIES  357 

tenth  year  he  may  have  $12,000  accumulated.  The  bank  pays  4%,  interest 
compounded  semiannually.  What  is  the  semiannual  deposit  required?  Pre- 
pare formula,  solution,  and  verification. 

3.  Brown  wishes  to  save  $8,000  by  making  equal  deposits  on  the  first  of  each 
quarter  for  10  years.     His  bank  allows  him  4%,  interest  compounded  quarterly. 
What  is  the  quarterly  deposit  required? 

4.  A  wishes  to  have  $5,000  saved  at  the  end  of  3  years.     He  decides  to  make 
equal  deposits  on  the  first  of  each  month.     His  bank  allows  him  6%,  interest 
compounded  monthly.     What  is  the  monthly  deposit  required? 

Rent  of  the  present  value  of  an  annuity  due.  The  rent  of  the 
present  value  of  an  annuity  due  may  be  found  by  the  following 
procedure: 

Procedure:  (a)  Use  the  Second  Method  (see  page  354)  to  com- 
pute the  present  value  of  an  annuity  due  of  1  for  the  required 
number  of  periods  and  at  the  given  rate  per  cent. 

(6)  Divide  the  given  present  value  of  the  annuity  due  by  the 
present  value  of  an  annuity  of  1,  as  computed  in  (a). 

Example 

A  man  owes  $5,000.  He  wishes  to  know  the  size  of  each  of  ten  equal  annual 
payments  which  will  exactly  cancel  the  debt,  and  pay  the  accrued  interest  due 
on  the  date  of  each  payment.  The  first  payment  is  to  be  made  at  once.  Money 
is  worth  0%  per  annum. 

Form  ula 
P 


A  rithmetical  Substitution 


i  .o«)Vo  ' 


Solution 

(1.06)9  =  1.6S9479,  compound  amount  of  1  for  9  periods 

at  6% 

1  -5-  1.689479  =  .591898,  present  value  of  I  for  9  periods  at  6% 
1  —  .591898  =  .408102,  compound  discount  on  1  for  9  periods 

at  6% 
.408102  -T-  .06  =  6.8017,  present  value  of  an  ordinary  annuity  for 

9  periods  at  6%,  or  a9;6%  from  Table  5, 
page  532. 

6.8017  +  1  =  7.8017,  present  value  of  an  annuity  due  for 

10  periods  at  6% 

$5,000  -*-  7.8017  ===  $640.89,  present  value  of  an  annuity  due  of 
$5,000 


358  SPECIAL  ANNUITIES 

Problems 

1.  Give  the  formula,  solution,  and  verification  for  each  of  the  following: 

Debt  to  Be 

Retired  Payments  Rents  Per  Cent  Compounded 

(a)    $1,000  00               4  ........               6  Annually 

(6)     2,7(32  50             8  ........             4  Semiannually 

(c)     4,875  40            16  ........              6  Quarterly 

2.  A  purchases  a  farm  for  $15,000,  with  interest  at  6%.     He  desires  to  pay 
'or  it  in  twelve  equal  annual  payments,  the  first  payment  to  be  made  immediately. 
What  is  the  amount  of  each  payment  necessary  to  cancel  the  debt  and  interest? 

Effective  interest  on  an  annuity  due.  When  the  interest  is 
compounded  more  often  than  the  payments  are  made,  the  effective 
rate  should  be  found  for  a  period  corresponding  to  the  period  of  an 
annuity  payment. 

Procedure:  (a)  Reduce  the  given  interest  rate  to  an  effective 
rate  for  a  period  corresponding  to  the  period  of  an  annuity  pay- 
ment. 

(b)  Using  as  the  rate  for  each  period  the  effective  rate  found  in 
(a),  proceed  by  following  the  instructions  given  in  preceding  pages 
of  this  chapter. 

Example 

What  is  the  amount  of  an  annuity  due,  the  annual  payments  of  which  are 
$100  for  3  years,  and  the  interest  on  which  is  0%,  compounded  quarterly? 

PAHT  1 
Formula  Arithmetical  Substitution 


+   - 


-)    -  l  =  Effective  rule  (  1  +  -'^-J    -  1  =  .0613635 


Solution  to  Part  1 


.06  -r  4  =  .015,  quarterly  rate 
1  H-  .015  =  1.015,  ratio  of  increase  for  1  quarter 

(1.015)4  =  1.0613635,  compound  amount  of  1  for  4  periods 
1.0613635  —  1  =  .0613635,  effective  interest  for  an  annuity  pay- 
*  ment  period 

PART  2 
Formula 

((I  _i_  f\«+i  __  i         \ 
1 1  j  «=  Amount  of  annuity  due 

An 

1W(» 


Arithmetical  Substitution 
.  0613635)'  -  1 


-0613635 


SPECIAL  ANNUITIES  359 

Solution  to  Part  2 
Substituting  for  i  the  rate  found  in  Part  1,  the  solution  is: 

(1.0G13635)4  =  1.2689855,  compound  amount  of  1  for  4 

periods  at  '•  '  >•'••{"» 
1.2689855  —  1  =  .2689855,  compound  interest  on  1  for  4 

periods  at  i'».i.{« ».{.">•  t 
.2689855  -J-  .0613635  =  4.3834,  amount  of  an  ordinary  annuity  of 

1  for  4  periods  at  6.13635% 
4.3834  -  1  =  3.3834,  amount  of  an  annuity  due  for  3 

periods  at  •'»  1 3« ),{,">  / 

$100  X  3.3834  -  $338.34,  amount  of  annuity  due  of  $100 
for  3  years 

Problems 

1.  What  will  be  the  amount  at  the  end  of  4  years  of  an  annuity  the  rents  of 
which  are  $1,000  payable  at  the  end  of  each  year,  and  the  nominal  rate  4%, 
interest  compounded  semiannually?     Give  formula,  solution,  and  verification. 

2.  A  company  desires  to  know  how  much  will  be  accumulated  at  the  end 
of  5  years  if  $5,000  is  placed  in  a  sinking  fund  at  the  end  of  each  year.     The 
fund  bears  a  nominal  rate  of  6%,  interest  compounded  semiannually.     Give 
verification. 

3.  The  R-M  Company  desires  to  accumulate  a  sinking  fund  of  $50,000  to 
meet  a  bond  issue  of  that  amount  which  is  due  in  10  years.     What  will  be  the 
annual  payments  made  on  the  last  day  of  each  year,  if  the  fund  is  so  placed 
that  it  will  bear  an  annual  rate  of  6%,  interest  to  be  compounded  semiannually? 
Construct  columnar  table  showing  payments,  interest,  addition  to  fund,  arid 
amount  of  fund. 

4.  The  R.P.S.  Company  has  a  bond  issue  of  $50,000  coming  due  at  the  end 
of  5  years.     The  directors  desire  to  know  how  much  money  must  be  placed  in 
the  bank  at  the  end  of  each  year  in  order  that  this  fund  may  exactly  cover  the 
bond  issue  at  the  end  of  the  required  time.     The  bank  pays  4%,  interest  com- 
pounded quarterly.     Construct  a  columnar  table,  as  in  the  preceding  problem. 

5.  The  annual  rents  of  an  ordinary  annuity  are  $500,  the  time  is  6  years,  and 
money  is  worth  6%,  interest  compounded  serniannually;  what  is  the  present 
value?     Construct  a  columnar  table. 

6.  What  is  the  present  value  of  a  contract  in  which  a  company  agrees  to 
pay  $500  at  the  beginning  of  each  year  for  5  years,  interest  to  be  allowed  at  the 
rate  of  5%  per  annum,  compounded  quarterly?     Construct  a  columnar  table. 

Deferred  annuity.  A  deferred  annuity  is  an  annuity  in  which 
a  number  of  periods  are  to  expire  before  the  periodic  payments  or 
rents  are  to  begin. 

The  amount  of  a  deferred  annuity  is  the  same  as  the  amount  of 
one  which  is  not  deferred,  since  no  payments  are  made  until  after 
the  time  of  deferment  has  expired. 

The  present  value  of  a  deferred  annuity  is  the  value  at  the 
beginning  of  the  period  of  deferment. 

There  are  two  methods  of  computing  the  present  value  of  a 


360  SPECIAL  ANNUITIES 

deferred  annuity.  In  each  of  these  two  distinct  operations  are 
necessary. 

First  method.  Procedure:  (a)  Determine  the  present  value  of 
an  ordinary  annuity  of  1  at  the  given  rate  and  for  the  number  of 
periods  corresponding  to  the  number  of  rents. 

(6)  Multiply  the  present  value  of  1  found  in  (a)  by  the  number 
of  dollars  in  each  rent. 

(c)  Multiply  the  present  value  of  the  annuity  found  in  (6)  by 
the  present  value  of  1  for  the  number  of  deferred  periods. 

Example 

Find  the  present  value  of  an  annuity  contract  which  calls  for  four  equal 
annual  payments  of  $100  each,  the  first  rent  to  be  paid  at  the  end  of  the  seventh 
year.  Interest  is  to  be  calculated  at  6%. 

An  analysis  shows  that  this  is  an  ordinary  annuity  for  4  years,  deferred  for 
6  years. 

Formula 

Let:  n  =  the  number  of  rents 

m  =  the  number  of  deferred  periods 

Ran\t  -  vrn  =  Present  value  of  deferred  annuity. 
A  rithmetical  Substit  ution 


100 


1        (1.06V 
.06 


P-l  - 
L(1.06)'J 


$244.28. 


Short  Solution,  Part  1 

1  -r-  1.262477  =  .7920937,  present  value  of  1  due  at  the  end  of 

4  years  at  6% 
1  -  .7920937  =  .2079063,  compound  discount  on  1  due  at  the 

end  of  4  years  at  6  % 

.2079063  -f-  .06  =  3.465105,  present  value  of  annuity  of  1  for  4 
years  at  6 %,  or  a^6%  from  Table  5,  page  532 
$100  X  3.465105  =  $346.51,  present  value  of  annuity  of  $100  for 
4  years 

Short  Solution,  Part  2 

(1.06)6  =  1.418519 
1  -^  1 .418519  =  .7049605,  present  value  of  1  due  at  the  end  of 

6  years  at  6% 
$346.51  X  .704605  =  $244.28,  present  value  of  deferred  annuity 

Verification,  Part  1 

Present  time: 

Present  value  of  deferred  annuity $244  28 

Multiply  by  (1.06)6  1  418519 

End  of  deferred  period: 
Value  at  end  of  deferred  period  $346.51 


SPECIAL  ANNUITIES  361 

End  of  1st  period: 

Rent $100  00 

Interest  on  $346.51  at  6% 20  79 

Amortization 79 . 21 

Balance $267~30 

End  of  2nd  period: 

Rent $100  00 

Interest  on  $267.30  at  6 % 1 6.04 

Amortization 83  96 

Balance $183.34 

End  of  3rd  period: 

Rent $100  00 

Interest  on  $183.34  at  6% 1 1  00 

Amortization 89  00 

Balance $  "94"  34 

End  of  4th  period: 

Rent $100  00 

Interest  on  $94.34  at  6  % 5  66 

Amortization 94  34 

Second  method.  Procedure:  (a)  Compute  the  present  value  of 
an  ordinary  annuity  of  1  at  the  given  rate  per  cent,  for  a  number  of 
periods  equal  to  the  sum  of  the  periods  of  the  annuity  and  deferred 
periods,  or  n  +  m  periods. 

(b)  Compute  the  present  value  of  an  ordinary  annuity  of  1,  at 
the  given  rate  and  for  a  number  of  periods  equal  to  the  deferred 
periods,  or  m  periods. 

(c)  Find  the  difference  between  the  present  values  found  in  (a) 
and  (6). 

(d)  Multiply  the  difference  found  in  (c)  by  a  number  equal  to 
the  number  of  dollars  in  each  rent. 

Formula 
K(<ln+m\i    —  amlt)  —  Present  value  of  deferred  annuity. 

A  rith  m  ctica  I  S  u  bstit  ution 
1 


100 


1 


=  $244.28. 


Solution 


(LOG)10  =  1.790847,  compound  amount  of  1  at  6% 

for  10  periods 
1  -v-  1.790847  =  .5583948,  present  value  of  1  at  6%  for  10 

periods 
1  —  .5583948  =  .4416052,  compound  discount  on  1  at  6% 

for  10  periods 


362  SPECIAL  ANNUITIES 

.4416052  -f-  .06  =  7.360087,  present  value  of  annuity  of  1  at 
6%     for     10    periods,    or    a10  6%    from 
Table  5,  page  532. 
(1.06)6  =  1.418519,  compound  amount  of  1  at  6% 

for  6  periods 
1  -T-  1.418519  =  .7049605,   present  value  of  1   at  6%  for 

6  periods 
1  -  .7049605  =  .2950395,  compound  discount  on  1  at  6% 

for  6  periods 

.2950395  4-  .06  =  4.917324,  present  value  of  annuity  of  1  at 
6%  for  6  periods,  or  a6l6%  from  Table  5, 
page  532. 

7.360087   -  4.917324  =  2.442763,    difference   in   present   value   of 
annuities  of  1  for  10  periods  and  6  periods 
$100  X  2.442763  =*  $244.28,  present  value  of  deferred  annuity 
of  $100 

Probably  most  of  the  difficulties  in  the  solution  of  problems 
such  as  the  above  arise  from  failure  to  make  a  complete  analysis 
before  beginning  the  work.  Problems  of  this  type  may  be  analyzed 
and  solved  in  various  ways.  Be  sure  to  see  each  problem  in  all  its 
parts  before  attempting  to  calculate  the  amounts. 

Problems 

Find  the  present  value  of  each  of  the  following  deferred  annuities.  (NOTE: 
If  interest  is  to  be  compounded  semianrmally,  quarterly,  or  monthly,  this  same 
condition  usually  prevails  during  the  period  of  deferment.) 


Number  of          Years 

Payments 

Payments  Made 

Kate 

Rents 

Deferred 

1. 

$100 

Annually 

5% 

5 

5 

2. 

$500 

Annually 

4% 

6 

4 

3. 

$250 

Semiannually 

4% 

10 

5 

4. 

$200 

Quarterly 

6% 

16 

3 

6. 

$100 

Quarterly 

6% 

12 

5 

6. 

$  50 

Monthly 

6% 

48 

3 

7.  What  is  the  present  value  of  an  annuity  contract  in  which  the  A.B. 
Company  agrees  to  pay  to  Mr.  Ladd  a  monthly  installment  of  $40  for  10  years, 
the  first  payment  to  be  made  10  years  hence?     Money  is  worth  6%,  interest 
convertible  monthly  during  the  annuity  period,  and  annually  during  the  deferred 
period. 

8.  A  company  is  issuing  $100,000  of  4%,  20-year  bonds,  which  it  wishes  to 
pay  at  maturity  by  means  of  a  sinking  fund  in  which  equal  annual  deposits  are 
to  be  made.     The  board  of  directors  wishes  to  assume  that  this  fund  will  earn 
5%  interest  for  the  first  10  years,  and  4%  for  the  last  10  years.     What  is  the 
annual  deposit  required? 

Given: 

5%          4c/o 

810 12.578     12  006 

(1  -f  t)10 1  629       1 .480 


SPECIAL  ANNUITIES  363 

9.  A  lease  has  5  years  to  run  at  $1,200  a  year,  with  an  extension  for  a  further 
>  years  at  SI, 500  a  year.  The  payments  are  due  at  the  end  of  each  year.  If 
noney  is  worth  59o,  what  should  be  the  sum  paid  now  in  lieu  of  the  10  years' 
•ent? 

10.*  On  December  31,  1943,  A  is  indebted  to  B  in  the  following  amounts: 

$1,500,  due  December  31,  1944,  without  interest 

$3,500,  due  December  31 ,  1946,  with  interest  at  the  rate  of  6%  pay- 
able annually 

$5,000,  due  December  31,  1948,  with  interest  at  the  rate  of  6%  from 
December  31,  1943,  not  payable  until  maturity  of  note  but  to  be 
compounded  annually 

$6,000,  due  December  31,  1949,  with  interest  at  the  rate  of  5%  pay- 
able annually 

On  this  date  (December  31,  1943),  .4  learns  that  on  December  31,  1947,  he 
vill  fall  heir  to  $200,000,  and  he  arranges  with  B  to  cancel  the  four  notes  in 
exchange  for  one  note  due  in  4  years.  It  is  then  agreed  that  the  new  note  shall 
nclude  interest  to  maturity  calculated  at  5%,  compounded  annually,  and  that  B 
ihall  not  lose  by  the  exchange. 

What  will  be  the  amount  of  the  new  note? 
Given  at  5%: 

i'1  =  .9523X10  (1  -hi)1  =  1.0500000 
v2  =  .9070295  (1  -f  i)2  =  1.1025000 
i'3  =  .S63X376  (1  +  i)3  -  1.1576230 
vb  =  .7S35262  (1  +  i)4  -  1.2155062 
Given  at  6%: 

(1  +  i)3  =  1.1910160 
(1  +  i)b  =  1.3382256 

11.*  On  January  1,  1938,  A  leased  a  building  to  B  for  the  period  ending 
3ecember  31,  1952,  at  an  annual  rental  of  $7,000  payable  annually  in  advance. 
Subject  to  this  lease,  A  leased  the  same  property  to  C  on  January  1,  1944,  for  a 
,erm  of  50  years  at  an  annual  rental  of  $10,000,  payable  annually  in  advance, 
7  to  receive  the  rental  of  $7,000  payable  by  B  during  the  remainder  of  B's  lease. 
?or  this  lease  C  paid  to  A  an  additional  $1,500  as  a  bonus. 

Omitting  all  consideration  of  income  tax  questions,  how  should  the  various 
tccounts  appear  on  C"s  books  if  he  calculates  interest  on  the  investment  at  6% 
>er  annum? 
Given  at  6%: 

(1  _f  i)»  =  1.689479  v»  =  .591899 

(1  -|-  {)io  =  1.790848  v10  =  .558395 

(1  +  i)41  =  10.902861         v41  =  .091719 

12. f  The  Belgian  pre-armistice  debt  to  the  United  States  amounted  to 
>1 7 1,800, 000.  The  settlement  provided  that  no  interest  was  to  be  charged  on 
his  part  of  the  war  debt  and  that  graduated  payments  on  account  of  principal 
vere  to  be  made,  totaling  $9,400,000,  by  June  15,  1931,  the  balance  to  be  payable 
it  the  rate  of  $2,900,000  per  annum  for  56  years. 

Assuming  an  interest  rate  of  3%  per  annum,  calculate  the  loss  to  the  United 
States  by  the  waiving  of  interest  calculated  at  June  15,  1931. 


*  Adapted  from  American  Institute  Examination, 
f  American  Institute  Examination. 


364  SPECIAL  ANNUITIES 

The  present  wlue  of  $1.00  at  3%  due  in  56  years  is  $0.1910361,  and  in  56 
years  $1.00  at  3%  will  amount  to  $5.2346131. 

13.  A  note,  the  face  value  of  which  is  $1,000,  bears  interest  at  the  rate  of 
8%  per  annum,  and  is  payable  in  monthly  installments  of  $25,  including  interest. 
It  is  desired  to  discount  this  note  at  the  bank,  so  that  the  bank  shall  have  an 
effective  rate  of  12%  per  annum.  What  is  the  amount  of  the  discount  to  be 
deducted  by  the  bank? 

Perpetuity.  A  perpetuity  is  defined  as  a  series  of  periodic  pay- 
ments which  are  to  run  indefinitely.  This  form  of  annual  pay- 
ment is  most  often  found  as  the  effect  of  the  establishment  of  an 
endowment  fund,  the  rents  of  which  are  to  be  used  for  a  special 
purpose.  As  the  endowment  fund  is  never  to  be  returned,  the 
amount  has  no  meaning,  but  its  present  value  is  the  value  of  the 
periodic  payment  divided  by  the  prevailing  rate  of  interest. 
The  present  value  of  a  perpetuity  is  denoted  by  aw. 

Procedure:  Divide  the  periodic  rent  by  the  current  rate 
per  cent  expressed  decimally,  to  find  the  present  value  of  the 
perpetuity. 

Example  1 

It  is  desired  to  establish  a  fund  the  annual  income  from  which,  at  6%,  will 
be  $300.  Find  the  present  value  of  the  perpetuity. 

Formula  Arithmetical  Substitution 

*f  -A.  ^  -  W.OOO 

Solution 
$300  -5-  .06  =  $5,000,  the  amount  of  the  endowment  fund 

Example  2 

Find  the  present  value  of  a  perpetuity  of  $50  a  month  at  6%  nominal  con- 
vertible monthly. 

Solution 

S  - $10'000 

Example  3 

What  is  the  present  value  of  a  perpetuity  of  $500  a  year,  if  money  is  worth 
4%  convertible  semiannually? 

Solution 

- $12'376-24 

Perpetuities  payable  at  intervals  longer  than  a  year.  In  this 
case  the  annual  interest  on  the  principal  will  accumulate  during 
the  interval  to  equal  the  payment  due  at  the  end  of  that  time. 


SPECIAL  ANNUITIES  365 

Example 

What  is  the  present  value  of  a  perpetuity  of  $1,000  every  5  years,  if  interest 
is  at  4%  a  year? 

Solution 
$1,000    1 


•04 


=  $25,000  X  0.1846271  =  $4,615.68 


Problems 

1.  If  a  farm  produces  a  net  annual  income  of  $3,600,  what  is  its  present  value 
if  money  is  worth  5%? 

2.  What  is  the  present  value  of  a  perpetuity  of  $3,500  payable  every  5  years, 
if  money  is  worth  4%  convertible  semiannually? 

3.  Find  the  present  value  of  a  perpetuity  of  $250  a  year  if  money  is  worth  6  %. 

4.  What  is  the  present  value  of  a  perpetuity  of  $(>00  a  year  if  money  is  worth 
4%  convertible  quarterly? 

5.  Find  the  annual  rent  of  a  perpetuity  whose  present  value  is  $15,000,  if  in- 
terest is  5%  a  year. 

6.  Find  the  present  value  of  a  perpetuity  of  $10,000,  payable  every  5  years, 
if  money  is  worth  4^%  a  year. 

Review  Problems 

1.  Brown  sets  aside  $500  at  the  beginning  of  each  year  to  provide  for  his 
daughter's  college  education.     If  he  invests  the  money  at  3%  effective,  what 
will  be  the  amount  following  the  10th  payment? 

2.  Find  the  present  value  of  a  premium  of  $27.42,  assuming  that  the  insured 
will  live  to  pay  20  premiums,  money  being  worth  4%  effective. 

3.  White  bought  a  house,  agreeing  to  pay  $1,000  down  and  $500  each  six 
months  until  he  paid  $8,000.     If  money  is  worth  5%  effective,  what  should  be 
the  cash  price  of  the  house? 

4.  Find  the  present  value  of  an  annuity  of  $600  a  year  for   10  years,  if 
money  is  worth  4%  effective:  (a)  deferred  5  years,  (b)  deferred  8  years. 

5.  If  you  deposit  $150  in  a  savings  bank  at  the  beginning  of  each  quarter 
and  the  bank  pays  3%  nominal  convertible  quarterly,  how  much  will  you  have 
to  your  credit  at  the  end  of  5  years? 

6.  Cole,  at  age  22,  takes  a  20-payment  life  insurance  policy  of  $1,000  on 
which  the  premium  is  $27.42  payable  at  the  beginning  of  each  year.     If  he 
should  die  at  the  end  of  10  years  just  before  the  eleventh  premium  is  due,  by 
how  much  would  his  estate  be  increased  by  having  taken  the  insurance  instead 
of  having  put  the  premiums  into  a  savings  bank  paying  2%  effective? 

7.  Find  the  present  value  of  an  annuity  of  $1,000  a  year  to  be  paid  in 
quarterly  installments  for  12  years,  deferred  for  5  years,  if  money  is  worth  3% 
effective. 

8.  An  insurance  premium  of  $64.50  is  payable  semiannually  in  advance  for 
20  years.     If  interest  is  at  3  %  convertible  semiannually,  find  the  amount  of  the 
payments  at  the  end  of  20  years. 


366  SPECIAL  ANNUITIES 

9.  Attached  to  a  $100  bond  are  coupons  worth  $2.50  each  six  months  for 
the  next  20  years.  What  is  the  present  value  of  these  coupons,  assuming  money 
to  be  worth  4%  effective? 

10.  How  long  will  it  take  to  accumulate  $10,000  by  investing  $500  each  six 
months  at  3%  convertible  semiannually? 

11.  What  rate  of  interest  compounded  monthly  is  equivalent  to  6%  com- 
pounded quarterly? 

12.  Find  the  annual  payment  required  to  accumulate  $3,000  in  10  years, 
when  money  is  worth  3%  convertible  semiannually. 

13.  If  it  takes  an  orchard  6  years  to  reach  profitable  maturity,  and  for  a 
period  of  20  years  it  is  expected  to  yield  a  net  income  of  $3,500  a  year,  what  is 
the  cash  value  of  the  orchard  if  money  is  worth  4%  effective? 

14.  Find  the  present  value  of  an  annuity  due  of  $250  a  year  payable  annually 
for  6  years,  if  money  is  worth  5%. 

15.  Find  the  present  value  of  an  annuity  of  $75  a  month  for  8  years,  6  months, 
if  money  is  worth  6  %  nominal  converted  monthly. 


CHAPTER  33 
Bond  and  Bond  Interest  Valuation 

Definitions.  A  bond  is  a  promise  to  pay  a  specified  sum  of 
money  at  a  de terminable  future  date.  It  differs  from  a  note,  in 
general,  in  that  it  is  usually  a  long  term  obligation.  Bonds  are 
generally  issued  by  a  city,  state,  nation,  or  corporation,  and  seldom 
by  individuals.  The  sum  written  in  the  body  of  the  instrument  is 
known  as  the  face,  par,  or  nominal  par,  and  is  the  amount  on  which 
interest  is  calculated.  Bonds  usually  have  a  par  of  $100,  $500,  or 
$1,000. 

Interest  payments  may  be  made  annually,  semiannually,  or 
quarterly.  These  interest  payments  are  known  as  nominal  inter- 
est, or  cash  interest. 

The  rate  per  cent  earned  on  the  actual  money  invested  is 
termed  the  effective  or  investment  interest.  The  cash  rate  and 
the  effective  rate  are  not  the  same  unless  the  bond  is  purchased  at 
par.  Quotations  are  sometimes  made  "on  a  basis/7  which  means 
at  an  effective  rate  on  the  money  invested.  The  price  of  a  bond 
will  usually  be  either  above  or  below  par;  because  the  cash  rate  is 
either  above  or  below  the  effective  rate. 

A  bond  is  said  to  be  "redeemed"  when  it  is  bought  back  by  the 
company  which  issued  it.  If  the  market  price  of  a  bond  is  more 
than  the  par  value,  the  bond  is  said  to  be  above  par;  or  if  less,  the 
bond  is  said  to  be  below  par.  Bonds  sold  above  par  are  said  to  be 
sold  "  at  a  premium,"  and  if  sold  below  par,  they  are  said  to  be  sold 
"at  a  discount." 

Bonds  are  usually  sold  on  the  open  market  for  whatever  they 
will  bring.  Whether  they  bring  more  or  less  than  par  depends 
upon: 

(1)  The  cash  or  coupon  rate  of  interest. 

(2)  The  redemption  price. 

(3)  The  current  rate  of  interest. 

(4)  The  length  of  time  until  maturity. 

(5)  The  character  of  the  security. 

Only  the  first  four  of  the  above  can  be  mathematically  con- 
sidered. 


368  BOND  AND  BOND  INTEREST  VALUATION 

Bonds  sold  at  par.  It  is  apparent  that  if  a  man  desires  to  buy 
a  certain  bond  which  has  a  face  value  of  $1 ,000,  and  this  bond  bears 
a  cash  rate  of  interest  exactly  the  same  as  that  which  his  money  is 
worth  on  the  market  for  other  securities  of  the  same  general  class, 
he  will  be  willing  to  pay  $1,000  for  the  bond.  In  such  a  case,  the 
interest  that  he  will  receive  from  this  bond  will  be  equivalent  to 
the  interest  that  he  would  receive  from  any  other  investment  of  the 
same  general  class.  But  if  he  considered  the  purchase  of  another 
bond  of  $1,000,  which  had  a  cash  rate  of  interest  lower  than  that 
of  other  investments  of  the  same  general  class,  it  is  apparent  that 
he  would  not  pay  a  full  $1,000  for  this  bond.  And  again,  if  there 
were  on  the  market  securities  of  the  same  general  class  paying  a 
higher  rate  per  thousand,  he  would  be  willing  to  pay  more  than  tln» 
par  value  for  a  bond  of  this  class. 

This  may  be  illustrated  graphically,  as  follows: 

5% 
4% 

^       \ 

Cash  or  Bond 
Rate  of  Interest 


Par  of  Bond 


1.  Cash  or  bond  rate  of  interest  has  a  tendency  to  lift  the  price  of  the  bond 
above  par. 

2.  Effective  rate  of  interest  has  a  tendency  to  bring  the  price  of  the  bond 
below  par. 

a.  This  is  an  illustration  of  an  equal  pull  of  cash  and  effective  interest. 
If  all  other  factors  were  equal,  the  result  would  be  a  price  at  par. 

6.  If  the  cash  rate  is  larger  than  the  effective  rate,  the  price  should  be 
above  par. 

c.  If  the  effective  rate  is  larger  than  the  cash  rate,  the  pull  is  below  par. 

Bonds  purchased  at  a  discount  or  at  a  premium.  If  a  bond  is 
purchased  at  a  discount,  the  effective  rate  io  higher  than  the  cash 
rate,  for  two  reasons:  (a)  the  investment  is  less  than  par;  and  (6) 
the  investor  makes  additional  income  equal  to  the  difference 
between  the  cost  and  the  par  to  be  collected  at  maturity.  On  the 
other  hand,  if  a  bond  is  purchased  at  a  premium,  the  effective  rate 
is  lower  than  the  cash  rate,  for  two  reasons :  (a)  the  investment  is 


BOND  AND  BOND  INTEREST  VALUATION 


369 


more  than  par;  and  (6)  the  investor  loses  the  difference  between 
the  price  paid  and  the  par  to  be  collected  at  maturity. 

Price  and  rate  of  yield.  In  business,  the  usual  questions 
which  arise  with  regard  to  bonds  are:  "What  price  should  be  paid 
for  bonds  if  a  certain  investment  rate  of  interest  is  desired  by  the 
investor?"  and  "What  rate  will  a  bond,  purchased  at  a  certain 
price,  produce?" 

Use  of  bond  tables.  In  the  calculation  of  the  cost,  bond  tablevS 
are  generally  used.  They  are  reproduced  below  in  order  that  the 
student  may  familiarize  himself  with  their  use. 

Bond  table,  first  form.  The  following  is  a  very  simple  form  of 
bond  table,  in  which  only  values,  effective  rates,  and  nominal  rates 
are  shown: 

TABLE  OF  BOND  VALUES 

FIVE  YEARS,  INTEHEST  PAYABLE  SEMIANNUALLY 

. 

Xowinal  Rate 


Effective  Rate 

3 

3i 

4 

4i 

5 

6 

7 

"4 

89  20 

91   30 

93  52 

95  08 

97  84 

102.16 

106.48 

5  )i 

SS.70 

90  85 

93  00 

95.10 

97  31 

101.61 

105  92 

ST 

XX  20 

90  34 

92  49 

94.63 

96  78 

101.07 

105  37 

54 

87  .  70 

89  84 

91   98 

94.12 

96  26 

100.53 

104  81 

6 

87  20 

89  34 

91.47 

93.60 

95  73 

100.00 

104  27 

Example 

What  is  the  purchase  price  of  a  5-year,  5%  bond  (interest  payable  .semi- 
it  nnually),  bought  on  a  (}'/0  basis? 

Referring  to  the  table,  the  answer  is  found  to  be  $95  73. 

Problems 

1.  A  $1,000  bond  due  in  5  years  bears  interest  at  4%,  payable  semiannually, 
and  is  bought  on  a  5^-%  basis.     What  is  the  purchase  price,  or  value? 

2.  A  $1,000  bond  due  in  5  years  bears  interest  at  6%,  payable  semiannually, 
and  is  purchased  on  a  5^%  basis.     State  the  value  of  the  bond. 

3.  What  price  can  be  paid  for  city  bonds  of  $1,000  each,  due  in  5  years, 
interest  at  5%,  payable  semiannually,  bought  on  a  6%  basis? 

4.  If  John  Jones  bought  one  4^%,  $1,000  bond  for  $946.30,  what  per  cent 
may  he  expect  on  his  investment? 

5.  The  coupon  rate  is  4jr%;  the  number  of  bonds  is  5;  the  purchase  price  is 
$4,680.00.     What  per  cent  is  made  on  the  investment? 

Bond  table,  second  form.  The  most  common  form  of  bond 
table  shows  the  "Price"  as  indexed  between  the  "Nominal"  and 
the  "Effective"  rates.  A  new  and  very  efficient  form  of  bond 


370  BOND  AND  BOND  INTEREST  VALUATION 

table  is  that  used  by  Johnson,  Stone,  Cross,  and  Kircher  in  their 
volume,  Yields  of  Bonds  and  Stocks  (New  York,  Prentice-Hall, 
Inc.,  enlarged  edition,  1938).  In  this  form  of  table  the  "Effec- 
tive" rate  is  indexed  between  the  " Nominal"  rate  and  the  "Price." 
An  additional  feature  is  the  showing  of  the  cash  rate  of  return. 
A  page  from  Yields  of  Bonds  and  Stocks  is  reproduced  on  page  371. 

Example 

What  will  be  the  rate  of  yield  of  a  bond  purchased  at  $97.25,  if  it  bears  5% 
interest,  payable  semiannually,  and  matures  in  5  years? 
Procedure:  (a)  Turn  to  the  5%  table. 
(6)  Find  the  column  headed  5  years. 

(c)  In  the  column  at  the  left,  find  the  price  $97.25. 

(d)  Find  the  rate  at  the  intersection  of  the  year  column  and  the  price  line. 
In  this  case  it  is  5.639%. 

Interpolating  in  bond  tables.  It  frequently  happens  that  a 
given  yield  or  a  given  price  is  not  listed  in  a  bond  table.  In  such 
cases,  however,  the  desired  price  or  yield  rate  can  be  obtained  by 
interpolation. 

Example 

What  price  can  an  investor  pay  for  a  bond  due  in  5  years,  if  he  wishes  to 
obtain  a  yield  of  6%  and  the  bond  bears  interest  at  5%,  payable  semiannually? 

Procedure:  (a)  Turn  to  the  5%  table. 

(b)  Find  the  column  headed  5  years. 

(c)  Select  from  this  column  two  effective  rates,  one  just  above  and  the  other 
just  below  the  rate  desired. 

(d)  In  the  price  column  at  the  left,  select  the  prices  corresponding  to  the 
rates  selected  in  (c). 

(e)  The  rate  of  yield  is  obtained  by  interpolating  between  the  yields  of  the 
prices  found  in  (c). 

Solution 

When  yield  =  6.057,  the  price  is  $95.50 

"        "     =  6.000,    "       "     " 

"         "     =  5.996,    "       "      "    95.75 
6.057  -  5.996  =  0.061 
6.057  -  6.000  =  0.057 
$95.75  -  $95.50  =  $.25 

Since  a  difference  of  0.061  in  the  rate  of  yield  means  a  difference  of  25^  in  the 
price,  a  difference  of  0.057  in  the  rate  of  yield  will  mean  a  difference  of  fy  of  .25; 
hence  the  price  sought  is  fy  X  .25  greater  than  $95.50.  That  is: 

||  X  .25  =  .2336 
$95.50  +  .2336  =  $95.7336  (Answer) 

Example 

What  is  the  rate  of  yield  on  a  5%  bond  due  in  4  years,  interest  payable 
semiannually,  if  the  bond  is  purchased  at  96£? 


BOND  AND  BOND  INTEREST  VALUATION 


371 


TABLE  OF  YIELDS  OF  6%  BOND 

YIELDS  IN  PER  CENT  PER  ANNUM,  CORRECT  TO  THE  NEAREST  FIVE  TEN- 
THOUSANDTHS  OP  1%,  INTEREST  PAYABLE  SEMIANNUALLY 


Price 

3 

Years 

3J 

Years 

4 
Years 

4J 
Years 

5 

Years 

5} 
Years 

6 
Years 

Current 
Income 

94 

7.262 

6.961 

6.736 

6.561 

6  422 

6  308 

6  213 

5.319 

94} 

7.164 

6.876 

6.661 

6.494 

6.361 

6.252 

6.161 

5.305 

94* 

7.067 

6.792 

6.586 

6.427 

6.299 

6  19,5 

6.109 

5.291 

94| 

6.970 

6.708 

6.512 

6.360 

6.238 

6.139 

6.057 

5.277 

95 

6  873 

6.624 

6.438 

6.293 

6.178 

6  083 

6  005 

5.263 

95} 

6.776 

6.540 

6.364 

6.227 

6.117 

6  028 

5.953 

5  249 

96^ 

6  680 

6.457 

6.290 

6.160 

6.057 

5.972 

5  902 

5.236 

951 

6.584 

6.374 

6.216 

6.094 

5.996 

5.917 

5.850 

5.222 

96 

6.489 

6.291 

6.143 

6.028 

5.936 

5.861 

5  799 

5.208 

96} 

6.394 

6.209 

6  070 

5.962 

5.876 

5  806 

5.748 

5  195 

96  J 

6  299 

6.126 

5  997 

5.897 

5.817 

5.751 

5  697 

5  181 

961 

6.204 

6.044 

5.924 

5.832 

5.757 

5.697 

5  646 

5  168 

97 

6.110 

5.962 

5.852 

5.766 

5  698 

5.642 

5  596 

5  155 

971 

6.016 

5.881 

5.780 

5.701 

5  639 

5.588 

5  545 

5  141 

97  i 

5  922 

5.800 

5.708 

5.637 

5.580 

5.533 

5  495 

5  128 

97J 

5.828 

5.718 

5.636 

5.572 

5.521 

5.479 

5.445 

5  115 

98 

5.735 

5.638 

5.565 

5.508 

5.463 

5  425 

5  395 

5  102 

98} 

5  642 

5  557 

5  493 

5.444 

5.404 

5  372 

5  345 

5  089 

98* 

5.550 

5.477 

5  422 

5.380 

5  346 

5  318 

5  295 

5  076 

981 

5.457 

5.397 

5  351 

5.316 

5.288 

5.265 

5  .  246 

5.063 

99 

5.365 

5.317 

5  281 

5.252 

5.230 

5.211 

5  196 

5  051 

99} 

5.274 

5.237 

5  210 

5.189 

5.172 

5.158 

5  147 

5  038 

99| 

5  182 

5.158 

5  140 

5.126 

5.115 

5.105 

5  098 

5  025 

991 

5.091 

5.079 

5.070 

5.063 

5.057 

5.053 

5  049 

5  013 

100 

5.000 

5.000 

5.000 

5.000 

5.000 

5  000 

5  000 

5  000 

100} 

4.909 

4.921 

4  930 

4.937 

4  943 

4.948 

4  951 

4  988 

100| 

4.819 

4.843 

4.861 

4.875 

4.886 

4.895 

4.903 

4.975 

1001 

4.729 

4.765 

4.792 

4.813 

4.829 

4.843 

4.854 

4  963 

101 

4.639 

4.687 

4.723 

4  751 

4.773 

4.791 

4.806 

4  950 

1011 

4.550 

4.609 

4.654 

4.689 

4.716 

4  739 

4.758 

4  938 

10U 

4.460 

4.532 

4.585 

4  627 

4.660 

4.687 

4.710 

4  926 

1011 

4.371 

4.454 

4.517 

4.565 

4.604 

4.636 

4.662 

4  914 

102 

4.282 

4  377 

4.449 

4.504 

4.548 

4.584 

4.615 

4  902 

102} 

4.194 

4.301 

4  381 

4.443 

4.493 

4.533 

4  567 

4  890 

102^ 

4.106 

4.224 

4  313 

4  382 

4.437 

4.482 

4.520 

4.878 

1021 

4.018 

4.148 

4.245 

4.321 

4.382 

4.431 

4.472 

4.866 

103 

3.930 

4  072 

4.178 

4.260 

4.326 

4.380 

4.425 

4.854 

103} 

3.843 

3  996 

4  111 

4  200 

4.271 

4.330 

4  378 

4.843 

103] 

3.755 

3.920 

4.044 

4.140 

4.216 

4.279 

4.331 

4  831 

1031 

3.669 

3.845 

3.977 

4.079 

4.162 

4,229 

4.285 

4.819 

*  From  Johnson,  Stone,  Cross,  and  Kircher,  Yields  of  Bonds  and  Stocks.     New 
York:  Prentice-Hall,  Inc.,  enlarged  edition,  1938. 


372  BOND  AND  BOND  INTEREST  VALUATION 

Solution 

When  price  =  $96.75,    the  yield  is  5.924 

"         "      =    96.875,    "       "     "  

"      =    97.000,    "       "     "  5.852 
$97.00  -  $96.75    =  $.25 
$97.00  -    96.875  =  $.125 
5.924  -      5.852  =  0.072 

Since  a  difference  of  25^  in  the  price  means  a  difference  of  0.072  in  the  rate 
of  yield,  a  difference  of  12^  in  the  price  will  mean  a  difference  in  the  rate  of 

yield  of  i-~  of  0.072;  hence  the  sought  for  rate  of  yield  is  12^/25  X  0.072  less 
than  5.924.     That  is: 

Yield  sought  =  5.924  -  (ljj  X  0.072  J 

=  5.924  -  0.031) 
=  5.888%  (Answer) 


Problems 

By  means  of  the  foregoing  bond  table,  supply  the  missing  factor  i 
the  following: 


in  each  of 


1. 

Par  of 
Bonds 
$  1,000 

Nominal 
Price            Rate 
$    977  50         5% 

Effective                             Interest 
Rate            Years            Payable 
3            Semiannually 

2. 

1,000 

fro/ 

6  (/                4 

3. 

5,000 

5,087.50 

5% 

34 

4. 

4,000 

fyO/ 

4^  c/              5 

5. 

10,000 

9,812.50 

5% 

5i 

6. 

2.000 

5% 

STJT  %                     6 

Bond  values  computed  without  tables.  If  no  bond  tables  are 
available,  the  values  of  bonds  sold  at  a  discount  or  at  a  premium 
may  be  found  by  either  of  two  methods  of  calculation. 

Bonds  sold  at  a  discount.  The  discount  on  bonds  is  the  dif- 
ference between  the  cost  (which  is  below  par)  and  the  par  value. 

Let  C  =  the  par  value  of  the  bond; 

C"  =  the  redemption  price  when  more  than  par; 

r  =  the  rate  named  in  the  bond,  that  is,  the  coupon  or  cash  interest; 
i  -  the  rate  of  yield  expected ; 
n  =  the  number  of  years  to  maturity; 
P  =  the  cost  of  the  bond. 

Two  methods  of  calculation  are  given. 

First  method.  Procedure :  (a)  Compute  the  present  value  of  the 
par  of  the  bond  at  the  investment  rate  of  interest  and  for  the 
number  of  periods  between  the  date  of  purchase  and  the  maturity 
of  the  bond,  Cvn. 


BOND  AND  BOND  INTEREST  VALUATION  373 

(6)  Compute  at  the  effective  interest  rate  the  present  value  of 
an  annuity  for  a  number  of  periods  equal  to  the  number  of  bond- 
interest  or  cash-interest  payments,  the  rents  of  the  annuity  to  be 
of  the  same  amount  as  the  periodic  bond-interest  or  cash-interest 
payments,  rCa~\it 

(c)  Add  the  present  value  of  the  par  of  the  bond  found  in  (a), 
and  the  present  value  of  the  annuity  found  in  (6).  The  sum  will 
be  the  present  value  of  the  bond. 

Example 

What  will  be  the  cost  of  a  5%  bond  for  SI 00,  maturing  in  4  years,  bought  so 
as  to  produce  6%  effective  interest,  payable  semiannually? 

Formula 
G>  +  r(1<rnt  =  P 

A  rith  mctical  8 ubst it ation 
100 1        '    .J  +  2.50  \          ,;"  =  . 196.49. 


Xolutioti,  Part  1 
Finding  the  present  value  of  the  par  of  the  bond: 

(1.03)8  =  1. 20(5770 1,  compound  amount  of  1  at  3%  for 

<S  periods 

1  -T-  1.2007701  =  .7S94092,  present  value  of  1  at  3%  for  8  periods 
$100  X  .7894092  =  $78.94,   piescnt  value  of  $100  at  3%  for  8 
periods 

Solution,  Part  '2 

Finding  the  present  value  of  an  annuity  the  rents  of  which,  $2.50,  are  the 
aiime  as  the  periodic  interest  payments  on  the  bond: 

.7S94092  =  present  value  of  1 ,  as  found  above 
1  -  .7894092  =  .2105908,  compound  discount  on  1  at  3%  for 

8  periods 

.2105908  -r-  .03  =  7.01909,  present  value  of  an  annuity  of  1 
$2.50  X  7.01909  =  $17.549,  present  value  of  an  annuity  of  $2.50 

Solution,  Part  3 

Adding: 

$78.94  +  $17.55  =  $90.49,  cost  of  bond 

A  convenient  and  condensed  statement  showing,  for  each 
period,  the  carrying  value,  the  accumulation  of  discount,  the 
coupon  interest,  and  the  effective  interest  may  be  made  as  follows. 


374 


BOND  AND  BOND  INTEREST  VALUATION 


End  of 
Period 


1  

$2  89 

$2  50 

2  .... 

....   2  91 

2  50 

3  ... 

.   .  .   2  92 

2  50 

4  

2  93 

2  50 

5  

2  94 

2  50 

6  

2  96 

2  50 

7  

.   .     2  97 

2  50 

8  

2  99 

2  50 

Effective     Coupon       Accumulation  Carrying 

Interest      Interest         of  Discounts  Value 

$  96  49 

$  39  96  88 

.41  97  29 

.42  97  71 

.43  98  14 

.44  98  58 

.46  99  04 

47  99  51 

.49  100  00 


Second  method.  The  theory  of  this  method  is  that  the  $2.50 
interest  received  will  offset  $2.50  of  the  $3.00  expected;  therefore, 
the  cost  of  the  bond  will  be  the  par  value  of  the  bond  less  the 
present  value  at  the  effective  interest  rate  of  an  annuity  of  $.50. 

Procedure:  (a)  Compute  the  present  value  of  an  annuity  of  1  at 
the  effective  interest  rate  for  a  number  of  periods  equal  to  the 
number  of  periods  that  the  bond  has  yet  to  run,  a^. 

(b)  Calculate  the  difference  between  the  number  of  dollars  of 
income  per  period  at  the  desired  rate  and  at  the  cash  rate,  using  as 
the  basis  in  both  cases  the  par  of  the  bond,  Ci  —  Cr. 

(c)  Multiply  the  present  value  of  the  annuity  found  in  (a),  by 
the  difference  found  in  (6);  the  result  will  be  the  discount  on  the 
par  value  of  the  bond,  (Ci  —  Cr)  •  ani. 

(d)  Deduct  the  discount  found  in  (c)  from  the  par  value  of  the 
bond,  and  the  result  will  be  the  price. 

Formula 
C  -  [(Ci  -  Cr)  -  «nl]  =  P 

Arithmetical  Substitution 
/.  1      ' 


100  - 


(3.00  -  2.50) ' 


(1.03T 
".03         > 


=  $96.49 


Solution,  Part  1 

(1.03)8  =  1.2667701,  compound  amount  of  1  at  3%  for 

8  periods 

1  -f-  1.2667701  -  .7894092,  present  value  of  1  at- 3%  for  8  periods 
1  -  .7894092  =  .2105908,  compound  discount  on  1  at  3%  for 

8  periods 
$.2105908  +  .03  =  $7.01969,  present  value  of  an  annuity  of  1 

Solution,  Part  2 

3.00  —  2.50  =  .50,    difference    between    effective    and    cash 

interest  for  1  period 
$7.01969  X  .50  =  $3.51,  discount  on  bond 


BOND  AND  BOND  INTEREST  VALUATION  375 

Solution,  Part  3 
$100  -  $3.51  =  $96.49,  cost  of  bond 

Verification 
First  Period : 

Cost  of  bond $  96. 49 

Interest  at  3%  on  $9C .49          $2.89 

Coupon  interest  2 . 50 

Accumulation  of  discount 39 

$  961J8 
Second  Period: 

Interest  at  3%  on  $96.88 $2  91 

Coupon  interest       2  50 

Accumulation  of  discount 41 

$  97729 
Third  Period: 

Interest  at  3%  on  $97.29 $2.92 

Coupon  interest  !J. 50 

Accumulation  of  discount 42 

$  97"  71 

Fourth  Period: 

Interest  at  3  %  on  $97.71 $2  93 

Coupon  interest     2 . 50 

Accumulation  of  discount 43 

$  98714 

Fifth  Period: 

Interest  at  3%  on  $9S.14  $2.94 

Coupon  interest  2  50 

Accumulation  of  discount .44 

$~9~8~58 

Sixth  Period: 

Interest  at  3%  on  $98.58   $2  96 

Coupon  interest  2  50 

Accumulation  of  discount 46 

$  99~04 

Seventh  Period: 

Interest  at  3%  on  $99.04 $2.97 

Coupon  interest  2 . 50 

Accumulation  of  discount .47 

$~99~5I 

Eighth  Period: 

Interest  at  3  %  on  $99.51 .  $2 . 99 

Coupon  interest     . .  2  50 

Accumulation  of  discount _       .49 

Par  of  bond $100  00 

Problems 

1.  What  should  be  the  purchase  price  of  a  $1,000,  5-year,  5%  bond  (interest 
payable  semiannually),  bought  so  that  it  will  produce  6£%?  Prove  your  work 
by  means  of  the  table. 


376  BOND  AND  BOND  INTEREST  VALUATION 

2.  If  money  is  worth  6%,  interest  payable  semiannually,  what  should  be  the 
purchase  price  of  five  $100  bonds,  bearing  a  cash  rate  of  5%,  and  having  5  years 
to  run?     Prove  your  answer  by  means  of  the  table. 

3.  Construct  in  columnar  form  a  table  showing  the  carrying  value,  the  cash 
interest,  the  effective  interest,  and  the  amortization  of  a  5-year,  6%  bond  of 
$500,  bought  on  a  7%  basis  (interest  payable  semiannually). 

4.  A  $500  bond,  maturing  in  6  years  and  bearing  interest  at  6%,  payable 
semiannually,  is  bought  on  an  8%  basis.     Construct  a  columnar  table,  as  in 
problem  3. 

6.  Show  in  columnar  form  the  carrying  value,  the  cash  interest,  the  effective 
interest,  and  the  accumulation  of  discount  for  a  4-year  bond,  the  par  value  of 
which  is  $1,000.  The  bond  bears  5%  interest,  payable  semiannually;  the  effec- 
tive rate  is  6%,  convertible  semiannually. 

Bonds  sold  at  a  premium.  As  stated  previously,  if  the  effective 
rate  is  less  than  the  coupon  rate,  the  bond  will  sell  at  a  premium, 
which  means  that  it  will  be  priced  above  par.  When  bonds  sell  at 
a  premium,  part  of  the  money  received  for  each  coupon  is  used  to 
cancel  part  of  the  premium  paid  for  the  bond. 

As  in  the  case  of  discount  on  bonds,  two  methods  of  calculating 
the  price  of  ,a  bond  sold  at  a  premium  are  in  common  use. 

First  method.  Procedure:  The  procedure  here  is  the  same  as 
the  procedure  for  the  first  method  of  finding  the  price  of  a  bond 
purchased  at  a  discount. 

Example 

What  price  should  be  paid  for  a  $100,  0%,  4-year  bond,  in  order  that  the 
investor  may  realize  5%  on  his  investment?  Interest  coupons  are  payable 
semiannually. 

Formula 

O  +  Cr  -  a-]t  =  P 
Arithmetical  fiubstit ution 


-  $103.58. 
/ 

Solution,  Part  1 
Finding  the  present  value  of  the  face  of  the  bond: 

(1.025)8  =  1.2184029,  compound  amount  of  1  at  2^%  for 

8  periods 
1  -T-  1.2184029  -  .8207466,  present  value  of  1  at  2£%  for  8 

periods 
$100  X  .8207460  =  $82.07,  present  value  of  face  of  bond 

Solution,  Part  2 
Finding  the  present  value  of  the  annuity  of  $3: 

1  -  .8207466  =  .1792534,  compound  discount  on  1  at  2i%  for 
8  periods 


BOND  AND  BOND  INTEREST  VALUATION 

.1792534  -T-  .025  =  7.1701372,  present  value  of  an  annuity  of  1  at 

2i9o  for  8  periods 
?3  X  7.1701372  =  $21.51,  present  value  of  an  annuity  of  $3 


377 


Solution,  Part  3 

$82.07  +  $21.51  =  $103.58,  cost  of  bond 
COLUMNAR  TABLE  SHOWING  VERIFICATION 


Effective 
Interest 

(Coupon 
Interest 

A  tnortization 

Carrying 
Value 

$2  59 

S3  00 

S  41 

$103  58 
103  17 

2  58 

3  00 

42 

102  75 

.    .          2  57 

3  00 

43 

102  32 

2  56 

3  00 

44 

101  88 

2  55 

3  00 

45 

101.43 

2  54 

3  00 

46 

100  97 

2  52 
251 

3  00 
3  00 

.48 
49 

100.49 
100  00 

Adding : 


End  of 
Period 

\ 
2 

3     . 
4 


/ 

8 

Second  method.  Procedure:  (a)  Compute  the  present  value  of 
an  annuity  of  1  at  the  investment  rate  of  interest  and  for  a  number 
of  periods  equal  to  the  number  of  unexpired  periods  of  the  bond, 

a  -, . 

n\i 

(h)  From  the  number  of  dollars  of  one  cash  interest  subtract 
the  number  of  dollars  found  by  multiplying  the  par  of  the  bond  by 
the  effective  rate  of  interest,  Cr  —  Ci. 

(c)  Multiply  the  present  value  of  the  annuity  found  in  (a)  by 
the  number  of  dollars  of  the  excess  of  the  investment  interest 
found  in  (6).     The  result  obtained  is  the  premium  on  the  bond, 
(Cr  -  Ci)  •  a^, 

(d)  Add  the  par  value  of  the  bond  and  the  premium  found  in 
(c)  to  obtain  the  present  value  of  the  bond  at  a  premium. 

Formula 
[(Cr  -  Ci)  -  oj  +  C  =  P 

A  rith metical  S ubstit  ution 

i_\ 

(L025^8  I 


(3  00  -  2.50) 


+  100  =  $103.58 


.       -025       /J 
Solution ,  Part  1 
(1.025)8  =  1.2184029,  compound  amount  of  1  at 


for 

for  8 


8  periods 
1  -f-  1.2184029  =--  .8207466,  present  value  of  1  at 

periods 

1  -  .8207466  =  .1792534,  compound  discount  on  1  at  2£%  for 
8  periods 


378  BOND  AND  BOND  INTEREST  VALUATION 

.1792534  -T-  .025  =  7.1701372,  present  value  of  an  annuity  of  1  at 

2-3-%  for  8  periods 
$.50  X  7.1701372  =  $3.58,  present  value  of  an  annuity  of  $.50 

Solution,  Part  2 
Adding:  $100  +  $3.5S  =  $103.58 

Verification 

First  Period: 

Cost  of  bond  ...............................  $103  .  58 

Coupon  interest  .........................    S3  00 

Less  2i%  interest  on  $103.58  .................      2  59 

Amortization  of  premium  ....................  .41 

$103.  17 

Second  Period: 

Coupon  interest   ..........................    $3  00 

Less  2^%  interest  on  $103.1  7  .................    J2J5S 

Amortization  of  premium  ....................  .42 

$102.75 

Third  Period: 

Coupon  interest  .............................    $3  00 

Less  2i%  interest  on  $102.75   ................      2  57 

Amortization  of  premium  ...................  43 

$102.32 

Fourth  Period: 

Coupon  interest  ...........................   $3  00 

Less  2i%  interest  on  $102.32  .................      2  56 

Amortization  of  premium  ...................  .44 


Fifth  Period: 

Coupon  interest  .............................    $3  00 

Less  2^%  interest  on  $101  .88   ................      2  55 

Amortization  of  premium  ....................  45 

$FoT43 

Sixth  Period: 

Coupon  interest  ............................   $3  00 

Less  2£  %  interest  on  $101  .43       ..............      2  54 

Amortization  of  premium  ...................  46 

$100.97 

Seventh  Period: 

Coupon  interest  .............................    $3  00 

Less  2|%  interest  on  $100.97  ................      2  52 

Amortization  of  premium  ....................  .48 

$100  49 

Eighth  Period: 

Coupon  interest  .............................   $3  00 

Less  2i%  interest  on  $100.49  .................     2.51 

Amortization  of  premium    ...................  .49 

Par  of  bond  ................................  $100.00 


BOND  AND  BOND  INTEREST  VALUATION  379 

Problems 

Fill  in  the  price  in  each  of  the  following;  the  interest  is  payable  semiannually: 


1, 

Face  of 
Bond 
$    100 

Time  to 
Run 
5  years 

Cash 
Interest 
6% 

Effective 
Interest        Price 
7%          $  

9,. 

1,000 

10  years 

5% 

6%            

3 

5,000 

15  years 

5i% 

6% 

4, 

2,000 

12  years 

6% 

5^%          

ft. 

3,000 

6  years 

6^% 

4i%          

6. 

5,000 

20  years 

&S°/n 

5% 

In  problems  7,  8,  9,  and  10,  set  up  a  columnar  table  showing:  (a)  the  number 
of  periods;  (b)  the  effective  interest;  (c)  the  coupon  interest;  (d)  the  amortization 
of  premium  or  discount;  and  (e)  the  carrying  value.  The  interest  is  payable 
semiannually. 


Price 


Face  of 

Time  to 

Cash  or 

Effective 

Bond       Run,  Years 

Coupon  Interest       Interest 

1. 

$    100 

4 

5% 

6% 

8. 

3,000 

3i 

6% 

5% 

9. 

7,500 

4 

fiv("o 

5^  % 

10. 

5,000 

5 

4% 

5% 

11.*  A  $10,000,  5%  coupon  bond  is  bought  on  a  4%  basis.  It  is  due  1-j  years 
hence,  and  interest  is  payable  semiannually.  Find  the  cost  of  the  bond. 

12.  What  is  the  difference  in  the  purchase  price  of  two  $1,000,  20-year  bonds, 
bought  to  yield  6%,  if  one  of  the  bonds  has  a  semiannual  coupon  of  $25,  while 
the  other  has  a  semiannual  coupon  of  $35? 

13.  Davis  died  on  April  1,  1933.     His  estate  contained  five  $1,000   bonds 
of  the  X.Y.Z.  Company,  bearing  6%  interest,  payable  July  1st  and  January  1st. 
The  bonds  were  due  on  July  1,  1938,  and  were  inventoried  at  104^.     On  July  1, 
1933,  the  trustee  purchased  five  more  of  the  same  bonds  on  a  5%  basis.      Com- 
pute the  price  paid  by  the  trustee  for  the  bonds.     Assume  the  value  of  $1,  due 
after  ten  periods  at  2jr%,  to  be  $.781198402. 

14.  Find  the  price  for  a  $1,000  bond  bearing  interest  at  5^%,  payable  May  1 
and  November  1,  maturing  May  1,  1953,  if  bought  on  May  1,  1942  at  a  price 
to  yield  the  purchaser  5%. 

15.  Suppose  the  bond  described  in  Problem  14  were  bought  to  yield  the 
purchaser  6%.     Find  the  price. 

Values  of  bonds  between  interest  dates.  Heretofore,  in  finding 
the  values  of  bonds  we  have  used  even  periods  in  the  calculation 
of  the  interest.  However,  if  it  is  desired  to  find  the  value  of  a  bond 
at  a  date  other  than  an  interest  date,  additional  computations  are 
necessary.  Two  factors  must  be  taken  into  account:  (1)  accrued 
interest  at  the  cash  rate  must  be  computed  for  the  fraction  of  an 


*  C.  P.  A.  Examination. 


380  BOND  AND  BOND  INTEREST  VALUATION 

interest  period  elapsed;  and  (2)  the  amortization  of  the  premium 
or  the  accumulation  of  the  discount  must  be  computed  for  the 
fraction  of  an  interest  period  elapsed. 

Interest  accrued  between  interest  dates.  Finding  the  amount 
of  the  accrued  interest  for  a  fractional  part  of  a  period  is  a  simple 
computation  which  requires  no  explanation.  The  accrued  interest 
must  always  be  considered  when  the  exact  value  of  an  investment 
is  being  determined. 

Bond  discount  or  premium  between  interest  dates.  For  prac- 
tical purposes,  the  amortization  of  the  premium  or  the  accumula- 
tion of  the  discount  between  interest  dates  may  be  calculated  on  a 
proportional  basis,  by  means  of  interpolation.  This  method  gives 
a  fair  degree  of  accuracy.  The  amount  may  be  readily  found  if 
bond  tables  are  used. 

Illustration  of  the  practical  process  of  calculating  the  value  of  a 
bond  bought  at  a  discount. 

Procedure:  (a)  By  the  use  of  bond  tables,  or  of  formulas  pre- 
viously given,  determine  the  value  of  the  bond  at  the  interest  date 
just  preceding  the  purchase  date,  and  the  value  of  the  bond  at  the 
interest  date  just  subsequent  to  the  purchase  elate. 

(6)  Calculate  the  discount  for  one  period  by  finding  the  dif- 
ference between  the  values  determined  in  (a). 

(c)  Calculate  the  part  of  the  discount  which  is  in  the  same  pro- 
portion to  the  discount  for  one  period,  found  in  (fr),  as  the  fractional 
part  of  the  period  which  has  elapsed  is  to  the  total  period. 

(rf)  Add  the  discount  for  the  fractional  period,  found  in  (c),  to 
the  value  of  the  bond  at  the  interest  date  just  preceding  the  pur- 
chase date. 

(e)  Determine  the  accrued  interest,  at  the  rate  specified  in  the 
bond,  on  the  par  of  the  bond  for  the  time  expired  since  the  last 
interest  date. 

(/)  Add  the  accrued  interest  found  in  (e)  to  the  amount  found 
in  (d) ;  the  sum  is  the  value  of  the  bond,  with  interest. 


Example 

On  March  1,  1944,  what  was  the  value  of  a  $1,000,  4^-%  bond,  due  January 
1,  1949?  Interest  coupons  are  payable  January  1  and  July  1,  and  money  is 
worth  6  %,  interest  compounded  semiannually . 

Solution 

Value  9  periods  before  maturity,  at  6%  $941  60 

Value  10  periods  before  maturity,  at  6%       936  02 

Accumulation  of  discount  during  1  period     $     5  58 


BOND  AND  BOND  INTEREST  VALUATION  381 

Interpolation 

Accumulation  of  discount  during  1  period  ..............   $     5.5S 

As  two  months  of  the  six  months'  period  have  elapsed,  the 

simple  proportional  part  of  the  discount  accumulated 

is  i  of  $5.58,  or  $1.86. 

Value  10  periods  before  maturity,  .  .  036  02 

Add  2  months'  accumulation  of  discount  ....  .  I  86 

Value  9  periods  and  4  months  before  maturity  .  .   $937.88 

Add  accrued  portion  of  next  interest  coupons  7  50 

Total  value  of  bond,  with  interest  ..............  .   $945  38 

Theoretical  procedure  illustrated.  As  the  discount  accumu- 
lates at  the  rate  of  3%  for  one  whole  period,  and  as  exactly  one 
third  of  a  period  has  expired,  the  accumulation  may  be  expressed 
by  a  fraction: 


- 

(1.03J-1 
When  the  3rd  root,  of  (1.03)  is  extracted,  the  fraction  becomes: 

(1.009002)  -  1 
(1.03)  -  1""  X     ' 

009902 
Simplifying,  X  5.5S       ............     $,842 


Accumulation  of  discount  by  interpolation  (as  above)  .  1  .  SO 

Accumulation  of  discount  by  theoretical  process  1   842 

Error  by  interpolation  in  the  practical  process  of  calculation  $     018 

Illustration  of  the  practical  process  of  calculating  the  value  of  a 
bond  bought  at  a  premium. 

Procedure:  (a)  Determine  from  bond  tables  the  value  of  the 
bond  at  the  interest  date  just  preceding  the  purchase  date,  and  the 
value  of  the  bond  at  the  first  interest  date  subsequent  to  the  pur- 
chase date. 

(6)  Determine  the  premium  for  one  period  by  finding  the  dif- 
ference between  the  values  found  in  (a). 

(c)  Calculate  the  part  of  the  premium  which  is  in  the  same 
proportion  to  the  premium  for  one  period  as  the  expired  part  of  the 
period  is  to  the  whole  period. 

(d)  Deduct  the  premium  found  in  (c)  from  the  value  of  the 
bond  at  the  interest  date  just  preceding  the  purchase  date. 

(e)  Determine  the  accrued  interest  on  the  par  of  the  bond  for 
the  expired  fractional  part  of  the  period. 

(/)  Add  the  accrued  interest  found  in  (e)  to  the  amount  found 
in  (d)]  the  sum  is  the  purchase  price  of  the  bond,  with  accrued 
interest. 


382  BOND  AND  BOND  INTEREST  VALUATION 

Example 

On  May  1,  1943,  what  should  have  been  paid  for  a  $1,000,  6%  bond,  due 
January  1,  1947,  if  interest  coupons  were  payable  January  1  and  July  1,  and 
money  was  worth  5%,  interest  compounded  semiannually? 

Solution 

Value  8  periods  before  maturity,  at  5%  .  ....     $1,03585 

Value  7  periods  before  maturity,  at  5%     .         .      .        .      1,031  75 
Amortization  of  premium  during  1  period  .  .   $        4  10 

Interpolation 

Amortization  of  premium  during  1  period  $  4  10 

As  4  months  of  the  8th  period  have  expired,  the 

proportional  part  of  the  6  months'  period  is 

two-thirds.     Therefore,    the   amortization 

which  has  taken  place  is  f  of  $4.10,  or  $2.73. 

Value  8  periods  before  maturity $1,035  85 

Deduct  4  months'  amortization  of  premium  2  73 

Value  7  periods  and  2  months  before  maturity  $1,033   12 

Interest  at  0%  for  1  period .      $30  00 

As  4  months  have  expired  since  any  interest  was 

paid,  there  is  due  £,  or  -f ,  of  $30     .  20  00 

Value  of  bond,  with  accrued  interest $1,053  12 


Bonds  bought  on  a  yield  basis.  For  bonds  bought  on  a  strictly 
yield  basis,  the  following  procedure  may  be  used : 

To  the  price  of  the  bond  at  the  last  preceding  interest  date,  add 
interest  thereon  at  the  effective  (yield)  rate  for  the  expired  portion 
of  the  period  during  which  the  purchase  is  made. 

Example 

A  $1,000  bond  maturing  October  1,  1952  with  interest  at  6%  payable  April  1 
and  October  1  was  bought  on  July  1,  1942  at  a  price  to  yield  the  investor  5%. 
What  price  was  paid  for  the  bond? 

Solution 

(a)  To  obtain  the  price  of  the  bond  on  April  1,  1942,  which  was  the  last 
interest  payment  date  prior  to  the  date  of  purchase,  follow  the  procedure  on 
page  376.     The  price  of  the  bond  on  April  1,  1942  is  found  to  be  $1,080.92. 

(b)  To  determine  the  price  of  the  bond  on  July  1,  1942,  compute  the  elapsed 
time  from  April  1  to  July  1 ,  which  is  3  months  or  one-half  a  period.     Then, 

$1,080.92  X  (1.025)*  =  $1,094.35 

Problems 

1.  A  $100  bond  bears  5%  interest,  payable  semiannually,  and  is  due  in 
5  years  and  4  months.  What  price,  plus  accrued  interest,  should  an  investoi 
pay  for  the  bond  if  he  wishes  his  investment  to  produce  4%? 


BOND  AND  BOND  INTEREST  VALUATION  383 

2.  A  $1,000,  5%  bond  is  due  in  6  years  and  3  months,  and  interest  is  payable 
semiannually.     The  effective  interest  rate  is  6%.     What  is  the  value  of  the 
bond,  with  accrued  interest? 

3.  A  $1,000,  5-g-%  bond,  with  interest  payable  annually,  is  redeemable  at 
par  in  20  years  and  4  months,  and  is  bought  on  a  6%  basis.     Find  the  purchase 
price. 

4.  A  $1,000,  5%  bond,  with  interest  payable  semiannually,  is  redeemable 
at  par  in  8  years  and  5  months,  and  is  bought  on  a  4-g-%  basis.     Find  the  cash 
price. 

6.  Five  $1,000,  6%  bonds,  with  interest  payable  semiannually,  are  due  in 
4  years  and  4  months,  and  are  purchased  on  a  4^%  basis.  Find  the  value  of 
the  bonds. 

Bonds  to  be  redeemed  above  par.  A  form  of  bond  which  is 
redeemable  (usually  at  the  option  of  the  maker)  at  a  premium  is  a 
callable  bond. 

To  find  the  value  of  a  bond  redeemable  above  par  at  the  option 
of  the  maker,  before  it  is  due,  it  is  necessary  to  determine  the  value: 

(1)  On  the  assumption  that  the  bond  will  be  redeemed  at  the 
optional  redemption  date,  and  at  the  optional  price. 

(2)  On  the  assumption  that  the  maker  will  not  redeem  the 
bond  until  maturity,  and  that  it  will  then  be  redeemed  at  the  par 
value. 

After  the  two  prices  have  been  found,  the  purchaser  should  pay 
the  lower  of  the  two  prices. 

Example 

A  $1,000,  6%  bond,  with  interest  payable  semiannually,  is  due  in  15  years, 
but  the  maker  has  the  option  of  redeeming  it  at  the  end  of  10  years  at  105.  What 
price  should  an  investor  pay  for  the  bond,  if  he  purchases  it  on  a  5%  basis? 

(1)  Calculation  on  the  assumption  that  the  bond  will  be  redeemed  at  the 
end  of  10  years  at  105: 

Formula 

CV  +  Cr  -  an]l  =  P 

A rithmctical  S  ubstitution 


\(1.025)2V 


/I  - 
1,0501  „-  -~^}+  30  I (^-l  =  $1,108.45. 


Solution,  Part  1 

(1.025)20  =  1.638616,  compound  amount  of  1  at  2£%  for 

20  periods 

1  -T-  1.638616  =  .61027,  present  value  of  1  at  2^%  for  20  periods 
$1,050  X  .61027  =  $640.78,  present  value  of  redemption  price  of 
bond 


384  BOND  AND  BOND  INTEREST  VALUATION 

Solution,  Part  2 

.61027  =  present  value  of  1  at  2£%  for  20  periods 
1  -  .61027  =  .38973,   compound  discount  on   1   at  2i%  for 

20  periods 

.38973  -T-  .025  =  15.5892,  present  value  of  annuity  of  1 
$30  X  15.5892  =  $467.67,  present  value  of  annuity  of  $30 

Solution,  Part  8 
$640.78  +  $467.67  =  $1,108.45,  value  based  on  optional  redemption 

(2)  Calculation  on  the  assumption  that  the  bond  will  not  be  paid  until 
maturity: 

Formula 

Cv»  +  Cr  •  a-,,  =  P 
Arithmetical  Substitution 


Solution,  Part  1 

(1.025)30  =  2.097567,  compound  amount  of  1  at  2j-%  for 

30  periods 
1  -f-  2.097567  =  .476742,  present  value  of  1  at  2i%  for  30 

periods 
$1,000  X  .476742  =  $476.74,  present  value  of  par  of  bond 

Solution,  Part  2 

.476742  =  present  value  of  1  at  2i%  for  30  periods 
1  -  .476742  =  .523258,  compound  discount  on  1  at  2^%  for 

30  periods 

.523258  -i-  .025  =  20.93029,  present  value  of  an  annuity  of  1 
$30  X  20.93029  =  $627.91,  present  value  of  an  annuity  of  $30 

Solution,  Part  8 

$476.74  -f  $627.91  =  $1,104.65,  price  of  bond  based  on  par 
A  comparison  of  the  two  results  shows: 

Value  based  on  15-year  redemption  price  ............   $1,108  45 

Value  based  on  20-year  redemption  price   ...........     1,104.65 

Therefore,  the  purchaser  should  pay  the  lower  price,  or  $1,104.65. 

Problems 

1.  A  $1,000,  5^%  bond,  with  interest  payable  semiannually,  matures  iiv 
10  years,  but  the  company  has  the  option  of  redemption  at  the  end  of  5  year& 
at  104.     What  price  should  an  investor  pay  for  the  bond,  if  he  purchases  it  on 
a  5%  basis? 

2.  A   $1,000,    5%   bond,   with  interest   payable  semiannually,   matures   in 
20  years,  but  the  company  has  the  option  of  redeeming  the  bond  at  the  end  of 


BOND  AND  BOND  INTEREST  VALUATION  385 

15  years  by  paying  a  bonus  or  premium  of  10%.     What  price  should  an  investor 
pay  for  five  of  these  bonds,  if  he  purchases  them  on  a  4%  basis? 

3.  A  $1,000,  5^%  bond,  with  interest  payable  annually,  is  redeemable  in 
5  years  with  a  bonus  of  10%.     What  price  should  be  paid  for  this  bond  by  a 
purchaser  who  desires  to  realize  6%  on  his  investment?     (v*  at  6%  =  .7473.) 
Construct  a  table  of  verification. 

Prepare  the  formula,  solution,  and  verification  for  each  of  the  following: 

Effective  Coupon  Amount  Redeemable         Time             Coupons 

Interest  Interest  of  Bond          Value  to  Run           Payable 

4.  5%             6%  $    100             104  6  years  Annually 
6.         6%            8%  1,000             105  4i    "  Annually 

6.  5%  6%  500  108  4^    "          Semiannually 

7.  6%  4%  500  110  5i    "         Semiannually 

Serial  redemption  bonds.  Many  public  and  private  corpora- 
tions desire  to  pay  off  a  portion  of  their  bonds  each  year  instead  of 
setting  up  a  sinking  fund.  Serial  redemption  bonds  may  be 
redeemed  in  equal  or  unequal  periodic  amounts. 

If  the  bonds  are  not  redeemed  in  equal  periodic  amounts,  it  is 
difficult  to  derive  a  formula  or  plan  by  which  computations  may  be 
shortened  or  systematized.  But  if  the  redemptions  are  to  be  made 
in  equal  amounts  and  at  regular  periodic  dates,  formulas  and  solu- 
tions for  finding  the  value  of  such  an  issue  may  be  derived. 

For  the  purpose  of  finding  the  present  value  of  bonds  to  be 
redeemed  in  a  series,  it  is  well  to  analyze  the  issue  into  its  com- 
ponent parts,  and  to  calculate  the  value  of  each  component  part 
separately.  The  following  example  will  illustrate  this  point : 

Example 

What  is  the  present  value  of  a  bond  issue  of  $10,000  bearing  5%  interest, 
payable  annually?  These  bonds  are  to  be  redeemed  serially,  $2,000  each  year. 
Money  is  worth  6%,  effective  interest. 

ANALYSIS  OF  THE  CALCULATION  OF  THE  VALUE  OF  A  SERIAL 
REDEMPTION  BOND 

Present  Present 

Multiplied       Value  of         Value  of 

by  Present        Annuity  a  Series  of 

First        Second       Third       Fourth        Fifth          Value  of  of  Annuities 

Period       Period       Period       Period       Period    Annuity  of  1     Principal  of  Interestt 

Principal $2,000      $2,000      $2,000      $2,000      $2,000     $4212363     $8,424.73 

Interest  payments           100                                                                                   94339  $      94  34 

100           100                                                         1  83339  183.34 

100            100            100                                           2  67301  267.30 

100           100           100           100                        3.46511  346.51 

100            100            100            100            100        4.21236 421   24 

$8.'424.73  $1.312  73 

Summary 

Present  value  of  annuity  of  $2,000  at  6% $8,424.73 

Present  value  of  5  annuities  of  $100  each  (the  number 

of  rents  varies  from  1  to  5,  as  shown  above)  at  6%  1,312. 73 

Present  value  of  the  serial  redemption  bonds $9,737.46 


386  BOND  AND  BOND  INTEREST  VALUATION 

A  study  of  the  above  analysis  shows  two  general  divisions : 

(1)  The  calculation  of  the  present  value  of  an  annuity,  the  rents 
of  which  are  the  same  as  the  amounts  of  the  bonds  which  are 
redeemed  annually. 

(2)  The  calculation  of  the  present  value  of  a  series  of  annuities, 
the  rents  of  which  are  the  same  as  the  periodic  interest  payments 
on  each  series  of  bonds. 

The  first  part  needs  no  explanation,  since  the  computation  is 
similar  to  that  of  the  present  value  of  an  ordinary  annuity. 

The  sum  of  the  values  of  the  second  part  may  be  found  by  cal- 
culating the  value  of  each  separate  annuity  and  then  adding  these 
values. 

Let  C  =  the  par  value  of  the  bond; 
R  —  annual  amount  redeemed; 

r  =  the  rate  named  in  the  bond,  that  is,  the  coupon  or  cash  rate; 
i  =  the  rate  of  yield  expected; 
n  =  the  number  of  years  to  maturity; 
N  =  the  number  of  equal  redemptions  to  retire  the  issue; 
P  =  the  cost  of  the  bond. 

Procedure,  Part  1 :  (a)  Find  the  present  value  of  an  annuity  of 
1  at  the  effective  rate  per  cent  for  as  many  periods  as  the  bond  has 
serial  payments,  a^.. 

(b)  Multiply  the  present  value  of  the  annuity  of  1  found  in  (a) 
by  the  number  of  dollars  to  be  paid  each  period  on  the  principal 
of  the  bonds,  Ra-{t* 

Procedure,  Part  2 :  (a)  Use  the  present  value  of  an  annuity  of  1 
found  in  Part  1  (a). 

(b)  From  the  number  of  annuities  subtract  the  present  value 
of  an  annuity  of  1  found  in  (a),  N  —  a-  . 

(c)  Divide  the  annuity  discount  found  in  (6)  by  the  effective 
rate  to  obtain  the  cumulative  present  worth  of  the  annuities, 
N  -  a-,. 

nit. 

i 

(d)  Multiply  the  average  annuity  price  of  1  by  the  number  of 
dollars  in  each  interest  payment  or  rent.     The  result  will  be  the 

value  of  a  series  of  annuities,  Cr  ( = — —  )• 

Procedure,  Part  3 :  Add  the  result  found  in  1  (b)  to  that  found 
in  2  (d). 

Formula 
t  +  Cr| 


BOND  AND  BOND  INTEREST  VALUATION  387 

Arithmetical  Substitution 


Solution,  Part  1 

(1.06)5  =  1.3382256,  compound  amount  of  1  at  6% 

for  5  periods 
1  -f-  1.33S2256  =  .7472582,   present  value  of   1   at  6%  for 

5  periods 

1  -  .7472582  -  .2527418,  compound  discount  on  1  at  6% 

for  5  periods 
.2527418  -r-  .06  =  4.2123638,  present  value  of  annuity  of  1  at 

6  %  for  5  periods     (See  Table  5,  page  532.) 
$2,000  X  4.2123638  =  $8,424.73,  present  value  of  annuity  of  $2,000 

Solution,  Part  2 

4.2123638  =  present  value  of  annuity  of  1  at  6%  for  5  periods 
5  —  4.2123638  =  .7876362,  annuity  discount  on  5  annuities 
.7876302  -r-  .06  =  13.12727,  cumulative  present  value  of  annuities 
$100  X  13.1273  =  $1,312.73,  cumulative  present  value  of  annuity 
of  $100 

Solution,  Part  3 
$8,424.73  -f  $1,312.73  =  $0,737.46,  sum  of  present  value  of  annuities 

Problems 

1.  Compute  the  purchase  price  of  $5,000  of  serial  bonds,  issued  January  1, 
1934,  with  5%  interest,  payable  annually,  and  dated  to  mature  in  five  equal 
annual  installments.     Money  is  worth  5^%. 

2.  Verify  the  solution  of  the  above  problem  by  setting  up  a  columnar  table 
showing:  (a)  date  of  maturity;  (b)  bonds  outstanding;  (c)  coupon  interest  each 
year;  (d)  effective  interest  each  year;  (c)  accumulation  of  discount;  (/)  carrying 
value. 

3.  Compute  the  purchase  price  of  $50,000  of  serial  bonds,  issued  January  1, 
1934,  bearing  5%  interest,  coupons  due  annually.     These  bonds  were  to  mature 
serially  in  equal  annual  payments,  beginning  January  1,  1935,  and  each  year 
thereafter  for  10  years.     Money  was  worth  4%. 

4.  Prepare  a  columnar  table  for  Problem  3. 

5.  A  $20,000  serial  bond  issue,  with  interest  at  5^%,  payable  semiannually, 
is  redeemable  in  ten  equal  semiannual  installments.     Money  is  worth  5%,  con- 
vertible semiannually.     Set  up  a  columnar  table  similar  to  that  in  Problem  2. 

Frequency  of  redemption  periods.  In  the  example  on  page 
385,  bonds  were  redeemed  at  each  interest  date.  It  is  more  usual, 
however,  to  find  that  the  interest  is  payable  semiannually,  while 
the  bond  redemptions  occur  once  a  year. 


388 


BOND  AND  BOND  INTEREST  VALUATION 


P-(  Pk  Pu, 


BOND  AND  BOND  INTEREST  VALUATION  389 

Example 

A  $6,000  issue  of  serial  bonds  is  to  be  redeemed  in  equal  installments  of 
$2,000,  on  January  1  of  each  year.  The  coupons  are  at  the  rate  of  5%,  payable 
semiannually.  If  money  is  worth  6%,  interest  convertible  semiannually,  what 
is  the  present  value  of  the  issue?  (For  the  purpose  of  illustration,  short-term 
bonds  are  used.) 

A  study  of  the  analysis  on  page  388  shows  three  divisions: 

(1)  The  calculation  to  find  the  present  value  of  an  annuity  of 
$2,000,  the  annual  payment  on  the  bond  issue,  at  the  effective 
annual  rate  of  6.09%. 

(2)  The  calculation  to  find  the  present  value  of  an  annuity  of 
$25,  the  amount  of  the  interest  payment  due  at  the  end  of  each 
year. 

(3)  The  calculation  to  find  the  present  value  of  the  series  or 
six  rents  of  $25  each  at  3%  semiannual  interest. 

It  should  be  noticed  that  one  of  the  interest  payments  of  $25 
due  at  the  end  of  each  year  is  separated  from  the  other  interest 
payments.  This  separation  is  made  to  reduce  the  remaining  inter- 
est payments  to  a  series  of  annuities  of  regularly  increasing  terms. 

The  computation  may  be  shortened  if  the  annual  bond  redemp- 
tion payment  of  $2,000  and  the  annual  interest  payment  of  $25  are 
combined.  It  is  then  necessary  to  find  the  present  value  of  an 
annuity  the  rents  of  which  are  $2,025  at  6.09%  per  annum.  The 
formula  and  solution  are  derived  by  a  method  similar  to  that 
explained  on  page  386. 

Let  g  represent  the  value  of  one  coupon  on  the  periodic  cash  interest  on  one 
bond. 


A'    /"') 


Formula 
(R  +  g)  •  an{t  +  g 

in  which  o-|f  in  the  first  part  of  the  formula  is  calculated  at  the  effective  annual 
rate. 

A  rithmetical  Substitution 

/,        i_\       /'"(w1 

2025  (L0609)3    +25\  -°3 

2'°25\       -;0609~~/  +  25r       ~^3~ 

Solution,  Part  1 

(1.03)2  =  1.0609,  effective  ratio  of  increase 
1.0609  -  1  =  .0609,  effective  rate  per  annum 

(1.0609)3  =  1.19405228,  compound  amount  of  I  at  6.09% 
for  3  periods 


390  BOND  AND  BOND  INTEREST  VALUATION 

1  -4-  1.19405228  =  .8374843,  present  value  of  1  at  6.09%  for 

3  periods 
1  -  .8374843  =  .1625157,  compound  discount  on  1  at  6.09% 

for  3  periods 
.1625157  -f-  .0609  =  2,668568,  present  value  of  an  annuity  of  1  at 

6.09%  for  3  periods 

$2,025  X  2.668568  =  $5,403.85,  present  value  of  annuity  of  $2,025 
at  6.09%  for  3  periods 

Solution,  Part  2 

(1.03)°  =  1.19405228,  compound  amount  of  1  at  3%  for 

6  periods 

1  -T-  1.19405228  =  .8374843,  present  value  of  1  at  3%  for  6  periods 
1  -  .8374843  =  .1625157,  compound  discount  on   1  at  3%  for 

6  periods 
.1625157  -T-  .03  =  5.4171914,  present  value  of  an  annuity  of  1  at 

3%  for  6  periods 

6  -  5.4171914  =  .5828086,  annuity  discount  on  6  rents 
.5828086  -5-  .03  =  19.42695,  present  value  of  6  rents  of  1  at  3% 

for  6  periods 
$25  X  19.42695  =  $485.67,  present  value  of  6  rents  of  $25  each 

Solution,  Part  3 
$5,403.85  +  $485.67  =  $5,889.52,  value  of  serial  bond 

Alternative  solution.  The  following  method  of  solving  the  pre- 
ceding example  is  preferred  when  the  number  of  redemptions  are 
few.  The  procedure  is  that  of  rinding  the  present  value  of  the 
respective  payments  of  interest  and  principal. 

July  1  Interest                    $    150  X  .970873  =  $    145  63 

Jan.  1  Int.  and  Principal   2,150  X  .942595  =     2,026  58 

July  1  Interest                   100  X  .915141  -          91   51 

Jan.  1  Int.  and  Principal  2,100  X  .888487  =     1,865  82 

July  1  Interest                 50  X  .862608  =         43  13 

Jan.  1  Int.  and  Principal 2,050  X  .837484  =     1,71 6 _84 

Total  =  $5,889jy. 

Bonds  redeemed  by  other  than  equal  annual  payments.     If  the 

bonds  are  to  be  redeemed  in  any  other  way  than  by  equal  annual 
payments  beginning  at  the  end  of  the  first  year,  an  analysis  must 
be  made  of  the  component  parts,  and  computations  made  for  each 
part  separately. 

Example 

What  is  the  value  of  an  issue  of  5%  serial  redemption  bonds  for  $10,000,  if 
$2,000  is  to  be  redeemed  at  the  end  of  6  years  and  $2,000  at  the  end  of  each  year 
thereafter  until  all  the  bonds  have  been  redeemed?  Interest  coupons  are  payable 
semiannually.  Money  is  worth  6%,  interest  converted  serniannually. 


BOND  AND  BOND  INTEREST  VALUATION 


391 


ANALYSIS  OF  A  SERIES  OF  SERIAL  REDEMPTION  BONDS,  WITH 
INTEREST  PAYMENTS 

SEMIANNUAL  PERIODS 
1       2       3       4       5       6       7       8       9        10         11      12     13     14     15     16     17     18     19     20 


1ST 

SERIES 

25  25  25  25  2 
25  25  25  25  2 

25  25  25  25  25 
25  25  25  25  25 

25s- 

IN*        <N        <N        <N         fri 

-N25 
25\ 

2ND 

SERIES 

25  25  25  25  2 
25  25  25  25  2 

25  25  25  25  25 
25  25  25  25  25 

25 
25 

25  25  \25 
25  25  25\ 

3RD 

SERIES 

25  25  25  25  2 
25  25  25  25  2 

25  25  25  25   25 
25  25  25  25  25 

25 
25 

25   25   25  25\25 
25  25  25  25  25  x 

4m 
SKKIKH 

25  25  25  25  2 
25   25  25  25  2 

25  25  25  25  25 
25   25  25   25   25 

25  25  25  25  25  25   25 
25  25   25  25  25   25  25 

OTH 
SERIES 
TOTALS 

25  25  25   25  2 
25  25  25  25  2 
250  250  250  250  25 

25  25  25  25  25 
250  250  250  250  250 

25 
25 

25   25   25   25   25   25   25   25   25 
25  25  25  25  25  25  25  25  25 

From  the  above  analysis  there  are  found  to  be  four  component 
parts  in  the  example : 

(1)  An  annuity  of  $2,000,  the  payments  of  which  are  to  begin 
at  the  end  of  the  sixth  year  and  are  to  be  made  at  the  end  of  each 
year  thereafter,  at  an  effective  interest  rate  of  6%,  convertible 


0.09% 


annually.     This   series   of   annuities 


semiaimually,    or 
deferred  for  5  years. 

(2)  An  annuity,  the  rents  of  which  are  $25,  payable  at  the  end 
of  the  sixth  year,  and  annually  thereafter,  at  6.09%. 

(3)  A  series  of  ten  annuities  of  unequal  length,  the  rents  of 
which  are  $25,  payable  at  the  end  of  each  half-year.     Each  annuity 
is  for  1  period  longer  than  the  preceding  one.     The  rate  of  effective 
interest  is  3%.     This  series  of  annuities  is  deferred  for  5  years. 

(4)  A  series  of  ten  annuities  of  equal  length,  the  rents  of  which 
are  $25,  payable  at  the  end  of  each  period,  with  interest  at  3%. 

Formulas  will  not  be  given  for  this  example,  which  does  not 
involve  anything  new,  but  only  the  combining  of  certain  principles 
already  explained. 

To  shorten  the  calculation,  (1)  and  (2)  above  can  be  combined, 
making  a  deferred  annuity  the  rents  of  which  are  $2,025  for  5 
periods  at  6.09%. 

Procedure:  (a)  Find  the  present  value  at  the  beginning  of  the 
sixth  year  of  (1)  and  (2)  combined. 

(6)  Find  the  present  value  of  (3)  at  the  beginning  of  the  sixth 
year. 

(c)  Multiply  the  sum  of  the  present  values  found  in  (a)  and  (6) 
by  the  present  worth  of  1  for  ten  periods  at  3%.  The  result  is  the 


392  BOND  AND  BOND  INTEREST  VALUATION 

present  value  at  the  beginning  of  the  first  period  of  all  the  bonds 
and  interest  payments  falling  due  after  the  end  of  the  fifth  year. 

(d)  Calculate  the  present  value  of  (4) ;  this  result  is  the  value 
at  the  beginning  of  the  first  year. 

(e)  Add  the  results  found  in  (c)  and  (d).     Their  sum  is  the 
present  value  of  the  series  of  serial  redemption  bonds,  with  all  the 
interest  payments. 

Solution,  Part  1 

NOTE:     The  compound  amount  and  present  value  of  (1. 0609) 5  are  the  same 
as  the  compound  amount  and  present  value  of  (1.03)10. 

(1.0609)5  =  1.3439164,   compound  amount  of   1   for  5 

periods  at  6.09% 
1  -T-  1.3439164  =  7440939,  present  value  of  1  for  5  periods  at 

6.09% 
1  -  .7440939  =  .2559061,  compound  discount  on   1  for  5 

periods  at  6.09% 
.2559001  -T-  .0609  =  4.2020707,  present  value  of  an  annuity  of  1 

for  5  periods  at  6.09% 

$2,025  X  4.2020707  =  $8,509.19,  value  of  an  annuity  of  $2,025  for 
5  periods  at  6.09% 

Solution,  Part  2 

.7440939  =  present  value  of  I  for  10  periods  at  3%  (same 

as  1  for  5  periods  at  6.09%) 
1  —  .7440939  =  .2559061,  compound  discount  on  1  for  10  periods 

at  3% 
.2559061  -r-  .03  =  S.530202,  present  value  of  an  annuity  of  I  for 

10  periods  at  3% 
10  -  8.530202  =  1.469798,  annuity  discount  on  the  series  of  10 

annuities  of  1 

1.469798  -r-  .03  =  48.9932,  present  value  of  the  series  of  10  annui- 
ties of  1 

$25  X  48.9932  =  $1,224.83,    present   value   of  the   series   of    10 
annuities  of  $25  each 

Solution,  Part  8 

$8,509.19  +  $1,224.83  =  $9,734.02,  amount  of  present  value  at  the 

beginning  of  the  sixth  year 
.7440939  =  present  value  of  1  for  10  periods  at  3% 

$9,734.02  X  .7440939  =  $7,243.02,  present  value  of  serial  bonds 
and  of  the  series  of  10  annuities 

Solution,  Part  4 

8.530202  =  present  value  of  an  annuity  of  1  for  10  periods 

at  3% 
$25  X  10  =  $250,  sum  of  the  10  payments  at  the  end  of 

each  period 

$250  X  8.530202  =  $2,132.55,   present   value   of  the   annuity   of 
$250  for  10  periods  at  3% 


BOND  AND  BOND  INTEREST  VALUATION 


393 


Solution,  Part  5 

$7,243.02  +  $2,132.55  =  $9,375.57,  present  value  of  serial  bonds, 

with  all  interest  payments 

VERIFICATION  OF  CALCULATION  OF  THE  VALUE  OF  A  SERIES 
OF  SERIAL  REDEMPTION  BONDS 


Knd  of 
Period 

1.. 

2 

3 

4  . 

5 

6 

7 

8 

9 
10 
11 
12 
13 
14 
15 
16 
17 
18 
19 
20.. 


Bonds        Effective     Coupon 
Redeemed      Interest      Interest 


$2,000.00 
2,000  00 
2,000  00 
2,000  00 
2,000.00 


$281.27 

$250  00 

282.20 

250  00 

283.17 

250  00 

284.17 

250  00 

285  19 

250  00 

286  .  25 

250  00 

287.  S3 

250.00 

288.45 

250.00 

289.61 

250.00 

290.80 

250  00 

292  02 

250  00 

293.28 

250  00 

234.58 

200  00 

235  62 

200  00 

176.69 

150  00 

177.49 

150  00 

118.31 

100  .  00 

118  86 

100  00 

59  43 

50  00 

59.71 

50  00 

Amortization 

Bonds  Less 

of  Discount 

Discount 

$9,375.57 

$31  27 

9,406  84 

32  20 

9,439  04 

33.17 

9,472.21 

34  17 

9,506  38 

35.19 

9,541  57 

36  25 

9,577  82 

37.33 

9,615.15 

38  45 

9,653.60 

39  61 

9,693  21 

40  80 

9,734.01 

42  02 

9,776.03 

43  28 

7,819  31 

34  58 

7,854.89 

35  62 

5,889  51 

26  69 

5,916  20 

27  49 

3,943  69 

IS  31 

3,962  00 

18  86 

1,980  86 

9  43 

1,990  29 

9  71 

0  00 

Problems 

1.  The  State  Highway  Department  of  Michigan  desires  to  know  the  value 
of  a  series  of  road  bonds  which  it  is  about  to  issue.     The  bonds  will  have  a  pal- 
value  of  $200,000,  and  will  bear  5%  interest,  payable  semiannually.     They  are 
to  be  redeemed  serially,  in  installments  of  $40,000.     The  first  redemption  pay- 
ment is  to  be  made  one  year  from  the  date  of  issue,  and  the  other  payments 
are  to  be  made  annually  thereafter.     Money  is  worth  6%,  interest  compounded 
semiannually.     Find  the  value  of  the  bonds,  and  draw  up  a  table  of  analysis 
as  a  proof  of  your  solution. 

2.  A  corporation  wishes  to  float  an  issue  of  serial  bonds  for  $100,000.     These 
bonds  are  to  be  redeemed  in  yearly  installments  of  $20,000,  the  first  redemption 
payment  to  be  made  at  the  beginning  of  the  sixth  year.     The  interest  rate  is  5%, 
payable  semiannually,  and  the  effective  rate  is  4^%,  interest  convertible  semi- 
annually.    Draw  up  a  table  of  analysis.     From  the  analysis,  prepare  the  formula 
and  solution. 

3.  An  issue  of  serial  bonds  bearing  4%  interest,  payable  semiannually,  is  to 
be  redeemed  serially  in  installments  of  $4,000.     The  first  redemption  payment 
is  to  be  made  at  the  end  of  the  fifth  year,  and  the  other  payments  are  to  be 
made  annually  thereafter.     At  what  price  must  $20,000  of  these  serial  bonds 
be  purchased  in  order  to  net  the  purchaser  5%  annual  effective  interest? 


394  BOND  AND  BOND  INTEREST  VALUATION 

4.  An  issue  of  $50,000  of  bonds  bearing  interest  at  5%,  payable  semiannually,, 
is  sold  to  produce  5^%  effective  interest,  convertible  semiannually.  The  bonds 
are  to  be  retired  serially,  as  follows: 

$10,000  at  104  in  6  years 
10,000  at  103  in  7  years 
10,000  at  102  in  8  years 
10,000  at  101  in  9  years 
10,000  at  par  in  10  years 

Set  up  a  schedule  in  columnar  form,  showing  the  book  value,  cash  interest, 
effective  interest,  amortization  of  discount,  and  par  value  of  bonds  outstanding 
each  year. 

Bonds  redeemable  from  a  fund.  Frequently,  bonds  are 
redeemed  periodically  from  a  fund;  that  is,  as  soon  as  money  is  put 
into  the  fund,  or  when  money  becomes  available  at  the  end  of  an 
interest  period,  it  is  at  once  used  to  redeem  outstanding  bonds. 
No  difficulty  would  be  encountered  in  making  the  computations 
necessary  in  this  system  of  redemption,  except  for  the  fact  that 
bonds  are  usually  issued  in  denominations  of  $100,  $500,  or  $1,000, 
and  the  payments  into  the  fund  may  exceed  the  expenditures  from 
the  fund  for  the  redemption  of  bonds.  A  residue  would  then  be 
left  in  the  fund,  and  this  residue  should  earn  interest. 

Example 

The  X.Y.Z.  Company  issues  $100,000  of  bonds,  par  value  $100,  and  sets  up 
a  sinking  fund  for  their  periodic  redemption.  The  bond  interest  and  the  sinking 
fund  interest  are  each  6%.  The  bonds  arc  to  run  for  5  years,  with  interest 
payable  semiannually,  and  are  to  be  kept  alive  in  the  treasury.  Sinking  fund 
payments  are  to  be  made  semiannually.  Interest  is  to  be  paid  by  the  trustee 
on  the  balance  remaining  in  the  fund.  Show:  (a)  the  periodic  payments  into 
the  sinking  fund;  (b)  the  interest  accrued  periodically  on  bonds  redeemed;  (c)  the 
total  amount  that  must  be  invested  in  the  sinking  fund  each  period;  (d)  the 
amount  of  bonds  purchased  periodically  for  the  sinking  fund;  (e)  the  cash  balance 
held  in  the  treasury;  and  (/)  the  interest  on  the  cash  balance  held  in  the  treasury. 

Let:  i  =  the  effective  interest  per  period. 

And:  n  =  the  number  of  periods. 

Formula  Arithmetical  Substitution 

=  R  —  —  - —  X  $100,000  =  $11,723.05 

\J~  (1.03V0 
~~  .03 

From  Table  6,  page  534,  the  value  of is  found  to  be  .1172305, 

1  ""  (L03T10 

.03 
and  .1172305  X  $100,000  =  $11.723.05. 


BOND  AND  BOND  INTEREST  VALUATION 


395 


The  remaining  part  of  the  solution  can  be  derived  from  the  following  table: 


Periodic 
Payment 

Balance 
from 
Preceding 
Period 

Total 
Amount 
Available 

Bond 
Interest 

Bonds 
Re- 
deemed 

Cash 
Balance 

Interest 
on  Cash 
Balance 

Bonds 
Out- 
standing 

$100,000 

$11,723 

05 

$11,723 

.05 

$3,000 

$  8,700 

$23 

05 

$ 

.69 

91,300 

11,723 

05 

$23  74 

11,746 

79 

2,739 

9,000 

7 

79 

23 

82,300 

11,723 

05 

8  02 

11,731 

07 

2,469 

9,200 

62 

07 

1 

86 

73,100 

11,723 

05 

63.93 

11,786 

98 

2,193 

9,500 

93 

98 

2 

82 

63,600 

11,723 

05 

96  80 

11,819 

85 

1,908 

9,900 

11 

85 

.36 

53,700 

11,723 

05 

12  21 

11,735 

26 

1,611 

10,100 

24 

26 

.73 

43,600 

11,723 

05 

24  99 

11,748 

.04 

1,308 

10,400 

40 

01 

1 

.20 

33,200 

11,723 

05 

41  24 

11,764 

29 

996 

10,700 

68 

29 

2 

.05 

22,500 

11,723 

05 

70  34 

11,793 

39 

675 

11,100 

18. 

39 

,55 

11,400 

11,723 

06 

18.94 

11,742. 

00 

342 

11,400 

In  the  above  example,  the  first  payment  into  the  fund  is 
$11,723.05.  The  payment  on  the  interest  will  be  $3,000,  leaving 
$8,723.05  to  be  used  for  the  redemption  of  bonds.  As  the  bonds 
issued  are  of  $100  denomination,  only  $8,700  of  this  fund  can  be 
used  for  the  redemption  of  bonds,  leaving  a  balance  of  $23.05  cash 
in  the  hands  of  the  trustee. 

In  order  to  verify  the  solution  of  a  problem  of  this  kind,  it  is 
necessary  to  charge  the  trustee  with  interest  at  the  sinking  fund 
rate  on  the  balance  remaining  in  his  hands.  The  interest  on 
$23.05,  the  cash  balance  in  the  trustee's  hands,  for  6  months  at 
3%  is  $.69.  This  interest  added  to  the  cash  balance  gives  the 
balance  from  the  preceding  period. 


Problems 

1.  A  $200,000  bond  issue,  maturing  in  8  years,  bears  interest  at  6%,  payable 
semiannually.     The  par  value  of  the  bonds  is  $1,000.    A  sinking  fund  is  set  up, 
and  the  trustee  is  to  purchase  bonds  semiannually  at  par  and  keep  them  alive 
in  the  treasury.     Money  is  worth  6%,  interest  convertible  semiannually.     Pre- 
pare a  table,  showing:  (a)  the  semiannual  periodic  payments  to  the  sinking  fund; 
(b)  the  interest  accrued  on  the  bonds;  (c)  the  par  value  of  the  bonds  purchased 
semiannually;  (d)  the  cash  balance  in  the  hands  of  the  trustee;  and  (e)  the  interest 
on  the  cash  balance. 

2.  An  issue  of  $200,000  of  6%,  10-year  bonds  is  floated  by  a  corporation. 
The  par  value  is  $100,  arid  the  interest  coupons  are  payable  semiannually.     The 
bonds  are  to  be  redeemed  semiannually,  and  the  sinking  fund  payments  neces- 
sary for  the  redemption  of  the  principal  and  the  payment  of  the  interest  are 
placed  in  the  hands  of  the  trustee.    The  sinking  fund  rate  is  6%.     Prepare  a 
table,  as  in  Problem  1. 

Effective  rate  of  interest  on  bonds.  One  question  which  is 
very  important  to  the  investor  is,  ''What  rate  of  interest  will  I 
receive  on  this  bond?"  or  "What  will  be  the  effective  rate  on  the 


396  BOND  AND  BOND  INTEREST  VALUATION 

money  in  vested  ?"  The  investor  knows  the  amount  of  each  inter- 
est coupon,  but  the  coupon  rate  is  based  on  par  value  and  not  on 
the  amount  of  money  which  has  been  invested. 

Because  of  the  fact  that  the  unknown  effective  interest  rate 
must  be  used  more  than  once  in  the  algebraic  formula  for  the  cal  - 
culation  of  the  price  of  a  bond,  the  absolute  effective  rate  is  difficult 
to  find. 

However,  two  methods  which  give  a  close  approximation  may 
be  used.  The  rate  may  be  found : 

(1)  By  the  use  of  bond  tables,  or  test  rates,  and  interpolation. 

(2)  By  formulas  specially  constructed  to  give  approximately 
the  required  rate.     Most  formulas,  and  sometimes  a  method  of 
averages,  will  give  the  rate  accurately  enough  for  ordinary  purposes. 

Effective  rate  on  bonds  sold  at  a  premium. 

Example 

If  a  $100,  3-year,  6%  bond,  bearing  semiannual  interest  coupons,  is  bought 
at  $105.38,  what  rate  of  interest  will  be  realized  on  the  investment? 

SECTION  OF  BOND  TABLE 

CASH  INTEREST  PAYABLE  SEMIANNUALLY 


Effective 
Interest 

Rate 

3% 

$"2T  /o 

4%         4\% 

5% 

6% 

7% 

3  75 

$97  89 

$99  30 

$100 

70 

$102 

.11 

$103 

.52 

$106 

33 

$109  14 

3  80 

97.75 

99.16 

100 

56 

101 

97 

103 

.37 

106 

18 

108  99 

3.875 

97.54 

98  95 

100 

35 

101 

75 

103 

16 

105 

96 

108.77 

3  90 

97  48 

98  88 

100, 

,28 

101 

.68 

103 

09 

105 

89 

108  70 

4.00 

97.20 

98  60 

100 

00 

101 

.40 

102 

80 

105 

.60 

108  40 

4  10 

96  92 

98  32 

99 

72 

101 

12 

102 

52 

105 

31 

108  11 

4.12i 

96.86 

98.25 

99 

65 

101 

05 

102 

.45 

105 

24 

108  04 

4.20 

96.65 

98.05 

99 

44 

100 

84 

102 

.23 

105 

.02 

107  82 

4.25 

96.51 

97.91 

99, 

,30 

100 

.70 

102 

.09 

104 

.88 

107  67 

Solution 

In  the  6%  column  of  the  section  of  the  bond  table  given  above  will  be  found, 
opposite  4%,  the  price  of  $105.60,  and  opposite  4.1%,  the  price  of  $105.31. 
The  rate  is  therefore  somewhere  between  4%  and  4.1%.  A  more  exact  approxi- 
mation may  be  determined  as  follows: 

Interpolation 

Value  on  a  basis  of  4% $105  60 

Value  on  a  basis  of  4.1  % 105  31 

Difference  in  value  caused  by  a  difference  in  rate  of  .1  %. .   $        .29 

Value  on  a  basis  of  4%   $105  60 

Price  paid 105.38 

Difference  between  price  paid  and  value  on  a  basis  of  4  % .  $       .22 

If  the  difference  between  the  price  paid  and  the  value  on  a  basis  of  4%  is 
$.22,  and  the  difference  in  the  value  caused  by  a  difference  of  .1%  in  the  rate 
is  $.29,  the  rate  earned  will  be  ff  of  .1  %  greater  than  4%,  or  4,070%. 


BOND  AND  BOND  INTEREST  VALUATION  397 

Effective  rate  on  bonds  sold  at  a  discount. 

Example 

What  will  be  the  rate  of  income  on  an  investment  in  a  4%,  semiannual,  3-year 
bond  bought  at  $99.35? 

Solution 

By  reference  to  the  table  above,  the  computation  may  be  made  as  follows: 

Value  of  a  4%  bond  at  4.2%  effective  interest       .    .    .    $99  44 

Value  of  a  4%  bond  at  4.25  %  effective  interest 99  30 

Difference  in  value  caused  by  a  difference  in  rate  of  .05  %  $       1 4 

Value  on  a  basis  of  4.2% $99  44 

Price  of  bond 99  35 

Difference $     .09 

A  of  -05% 032% 

4.20%  +  .032% 4.232% 

Hence,  the  approximate  rate  is  4.232%. 

Computation  when  bond  table  is  not  available.  If  no  bond 
table  is  available,  an  approximate  rate  may  be  computed  by  the 
use  of  test  rates,  but  care  must  be  exercised  in  the  choice  of  the 
rates,  which  must  be  as  close  to  the  actual  rate  as  it  is  possible  to 
estimate. 

Example 

If  a  $100,  5%,  5-year  bond,  with  interest  payable  semiannually,  is  bought  at 
$97.31,  what  rate  of  interest  will  be  realized  on  the  investment? 

Solution 

The  first  step  is  to  find  the  cost  at  an  estimated  effective  rate.  Assume  this 
rate  to  be  5^%.  By  the  formula,  previously  given,  for  finding  the  purchase 
price  of  bonds,  this  bond  at  5.5%  is  worth  $97.84.  As  this  price  is  higher  than 
the  price  paid,  the  rate  is  too  low,  and  it  is  necessary  to  try  a  higher  rate.  Make 
a  second  trial  at  5.75%.  The  purchase  price  is  then  found  to  be  $96.78.  As 
one  price  is  slightly  above  and  the  other  slightly  below  the  actual  price  paid,  the 
approximate  rate  can  be  found  by  interpolation. 

Interpolation 

Value  on  a  basis  of  5.50% $97  84 

Value  on  a  basis  of  5.75% 96_Z? 

Difference  in  value  caused  by  a  difference  in  rate  of  ^  %  .  .  $  1 . 06 

Value  on  a  basis  of  5.50  %     $97^*84 

Price  paid  for  bond 97  31 

Difference  between  price  at  5.50%  and  price  paid $     .53 

Since  the  difference  between  the  price  paid  and  the  price  at  5.50%  is  $.53, 
and  the  difference  between  the  value  on  a  5.50%  basis  and  the  value  on  a  5.75% 
basis  is  $1.06,  the  rate  earned  will  be  approximately  5.50%  plus  y5^  of  the 
difference  between  5.50%  and  5.75%,  or  5.625%. 

Approximation  by  averages.  A  fair  approximation  may  be 
made  by  the  use  of  averages.  In  the  above  example,  the  rate 
would  be  found  as  shown  on  the  next  page. 


398 


BOND  AND  BOND  INTEREST  VALUATION 


Semiannual  yield      $    2. 50 

Total    gain    on    redemption,    $100  — 

$97.31  =  $2  69 
Average  gain  on  redemption  ($2.69  -f- 

10) .269 

Average  yield  per  period     .       ...  $     2  769 
Capital  invested  at  beginning  of  10th 

period  before  maturity $  97  31 

Capital  invested  at  beginning  of  last 

period  before  maturity  ($100  -  .269)  99  731 

Total $197041 

Dividing  by  2,  to  find  average  capital  $  9S  5205 

Average  yield  -f-  average  capital     .  Rate 

2.769  -T-  9S.5205     ..               .       ...  02811 

Multiplying  by  2                             .  05622,  or  5  622% 

Correct  yield  .                         0562468,  or  5.625% 

Problems 

In  each  of  the  following,  find  the  effective  rate  by  two  methods:  (1)  by  the 
use  of  bond  tables,  or  test  rates,  and  interpolation;  (2)  by  the  use  of  averages. 


1. 
2. 
3. 
4. 
5. 
6. 
7. 
8. 
9. 
10. 


Purchase 

Nominal 

Price 

Rate 

$      92  00 

4% 

1,015  00 

4% 

$9,750  00 

4% 

545  00 

5% 

925  00 

5% 

983  75 

5% 

9,732  50 

5J% 

5,201  50 

5£% 

96  85 

6% 

73  55 

6% 

Par        Time  in        Interest 


Value 

$      100  00 

1,000  00 

10,000  00 

500  00 

1,000  00 

1,000  00 

10,000  00 

5,000  00 

100  00 

100  00 


Effective  Kate 


Fears          Payable  Method  I      Metho 


3 

9 

29 

8 

15 

18 

5 

9 

4 

6 


Scmianiuuilly 


Amortization  of  discount,  premium,  or  discount  and  expense 
on  serial  redemption  bonds.  The  calculation  of  the  amortization 
of  bond  discount,  bond  premium,  or  bond  discount  and  expense, 
when  bonds  are  to  be  redeemed  serially,  or  in  unequal  amounts,  is 
a  problem  which  requires  particular  attention,  since  the  distribu- 
tion over  the  period  of  years  must  be  equitable.  The  two  methods 
which  are  most  commonly  used  are : 

(1)  The  bonds  outstanding  method. 

(2)  The  scientific  or  effective-interest  method. 

Bonds  outstanding  method.  It  would  be  incorrect  to  write  off 
the  discount  or  premium,  or  the  discount  and  expense,  on  serial 
redemption  bonds  or  on  a  bond  issue  which  has  no  regular  redemp- 
tion period,  by  the  straight-line  method.  In  some  cases  it  is 
difficult  to  calculate  the  portion  to  be  amortized  by  the  scientific 
method.  Because  of  the  ease  of  the  calculations  and  the  fair 


BOND  AND  BOND  INTEREST  VALUATION 


399 


degree  of  accuracy  which  it  affords,  the  bonds  outstanding  method 
is  the  one  most  commonly  used. 

Procedure :  (a)  Find  the  sum  of  the  bonds  outstanding  during 
each  period  of  the  life  of  the  bond  issue. 

(6)  Use  the  sum  found  in  (a)  as  the  denominator  of  a  series  of 
fractions,  and  use  the  sum  of  the  bonds  outstanding  each  period  as 
successive  numerators.  The  sum  of  all  these  fractions,  of  course, 
will  in  every  problem  be  equal  to  1. 

(c)  Multiply  the  total  bond  discount  or  the  premium,  or  the 
total  bond  discount  arid  expense,  by  the  appropriate  fraction,  to 
obtain  the  portion  of  discount  or  premium  to  be  amortized  each 
period. 

Example 

An  issue  of  ten  bonds  of  $1,000  each,  bearing  5%  interest,  payable  semi- 
annually,  is  to  be  redeemed  as  follows:  $3,000  at  the  end  of  the  sixth  year;  $3,000 
at  the  end  of  the  eighth  year;  and  $2,000  at  the  end  of  each  year  thereafter. 
The  bonds  are  sold  at  a  discount  of  $400.  Compute  an  equitable  amortization 
ot  the  discount  over  the  life  of  the  bonds. 


Solution 


Periods  of  On  e-h  a  If        Bonds 

Amortization 

Discount 

Bonds  Less 

Year  Each 

Outstanding 

Fraction 

Written  Off 

on  Bonds 

Discount 

$10,000  00 

$400  00 

$9,600  00 

1 

10,000  00 

10/160 

$  25  00 

375  00 

9,625  00 

2 

10,000.00 

10/160 

25  00 

350  00 

9,650.00 

3 

10,000  00 

10/160 

25  00 

325  00 

9,675  00 

4 

10,000  00 

10/160 

25  00 

300  00 

9,700  00 

5 

10,000  00 

10/160 

25  00 

275  00 

9,725  00 

6 

10,000  00 

10/160 

25  00 

250  00 

9,750.00 

7 

10,000  00 

10/160 

25  00 

225.00 

9,775  00 

8 

10,000.00 

10/160 

25  00 

200.00 

9,800.00 

9 

10,000.00 

10/160 

25  00 

175.00 

9,825  00 

10 

10,000  00 

10/160 

25  00 

150  00 

9,850  00 

11 

10,000  00 

10/160 

25  00 

125  00 

9,875.00 

12 

7,000  00 

10/160 

25  00 

100  00 

6,900  00 

13 

7,000  00 

7/160 

17  50 

82  50 

6,917  50 

14 

7,000  00 

7/160 

17  50 

65  00 

6,935  00 

15 

7,000  00 

7/160 

17  50 

47  50 

6,952  50 

16 

4,000  00 

7/160 

17  50 

30  00 

3,970  00 

17 

4,000  00 

4/160 

10  00 

20  00 

3,980  00 

18 

2,000.00 

4/160 

10  00 

10  00 

1,990  00 

19 

2,000  00 

2/160 

5.00 

5  00 

1,995  00 

20 

000  00 

2/160 

5.00 

0.00 

000  00 

160/160 

400.00 

400  BOND  AND  BOND  INTEREST  VALUATION 

Scientific  method.  To  find  by  the  scientific  method  the 
amount  of  discount  or  premium  to  be  amortized  on  an  issue  of 
serial  redemption  bonds,  it  is  necessary  to  find  the  approximate 
effective  rate  of  interest  which  these  bonds  will  bear;  and  to  find 
the  approximate  effective  rate  it  is  necessary  to  use  other  approxi- 
mations. 

Procedure:  (a)  Determine  the  average  life  of  the  bonds,  in 
periods. 

(6)  Determine,  by  the  use  of  a  bond  table  or  by  annuity  calcu- 
lations, the  approximate  effective  interest  rate  for  one  bond  having 
a  life  of  the  same  number  of  periods  as  the  average  life  found  in  (a). 

(c)  From  a  bond  table  or  by  annuity  calculations,  find  the 
value  of  one  bond  at  each  of  the  annual  redemption  dates,  at  the 
approximate  effective  rate  found  in  (6). 

(d)  By  using  the  values  found  in  (e),  find  the  total  value  of  the 
bonds  redeemed  at  each  redemption  date. 

(e)  Add  the  values  found  in  (d). 

(/)  Compare  the  sum  found  in  (e)  with  the  actual  price  received 
for  the  bonds. 

(</)  By  the  same  process,  determine  another  approximate  rate. 

(h)  By  interpolation,  determine  the  error,  using  the  approxi- 
mate rates  found  above. 

The  computation  of  the  periodic  amortization  is  comparatively 
simple  when  the  cost  of  the  serial  redemption  bonds,  the  nominal 
or  coupon  rate,  and  the  effective  rate  of  interest  are  known.  Val- 
uation of  each  member  of  the  series  is  equally  simple.  Refer  to  the 
table  on  page  402,  which  shows  the  amortization  of  discount  for  a 
series  of  serial  redemption  bonds.  The  periodic  amortization  is  the 
difference  between  the  effective  income  and  the  actual  cash  income. 
It  may  be  observed  that  the  difficulty  of  the  whole  calculation  is 
the  determination  of  the  effective  rate. 

The  example  that  was  previously  given  under  the  bonds  out- 
standing method  would  be  solved  by  the  scientific  method  as 
follows : 

Solution 
(a) 


Bonds 

Maturing  In 

Product 

3 

6  years 

18 

3 

8     " 

24 

2 

9     " 

18 

2 

10     " 

20 

10 

80 

80  -r  10  =  8,  or  an  average  life  of  8  years 
9,600  -T-  100  =  $96,  the  average  sales  price  per  hundred 


BOND  AND  BOND  INTEREST  VALUATION  401 

(6)  Refer  to  a  bond  table  which  shows  the  values  at  different  effective  rates 
of  a  5%  bond  maturing  in  8  years,  with  interest  payable  semiannually.  The 
price  nearest  to  96  is  found  to  be  96.02.  This  amount  is  opposite  the  effective 
rate  of  5f%. 

(c)  The  next  step  is  to  find  the  value  at  5f  %  of  all  the  bonds  in  the  series. 
To  do  this,  refer  to  the  bond  table  which  shows  a  cash  rate  of  5%  and  an  effective 
rate  of  5f%. 

(d)  The  results  will  be: 


Par  Value      Years       Value  at 
of  Bonds      to  Run        <5f%        Total  Value 


$3,000  00 

6 

$96.85 

$2,905  50 

3,000  00 

8 

96  02 

2,880.60 

2,000  00 

9 

95  63 

1,912  60 

2,000.00 

10 

95.27 

1,905  40 

(e)  Value  of  series $9,604  10 


(/)  As  the  sales  price  of  $9,600  for  the  bonds  is  below  the  price  based  on 
a  5f  %  rate,  the  effective  rate  is  a  little  larger  than  5f%.  A  test  at  5|%  would 
result  in  the  following: 


Par  Value 

Years 

Value  at 

of  Bonds 

to  Rim 

*f% 

Total  Value 

$3,000  00 

6 

$96  24 

$2,887  20 

3,000  00 

8 

95  24 

2,857  20 

2,000  00 

9 

94  79 

1,895  80 

2,000  00 

10 

94  36 

1,887  20 

Value  of  series 19,527  40 

(h) 


Interposition 

Value  of  series  at  5f  % $9,604  10 

Value  of  series  at  5f  % 9,52740 

Difference  caused  by  a  difference  in  rate  of  -g-%  $      76. 70 

Value  of  series  at  5g  % $9^604710 

Sales  price     ...  ...  .     9,600  JX) 

Difference .  .  $        47IO 

410/7670  of  fc%  .         ...  ""     1)06682 

Trial  rate ~™^T625™ 

Add  fractional  rate  found  .006682 

Effective  rate ITJI6J16^ 

Effective  rate  to  be  used  semiannually  (5.631682 

-7-2)   2.815841% 


402 


BOND  AND  BOND  INTEREST  VALUATION 


TABLE  SHOWING  AMORTIZATION  OF  DISCOUNT  FOR  SERIES 
OF  SERIAL  REDEMPTION  BONDS 


End  of        Bonds 

Effective  Rate, 

Coupon  Rate, 

Amortization 

Bonds  Less 

Period      Redeemed 

2.815841  % 

2.5% 

of  Discount 

Discount 

$9,600  00 

1 

$270.32 

$250.00 

$20  32 

9,620  32 

2 

270  89 

250  00 

20  89 

9,641  21 

3 

271  48 

250  00 

21   48 

9,662.69 

4 

272  09 

250  00 

22  09 

9,684  78 

5 

272.71 

250  00 

22  71 

9,707  49 

6 

273  35 

250  00 

23  35 

9,730  84 

7 

274  01 

250  00 

24  01 

9,754  85 

8 

274  68 

250  00 

24.68 

9,779  53 

9 

275  38 

250.00 

25  38 

9,804  91 

10 

276  09 

250  00 

26.09 

9,831  00 

11 

276  83 

250  00 

26  83 

9,857  83 

12        $3,000  00 

277  58 

250  00 

27  58 

6,885  41 

13 

193  88 

175.00 

18.88 

6,904  29 

14 

194.41 

175.00 

19  41 

6,923  70 

15 

194  96 

175.00 

19  96 

6,943  66 

16           3,000  00 

195.52 

175.00 

20  52 

3,964  18 

17 

111.63 

100  00 

11.63 

3,975  81 

18           2,000  00 

111  95 

100  00 

11  95 

1,987  76 

19 

55  97 

50.00 

5  97 

1,993  73 

20           2,000  00 

56  14 

50  00 

6  14 

.13* 

*  Error  caused  by 

approximation 

and  by  the  use 

of  bond  tables 

having  only 

places. 

Problems 

1.*  On  January  1,  1934,  a  corporation  floated  a  bond  issue  of  $300,000  to  be 
retired  serially  over  a  period  of  8  years  as  follows : 


Dec.  31,  1934 
Dec.  31,  1935 
Dec.  31,  1936 
Dec.  31,  1937 


$10,000  Dec.  31,  1938  $  30,000 

15,000  Dec.  31,  1939  .           35,000 

20,000  Dec.  31,  1940  40,000 

25,000  Dec.  31,  1941  125,000 


The  discount  and  expense  of  issuing  the  bonds  amounted  to  $33,000. 

Draft  a  schedule,  showing  how  much  of  such  bond  discount  and  interest  you 
would  claim  as  a  deduction  from  gross  income  for  federal  income  tax  purposes 
for  each  of  the  years  1934  to  1941,  inclusive. 

2.*  A  city  wishes  to  buy  new  fire  equipment.  The  cost  will  be  $500,000, 
and  the  equipment  will  have  an  estimated  life  of  10  years,  and  no  salvage  value. 
It  is  necessary  to  issue  bonds  to  pay  for  this  purchase,  although,  at  the  present 
time,  interest  rates  are  high — 6  %,  payable  annually. 

How  would  you  suggest  that  these  bonds  be  issued,  and  what  will  be  the 
annual  cost  to  the  taxpayers? 

It  is  expected  that  a  sinking  fund  would  not  earn  more  than  an  average 
of  3%. 


'  American  Institute  Examination. 


BOND  AND  BOND  INTEREST  VALUATION  403 

The  bonds  will  be  issued  in  denominations  of  $100  and  multiples  thereof. 

Given: 

(LOG)9  =  1.679479  (1.06)10  =  1.790848 

(1.03)9  =  1.304773  (1.03)10  =  1.343916 

3.*  A  series  of  5%  bonds  totalling  $100,000,  dated  January  1,  1934,  is  redeem- 
able at  par  by  ten  annual  payments  of  $10,000  each,  beginning  December  31, 
1944.  What  equal  annual  payments  to  a  sinking  fund  are  required  to  be  pro- 
vided on  a  4%  basis  in  order  to  pay  off  the  bonds  as  they  mature? 

The  first  payment  to  the  sinking-fund  trustees  is  to  be  made  on  December  3 1 , 
1934,  and  the  further  payments  are  to  be  made  annually  thereafter. 

What  is  the  status  of  the  sinking  fund  on  December  31,  1943,  1944,  and 
1945? 

Given  at  4%: 

(1  +  t)10  =  1.48024428  (1  +  i)20  =  2.19112312 

v™  =  .67556417  v20  =  .45638695 

Given  at  5%: 

(1  +  t)10  =  1.6288946  (1  +  i)20  =  2.6532977 

i'10  =    .6139133  v20  =    .3768895 

4.  On  June  1,  a  corporation  sold  a  $3,000,000  issue  of  6%,  first  mortgage, 
20-year  bonds  at  a  discount  of  $300,000.     According  to  the  terms  of  sale  these 
bonds  were  to  be  retired  at  the  rate  of  $150,000  each  year,  the  purchases  to  be 
made  in  the  open  market.     The  first  retirement  in  the  amount  of  $75,000  was 
to  be  made  on  March  1  following  the  date  of  issue,  and  $75,000  was  to  be  retired 
each  six  months  thereafter. 

Any  premium  paid  or  discount  received  on  bonds  purchased  for  retirement 
was  to  }>e  added  or  deducted,  whichever  the  case  might  be,  to  that  year's  portion 
of  discount,  and  was  to  be  amortized  as  shown  by  the  schedule  of  amortization. 

Prepare  a  schedule,  showing  the  amortization  of  bond  discount  by  the  bonds 
outstanding  method. 

5.  An  issue  of  $175,000  of  6^%  bonds  was  sold  for  95,  the  amount  of  the 
discount  being  $8,750.     Other  expenses  pertaining  to  the  issue  of  the  bonds 
amounted  to  $596.67,  making  the  total  of  bond  discount  and  expense  $9,346.67. 
These  bonds  are  dated  March  30,  1944,  arid  the  due  dates  are  as  follows: 

$  3,500  due  April  15,  1944  $  12,000  due  April  15,  1949 

5,000  due  April  15,  1945  14,500  due  April  15,  1950 

7,000  due  April  15,  1946  15,000  due  April  15,  1951 

8,000  due  April  15,  1947  100,000  due  April  15,  1952 

10,000  due  April  15,  1948 

Prepare  a  schedule,  showing  the  annual  charge  for  amortization  of  bond 
discount  and  expense  as  of  the  close  of  each  year,  December  31. 

6.f  On  April  1,  1934,  Southern  Railway  Equipment  Gold  4^'s,  due  serially 
on  each  successive  coupon  date  in  October  and  April  between  October  1,  1934, 
up  to,  and  including,  April  1,  1948,  were  sold  to  yield  4f  %.  The  Bank  of 


*  American  Institute  Examination. 

t  Moore,  Justin  H.,  Handbook  of  Financial  Mathematics.     New  York,  Prentice- 
Hall,  Inc.,  1929. 


404  BOND  AND  BOND  INTEREST  VALUATION 

Montrose  purchased  2  bonds  due  April  1,  1937,  4  due  October  1,  1938,  1  due 
October  1,  1942,  7  due  April  1,  1943,  and  1  due  April  1,  1945. 

(a)  What  was  the  total  price  paid?  (b)  Set  up  a  schedule  showing  the 
investor's  situation  in  regard  to  the  book  value  of  this  investment  at  the  begin- 
ning of  each  six-month  period  until  the  final  maturity. 

7.*  On  November  1,  1933,  an  investor  purchased  20  bonds,  each  with  a  par 
value  of  $1,000,  and  maturing  as  follows: 

Number 
of  Bonds 

May  1,  1934 1 

Nov.  1,  1934  .  3 

May  1,  1935  ...  2 

May  1,  1936 5 

May  1,  1937 9 

The  price  paid  for  the  20  bonds  was  $20,417.11.  The  coupon  rate  is  6%, 
payable  May  1  and  November  1. 

(a)  Determine  the  rate  of  yield,  (b)  Set  up  a  schedule  showing  the  investor's 
situation  in  regard  to  the  book  value  of  this  investment  at  the  beginning  of  each 
six-month  period  until  the  final  maturity. 

Review  Problems 

1.  A  serial  issue  of  $20,000  in  denominations  of  $1,000  with  interest  at  4% 
has  maturity  of  $1,000  at  each  interest  date,  March  1  arid  September  1.     Find 
the  price  at  the  date  of  issue,  March  1,  to  yield  the  purchaser  3^%. 

2.  If,  in  Problem  1,  the  first  maturity  were  to  occur  2  years  following  the 
date  of  issue,  and  then  each  six  months  thereafter,  find  the  price  at  the  date  of 
issue,  March  1,  that  would  yield  the  purchaser  3^ -%. 

3.  A  $1,000  bond  with  interest  at  4y%,  maturing  April  1,  1954  and  redeem- 
able at  102  on  any  interest  date  after  January  1,  1945,  was  sold  on  July  21,  1942 
on  a  5%  basis.     What  price  was  paid  for  it? 

4.  A  loan  of  $5,000  at  5%  payable  scmianiiually  is  repayable  on  each  interest 
date  in  installments  of  $1,250.     Find  the  purchase  price  to  yield  the  investor  4%. 

5.  A  loan  of  $5,000  with  interest  at  6%  payable  semiannually  is  to  run  5  years 
and  then  be  redeemed  in  annual  installments  of  $1,000.     What  is  the  purchase 
price  to  yield  5%  convertible  semiannually? 


*  Mooro,  Justin  H.,  Handbook  of  Financial  Mathematics      New  York,  Prentice- 
Hall,  Inc.,  1929. 


CHAPTER  34 
Asset  Valuation  Accounts 

Asset  valuation.  At  the  moment  that  an  asset  is  purchased 
for  use,  it  is  said  to  be  worth  cost.  When  this  asset  can  no  longer 
be  used  for  the  purpose  for  which  it  was  purchased,  and  can  be 
sold  for  little  or  nothing,  it  is  said  to  be  worth  scrap  value.  The 
difference  between  the  scrap  value  and  the  cost  is  the  depreciation. 

Depreciation.  Depreciation  is  the  decline  in  value  of  a  physi- 
cal asset  caused  by  wear,  tear,  action  of  the  elements,  obsolescence, 
supersession,  or  inadequacy  and  resulting  in  an  impairment  of 
operating  effectiveness. 

Depletion.  Depletion  is  the  progressive  extinction  of  a  wasting 
asset  by  a  reduction  in  the  quantity.  A  coal  mine  is  depleted 
year  by  year  as  the  coal  is  mined. 

Depreciation  methods.  There  are  many  methods  of  calculat- 
ing the  periodic  depreciation  charge.  Probably  the  following  are 
the  most  serviceable: 

(1)  Straight-line  method. 

(2)  Working-hours  method,  or  unit-product  method. 

(3)  Sum-of-digits  method. 

(4)  Sinking-fund  method. 

(5)  Annuity  method. 

(6)  Fixed-percentage-of-diminishing-value  method. 

The  accountant  should  know  how  to  calculate  depreciation  by 
each  method,  should  be  able  to  discuss  the  advantages  and  dis- 
advantages of  each,  and  should  be  able  to  formulate  tables  of 
comparison  showing  the  results  of  each. 

Straight -line  method.  This  is  the  simplest  method,  and  is  the 
one  most  commonly  used. 

Procedure:  (a)  Find  the  difference  between  the  cost  and  the 
scrap  value. 

(6)  Divide  the  difference  found  in  (a)  by  the  number  of  periods 
which  the  asset  is  expected  to  be  of  service.  The  result  is  the 
depreciation  charge  per  period. 

Example 

What  will  be  the  depreciation  charge  and  the  asset  valuation  at  the  end  of 
each  year  for  an  asset  costing  $1,000,  and  having  an  estimated  life  of  10  years 
and  an  estimated  scrap  value  of  $100? 

405 


406 


ASSET  VALUATION  ACCOUNTS 


Formula 

Cost  —  Scrap  =  Depreciation 
Depreciation  -f-  Number  of  years  =  Annual  charge 

A  rithmetical  Substitution 

$1,000  -  $100  -  $900 
$900  4-  10  =  $90 


TABLE  OF  DEPRECIATION 


Periodic 
Depreciation 
Years  Charge 


1 
2 
3 
4 
5 
6 
7 
8 
9 
10 


$90 
90 
90 
90 
90 
90 
90 
90 
90 
90 


Accumulated 

Depreciation 

Reserve 

$  90 
180 
270 
360 
450 
540 
630 
720 
810 
900 


Asset 
Value 
$1,000 
910 
820 
730 
f>4() 
550 
460 
370 
280 
190 
100 


Problems 

1.  Set  up  a  table  showing  the  annual  depreciation,  carrying  value,  and 
accumulated  depreciation  for  an  asset  costing  $3,500,  and  having  a  life  of  10 
years  and  an  estimated  scrap  value  of  $500. 

2.  Prepare  tables  showing  by  the  straight-line  method  the  depreciation  on 
the  following  machines: 


Assets  Cost 

Lathes        $5,000 

Milling  machines     3,000 

Power  equipment 4,100 

Furniture 600 


Estimated 

tier  a  p  Yalw 

$600 

400 

200 

150 


Estimated 

Life  in 

Years 

10 

12 

10 

15 


3.  The  following  fixed  assets  belong  to  the  Western  Hardware  Company: 


Asset  Cost 

Buildings  ..  $100,000 

Machinery 70,000 

Tools 20,000 

Patterns 10,000 


Estimated 
Scrap  Value 
$35,000 
25,000 
5,000 
none 


Estimated 

Life  in  Years 

20 

15 

10 

8 


Compute  the  amount  of  annual  depreciation  by  the  straight-line  method. 

4.  Complete  the  following  schedule  of  fixed  assets  and  depreciation,  for  the 
purpose  of  supporting  an  income  tax  return  (allow  six  months'  average  or 
additions) : 


ASSET  VALUATION  ACCOUNTS 


407 


Previous 

Current 

Cost 

Additions 

Kate 

Reserve 

Depreciation 

.   $75,000  00 

none 

none 

none 

none 

s 

92,519  61 

SI,  046  91 

3% 

$  9,461   92 

$     

ngs 

35,654  IS 

1,126  19 

5% 

12,319  14 

1  tools 

51,252  19 

9,217.62 

10% 

11,463  21 

B 

3,469  52 

417  51 

10% 

1,121   44 

-  . 

icks 

.       3,219  52 

1,750  19 

25% 

1,749  32 

712  92 
$""."...."./ 

none 

$""  "... 

5',o 

12S  34 
».."..."..".  

$ 

Asset 
Land 

Brick  buildings 
Wooden  buildings 
Machinery  and  tools- 
Office  furniture 
Automobile  trucks 
Spur  track 

Working -hours  or  unit-product  method.     This  mot  hod  is  based 

on  the  number  of  hours  which  the  asset  is  in  use,  or  on  the  number 
of  units  produced. 

Example 

A  certain  one-purpose  machine  which  costs  $1,000,  and  has  no  scrap  value, 
lias  been  installed  in  a  factory.  A  machine  of  this  class  produces  10,000  units 
of  product  during  its  life.  Assuming  that  the  annual  production  is  as  given 
below,  set  up  a  table  shoeing  the  depreciation  to  be  written  oil  each  year. 


First  year 

1,000  units 

Fifth  year 

1,000  units 

Second  year 

2,000      " 

Sixth  year 

1,200      " 

Third  year 

1,SOO      " 

Seventh  year 

1,200      " 

Fourth  year 

1,000      " 

Eighth  year   . 

.      800     " 

Solution 
TABLE  OF  DEPRECIATION 


I'm! 

Periodic 

A  ecu  mutated 

Fraction 

Depreciation 

Depreciation 

Asset 

Year 

of  Cost 

Charge, 

R(  serve 

Value 

$1,000 

[ 

1,000/10,000 

$100 

$    100 

900 

2 

2,000/10,000 

200 

300 

700 

3 

1,800/10,000 

ISO 

480 

520 

4 

1,000/10,000 

100 

580 

420 

5 

1,000/10,000 

100 

080 

320 

6 

1,200/10,000 

120 

SOO 

200 

7 

1,200/10,000 

120 

920 

80 

8 

800/10,000 

80 

1,000 

0 

Problems 

1.  Show  in  appropriate  form  the  yearly  depreciation,  accumulated  deprecia- 
tion, and  asset  value  of  a  machine  which  cost  $7,400,  and  which  will  have  a  scrap 
value  of  $200.     Assume  that  machines  of  this  class  have  a  working-hour  average 
life  of  24,000  hours;  also  assume  that  the  machine  will  be  run  as  follows: 

First  year  2,000  hours  Sixth  year  2,000  hours 

Second  year  2,000      "  Seventh  year  3,000      " 

Third  year  1,800      "  Eighth  year  3,000      " 

Fourth  year  2,600      "  Ninth  year  .       3,000      " 

Fifth  year  2,800       "  Tenth  year  1,800       " 

2.  An  aircraft  motor  costing  $7,700,  and  having  an  estimated  scrap  value 
of  $1,000,  is  assumed  to  have  a  useful  life  of  2,000  hours.     If  this  motor  is  oper- 


408  ASSET  VALUATION  ACCOUNTS 

ated  350  hours  during  a  certain  month,  what  should  be  the  charge  for  depreciation 
in  that  month? 

Sum-of -digits  method.  Those  who  believe  th  it  the  deprecia- 
tion charge  should  be  large  during  the  early  years  of  the  useful  life 
of  the  asset,  will  find  the  surn-of-digits  method  of  value. 

Procedure:  (a)  Find  the  sum  of  the  digits,  or  numbers  repre- 
senting the  periods  of  useful  life  of  the  asset.  Use  this  sum  as  the 
denominator  of  certain  fractions. 

(6)  Use  the  same  digits  or  numbers  in  inverse  order  as  the 
numerators  of  these  fractions. 

(c)  Compute  the  periodic  depreciation  by  multiplying  the 
total  depreciation  by  the  fractions  obtained  in  (a)  and  (6). 

Example 

An  asset  is  valued  at  $1,000,  and  has  a  scrap  value  of  $100.  What  should 
be  the  depreciation  charges  if  the  asset  is  to  be  written  down  in  9  years  by  the 
sum-of-digits  method? 

Solution 
TABLE  OF  DEPRECIATION 


Year 

1 
2 
3 
4 
5 
0 
7 
8 
9 
45  45/45  $900 

The  denominator  of  the  fractions  used  in  the  second  column  is  found  by 
adding  the  first  column.  The  numerators  are  the  same  numbers  taken  in 
inverse  order. 

Problem 

Compute  by  the  sum-of-digits  method  the  depreciation  charges,  the  asset 
valuation,  and  the  depreciation  provision  for  each  year  on  each  of  the  following 
machines : 

Life  of  Asset, 
Asset  Cost        Scrap  Years 

Power  lathe     $1,300       $200  10 

Hack  saw 350  38  12 

Turret  lathe .      3,000         200  7 

Boiler.    .    .  ...          2,700         180  8 

Power  equipment  .  800  50  5 

Delivery  equipment         .      .        .        2,000         500  5 

Present  the  information  by  means  of  table? 


Periodic 

Accumulated 

Fractional 

Depreciation 

Depreciation 

Part 

Charge 

Reserve 

Atawt  Value 

$1,000 

9/45 

$180 

$180 

820 

8/45 

160 

340 

660 

7/45 

140 

480 

520 

6/45 

120 

600 

400 

5/45 

100 

700 

300 

4/45 

80 

780 

220 

3/45 

b'O 

840 

160 

2/45 

40 

880 

120 

1/45 

20 

900 

100 

ASSET  VALUATION  ACCOUNTS  409 

Sinking-fund  method.  This  method  is  based  on  the  assump- 
tion that  a  fund  will  be  provided  to  replace  the  asset  at  the  expira- 
tion of  its  life.  Seldom  is  such  a  fund  provided,  but  the  method 
can  be  applied  without  actually  accumulating  the  fund.  The 
procedure  is  the  same  as  though  the  fund  were  actually  created. 

Procedure :  (a)  Find  the  amount  of  the  total  depreciation  of  the 
asset  by  deducting  the  scrap  value  from  the  cost. 

(6)  Divide  the  total  depreciation  found  in  (a)  by  the  amount 
of  an  ordinary  annuity  of  1  at  the  sinking  fund  interest  rate,  for  the 
number  of  periods  of  the  life  of  the  asset,  s^t-.  This  will  give  the 
periodic  sum  to  be  placed  in  the  sinking  fund. 

(c)  To  the  periodic  sum  found  in  (6),  add  a  sum  equal  to  the 
interest  on  the  sinking  fund  for  the  period.  This  will  give  the 
periodic  charge  to  depreciation,  and  the  credit  to  reserve  for 
depreciation. 

Example 

An  asset  costs  $1,000,  and  has  a  scrap  value  of  $100  at  the  end  of  10  years. 
Determine  the  periodic  depreciation  charge  by  the  sinking-fund  method,  on  a 
(>%  interest  basis. 

Formula  Arithmetical  Substitution 

Cost  -  Scrap        0    .    ..     .         .,  1,000-100 

—    —   =  Periodic  deposit  /,~^^r;,r~     ,   =  $(>8.28. 

to  sinking  fund.  (l^^l 
*  .06 

The  formula  and  substitution  just  shown  give  only  the  first  periodic  charge 
to  depreciation.  Each  periodic  charge  thereafter  is  an  amount  equal  to  the 
sum  of  the  first  periodic  payment  and  the  interest  on  the  accumulated  depreci- 
ation reserve.  Table  II,  on  page  410,  shows  the  periodic  charges  to  depreciation 
and  the  credits  to  the  reserve  account  for  each  of  the  10  years. 

The  depreciation  entries  are  independent  of  the  fund  entries.  If  a  sinking 
fund  were  provided  for  the  above  example,  the  entries  for  the  fund  would  be  as 
shown  in  Table  I: 

TABLE  I— ENTRIES  TO  THE  SINKING  FUND 

Debit  to          Credit  to       Credit  to     Accumulation 


Year 

Sinking  Fund 

Cash 

Interest 

of  Fund 

1 

$  68  28 

$  68.28 

$.-. 

$  68  28 

2 

72  38 

68.28 

4.10 

140  66 

3 

76  72 

68  28 

8.44 

217.38 

4 

81.32 

68  28 

13.04 

298  70 

5 

86.20 

68  28 

17  92 

384.90 

6 

91.37 

68  28 

23  09 

476.27 

7 

96.86 

68.28 

28  58 

573.13 

8 

102.67 

68.28 

34.39 

675.80 

9 

108  83 

68.28 

40  55 

784.63 

10 

115  37 

68.29 

47.08 

900.00 

$900.00  $682.81         $217.19 


410  ASSET  VALUATION  ACCOUNTS 

TABLE  II— DEPRECIATION  ENTRIES  BY  THE  SINKING-FUND 
METHOD  OF  DEPRECIATION 

Depreciation     Accumulated 


End  of 
Year 

Charge  and 
Reserve  Credit 

Depreciation 
Reserve 

Asset 
Value 

$1,000  00 

1 

$  68  28 

$  68  28 

931  72 

2 

72  38 

140  66 

859  34 

3 

76  72 

217  38 

782  62 

4 

81  32 

298  70 

701  30 

5 

86  20 

384  90 

615  10 

6 

91  37 

476  27 

523  7£ 

7 

96  86 

573.13 

426  87 

8 

102  67 

675  80 

324  20 

9 

108  83 

784  63 

215  37 

10 

115  37 

900  00 

100.00 

Problems 

1.  Set  up  sinking  fund  depreciation  tables  for  the  following  assets,  using  5% 
as  the  sinking  fund  rate. 

Asset                                   Cost  Scrap         Life 

Office  furniture  .                $      500  $    125  8  years 

Factory  furniture                                             1,000  100  10  years 

Machinery  ..  .                  15,000  3,000  8  years 

Delivery  equipment  .                   4,000  500  5  years 

2.*  The  owner  of  an  unimproved  building  site  who  is  desirous  of  developing 
it  so  that  it  will  produce  an  income,  receives  from  a  proposed  lessee  a  proposition 
relative  to  the  erection  of  a  building  at  a  cost  of  $100,000.  In  calculating  the 
annual  expenses,  which  are  to  be  made  the  basis  of  rentals,  the  owner  assumes 
a  life  of  50  years  for  the  proposed  building,  and  calculates  by  the  straight-line 
method  that  the  depreciation  charge  should  be  $2,000  per  year.  The  prospective 
lessee  contends  that  this  depreciation  charge  is  too  large,  that,  as  depreciation 
charged  into  expense  does  not  represent  actual  expenditures,  an  amount  of  cash 
equal  to  the  depreciation  charge  should  be  set  aside  annually  and  invested  in 
interest-bearing  securities,  and  the  interest  obtained  annually  reinvested.  He 
demonstrates  by  calculation  that  an  annual  depreciation  charge  of  $477.68  so 
handled  will  at  an  interest  rate  of  5%  amount  to  $100,000  at  the  end  of  50  years, 
and  hence  argues  that  this  amount  rather  than  $2,000  should  be  taken  into 
consideration  in  determining  the  annual  rental. 

You  are  asked  by  the  owner  to  give  your  opinion  as  to  which  of  the  two 
methods  should  be  used,  and  why.  Give  your  answer. 

Annuity  method  of  depreciation.  The  theory  applied  in  this 
method  is  that  the  depreciation  charge  should  include,  in  addition 
to  the  amount  credited  to  the  reserve,  interest  on  the  carrying 
value  of  the  asset. 

The  investment  in  property  is  regarded,  first,  as  the  amount  of 
scrap  value  which  draws  interest,  and  second,  as  an  investment  in 


*  C.  P.  A.,  Wisconsin. 


ASSET  VALUATION  ACCOUNTS  411 

an  annuity  to  be  reduced  by  equal  periodic  amounts.  The  interest 
on  the  scrap  value  plus  the  equal  periodic  reduction  of  the  invest- 
ment is  the  charge  to  depreciation,  offset  by  a  credit  to  interest 
computed  on  the  diminishing  value  of  the  property,  and  a  credit  to 
the  reserve  account  for  the  balance.  This  charge  to  depreciation 
is  the  same  each  period  during  the  life  of  the  property.  The  theory 
of  an  investment  in  an  annuity  is  that  the  annuity  is  to  be  reduced 
by  equal  periodic  payments,  and  as  the  credits  to  interest  wUl 
decrease,  the  credits  to  the  reserve  must  correspondingly  increase. 

Procedure:  (a)  Find  the  difference  between  the  cost  and  the 
scrap  value. 

(6)  Divide  the  difference  found  in  (a)  by  the  present  value  of 
an  annuity  of  1. 

(c)  Calculate  the  interest  on  the  scrap  value  for  one  period  at 
the  given  rate  per  cent. 

(d)  Determine  the  sum  of  (6)  and  (c).     This  sum  will  be  the 
periodic  charge  to  depreciation. 

Example 

Calculate  by  the  annuity  method  the  annual  charge  to  depreciation  for  an 
asset  valued  at  $1,000,  with  a  scrap  value  of  $100,  which  is  to  be  written  off  in 
10  years  on  a  6%  basis. 

Formula 

[  Cost  —  Scrap  1        .  .  .         ._    .    ..      . 

I    „__ _j_  (h(.ra,p  x  i)  =  Periodic  charge. 

L          ««I*  J 

A rith metica I  Kiibstit  ut  io  n 


(LOG)1 


Solution,  Par  I  1 

$1,000  —  $100  =  $900,  sum  to  be  depreciated 

(1.06)™  =  1.7908477,    compound    amount    of    1    for    10 

periods  at  6% 
1  -r-  1.790S477  -  .5583948,  present  value  of   1   for   10  periods 

at  6% 
1  -  .5583948  =  .4416052,    compound    discount   on    1    for    10 

periods  at  6% 
.4416052  -f-  .06  =  7.360087,  present  value  of  an  annuity  of  1  for 

10  years  at  6% 
$900  •*-  7.360087  =  $122.28,  rent  of  the  present  value  of  annuity 


412  ASSET  VALUATION  ACCOUNTS 

Solution,  Part  2 
$100  X  .06  *=  $6.00,  interest  on  scrap  value 

Solution,  Part  3 
$122.28  -h  $6.00  =  $128.28,  periodic  charge  to  depreciation 

In  the  above  example,  the  $900  represents  the  present  value  of  the  sum  to 
be  spread  over  the  life  of  the  asset,  and  the  $100  represents  the  scrap  value.  In 
the  following  tables,  the  fifth  column  always  contains  the  carrying  value  of  the 
annuity,  plus  $100.  The  two  tables  are  given  to  show  the  similarity  between  an 
annuity  in  which  an  investment  was  made  and  equal  annual  rents  withdrawn, 
and  the  annuity  method  of  depreciation. 

TABLE  OF  REDUCTION  OF  AN  ANNUITY 

End  of         Rents  Credits  to  Amortization  of  Present  Value  of 

Period     Withdrawn  Interest  Investment  Annuity,  Plus  $100 

$1,000  00 

1  $    128  28  $  60  00  $  68.28  931   72 

2  128  28  55  90  72  38  859  34 

3  128  28  51  56  76  72  782  62 

4  128  28  46  96  81.32  701   50 

5  128  28  42.08  86  20  615   10 

6  128.28  36.91  91  37  523  73 

7  12828  31.42  9686  42687 

8  128  28  25.61  102  67  324  20 

9  128  28  19.45  10S  83  215.37 
10              128  29  12  92  115  37  100.00 

£f,282  81  $382  81  $900. 00 

TABLE  OF  REDUCTION  OF  THE  VALUE  OF  AN  ASSET 

End  of     Depreciation      Credits  to       Credits  to        Value  of 
Period 

1 
2 
3 

4  ' 
5 
6 
7 
8 
9 
10 

$1,282.81         $382.81         $900.00 

Problem 

The  Acme  Manufacturing  Company,  believing  that  the  annuity  method  of 
depreciation  is  the  correct  one,  desires  that  you  construct  tables  for  the  following 
machines  (one  table  for  each) : 


Charge 

Interest 

Reserve 

Asset 

$1,000  00 

128.28 

$  60  00 

$  68  28 

931  72 

128  28 

55  90 

72  38 

859  34 

128  28 

51.56 

76.72 

782  62 

128  28 

46.96 

81  32 

701  50 

128  28 

42  08 

86  20 

615  10 

128  28 

36  91 

91  37 

523  73 

128  28 

31.42 

96  86 

426  87 

128.28 

25  61 

102  67 

324  20 

128.28 

19  45 

108  83 

215  37 

128  29 

12.92 

115  37 

100.00 

ASSET  VALUATION  ACCOUNTS  413 

Interest 

Assets  Cost       Scrap         Life          Rate 

Lathes  ....................   $5,000    $    500     10  years       5% 

Milling  machines  ...........     4,500       1,000       8  years       5% 

Grinders  ...................      1  ,200          200       5  years       5% 

Motors  ..................     2,000          200       0  years       5% 

Fixed-percentage-of-diminishing-value  method.  By  this 
method  a  uniform  rate  on  diminishing  value  gives  the  amount  to 
be  charged  to  depreciation  each  year.  As  the  book  value  declines 
each  year,  the  percentage  of  book  value  declines  similarly. 

The  difficulty  encountered  in  this  method  is  that  of  finding  the 
rite  per  cent  to  be  used  in  the  calculation  of  the  charge. 

Procedure:  (a)  Divide  the  scrap  value  by  the  cost. 

(6)  Extract  the  root,  the  index  of  which  corresponds  to  the 
number  of  periods  of  depreciation  to  be  taken  on  the  life  of  the 
asset,  of  the  quotient  obtained  in  (a). 

(c)  Deduct  from  1  the  result  obtained  in  (b),  to  find  the  rate 
per  cent  to  be  used. 

(d)  Multiply  the  net  asset  value  or  the  carrying  value  of  the 
asset  at  the  beginning  of  each  period  by  the  rate  found  in  (c),  to 
obtain  the  depreciation  charge  for  each  period. 

Example 

What  will  be  the  depreciation  charges  for  an  asset  valued  at  $1,000,  with  a 
scrap  value  of  $100,  which  is  to  he  written  off  in  10  years  by  the  fixed-percentage- 
of-diminishing-  value  method9 

The  following  are  the  formula  and  solution  for  the  calculation  of  the  rate: 

Formula  Arithmetical  Substitution 


n/Sc7ap  value  t        10/  100         on  ™0/w 

-  V  lvalue  "  '•  '  ~  Vl,0      =  20-S072%' 


10 
,000 

Solution,  Part  1 
100  -T-  1,000  =  .1 

log  .1  =  1.000000 

Changed,  T.OOOOOO  =  9.000000  -  10 
9.000000  -  10  -h  10  =  .900000  -  1 

Changed  =  1.900000 
The  antilog  of  T.900000  =  .79432K 

1  -  .794328  =  .205672,  or  20.567% 

Solution,  Part  2 

$1,000  X  20.567%  =  $205.67,  first  depreciation  charge 

$1,000  -  $205.67  =  $794.33,  new  asset  value 
$794.33  X  20.567%  =  $163.37,  second  depreciation  charge 

his  process  is  continued  for  each  of  the  10  years. 


414 


ASSET  VALUATION  ACCOUNTS 


TABLE  OF  DEPRECIATION 

(Rate,  20.567%) 


Periodic 

Accumulated 

Depreciation 

Depreciation 

Asset 

Year 

Charge 

Reserve 

Value 

$1,000  00 

1 

$205  67 

$205  67 

794  33 

2 

163.37 

369  04 

630  96 

3 

129.77 

498  81 

501   19 

4 

103  08 

601  89 

398  11 

5 

81    88 

683  77 

316  23 

6 

05  04 

748  81 

251   19 

7 

51   66 

SOO  47 

199  53 

8 

41   04 

841   51 

158  49 

9 

32  60 

874  11 

125  89 

10 

25  89 

900  00 

100  00 

Problems 

1.  Construct  a  comparative  columnar  table  showing  the  periodic  depreciatior 
charges  computed  by  the  straight-line,  sum-of-digits,  sinking-fund,  annuity,  and 
fixed-percentage-of-diminishing-value  methods  for  an  asset  costing  $10,000,  and 
having  a  probable  life  of  10  years  and  a  scrap  value  of  $1,500.     Use  an  interest 
rate  of  4%  per  annum. 

2.  An  asset  costing  $2,000  has  a  life  of  5  years.     It  has  no  scrap  value,  but 
for  the  purposes  of  calculation,  use  $1.     Money  is  worth  5%.     Construct  com 
partitive  columnar  tables  showing  the  carrying  value  and  the  annual  depreciation 
charge  computed  by  the  straight-line,  sum-of-digits,  sinking-fund,  annuity,  and 
fixed-percentage-of-diminishing-value  methods. 

3.  A  businessman,  having  heard  much  about  correct  depreciation  but  under- 
standing little  of  the  methods  of  calculation  used,  calls  on  you  to  explain  to  him 
by  means  of  comparison  the  five  most  important  methods.     As  an  illustration, 
use  an  asset  costing  $4,000,  with  a  scrap  value  of  $500  and  a  life  of  5  years,  and 
an  interest  rate  of  6%. 

Composite  life.  Often  the  depreciable  assets  of  a  business  have 
wearing  values  (cost  less  scrap  value)  and  teiins  of  effective  life 
that  vary  widely,  yet  it  is  desirable  to  ascertain  the  life  of  the  plant 
as  a  whole,  as  when  bonds  secured  by  a  mortgage  on  buildings  and 
equipment  are  issued.  The  bonds  should  not  be  issued  for  a  term 
of  years  exceeding  the  composite  life  of  the  plant;  and,  for  a  margin 
of  safety,  the  term  of  the  bonds  should  be  considerably  shorter  than 
the  composite  life  of  the  plant. 

If  interest  is  not  a  factor,  the  composite  life  is  found  by  dividing 
the  total  wearing  value  by  the  total  depreciation. 

Wearing  Annual 

Cost        8 crap       Value  Charge 

$45,000     $5,000     $40,000  $    800 

12,000       2,000       10,000  400 

30,000       5,000       25,000  1,000 

12,000       2,000       10,000  500 


Unit  Life 

Bldg.  (Brick) 50 

Bldg.  (Frame) 25 

Heavy  Machinery 25 

Boiler 20 


85,000  -^  2,700 


$85,000 
31.5,  approximate  years 


$2,700 


ASSET  VALUATION  ACCOUNTS  415 

If  interest  is  a  factor,  as  when  depreciation  is  computed  on  the 
sinking  fund  basis,  the  annual  charge  for  depreciation  is  the  annual 
rent,  the  accumulation  of  which  should  equal  the  wearing  value. 
Making  use  of  the  data  from  above  and  using  5%  as  the  interest 
rate,  we  have: 

Unit                     Life      Wearing  Value  Annual  Charge 

Bldg.  (Brick) 50             $40,000  $    191 .07 

Bldg.  (Frame) 25               10,000  209 . 53 

Heavy  Machinery 25                25,000  523  81 

Boiler 20              J0,000  J*02 .43 

$85,000  $17226784 
1,226.84  -T-  85,000  =  1.4433% 

The  per  cent  is  low  because  the  major  asset  lias  a  50-year  life, 
and  because  over  long  periods  the  interest  is  also  a  large  factor. 

To  find  the  composite  life,  let  r  denote  the  rate  of  depreciation 
and  i  the  interest  rate.  Then, 


loK(l+i) 

loi 
For  the  foregoing  problem 


log  (  1  +  i) 


log  1.05 

__  log  4.464  _  .649724 

loig'LOS  ~~  .02U89 

.649724  4-  .021189  =  30.66  years. 

Problems 

1.  A  company's  plant  consists  of:  (a)  Buildings:  cost,  $100,000;  life,  40  yean»» 
scrap  value,   $10,000.     (6)   Engine:  cost,  $40,000;  life,  25  years;  scrap  valu*»- 
$5,000.      (c)  Boiler:  cost,  $12,000;  life,  20  years;  scrap  value,  $2,000.      (d)  Elec- 
trical equipment:  cost,  $7,500;  life,  15  years;  scrap  value,  $1,500.     Compute  the 
charge  for  depreciation  by  the  sinking  fund  method,  on  a  4%  basis,  and  find  the 
composite  life  of  the  plant. 

2.  Find  the  composite  life  of  a  plant  consisting  of  the  following: 

Item 
A 
B 
C 
D     

Interest  at  5%. 

Depreciation  Problems  from  C.  P.  A.  Examinations 

1.*  A  manufacturing  company  has  a  factory  building  which  cost,  with  its 
equipment,  $100,000.     The  company  has  set  up  a  depreciation  reserve  of  $30,000. 

*  C.  P.  A.,  Michigan. 


Cost       Scrap  Value        Life 

$  6,000 

$    200 

10  years 

3,500 

500 

15  years 

12,000 

1,000 

20  years 

15,000 

2,500 

12  years 

416  ASSET  VALUATION  ACCOUNTS 

An  appraisal  made  shows  the  replacement  cost  to  be  $160,000,  and  the  depreci- 
ated sound  value  to  be  $130,000. 

(a)  Prepare  the  necessary  entries  to  give  effect  to  the  appraisal  figures. 

(6)  How  would  you  treat  the  item  of  depreciation  on  the  increased  values, 
for  the  purpose  of  determining  costs? 

2.*  A  machine  which  cost  $1,200  has  been  used  for  5  years,  and  has  depreci- 
ated annually  10%.  The  latter  amount  has  been  credited  to  the  Reserve 
account. 

(a)  At  the  end  of  the  first  5  years,  the  machine  is  traded  for  a  new  one  which 
costs  $1,500;  an  allowance  of  $300  is  made  on  the  old  machine,  the  balance  being 
paid  in  cash.  Prepare  the  necessary  entries  to  take  care  of  this  transaction. 

(6)  Assume  that  the  machine  is  traded  for  one  costing  $1, 700,  and  that  an 
allowance  of  $700  is  made,  the  balance  being  paid  in  cash.  Prepare  the  necessary 
entries. 

3.f  A  manufacturing  plant,  operating  to  the  date  of  negotiations  relative 
to  its  disposition,  was  acquired  by  a  newly  formed  corporation,  the  price  being 
based  on  the  present  sound  values,  which  were  stated  as  follows: 

Present 

tiound  Value  Age 

Machinery     $116,500  4^- years 

26,300  4  years 

217,300  2\  years 

16,750  2  years 

57,550  I  year 

Equipment $  13,300  6  years 

1 1 ,650  2  years 

27,660  1  year 

Buildings:  A                     .                                   $285,700  12  years 

A         .                                                   15,000  5i  years 

A   .                                                          16,600  1  year 

B   .                                                       525,000  5  years 

The  estimated  life  of  the  machinery  is  10  years  from  the  date  of  original 
purchase;  of  the  equipment,  15  years  from  the  (late  of  purchase;  of  buildings  A, 
30  years;  and  of  building  #,  45  years. 

It  is  desired  to  set  up  the  assets  on  the  books  at  present  reproductive  values, 
with  a  corresponding  depreciation  reserve  to  bring  the  net  book  value  to  the 
amount  of  the  "sound  values "  given  above.  Compute  the  " reproductive  value  " 
and  the  depreciation  reserve,  and  give  the  future  annual  depreciation  provision, 
all  on  the  basis  of  a  uniform  rate  each  year  until  the  book  value  is  extinguished. 

It  may  be  assumed  for  the  purposes  of  your  answer  that  the  assets  will  have 
no  salvage  value. 

4.f  The  City  Dairy  Company  bottles  and  distributes  milk.  Its  sales  average 
40,000  Ibs.  per  day.  It  operates  three  pasteurizers,  each  of  which  has  a  capacity 
of  2,500  Ibs.  per  hour. 

These  machines  cost  $1,200  apiece,  installed,  and  they  have  been  in  use  for 
3  years.  At  the  time  that  they  were  installed,  their  life  was  estimated  at  15 


*C.  P.  A.,  Michigan. 

|  American  Institute  Examination. 

J  C.  P.  A.,  Wisconsin. 


ASSET  VALUATION  ACCOUNTS  417 

years,  and  their  salvage  value  at  the  end  of  that  period  at  $50  each.  Experience 
has  shown  that  repair  and  maintenance  charges  on  these  machines  will  average 
3  %  of  their  cost  per  year. 

The  Smith  Dairy  Machinery  Company  manufactures  a  new  type  of  machine 
which  is  guaranteed  to  have  a  productive  capacity  of  12,000  Ibs.  per  hour,  and 
to  save,  in  comparison  with  the  old  type,  80%  of  the  cost  of  live  steam  and 
refrigeration  used  in  pasteurization.  The  life  of  these  machines  is  estimated  at 
10  years,  and  their  salvage  value  at  the  end  of  that  period  at  $100  each.  The 
manufacturers  offer  to  install  them  ready  to  operate  at  $8,500  each,  and  to 
remove  the  old  machines  and  make  an  allowance  for  their  estimated  scrap  value. 
Their  guarantee  also  provides  for  replacement  of  broken  or  defective  parts,  and 
for  complete  maintenance  to  keep  the  machines  in  good  working  order  for  1  year. 
Subsequent  repair  and  maintenance  charges  may  be  assumed  to  average  5% 
annually  of  the  machines'  original  cost. 

You  are  called  upon  to  make  a  special  examination  of  the  accounts,  with  a 
view  to  determining  whether  it  would  be  advantageous  from  a  profit  and  loss 
standpoint  to  install  the  new  type  of  machine.  Your  examination  discloses  that 
the  present  cost  of  live  steam  and  refrigeration  used  in  pasteurization  averages 
44^  per  1,000  Ibs.  of  milk. 

Required:  (a)  Assuming  future  production  to  average  5%  increase  over  the 
sales  given  above,  state  the  conclusions  that  you  would  report  to  your  client, 
and  show  your  method  of  arriving  at  them. 

(6)  Assuming  that  the  manufacturer's  proposition  has  been  accepted,  draft 
the  entries  which  you  consider  should  be  made  to  record  the  changes. 

5.*  The  Plant  and  Equipment  and  Reserve  for  Depreciation  accounts,  pre- 
sented below,  represent  the  transactions  of  the  A.  Company  for  the  year  1944, 
as  recorded  by  the  bookkeeper,  from  January  1  of  that  year. 

PLANT  AND  EQUIPMENT 

1944  1944 

Jan.     1  Balance $500,000     Jan.   31  Screw-cutting  lathe.   $        150 

17  Planer     2,000     Apr.   17  Steam  engine 300 

Mar.  21  Bolt  machine             .  1,250     Sept.  30  Steel  and  lumber.  .  .             400 

Apr.  16  Crane 3,000     Dec.  31  Balance 524,650 

May    3  Electrical  equipment 

for  crane  . .        .    .  1,200 
27  Roof  of  machine  shop  3,500 
June    3  Lathe  belting       .  .    .  750 
Aug.  20  Wm.     Smith,     Con- 
tractor          10,000 

Dec.  31  Machine  shop.   .     ..  3,800 

«525,500  $525,500 

RESERVE  FOR  DEPRECIATION— PLANT  AND  EQUIPMENT 
1944  1944 

Dec.  31  Balance $175,000    Jan.     1  Balance $125,000 

Dec.  31  Depreciation  at  10% 

per  annum 50,000 

$175,000  $175,000 


1  Adapted  from  American  Institute  Examination. 


418  ASSET  VALUATION  ACCOUNTS 

The  following  is  a  description  of  the  transactions;  you  are  required  to  make 
any  entries  that  you  deem  necessary  to  correct  the  accounts,  giving  reasons 
therefor,  and  setting  up  corrected  accounts. 

The  balances  at  the  beginning  are  assumed  to  be  correct. 

Planer,  $2,000,  is  a  standard  machine,  purchased  new. 

Bolt  machine  was  made  in  company's  own  shop.  The  SI, 250  represents 
cost  of  castings,  $500,  and  direct  labor,  $750.  The  machine  shop  pay  roll  was 
$20,000  ($15,000  direct,  and  $5,000  indirect)  during  the  year;  castings  and  parts 
purchased  were  $17,000;  general  supplies  were  $4,000;  rent  was  $2,500;  light 
heat,  and  power  were  $3,500. 

Crane  and  equipment,  $4,200,  are  standard  machinery,  purchased  new. 

Roof  of  machine  shop  was  destroyed  by  weight  of  snow  during  the  winter. 

Belting  for  all  equipment,  amounting  to  $25,000,  was  charged  to  plant  and 
equipment  when  the  plant  was  opened,  and  has  not  siiu-e  been  depreciated. 

William  Smith  is  engaged  in  erecting  an  addition  to  the  plant  buildings. 
$10,000  is  the  first  payment  on  the  uncompleted  work. 

Machine  shop,  $3,800,  represents  the  cost  of  making  tools,  setting  machines, 
•ind  installing  new  machinery,  as  follows: 

Tool  making  $1,000 

Setting  machines  for  special  work       .  1,800 

Installing  planer  .        .  .          300 

Installing  bolt  machine*   ...  .  .        .  200 

Installing  crane     .  .         500 

$3,800 

Screw-cutting  lathe— cost,  1937,  $2,000. 

Steam  engine—cost,  1934,  $15,000. 

Steel  and  lumber,  $400,  represents  salvage  from  machine  shop  roof. 

Prior  to  December  31,  1943,  a  separate  account  was  kept  for  land  and 
buildings. 

Ten  per  cent  per  annum  depreciation  on  plant  and  equipment  has  been 
written  oil. 

6.*  A  machine  costing  $81  is  estimated  to  have  a  life  of  4  years  and  a  residual 
value  of  $16.  Prepare  a  statement  showing  the  annual  charge  for  depreciation 
computed  by  each  of  the  following  methods:  (a)  straight-line;  (b)  constant- 
pereentage-of-diminishmg-value;  (r)  annuity.  (For  convenience  in  arithmetical 
calculation,  assume  the  rate  of  interest  to  be  10%.) 

Depletion.  The  provision  for  the  extinction  of  wasting  assets, 
such  as  mines,  timber  lands,  or  gravel  pits,  is  called  a  provision  for 
depletion. 

We  shall  deal  with  two  classes  of  problems;  namely,  the  deter- 
mination of  the  amount  of  depletion  each  year,  and  the  capitaliza- 
tion of  the  wasting  asset. 

Calculation  of  depletion.  The  amount  of  depletion  usually 
stands  in  the  same  proportion  to  the  total  cost  of  the  wasting  asset 
as  the  units  of  product  removed  stand  to  the  total  units  of  product 
that  it  was  estimated  the  asset  would  produce  when  new;  but  if  the 


1  American  Institute  Examination. 


ASSET  VALUATION  ACCOUNTS  419 

property  is  leased  and  it  is  not  possible  to  remove  all  the  product 
before  the  lease  expires,  it  is  plain  that  depletion  should  be  based 
on  the  quantity  to  be  extracted  during  the  period  of  the  lease. 

Problems 

1.  The  estimated  recoverable  tonnage  in  a  coal  mine  was  placed  at  2,214,363 
tons.     The  value  of  "Coal  Lands"  was  $90,443.62.     Compute  the  depletion 
charge  per  ton  of  coal  mined. 

2.  A  tract  of  timber  was  valued  at  $25,965.86,  and  its  footage  was  estimated 
at  17,228,000.     The  following  year  the  timber  cut  was  5,184,336  feet.     Compute 
the  rate  of  depletion  per  thousand  feet,  and  the  depletion  charge  for  the  year. 

3.  A  mining  property  was  valued  at  $50,000,  and  the  estimated  recoverable 
tonnage  placed  at  1,000,\)00  tons. 

During  the  next  7  years,  tonnage  was  removed  as  follows: 

First  year  50,000  tons 

Second  year  .                             00,000  tons 

Third  year  70,000  tons 

Fourth  year  .    .  .                             X0,00l)  tons 

Fifth  year                                         .  80,000  tons 

Sixth  year  80,000  tons 

Seventh  year  .                          80,000  tons 

Prospecting  and  development  toward  the  close  of  the  seventh  year  cost 
$25,000,  and  resulted  in  an  estimated  recoverable  tonnage  of  2,000,000  tons. 
Appreciation  due  to  discovery  was  placed  on  the  books  at  $60,000. 

The  tonnage  removed  during  the  eighth  and  ninth  years  was  100,000  tons 
and  120,000  tons,  respectively. 

Calculate  the  annual  charges  for  depletion. 

[Suggestion:  Set  up  two  accounts,  Mining  Property  (Cost),  and  Mining 
Property  (Discovery  Value),  and  credit  these  accounts  with  the  proper  depletion 
charges  each  year.  This  problem  is  illustrative  of  the  complications  that  arise 
when  there  is  more  than  one  valuation  on  a  particular  property.] 

Capitalized  cost.  Capitalization  is  the  cost  of  an  indefinite 
number  of  renewals  of  anything.  The  value  is  found  as  a  per- 
petuity, page  364.  Capitalized  cost  is  the  cost  of  the  renewals 
plus  the  original  cost.  When  an  endowment  fund  provides  for  the 
periodic  replacement  of  a  useful  memorial,  such  as  a  building,  a 
bridge,  and  so  forth,  the  amount  of  the  fund  is  the  capitalized  cost 
of  the  memorial. 

Procedure:  (a)  Divide  the  cost  of  the  asset  by  the  given  rate 
per  cent  expressed  decimally,  to  find  the  capitalization. 

(6)  Divide  the  capitalization  found  in  (a)  by  the  amount  of  an 
ordinary  annuity  of  1  at  the  given  rate  per  cent  and  for  the  time, 
expressed  in  periods,  which  represents  the  life  of  the  asset.  (This 
will  give  the  amount  of  an  endowment  fund  necessary  to  replace 
the  asset  periodically  at  the  end  of  each  term  of  years.) 


420  ASSET  VALUATION  ACCOUNTS 

(c)  Add  the  cost  of  the  asset  to  the  amount  of  the  endowment 
fund  found  in  (6).  The  sum  of  these  two  items  is  the  amount 
necessary  to  provide  for  the  first  cost  and  for  the  replacement  fund. 

Example 

It  is  desired  to  set  aside  a  fund  which  will  provide  for  the  erection  of  a  memorial 
costing  $2,000,  and  for  the  replacement  of  the  memorial  at  the  end  of  each  5  years. 
Money  is  worth  6%.  Find  the  amount  of  the  fund. 

Formula  Arithmetical  Substitution 

Cost  2,000 

t  0(> 

—     +  Cost  =  Capitalized  cost.  '  +  2,000  -  $7,913.21. 

8    i  (1.0o)°  —   1 

.06 

Solution 

2,000  -f-  .()()  -  33,333.33,  capitalization  of  asset 

(I. (Mi)5  -  1.33S225(),  compound  amount  of  1  at  6% 

for  5  year* 
1.338225(5  -  I  -  .33S225(i,  compound  interest  on  1  at  6% 

for  5  years 
.33H2250  -i-  .0<>  =  5.(>37093,  amount  of  ordinary  annuity  of 

1  at  (>%  for  5  years 
33,333.33  •*•  5.037093  =  5,913.21,  sum  available  for  investment  at 

compound  interest 
$5,913.21  -f  $2,000  =  $7,913.21,  capitalized  cost 

Verification 

$2,000  —  present  cost  of  memorial 
$7,913.21  -  $2,000  -  $5,913.21,  sum  available  for  investment 

at  compound  interest 
$5,913.21  X  1.3382250  =  $7,913.21,  total  fund  5  years  hence 

Problems 

1.  The  cost  of  the  Davis  Memorial  Building  was  $150,000.     It  is  estimated 
that   the   building   will    last   50   years.     Money   is   worth   4%.     Calculate    the 
capitalized  cost  of  the  building. 

2.  A  philanthropist  desires  to  provide  a  fund  for  the  erection  of  a  college 
building  costing  $200,000,  and  for  the  replacement  of  the  building  at  the  end 
of  each  50  years.     Money  is  worth  4%.     Calculate  the  amount  of  the  fund, 
assuming  that  repairs  and  renewals  necessary  during  the  50-year  periods  are 
not  to  be  paid  for  out  of  the  fund. 

Perpetuity  providing  for  ordinary  annual  expenses  and  for 
replacement  of  asset.  Many  endowments  provide  for  ordinary 
annual  expenses  as  well  as  for  the  replacement  of  the  asset. 

Procedure:  (a)  Divide  the  cost  of  the  asset  by  the  given  rate 
per  cent  expressed  decimally,  to  find  the  capitalization. 

(6)  Divide  the  capitalization  of  the  asset  by  the  amount  of  an 
ordinary  annuity  of  1  at  the  given  rate  per  cent  for  a  number  of 


ASSET  VALUATION  ACCOUNTS  421 

periods,  expressed  in  years,  equal  to  the  life  of  the  asset.     This 
will  give  the  replacement  fund. 

(c)  Divide  the  annual  upkeep  by  the  given  rate  per  cent,  to 
find  the  capitalization  of  the  upkeep  fund. 

(d)  Add  the  original  cost  of  the  asset,  the  replacement  fund, 
and  the  upkeep  fund  to  find  the  total  fund  necessary. 

Example 

The  Rutledge  Home  cost  $75,000.  The  annual  expenses  of  running  the 
institution,  including  repairs  and  upkeep,  are  estimated  at  $5,000.  If  the  life 
of  the  building  is  50  years,  and  money  is  worth  4%,  what  is  the  amount  of  the 
fund  necessary  to  pnnide  for  the  perpetuity  of  the  home? 

Formula 
Cost 


,    r^    A    .  o 

—  -     +  Cost  +         .        =  Sum. 
,s  i 

A  rithmcticnl  Substitution 
75,000 


.04 

Solution,  J'art  1 

$75,000  -5-  .04  -  $1,S75,000,  capitalization 
$1,S75,000  ~  $15200708  =  $12,281.03,  building  replacement  fund 

Solution,  I*  art  2 
$75,000  =  cost  of  building 

Solution,  Part  ft 
$5,000  -f-  .04  =  $125,000,  upkeep  fund 

Summary 

Building  fund  $  12,281.03 

Cost  of  building  75,000  00 

Upkeep  fund  125,000  00 

Total  fund  necessary  .        $212,281   63 

Problems 

1.  The  original  cost  of  a  public  library  is  $100,000,  the  estimated  life  of  the 
building  is  50  years,  and  the  annual  cost  of  upkeep  is  $12,000.     Money  is  worth 
4%.     Calculate  the  amount  of  the  fund  necessary  to  provide  for  the  perpetuity 
of  the  library. 

2.  The  cost  of  a  memorial  is  $55,000,  its  life  is  25  years,  and  its  upkeep  is 
$2,400  a  year.     Money  is  worth  4%.     Calculate  the  amount  of  the  fund  neces- 
sary to  provide  for  building  the  memorial,  keeping  it  in  repair,  and  replacing  it 
at  the  end  of  each  25-year  period. 


422  ASSET  VALUATION  ACCOUNTS 

3.  A  charitable  institution  is  to  be  built  and  maintained  by  a  trust  com- 
mittee. It  is  estimated  that  the  building  will  cost  SI 00,000,  arid  that  its  life 
will  be  60  years.  The  estimated  annual  income  and  costs  are  as  follows: 

Income : 

Donations         $4,000 

Expenses  : 

Miscellaneous  service  charges               .            .  .          .  .          1,000 

Heat              ...                  2,000 

Matron  ...  .  1,800 

Help          .                           3,000 

Food  . .                    .                   5,000 

Medical  attention                            .                .    .  .          1,200 

Incidentals  ...                  .  .            .  1,000 

Find  the  value  of  the  endowment  at  5%,  interest  convertible  annually. 

Capitalization  of  a  wasting  asset.  Investment  in  a  wasting 
asset  such  as  a  mine  or  timberlands  should  yield  not  only  interest 
on  the  investment,  but  additional  income  to  provide  funds  for  the 
restoration  of  the  capital  originally  invested.  A  sinking  fund, 
(•ailed  a  redemption  fund,  is  created  for  the  purpose  of  restoring  the 
original  capital.  The  investment  rate  will  usually  be  higher  than 
the  rate  which  can  be  earned  on  the  redemption  fund. 

To  find  the  fair  market  value  at  which  a  wasting  asset  may  be 
capitalized,  divide  the  estimated  annual  average  income  by  the 
sum  of :  (a)  the  rent  of  an  ordinary  annuity  of  1  at  the  redemption 
fund  rate  for  the  length  of  time  estimated  to  deplete  the  asset 
(this  to  provide  the  annual  sinking  fund  payment  to  restore  the 
investment),  and  (6)  a  fair  annual  rate  of  income  (this  to  provide  a 
return  to  the  investor). 

Example 

A  mine  produces  an  average  annual  operating  income  of  $10,000.  At  tli<* 
present  rate  of  depletion,  it  is  estimated  that  the  mine  will  last  8  years.  If 
the  sinking  fund  will  earn  4%,  and  the  owners  are  to  receive  a  dividend  of  6%. 
what  should  be  the  capitalized  value? 


Formula 
Operating  profit 


h  Dividend  rate 


Capitalized  value 


Arithmetical  Substitution 


.04 


ASSET  VALUATION  ACCOUNTS 

Solution 

(1.04)8  =  1.3685691,   compound  amount  of  1  for  8 

periods  at  4% 
1.3685691  -  1  «  .3685691,   compound  interest  on   1   for    8 

periods  at  4% 
.3685691  ^  .04  =  9.214226,   amount  of  annuity  of  1   for  8 

periods  at  4% 
1  •*-  9.214226  -  .1085278,  the  rent  of  an  annuity  that  will 

amount  to  I 
.1085278  +  .06  =  .1685278,  rent  of  annuity  plus  the  dividend 

rate 
$10,000  -v-  .1685278  =  $59,337.39,  capitalized  value 


423 


TABLE  OF 

Annual 
Years      Income 
\          $10,000 
2            10,000 
3            10,000 
4           10,000 
5           10,000 
6           10,000 
7           10,000 
8           10,000 

CAPITALIZATION  OF 

6%  on           Sinking 
Capitalized        Fund 
Value         Payments 
$3,560  24      $6,439.76 
3,560  24        6,439  76 
3,560  24        6,439  76 
3,560  24        0,439  76 
3,560  24        6,439  76 
3,560  24       6,439  76 
3,560.24       6,439  76 
3,560.24       6,439  76 

WASTING 

Interest 
on  Sinking 
Fund 

$    . 

ASSET 

Sinking  Fund 
Accumulations 
$  6,439  76 
13,137  11 
20,102  35 
27,346  20 
34,879  81 
42,714  76 
50,863  11 
59,337.39 

257  59 
525  48 
804  09 
1,093  85 
1,395  19 
1,708  59 
2,034  52 

Problems 

1.  Find  the  capitalized  value  of  a  coal  mine  which  will  produce  a  net  income 
of  $20,000  a  year  for  30  years;  the  annual  income  rate  is  6%,  and  a  sinking  fund 
is  to  be  accumulated  at  4%. 

2.  A  tract  of  timber  will  yield  an  annual  revenue  of  $20,000  for  20  years. 
If  annual  dividends  are  declared  at  5%,  and  payments  are  made  annually  into 
a  sinking  fund  which  bears  4%  interest,  what  is  the  value  of  the  timber  rights? 

3.  A  gravel  pit  is  estimated  to  contain  3,500,000  cubic  yards  of  gravel.     This 
pit  is  leased  at  a  royalty  of  10^  per  cubic  yard  of  gravel  extracted,  and  the  average 
annual  output  is  150,000  cubic  yards.     If  a  6%  dividend  is  paid  on  the  stock, 
and  a  fund  equal  to  the  capital  stock  is  accumulated  at  4%,  what  should  be  the 
capitalized  value  of  the  property? 


Review  Problems 

1.  A  machine  that  cost  $1,000  is  estimated  to  have  a  life  of  10  years  and  a 
scrap  value  of  $200.     Compute  the  annual  depreciation  charge  by: 

(a)  The  straight  line  method. 

(6)  The  fixed  percentage  of  diminishing  value  method. 

(c)  The  sinking  fund  method,  using  6%  effective  interest 

(d)  The  annuity  method,  using  6%  effective  interest. 

2.  A  mine  with  a  net  annual  yield  of  $75,000  will  be  exhausted  in  15  years 
at  the  present  rate  of  output.    What  is  the  mine  worth,  on  a  5%  basis? 


424  ASSET  VALUATION  ACCOUNTS 

3.  A  certain  make  of  bench  drill  costs  $17.50  and  lasts  3  years.     How  much 
can  be  paid  for  a  better  grade  of  drill  that  will  last  6  years,  money  being  worth  4 %? 
(HINT:  Capitalized  costs  must  be  equal.     Solve  for  x.) 

4.  A  roof  made  of  one  material  will  cost  $300  and  last  for  20  years.     If  made 
of  another  type  of  material,  it  will  last  for  the  life  of  the  building,  which  is  esti- 
mated to  be  75  years.    How  much  can  one  afford  to  pay  for  the  permanent  type 
of  roof  if  money  is  worth  5%? 

6.  Calculate  the  fixed  percentage  to  be  written  off  each  year  for  an  asset 
costing  $2,400,  estimated  life  6  years  and  scrap  value  $400. 


CHAPTER  35 
Building  and  Loan  Associations 

Control.  Building  and  loan  associations  are  organized  under 
state  laws,  and  in  most  cases  are  under  the  direct  supervision  of 
the  state  banking  department.  The  banking  department  requires 
semiannual  or  annual  reports,  and  the  associations  are  subject  to 
special  examinations  from  time  to  time  by  the  state  bank  exam- 
iners. In  addition  to  this,  annual  audits  are  usually  made  by 
committees  of  stockholders,  and  in  a  great  many  cases  independent 
audits  are  made  by  certified  public  accountants. 

Classes  of  stock.  The  amount  of  capital  stock  of  a  building 
and  loan  association  is  fixed  by  the  charter,  and  the  minimum 
capital  and  the  par  value  per  share  are  stated  at  the  beginning  of 
the  charter.  The  classes  of  stock  issued  are  installment  stock  and 
fully-paid  stock. 

Installment  stock.  Ownership  of  this  class  of  stock  is  generally 
evidenced  by  a  pass  book,  in  which  the  weekly  or  monthly  pay- 
ments are  recorded.  The  monthly  payments  are  usually  $1  for 
each  share  with  a  maturity  value  of  $200,  or  50ji  for  each  share  with 
a  maturity  value  of  $100.  In  some  organizations,  if  it  is  desired 
to  mature  the  shares  in  a  shorter  time,  double  payments  may  be 
made.  Shares  of  this  class  participate  in  all  the  earnings  of  the 
association. 

When  the  monthly  installments,  called  dues,  and  the  profits  or 
dividends  credited  to  the  stockholder  equal  the  face  value  of  the 
shares,  the  shares  are  matured  or  paid-up.  The  amount  may  then 
be  withdrawn,  or  fully-paid  shares  may  be  issued. 

If  the  stockholder,  also  called  a  member,  has  borrowed  from  the 
association,  the  maturing  of  his  stock  effects  a  reduction  in  his 
indebtedness. 

Fully-paid  stock.  Ownership  of  this  class  of  stock  is  evidenced 
by  a  stock  certificate,  and  the  stockholder  usually  receives  a  given 
rate  of  interest,  although  in  some  cases  stockholders  participate  in 
earnings  in  the  same  manner  as  holders  of  installment  stock. 

Withdrawal  of  funds.  In  most  cases,  funds  deposited  in  an 
association  must  be  left  for  at  least  six  months.  After  that  time 
any  member  may  withdraw  all  or  a  part  of  his  funds,  together  with 
the  dividends  credited  and  not  already  paid.  In  the  event  of  with- 
drawal before  the  maturity  of  the  shares,  some  associations  retain 

425 


426  BUILDING  AND  LOAN  ASSOCIATIONS 

a  membership  or  withdrawal  fee  of  2%  of  the  par  value  of  each 
share,  and  issue  membership  certificates  which  are  transferable 
upon  the  books  of  the  association.  The  member  may  retain  this 
certificate,  or  may  assign  it  to  someone  else,  as  he  sees  fit.  When 
another  account  is  opened  in  the  association,  either  by  the  member 
or  by  anyone  else  holding  his  certificate  of  membership,  credit  is 
given  for  the  amount  which  the  certificate  represents. 

Other  associations  permit  the  withdrawal  of  the  face  amount 
of  the  deposits,  and  allow  the  holder  interest  for  the  equated  time. 
The  rate  of  interest  is  fixed  by  the  association,  arid  is  usually  lower 
than  the  per  cent  earned  by  the  shares.  This  difference  in  rates 
results  in  a  profit  to  the  association.  Such  profit  on  withdrawals 
is  added  to  the  other  profits,  and  distributed  to  the  shares  remain- 
ing in  the  association. 

Plans  of  organization.  There  are  three  principal  plans  upon 
which  building  and  loan  associations  are  organized :  the  terminating 
plan;  the  serial  plan;  and  the  permanent  or  perpetual  plan,  also 
called  the  Dayton  or  Ohio  plan. 

Terminating  plan.  The  life  of  the  association  is  limited  under 
the  terminating  plan  to  any  number  of  years  that  may  be  agreed 
upon.  When  the  specified  number  of  years  elapses,  the  association 
goes  out  of  existence,  or  a  new  one  may  be  formed  for  another 
period  of  time.  On  becoming  a  member  subsequent  to  the  date 
of  issue  of  the  stock,  the  purchaser  pays  the  book  value  of  the  share 
or  shares  purchased.  The  book  value  consists  of  back  dues,  plus 
dividends  that  have  accumulated  to  the  credit  of  the  stock  since 
the  date  of  original  issue. 

The  amount  of  the  monthly  payments  necessary  to  retire  the 
stock  is  determined  in  the  same  manner  as  is  the  rent  of  the  present 
worth  of  an  annuity. 

Determination  of  the  amount  to  be  paid  monthly  under  the  termi- 
nating plan. 

Procedure :  (a)  Compute  the  interest  rate  per  period  by  dividing 
the  annual  rate  by  the  number  of  times  that  conversion  takes  place. 

(6)  Compute  the  number  of  periods  by  multiplying  the  time 
in  years  by  the  number  of  times  that  the  interest  is  converted 
annually. 

(c)  Compute  the  present  value  of  an  annuity  of  1,  using  the 
periodic  rate  found  in  (a)  and  the  number  of  periods  found  in  (6). 

(d)  Divide  the  par  value  of  one  share  by  the  present  value  of 
an  annuity  of  1  found  in  (c). 

Example 
What  should  be  the  monthly  payment  per  share,  each  share  having  a  par 


BUILDING  AND  LOAN  ASSOCIATIONS  427 

value  of  $100,  if  it  is  desired  to  mature  the  shares  in  5  years,  money  being  worth 
6%? 

Solution 

p 
—  =  Monthly  payment 

«„;»"- 51 .7256 

100  -5-  51.7256  =  1.933 

.*.  $1.93  =  Monthly  payment. 

By  tliis  plan,  the  association  would  be  dissolved  at  the  end  of  the  5-year 
period. 

Serial  plan.  Under  this  plan,  stock  is  issued  at  specified  dates. 
Each  issue  constitutes  a  new  series,  and  shares  in  the  profits  in 
proportion  to  the  length  of  time  that  the  series  is  outstanding. 
The  distribution  of  profits  under  this  plan  is  similar  to  the  distri- 
bution in  a  partnership  using  the  average  investment  method. 
New  members  may  pay  all  back  payments  to  the  beginning  of  the 
current  series  plus  a  charge  for  interest  on  these  payments,  or  they 
may  wait  for  a  new  series  to  begin;  a  new  series  may  begin  annually, 
semiannually,  quarterly,  or  monthly,  depending  on  the  demand  for 
shares.  As  soon  as  a  new  series  is  opened,  issuance  of  shares  in  the 
previous  series  is  stopped. 

Distribution  of  profits.  When  several  series  of  stock,  maturing 
at  as  many  different  dates,  have  been  issued,  the  distribution  of 
profits  is  a  complicated  matter.  Of  the  many  methods  of  deter- 
mining the  proper  profit  distribution,  the  most  familiar  are  the 
partnership  method  and  Dexter's  Method,  commonly  called 
"Dexter's  Rule."  The  purpose  of  these  methods  is  to  secure  an 
equitable  distribution  of  profits  among  the  members  of  the  associa- 
tion, upon  the  basis  of  the  amounts  that  they  have  contributed 
against  the  face  value  of  the  shares  registered  in  their  names,  and 
upon  the  basis  of  the  time  that  each  dollar  paid  by  the  association 
members  has  been  in  the  possession  of  the  association. 

Partnership  method.  The  name  indicates  the  substance  of 
this  method,  and  an  example  and  solution  are  given  to  show  its 
application. 

Example 

The  Alpha  Building  and  Loan  Association  issued  six  series  of  shares,  as 
follows: 


Series 
1 

Date            Number  of  Shares 
Jan.  1    1942                  500 

2 
3 

July  1 
Jan.  1 

1942 
1943 

500 
400 

4 
5 

July  1 
Jan.  1 

1943 
1944 

300 
400 

6 

July  1 

1944 

400 

428 


BUILDING  AND  LOAN  ASSOCIATIONS 


The  dues  in  each  series  were  $1  a  share,  payable  monthly.  The  net  profits 
from  interest,  fines,  and  so  forth,  less  the  operating  expenses  for  the  half-year 
ended  Dec.  31,  1944,  were  $965.22,  and  the  undivided  profits  on  July  1,  1944, 
were  $4,272.78. 

The  status  of  the  shares  on  July  1,  1944,  was  as  follows: 


Series 
1 
2 
3 
4 
5 


Date  of  Issue  Shares 

Jan.  1,  1942  500 

July    ,  1942  500 

1943  400 

1943  300 

1944  400 


Number  of   Paid  per    Profit  per    Value  per 


Jan. 
July 
Jan. 


Share 

Share 

Share 

$30 

$4  12 

$34  12 

24 

2.66 

26  66 

18 

1.51 

19  51 

12 

.69 

12  69 

6 

.19 

6  19 

Distribute  the  profits  fur  the  half-year  ending  December  31,  1944,  by  the 
partnership  method. 

Solution 

On  December  31,  1944,  the  amount  paid  on  each  share  of  each  series  was 
as  follows: 

Paid  per  Paid  per 

Series                                Share  Series  Share 

\ $36  4  ....           $18 

2  30  5  .12 

3  24  6  6 

Dues  of  $1  have  been  paid  at  the  beginning  of  the  month  on  each  of  the 
500  shares  in  the  first  series  for  36  months.  The  average  time  is  found  to  be 
18^  months.  That  is,  $1  was  invested  for  36  months;  $1  for  35  months;  $1  for 
34  months;  and  so  forth.  The  average,  18^  months,  is  the  sum  of  the  first  term 

and  the  last  term,  divided  by  2;  thus,       -  -  —  =  18^.     Solving  for  series  2,  3,  4,  5, 

2i 

and  6,  we  have  15^-,  12^-,  9^,  63,  and  3^,  respectively,  as  the  average  time  for 
each  series. 

As  there  are  500  shares  in  the  first  series,  and  each  share  has  paid  $36,  the 
capital  of  the  first  series  is  $18,000.  $18,000  for  18^  months  equals  $333,000  for 
1  month.  Solving  for  each  series,  we  have  the  following: 

Series  Dollars  for  1  Month 


2       30  X  15*        "        X  500 

232  500 

3       24  X  12^        "         X  400 

120,000 

4       18X9^       "       X  300 

51,300 

5       12  X    6^       "        X  400 

31  200 

6        6  X    3^       "        X  400 

8  400 

Undivided  profits,  July  1,  1944  

$776,400 
$     4,272  78 

Earnings,  July  1,  1944,  to  Dec.  31, 

1944  965  22 

Profits  available  for  distribution 

$     5,238.00 

The  profits  available  for  distribution,  $5,238,  are  prorated  among  the  series 
in  the  proportion  that  the  equated  capital  of  each  series  bears  to  the  total  equated 
capital,  as  follows: 


BUILDING  AND  LOAN  ASSOCIATIONS 


429 


Series 


o  QQQ 

1       '    >4  of  $5,238  =  $2,246  59  for  500  shares,  or  $4.49  per  share 

"       =  1,568  57  "  500  "  "  3.14  " 

"       =  809  58  "  400  "  "  2.02  " 

"       =  346.10  "  300  "  "  1.15  " 

"       =  210  49  "  400  "  "  .53  "       " 

"       =  56_67  "  400  "  "  .14  "       " 
$5,238~00 


2  ^^ 

3  lj.200  lt 
7,764 

4  513  f< 
7,764 

5  312  •< 
5  7,764 


A 

6 


H4 

7,764 


Dexter' s  rule  for  distribution  of  profits.  This  method  is  a  modi- 
fication of  the  partnership  method.  Its  principles  are  illustrated 
in  the  following  solution,  which  is  based  on  the  example  given 
under  the  partnership  method. 

Solution 

To  the  capital  of  each  series  on  July  1,  1944,  as  shown  by  the  column  "Value 
per  Share"  in  the  previous  tabulation,  should  be  added  the  contribution  of  $1  per 
share  for  each  of  the  six  months  of  the  current  half-year,  computed  by  the 
average  method. 


1  paid 

July 

1 

= 

$1 

for  6 

months,  or  $  6 

for 

moil 

1     " 

Aug. 

1 

= 

1 

1 

5 

a 

<       5 

n 

i 

1     " 

Sept. 

1 

= 

1 

1 

4 

kl 

'      4 

(i 

i 

1     " 

Oct. 

1 

= 

1 

1 

3 

n 

'       3 

it 

1 

1     " 

Nov. 

1 

= 

1 

1 

2 

u 

<       2 

tl 

< 

1     " 

Dec. 

1 

= 

i 

1     1 

month,      '       1 

"  i 

$21    "    I        " 
$21  -T-  6  =  $3.50,  average  for  0  months 

Add  $3.50  to  the  value  of  each  share,  and  multiply  by  the  number  of  shares, 
to  find  the  capital  value  of  the  series. 


Series       Shares 

Dollars  per  Share 

Capital  per 

Scries 

1 

500 

$37 

62 

$18,810 

00 

2 

500 

30 

16 

15,080 

00 

3 

400 

23 

01 

9,204 

00 

4 

300 

16 

19 

4,857 

00 

5 

400 

9 

69 

3,876 

00 

6 

400 

3 

50 

1,400 

00 

Total  capital 

$53,227 

66 

The  previous  distributions  of  profits  are  unchanged.  To  these  are  added 
the  profits  of  the  current  period,  computed  on  the  basis  of  the  present  earning 
capitals ;  in  other  words,  the  capital  $53,227  earned  a  profit  of  $965.22,  or  1 .81 34  %. 


430  BUILDING  AND  LOAN  ASSOCIATIONS 

Hence,  the  profit  for  each  series  and  for  each  share  in  a  series  is  calculated  as 
follows : 


Profit 

Profit  per 

per 

Share  for 

Series 

Capital 

Rate 

Series 

Shares 

Last  Period 

1 

$18,810 

.018134 

$341.10 

500 

$0.68 

2 

15,080 

.018134 

273.46 

500 

.55 

3 

9,204 

.018134 

166  91 

400 

.42 

4 

4,857 

.018134 

88  08 

300 

.29 

5 

3,876 

.018134 

70  29 

400 

.18 

6 

1,400 

.018134 

25  38 

400 

.06 

$965~22 

Adding  the  profit  per  share  for  the  last  period  to  the  value  of  the  share  as 
of  July  1,  1944,  and  including  the  monthly  payments  for  the  period,  gives  the 
value  per  share  as  of  December  31,  1944. 

COMPARISON  OF  PARTNERSHIP  PLAN  AND  DEXTER'S  RULE 

PARTNERSHIP  PLAN  DEXTER'S  RULE 


Profit  per 

Profit  per 

Profit  per 

Profit  per 

Share  to 

Share  to 

Share  for 

Share  for 

Difference 

Series 

July  1,1944 

Dec.  SI,  1944 

Last  6  Mos. 

Last  6  Mos. 

Inc. 

Dec. 

1 

$4   12 

$4  49 

$0  37 

$0.68 

$0.31 

2 

2  66 

3.14 

.48 

.55 

.07 

3 

1   51 

2.02 

.51 

.42 

$0.11 

4 

.69 

1.15 

.46 

.29 

.17 

5 

.19 

.53 

.34 

.18 

.16 

6 

.00 

.14 

.14 

.06 

.08 

The  above  comparison  shows  that  the  partnership  plan  favors 
the  newer  series  at  the  expense  of  the  old.  This  is  unjust,  because 
it  was  really  the  old  series  that  produced  the  profits.  Dexter's 
Rule  is  thus  a  more  equitable  method  of  calculating  the  distribu- 
tion of  profits  in  a  building  and  loan  association  using  the  serial 
plan. 

Problems 

1.  A  building  and  loan  association  issued  a  new  series  each  year,  as  follows: 


Number  of 

Series 

Date  of  Issue 

Shares 

1 

Jan.  1,  1941 

300 

2 

Jan.  1,  1942 

400 

3 

Jan.  1,  1943 

500 

4 

Jan.  1,  1944 

300 

The  dues  were  $1  per  month  per  share.  The  profits  to  the  end  of  the  fourth 
year  were  $4,800.  Find  the  value  of  one  share  in  each  series  at  the  end  of  the 
fourth  year,  using  the  partnership  method. 

2.  The  first  series  of  a  certain  building  and  loan  association  is  3  years  old  and 
has  1,000  shares;  the  second  series  is  2  years  old  and  has  500  sharas;  and  the 
third  series  is  1  year  old  and  has  400  shares.  The  net  assets  are  $60,650.00 
Payments  were  $1  per  month  per  share. 


BUILDING  AND  LOAN  ASSOCIATIONS  431 

By  the  partnership  method,  compute:  (a)  net  profits;  (b)  profit  for  1  share 
in  each  series;  (c)  value  of  1  share  in  each  series. 

3.  By  Dexter *s  Rule,  find  the  value  as  of  June  30,  1943,  of  1  share  in  each 
series : 

STATEMENT,  DECEMBER  31,  1942 

Number  of  Paid  per  Profit  per  Value  per 

Series        Date  Issued        Shares  Share  Share  Share 

1  Jan.  1,  1941          500  $24  $3  32  $27  32 

2  July  I,  1941          400  18  I  89  19  89 

3  Jan.  1,  1942          300  12  .86  12  86 

4  July  1,  1942         400  6  23  6  23 

The  fifth  scries  was  issued  January  1,  1943,  and  comprised  300  shares.  The 
dues  in  each  series  were  $1  per  month  per  share.  The  profits  for  the  six  months 
ended  June  30,  1943,  after  all  expenses  had  been  deducted,  amounted  to  $648.75. 

4.  The  sixth  of  the  above  series  was  issued  July  1,   1943,  and  comprised 
400  shares.     The  net  profits  for  the  six  months  ended  December  31,  1943,  were 
$698.75.     Find  the  value  of  1  share  in  each  series  as  of  December  31,  1943. 

Withdrawal  value.  If  20  shares  of  the  third  series  in  the 
example  on  page  427  are  withdrawn  on  October  1,  1944,  what  is 
their  withdrawal  value,  assuming  that  the  association  allows  5% 
interest  on  shares  withdrawn  before  maturity? 

Each  share  of  the  third  series  lias  a  paid  up  value  on  October 
1,  1944,  of  $21;  hence  the  20  shares  are  worth  $420.  To  this 
amount  add  5%  interest  for  the  equated  time,  11  months, 

(       - — ]>   or  $19.25;  this  gives  a  withdrawal  value  of  $439.25. 

The  book  value  of  each  share  of  this  series  is  $22.51  ($21  in  each 
payment,  plus  $1.51  profits  as  of  July  1,  1944).  Twenty  shares  at 
$22.51  gives  a  book  value  of  $450.20.  The  difference,  $10.95,  is 
the  profit  on  the  transaction,  and  makes  the  divisible  profits  for 
the  half-year  $976.17  ($965.22  +  $10.95). 

Problems 

(Based  on  example,  page  427.) 

1.  Find  the  withdrawal  value  of  10  shares  of  the  second  series,  withdrawn 
December  31,  1944,  interest  allowed  at  4%. 

2.  Find  the  withdrawal  value  of  20  shares  of  the  fourth  series,  withdrawn 
November  1,  1944,  interest  allowed  at  5%. 

3.  Find  the  withdrawal  value  of  10  shares  of  the  third  series,  withdrawn 
July  1,  1944,  assuming  that  the  association  permits  withdrawals  at  book  value, 
but  retains  a  membership  fee  of  2%  of  the  par  value  of  each  share. 

Dayton  or  Ohio  plan.  The  plan  most  commonly  used  by 
building  and  loan  associations  is  the  Dayton  or  Ohio  plan.  Use  of 
this  plan  eliminates  the  uncertainty  as  to  the  time  of  the  maturity 
of  a  loan,  thus  giving  the  borrower  a  definite  contract.  Under 


432  BUILDING  AND  LOAN  ASSOCIATIONS 

other  plans  a  successful  association  would  soon  mature  its  stock, 
while  an  unsuccessful  one  would  greatly  prolong  the  time  during 
which  the  borrower  would  have  to  continue  his  interest  payments. 
Furthermore,  under  the  Dayton  plan  the  borrower  does  not  have- 
to  own  stock.  Each  payment  that  the  borrower  makes  does  two 
things:  first,  it  pays  the  interest;  second,  it  reduces  the  principal. 
The  same  idea  is  applied  in  the  Federal  Farm  Loan  Act,  and  is  an 
application  of  the  subject  " Payment  of  debt  by  installments," 
discussed  on  page  339.  The  monthly  payment  is  found  by  the 
formula  used  under  the  terminating  plan. 

Problems 

1.  Find  the  monthly  payment  which  will  cover  both  principal  and  interest 
of  a  $4,000  loan  at  5%  for  10  years. 

2.  Construct  a  schedule  for  the  first  2  years. 

3.  Construct  a  schedule  for  the  last  2  years,  showing  the  final  amortization 
of  the  debt. 

To  find  the  time  required  for  stock  to  mature  (rate  of  interest 
given).  Such  a  problem  is  simply  that  of  finding  the  time  that  it 
takes  an  annuity  of  annual  rents  payable  in  twelve  monthly  install- 
ments to  accumulate  to  a  certain  amount. 

Procedure:  (a)  Divide  the  maturity  value  by  the  number  of 
dollars  in  the  periodic  payment. 

(6)  Multiply  the  quotient  found  in  (a)  by  the  periodic  rate  pel- 
cent. 

(c)  Determine  the  logarithm  of  1  plus  the  product  found  in  (6). 

(d)  Determine  the  logarithm  of  1  plus  the  periodic  rale  per 
cent. 

(e)  Divide  the  logarithm  found  in  (c)  by  the  logarithm  found 
in  (d). 

Example 

The  Washington  Building  and  Loan  Association  yields  the  investor  a  nominal 
rate  of  7%,  convertible  monthly.  What  is  the  time  required  for  payments 
of  $1  a  month  to  mature  $100? 

For  nnda 


.      I  /  Maturity  value         .\  1 

log     1  +  (  u  -  -  - -  X  i ) 

L          \Perionic  payment        /  J 


^r~~Ti — . — ^ ^  Term. 

log  (1  +  i) 


A  rith  metica I  S  ubstitut  ion 


log  [l  +  (^  X  .0058333)1 


log  1.0058333  -79  periods. 


BUILDING  AND  LOAN  ASSOCIATIONS  433 

Solution 

Dividing:  100  -r  1  =  100  (1) 

Multiplying:  100  X  .0058333  =  .58333  (#) 

Adding  1:  1  +  .58333  =  1.58333  (3) 

log:  1.58333  =  0.199572  (4) 

log:  1.0058333  =  0.002520  (5) 

Dividing:  log  0.199572  -f-  log  0.002526  =  79  (upprox.)  (6) 

Problems 

1.  If  the  nominal  rate  of  interest  in  the  above  example  had  been  5%,  what 
would  have  been  the  time  of  maturity? 

2.  If  the  monthly  payment  is  50^  and  the  nominal  rate  is  5%,  convertible 
monthly,  what  time  is  required  to  mature  a  $100  share? 

3.  If  payments  of  50  £  a  month  on  a  $100  share  earn  for  the  investor  an 
effective  rate  of  6%,  convertible  annually,  in  what  time  will  the  stock  mature? 

To  find  the  effective  rate  of  interest  on  money  invested  in 
installment  shares.  This  is  a  problem  similar  to  that  of  finding 
the  effective  rate  earned  on  an  annuity. 

Example 

On  July  1,  1937,  the  Zenith  Building  and  Loan  Association  issued  $100  par 
value  stock  on  which  monthly  payments  of  $1  per  share  were  to  be  made.  The 
stock  matured  on  January  1,  1944,  by  the  association's  accepting  2^  on  the 
SOth  payment  of  $1.  What  was  the  effective  rate  of  interest  earned? 

To  find  the  effective  rate,  it  is  necessary  to  use  estimated  rates,  as  explained 
under  the  heading  "Computation  of  the  rate  of  an  annuity,"  page  343.  The 
formula  for  the  first  estimated  rate  is  as  follows: 

Formula  Arithmetical  Substitution 

„  Amount. 


A,  +  OJL-  .\ 

\  i  / 


.005660 
Solution 


First  trial  rate,  6.8%: 

.068  ~  12  =  .005f,  monthly  rate 
1.005|  to  79th  power  (by  logs)  =  1.5625808,  compound  amount 

1  .5625808  -  1  =  .5625808,  compound  interest 
.5625808  -j-  .005f  =  $99.28,  amount  of  annuity 

The  first  trial  rate  is  found  to  be  too  small. 
Second  trial  rate,  7.2%: 

.072  -T-  12  =  .006,  monthly  rate 
1.006  to  79th  power  (by  logs)  =  1.6041402,  compound  amount 

1.6041402  -  1  =  .6041402,  compound  interest 
.6041402  -T-  .006  =  $100.69,  amount  of  annuity 

The  second  trial  rate  is  found  to  be  too  large. 


434  BUILDING  AND  LOAN  ASSOCIATIONS 

Interpolation 

Value  at  7.2%  ...  $100  69 

Value  at  6.8%  J)9  2$ 

Difference  of  .4% -S~l.il 

$1.41  -T-  4  =  .35,  the  difference  represented  by  .1  % 

$100  —  .02,  excess  of  last  payment $99  9<S 

Value  of  79  payments  at  unknown  rate                99  98 

Value  of  79  payments  at  6.8%                                 9928 

Excess  over  rate  of  6.8%            .                                           ..  $"~70 

Difference  in  value  represented  by  difference  of  .1  %  .3f- 

-70  -f-  .35  =  2,  or  .2%  to  be  added  to  6.8%,  or  7% 
Problems 

1.  If,  in  the  foregoing  example,  the  stock  had  been  matured  by  the  associ- 
ation's acceptance  of  the  full  amount  of  the  70th  payment,  what  would  have 
been  the  effective  rate  earned? 

2.  Payments  of  50^  a  month  mature  $100  in  10  years,  4  months,  without  its 
being  necessary  for  any  part  of  the  125th  payment  to  be  made.     What  is  the 
effective  rate  of  interest  earned? 

Classified  Problems  on  Building  and  Loan  Associations 
Distribution  of  profits  to  shareholders. 

1.  Five  shares  of  stock  in  a  building  and  loan  association  had  a  book  value 
of  $215.80  at  the  beginning  of  a  6  months'  period.     The  dues  of  $5  per  month 
for  the  next  6  months,  payable  in  advance,  were  paid  when  due.     What  is  the 
average  book  value  for  the  period  that  should  be  used  in  the  distribution  of 
profits  to  these  5  shares? 

2.  B  subscribed  for  20  shares  in  a  building  and  loan  association,  and  because 
his  subscription  was  made  at  a  time  between  dividend  dates,  he  had  to  pay  $20 
in  dues  each  month  for  4  months.     What  was  the  book  value  of  his  payments? 

3.  Twenty  shares  of  stock  had  a  book  value  of  $800  at  the  beginning  of  a 
6  months'  period.     The  shareholder  became  delinquent  for  3  months.     At  the 
beginning  of  the  fourth  month  he  paid  $80,  and  then  paid  $20  each  month  for 
the  next  2  months.     What  was  the  average  book  value  of  his  20  shares?     If  his 
delinquency  had  been  covered  by  fines,  and  he  had  therefore  been  allowed  full 
participation  in  profits,  what  would  have  been  the  book  value? 

4.  Ten  shares  of  stock  in  the  X.  Building  and  Loan  Association  had  a  book 
value  of  $365.80  on  July  1,  1943.     Dues  of  $1  per  month  per  share  were  paid 
for  the  next  6  months.     On  December  31,  1943,  the  average  book  value  of 
holdings  in  the  association  was  $126,178.36.     The  association  reported  a  net 
gain  for  the  6  months  amounting  to  $4,116.80.     Find:  (a)  the  rate  per  cent 
earned;  (6)  the  dividend  on  the  10  shares,  December  31,  1943;  and  (c)  the  book 
value  of  the  10  shares,  December  31,  1943. 

5.  Adams  paid  $70  in  advance  for  1  share  of  paid-up  stock  in  the  Ames 
Building  and  Loan  Association.     The  maturity  value  of  the  shares  was  placed 
at  $100.     The  per  cents  of  earnings  for  the  5  succeeding  semiannual  periods  were 
4.6%,  3.9%,  4.2%,  5.1%,  and  4.9%,  respectively.     What  was  the  book  value 
of  Adams'  share  at  the  beginning  of  the  sixth  period? 


BUILDING  AND  LOAN  ASSOCIATIONS  435 

6.  The  Garfield  Building  and  Loan  Association  issued  shares  at  the  beginning: 
of  each  month.  Smith  subscribed  for  40  shares  just  1  month  before  the  end 
of  a  6  months'  period.  He  paid  for  these  shares  at  the  rate  of  $40  a  month. 
What  was  the  average  book  value  of  these  40  shares  that  was  used  in  the  dis- 
tribution of  profits  for  the  6  months'  period?  How  much  was  Smith  entitled  to 
receive  as  dividends  if  the  association  showed  a  net  gain  of  $2,631.25,  and  a  book 
value  of  $85,160.72? 

Shares  issued  in  series. 

1.  B  has  20  shares  of  $100  each  in  each  of  2  series  of  the  Capital  City  Building 
and  Loan  Association.  Twenty  shares  are  of  a  series  which  is  ending  its  second 
6  months'  period,  and  20  shares  are  of  a  series  which  is  ending  its  first  6  months' 
period.  B  has  paid  his  dues  of  $20  a  month  on  each  group  of  shares.  The 
association's  rate  of  profit  for  the  6  months'  period  just  ended  is  5^%.  What 
are  the  dividends  on  each  group  of  shares? 

Withdrawal  values. 

1.  Lee  paid  $20  a  month,  for  54  months,  on  20  shares  of  stock  in  the  Midway 
Building  and  Loan  Association.     When  the  55th  payment  was  due,  he  withdrew 
his  money  for  the  withdrawal  value.     The  association  allowed  him  the  sum  of 
his  payments,  and  simple  interest  at  5%  a  year.     The  value  of  the  stock  had 
been  accumulating  at  6%,  interest  convertible  monthly.     What  was  the  book 
value  of  the  stock?     What  was  the  withdrawal  value?     How  much  profit  was 
retained  by  the  association? 

2.  If,  in  Problem  1,  the  interest  paid  on  withdrawals  had  been  calculated  at 
4%,  what  would  have  been  the  difference  between  the  book  value  and  the  with- 
drawal value? 

3.  If,  in  Problem  1,  the  stock  had  been  accumulating  at  7%,  interest  con- 
vertible monthly,  and  simple  interest  at  5%  was  paid  on  withdrawals,  what 
would  have  been  the  difference  between  the  book  value  and  the  withdrawal 
value? 

4.  A  stockholder  who  has  been  making  payments  at  the  rate  of  $10  a  month 
for  85  months,  withdraws  at  the  date  of  the  <H6th  payment.     The  stock  has  been 
accumulating  at  tiie  rate  of  5g-%.     The  association  allows  4%  simple  interest 
on  the  payments  withdrawn.     What  is  the  difference  between  the  book  value 
and  the  withdrawal  value  if  the  stock  has  a  maturity  value  of  $200  a  share? 

The  interest  rate  from  the  borrower's  standpoint 

1.  A  certain  building  and  loan  association  operating  on  a  7  %  nominal  interest 
basis  will  finance  the  building  of  your  house.     If  you  make  payments  of  $1  a 
month  on  each  $100  share,  your  loan  will  mature  with  the  79th  monthly  payment. 
On  the  other  hand,  however,  the  Mutual  Life  Insurance  Company,  through  its 
financial  agents  in  this  city,  is  offering  loans  on  real  estate  at  5-Jr%.     Suppose 
that  you  pay  the  5i%  interest  monthly  in  advance,  and  invest  the  difference 
between  this  amount  and  the  $1  per  share  payable  to  the  building  and  loan 
association  in  a  sinking  fund  at  4%  interest,  payable  monthly,  will  this  prove 
to  be  a  better  proposition  at  the  end  of  78  months? 

2.  If,  in  Problem  1,  the  difference  in  the  monthly  payments  were  placed  in 
a  savings  bank  at  4%,  interest  convertible  semiannually,  would  the  insurance 
company's  proposition  be  the  better  one? 


436  BUILDING  AND  LOAN  ASSOCIATIONS 

The  interest  rate  from  the  borrower's  standpoint  when  the  interest  and 
dues  are  considered  together  as  a  single  sum  for  the  payment  of  interest  and 
principal. 

1.  Jones  borrowed  $1,000  from  an  association  operating  on  the  basis  of  a  6% 
nominal  interest  rate,  convertible  monthly.     Each  month  he  paid  $5  dues  and 
$5  interest.     His  stock  matured  at  the  end  of  11  years  and  3  months,  after 
Jones  had  made  the  monthly  payment  of  dues  and  interest  required  at  that 
time.     What  effective  rate  of  interest  has  Jones  paid  on  his  loan?     (NOTE. — It 
will  be  necessary  to  use  an  estimated  rate,  and  to  solve  by  interpolation.) 

2.  An  association  is  operating  on  a  7%  basis.     Smith  borrows  $1,200,  paying 
$7  interest  and  $12  dues  each  month.     The  79th  payment  is  $7  interest  and 
$2.40  dues.     This  is  the  final  payment,  and  matures  the  stock.     What  is  the 
effective  interest  rate? 

Review. 

1.*  At  the  end  of  its  fourth  year,  the  Thrifty  Building  and  Loan  Association 
has  500  shares  in  the  first  series,  400  in  the  second,  1,000  in  the  third,  and  800  in 
the  fourth,  and  its  total  net  earnings  for  all  years  amount  to  $8,364. 

(a)   Compute  the  rate  of  earnings  under  the  simple  interest  partnership  plan. 

(6)  Prepare  a  share  statement  which  includes  the  profit  per  series,  the  book 
value  per  series,  the  book  value  per  share,  and  such  other  details  as  may  be 
necessary. 

(c)  Compute  the  book  value  of  A/'s  50  shares  of  stock  in  the  first  series. 

(d)  Compute  the  withdrawal  value  of  A^'s  10  shares  in  the  second  series, 
assuming  that  4%  interest  is  allowed  on  withdrawals. 

2.  The  Ypsilanti  Building  and  Loan  Association  desires  that  you  present  to 
them  a  statement  that  they  may  use  to  inform  their  customers  as  to  the  com- 
parative cost  and  returns  of  their  loans  compared  with  loans  of  a  similar  nature 
obtained  from  other  sources.     Bank  loans  in  Ypsilanti,  on  good,  salable  property, 
may  be  obtained  by  the  payment  of  7%  semiannual  interest,  and  an  equal 
semiannual  reduction  of  the  principal.     The  building  and  loan  association  will 
lend  on  good,  salable  property,  on  condition  that  the  borrower  will  pay,  for 
each  $1 ,000  borrowed,  $5  each  month  as  a  repayment  of  the  loan,  and  $6  interest 
each  month.     These  payments  are  to  continue  until  the  $5  and  the  cumulative 
interest  shall  be  a  sum  sufficient  to  repay  the  loan.     Interest  is  allowed  on  the 
repayment  of  the  loan  at  8%,  compounded  semiannually,  January  and  July. 
Eight  per  cent  simple  interest  is  allowed  on  each  monthly  payment  until  the 
date  of  compounding. 

3.  Distribute  a  profit  of  $2,940  on  the  following  series  on  the  partnership 
plan. 

Paid  in          Total        Paid  in          Average          Total  for         Profit 
Series     Per  Share       Shares     Per  Series    Aro.  *\fonths    One  Month   Per  Series 

1  $60  100        30  5 

2  48  200  24  5  

3  36  300  18.5  

4  24  250  12.5  

5  12  400  6.5 

$2,940.00 


*  C.  P  A.,  Pennsylvania. 


BUILDING  AND  LOAN  ASSOCIATIONS  437 

4.  How  many  years  will  be  required  to  mature  stock  with  a  par  value  of 
$100  a  share  if  monthly  payments  of  $1.55  are  made  regularly,  interest  at  6%? 

5.  If  stock  with  a  par  value  of  $100  a  share  matures  in  7  years,  payments 
being  $1  a  share  monthly,  what  effective  rate  of  interest  is  earned  on  the 
investment? 

6.  A  building  and  loan  association  organized  on  the  serial  plan  issued  500 
shares  of  Series  A  stock,  par  value  $100  a  share,  on  the  first  day  of  its  fiscal 
year,  and  500  shares  quarterly  thereafter  for  a  period  of  two  years.     The  first 
of  each  month,  payments  of  50  cents  a  share  were  made  on  this  stock.     At  the 
end  of  two  years  it  was  found  that  profits  for  the  last  quarter-year  available  for 
distribution  to  shareholders  amounted  to  $720.     Find  the  profit  per  share  for 
the  quarter  on  the  basis  of  the  equated  capitals  of  the  respective  series. 

7.  If  in  Problem  6  dividends  have  been  credited  at  the  rate  of  30  cents  a  share 
each  quarter,  find  the  profits  per  share  if  the  $729  were  to  be  distributed  on  the 
basis  of  earning  capital. 


CHAPTER  36 
Permutations  and  Combinations 

Permutation.  A  permutation  is  each  arrangement  which  can 
be  made  by  using  all  or  part  of  a  number  of  tilings.  The  "number 
of  permutations  of  n  things  taken  r  at  a  time,"  represented  by  the 
symbol  „/%,  is  the  number  of  arrangements  of  r  tilings  that  can  be 
formed  from  n  things.  Thus,  using  the  three  letters  a,  6,  and  c 
taken  two  at  a  time,  the  permutations  are  ab,  ac,  6a,  6c,  ca,  and  cb. 
Using  all  of  them  at  the  same  time,  the  permutations  are  abc}  ach, 
bac,  bca,  cao,  arid  cba. 

Since  nPr  is  used  to  denote  the  number  of  permutations  of  n 
things  taken  r  at  a  time1,  its  value  is  determined  as  follows: 

For  first  place:        any  one  of  n  things  may  l>o  chosen; 

For  second  place:  any  one  of  the  remaining,  or  n  —  1,  things  may  he  chosen; 
For  third  place:  any  one  of  the  remaining,  or  n  —  2,  things  may  be  chosen; 
For  fourth  place:  any  one  of  the  remaining,  or  n  —  3,  things  may  be  chosen; 

and  so  forth; 
For  the  last  or 

rth  place:  there  remains  a  choice  of  n  —  (r  —  1)  or  n  —  r  +  1  things, 

Therefore,  J\  =  H(H  -  l)(w  -  2)0*  -  3)   -   •   •   (n  -  r  +  1). 

Take  the  three  letters  a,  fr,  and  c  two  at  a  time.  For  the  fh>fc 
place,  there  is  a  choice  of  3  letters;  for  the  second  place,  there  is  a 
choice  of  3  —  1  letters;  therefore,  3/^2  =  3(3  —  1)  =  0,  the  number 
of  permutations  of  three  letters  taken  two  at  a  time,  as  shown  in  the 
first  paragraph. 

Using  the  three  letters  a,  &,  and  c  all  at  the  same  time,  r  -  n. 
Therefore,  the  symbol  may  be  expressed  nPn  when  all  of  the  n 
things  are  taken  at  once,  and 

nPn  —  n(n  —  \)(n  —  2)(n  —  3)   •  •  •   (until  n  factors  are  used). 

The  symbol  n!,  called  "factorial  n"  denotes  the  product  of  all 
integers  from  n  to  1  inclusive,  and  the  expression  is  abbreviated  to 
nPn  =  n\.  Solving  the  foregoing  example,  we  have  3/^3  =  3  •  2  •  1 
=  6,  the  number  of  permutations  of  the  three  letters  taken  three 
at  a  time,  as  shown  in  the  first  paragraph. 

NOTE:  To  avoid  confusing  the  multiplication  sign  (X)  and 
the  sign  for  quantity  (x),  use  is  made  of  the  •  placed  above  the  line 
of  writing,  and  is  read  "times"  or  "multiplied  by."  Also,  the 

430 


440  PERMUTATIONS  AND  COMBINATIONS 

.  .  .  placed  on  the  line  of  writing  indicates  omission  of  the  "in- 
between"  factors. 

Example  1 

Determine  the  number  of  three-letter  code  words  that  can  be  made  from  the 
letters  of  the  word  bunch,  not  repeating  a  letter  in  any  word. 

Solution 

The  answer  is  the  number  of  arrangements  that  can  be  made  from  five 
objects  (the  letters  of  the  word  bunch)  taken  three  at  a  time. 

Formula 
rfr  =  n(n  -  1)   •  •   •   (n  -  r  +  1) 

Arithmetical  S institution 
J>z  =  5(5  -  1)(5  -  2)  =  5  •  4  •  3  =  60 

Example  2 

Determine  the  number  of  five-letter  code  words  obtainable  in  the  foregoing 
example. 

Solution 

The  answer  is  the  number  of  arrangements  that  can  be  made  from  five 
objects  taken  all  at  the  same  time,  or  J*n  =  til]  and,  since  n\  is  the  product  of  all 
the  integers  from  n  to  1 ,  we  have 

n\  =  n(n  —  l)(n  —  2)(w  —  3)  (ft  —  4),  or  five  factors  in  all. 

Arithmctica I  ft ubst it utio n 
J\  =  5  •  4  •  3  •  2  •  1  =  120 

Notice  thht  the  last  factor  (n  —  r  +  1)  is  one  more  than  the 
difference  be6ween  the  number  of  things,  n,  and  the  number  of 
places,  r.  1  hus,  if  n  is  7,  and  r  is  5,  the  last  factor  is  3 ;  also,  the 
number  of  factors  will  be  equal  to  r.  So  we  may  write  the  formula 
nPr  =  n(n  -  i)(n  —  2)  •  •  •  (until  r  factors  are  used). 

Example  3 

Five  perrons  enter  a  doctor's  waiting  room  in  which  there  are  seven  vacant 
chairs.  In  how  many  ways  can  they  take  their  places? 

Solution 
Formula 

nPr  —  n(n  —  \}(n  —  2)    •   •   •   (until  r  factors  are  used) 
Arithmetical  Substitution 

-jP,  =  7(7  -  1)(7  -  2) (7  -  3) (7  -  4)  (five  factors) 
=  7  •  6  •  5  •  4  -  3  =  2,520 

n\ 
The  formula  as  used  above  may  be  expressed  as  nPr  =  -: -—TV 

(n  -  r) ! 

Using  the  data  in  Example  3,  we  have: 


PERMUTATIONS  AND  COMBINATIONS)  441 


_7-0-:.fi:4.;8.  1^  =  2,520 

Notice  that  the  last  two  factors  cancel  the  two  below  the  line, 
and  that  the  remaining  factors  are  the  same  as  in  the  preceding 
solution. 

Using  the  three  letters  a,  b,  and  c,  two  at  a  time,  we  have* 

n\        _        3!^       _  3-2-  1   _    , 

"    r  ~  (n  -  r}\  "  (3"-~2)"l  1        "   ' 

In  the  foregoing  illustrations,  the  objects  were  distinct,  that  is, 
there  were  no  repetitions  of  objects.  If  the  objects  are  not 
distinct,  the  formula  is  altered  to  read: 

/',=     -WU 

alblcl 

where,  of  the  n  things,  there  are  a  alike,  b  alike,  and  c  alike. 

Example  1 

How  many  permutations  may  be  made  of  the  letters  of  the  word  Illinois? 

Solution 

There  are  eight  letters  in  the  word  Illinois,  but  three  are  i's  and  two  are  I'n. 
Then  we  have: 

8!          40,320 


3! -2!  12 


3,360 


Example  2 

How  many  permutations  may  be  made  of  the  letters  of  the  word  Indianolaf 

Solution 

There  are  nine  letters  in  the  word  Indianola,  but  two  are  a's,  two  are  I'H,  and 
two  are  n's.     Then  we  have: 

9!  362,880 

2!- 21-21-     -*--«•»» 

Number  of  ways  of  doing  two  or  more  things  together.     If  a 

certain  tiling  can  be  done  in  m  ways,  and  a  second  thing  can  be 
done  in  n  ways,  the  two  things  can  be  done  in  succession  in  m  •  n 
ways,  or  mn  ways.  The  principle  can  be  extended  to  find  the 
number  of  ways  of  doing  three  or  more  things  together,  as  m  •  n  •  p  • 
.  .  .  ways. 


442  PERMUTATIONS  AND  COMBINATIONS 

Example 

In  how  many  ways  can  two  positions,  the  one  that  of  bookkeeper  and  the 
other  that  of  stenographer,  be  filled  when  there  are  five  applicants  for  the  position 
of  bookkeeper  and  three  applicants  for  that  of  stenographer? 

Solution 

Assuming  that  all  applicants  are  qualified,  there  are  five  ways  of  filling  the 
position  of  bookkeeper,  and  for  each  of  these  there  is  a  choice  of  three  stenogra- 
phers; hence,  the  two  positions  can  be  filled  in  5-3  =  15  ways. 

For  each  of  the  m  ways  of  filling  the  position  of  bookkeeper  there  are  n  ways 
of  filling  the  position  of  stenographer;  that  is,  there  are  n  ways  of  filling  both 
positions  for  each  way  of  filling  the  position  of  bookkeeper.  Therefore,  there 
are  in  all  mn  ways  of  filling  the  two  positions  together. 

Problems 

1.  If  three  dice  are  thrown  together,  in  how  many  ways  can  they  fall? 

2.  There  are  eight  vacant  seats  to  be  filled  by  five  persons.     In  how  many 
ways  can  they  take  their  places? 

3.  How  many  five-place  numbers  can  be  made  from  the  digits  1,  2,  3,  4,  f> 
0,  and  7? 

4.  What  is  the  number  of  permutations  of  the  letters  (a)  of  the  word  Indiana; 
(b)  of  the  word  Illiopolis? 

6.  Using  three  letters  at  a  time,  how  many  permutations  can  be  formed 
with  the  letters  abed? 

6.  (a)  How  many  permutations  of  the  letters  abcde  can  be  formed  four  at 
a  time?     (b)  Five  at  a  time?     (c)  Three  at  a  time? 

7.  How  many  permutations  may  be  made  of  six  objects  taken:  (a)  six  at  a 
time?  (6)  five  at  a  time?  (r)  two  at  a  time? 

8.  Given  the  numbers  2,  3,  4,  5,  and  6.     How  many  four-phictj  numbers 
can  be  formed  therefrom? 

9.  The  Greek  alphabet  contains  24  letters.     If  no  repetition  of  letters  arc 
allowed,  how  many  three-letter  fraternities  can  be  named  therefrom? 

10.  A  signal  man  has  five  flags,  no  two  of  which  are  alike,  (a)  How  many 
different  signals  can  be  made  by  placing  them  in  a  row  using  all  five  of  them 
each  time?  (6)  How  many  by  using  three  at  a  time? 

Combinations.  A  combination  is  a  set  or  selection  of  r  things 
out  of  a  total  of  n  things  without  reference  to  the  order  within  the 
selection;  therefore,  ab  and  ba  are  the  same  combination.  The 
"number  of  combinations  that  can  be  made  from  a  total  of  n 
things  taken  r  at  a  time"  is  denoted  by  the  symbol  nCr.  Since 
each  combination  can  be  arranged  in  more  than  one  way,  the 
number  of  permutations  is  denoted  by  r!,  and  the  total  number  of 
permutations  for  all  combinations  is  rlnCr.  The  total  number  of 
permutations  of  n  things  taken  r  at  a  time  is  denoted  by  the  symbol 
nPr,  as  in  previous  paragraphs. 


PERMUTATIONS  AND  COMBINATIONS  443 

Therefore,  r!nCr  =  «Pr 

and  r 


(n-r)! 
Substituting  and  dividing  by  r!, 


??: 


Example 

Find  the  number  of  combinations  \\hich  can  be  made  with  the  four  letters 
a,  6,  c,  and  d  taken  three  at  a  time. 

/Solution 


=     -_     ^.          = 

4/3       3!(4-3J!         3-2-1 

These  combinations  are:  abc,  abd,  acrf,  and  bed.  Notice  in 
permutations  that  the  three  letters  a,  b,  and  c  form  six  permuta- 
tions, abc,  acbj  bac,  bca,  cab,  and  cba,  but  there  is  only  one  combina- 
tion abc,  since  all  others  are  merely  a  rearrangement  of  the  same 
letters.  The  addition  of  the  fourth  letter  makes  possible  three 
more  com!  )inations. 

Example 

How  many  lines  ran  be  drawn  connecting  seven  points,  no  throe  of  which  are 
in  the  same  straight  line? 

Solution 

If  we  let  the  points  be  represented  by  the  letters  a,  b,  c,,  d,  c,  /,  arid  g,  and 
any  line  connecting  two  of  them  by  the  symbol  ah,  nc,  and  so  on,  we  find  that 
ab  and  ha  is  the  same  line,  that  ac  and  ca  is  another  line,  and  so  forth;  therefore, 
the  problem  is  that  of  finding  the  number  of  combinations  of  seven  objects 
taken  two  at  a  time. 

,,    _         7!  __  _  7-6  _ 
72  ~  2!(7  -  2)!  ~     21     ~  Zl 

NOTE:  The  factors  from  o  to  1  above  the  line  cancel  the  same  factors  below 
the  line,  leaving  the  factors  indicated  in  the  solution. 

If  r  is  large  and  the  difference  between  n  and  r  is  small,  the  following  formula 
will  save  considerable  work:  nC!r  =  nCn  r. 

Example 

Find  the  value  of  25^23 

Solution 

25^23    =    26^25-23    =    26^'%    ==  =    300 


Problems 

1.  How  many  combinations  can  be  made  with  the  five  letters  a,  6,  c,  d,  and 
e  taken  three  at  a  time? 


444  PERMUTATIONS  AND  COMBINATIONS 

2.  If  twelve  members  of  an  association  are  available  for  committee  assign* 
ments,  how  many  different  committees  of  four  each  can  be  selected? 

3.  You  have  nine  friends  that  you  wish  to  invite  to  dinner  parties  of  four 
guests  each.     How  many  dinner  parties  can  you  have  without  having  the  same 
company  of  four  twice? 

4.  A  committee  consisting  of  two  men  and  one  woman  is  to  be  formed  from 
a  party  of  five  men  and  four  women.    In  how  many  ways  can  the  committee  be 
chosen? 


CHAPTER  37 
Probability 

Probability.  One  of  the  principal  applications  of  permutations 
and  combinations  is  found  in  the  theory  of  probability.  The 
probabilities  for  the  occurrence  of  one  or  more  events,  in  cases  in 
which  it  is  possible  to  count  the  number  of  equally  likely  ways  in 
which  the  event  can  happen  or  fail,  are  known  as  priori  probabilities. 

Counting  of  some  sort  is  the  background  of  probability.  For 
example,  if  a  coin  is  tossed,  the  chances  are  even  between  heads 
and  tails.  If  a  die  is  thrown,  the  chances  of  throwing  any  one  of 
the  numbers  1  to  6  is  1  in  G,  as  there  are  six  surfaces  numbered 
from  1  to  6.  The  chance  of  drawing  an  ace  from  a  well-shuffled 
pack  of  52  cards  is  evidently  4  in  52. 

If  an  event  can  occur  in  m  ways  and  fail  in  n  ways,  and  if  each 
of  these  ways  is  equally  likely,  then  the  probability  of  its  occurring  is 

m 
m  +  n 

and  the  probability  of  failure  ir»  an  event  is 

n 
m  +  n 

Example 

Compute  the  probability  of  throwing  a  3  in  the  first  throw  of  a  die. 

Solution 
m  1  1 


P  = 


m +  n        1+5       0 


Example 

Compute  the  probability  of  failing  to  throw  a  3  in  the  first  throw  of  a  die- 

Solution 


m  +  n       1+5       6 

The  mathematician  computes  the  ratio  of  successes  to  the 
total  number  of  ways  in  which  the  event  can  occur.     For  example, 

445 


446  PROBABILITY 

he  would  say  that  the  chances  of  throwing  a  4  in  one  toss  of  a  die 
is  "one  chance  in  six."  The  average  person  usually  calculates  the 
ratio  of  the  successes  to  the  failures,  and  would  say  "the  odds  are 
five  to  one  against  throwing  a  4." 

Problems 

1.  A  bag  contains  ten  black  balls  and  fifteen  white  ones.     What  is  the 
probability  that  a  ball  drawn  at  random  will  be  black? 

2.  A  box  contains  six  times  as  many  black  balls  as  white  ones  and  one  ball 
is  drawn  at  random.     What  is  the  probability  that  the  ball  drawn  will  be  black? 

3.  If  you  are  to  win  a  prize  valued  at  $12.00  by  throwing  an  ace  in  a  single 
throw  with  a  die,  what  is  the  value  of  your  expectation? 

Permutations  and  combinations  in  probability.  The  following 
examples  will  illustrate  the  statement  made  at  the  beginning  of 
this  chapter. 

Example 

What  is  the  probability  of  obtaining  a  6  if  two  dice  are  tossed? 

Solution 

Under  permutations  we  learned  that  a  succession  of  acts  can  be  performed 
together  in  as  many  ways  as  the  result  of  their  continued  product.  Since  each 
die  has  6  faces,  the  two  dice  can  fall  in  0  X  6,  or  36,  ways.  In  these  36  ways, 
the  sum  6  can  appear  in  any  one  of  the  following  5  ways:  5  and  1  ,  1  and  5,  4  and  12, 
2  and  4,  3  and  3.  Therefore,  the  probability  of  throwing  a  6  is  /V 

Example 

If  two  cards  are  drawn  from  a  complete  deck  of  52  cards,  what  is  the  proba- 
bility that  both  are  hearts? 

Solution 

First  is  the  determination  of  the  number  of  combinations  of  52  objects  taken 
2  at  a  time. 

n\  52' 

C     -  -   ___  ^   -    1    Q9fi 

"   P"»!(n  -r)I  "  21(52-2)1  "     ' 
Second  is  the  determination  of  the  number  of  selections  of  two  hearts. 


nr      _-- 
2!(13  -  2)! 

Therefore,  the  probability  of  selecting  two  hearts  is  T^l  «-  or  -pr- 


Example 

Two  prizes  are  offered  in  a  lottery  of  20  tickets.     What  is  your  probability 
of  winning  a  prize  if  you  hold  five  tickets? 

Solution 
First,  determine  the  number  of  ways  in  which  five  tickets  can  be  selected. 


PROBABILITY  447 

"Cr  =  81(20-5)!  =  15>5°4 

If  you  hold  the  two  prize  tickets,  the  remaining  tickets  may  be  any  three 
of  the  remaining  18,  so  the  number  of  selections  containing  both  prizes  is 

18! 

nCr  =  =  816 


Next,  determine  the  number  of  selections  containing  the  first  prize  and  not 
the  second. 

nCr  =  C  =  3'°6° 


The  number  of  selections  containing  the  second  prize  and  not  the  first  is 
evidently  the  same,  3,060. 

Therefore,  the  probability  of  winning  a  prize  is 

816  +  (2  X  3,060)  =   6,936  =  17 
15,504  ~  15,504  ~~  38 

Example 

In  the  foregoing  example,  what  is  the  probability  that  you  will  not  win  ?, 
prize? 

Solution 

As  two  of  the  tickets  are  winners,  18  are  not  winners,  and  this  is  the  number 
from  which  five  tickets  must  be  selected,  then, 

18!  8,568 


n   r       51(18  -  5)! 
A.S  before, 


The  probability  that  there  will  be  no  winner  is 

8,568    _  21 
15,504  "  38 

Checking  the  answer  to  the  preceding  problem,  we  have: 

i     _    2  1     _    17 

Problems 

1.  A  complete  deck  of  cards  numbers  52  and  is  made  up  of  13  cards  in  each 
of  the  four  suits.     If  four  cards  are  drawn,  find  the  following  probabilities: 

(a)  That  all  are  hearts; 

(b)  That  there  is  one  card  of  each  suit; 

(c)  That  there  are  two  diamonds  and  two  clubs. 

2.  (a)  In  a  single  throw  of  two  dice,  what  is  the  probability  of  throwing  a 
ten?     (b)  What  would  be  the  probability  in  a  single  throw  of  three  dice? 


448  PROBABILITY 

3.  If  you  toss  six  coins,  what  is  the  probability  that  there  are  four  heads 
and  two  tails? 

4.  The  cash  drawer  contains  five  ten-dollar  bills,  six  five-dollar  bills,  and 
seven  one-dollar  bills.     How  many  different  sums  may  be  formed  with  three 
bills  taken  out  at  random? 

Compound  events.  The  j  oint  occurrence  of  two  or  more  simpler 
events  in  connection  with  one  another  is  called  a  compound  event. 
If  two  or  more  events  occur  without  influencing  one  another,  they 
are  said  to  be  independent ;  but  if  any  one  of  them  does  affect  the 
occurrence  of  the  others,  they  are  said  to  be  dependent.  When  the 
occurrence  of  any  one  of  the  events  excludes  the  occurrence  of  any 
other  on  that  occasion,  the  events  are  said  to  be  mutually  exclusive. 

Independent  events.  The  probability  that  n  independent 
events  will  happen  favorably  on  a  given  occasion  (when  all  of  them 
are  in  question)  is  the  product  of  their  separate  probabilities.  If 
the  separate  probabilities  that  an  event  can  occur  favorably  are 
represented  by  p\,  p*,  .  .  .  pn,  and  P  equals  the  probability  that 
all  these  events  will  happen  together  at  a  given  trial,  then, 

P    =    Pl    X   P2    X     '     •     •     Pn 

Example 

What  is  the  probability  that  2,  3,  and  4  are  thrown  in  succession  with  a  die? 

Solution 

The  probability  of  getting  2  is  ^-,  that  of  getting  3  is  -g-,  and  that  of  getting  4 
is  #.  As  these  are  the  probabilities  of  independent  events,  the  joint  probability 
will  be  |  X  i  X  i  =  TTTT. 

Example 

What  is  the  probability  of  getting  2,  3,  and  4  in  one  throw  with  throe  dice? 

Solution 

Consider  the  three  dice  as  being  thrown  separately.  Then  there  are  3  chances 
in  6  of  getting  the  first  number,  2  chances  in  6  of  getting  the  second  number,  and 
one  chance  in  6  of  getting  the  third  number.  Since  these  events  are  all  inde- 
pendent of  one  another,  the  joint  probability  will  be  P  =  f  X  f  X  i  =  -£$• 

Example 

From  a  box  containing  5  brown  marbles  and  4  green  marbles,  3  marbles  are 
drawn.  What  is  the  probability  that  all  three  will  be  brown? 

Solution 

Consider  each  drawing  an  independent  event.  Since  there  are  at  first 
9  marbles  and  5  are  brown,  the  probability  on  the  first  drawing  will  be  -J.  If  a 
brown  marble  has  been  drawn,  then  on  the  second  drawing  the  probability  will 
be  f-.  Now,  if  2  brown  marbles  have  been  drawn,  on  the  third  drawing  the 
probability  will  be  y.  Since  these  three  probabilities  have  been  independent 
events,  the  joint  probability  is 


PROBABILITY  449 

P=t  xix*  =  A 

Mutually  exclusive  events.  Let  the  number  of  mutually  exclu- 
sive events  be  represented  by  pb  p2,  .  .  .  pn.  The  probability 
that  some  one  of  these  events  will  occur  is  equal  to  their  sum; 
therefore,  P  =  p\  +  p2  +  *  *  '  pn- 

Example 

Three  horses  are  entered  in  a  race.  Snowball's  chances  of  winning  are  -J-, 
Thunderbolt's  chances  are  ^-,  and  Fleetwind's  is  -J-.  What  is  the  probability  that 
the  race  will  be  a  tie? 

Solution 

The  winning  of  the  race  by  Snowball,  Thunderbolt,  or  Fleetwind  forms  a 
set  of  mutually  exclusive  events,  since  only  one  can  be  the  winner.  The  proba- 
bility that  one  of  them  wins  the  race  is 


Since  the  race  may  be  a  tie,  that  probability  is 


Example 

If  my  chance  of  completing  a  certain  engagement  is  -^,  and  your  chance  of 
completing  it  is  ^,  what  is  the  probability  that  the  engagement  will  be  completed 
if  we  both  work  independently  of  one  another? 

Solution 
If  we  work  together  to  complete  the  engagement, 

P.  =  i  x  *  =  A 

If  I  complete  the  engagement  and  you  fail  to  complete  it, 

Pt  =  t  X  (1    -  *)   =  TV 
If  you  complete  it  and  I  fail, 

p,  =  £  X  (1  -  t)  =  A 

The  sum  of  pi  +  p*  +  p$  =  •&  +  T2"  +  A  =  f,  the  probability  that  the 
engagement  will  be  completed  by  one  or  the  other  of  us. 

This  result  can  be  checked  by  assuming  that  both  will  fail: 


From  a  box  containing  5  brown  marbles  and  4  green  marbles,  2  marbles  are 
drawn  at  random.     What  is  the  probability  that  both  are  of  the  same  color? 

Solution 
The  probability  that  the  2  are  brown  is  determined  as  shown  on  the  next  page. 


450  PROBABILITY 

5'        -.0       .nrt        ^5^-36;       g.i 


2!(5_2)!       ^        2!(9-2)!      ""'        36       18 

The  probability  that  the  2  are  green  is  determined  in  the  same  manner. 

=    6        and  ' 


21(4-2)1  21(9-2)1      ""'        36       18 

These  two  events  are  mutually  exclusive;  hence,  the  probability  that  both 
marbles  are  of  the  same  color  is 

TF  +  T8"  =  ¥ 

The  probability  that  there  will  be  one  of  each  color  is 

5X4      = 5X4 = 5 
~9T  36     ~  9 


2!(9  -  2)! 

Problems 

1.  A  and  B  engage  in  a  game  of  checkers.     The  probability  that  A  will  wiii 
a  game  is  f  and  that  B  will  win  a  second  game  is  -5-.     What  is  the  probability 
that  both  win? 

2.  From  a  bowl  containing  5  red  marbles  and  6  white  ones,  4  marbles  are 
drawn  at  random.     What  is  the  probability  that  they  are  all  white? 

3.  In  Problem  2,  what  is  the  probability  that  the  4  marbles  drawn  at  random 
are  red? 

4.  In  Problem  2,  determine  the  probability  that  of  the  4  marbles  drawn,  2  aro 
red  and  2  are  white. 

6.  If  three  cards  arc  drawn  from  an  ordinary  pack  of  playing  cards,  what 
is  the  probability  that  all  three  will  be  spades? 

6.  In  Problem  5,  what  is  the  probability  that  all  three  cards  are  black? 

7.  What  is  the  probability  that  all  three  will  be  of  the  same  suit? 

8.  If  the  cards  are  replaced  after  each  drawing,  what  is  the  probability  that 
the  first  three  are  hearts? 

9.  At  an  election,  500  of  the  registered  voters  in  the  precinct  cast  their 
ballots.     Two  hundred  voted  in  favor  of  a  certain  amendment  and  300  voted 
against  it.     If  five  voters  are  chosen  at  random,  what  is  the  probability  that 
they  all  voted  for  the  amendment? 

Empirical  probability.  The  practical  application  of  probabili- 
ties leads  to  a  consideration  of  probabilities  which  are  derived 
from  experience.  These  are  called  empirical  probabilities.  The 
connection  between  probability  and  statistics,  a  subject  devoted 
to  the  analysis  and  interpretation  of  data,  is  found  in  empirical 
probability. 

Example 

The  following  table  gives  the  average  daily  sales  of  92  market  gardeners  in  a 
certain  public  market. 


PROBABILITY 


451 


Average 
Daily 
Sales 

2  50- 

7  49 

7  50- 
12.49 

12  50- 
17  49 

17  50- 
22.49 

22  50- 
27.49 

27  50- 
32.49 

32  50- 
37  49 

37  50- 
42  49 

42  50- 
47  49 

47.50- 
52  49 

Num- 

ber of 
Garden- 

2 

8 

27 

21 

16 

3 

11 

2 

1 

1 

ers 

What  is  the  probability  of  a  person  engaging  in  market  gardening  in  this 
market  of  having  average  daily  sales  of  less  than  $17.50? 

Solution 

The  table  shows  that  of  92  gardeners,  the  number  who  make  less  than  $17.50 
average  daily  sales  is  2  +  8  +  27  =  37.  The  required  probability  is,  therefore, 
|r£,  expressed  decimally  as  0.402. 

Problems 

1.  A  study  of  market  gardening  showed  that  100  gardeners  had  marketed 
in  a  particular  market  as  follows: 

Number  of 

Years  Gardeners 

1  to    5  .  .  .    .     31 

6  to  10  .    .     22 

11  to  15  10 

16  to  20  19 

21  to  25  .  .       6 

26  to  30  .  ...       3 

31  to  35  ...  5 

36  to  40 ...       3 

41  to  45 .       \_ 

100 

What  is  the  probability  of  a  gardener  being  in  this  market  for  20  years  or  less? 

2.  A  survey  among  farmers  to  determine  their  average  income  disclosed  tho 
following  facts: 

Per  Cent  of 


Income  Range 
Negative  income  (Loss) 

0  to  $  500 
$  500  to  $  1,000 
$  1,000  to  $  1,500 
$  1,500  to  $  2,000 
$  2,000  to  $  3,000 
$  3,000  to  $  4,000 

$  4,000  to  $  5,000 

$  5,000  to  $  7,500  . 
$  7,500  to  $10,000.. 
$10,000  and  over 


Farmers 

4.55 

12.53 

21.89 

17.01 

14.69 

13  31 

7.66 

3.25 

3.17 

0.93 

1.01 

100.00 


452 


PROBABILITY 


Based  on  the  above  survey,  what  is  C's  probability  of  making  $2,000  to 
$3,000  a  year  if  he  engages  in  farming? 

3.  A  manufacturer  of  electric  light  bulbs  made  a  test  with  200  bulbs  of 
uniform  design.     The  results  were  tabulated  as  follows: 


Life  in 
Hours 

400 
to 
500 

500 
to 
600 

600 
to 
700 

700 
to 
800 

800 
to 
900 

900 
to 
1,000 

1,000 
to 
1,100 

1,100 
to 
1,200 

1,200 
to 
1,300 

1,300 
to 
1,400 

1,400 
to 
1,500 

1,500 
to 
1,600 

Number  of 

Bulbs 

2 

4 

7 

16 

23 

27 

37 

31 

24 

18 

8 

3 

What  is  the  probability  that  a  bulb  will  burn  out  in  less  than  800  hours, 
based  on  the  experience  of  the  foregoing  test? 

4.  If  three  new  bulbs  are  placed  in  operation  at  the  same  time,  what  is  the 
probability  that  all  of  them  will  last  1,200  hours? 

HINT:  Cube  of  a  single  probability. 

5.  Find  the  probability  that  a  bulb  will  be  "alive"  between  900  and  1,300 
hours. 


CHAPTER  38 
Probability  and  Mortality 

Life  insurance.  Life  insuiance  is  based  upon  probabilities 
determined  by  the  actual  study  of  large  collections  of  mortality 
statistics.  If  an  event  has  happened  m  times  in  n  possible  cases 
(where  n  is  a  large  number),  then,  in  the  absence  of  further  knowl- 

171 

edge,  it  may  be  assumed  for  many  practical  purposes  that  —  is  the 

n 

best  estimate  of  the  probability  of  the  event  and  that  confidence 

771 

in  this  estimate  may  increase  as  n  increases.     The  fraction  —  is 

n 

called  the  frequency  ratio. 

According  to  the  American  Experience  Table  of  Mortality  (see 
page  536),  of  69,804  men  living  at  age  50,  the  number  living  ten 
years  later  will  be  57,917.  The  probability  that  a  man  aged  50 
will  live  ten  years  is  taken  to  be 

57,917 
69^04 

Mortality  table.  Application  of  the  theory  of  probability  is 
made  in  the  study  of  problems  involving  the  duration  of  human 
life,  such  as  life  insurance,  life  annuities,  pensions,  and  so  forth. 
Tables  that  show  the  number  of  deaths  expected  to  occur  during  a 
given  age  are  used  in  the  solution  of  these  problems.  Census 
records  and  vital  statistics  gathered  by  governmental  agencies  are 
the  basis  of  some  mortality  tables.  Others  are  based  upon  the 
records  of  life  insurance  companies.  Results  based  upon  mortality 
tables  are  applicable  only  to  large  groups  of  individuals. 

The  table  on  page  454  is  taken  from  the  American  Experience 
Table.  (The  entire  table  is  given  in  Table  7,  in  the  Appendix.) 

Column  (1)  is  the  age  column  and  contains  the  age  of  100,000 
people  or  their  survivors. 

Column  (2)  indicates  the  number  of  people  living  at  the  begin- 
ning of  the  year  designated  on  the  same  line  in  column  (1).  The 
table  starts  with  100,000  people  alive  at  age  10  and  at  age  11  shows 
that  only  99,251  have  survived.  The  number  that  have  died  in 
the  interval,  749,  is  shown  in  column  (3). 

453 


454 

Column 
Column 


(4)  is 

(5)  is 


PROBABILITY  AND  MORTALITY 

749 


100,000 

:«  ">251 
'"  100,000 


=  .00749  at  age  10,  and  so  on  for  each  age. 
=  .99251  at  age  10,  and  so  on  for  each  age. 


AMERICAN  EXPERIENCE  TABLE  OF  MORTALITY 

(1) 

(2) 

(3) 

(4) 

(5) 

Age 

Number 
Living 

Number  of 
Deaths 

Yearly 
Probability 
of  Dying 

Yearly 
Probability 
of  Living 

X 

I, 

d, 

V* 

Px 

10 
11 
12 
13 
14 

15 
16 
17 

18 
19 

20 

100,000 
99,251 
98,505 
97,762 
97,022 

96,285 
95,550 
94,818 
94,089 
93,362 

92,637 

749 

746 
743 
740 
737 

735 
732 
729 

727 
725 

723 

0.007490 
0  007516 
0.007543 
0.007569 
0.007596 

0  007634 
0  007661 
0.007688 
0  007727 
0.007765 

0  007805 

0  992510 
0.992484 
0.992457 
0.992431 
0.992404 

0.992366 
0.992339 
0.992312 
0.992273 
0.992235 

0  992195 

30 

85,441 

720 

0.008427 

0  991573 

50 

69,804 

962 

0.013781 

0.986219 

70 

38,569 

2,391 

0.061993 

0.938007 

90 

847 

385 

0.454545 

0.545455 

95 

3 

3 

1.000000 

0.000000 

Notation.  For  convenience,  the  letter  I  is  used  to  designate 
entries  in  the  "living"  column.  Subscripts  appended  to  the  I 
denote  specific  entries  in  this  column;  thus,  /i6  indicates  the 
number  living  at  age  15. 

It  is  customary  to  refer  to  the  age  as  x,  that  is,  any  age ;  there- 
fore, lx  would  apply  to  any  entry  in  column  (2). 

The  number  dying  is  denoted  by  d,  which  with  the  subscript,  as 
in  di5,  indicates  the  number  dying  between  the  age  indicated  and 
the  next.  Since  x  stands  for  any  year,  dx  indicates  any  entry  in 
column  (3). 


PROBABILITY  AND  MORTALITY  455 

The  probability  of  a  person  dying  is  expressed  by  q.  With  the 
subscript,  as  in  #15,  it  denotes  the  probability  of  a  person  of  the 
age  indicated  dying  before  reaching  the  following  age.  Similarly, 
qx  stands  for  the  probability  of  a  person  aged  x  dying  before  reach- 
ing age  x  +  1.  This  probability  is  shown  in  column  (4). 

The  probability  of  living  is  denoted  by  the  letter  py  and  pi6 
denotes  the  probability  that  a  person  aged  15  will  live  to  become 
age  16.  This  probability  is  shown  by  the  table,  column  (5),  to  be 
.992366.  The  symbol  px  denotes  the  probability  that  a  person 
aged  x  will  live  to  age  x  +  1. 

For  convenience,  the  symbols  already  explained  and  others 
based  on  them  are  shown  in  the  following  summation  : 

x  =  a  person  or  a  life  aged  x  years 
lx  =  the  number  of  persons  living  at  age  x 
lx+i  =  the  number  living  at  age  x  -f-  1 
/,+n  =  the  number  living  at  age  x  -\-  n 

dx  =  the  number  of  persons  dying  in  the  age  interval  x  to  x  +  1 
dx+i  =  the  number  dying  in  the  age  interval  x  +  1  to  x  +  2 
px  =  the  probability  that  a  person  of  age  x  will  live  one  year 
qx  =  the  probability  that  a  person  of  age  x  will  die  within  one  year 
npx  =  the  probability  that  a  person  of  age  £  will  live  at  least  n  years 
\nQx  —  the  probability  that  a  person  of  age  x  will  not  live  n  years 
n\q*  =  the  probability  that  a  person  of  age  x  will  die  within  one  year  after  reach- 

ing the  age  of  x  +  n 
Pxy  =  the  probability  that  two  persons,  of  ages  x  and  ?/,  respectively,  will  live 

at  least  a  year 

r.pxy  —  the  probability  that  two  persons,  of  ages  x  and  y,  respectively,  will  live 
at  least  n  years 

Probability  of  living.     On  page  453  was  shown  the  probability 


r  *j 

that  a  man  aged  50  will  live  ten  years  is  taken  to  be  r  Q'         =  .8297. 

O«_/, 


The  probability  that  a  person  aged  50  will  survive  10  years  is 
equal  to  the  ratio  between  the  number  living  at  age  60  and  the 
number  living  at  age  50.  Expressed  as  a  formula, 


_ 
10^50    —    7~~ 

tso 


The  probability  that  a  person  aged  x  will  live  to  age  x  +  1  is 
equal  to  the  ratio  of  the  number  of  people  living  at  age  x  +  1  to  the 
number  living  at  age  x,  or 


Example 

What  is  the  probabilitv  that  a  person  of  age  30  will  live  at  least  a  year? 


456  PROBABILITY  AND  MORTALITY 

Solution 

_  U,  _  /a,  _  84721  _ 
P*  ~  IT  ~  Hi  ~  85441  ~  -991573 

The  probability  that  a  person  will  live  longer  than  one  year  or 
n  years  is  expressed  by  the  formula 


Example 

Determine  the  probability  that  a  person  age  40  will  live  to  be  60. 

Solution 


/GO  _  57917  _ 

/To  "  78106  ~  '741 


Probability  of  dying.     The  probability  that  a  person  aged  x  will 
die  within  a  year  is  expressed  by 


Example 

What  is  the  probability  that  a  person  aged  40  will  die  within  one  year? 


Solution 


The  probability  that  a  person  aged  x  will  not  live  to  age  x  +  n 
is  ascertained  by  the  following  formula: 


What  is  the  probability  that  a  person  aged  30  will  not  live  to  age  40? 

Solution 

,  'so  —  '30+10 

MSO  =  ~ ; 

£30 

_  85441  -  78106 
~    8544 J 
=  .085849 


PROBABILITY  AND  MORTALITY  457 

Since  the  sum  of  the  probability  of  living  and  the  probability 
of  dying  equals  1,  or  certainty,  the  foregoing  example  may  be 
solved  as  follows  : 


|lO#30    =     1     —    IOPZO    =    1—7— 
/30 

_    781Q6 
85441 

=  .085849 


The  probability  that  a  person  aged  x  will  die  within  one  year 
after  reaching  age  x  +  n  is  ascertained  by  the  following  formula  : 


Example 

What  is  the  probability  that  a  person  aged  30  will  die  within  one  year  after 
reaching  age  40? 

Solution 


85441 


Joint  life  probabilities.  The  probability  that  two  persons  (x) 
and  (y)  will  survive  at  least  a  year  is  denoted  by  pzy.  As  the 
probabilities  of  life,  or  of  death,  of  two  or  more  persons  are  assumed 
to  be  independent  of  each  other,  it  follows  that 


Example 

A  husband  and  wife  are  aged  37  and  32,  respectively.     What  is  the  proba- 
bility that  both  will  be  alive  at  the  end  of  20  years? 

Solution 
The  probability  that  the  husband  will  be  alive  at  the  end  of  20  years  it? 

WtM,  or  0.7729 

The  probability  that  the  wife  will  be  alive  at  the  end  of  20  years  is 

,  or  0.8076 


Since  the  probability  of  two  separate  and  distinct  events  is  the 
product  of  the  probabilities  of  each  event,  the  probability  that 
both  will  be  alive  at  the  end  of  20  years  is 

0.7729  •  0.8076  =  0.6242 


458  PROBABILITY  AND  MORTALITY 

NOTE  :  Since  (x)  is  used  in  life  insurance  to  denote  the  age  of 
a  person,  it  is  confusing  to  use  it  to  represent  times,  or  multiplied 
by;  therefore,  multiplication  is  indicated  by  the  period  placed  above 
the  line.  The  formula  may  also  be  written 


— 
Pxy   — 


Problems 

1.  What  is  the  probability  that  a  child  aged  12  will  die  between  the  ages 
15  and  16? 

2.  A  father  and  son  are  aged  35  years  and  13  years,  respectively.     Find 
the  probability  that  both  will  be  living  on  the  son's  twenty-first  birthday. 

3.  What  is  the  probability  that  a  man  aged  35  will  die  within  5  years? 
What  is  the  probability  that  he  will  die  in  the  year  after  he  reaches  age  40? 

4.  A  husband  and  wife  are  aged  42  and  40,  respectively.     The  husband 
has  purchased  an  annuity,  the  first  payment  to  be  on  his  65th  birthday.     What 
is  the  probability  that  both  will  be  living  at  that  time? 

5.  What  is  the  probability  that  neither  will  be  living  when  the  first  payment 
of  the  annuity  described  in  Problem  4  is  due? 

6.  If  the  annuity  described  in  Problem  4  is  payable  for  20  years,  what  is  the 
probability  that  both  husband  and  wife  will  be  living  when  the  twentieth  pay- 
ment is  due? 

7.  A  husband  and  wife  are  aged  26  and  24,  respectively,  at  the  date  of  their 
marriage.    What  is  the  probability  that  they  will  live  to  celebrate  their  golden 
wedding  anniversary? 

8.  Y  is  aged  40  and  Z  is  aged  35.     Calculate  the  following: 

(a)  That  Y  will  survive  the  first  year  but  Z  will  not. 
(6)  That  both  will  survive  one  year. 

(c)  That  Z  will  survive  the  first  year  but  Y  will  not. 

(d)  That  both  will  survive  15  years. 

(e)  That  both  will  die  during  the  first  year. 

9.  Find  the  probability  that  a  person  aged  50  will  live  to  age  70. 

10.  Find  the  probability  that  a  person  aged  25  will  not  live  to  age  35.     What 
is  the  probability  that  this  person  will  die  between  the  ages  of  35  and  36? 


CHAPTER  39 
Life  Annuities 

Factors  involved.  In  Chapter  30  it  is  shown  that  the  present 
value  of  a  sum  of  money  payable  n  years  in  the  future  depends 
upon  the  rate  of  interest  which  can  be  earned. 

If  the  payment  of  this  sum  of  money  at  a  future  time  is  contin- 
gent on  some  person  being  alive  at  such  future  time,  the  present 
value  depends  upon  the  rate  of  interest  and  also  upon  the  probabil- 
ity that  the  person  will  be  living.  For  example,  if  two  equally 
good  insurable  risks  aged  25  and  65,  respectively,  are  to  receive 
$1,000  each  upon  attaining  ages  35  and  75,  respectively,  the  present 
value  of  the  promised  payment  to  the  person  aged  25  would  be 
relatively  much  greater  than  to  the  person  aged  65. 

Pure  endowment.  A  pure  endowment  contract  promises  to 
pay  to  the  holder  thereof  a  definite  sum  of  money  if  he  is  living  at 
the  end  of  a  specified  period,  but  nothing  to  his  beneficiaries  if  he 
fails  to  survive  this  period. 

The  present  value  of  an  n-year  pure  endowment  of  1  to  a  person 
now  aged  x  is  expressed  by  the  symbol  nEX9  which  is  equivalent  to 
the  present  value  of  1  to  be  received  at  the  end  of  n  years,  multi- 
plied by  the  probability  npx  that  a  person  aged  x  will  survive  n 
years. 

If  nEx  denotes  the  present  value  of  an  n  years'  pure  endowment 
to  a  person  of  age  xy  we  have 

TJ1       I     f I       T)  T~)        *  \  n 

n-I-J  x  I/I       i       "\       I  £  x  /~ 

OH 


Example 

A  person  aged  20  is  to  receive  $5,000  upon  attaining  age  25.     Find  the 
present  value  of  the  probability,  interest  at  8^%. 

Solution 

*25 


5,000^*0  =  5,000    (L03*)5 


459 


460  LIFE  ANNUITIES 

The  probability  that  the  person  will  receive  the  money  is 


-  j—  =  .8419732,  the  present  value  of  1  for  5  years  at  3i% 
(1.03-jf) 

The  present  value  to  the  person  aged  20  is,  then, 

$5,000  X  .8419732  X  .9610846  =  $4,046.04 

Problems 

1.  A  girl  aged  10  is  to  receive  $5,000  upon  attaining  age  18.     Find  the  present 
value  of  the  inheritance,  interest  at  3^%. 

2.  A  persor,  aged  20,  is  to  receive  $10,000  upon  reaching  age  30.     Find  the 
present  value  of  his  expectation  on  the  basis  of  3^%  interest  and  the  American 
Experience  Table  of  Mortality. 

3.  Find  the  present  value  of  a  pure  endowment  of  $2,000  to  a  person  aged  30 
payable  if  he  reaches  the  age  of  60,  on  a  3-g-%  basis. 

Life  annuity.  A  series  of  periodical  payments  during  the  con- 
tinuance of  one  or  more  lives  constitutes  a  life  annuity.  The 
simplest  form  of  a  life  annuity  to  a  person  age  x  is  the  payment  of 
1  at  the  end  of  each  year  so  long  as  the  person  now  aged  x  lives. 
Such  an  annuity  consists  of  the  sum  of  pure  endowments  of  1  each 
year.  The  symbol  for  a  life  annuity  is  ax\  therefore,  ax  =  iEx  + 
2EX  +  3#*  •  •  •  +  nEx  •  •  •  to  table  limit. 

Substituting  these  values  gives  : 

vlI+i  +  v*lx+2  +  vHf+3  '  '  '  +  vnlx+n 
ax  =  —  ±—  --  ±  -----  —  —  --   ----  --   •  •  •  to  table  limit. 

Lx 

Example 

Find  the  value  of  a  life  annuity  of  $1,000  a  year  to  a  person  now  aged  90, 
interest  at  3-j%. 

.035)-4/94)  +  ((1.035)-  V) 


Present  values  are  found  in  Table  3.     Values  of  ko,  ki,  and  so  forth,  are 
found  in  Table  7. 

Substituting  all  the  indicated  values  and  solving,  we  have 

ago  =  .8738 
$1,000  X  .8738  =  $873.80 

Problem 

A  life  pension  of  $500  a  year,  payable  at  the  end  of  each  year,  is  granted  to  a 
person  now  aged  91.     What  is  the  present  value  of  this  pension,  interest  at  3^%? 

Commutation  columns.     In  the  examples  and  in  the  preceding 
problem,  the  computations  are  not  particulary  arduous,  because  the 


LIFE  ANNUITIES  461 

of  the  annuitant  made  it  necessary  to  make  only  a  few  com- 
putations. But,  in  cases  where  the  annuitant  is  younger  —  for 
example,  age  20  —  it  is  evident  that  a  great  amount  of  work  would 
be  required  in  order  to  solve  the  problem.  Much  of  this  compu- 
tation may  be  eliminated  by  the  use  of  tables  called  '  '  commutation 
columns"  (see  Table  8). 

The  first  column  of  this  table,  the  Dx  column,  has  been  con- 
structed of  the  products  of  similar  ?/s  and  Z's  and  the  product 
denoted  by  the  letter  D.  The  computation  was  therefore  reduced 
to  the  addition  of  the  values  found  in  the  Dx  column.  To  save 
time  in  adding  these  values,  another  commutation  column  was 
formed,  containing  the  sums  of  all  the  Z>'s  from  any  particular 
value  of  Dx  to  the  table  limit.  This  is  the  Nx  column  of  Table  8. 

Therefore,  the  work  is  materially  reduced  by  using  the  tables 
and  the  formula  : 


The  solution  to  the  example  on  page  460  now  becomes  : 


A^t      AWi        33.47 
~D,  '=~7>;r  ^  38:3047  = 
$1,000  X  .8738  =  1873.80 


Using  the  commutation  tables,  the  present  value  of  the  pure 
endowment  on  page  459  may  be  found  by  the  formula 


Substituting  the  values  for  the  example  on  page  459,  we  have 
»*«•  -  P£l  -  "S(>9207 

40556.2 

and 

$5,000  X  .809207  -  $4,046.04 

From  the  foregoing  formula  it  may  be  found  that  1  at  age  a 
will  purchase  an  n-year  pure  endowment  of 

Dz 


Dx+n 

Problems 

1.  What  is  the  present  value  of  a  life  annuity  of  $3,000  to  a  person  aged  30, 
interest  at  3i%? 

2.  Find  the  value  of  a  life  annuity  of  $2.500  at  3^%  to  a  person  aged  35. 


462  LIFE  ANNUITIES 

Life  annuities  due.  The  principles  of  annuities  apply  in  life 
insurance.  The  preceding  illustration  was  that  of  a  life  annuity 
where  the  payment  was  made  at  the  end  of  each  year.  When  the 
payments  are  to  be  made  at  the  beginning  of  each  year,  the  life 
annuity  is  a  life  annuity  due.  Actuaries  use  the  symbol  a,  to 
represent  the  present  value  of  an  annuity;  and,  since  an  annuity 
due  differs  from  an  ordinary  annuity  by  an  additional  payment 
made  at  the  beginning  of  the  period,  the  present  value  of  an  annuity 
due  is  1  +  ax,  and  the  symbol  becomes 

a,  =  1  +  ax 

using  a  different  type  "a"  from  that  used  in  the  ordinary  life 
annuity.  Since  the  different  type  "a"  is  somewhat  difficult  to 
make,  the  regular  a  may  be  used  and  distinguished  by  a  bar 
over  it,  thus, 

tin  =  1  -f-  ®x 

N  +i 
Use  of  commutation  table.     Since  ax  =  —~-  >  the  life  annuity 

due  formula  may  be  written  as 

&x=sl+~157 
arid  if  for  1  we  substitute  -rr'  we  have 


which  is  equivalent  to 


the  formula  for  the  present  value  of  a  life  annuity  due  of  1  payable 
to  a  person  aged  x.  The  values  may  be  obtained  from  the 
commutation  table. 

Example 

Find  d3o 

.        N, 


596804   .          .     A  U1 
a3o  =  OQ44Q  o  from  ttie  table 

a3o  =»  19.6054,  also  shown  in  the  table  in  the  1  —  az  column. 


LIFE  ANNUITIES  463 

Problems 

(Use  the  commutation  table.) 

1.  Find  #20. 

2.  Find  D35. 

3.  If  x  =  75,  find  Dx. 

4.  Find  Nxitx  =  22. 

6.  When  Dx  =  25630.1,  what  is  the  value  of  *•? 

6.  When  Nx  =  20S510,  what  is  the  value  of  j? 

7.  At  what  age  does  Ar,  =  157,255? 

8.  At  what  age  does  Dx  =  19S7.87? 

9.  What  is  the  difference  in  value  between  <740  and  a40? 
10.  Find  (1)  a30;  (2)  a3o. 

Deferred  annuity.  When  the  first  payment  under  a  life  annu- 
ity is  to  be  made  after  the  lapse  of  a  specified  number  of  years 
(contingent  upon  the  annuitant  (x)  being  alive),  instead  of  being 
made  a  year  after  the  payment  of  the  single  premium,  the  annuity 
is  deferred. 

Since  under  an  ordinary  annuity  the  first  payment  is  made  at 
the  end  of  one  year,  then  if  an  annuity  is  deferred  n  years,  the  first 
payment  is  made  at  the  end  of  n  +  I  years;  but  an  annuity  provid- 
ing for  the  first  payment  at  the  end  of  n  years  is  deferred  n  —  I  years, 
for  the  annuity  is  entered  upon  at  the  end  of  n  —  1  years,  and  the 
first  payment  is  not  made  until  one  year  later;  and  it  is  a  deferred 
life  annuity  due. 

The  present  value  of  a  life  annuity  of  $1.00  deferred  for  n  years 
is  expressed  by  the  symbol 

but  in  n  years  the  annuitant's  age  will  be  x  +  n,  and  the  value  of 
the  annuity  will  be  a,+n;  and,  since  it  is  desired  to  find  the  value  of 
this  annuity  now,  we  discount  it  by  multiplying  ax+n  by  the  regular 
present-value  symbol,  vn. 

However,  three  factors  are  to  be  considered  as  follows : 

(a)  The  value  of  the  life  annuity,  ax+n; 

(6)  The  present  value  of  1  in  n  years,  vn\ 

(c)  The  probability  that  the  person  aged  x  will  be  living  n  years  from  now,  npx. 

Therefore,  the  formula  becomes : 

and,  making  substitutions  so  that  the  solution  may  be  obtained 
from  the  commutation  table,  we  have 

n|a*  =  ~~DT 


464  LIFE  ANNUITIES 

Example 

What  single  premium  will  a  person  aged  30  have  to  pay  to  obtain  a  life 
annuity  of  $2,500,  so  that  he  will  receive  his  first  annuity  payment  at  the  end 
of  his  46th  year? 

Solution 


D 


From  the  commutation  tables,  it  is  found  : 

253745 
"|flso  ~  1577l7> 

=  16.08669 
$2,500  X  16.08669  -  $40,216.73 

Deferred  life  annuity  due.  The  first  payment  of  an  annuity 
due  would  be  made  1  year  before  that  of  an  ordinary  deferred 
annuity;  therefore,  the  deferred  life  annuity  due  is  the  equivalent 
of  an  ordinary  life  annuity  deferred  for  n  —  1  years,  and  the 
formula  is 


,  . 

nax  =  n-i\ax         or         nax  = 


Example 

Y  is  aged  55,  and  he  desires  to  purchase  a  life  annuity  of  $2,500,  the  first 
payment  to  be  made  at  age  65.     What  is  the  single  premium  payment? 

Solution 


and 


_  4S616.4 
~  9733.40 
=  4.994cS 
$2,500  X  4.994S  =  $12,487.00 

Problems 

1.  A  child  15  years  of  age  is  to  receive  $2,400  a  year  for  life,  the  first  payment 
to  be  made  at  age  21.     Calculate  the  value  of  this  annuity  at  3^%. 

2.  What  single  premium  payment  will  a  person  aged  35  have  to  pay  to 
obtain  a  life  annuity  of  $3.000  from  which  he  will  receive  his  first  annuity  pay- 
ment at  age  60? 


LIFE  ANNUITIES  465 

Temporary  life  annuities.  A  temporary  life  annuity  continues 
for  n  years,  contingent  on  the  annuitant  living  that  long;  hence,  it 
is  not  an  annuity  certain.  The  symbol  for  a  temporary  life  annu- 
ity is  axn\j  and  the  formula  is 


'"'  D, 

Example 

Find  the  present  value  of  a  life  annuity  of  $2,000  for  25  years,  to  a  person 
aged  40. 

Solution 


=  324440  -  43343J 

19727.4""" 
-  14.24906 
$2,000  X  14.24906  -  $28,498.12 

Temporary  annuities  due.  The  present  worth  of  a  temporary 
life  annuity  due,  also  termed  an  immediate  temporary  annuity,  is 
equivalent  to  the  difference  between  the  present  worth  of  a  whole 
life  annuity  due  and  a  deferred  life  annuity  due,  and  may  be 
expressed  as 

Uxn\    ~    <*•*    —    n|«x 

Substituting  values,  the  formula  for  use  with  commutation 
tables  becomes 

Nx  -  Nx+n 


Example 

Y  buys  a  temporary  life  annuity  of  $1,200  for  his  widowed  mother  aged  50. 
Payments  are  to  begin  at  once  and  to  continue  until  age  75.  What  is  the  present 
value  of  this  annuity  due? 

Solution 


"'    />* 

#50  25  ~  fT— 


_  181663  -  11728.9 

12498.6 
=  13.59625 
$1,200  X  13.59625  =  $16,315.50 


466  LIFE  ANNUITIES 

Problems 

1.  Find  the  present  value  of  a  temporary  life  annuity  of  $1,500  for  5  years 
co  a  person  aged  65. 

2.  Find  the  values  of  a20ioi,  nl6-M[j  and  r/3510> 

3.  Find  the  values  of  io|«2o,  2o|«i6,  and  io|^35. 

Life  annuities  with  payments  m  times  a  year.  Annuity  con- 
tracts often  provide  that  payments  shall  be  made  more  frequently 
than  once  a  year,  such  as  quarterly  or  monthly,  the  latter  being 
more  common.  For  an  annuity  payable  m  times  a  year,  the 
symbol  ar(m)  is  used  to  denote  its  present  value,  and  the  formula 
used  to  determine  the  value  when  the  payments  are  made  at  the 
end  of  the  period  is 

,   w  -  1 

«x(w)  =  fix  H  —  o  — 
2m 

Example 

Find  the  present  value  of  a  life  annuity  of  $(500  a  year  payable  monthly,  the 
first  installment  to  be  paid  in  one  month,  for  a  person  35  yeais  of  age. 

Solution 


o 

2-rii 

Substituting  values, 

«35(12)    =    035  +  U 

_  #36  _    482326 
035  ~  AT*  ~  24^447 

=  17.0143 
U  =  -45S3 

17.6143  +  .4583  -  1S.0726 
$600  X  18.0726  -  $10,843.56 

If  the  payments  are  made  at  the  beginning  of  the  period,  they 
constitute  an  annuity  due,  and  its  present  value  will  be: 


2m 


For  a  deferred  life  annuity  of  1  a  year,  payable  in  in  install- 
ments a  year,  the  present  value  is 


For  a  temporary  life  annuity  for  n  years,  payable  in  m  install- 
ments a  year,  the  present  value  is 


LIFE  ANNUITIES  467 

Example 

The  value  of  a  temporary  life  annuity  of  $180  a  year  payable  annually  for 
15  years  to  a  person  aged  45  is  $1,873.96. 

What  is  the  present  value  of  an  annuity  of  the  same  annual  rent  if  paid  in 
monthly  installments  of  $15  ea^h,  the  first  payment  one  month  hence? 

Solution 
$l80a46fri  =  $1,873.96 

a^I10'4109  11 

"  1B^  =  §=1  |^  .46606 
Therefore, 


a46<12)16l-  =  10.4109  +  fl 

=  10.4109  +  .2447  -  10.6556 
$180(10.6556)  -  $1918.01 

Problems 

1.  Find  the  present  value  of  a  pension  of  $75  a  month  payable  at  the  end 
of  each  3  months  to  a  pensioner  aged  65. 

2.  A  corporation  executive  aged  5S  is  to  be  retired  at  age  65.     During  retire- 
ment he  will  receive  $3,600  a  year  payable  in  monthly  installments.     Find  the 
present  value  of  this  retirement  allowance  on  a  3^%  basis. 

3.  A  life  annuity  contract  provided  for  a  payment  of  $750  a  year  for  15 
years,  the  first  payment  to  be  made  at  age  60.     At  age  60  the  annuitant  desired 
monthly  payments.     Find  the  amount  of  the  monthly  payments. 

4.  A  widow  was  to  receive  $1,800  a  year  for  life  in  annual  payments,  the 
first  payment  to  be  made  one  year  after  her  husband's  death.     When  the  first 
payment  was  due,  the  widow  was  65  and  asked  that  the  payments  be  made 
monthly.     What  amount  should  she  receive  monthly? 

Forborne  temporary  annuity  due.  A  forborne  temporary 
annuity  due  is  created  when  a  person  who  is  entitled  to  a  life 
annuity  due  of  1  a  year  forbears  to  draw  it  and  agrees  that  the 
unpaid  installments  are  to  accumulate  as  pure  endowments  until 
he  is  aged  x  +  n. 

On  page  465  the  present  value  of  a  temporary  annuity  due  was 
found  by  the  formula 


and  on  page  461  it  is  given  that  1  at  age  x  will  purchase  an  n-year 
pure  endowment  of 


Then  the  present  value  would  buy  a  pure  endowment  equal  to 


468  LIFE  ANNUITIES 

7)  AT    —  AT  A7    —  N 

D^^N^N^       or        N*      Nl" 


Dx+n 

Problems 

1.  Find  the  amount  at  age  60  of  a  forborne  temporary  annuity  due  of  1  a  year 
that  is  to  be  accumulated  for  a  person  now  aged  40. 

2.  A  man  was  to  receive  a  life  annuity  of  $1200  a  year,  the  first  payment  to 
be  made  one  year  mter  his  60th  birthday.    At  that  time  he  was  still  employed 
at  a  good  salary,  and  so  decided  to  postpone  the  beginning  of  the  annuity  for 
5  years.    What  yearly  sum  will  he  receive,  the  first  payment  to  begin  on  his 
66th  birthday?     (Use  American  Experience  Table  of  Mortality  and  83- %,  and 
treat  the  postponed  annuity  as  a  forborne  annuity.) 


CHAPTER  40 
Net  Premiums 

Net  single  premium.  The  net  single  premium  is  equal  to  the 
present  value  of  the  benefit  influenced  by  rates  of  mortality  and 
interest. 

The  net  single  premium  for  a  whole  life  policy  (a  policy  pay- 
able at  death  only)  is  denoted  by  Ax.  Solution  of  a  problem  of 
this  type  is  simplified  by  use  of  the  commutation  columns  Mx  and 
Ac,  thus: 

A    -^ 
x  ~    Df 

Example 

Find  the  net  single  premium  for  $3,000  whole  life  insurance  on  a  person 
aged  24. 

Solution 

A~<  =  ^ 


$3,000  X  .303644  =  $910.93 

Annual  premiums.  Life  insurance  premiums  are  most  fre- 
quently paid  in  equal  annual  payments,  but  they  may  be  paid 
semiannually,  quarterly,  or  monthly,  and,  in  the  case  of  industrial 
insurance,  weekly.  Rates  other  than  annual  are  greater  in  pro- 
portion than  annual  rates,  for  they  include  interest  and  additional 
overhead  or  administrative  costs. 

On  an  ordinary  life  policy,  payments  continue  throughout  the 
life  of  the  insured.  On  a  limited  payment  life  policy,  the  premium 
payments  are  limited  to  a  certain  number  of  years,  such  as  20 
years  on  a  20-payment  life  policy. 

The  net  annual  premium  is  the  annual  payment  made  at  the 
beginning  of  each  policy  year,  the  sum  thereof  being  the  equivalent 
of  the  net  single  premium.  The  annual  premiums  constitute  an 
annuity  due  payable  by  the  policy  holder  to  the  insurance  com- 
pany. Px  is  the  symbol  used  for  the  net  annual  premium  and, 
using  commutation  columns, 


469 


470  NET  PREMIUMS 

Example 

Find  the  net  annual  premium  for  an  ordinary  life  policy  of  $1,000  issued  to 
a  person  aged  30. 

Solution 

_3/3o_  10259.0  _ 
1  3°  ~  ~N^  ~  596804   --017189 
$1,000  X  .017189  =  $17.19 

If  the  premium-paying  period  is  limited  to  a  certain  number  of 
years,  such  as  10  years  in  a  10-payment  life  policy,  then  the  pay- 
ments are  equivalent,  interest  and  mortality  considered,  to  the 
single  net  premium.  nPx  is  the  symbol  used  for  the  net  annual 
premium  for  an  n-payment  life  policy  to  a  person  aged  x,  and,  in 
terms  of  commutation  columns: 


Example 

Find  the  net  annual  premium  for  a  20-payment  lift1  policy  for  $2000  issued 
to  a  person  aged  45. 

Solution 

MK  7192.S1  7192.S1 


20  46       #45  -  A',*       253745  -  4S616       205129 
$2000-  .03506  =  $70.12 

Term  insurance.  Other  than  group  life  insurance,  term  insur- 
ance is  the  lowest-cost  life  insurance  obtainable.  The  term  may 
be  one  year,  five  years,  or  ten  years,  and  so  forth,  and  the  face  value 
of  the  policy  is  payable  in  the  event  of  death  within  the  stated  term. 

The  net  single  premium  for  term  insurance  may  be  ascertained 
from  commutation  columns,  using  the  formula 


Example 

Find  the  net  single  premium  for  a  10-year  term  insurance  of  $5,000  at  age  2£ 

Solution 


11()31.1  -  9094.96 


37673.6 
$5.000  X  .067318  =  $336.59 


37673.6 

=  .0673,8 


NET  PREMIUMS  471 

Annual  premium  for  term  insurance.  The  payments  of  pre- 
mium constitute  an  annuity  due  for  a  definite  term,  and  the  net 
annual  premium  may  be  determined  from  commutation  columns, 
using  the  formula 


Example 

Find  the  net  annual  premium  for  a  10-year  term  insurance  of  $3,000  at  Age  20. 

Solution 

*    20  io  I  ~  "T7  \r 

A  -jo  —  A  ao 

_  13207.3  -  10259.0 
9S4400  -  59(>S04 

"  So  =  -007701 
$3,000  X  .007761  =  $23.38 

Net  single  premium  for  endowment  insurance.  An  endow- 
ment policy  provides  for  payment  of  the  face  value  of  the  policy 
at  the  end  of  the  stated  period  if  the  insured  he  living,  or  to  the 
named  beneficiary  or  beneficiaries  should  death  occur  before  the 
end  of  the  stated  period. 

Endowment  insurance  may  be  considered  as  term  insurance  of 
1  for  n  years  plus  an  n-year  pure  endowment  of  1,  and  the  net 
single  premium  may  be  found  from  commutation  columns  by  the 
use  of  the  following  formula: 

Mx  —  Mjcm  -h  />>,+„ 

A.n\  = -f)~ • 

Example 

Find  the  net  single  premium  on  a  10-year  endowment  policy  for  $5,000  at 
rtgc  30. 

Solution 

_  Mao  —  A/40  +  (>w 

^30   10*1    ~~     ~~  ~~  TV 

^30 

_  10259.0  -  8088.91  +  19727.4 
30440.* 

_  21897.49  _ 
~  1^04408   ~ 
$5,000  X  .719346  =  $3,596.73 

Annual  premium  for  endowment  insurance.  The  net  annual 
premium  for  r  years  for  an  n-year  endowment  insurance  of  1  may 
be  found  from  commutation  columns,  using  the  formula 


472  NET  PREMIUMS 

M,  -  M1+n  +  L 


Example 

Find  the  net  annual  premium  on  a  15-year  endowment  policy  for  $10,000 
purchased  at  age  40. 

Solution 


__  ~    A/55   +   £>55 

40  161    ~  vr  \T~ 

N  40   —   A/55 

^  8Q88-01  -  5510.54  +  9733.40 
344167  -  124876" 


$10,000  X  .056143  =  $561.43 

Miscellaneous  Problems 

1.  Find  the  net  single  premium  for  a  whole  life  insurance  of  $1,000  at 
age  30. 

2.  What  is  the  increase  in  the  net  single  premium  for  a  whole  life  insurance 
of  $2,000  from  age  25  to  26? 

3.  What  is  the  present  value  of  a  life  annuity  of  $1,000  a  year  at  age  25? 

4.  Find  the  net  annual  premium  for  an  ordinary  life  policy  of  $1,000  at 
age  20. 

5.  Find  the  net  annual  premium  for  a  20-payment  life  policy  for  $10,000 
at  age  30. 

6.  What  is  the  net  annual  premium  for  a  10-payment  life  policy  for  $2,000 
at  age  60? 

7.  What  is  tne  net  single  premium  for  10-year  term  insurance  of  $5,000  at 
age  20V 

8.  What  is  the  net  annual  premium  on  a  10-year  endowment  policy  for 
$5,000  at  age  45? 

9.  Find  the  net  annual  premium  for  a  20-payment  endowment  at  age  65 
for  $5,000  if  the  insured  is  45  at  date  of  issue. 

10.  Find   the  difference  in   annual  premiums   between   a  20-payment  life 
policy  and  a  20-year  endowment  policy,  each  for  $5,000,  issued  at  age  30. 


CHAPTER  41 
Valuation  of  Life  Insurance  Policies 

Mortality  and  the  level  premium.  If  the  cost  of  insurance  on 
a  group  of  men  were  to  be  met  each  year  by  payment  into  a  fund 
of  just  the  amount  necessary  to  meet  that  year's  death  loss,  the 
amount  would  be  low  at  first,  and  finally  prohibitive,  hence  the 
necessity  of  a  level  premium.  In  order  to  have  a  level  premium, 
an  excess  over  current  death  losses  is  collected  during  the  early 
years  to  bear  the  burden  of  later  years  when  losses  exceed  the 
premium  income. 

This  excess  premium  is  known  as  the  "  reserve."  When  cal- 
culated on  the  basis  prescribed  by  law,  it  is  called  the  "  legal 
reserve/'  and  "legal  reserve"  companies  are  referred  to  as  "old 
line"  companies. 

Policy  reserves.  To  show  the  meaning  of  insurance  reserves, 
a  simple  illustration  is  given. 

Assume  that  an  ordinary  life  policy  for  $1,000  is  purchased  at 
age  20.  The  net  annual  premium,  calculated  from  Table  8. 
would  be 


$1,000  X  .013847  =  $13.85 

Term  insurance  for  one  year  at  the  same  age  would  be  : 

1st  year: 

.          C20       351.07         -_„. 

A20  =   yr-  =  =   .00754 

DM      40556.2 

$1,000  X  .00754  =  $7.54 
3th  year: 

C25        293.55 

A25  =  ^  =  3767^6  =  -00779 
$1,000  X  .00779  =  $7.79 

and  so  on. 

C  Comparing  these  premiums  over  a  period  of  years,  we  have: 

473 


474  VALUATION  OF  LIFE  INSURANCE  POLICIES 

Ordinary  One  Year 

Age              Life  Term 

20  $13.85  $  7.54 

25              13.85  7.79 

30               13  85  8.14 

35              13  85  8  64 

40               13  85  9.46 

45              13  85  10  79 

50               13  85  13  32 

55              13  85  17  94 

60               13  85  25  79 

65               13  85  38.77 

The  excess  of  the  premium  on  ordinary  life  over  one-year  term 
insurance  is  the  amount  placed  in  the  reserve  to  be  accumulated 
for  heavier  losses  which  will  occur.  It  will  be  noticed  that  between 
50  and  55  and  from  that  point  on  the  ordinary  life  premiums  will 
be  insufficient;  therefore,  the  reserves  will  be  drawn  upon  to  meet 
the  difference. 

Interest  and  the  premium.  Reserves  are  invested  in  securities 
and  earn  interest  which  increases  the  reserves.  The  amount  of 
interest  earned,  therefore,  is  reflected  in  lower  premiums.  If  a 
company  assumes  a  rate  of  interest  lower  than  the  maximum 
permitted  under  insurance  law,  the  premium  is  higher  and  a  larger 
reserve  is  accumulated  during  the  early  policy  years. 

Loading.  Using  the  mortality  table  and  an  assumed  rate  of 
interest  to  be  earned  on  the  reserve,  the  actuary  arrives  at  the  net 
level  premium.  To  this  must  be  added  the  expense  of  doing 
business,  or  overhead,  which  amount  is  called  "loading."  The  net 
premium  plus  the  loading  is  the  premium  rate  to  the  purchaser  of 
insurance. 

Expenses  are  heaviest  in  the  first  policy  year;  therefore,  the 
plan  is  modified  to  permit  more  of  the  premium  to  be  used  for 
expenses,  and  this  is  balanced  by  lowering  the  amount  required 
for  the  reserve.  Methods  of  modification*  are  not  presented  in 
this  text. 

Dividends  and  net  cost.  Mortality  may  differ  from  that  shown 
by  the  table;  interest  may  be  earned  in  excess  of  the  rate  antici- 
pated; the  loading  may  exceed  the  actual  costs.  Such  savings 
result  in  dividends  to  policy  holders  in  mutual  companies  and  to 
holders  of  participating  policies  issued  by  stock  companies.  The 
net  cost  of  the  insurance  is  the  premium  paid  less  these  dividend 
refunds. 


*  An  extended  discussion  of  these  methods  will  be  found  in  Robert  Riegel  and 
H.  J.  Loman,  Insurance  Principles  and  Practice.  New  York:  Prentice-Hall,  Inc., 
rev.  ed.,  1929. 


VALUATION  OF  LIFE  INSURANCE  POLICIES  475 

Terminal  reserves.  When  a  policy  its  issued,  the  mathematical 
expectation  of  the  future  premiums  equals  the  benefit. 

As  the  insured  grows  older,  the  value  of  the  future  premiums 
becomes  less  and  the  value  of  the  benefit,  conversely,  becomes 
greater. 

The  value  of  the  benefit  is  represented  by  Ax+n,  and  the  net 
annual  premium  constitutes  a  life  annuity  with  a  value  represented 
by  Px(l  +  djr+n);  therefore,  nVx,  the  terminal  reserve,  is  found  by 
the  formula 

»r,  =  Ax+n  -Pt(l  +  «,,„) 

The  foregoing  method  of  valuation  is  called  the  prospective 
method. 

Example 

Find  the  terminal  reserve  of  the  20th  policy  year  on  an  ordinary  life  policy 
of  $1,000  issued  at  age  30. 

Solution 


1  +  13.5347  =  $14.5347 

_j|/,._1  0259.0 

lm~  A^~  590804 
Substituting  values  gives: 

20^30  =  .50849  -  (.017190  X  14.5347) 

=  .25864 
$1,000  X  .25864  =  $258.64 

The  surrender  value  is  the  sum  which  the  insurance  company 
pays  the  policy  holder  upon  the  surrender  and  cancellation  of  the 
policy.  Whether  the  amount  will  be  greater  or  less  than  an 
amount  as  calculated  in  the  foregoing  example  is  dependent  on 
averages  obtained  from  the  company's  records  instead  of  the 
theoretical  amount  so  calculated.  Policies  contain  a  table  showing 
the  company's  contractual  surrender  value  for  each  $1,000  of 
insurance. 

Problems 

1.  Find  the  terminal  reserve  of  the  tenth  year  on  an  ordinary  life  policy  of 
$3,000  taken  at  age  20. 

2.  Find  the  terminal  reserve  of  the  twentieth  year  on  an  ordinary  life  policy 
of  $5,000  taken  at  age  30. 


476  VALUATION  OF  LIFE  INSURANCE  POLICIES 

Retrospective  method.  Under  this  method  the  policy  value  is 
found  by  deducting  the  accumulated  losses  from  the  accumulated 
premiums.  The  formula  is : 

v  -  MI  Nx  "  Nz+n  -  Mx  ""  Mx+H 
n   *       Nx'       DI+n  Dx+n 

Problems 

1.  Check  the  answer  to  the  example,  using  the  retrospective  formula. 

2.  Find  the  surrender  value  in  Problems  1  and  2  on  page  475  by  the  retro- 
spective method. 

Transformation.  In  computing  the  terminal  reserve,  the  for- 
mula used  was 

and  the  net  single  premium  at  age  x,  denoted  by  Ax,  was  computed 
by  the  formula : 

A,  =  Px(l  -f  O 
Then,  substituting  for  Ax,  we  have: 

which  represents  the  policy  value  or  reserve,  for  the  policy  value 
is  equal  to  the  present  value  of  the  difference  between  the  net 
premiums  for  age  x  +  n  and  age  x  for  the  remainder  of  life. 

To  express  the  value  of  the  reserve  in  terms  of  annuities,  take 
the  formula: 

which  denotes  the  present  value  of  1  payable  at  the  end  of  the 
year  in  which  a  person  dies  (d  being  the  value,  at  the  beginning  of 
the  year,  of  the  interest  for  each  year  on  1),  and 

p*  =  nb;-« 

which  denotes  the  annual  premium  Px,  for  an  ordinary  life  policy 
expressed  in  terms  of  annuity  values.  Then  substitute  the  values 
of  Ax  and  Px  in  the  formula: 

nVx  =  Ax+n  -  P,(l  -f-  ax+n) 
Following  the  algebraic  processes  of  simplifying,  the  result  is 

nV,  =  1  -  \+,aw 


VALUATION  OF  LIFE  INSURANCE  POLICIES  477 

Problems 

1.  With  the  aid  of  the  table  of  life  annuities,  calculate  the  terminal  reserve 
of  the  twentieth  policy  year  on  an  ordinary  life  policy  for  SI,  000  issued  at  age  30. 

2.  Find  the  terminal  reserve  the  tenth  year  on  an  ordinary  life  policy  of 
$5,000  issued  at  age  25. 

Reserve   valuation  for  limited  payment  life   insurance.     In 

determining  the  reserve  valuation  for  such  policies  as  10-payment 
life,  20-payment  life,  and  so  forth,  the  following  principle  is  funda- 
mental :  the  terminal  reserve  of  the  nth  policy  year  equals  the  net 
single  premium  at  the  attained  age  of  the  insured  minus  the  present 
value  of  the  future  net  premiums.  Using  m  to  denote  the  number 
of  annual  payments,  we  have 

n:mVx  =  Ax},,  -  J\(]  +  rtxfnro_n_,i) 

but  only  when  n  is  less  than  m.  Where  n  is  equal  to  or  greater 
than  m,  the  terminal  reserve  is  simply  equal  to  the  net  single 
pi  emium. 

For  endowment  insurance,  the  formula  is 

nVKxr\     =    AKx+n    r_n]    -  PEzr]  (1    +   <lf  +  n:r--j=i]) 

for  an  r-year  endowment. 

Problems 

1.  Find  the  terminal  reserve  of  the  fifteenth  policy  year  for  a  $2000  20-year 
pay-life  policy  issued  at  age  30. 

2.  Find  the  terminal  reserve  of  the  fifteenth  policy  year  for  a  $3000  20-year 
endowment  issued  at  age  45. 

Preliminary  term  valuation.  Initial  expenses  of  securing  a 
policy,  such  as  agents'  commissions,  medical  and  inspection  fees, 
arid  other  expenses,  make  it  practically  impossible  to  provide  any 
reserve  out  of  the  first  year's  premium.  Under  the  net  level  pre- 
mium method,  the  loading  is  the  same  each  year;  therefore, 
expenses  exceed  income,  and  the  deficit  must  be  met  from  general 
funds.  To  avoid  this,  the  preliminary  term  valuation  is  used, 
whereby  the  first  year's  premium  becomes  available  for  expenses 
and  losses,  the  policy  is  renewed  at  the  beginning  of  the  second 
year,  and  policy  values  begin  with  that  year.  Therefore,  the 
first  year  is  simply  term  insurance. 

Under  this  plan,  assume  an  ordinary  life  policy  of  $1,000  at 
age  20,  the  premium  being  $16.50. 

The  net  premium  for  the  first  year  would  be  : 

*         351'07 


Z)20  "  46552 
$1,000  X  .00754  =  S7.54 
$16.50  -  $7.54  =  $8.06,  the  loading  for  the  first  year. 


478  VALUATION  OF  LIFE  INSURANCE  POLICIES 

For  the  second  and  subsequent  years,  the  net  premium  will  he 
the  level  net  premium  based  upon  age  21 : 

_  Jf»  _  12916.3 
^2l  "  IvTt  "  937843 
$1,000  X  .01377  =  $13.77 
$16.50  -  13.77  =  $2.73,  the  loading  for  each  year  after  the  first. 

Problems 

1.  Find  the  loading  for  the  first  and  second  years  on  an  ordinary  life  policy 
of  $2000  at  age  30,  with  an  annual  premium  of  $42.60,  using  the  preliminary 
term  evaluation. 

2.  Using  the  preliminary  term  evaluation,  find  the  loading  for  the  first  and 
second  years  on  an  ordinary  life  policy  of  $1000  at  age  40,  the  premium  being 
$28.80,  * 


APPENDIXES 


APPENDIX  PAOfe/ 

I.     Practical  Business  Measurements 481 

II.  Tables  of  Weights,  Measures,  and  Values 489 

III.  Tables: 

1.  Logarithms  ....  ...  ...  497 

2.  Compound  amount  of  1 :  ,s  =  (1  +  z)w     512 

3.  Present  value  of  1:  vn  =     ^         -  -  or  vn  =  (I  +  f)"n «r)20 

(1  +  i)n  -  1 

4.  Amount  of  annuity  of  1  :  .<?      =  .  527 

i 

5.  Present  value  of  annuity  of  1 :  a  ,    —  .  530 

alt  l 

1 

6.  Rent  of  present  value  of  annuity  of  1 :  —  =  .  .  .    533 

«»'*  i 

7.  Amorican  Experience  Table  of  Mortality.  .  .    530 

8.  Commutation  Columns,  3l%       538 


479 


APPENDIX  I 

Practical  Business  Measurements 

Practical  business  measurements.  The  measurements  discussed  in 
this  section  are  those  of  practical  use,  and  include  measurements  of  or 
pertaining  to  the  following:  angles;  surfaces;  triangles,  rectangles,  and 
other  polygons;  circles,  including  circumference,  radius,  diameter,  and 
area;  area  of  irregular  figures;  and  solids,  such  as  the  sphere,  cone,  cylin- 
der, cube,  and  prismatoid. 

Rectilinear  figures.  An  angle  is  the  difference  in  the  direction  of  two 
lines  proceeding  from  a  common  point  called  the  vertex. 

A  right  angle  is  an  angle  formed  by  two  lines  perpendicular  to  each 
other,  and  is  an  angle  of  90°. 

An  angle  that  is  less  than  a  right  angle  is  an  acute  angle,  and  one  that 
is  greater  is  an  obtuse  angle.  Acute  and  obtuse  angles  are  also  called 
oblique  angles. 

A  surface  has  two  dimensions — length  and  breadth. 

A  plane,  or  a  plane  surface,  is  a  level  surface.  A  straight  edge  will  fit 
on  it  in  any  position. 

A  plane  figure  is  a  figure  all  of  whose  points  lie  in  the  same  plane. 

A  quadrilateral  is  a  plane  figure  bounded  by  four  straight  lines. 

Quadrilaterals  are  of  three  classes  or  kinds:  the  trapezium,  which  has 
four  unequal  sides,  no  two  of  which  are  parallel ;  the  trapezoid,  which  has 
two  and  only  two  sides  parallel;  and  the  parallelogram,  which  has  two 
pairs  of  parallel  sides. 

The  altitude  of  a  quadrilateral  having  two  parallel  sides  is  the  perpen- 
dicular distance  between  those  sides. 

The  diagonal  of  a  quadrilateral  is  the  straight  line  connecting  two  of 
its  opposite  vertices. 

Parallelograms  are  of  three  classes  or  kinds:  the  rhomboid,  which  has 
one  pair  of  parallel  sides  greater  in  length  than  the  other  pair,  and  no 
right  angles;  the  rhombus,  all  of  whose  four  sides  are  equal;  and  the 
rectangle,  whose  angles  are  all  right  angles. 

A  square  is  a  rectangle  having  four  equal  sides.  It  is  also  a  rhombus 
whose  four  angles  are  each  90°. 

A  triangle  is  a  plane  figure  bounded  by  three  straight  lines.  If  the 
three  sides  are  of  equal  length,  the  triangle  is  called  equilateral.  If  two 
sides  are  of  equal  length,  it  is  called  isosceles.  If  the  three  sides  are  of 
different  lengths,  it  is  called  scalene.  If  one  of  the  three  angles  is  a  right 
angle,  the  triangle  is  called  a  right  triangle,  and  the  side  opposite  the 
right  angle  is  called  the  hypotenuse. 

Plane  figures  may  be  regular  or  irregular.  A  regular  plane  figure  has 
all  its  sides  and  all  its  angles  equal  The  smallest  regular  plane  figure  is 

481 


482 


APPENDIXES 


an  equilateral  triangle;  the  next,  a  square;  the  next,  a  pentagon;  and  so 
on.  Each  figure  derives  its  name  from  the  number  of  its  angles  or  sides- 
hexagon,  heptagon,  octagon,  nonagon,  decagon,  etc. 

The  perimeter  of  a  plane  figure  is  the  sum  of  the  lengths  of  its  sides 

The  apothem  of  a  polygon  is  a  perpendicular  line  drawn  from  the 
center  of  the  figure  to  the  middle  of  a  side,  the  center  being  the  point 
within  the  figure  which  is  equally  distant  from  the  middle  points  of  all 
the  sides. 

The  altitude  of  a  plane  figure  is  the  perpendicular  distance  from  the 
highest  point  above  the  base  to  the  base  or  to  the  base  extended. 

Circles.  A  circle  is  a  plane  figure  bounded  by  a  curved  line,  called 
the  circumference,  every  point  of  which  is  equally  distant  from  a  point 
within  called  the  center. 

The  diameter  of  a  circle  is  a  straight  line  drawn  through  the  center 
and  terminated  by  the  circumference. 

The  radius  of  a  circle  is  a  straight  line  drawn  from  the  center  to  the 
circumference,  and  is  equal  to  one-half  the  diameter. 


Figure  16. 

An  arc  of  a  circle  is  any  portion  of  the  circumference. 

A  sector  of  a  circle  is  bounded  by  two  radii  and  the  intercepted  arc. 

A  chord  is  the  straight  line  joining  the  extremities  of  an  arc. 

A  segment  is  bounded  by  an  arc  and  its  chord. 

A  zone  is  a  portion  of  a  circle  bounded  by  two  parallel  chords. 

A  tangent  to  a  circle  is  a  straight  line  having  only  one  point  in  common 
with  the  curve;  it  simply  touches  the  circle.  A  secant  enters  the  figure 
from  without. 

An  ellipse  is  a  plane  figure  bounded  by  an  oval  curved  line,  and  has  a 
long  and  a  short  diameter  or  axis. 

Measurement  of  triangles.  It  is  proved  in  geometry  that  the  square 
erected  on  the  hypotenuse  of  a  right  triangle  is  equal  to  the  sum  of  the 
squares  erected  on  the  other  two  sides.  This  may  be  illustrated  as  in 
Figure  16. 


PRACTICAL  BUSINESS  MEASUREMENTS 


483 


To  find  the  length  of  the  hypotenuse  of  a  right  triangle,  the  lengths  of 
the  other  two  sides  being  given,  add  the  squares  of  the  sides  forming  the 
right  angle,  extract  the  square  root  of  the  sum,  and  the  result  will  be  the 
length  of  the  hypotenuse. 

To  find  the  length  of  either  of  the  two  sides  other  than  the  hypotenuse, 
from  the  square  of  the  hypotenuse  subtract  the  square  of  the  given  side, 
extract  the  square  root  of  the  remainder,  and  the  result  will  be  the  length 
of  the  third  side. 

To  find  the  area  of  a  triangle,  the  base  and  the  altitude  being  given, 
multiply  the  base  by  one-half  the  altitude. 

To  find  the  area  of  a  triangle  when  the  lengths  of  the  three  sides  are 
given,  from  half  the  sum  of  the  three  sides,  subtract  the  length  of  each 
side  separately.  Find  the  continued  product  of  the  three  remainders  and 
the  half  sum.  The  square  root  of  the  result  will  be  the  area. 

Measurement  of  rectangles.  The  area  of  a  square  or  of  a  rectangle 
is  the  product  of  the  length  and  the  breadth. 

Either  dimension  of  a  rectangle  may  be  found  by  dividing  the  area  by 
the  given  dimension. 

Measurement  of  quadrilaterals.  The  area  of  a  trapezium  may  bo 
found  by  multiplying  one-half  the  sum  of  the  altitudes  by  the  diagonal. 

The  area  of  a  trapezoid  may  be  found  by  multiplying  the  sum  of  the 
parallel  sides  by  one-half  the  altitude. 


Figure  17.     Trapezium. 


Figure  18.     Trapezoid. 


The  area  of  a  parallelogram  may  be  found  by  multiplying  the  base 
by  the  altitude. 


Figure  19.     Parallelogram. 

The  area  of  polygons  having  equal  sides  and  equal  angles  may  be 
found  by  multiplying  the  square  by  one  of  the  equal  sides  by : 

.433,  if  the  figure  is  a  triangle 
1.7205,  if  the  figure  is  a  pentagon 
2.5981,  if  the  figure  is  a  hexagon 
4.8284,  if  the  figure  is  an  octagon 


484  APPENDIXES 

Measurement  of  circles.  It  is  shown  in  geometry  that  the  circum- 
ference of  a  circle  bears  a  fixed  ratio  to  its  diameter.  This  constant  ratio 
is  represented  by  TT  I'pmnoiim-nl  "pi")>  and  is  3.1416. 

From  this  relation  the  following  principles  are  derived: 

The  circumference  =  the  diameter  X  3.1416 
The  diameter  =  the  circumference  -r  3.1416 
The  area  =  the  circumference  X  half  the  radius 

The  area  of  a  circle  is  found  by  considering  the  surface  to  be  composed 
of  an  infinite  number  of  isosceles  triangles,  the  bases  of  which,  taken 
together,  equal  the  perimeter  of  the  circle.  The  common  altitude  of 
these  triangles  constantly  approaches  the  radius  of  the  circle,  and  will 
reach  that  length  when  the  perimeter  consists  of  very  short  straight  lines; 
hence,  perimeter  (circumference)  X  i  radius  =  area. 

To  find  the  circumference  of  a  circle,  multiply  the  diameter  by  3.1416, 
or  divide  the  area  by  one-fourth  of  the  diameter. 

To  find  the  diameter  of  a  circle,  divide  the  circumference  by  3.1416, 
or  divide  the  area  by  .7854  and  extract  the  square  root  of  the  result. 

To  find  the  area  of  a  circle,  multiply  the  circumference  by  one-half 
the  radius;  or,  multiply  the  diameter  by  one-fourth  of  the  circumference; 
or,  multiply  the  square  of  the  diameter  by  .7854;  or,  multiply  the  square 
of  the  radius  by  3.1416. 

To  find  the  area  of  an  ellipse,  multiply  the  major  axis  by  the  minor 
axis,  and  that  result  by  .7854. 

To  find  the  area  of  a  sector  of  a  circle,  multiply  one-half  the  length  of 
the  arc  by  the  radius;  or,  take  the  same  part  of  the  area  of  the  circle  as  the 
number  of  degrees  in  the  arc  is  of  360°. 

To  find  the  area  of  a  segment  which  is  less  than  a  semi-circle,  from 
the  area  of  a  corresponding  sector,  subtract  the  area  of  the  triangle 
formed  by  the  chord  and  radii;  to  find  the  area  of  a  segment  which  is 
greater  than  a  semi-circle,  add  the  area  of  the  triangle  formed  by  the 
chord  and  radii  to  the  area  of  a  corresponding  sector. 

To  find  the  area  of  a  zone,  from  the  area  of  the  circle  subtract  the 
areas  of  the  segments  not  included  in  the  zone. 

Problems 

1.  Harry  and  George  start  from  the  same  point,  Harry  going  4  miles  due 
west,  and  George  3  miles  due  north;  how  far  apart  are  they? 

2.  The  base  of  a  triangle  is  12  inches,  and  the  altitude  is  8  inches.     What 
is  the  area  of  the  triangle? 

3.  The  three  sides  of  a  triangular  plot  of  land  are  100  feet,  130  feet,  and 
150  feet.     What  is  the  area  of  the  plot? 

4.  A  rectangular  piece  of  land  is  40  rods  long  and  20  rods  wide.     What  is 
the  area  in  square  rods? 

6.  Find  the  cost  of  fencing  a  field  40  rods  wide  and  55  rods  long,  if  the 
fencing  costs  $2.25  a  rod. 


PRACTICAL  BUSINESS  MEASUREMENTS  485 

6.  A  field  is  in  the  form  of  a  trapezium,  having  a  diagonal  of  90  rods,  and 
altitudes  of  25  rods  and  40  rods.     What  is  the  area  in  square  rods? 

7.  One  of  the  parallel  sides  of  a  garden  is  60  yards  long,  and  the  other  is 
SO  yards  long.     The  garden  is  52  yards  wide.     How  many  square  yards  does  it 
contain? 

8.  Find  the  area  of  a  parallelogram  whose  base  is  10  feet,  and  whose  alti- 
tude is  4  feet. 

9.  The  side  of  a  hexagonal  building  is  20  feet.     What  is  the  floor  area? 

10.  A  cylindrical  tank  is  12  feet  in  diameter.     What  must  be  the  length  of 
a  piece  of  strap  iron  which  is  to  be  used  to  make  a  band  around  the  tank,  if  1  foot 
is  allowed  for  overlapping? 

11.  The  circumference  of  a  circle  is  44  feet.     What  is  its  diameter? 

12.  Find  the  area  of  the  circle  in  problem  11. 

13.  The  diameters  of  an  ellipse  are  00  feet  and  40  feet.     What  is  the*  area? 

14.  How  much  belting  will  be  required  to  make  a  belt  to  run  over  two  pulleys, 
each  30  inches  in  diameter,  if  the  distance  between  the  centers  of  the  pulleys 
is  18  feet? 

15.  If  there  is  a  steam  pressure  of  90  pounds  to  the  square  inch,  what  is  the 
pressure  on  a  9-inch  piston? 

16.  If  pieces  of  sod  are  12  inches  by  14  inches,  how  many  pieces  will  be 
required  to  sod  a  lawn  24  feet  wide  and  2S  feet  long? 

17.  Find  the  cost  of  painting  the  four  side  walls  of  a  room  14  feet  long, 
10  feet  6  inches  wide,  and  8  feet  high,  at  18  cents  a  square  yard,  no  allowance 
being  made  for  openings. 

18.  A  circular  walk  5  feet  wide  is  laid  around  a  plot  20  feet  in  diameter. 
What  is  the  cost  of  the  walk  at  $2.50  a  square  foot? 

19.  Find  the  number  of  paving  blocks  required  to  pave  a  street  one  mile 
long  arid  35  feet  wide,  if  the  blocks  are  one  foot  long  and  five  inches  wide. 

20.  What  part  of  an  acre  is  a  plot  of  land  78  feet  long  and  36  feet  wide? 

Solids.  A  solid  is  a  magnitude  which  has  length,  breadth,  and  thick- 
ness. Solids  include  the  prism,  the  cylinder,  the  pyramid,  the  cone,  the 
polyhedron,  and  the  sphere. 

A  prism  is  a  solid  whose  upper  and  lower  bases  are  equal  and  parallel 
polygons,  and  whose  sides,  or  lateral  faces,  are  parallelograms. 

A  rectangular  sold  is  bounded  by  six  rectangular  surfaces. 

A  cube  is  a  rectangular  solid  having  six  square  faces. 

A  triangular  prism  is  a  prism  whose  bases  are  triangles. 

A  cylinder  is  a  prism  having  an  infinite  number  of  faces  or  sides;  the 
two  bases  are  equal  parallel  circles. 

A  pyramid  is  a  solid  having  for  its  base  a  polygon,  and  for  its  other 
faces  three  or  more  triangles  which  terminate  in  a  common  point  called 
the  vertex  or  apex. 

A  cone  is  a  pyramid  having  an  infinite  number  of  faces;  or,  it  is  a  solid 
whose  base  is  a  circle,  and  whose  convex  surface  tapers  uniformly  to  a 
point  called  the  apeY. 


486  APPENDIXES 

A  polyhedron  is  a  solid  bounded  by  four  or  more  faces. 

A  sphere  is  a  solid  bounded  by  a  curved  surface,  every  point  of  which 
is  equally  distant  from  a  point  within,  called  the  center. 

The  frustum  of  a  pyramid  or  of  a  cone  is  the  solid  which  remains  when 
a  portion  which  includes  the  apex  is  cut  off  by  a  plane  parallel  to  the  base. 

The  axis  of  a  pyramid  or  of  a  cone  is  a  straight  line  that  joins  the  apex 
to  the  center  of  the  base. 

The  altitude  of  a  pyramid  or  of  a  cone  is  the  perpendicular  height 
from  its  apex  to  its  base. 

The  slant  height  of  a  pyramid  is  the  distance  from  the  apex  to  the 
midpoint  of  one  side  of  its  base. 

The  slant  height  of  a  cone  is  the  distance  from  its  apex  to  the  circum- 
ference of  its  base. 

The  diameter  of  a  sphere  is  a  straight  line  drawn  through  its  center 
and  terminated  at  both  ends  by  the  surface. 

The  radius  of  a  sphere  is  one-half  of  its  diameter. 

The  circumference  of  a  sphere  is  the  greatest  distance  around  the 
sphere. 

A  hemisphere  is  one-half  of  a  sphere. 

Measurement  of  solids.  To  find  the  contents  of  a  prism  or  of  a 
cylinder  when  the  perimeter  of  the  base  and  the  altitude  are  given, 
multiply  the  area  of  the  base  by  the  altitude. 

To  find  the  convex  surface  of  a  prism  or  of  a  cylinder,  multiply  the 
perimeter  of  the  base  by  the  height. 

To  find  the  entire  surface  of  a  prism  or  of  a  cylinder,  add  the  area  of 
the  bases  to  the  area  of  the  convex  surface. 

To  find  the  convex  surface  of  a  cone,  multiply  the  circumference  of  the 
base  by  one-half  the  slant  height. 

To  find  the  entire  surface  of  a  cone,  add  the  area  of  the  base  to  the 
area  of  the  convex  surface. 

The  slant  height  of  a  pyramid  or  of  a  cone  may  be  found  by  adding  the 
square  of  the  altitude  to  the  square  of  the  radius  of  the  base,  and  extract- 
ing the  square  root  of  the  sum. 

To  find  the  volume  of  a  pyramid  or  of  a  cone,  multiply  the  area  of  the 
base  by  one-third  the  altitude. 

The  volume  of  a  pyramid  is  one-third  as  much  as  the  volume  of  a 
prism  that  has  the  same  base  and  altitude. 

The  volume  of  a  cone  is  one-third  as  much  as  the  volume  of  a  cylinder 
that  has  the  same  base  and  altitude. 

To  find  the  convex  surface  of  a  frustum  of  a  pyramid  or  of  a  cone, 
multiply  one-half  the  sum  of  the  perimeters  of  the  two  bases  by  the  slant 
height. 

To  find  the  entire  surface  of  a  frustum  of  a  pyramid  or  of  a  cone,  add 
the  area  of  the  two  bases  to  the  area  of  the  convex  surface. 

To  find  the  volume  of  a  frustum  of  a  pyramid  or  of  a  cone,  find  the 
product  of  the  areas  of  the  two  bases,  and  extract  the  square  root  thereof 
This  result  is  the  area  of  a  base  which  is  a  mean  base  between  the  other 
two  Add  the  three  areas,  and  multiply  by  one-third  the  altitude. 


PRACTICAL  BUSINESS  MEASUREMENTS  487 

To  find  the  surface  of  a  sphere,  find  the  area  of  a  great  circle  of  the 
sphere,  and  multiply  this  area  by  4. 

To  find  the  volume  of  a  sphere,  multiply  the  convex  surface  by  one- 
third  the  radius. 

The  volume  of  a  spherical  shell  (a  hollow  sphere)  is  equal  to  the 
volume  of  the  outside  sphere  minus  the  volume  of  the  inside  sphere. 

Problems 

1.  A  cylindrical  tank  is  12  foot  in  diameter.     If  it  is  filled  with  water  to  a, 
depth  of  6  feet,  what  is  the  weight  of  the  water?     (1  cu.  ft.  of  water  weighs 
62.5  pounds.) 

2.  How  many  square  yards  of  sheet  metal  will  be  required  for  a  smoke- 
stack 2  feet  in  diameter  and  12  feet  in  height,  if  I  inch  is  allowed  for  overlapping? 

3.  Find  the  cost  of  painting  the  entire  surface  of  a  cylindrical  tank  10  feet 
in  diameter  and  20  icct  long,  at  10^  per  square  foot. 

4.  The  boundary  lines  of  the  Fort  Pombina  Airport  are  marked  by  cone- 
shaped  markers;  each  marker  is  3  feet  in  diameter  and  has  a  slant  height  of 
3  feet.     If  there  are  120  of  these  markers,  and  1   inch  \\as  allowed  for  over- 
lapping, how  many  square  feet  of  sheet  metal  were  required  for  their  construction? 

6.  If  a  freight  car  is  36  foot  long,  and  S  foot  6  inches  \\ido,  inside  measure, 
how  many  hi  she-Is  of  wheat  will  it  contain  when  filled  to  a  depth  of  5  feet? 
(A  cubic  foot  is  approximately  .8  of  a  bushel.) 

6.  How  many  tons  of  coal  will  fill  a  bin  20  feet,  by  16  feet,  by  8  feet,  if  there 
are  80  cubic  feet  to  a  ton? 

7.  The  measurements  of  a  railroad  embankment  are:  length,  400  feet;  height, 
10  feet;  width  of  base,  14  feet;  and  width  of  top,  8  feet.     How  many  cubic  yards 
of  earth  will  be  required? 

8.  The  measurements  of  a  funnel  are  as  follows:  larger  diameter,  12  inches; 
•»rnaller  diameter,  1  inch;  and  slant  height,  18  inches.     How  many  square  inches 
of  sheet  metal  will  be  required? 

9.  One  of  the  units  of  a  grain  elevator  is  a  concrete  cylinder  20  feet  in 
diameter  and  50  feet  in  height.     The  bottom  is  cone-shaped  to  facilitate  tho 
drawing  off  of  the  grain.     The  depth  of  this  cone  is  5  feet.     If  the  wheat  in  this 
unit  is  leveled  off  at  the  30-foot  mark,  how  many  bushels  are  in  the  unit,  assuming 
that  a  cubic  foot  is  approximately  .8  of  a  bushel? 

10.  A  bucket  is  16  inches  wide  at  the  top,  and  10  inches  wide  at  the  bottom. 
The  depth  is  12  inches.     How  many  gallons  of  water  will  the  bucket  hold? 
(231  cu.  in.  =  1  gal.) 

11.  What  number  of  square  feet  of  sheet  metal  will  be  required  to  make 
XOO  pails,  each  10  inches  dee]),  8  inches  in  diameter  at  the  bottom,  and  11  inches 
in  diameter  at  the  top?     The  allowance  for  seams  and  for  waste  in  cutting  is  10%. 

12.  How  many  tiles  1  inch  square  will  be  required  for  the  surface  of  a  tiled 
dome  in  the  form  of  a  hemispherical  surface,  if  the  diameter  of  the  dome  is 
24  feet? 

13.  The  top  of  a  vat  is  9  feet  square,  and  the  base  is  8  feet  square.     If  the 
slant  height  is  10  feet,  what  is  the  capacity  of  the  vat  in  cubic  feet? 


488  APPENDIXES 

14.  How  many  cubic  feet  are  there  in  a  spherical  body  whose  diameter  is 
10  feet? 

16.  The  base  of  a  church  steeple  is  in  the  form  of  an  octagon  measuring 
6  feet  on  each  side.  The  slant  height  of  the  steeple  is  80  feet.  What  will  be  the 
cost  of  painting  this  steeple  at  50^  per  square  yard? 

16.  A  tank  is  8  feet  long,  6  feet  wide,  and  3  feet  deep.     If  a  cubic  foot  of 
water  weighs  62.5  pounds,  what  is  the  weight  of  water  in  this  tank  if  it  is  two- 
thirds  full? 

17.  If  38  cubic  feet  of  coal  weigh  a  ton,  how  many  tons  can  bo  put  into  a  bin 
10  feet  long  and  8  feet  wide,  if  the  coal  is  leveled  off  at  an  average  depth  of 
5  feet? 

18.  Find  the  number  of  square  feet  of  sheet  metal  required  to  make  12  gross 
of  pails,  each  14  inches  deep,  8  inches  in  diameter  at  the  bottom,  and  11  inches 
in  diameter  at  the  top,  not  allowing  for  seams  or  waste  in  cutting. 

19.  The  diagram  is  that  of  a  cross  section  of  a  concrete  retaining  wall  150  feet 
long.     Find  the  number  of  cubic  yards  of  material  necessary  to  construct  such 
a  wall. 

2' 


20.  If  200  gallons  of  water  flow  through  a  pipe  2  inches  in  diameter  in  4  hours, 
how  much  water  will  flow  through  a  pipe  4  inches  in  diameter  in  the  same  time? 
HINT:  The  amounts  of  the  liquids  are  to  each  other  as  the  squares  of  the  like 
dimensions. 

21.  A  cylindrical  hot  water  tank  is  5  feet  high  and  11  inches  in  diameter. 
How  many  gallons  will  it  contain? 

22.  A  corn  crib  32  feet  by  10  feet  by  S  feet  is  filled  \\ith  oar  corn.     How 
many  bushels  will  it  contain  if  one  bushel  equals  1^  cubic  foot? 

23.  A  barn  loft  is  36  feet  by  24  feet  by  8  foet.     How  many  tons  of  hay  will 
it  hold  if  it  is  to  be  filled  with:  (a)  clover  hay  weighing  one  ton  for  600  cubic  foot; 
(6)  timothy  hay  weighing  one  ton  for  500  cubic  feet? 

24.  If  a  heaped  bushel  equals  1^-  cubic  feet,  how  many  bushels  of  potatoes 
may  be  stored  in  a  bin  that  is  12  feet  by  8  feet  by  6  feet? 

25.  If  a  cubic  foot  of  steel  weighs  484  pounds,  what  is  the  weight  of  a  hollow 
steel  cylinder  whose  length  is  10  feet  and  the  radii  of  whose  outer  and  innei 
circles  are  3  feet  and  2^  feet,  respectively? 


APPENDIX  II 


Tables  of  Weights,  Measures,  and  Values 


Long  Measure 


U.  S.  and  British  Standard 


12  inches 
3  feet 

5-gr  yards,  or  163-  feet 
320  rods,  or  5,280  feet 
,760  yards 
40  rods. 
8  furlongs 
3  miles 


foot 

yard 

rod 

mile 

mile 

furlong 

statute  mile 

league 


Metric  System 

10  millimeters. .  .       I  centimeter 

10  centimeters  .  1  decimeter 

10  decimeters  1  meter 

10  meters .      1  dekameter 

10  dekameters  .      1  hektometer 

10  hekto meters  .      1  kilometer 

10  kilometers     .  .      1  myriameter 


Comparisons  of  Long  Pleasures 

1  inch                         25.4001  millimeters     1  centimeter       .  .3937  inch 

1  foot 304S01  meter              1  meter.     ..  39.37  inches 

1  yard 914402  meter               I  meter. .  3.28083  feet 

1  rod 5.029  meters                1  meter.     .  .    1.09361 1  yards 

1  mile 1.00935  kilometers      1  kilometer  .  .    .02137  mile 


Square  Measure 

( r.  8.  and  British  Standard 


1 44  square 

9  square 

30J-  square 

272^  square 

40  square 

4  roods 

160  square 

640  acres. . 

43,560  square 

4,840  square 


inches 

feet 

yards 

feet 

rods 

rods 

feet. . . . 
yards   . 


100  square  millimeters 
100  square  centimeters 
100  square  decimeters 
100  square  meters  . 
100  square  dekameters 
100  square  hekto  meters 


Metric  System 


1  square  foot 
1  square  yard 

square  rod 

square  rod 

rood 

acre 

acre 

square  mile 

acre 

acre 


1  square  centimeter 
1  square  decimeter 
1  square  meter 
1  square  dekameter 
1  square  hektometer 
1  square  kilometer 


100  square  kilometers 1  square  myriameter 

489 


490 


APPENDIXES 


Comparisons  of  Square  Measures 

1  sq.  in 6.452  sq.  cm.     1  sq.  mm 00155  sq.  in. 

1  sq.  ft 0929  sq.  m.       1  sq.  cm 155  sq.  in. 

1  sq.  yd 8361  sq.  m.       1  sq.  m 10.764  sq.  ft. 

1  sq.  rd 25.293  sq.  m.     1  sq.  m 1.196  sq.  yds. 

1  sq.  mi 2.59  sq.  km.      1  sq.  km 3861  sq.  mi. 

1  sq.  km  247.11  acres 

1  sq.  Dm.,  or  1  are 1,076.41  sq.  ft. 

100  ares  =  1  hektare 2.4711  acres 

Solid  or  Cubic  Measure  (Volume) 

U.  S.  and  British  Standard  Metric  System 

1 ,728  cubic  inches  1  cubic  foot  1 ,000  cubic  millimeters ...  1  cu.  cm. 

27  cubic  feet.  .    1  cubic  yard  1,000  cubic  centimeters. .  .  1  cu.  dm. 

128  cubic  feet.  .       cord  of  wood  1,000  cubic  decimeters. ...  1  cu.  m. 

24f  cubic  feet    .       perch  of  stone  1,000  cubic  meters    1  cu.  Dm. 

2,150.42  cubic  inches      standard  bushel  1,000  cubic  dekameters.  .  .  1  cu.  Hm. 

231  cubic  inches      standard  gallon  1,000  cubic  hektometers.  .  1  cu.  Km. 

40  cubic  feet    .       ton  (shipping)  1,000  cubic  kilometers   ...  1  cu.  Mm. 

Comparisons  of  Solid  or  Cubic  Measures  (Volume) 

1  cu.  in 16.3872  cu.  cm.     1  cu.  cm 061  cu.  in. 

I  cu.  ft 02832  cu.  in.         1  cu.  m 35.314  cu.  ft. 

1  cu.  yd 7646  cu.  in.          1  cu.  m 1.3079  cu.  yds. 

1  cu.  dm.  =  1  liter 61.023  cu.  in. 

1  liter 1 .05671  liquid  quarts 

1  liter 9081  dry  quart 

1  hectoliter  or  decistore. . .   3.5314  cu.  ft.  or 

2.8375  U.  S.  Bushels 
1  stere,  kiloliter,  or  cu.  m  .    1.3079  cu.  yds.  or 

28.37  U.  S.  Bushels 

Liquid  Measure  (Capacity) 

U.  S.  and  British  Standard  Metric  System 

4  gills 1  pint  10  milliliters 1  centiliter 

2  pints 1  quart  10  centiliters 1  deciliter 

4  quarts 1  gallon  10  deciliters  ...        .1  liter 

31^  gallons 1  barrel  10  liters     1  dekaliter 

2  barrels 1  hogshead  10  dekaliters 1  hektolitei 

1  U.  S.  Gallon. ...  231  cubic  inches  10  hektoliters   1  kiloliter 

1  British  Imperial  10  kiloliters 1  myrialitei 

Gallon 277.274  cubic  inches 

7.4805  U.  S.  Gallons. . .  1  cubic  foot 

16  fluid  ounces. ...  1  pint 

1  fluid  ounce 1.805  cubic  inches 

1  fluid  ounce 29.59  cubic  centimeters 

1.2  U.  S.  Quarts. ...  1  Imperial  Quart 

1.2  U.  S.  Gallons. .  .  1  Imperial  Gallon 

1  gallon  gasoline . .  6  pounds  (approx.) 

1  gallon  oil 7^-  pounds  (approx.) 

1  gallon  water 8.3  pounds  (approx.) 

1  liter  gasoline.  .  .  1.59  pounds  (approx.) 

1  liter  gasoline .  .  .  0.72  kilograms 


TABLES  OF  WEIGHTS,  MEASURES,  AND  VALUES        491 


Dry  Measure 

U '.  8.  and  British  Standard 

2  pints 1  quart 

8  quarts  .  .  1  peck 

4  pecks  1  bushel 

2,150.42  cubic  inches  I  IT.  S.  Standard  Bushel 

1.2445  cubic  feet  1  U.  S.  Standard  Bushel 

2,218.192  cubic  inches    .  1  British  Imperial  Bushel 

1.2837  cubic  feet  .  1  British  Imperial  Bushel 


Metric  System 


[In  the  Metric  System, 
the  same  table  is  used 
for  both  Liquid  Meas- 
ure and  Dry  Measure.] 


Comparisons  of  Liquid  and  Dry  Measures 


1  liquid  quart. 

1  liquid  gallon 

1  dry  quart 

1  peck 

1  bushel 

1  milliliter 


1  liter  - 


28.317  liters 
4.543  liteis 
3.785  liters. 


decimeter 


.94636    liter 

3.78543  liters 

1.1012    liters 

S.80982  liters 

.35239    hektoliters 

.03381    liquid  ounce,  or 
.2705  apothecaries' 
dram 

61.023  cubic  indies 
.03531  cubic  foot 
.2642  U.  S.  Gallon 
2.202  pounds  of  water 
at  62°  F. 

1  cu.  ft. 

1  British  Imperial  Gal. 

1  U.  S.  Gal. 


Avoirdupois  Measure  (Weight) 

( Used  for  weighing  all  ordinary  substances  except  precious  metals,  jewels,  and 

drugs) 


U.  S.  and  British  Standard 

grains 1  dram 

16  drams 1  ounce 

16  ounces  1  pound 

25  pounds .  1  quarter 

4  quarters 1  hundredweight 

100  pounds 1  hundredweight 

20  hundredweight ...  1  ton 

2,000  pounds 1  short  ton 

2,240  pounds 1  long  ton 


Metric  System 

10  milligrams     .      .  1  centigram 

10  centigrams 1  decigram 

10  decigrams 1  gram 

10  grams 1  dekagram 

1 0  dekagrams I  hektogram 

1 0  hektograms 1  kilogram 

10  kilograms 1  myriagrani 


Troy  Measure  (Weight) 

(Used  for  weighing  gold,  silver,  and  jewels) 


24  grains 

20  pennyweights . 

12  ounces 


1  pennyweight 
I  ounce 
1  pound 


492  APPENDIXES 

Apothecaries'  Measure  (Weight) 

(Used  for  weighing  drugs) 


20  grains         

.    .  .        .          1  scruple 

3  scruples   

...              1  dram 

8  drams     

1  ounce 

12  ounces  

1  pound 

Comparison  of  Avoirdupois  and  Troy  Measures 

1  pound  troy 5,760  grains     1  ounce  troy  4X0  gniins 

1  pound  avoirdupois     .  .  .    7,000  grains     1  ounce  avoirdupois  437i  grains 

I  karat,  or  carat 3.2  troy  grains 

24  karats pure  gold 

Comparison  of  Avoirdupois  and  Troy  Measures  with  Metric  Weights 

grain  0648  gram  (  15.4324  grains 

ounce  (avoir.)..   28.3495  grams  1  gram <  .03527  ounce  (avoir.) 

ounce  (troy)..  .    31.10348  grains  (  .03215  ounce  (troy) 

pound  (avoir.) .    .45359  kilogram  (  2.20462  pounds 

pound  (troy) .  .    .37324  kilogram        i  kilogram <       (avoir.) 

(  2.67923  pounds  (troy) 
.9842    ton     of     2,240 


1  tonne,  or 
metric  ton 


pounds,     or      19.68 
hundredweight 


1.1023   tons   of   2,000 

pounds 

1,000  kilograms  2,204.6  pounds 

1.016  metric  tens,  or 

1,016  kilogrims.  .    1  ton  of  2,240  pounds 

Apothecaries*  Fluid  Measure  (Capacity) 

60  minims 1  fluid  dram 

8  fluid  drams 1  fluid  ounce 

16  fluid  ounces     .  I  pint 

8  pints  .    .  .1  gallon 

Comparisons  (Approximate  Liquid  Measure) 

Apothecaries'                            Common  Metric 

1  minim  1  to  2  drops  0.06  cu.  cm. 
60  minims,  or 

1  fluid  dram  1  teaspoonful  3.75  cu.  cm. 

2  fluid  drams  1  dessertspoonful  7.50  cu.  cm. 
4  fluid  drains  1  tablespoonful  15.00  cu.  cm. 
8  fluid  drams  1  fluid  ounce  28.39  cu.  cm. 
2  fluid  ounces  1  wineglassful  59.20  cu.  cm. 
4  fluid  ounces  1  teacupfui  118.40  cu.  cm. 

16  fluid  ounces  1  pint  473.11  cu.  cm 

NOTE:  Drops  are  not  accurate  measures,  but  for  practical  purposes  it  may 
be  considered  that  one  minim  equals  one  drop  of  watery  liquids  and  fixed  oils, 
but  two  drops  of  volatile  oils  and  alcoholic  liquids,  such  as  tinctures  and  fluid 
extracts. 


TABLES  OF  WEIGHTS,  MEASURES,  AND  VALUES        493 

MISCELLANEOUS  TABLES 

Surveyors1  Long  Measure 

7.92  inches 1  link 

25  links 1  rod 

4  rods,  or  100  links     .          1  chain 

80  chains     1  mile 

Surveyors'  Square  Measure 

625  square  links         1  square  rod 

16  square  rods.  .      .  .  .          1  square  chain 

10  square  chains  1  acre 

640  acres       .  .  1  square  mile 

36  square  miles .  1  township 

Mariners'  Measure 

6  feet 1  fathom 

120  fathoms     1  cable's  length 

7^  cable  lengths  I  mile 

5,280  feet 1  statute  mile 

6,080  feet 1  nautical  mile,  or  British  Admiralty  knot 

50.71H  feet 1  knot 

120  knots,  or 

1.152-|  statute  miles       1  nautical  or  geographical  mile 

3  geographical  miles 1  league 

60  geographical  miles,  or 

69.16  statute  miles 1  degree  of  longitude  on  the  equator,  or 

1  degree  of  meridian 
360  degrees.    . .  .  .        .  .    1  circumference 

NOTE:  A  knot  is  properly  T^TT  °f  a  marine  mile,  but  current  usage  makes  it 
equivalent  to  a  marine  mile.  Hence,  when  the  speed  of  vessels  at  sea  is  being 
measured,  a  knot  is  equal  to  a  nautical  mile,  or  6,086.08  feet,  or  2,028.69  yards. 

Circular  or  Angular  Measure 

60  seconds  (60")  ...    1  minute  (!') 

60  minutes  (600  •    •    1  degree  (L°) 

30  degrees     .    .  .1  sign 

90  degrees 1  right  angle  or  quadrant 

360  degrees  .  .  .  1  circumference 

NOTE:  One  degree  at  the  equator  is  approximately  60  nautical  miles. 

Counting 

12  units  or  things  .        .  1  dozen 

12  dozen,  or  144  units  .  .  1  gross 

12  gross .  1  great  gross 

20  units ...  1  score 

Paper  Measure 

24  sheets 1  quire 

20  quires .    .  ...              .  .    1  ream 

2  reams .  1  bundle 

5  bundles .    .  ....  1  bale 

NOTE:  Although  a  ream  contains  480  sheets,  500  sheets  are  usually  sold  aa 
a  ream. 


494 


APPENDIXES 


Books 

Books  are  printed  on  large  sheets  of  paper,  which  arc  folded  into  leaves 
according  to  the  size  of  the  book.  The  terms  folio,  quarto,  octavo,  and  so  forth, 
as  applied  to  printed  books,  are  based  on  sheets  about  IS  by  24  inches,  or  about 
half  the  size  now  generally  used,  and  indicate  the  number  of  leaves  into  which 
each  sheet  is  folded. 


A  sheet  folded  in    2  leaves  is  called  a  folio 


"  4 
"  X 
"  12 
"  16 
"  24 
"  32 


a  quarto,  or  4to 
an  octavo,  or  Svo 
a  12  mo 
a  16  mo 
a  24  mo 
a  32  mo 


and  makes    4  pages 

a  a  u         a 


32 

4^ 

04 


Sizes  of  Paper 

Book  Papers  Bo  fid,  Ledger,  and  Writing  Papers 

25  X  38                 38  X  50                          14  X  17  20  X  28 

3(4  X  41                 41  X  61                          16  X  21  23  X  31 

32  X  44                 64  X  44                          18  X  23  21  X  32 

33  X  44       66  X  44           17  X  28  16  X  42 
35X45       45X70          19X24  23  X  36 

1 7  X  22  22  X  34 


10  mills  .  . 
10  cents 
10  dimes 
10  dollars 


4  farthings 
12  pence. 
20  shillings. . 


MEASURES  OF  VALUE 
United  States  Money 


English  Money 


A  pound  sterling  =  $4.8665  (normal). 


1  cent 
1  dime 
I  dollar 
1  eagle 


1  penny  (d.) 
1  shilling  (s.) 
1  pound  (£) 


French  Money 


10  milliraes  (m.) . 
10  centimes.  .  .  . 
10  declines 


A  franc  =  $0.193  (normal). 


1  centime  (c.) 
1  decime  (d.) 
1  franc  (fr.) 


Comparison  of  Thermometer  Scales 

To  convert  from  °  F  to  °  C,  subtract  32  from  °  F  and  divide  by  1.8. 
To  convert  from  °  C  to  °  F  multiply  °  C  by  1.8  and  add  32. 


TABLES  OF  WEIGHTS,  MEASURES,  AND  VALUES        495 


Degrees  C 
-100 

-  50 

-  40 

-  20 

-  17  77 

-  15 

-  10 

-  5 
0 
5 

10 

15 

20 

25 

30 

35 

40 

45 

50 

55 

00 

65 

70 

75 

cSO 

85 

90 

95 
100 
150 
190 
200 
300 


Temperature  Equivalents 

Degrees  F 
-148 

-  58 

-  40 

-  4 
0 
5 

14 

23 

32  .... 

41 

50 

59 

OX 

77 

80 

95 
104 
113 
122 
131 
140 
149 
158 
107 
170 
185 
194 
203 
212 
302 
374 
392 
572 


Remarks 


Water  freezes 


Water  boils  at  sea  level 
(With    each    1,000   feet 
altitude,  boiling  point  of 
water    is    reduced    ap- 
proximately 1°  C.) 


Approximate  Weight  of  Substances 


Brick,  pressed,  host  .... 

Brick,  common,  hard 
Brick,  common,  soft 
Coal,  broken  (anlhra  ),  loose 
Coal,  broken  (bitu.),  loose 
Cement,  concrete,  limestone.. 
Cement,  concrete,  cinder 
Cement,  concrete,  stone 
Cement,  concrete,  trap  rock 

Granite     

Hemlock,  dry 

Hickory,  dry 

Ice     ... 

Iron,  cast 

Iron,  wrought.  


Lbs.  Per 
Cu.  Ft. 

150  Lead   

125  Limestone,  marble1,  ordinarily 

100  Limestone,  marble,  piled 

52--50  Masonry,  granite,  dressed 

47-52  Masonry,  sandstone 

148  Sand,  pure  quartz  dry  loose 

112  Ibs.  per  struck  bu 

150  Sand,  angular,  large  and  small. 

155  Sandstone,  dry  for  building 

170  Sandstone,  quarried,  piled    .  .  . 

25  Shales,  red  or  black 

53  Shales,  quarried,  piled   .        ... 

57-0  Slate 

450  Soapstone  or  steatite       

485  Steel,  heaviest,  lowest  in  carbon 


IJbx.  Per 

C?/.  Ft. 

709.6 

168 
96 

165 

145 
112  113 

90-106 

117 

151 
86 

162 
92 

175 

170 

490 


496 


APPENDIXES 


Coal,  anthracite  egg 

Coal,  anthracite  nut. ...          .... 

Coal,  anthracite  stove.  .          

Coal,  bituminous,  111  

Coal,  bit.,  Ind.  block   . 
Coal,  bit.,  Iowa  lump 

Coal,  bit.,  Pittsburgh  

Coal,  bit.,  Pocahontas  egg  and  lump 

Coal,  cannel 

Coke,  loose. . 
Charcoal,  hardwood 

Charcoal,  pine 

Peat,  dry 


WEIGHTS  AND  MEASURES 

Solid  Fuels 

Lbs.  Per  Tons  Per  Cu.  Ft. 

Cu.  Yd.  Cu.  Yd.  Per  Ton 

1514  .76  36 

1536  .77  36 

1521  .76  30 

1275  .64  42 

1161  .58  43 

1 256  63  42 

.        1255  .63  42 

1411  .71  38 

1328  .66  49 

S70-1026  51  60-65 

513  25  19 

486  .24  19 

1269  .63  42 


Anthracite  and  Pocahontas,  approximately  36  cu.  ft.  for  I  ton.  Othei 
bituminous  coal,  approximately  40^  cu.  ft.  for  1  ton.  Coke,  approximately 
60-65  cu.  ft.  for  1  ton. 

Bulk  Materials 

Lbs.  Per      Tons  Per 
Cu.  Yd.       Cu.  Yd. 

Ashes 1080  52 

Asphalt 2700  1  35 

Brick,  so "t  clay .                 .  2718  1   35 

Brick,  hard  clay. . .                                           .  3397  1  69 

Brick,  pressed      3806  1 . 90 

Bluestone .  .  2970  1 .48 

Cement,  Portland .      .  2430  1 .21 

Cinders .  1080  .54 

Clay,  dry .  1701  85 

Clay,  wet 2970  1.48 

Earth,  dry,  loose ....                                    .  1 890  .94 

Earth,  dry,  shaken .  2214  1  . 10 

Earth  and  sand,  dry,  loose.  2700  1   35 

Earth  and  sand,  dry,  rammed                        .  3240  1  .62 

Fire  brick .        .    .  3915  1   95 

Fire  clay .    .                .  3510  1  75 

Gravel,  dry .                        .  2970  148 

Granite 4536  2.26 

Lime,  quick,  shaken .            .    .  1485  .70 

Limestone,  loose ..  2592  1.29 

Marble,  loose 2592  1 . 29 

Mud,  river 2430  1 .21 

Pitch 1863  .93 

Rip-rap,  limestone 2160  1 .08 

Rip-rap,  sandstone 2430  1 . 21 

Rip-rap,  slate 2835  1.41 

Sand,  dry,  loose 2619  1  30 

Sand,  wet 3186  1 .  5t> 

Slag,  screenings 2700  1 . 35 

Street  sweepings 850  . 42 

Tar 1674  .83 


APPENDIX  III-TABLES 


Table  1 
TABLE  OF  LOGARITHMS 


H. 

0 

1 

2 

3 

4 

5 

6 

7 

8 

0 

D 

100 

000000 

000434 

000808 

001301 

001734 

002166 

002598 

003029 

003161 

003891 

432 

1 

4321 

4751 

5181 

5609 

o038 

tvn.t) 

6894 

7321 

7748 

8174 

428 

2 

8600 

9026 

9451 

9876 

010300 

010724 

011147 

011570 

011993 

01241ft 

424 

3 

012S37 

013259 

013680 

014100 

4521 

4940 

5360 

5779 

6197 

6616 

420 

4 

7038 

7451 

7868 

8284 

8700 

9116 

9532 

9917 

020361 

020775 

416 

105 

021189 

021003 

022016 

022428 

022841 

023252 

023664 

024075 

4480 

4896 

412 

6 

5306 

5715 

6125 

6533 

6942 

7350 

7757 

8  KM 

8571 

8978 

408 

7 

9384 

97S9 

03O1  95 

030000 

031004 

031408 

031812 

032216 

032619 

033021 

404 

8 

033424 

033826 

4227 

4628 

5029 

5130 

583O 

6230 

6629 

7028 

400 

9 

7426 

7825 

8223 

8020 

9017 

9414 

9811 

040?07 

040602 

040998 

397 

110 

041393 

041787 

042182 

042576 

042969 

043362 

043755 

044148 

044540 

044932 

893 

1 

r>323 

5714 

6105 

6495 

6885 

7275 

7664 

8053 

8-142 

8830 

890 

2 

9218 

9006 

9993 

0503HO 

050766 

051153 

051538 

051921 

052309 

052694 

386 

3 

O53078 

053463 

053816 

4230 

4613 

4996 

5378 

5760 

6142 

6524 

883 

4 

6905 

7286 

7666 

8016 

8426 

8805 

U185 

9563 

9942 

06032O 

379 

115 

060698 

061075 

061452 

061829 

062200 

062582 

062958 

063333 

063709 

4083 

376 

6 

4458 

4832 

5206 

5580 

5953 

6326 

OCW 

7071 

7443 

7815 

373 

7 

818€ 

85a7 

8928 

9298 

96(58 

070038 

070-1O7 

070776 

071145 

071514 

370 

8 

071882 

072250 

072617 

072985 

073352 

3718 

4085 

4451 

4816 

5182 

366 

9 

5547 

5912 

6276 

66-10 

7001 

7368 

7731 

8094 

8457 

8819 

363 

120 

079181 

079543 

079904 

080206 

080626 

050087 

081347 

081707 

082067 

082426 

360 

1 

082785 

083144 

083503 

3S(,l 

4219 

I57« 

4934 

52()1 

5047 

6004 

357 

2 

6360 

6716 

7071 

7126 

77K1 

8136 

81'H) 

8.S15 

9198 

9552 

355 

3 

91)05 

090258 

05)0611 

090903 

091315 

091067 

092018 

OU237O 

092721 

093071 

352 

4 

093422 

3772 

4122 

4471 

4820 

5169 

5518 

5806 

6215 

6562 

349 

125 

6910 

7257 

7604 

7951 

8298 

8C44 

8990 

9335 

9681 

100026 

846 

6 

100371 

100715 

1010.39 

101103 

101717 

102091 

102m 

102777 

103119 

3462 

343 

7 

3W)4 

4146 

4187 

4828 

5169 

5510 

5851 

6191 

6531 

6871 

341 

8 

7210 

7519 

7888 

8227 

8565 

S908 

9211 

9579 

9910 

110253 

338 

9 

110590 

110926 

111263 

111599 

111934 

112270 

112605 

112940 

113275 

3009 

335 

130 

113943 

114277 

114611 

1149H 

115278 

115611 

115943 

116276 

116608 

116940 

333 

1 

7271 

7603 

793  1 

X2fx> 

8595 

8<)26 

9256 

9580 

9915 

12O245 

330 

2 

120574 

120903 

1212',  1 

1215M 

121  888 

122216 

122511 

122S71 

123198 

3525 

828 

3 

3852 

4178 

4504 

4830 

5156 

5KS1 

5806 

6131 

6156 

6781 

325 

4 

7105 

7429 

7753 

8076 

839« 

8722 

9045 

9368 

9690 

130012 

323 

135 

130334 

130655 

130977 

131298 

131619 

131039 

132260 

132580 

132900 

3219 

321 

6 

3539 

3858 

4177 

411)6 

4814 

5133 

5151 

57  Ml 

6086 

6-103 

318 

7 

6721 

7037 

7354 

7671 

7987 

8303 

W>18 

MM  1 

9219 

9564 

816 

!    & 

9879 

140194 

140508 

140822 

141136 

141150 

1  1  1  763 

142076 

142389 

142702 

314 

9 

143U15 

3327 

3639 

3951 

4263 

4574 

4Sb5 

6196 

5607 

5818 

811 

140 

146128 

146438 

146718 

117058 

147367 

147676 

147985 

148294 

148603 

148911 

309 

1 

9219 

9527 

1)835 

150142 

150149 

150756 

151(Xi3 

151370 

151676 

151982 

807 

2 

152288 

152594 

152900 

3205 

3510 

.'',M5 

4120 

4124 

4728 

5032 

30r» 

3 

5336 

5640 

5943 

6246 

6519 

<i852- 

7154 

7457 

7759 

8001 

803 

4 

8362 

86C4 

8965 

9266 

9567 

9868 

160168 

100409 

160769 

161068 

301 

145 

161368 

161667 

161967 

162266 

162564 

162863 

3161 

3160 

3768 

4055 

299 

6 

4353 

4650 

4947 

5214 

5511 

5S38 

6134 

6130 

6726 

7022 

297 

7 

7317 

7613 

7908 

8203 

8497 

8792 

90H6 

1)380 

9674 

9968 

295 

8 

170262 

170555 

170848 

171141 

171434 

171726 

172019 

172311 

172603 

172895 

2!)3 

9 

3186 

3478 

3769 

4060 

4351 

4641 

4932 

5222 

5512 

5802 

291 

150 

176091 

176381 

176670 

176959 

177248 

177536 

177825 

178113 

178401 

178689 

289 

1 

8977 

9264 

9552 

9S39 

180126 

180413 

180099 

180986 

181272 

181558 

287 

2 

181844 

182129 

182415 

182700 

2985 

3270 

3555 

3839 

4123 

4407 

285 

3 

4691 

4975 

5259 

5542 

5825 

6108 

6391 

6674 

6956 

7239 

283 

4 

7521 

7803 

8084 

8366 

8647 

8928 

9209 

94JX) 

9771 

190051 

281 

155 

190332 

190612 

190892 

191  in 

191451 

191730 

192010 

192289 

192567 

2846 

279 

6 

3125 

3403 

3681 

3959 

4237 

4514 

4792 

5069 

r>346 

5623 

278 

7 

5900 

6176 

6453 

6729 

7005 

7281 

7556 

7832 

8107 

8382 

276 

8 

8657 

8932 

9206 

9481 

9756 

200029 

200303 

200577 

200850 

201124 

274 

9 

201397 

201670 

201943 

202216 

202488 

2761 

3033 

3305 

3577 

3848 

272 

N. 

0 

1 

2 

3 

4 

5 

6 

7 

8 

9 

D 

497 


498 


APPENDIXES 


N. 

0 

1 

2 

3 

4 

5 

6 

7 

8 

9 

D. 

160 

204120 

204391 

204663 

204934 

205204 

205475 

205746 

206016 

20628fl 

206556 

271 

1 

6826 

7006 

7365 

7634 

7904 

8173 

8441 

8710 

8979 

9247 

269 

2 

9515 

0783 

210051 

210319 

210586 

210853 

211121 

211388 

211654 

211921 

267 

3 

212188 

212451 

2720 

21*86 

3252 

3518 

3783 

401!) 

4314 

4579 

266 

4 

4844 

5109 

5373 

5688 

6902 

6166 

6430 

6694 

6957 

7221 

264 

165 

7484 

7747 

8010 

8273 

8536 

8798 

9060 

9323 

9585 

9846 

262 

6 

220108 

220370 

220631 

220892 

221153 

221414 

221675 

221936 

222196 

222456 

261 

7 

27  Ifi 

2976 

3236 

31!tf5 

3755 

4015 

4274 

4533 

4792 

5051 

259 

8 

5309 

5568 

5826 

6084 

6342 

6600 

6858 

71  1'> 

7372 

7630 

2f>8 

9 

7887 

8141- 

8400 

8657 

8913 

9170 

9426 

9682 

9938 

230193 

256 

170 

2304  19 

23070* 

230960 

231215 

231470 

231724 

231979 

232234 

232488 

232712 

205 

1 

29<>6 

3250 

3504 

3757 

4011 

4264 

4517 

4770 

5023 

5276 

2V! 

2 

6528 

5781 

6033 

6285 

6537 

6789 

7011 

7292 

7544 

7795 

2  52 

3 

8046 

8297 

8548 

87<)9 

9049 

9299 

95  ~jO 

9800 

240050 

240300 

2>O 

4 

240549 

240799 

241018 

241297 

211546 

241795 

242044 

242293 

2541 

2790 

249 

175 

3038 

8286 

3531 

3782 

4030 

4277 

4525 

4772 

5019 

5?f>6 

248 

6 

6513 

5759 

6006 

6252 

6499 

6745 

6991 

7237 

7482 

7728 

21<* 

7 

75)73 

821?) 

HUM 

870') 

8951 

9198 

9413 

9687 

9932 

250176 

li  J."> 

8 

250120 

250601 

250908 

2511".! 

25  131*5 

25HW8 

251881 

252125 

25236S 

2610 

2.3 

9 

2853 

3090 

3338 

3580 

3822 

4064 

4306 

4548 

4790 

6031 

242  I 

180 

255273 

255514 

255755 

255996 

256237 

256477 

256718 

256958 

257198 

257439 

211 

1 

7071) 

7918 

8158 

8398 

8637 

8877 

9116 

93,";5 

9594 

9833 

239 

2 

200071 

260310 

260548 

260787 

261025 

261263 

261501 

261730 

•VH976 

262214 

238 

3 

2451 

2<J88 

2925 

3162 

3399 

3636 

3873 

4109 

43  1  6 

4582 

237 

4 

4818 

5054 

5290 

6525 

5761 

5996 

6232 

6467 

6702 

6937 

235 

185 

7172 

7400 

7611 

7875 

8110 

8314 

8578 

8812 

9046 

9279 

234 

6 

9513 

9746 

99HO 

270213 

270446 

270679 

27091-2 

271114 

271377 

271609 

233 

7 

271812 

272074 

272306 

2538 

2770 

3001 

323,t 

:-54(U 

3690 

3927 

232 

8 

4r,8 

4,  '48') 

4620 

48r><> 

5081 

5311 

55  1  2 

5772 

MX)2 

(>2:-52 

230 

9 

0162 

6692 

6<>21 

7151 

7380 

7609 

7838 

8067 

829G 

8525 

229 

190 

278754 

278982 

279211 

279  1«9 

279667 

279895 

280123 

280351 

280578 

280806 

228 

1 

281033 

281201 

281488 

281715 

281942 

282169 

2396 

2622 

284!) 

307.1 

227 

2 

3301 

3527 

3753 

:5979 

4205 

4431 

4656 

4882 

5107 

5332 

2i6 

3 

f»p>57 

5782 

OO07 

6232 

6456 

6681 

6905 

7130 

7354 

7578 

225 

4 

7802 

8026 

8249 

8473 

8696 

8920 

9143 

9366 

95S9 

9812 

223 

195 

200035 

290257 

290480 

290702 

290925 

291117 

291360 

291591 

291813 

292034 

290 

6 

2256 

2178 

20<><J 

2920 

3141 

3.U>3 

3584 

3804 

402". 

4216 

2211 

7 

4106 

4687 

4907 

51  27 

6347 

5567 

5787 

6007 

622(5 

(>iio 

220 

8 

<iW)5 

6881 

7104 

7323 

7542 

7761 

7979 

81')S 

8116 

so;-;,') 

219 

9 

8853 

9071 

8289 

9507 

9725 

9943 

300161 

300378 

300595 

300813 

218 

200 

301030 

301217 

301  164 

301681 

801898 

802114 

802331 

302547 

302761 

302980 

217 

1 

3196 

3412 

3(>'28 

3844 

40  >9 

4275 

4491 

4706 

4921 

51.  {6 

2K. 

2 

5351 

6666 

5781 

591  >6 

6211 

6425 

6639 

6854 

7068 

7282 

215 

3 

74SH> 

7710 

79'24 

8137 

8351 

856  1 

8778 

8991 

9'20i 

9417 

213 

4 

9630 

9843 

310056 

310268 

310181 

310693 

310906 

311118 

311330 

311542 

212 

201 

811754 

311966 

2177 

2389 

2600 

2812 

3023 

8214 

3445 

3656 

211 

6 

38<>7 

4078 

42S9 

44<H* 

4710 

4920 

5130 

5340 

6551 

57<iO 

210 

7 

5970 

6180 

6390 

6599 

680'.) 

7018 

7227 

7430 

7646 

78:>4 

209 

8 

80(13 

8272 

8481 

8(589 

8898 

9106 

9314 

9522 

9730 

9938 

208 

9 

320146 

320354 

320562 

320769 

320977 

321184 

321391 

321598 

321805 

322012 

207 

210 

322210 

322426 

322633 

322839 

823046 

323252 

323458 

323665 

323871 

324077 

206 

1 

4282 

4488 

469  1 

4890 

5105 

5310 

5516 

5721 

5026 

6131 

205 

2 

<J336 

6541 

6745 

6950 

71  .55 

7359 

7563 

7767 

7972 

8176 

201 

3 

8380 

8583 

8787 

895)1 

9191 

9398 

9601 

9805 

330008 

330211 

203 

4 

330414 

330617 

330819 

831022 

331225 

331427 

331630 

331832 

2034 

2236 

202 

215 

2438 

2640 

2842 

3044 

3246 

3447 

364  Q 

3850 

4051 

4253 

202 

6 

4454 

4655 

4856 

5057 

5257 

5458 

5658 

5859 

6059 

6260 

201 

7 

6460 

6660 

6860 

7060 

7260 

7459 

7659 

7858 

8058 

8257 

200 

8 

8466 

8656 

8855 

9054 

9253 

9451 

9650 

9849 

340047 

340246 

199 

9 

340444 

340612 

340841 

341039 

341237 

341435 

341632 

341830 

2028 

2225 

198 

N. 

0 

1 

2 

3 

4 

5 

6 

7 

8 

9 

J>. 

TABLE  OF  LOGARITHMS 


499 


N. 

0 

1 

2 

3 

4 

5 

6 

7 

8 

9 

D. 

220 

342423 

342620 

342817 

843014 

34S212 

343409 

848606 

843802 

848999 

844196 

197 

1 

4392 

4589 

47S5 

4981 

5178 

5374 

5570 

5766 

6962 

6157 

196 

2 

6353 

0549 

6744 

6939 

7135 

7330 

7525 

7720 

7915 

8110 

195 

3 

8305 

8500 

86^1 

8889 

9083 

9278 

9472 

9(566 

98(50 

35005  4 

194 

4 

350248 

350442 

350636 

850829 

351023 

351216 

351410 

351603 

851796 

1089 

193 

225 

2183 

2375 

2568 

2701 

2954 

3147 

8339 

3532 

3724 

8916 

103 

6 

4108 

4301 

4493 

4685 

4876 

5068 

6260 

5452 

5643 

583  4 

192 

7 

6020 

6217 

6108 

(5599 

6790 

6981 

7172 

7863 

7554 

7714 

191 

8 

7935 

8125 

Ml  6 

8506 

8696 

88S6 

9076 

0?(>6 

9456 

9616 

190 

9 

9835 

360025 

360215 

360404 

360593 

360783 

360972 

361161 

861350 

361539 

189 

230 

361728 

361917 

362105 

362294 

362482 

362671 

302859 

363048 

363236 

363424 

18ft 

1 

3612 

3800 

398.x 

4176 

4363 

45.")! 

4739 

49'26 

5113 

5301 

18S 

2 

fit  88 

5675 

5802 

60  49 

6*236 

6T23 

66  1  0 

671M5 

61)83 

71()') 

187 

3 

7356 

7542 

7729 

7915 

8101 

82S7 

8473 

865') 

88  1  5 

!¥):«> 

186 

4 

9216 

9401 

9587 

9772 

9958 

370143 

870328 

870513 

370698 

370883 

185 

235 

371068 

371253 

371437 

371622 

371806 

1991 

2175 

2360 

2544 

2728 

184 

6 

2912 

30% 

32SO 

3161 

3617 

3831 

4015 

4  IDS 

43.S2 

45(>5 

184 

7 

4748 

4932 

5115 

52'  >8 

5181 

5664 

5816 

6029 

6212 

(53!  )t 

183 

8 

6577 

6759 

6912 

71'2I 

7806 

7488 

7670 

7S.V2 

NM  t 

821  (5 

182 

9 

8398 

8580 

8761 

Ml  3 

91  '2  4 

9806 

9487 

9668 

9819 

380030 

181 

240 

380211 

380392 

380578 

380754 

880934 

381115 

381296 

381476 

381666 

881837 

181 

1 

2017 

2197 

2377 

2557 

2737 

2917 

3097 

3277 

3156 

3036 

180 

2 

8815 

3995 

4174 

4353 

4533 

4712 

4891 

5070 

6219 

5428 

179 

3 

5606 

5785 

51)64 

61  4  '2 

6321 

6l<)<> 

6677 

(5856 

70?  4 

7212 

178 

4 

7390 

7563 

7746 

7923 

8101 

8279 

8456 

8631 

8811 

8989 

178 

245 

0166 

9343 

9520 

9608 

9875 

390051 

390228 

390405 

390582 

390759 

177 

6 

39093.") 

391112 

89128S 

3914t)l 

391611 

1817 

11)93 

216!) 

mi 

2521 

17(5 

7 

2697 

2873 

3048 

3'22  1 

3100 

3575 

37.">1 

31126 

•4101 

4277 

176 

8 

4152 

4(527 

4802 

4!>77 

5  IT)  2 

532(5 

6501 

5676 

r>8.io 

6023 

175 

9 

6199 

6374 

6548 

6722 

6896 

7071 

7245 

7119 

7592 

7766 

171 

250 

397940 

398114 

398287 

398461 

398634 

39880ft 

308981 

399154 

399328 

399501 

173 

1 

9674 

98  1  7 

4(KH)20 

400192 

400365 

400538 

400711 

400883 

401056 

401228 

173 

2 

401401 

401573 

1715 

1!M7 

2089 

2261 

2133 

2(>01 

2777 

•2949 

17'2 

3 

3121 

3292 

3464 

3035 

3807 

3978 

414!) 

4320 

4492 

46(53 

171 

4 

483  4 

6005 

5176 

5316 

6517 

5688 

5858 

6029 

6199 

6370 

171 

255 

6540 

6710 

6881 

7051 

7221 

7391 

7561 

7731 

7901 

8070 

170 

6 

8240 

8410 

8571) 

8749 

8918 

9087 

9257 

9126 

95!  15 

97(51 

169 

7 

1)03'* 

410102 

410-271 

410110 

410609 

410777 

410940 

411114 

411'283 

411451 

1(5') 

8 

411620 

1788 

1956 

2124 

2293 

2461 

21)29 

2796 

2!)61 

3  1  32 

168 

9 

3300 

3467 

3635 

880!} 

3970 

4137 

4305 

4472 

4(>39 

480(5 

167 

260 

414973 

415140 

415307 

415474 

415641 

415808 

415974 

416141 

416308 

416474 

167 

1 

6641 

(5807 

6973 

7139 

7xo<> 

7472 

7638 

7804 

7970 

8135 

16(3 

2 

8301 

8467 

8033 

8798 

WM>4 

9129 

92% 

94(K) 

9('»'2r> 

!)7U1 

161 

3 

9956 

420121 

420286 

420451 

420616 

420781 

420915 

421110 

421275 

421439 

16,1 

4 

421604 

1768 

1933 

2097 

2261 

2426 

2r>90 

2754 

2918 

3082 

161 

265 

3*246 

3410 

3574 

3737 

3901 

4065 

4228 

4392 

4555 

4718 

164 

6 

4882 

5045 

6208 

6371 

553  i 

5(597 

5860 

6023 

6186 

6349 

1(53 

7 

6511 

6674 

6836 

0999 

7161 

7324 

7480 

7618 

7811 

7973 

162 

8 

8135 

8297 

8459 

8621 

8783 

8944 

0106 

9268 

9129 

9591 

162 

9 

9752 

9914 

430075 

430236 

430398 

430559 

430720 

430881 

431012 

431203 

161 

270 

431364 

431525 

431685 

431816 

432007 

432167 

432328 

432488 

432649 

432809 

161 

1 

2969 

3130 

3290 

3460 

8610 

3770 

3930 

4090 

4249 

4109 

100 

2 

4569 

4729 

4888 

5048 

6207 

5367 

6526 

5685 

5844 

6004 

159 

3 

6i  as 

6322 

6481 

6640 

6799 

6957 

7116 

7275 

7133 

7592 

159 

4 

7751 

7909 

8067 

8226 

8384 

8542 

8701 

8859 

9017 

9175 

168 

275 

9333 

9191 

9648 

9806 

9964 

440122 

440279 

440437 

440591 

440752 

158 

6 

440909 

4410f>6 

411224 

441381 

441,138 

If  >')."> 

1852 

2009 

21(56 

2323 

157 

7 

2480 

2637 

2793 

29.50 

3106 

3263 

8119 

8576 

3732 

8889 

157 

8 

4045 

4201 

4357 

4513 

4669 

4825 

4981 

5137 

5293 

5449 

156 

9 

6604 

5760 

5915 

6071 

6226 

6382 

6537 

6602 

6848 

7003 

155 

N. 

0 

1 

2 

3 

4 

5 

6 

7 

8 

8 

1>. 

500 


APPENDIXES 


K. 

0 

1 

2 

3 

4 

5 

6 

7 

8 

9 

D. 

280 

447158 

447313 

447468 

447623 

447778 

447933 

448088 

448242 

448397 

448552 

155 

1 

8700 

8861 

9015 

9170 

9324 

9478 

9633 

9787 

9941 

450095 

154 

2 

450249 

450403 

450557 

450711 

450865 

451018 

461172 

451326 

451479 

1633 

154 

3 

1780 

1910 

2003 

2247 

2400 

2553 

2706 

2859 

3012 

3165 

153 

4 

3318 

3471 

3624 

3777 

3930 

4082 

4235 

4387 

4540 

4692 

153 

285 

4845 

4997 

5150 

6302 

5454 

5606 

5758 

5910 

6062 

6214 

152 

6 

6366 

0518 

6670 

6821 

6973 

7125 

7270 

7428 

7579 

7731 

152 

7 

7882 

8033 

8184 

8336 

8487 

8638 

8789 

8940 

9091 

9242 

151 

8 

9392 

9543 

9694 

0815 

9995 

400146 

460296 

460147 

460597 

460748 

151 

9 

460898 

461048 

461198 

461348 

461499 

1619 

1799 

1948 

2098 

2248 

150 

290 

462398 

462548 

462697 

402847 

462997 

463146 

463296 

463445 

4&S594 

463744 

150 

1 

3893 

4042 

4I<)1 

4340 

4490 

4639 

4788 

4936 

5085 

5234 

149 

2 

5388 

5532 

5680 

5829 

5977 

6126 

6274 

6423 

6571 

6719 

149 

3 

(5868 

7010 

7161 

7312 

7460 

7008 

7750 

7904 

8052 

8200 

148 

4 

8347 

8495 

8643 

8790 

8938 

9085 

9233 

93SO 

9527 

9675 

148 

295 

9822 

9909 

470116 

470263 

470410 

470557 

470704 

470851 

470998 

471115 

147 

6 

471202 

471  138 

1585 

1732 

1878 

2025 

2171 

2318 

2464 

2610 

146 

7 

2750 

2903 

3O49 

319.-, 

3341 

3487 

3033 

3779 

3925 

4071 

146 

8 

4218 

4302 

47)8 

4653 

4799 

4944 

5090 

5235 

538  1 

5526 

146 

9 

6671 

6810 

6962 

6107 

6252 

6397 

6542 

6687 

6832 

0976 

145 

300 

477121 

477266 

477411 

477555 

477700 

477844 

477989 

478133 

478278 

478422 

145 

1 

8506 

8711 

8855 

8999 

9143 

9287 

9431 

9575 

9719 

98M 

144 

2 

480007 

480151 

480294 

480438 

480582 

480725 

480869 

481012 

481150 

4812W 

114 

3 

1443 

1586 

1729 

1H72 

20  Hi 

2159 

2302 

2445 

2588 

2731 

143 

4 

2874 

3016 

3159 

3302 

3445 

3587 

3730 

3872 

4015 

4157 

11.1 

305 

4300 

4442 

4585 

4727 

4869 

5011 

5153 

52% 

5437 

55  7!) 

142 

6 

6721 

58(53 

0005 

6147 

6289 

6130 

6572 

6711 

6833 

6<J<)7 

142 

7 

7138 

7280 

7421 

7563 

7704 

7845 

7')86 

8127 

8209 

8410 

141 

8 

8551 

869*2 

8833 

8«)7t 

91  1  1 

9255 

931)6 

95X7 

9677 

9M8 

141 

9 

9958 

490099 

490239 

490380 

490520 

490061 

490801 

4901)41 

491081 

491222 

110 

310 

491302 

491502 

491642 

491782 

491922 

492062 

492201 

492341 

492481 

492021 

140 

1 

2700 

2900 

3010 

317!) 

3319 

3  138 

3397 

3737 

8876 

4015 

139 

2 

4155 

4294 

4133 

4572 

4711 

4850 

4<)M> 

5r>K 

52(»7 

5400 

139 

3 

5544 

5683 

5822 

5!MX) 

(*)<)<) 

6238 

0370 

»>5H 

f><)33 

0791 

139 

4 

0930 

7068 

7206 

7344 

7483 

7621 

7759 

7897 

80H5 

8173 

138 

315 

8311 

8448 

8586 

8724 

8862 

8099 

9137 

9275 

9412 

(rr>o 

138 

6 

WW7 

0824 

9962 

5<XK)99 

500236 

50037  1 

500511 

5000  18 

500785 

5(X)922 

137 

7 

60105!) 

601196 

501333 

1470 

J007 

1714 

1880 

2017 

2154 

2291 

137 

8 

2427 

2564 

2700 

2837 

2973 

3109 

3246 

3382 

3318 

3<>35 

130 

9 

3791 

3927 

4063 

4199 

4335 

4471 

4607 

4743 

4878 

6014 

136 

320 

605150 

605280 

505421 

505557 

505693 

505828 

5051)64 

500099 

506234 

506370 

136 

1 

6505 

6640 

0776 

6911 

7016 

7181 

7310 

7431 

7586 

7721 

135 

2 

7850 

7991 

8126 

8260 

8393 

8530 

8G(>1 

879U 

8934 

906b 

i.r> 

3 

9203 

9337 

9471 

9600 

9740 

9874 

510009 

510143 

510277 

510411 

131 

4 

610545 

610679 

510813 

510947 

611081 

511215 

1349 

1482 

1616 

1750 

134 

325 

1883 

2017 

2151 

2284 

2418 

2551 

208  1 

2818 

2951 

308  { 

133 

8 

3218 

3351 

3484 

3617 

3750 

3883 

4016 

4149 

4282 

4415 

133 

7 

4548 

4681 

4813 

4946 

5079 

52  LI 

5344 

5476 

5609 

5741 

133 

8 

6874 

6006 

0139 

0271 

6403 

0535 

6068 

6800 

6932 

7004 

132 

9 

7196 

7328 

7460 

7592 

7724 

7855 

7987 

8119 

8251 

8382 

132 

330 

618514 

518646 

518777 

518909 

519010 

519171 

519303 

619434 

519506 

519697 

131 

1 

9828 

9959 

520090 

620221 

520353 

520484 

520615 

620745 

520876 

521007 

131 

2 

621138 

521269 

1400 

1530 

1661 

1792 

1922 

2053 

2183 

2314 

131 

3 

2444 

2575 

2705 

2835 

2966 

3096 

3226 

3350 

3  4  SO 

3010 

130 

4 

3740 

3876 

4006 

4136 

4266 

4396 

4526 

4656 

4785 

4915 

130 

335 

5045 

5174 

6304 

5434 

5563 

5693 

5822 

5951 

6081 

6210 

129 

6 

6339 

6469 

6598 

6727 

6806 

6985 

7114 

7243 

7372 

7501 

129 

7 

7630 

7759 

7888 

8016 

8145 

8274 

8402 

8531 

8660 

8788 

129 

8 

8917 

9045 

9174 

9302 

9  430 

9359 

9687 

9815 

9943 

530072 

128 

9 

630200 

630328 

630456 

630584 

630712 

530840 

530968 

631096 

531223 

1351 

128 

K. 

0 

1 

2 

3 

4 

5 

6 

7 

8 

9 

D. 

TABLE  OF  LOGARITHMS 


501 


N. 

0 

1 

2 

3 

4 

5 

6 

7 

8 

9 

D. 

340 

531479 

531607 

531734 

531862 

531990 

532117 

532245 

632372 

532500 

632627 

128 

1 

2754 

2882 

3009 

3136 

3264 

3391 

3518 

3645 

3772 

3899 

127 

2 

4026 

4153 

4280 

4407 

4534 

4661 

4787 

4914 

5041 

5167 

127 

3 

5294 

5421 

5547 

5671 

6800 

5927 

6053 

61  SO 

6306 

6432 

120 

4 

6568 

66S5 

6811 

6937 

7063 

7189 

7315 

7441 

7567 

7693 

126 

345 

7819 

7945 

8071 

8197 

8322 

844S 

R574 

8699 

8825 

8951 

120 

6 

9076 

9202 

9327 

9452 

9578 

9703 

9829 

99.34 

540079 

540204 

25 

7 

540329 

540455 

540580 

540705 

540830 

54095.3 

541080 

641205 

1330 

1454 

25 

8 

1579 

1704 

1829 

1953 

2078 

2203 

2327 

2  152 

2570 

2701 

23 

9 

2825 

2950 

3074 

3199 

3323 

3417 

3571 

3690 

3820 

3944 

•24 

350 

544008 

541192 

544316 

544440 

544564 

544688 

544812 

544936 

545000 

545183 

24 

1 

5307 

5431 

5555 

5678 

5802 

5925 

601') 

6172 

6296 

6419 

24 

2 

6,343 

6666 

6789 

6913 

7036 

7  1  59 

72S2 

7  10.3 

7529 

70.32 

21 

3 

7775 

7898 

8021 

8144 

8267 

8381) 

8512 

8635 

8758 

88S1 

•23 

4 

9003 

9126 

9249 

9371 

9494 

9616 

9739 

1*86  1 

9984 

550106 

23 

355 

550228 

550351 

550473 

550595 

550717 

550840 

550962 

551084 

651206 

1328 

22 

6 

1450 

1572 

1694 

1816 

1938 

2060 

2181 

2303 

2125 

2547 

22 

7 

2668 

2790 

2911 

3033 

3155 

3276 

339S 

3.31!) 

3610 

3702 

21 

8 

3883 

4004 

4  1  L'O 

4217 

4368 

4489 

4610 

4731 

48,32 

4973 

21 

9 

5094 

5215 

5330 

5457 

5578 

5699 

6820 

5940 

6061 

6182 

21 

360 

556303 

556423 

556544 

556664 

556785 

556905 

557026 

557146 

557267 

557387 

120 

1 

7507 

7627 

7748 

7sus 

79SS 

8108 

8228 

8319 

MO!) 

8,3X9 

120 

2 

8709 

8X2D 

8«>18 

9068 

91  hS 

9.J08 

9128 

95-18 

9<iii7 

9787 

120 

3 

9907 

50(>0'2f> 

500140 

660265 

5003  S5 

500.304 

560024 

560713 

660803 

5(50982 

119 

4 

501101 

1221 

1310 

1  159 

]578 

1698 

1817 

1936 

2055 

2174 

ll'J 

365 

2293 

2112 

2531 

2650 

2769 

2887 

3006 

3125 

3244 

3362 

110 

6 

3181 

3600 

3718 

3837 

395,3 

4074 

4192 

4311 

4429 

4518 

119 

r 

4666 

47M 

4903 

5021 

5139 

5257 

53  7(> 

51')  4 

5612 

5730 

11H 

8 

5848 

5900 

OOS4 

020-2 

6320 

6437 

6,3.35 

6673 

6791 

6909 

11H 

9 

7026 

71  14 

7262 

7379 

7197 

7614 

7732 

7849 

7907 

8084 

118 

370 

568202 

568319 

568436 

568554 

568671 

568788 

568905 

569023 

569140 

609257 

117 

1 

9374 

9191 

%OH 

9725 

9812 

99.39 

670076 

570193 

570309 

670420 

117 

2 

570543 

570660 

570776 

57081)3 

571010 

571126 

1  2  13 

1359 

1176 

1592 

117 

3 

1709 

182.") 

1912 

2038 

2174 

2'291 

2107 

2523 

2039 

275,3 

lie. 

4 

2872 

2988 

3104 

3220 

3336 

3452 

3568 

3684 

3800 

3915 

110 

375 

4031 

4147 

4263 

4379 

4494 

4610 

4726 

4841 

4957 

5072 

no 

6 

5188 

5303 

MI9 

5334 

50.30 

57(55 

5880 

5996 

6111 

6220 

11.3 

7 

6341 

6157 

6,3/2 

6687 

(te02 

<>'H7 

7032 

7147 

7202 

7377 

11.3 

8 

7  192 

7007 

7722 

7836 

7931 

8066 

8181 

8295 

8110 

8525 

113 

9 

8639 

8754 

8808 

8983 

9097 

9212 

9320 

9441 

95.35 

900U 

114 

380 

579784 

579898 

580012 

580126 

580241 

580355 

580469 

580583 

680697 

580811 

114 

1 

580925 

581039 

1153 

1267 

1381 

1495 

1608 

1722 

1H30 

1950 

111 

2 

20fi3 

2177 

2291 

'2404 

2518 

2631 

271.3 

'2S.38 

2<)7'2 

30H5 

111 

3 

3199 

33  J  2 

3120 

3539 

3052 

3705 

387!) 

3992 

4  105 

4218 

113 

4 

4331 

4414 

4557 

4670 

4783 

4896 

5009 

5122 

5235 

5348 

113 

385 

5461 

5574 

5686 

5799 

5912 

6024 

6137 

6250 

6362 

6475 

113 

6 

6587 

6700 

6812 

6925 

7037 

7149 

7262 

7374 

74  SO 

759'  > 

112 

7 

7711 

7823 

7935 

8017 

MM) 

8272 

83h  1 

8496 

8008 

8720 

11  '2 

8 

8832 

8944 

9056 

9167 

9279 

«>:!!)] 

Til  13 

9615 

9720 

9838 

112 

9 

9950 

590061 

590173 

690284 

590396 

590507 

590619 

690730 

590842 

590953 

112 

390 

591063 

591176 

591287 

591399 

591510 

591621 

591732 

591843 

591955 

592066 

111 

1 

in  77 

22H8 

2399 

2510 

262J 

2732 

'2843 

2954 

3004 

3175 

111 

2 

3280 

3397 

3508 

3618 

3729 

3840 

3930 

4061 

4171 

4282 

111 

3 

4393 

4503 

4614 

4724 

4834 

4')  15 

50.3  r> 

5165 

5270 

6380 

110 

4 

5496 

5(506 

5717 

6827 

6937 

6047 

6157 

6267 

6377 

6487 

110 

395 

6597 

6707 

6817 

6927 

7037 

7146 

7256 

7366 

7476 

7586 

110 

6 

769.3 

7805 

7914 

8024 

8134 

8213 

8",.33 

8402 

8572 

8081 

110 

7 

8791 

8900 

9009 

9119 

9228 

9337 

9116 

9.356 

9065 

9774 

109 

8 

9883 

9992 

600101 

600210 

600319 

600428 

600537 

600646 

600755 

600864 

109 

9 

600973 

601082 

1191 

1299 

1408 

1517 

1625 

1734 

1843 

1951 

109 

N. 

0 

1 

2 

3 

4 

5 

6 

7 

8 

9 

D. 

502 


APPENDIXES 


N. 

0 

1 

2 

3 

4 

5 

6 

7 

8 

9 

D. 

400 

602060 

602169 

602277 

602386 

602494 

602603 

602711 

602819 

602928 

603036 

108 

1 

3144 

3253 

3361 

3469 

3577 

3686 

3794 

3902 

4010 

4118 

las 

2 

4226 

4334 

4412 

4550 

4658 

4766 

4874 

49S2 

5080 

5197 

lus 

3 

5305 

54  H 

5521 

5628 

5736 

5844 

5051 

6050 

6166 

6274 

1()S 

4 

6381 

6489 

6596 

«70t 

6811 

6910 

7026 

7133 

7241 

7348 

107 

405 

7455 

7562 

7609 

7777 

7884 

7991 

8098 

8205 

8312 

8410 

107 

6 

8526 

8633 

8740 

8847 

8954 

0061 

9167 

0274 

9381 

94feS 

107 

7 

051)4 

9701 

9808 

0914 

610021 

610128 

610234 

610341 

610447 

610.334 

107 

8 

610600 

610767 

610873 

610979 

108(5 

1102 

1298 

140") 

1511 

1017 

100 

9 

1723 

1820 

1030 

2012 

2148 

2254 

2300 

2466 

2572 

2678 

106 

410 

612784 

612890 

612996 

613102 

613207 

613313 

613110 

613525 

613C30 

613736 

106 

1 

3842 

3917 

4053 

4  159 

4204 

4370 

4475 

45M 

4(.80 

4792 

101, 

2 

4897 

5003 

5108 

5213 

5319 

5421 

5529 

5(>'U 

5740 

5845 

101 

3 

59  jO 

eo5.r» 

(51  (X) 

620,1 

6370 

6470 

65H1 

6CSO 

0790 

0895 

101 

4 

7000 

7105 

7210 

7315 

7420 

7525 

7020 

7734 

7839 

7943 

10.1 

415 

80  18 

8153 

8257 

8302 

8466 

8571 

8676 

8780 

8884 

8989 

105 

6 

0093 

9198 

9302 

9406 

9511 

%1.1 

0710 

%2I 

9928 

6200.i2 

101 

7 

620136 

620210 

620344 

620448 

620552 

620656 

6207(50 

620804 

6209(5H 

1072 

Id  I 

8 

1170 

1280 

1.-JH4 

1-ISH 

1,1<)2 

ib(n 

17'K) 

]«>(« 

2007 

2110 

104 

9 

22  U 

2318 

2421 

2525 

2628 

2732 

2835 

2939 

3042 

3140 

104 

420 

623249 

623353 

623456 

623550 

623063 

623706 

623809 

623973 

624076 

624170 

103 

1 

42H2 

4385 

4488 

4591 

4693 

4798 

4001 

5004 

5107 

5210 

10.4 

2 

5312 

5415 

5518 

5621 

5724 

5827 

5929 

6032 

6135 

C2H8 

103 

3 

6340 

G443 

6546 

(5618 

6751 

6H53 

0056 

7058 

7161 

72f>3 

103 

4 

7306 

7408 

7571 

7673 

7775 

7878 

7980 

8082 

8185 

8287 

102 

425 

8380 

8491 

8593 

869.') 

8707 

8000 

0002 

910  J 

0206 

0308 

102 

6 

OHO 

9512 

0613 

9715 

9817 

0919 

630021 

630123 

630224 

630320 

102 

7 

630128 

6305:50 

630631 

630733 

630835 

630030 

1038 

ii:w 

1241 

1312 

102 

8 

1441 

1515 

1617 

17  IS 

1849 

i(r>i 

2052 

2r>.j 

2211 

2310 

101 

9 

2157 

2559 

2660 

2701 

2802 

2903 

300  1 

3163 

3200 

3307 

101 

430 

633468 

633569 

633670 

633771 

633872 

633073 

634074 

634175 

634276 

634370 

101 

1 

4477 

4578 

4G79 

4770 

48SO 

4981 

6081 

51h2 

5283 

5.iNi 

101 

2 

5484 

558  1 

5685 

67K5 

688(5 

5980 

60S7 

61S7 

6287 

6388 

1IMI 

3 

6488 

0588 

6f>88 

6789 

688<> 

d(JS9 

70S9 

71SO 

7290 

7.M) 

100 

4 

7490 

7590 

7690 

7790 

7800 

7990 

8090 

8190 

8200 

8389 

100 

435 

8489 

8589 

8680 

8789 

8888 

8988 

0088 

9188 

0287 

0387 

100 

6 

9486 

9586 

068(1 

0785 

9885 

0984 

640084 

6401W 

640263 

640382 

0'» 

7 

640481 

640581 

640680 

640770 

640870 

640978 

1077 

1177 

1270 

1.-S7.1 

9') 

8 

1474 

1573 

1672 

1771 

1871 

1970 

20bO 

2108 

2207 

2J(i(> 

<)<) 

9 

2465 

25G3 

2662 

27G1 

2800 

2950 

3058 

3156 

3255 

3354 

09 

440 

643463 

643551 

6-13650 

643749 

643847 

643046 

644044 

644143 

644242 

644340 

08 

1 

4439 

4537 

4036 

4TC4 

4832 

4031 

5020 

5127 

5220 

5324 

08 

2 

5422 

5521 

6610 

5717 

6815 

5913 

0011 

6110 

0208 

6306 

OS 

3 

6404 

6502 

6600 

6698 

6706 

6804 

6002 

7089 

71S7 

7285 

OH 

4 

7383 

7481 

7579 

7676 

7774 

7872 

7969 

8067 

8165 

8202 

98 

445 

8360 

8158 

8555 

8653 

8750 

8848 

8945 

9043 

9140 

0237 

97 

6 

9335 

9432 

0530 

9627 

9724 

9821 

0010 

650016 

650113 

650210 

07 

7 

650308 

650405 

650602 

650590 

650C96 

650793 

650890 

0087 

1084 

1181 

97 

8 

1278 

1375 

1472 

1560 

3666 

1762 

1859 

1056 

2053 

2150 

97 

9 

2246 

2343 

2140 

2536 

2633 

2730 

2826 

2923 

3010 

3116 

97 

450 

653213 

653309 

653405 

653502 

653508 

653695 

653791 

653888 

653984 

654080 

96 

1 

4177 

4273 

4369 

4465 

45(52 

4658 

4754 

4850 

4010 

6042 

06 

2 

6138 

5235 

5331 

5427 

5523 

5610 

5715 

5810 

5906 

6002 

90 

3 

61)98 

6194 

6200 

6386 

6482 

6577 

6673 

6709 

6864 

6960 

% 

4 

7056 

7152 

7247 

7343 

7438 

7534 

7629 

7725 

7820 

7916 

96 

455 

8011 

8107 

8202 

8298 

8393 

8488 

8584 

8679 

8774 

8870 

95 

6 

8965 

0060 

9155 

9250 

9346 

9441 

0530 

9631 

0726 

98-21 

95 

7 

9916 

660011 

660106 

660201 

6602% 

660391 

660486 

660581 

660676 

660771 

95 

8 

660865 

0960 

1055 

1150 

1245 

1339 

1434 

1529 

1623 

1718 

95 

9 

1813 

1907 

2002 

2096 

2191 

2286 

2380 

2475 

2569 

2663 

95 

K. 

0 

1 

2 

3 

4 

5 

6 

7 

8 

9 

D. 

TABLE  OF  LOGARITHMS 


503 


N. 

0 

1 

2 

3 

4 

5 

6 

7 

8 

9 

D. 

460 

662758 

662852 

662947 

663041 

66S135 

663230 

66S324 

663118 

663512 

668607 

91 

1 

3701 

3705 

3889 

39S3 

4078 

4172 

4266 

43(50 

4154 

45(8 

91 

2 

4612 

4716 

4830 

4924 

60  IS 

5112 

5'206 

5299 

5393 

6487 

91 

3 

5581 

5675 

5769 

5862 

5956 

6050 

6113 

6237 

6331 

6424 

91 

4 

6518 

6612 

6703 

679'J 

681)2 

6t>Sli 

7079 

7173 

7266 

7300 

91 

465 

7453 

7546 

7640 

7733 

7820 

7020 

8013 

8106 

8109 

8293 

93 

6 

83M5 

8479 

8572 

8665 

8759 

8852 

8915 

9038 

9131 

9224 

93 

7 

9317 

9410 

9503 

9590 

9089 

97.v> 

9875 

99(17 

670000 

670153 

93 

8 

670246 

670339 

670431 

670524 

670617 

670710 

670802 

670895 

WSS 

1080 

93 

9 

1173 

1205 

1358 

1431 

1513 

103(5 

17-28 

1821 

1913 

2005 

93 

470 

672098 

672190 

6722S3 

672375 

672467 

672560 

672652 

672744 

672836 

672929 

92 

1 

3021 

3113 

3205 

3297 

3300 

3182 

3574 

3(560 

3758 

3850 

92 

2 

3012 

4034 

4126 

4218 

4310 

4  102 

44()1 

4f>Mi 

4677 

4769 

9! 

3 

4801 

4053 

5045 

5137 

6228 

53'2<> 

54  1  2 

6503 

5595 

5(587 

92 

4 

5778 

5870 

5962 

6053 

6145 

6230 

6328 

6119 

6511 

0602 

92 

475 

6694 

6785 

6876 

6968 

7059 

7151 

7212 

7333 

7421 

7516 

91 

6 

7007 

7098 

7789 

7881 

7972 

8063 

8154 

8215 

833(5 

8427 

91 

7 

8,')  18 

860) 

8700 

8791 

8882 

8973 

9064 

9155 

924(5 

9337 

91 

8 

94'28 

a">  19 

9010 

4700 

9791 

9S82 

9973 

6800<>3 

0801  51 

6802  \  r> 

91 

9 

680330 

680-126 

680,-)17 

G80607 

680098 

680789 

680879 

0970 

10(50 

1151 

91 

480 

681241 

681332 

681422 

681513 

681003 

681693 

681781 

681874 

681964 

682055 

90 

1 

21  15 

2235 

2326 

2116 

2500" 

2V  16 

20S<> 

2777 

2867 

2957 

90 

2 

3017 

3137 

3227 

3317 

3407 

3497 

35S7 

3077 

3767 

3S57 

90 

3 

3947 

40  i7 

41*27 

4217 

4307 

439« 

418(1 

4570 

4  000 

475(5 

90 

4 

4H15 

4935 

5025 

6114 

6204 

5291 

6383 

5473 

6503 

5052 

90 

485 

5742 

5831 

5921 

6010 

6100 

6189 

6279 

6308 

6158 

6617 

89 

6 

oo:;6 

6726 

6815 

690  1 

6')9i 

7083 

7172 

72(51 

7351 

7140 

89 

7 

7524 

7018 

7707 

779(5 

7880 

7975 

8061 

8153 

8212 

8331 

8<J 

8 

8420 

8509 

8598 

8687 

8770 

8805 

S9',.{ 

901-2 

9131 

9'2'20 

89 

9 

9309 

9398 

9486 

9575 

9664 

9753 

9811 

9930 

61)0019 

690107 

89 

490 

690196 

690285 

690373 

690462 

690550 

690639 

690728 

690816 

OM905 

690993 

89 

1 

1081 

1170 

125H 

1347 

1  135 

1  52  1 

1(512 

1700 

1789 

1877 

88 

2 

i06r> 

2053 

2112 

2230 

2318 

2lO(i 

2191 

2583 

2071 

2759 

88 

3 

2817 

293  > 

3023 

3111 

3199 

3'2S7 

3375 

3163 

3551 

3039 

88 

4 

3727 

3815 

3903 

399  1 

4078 

4166 

4254 

4312 

4430 

4517 

88 

495 

4605 

4693 

4781 

4868 

4956 

50  It 

5131 

5219 

5307 

5391 

88 

6 

54  82 

656C> 

5657 

57  1  1 

5832 

59t<> 

0007 

6091 

6182 

0269 

87 

7 

0350 

6444 

6r>3l 

6<ilH 

0700 

(.793 

6880 

6908 

7055 

7142 

87 

8 

7229 

7317 

7104 

71')l 

7578 

7665 

7752 

7839 

7920 

8014 

87 

9 

8101 

8188 

8275 

8302 

8419 

853ri 

8022 

8709 

8746 

8883 

87 

500 

698970 

699057 

6991  14 

699231 

699317 

699404 

699491 

699578 

699064 

099751 

87 

1 

9838 

9921 

700011 

700098 

7001  hi 

700271 

700358 

700144 

700531 

700017 

87 

2 

700704 

700790 

0877 

0963 

1050 

1130 

1222 

1309 

1395 

1482 

86 

3 

i:>r>8 

1654 

1741 

1827 

1913 

1999 

2086 

2172 

2258 

2344 

8(5 

4 

2431 

2517 

2603 

2089 

2775 

2861 

2917 

3033 

3119 

3205 

86 

505 

3291 

3377 

3103 

3510 

3(535 

3721 

3807 

3843 

3979 

4065 

86 

6 

4151 

4236 

4322 

4108 

4491 

4579 

4065 

4751 

4837 

4922 

86 

7 

5008 

5094 

5174 

62(15 

6350 

6430 

5522 

5007 

5093 

5778 

8(5 

8 

5864 

5919 

6035 

6120 

6200 

6291 

6376 

0  10'2 

6517 

6632 

85 

9 

6718 

6803 

6888 

6974 

7059 

7111 

7229 

7315 

7100 

7185 

85 

510 

707570 

707655 

707740 

707826 

707911 

707996 

708081 

7081  6f 

708251 

708336 

85 

1 

8421 

8506 

8591 

8070 

8701 

88  1C 

8931 

9015 

9100 

9185 

85 

2 

9270 

9355 

9140 

9524 

9004 

9691 

9779 

9863 

9448 

710033 

85 

3 

710117 

710202 

710287 

710371 

710150 

710510 

710625 

710710 

710791 

0879 

85 

4 

0963 

1018 

1132 

1217 

1301 

1385 

1470 

1554 

1639 

1723 

84 

515 

1807 

1892 

1976 

2060 

2111 

2229 

2313 

2397 

2181 

2566 

84 

6 

2650 

2734 

2818 

2902 

21)8« 

3070 

3154 

3238 

3323 

3407 

81 

7 

3491 

3575 

3659 

3712 

3826 

3910 

3994 

4078 

41(52 

4246 

84 

8 

4330 

4414 

4197 

4581 

4065 

4719 

4833 

4910 

fiOOO 

6084 

84 

9 

6167 

6251 

6335 

5418 

6602 

5586 

6669 

5753 

6830 

6920 

84 

K. 

0 

1 

2 

3 

4 

5 

6 

7 

8 

9 

D. 

504 


APPENDIXES 


N. 

0 

1 

2 

3 

4 

5 

6 

7 

8 

9 

D. 

520 

716003 

716087 

716170 

716254 

716337 

716421 

716504 

716588 

716671 

716754 

83 

1 

6838 

6921 

7004 

7088 

7171 

7254 

7338 

7421 

7504 

7587 

S3 

2 

7G71 

7754 

7837 

7920 

8003 

8086 

8169 

8253 

83^6 

8419 

83 

3 

8502 

8585 

8068 

8751 

8834 

8917 

9000 

9083 

9103 

9248 

83 

4 

9331 

9414 

9497 

9580 

9663 

9715 

9828 

9911 

91)94 

720077 

83 

525 

720159 

720242 

720325 

720407 

720490 

720573 

720655 

720738 

720821 

0903 

83 

6 

09H6 

1068 

mi 

1233 

1316 

1398 

1481 

1583 

1046 

1728 

82 

7 

1811 

1893 

1975 

2058 

2110 

2222 

2305 

2387 

2469 

2552 

82 

8 

20'',  1 

27  in 

2798 

2881 

2063 

3045 

3127 

3209 

3*291 

3374 

82 

9 

3450 

3538 

3020 

3702 

3784 

3866 

3918 

4030 

4112 

4194 

82 

530 

724276 

724358 

724440 

724522 

724604 

724685 

724767 

724849 

724931 

725013 

82 

1 

5005 

5176 

5258 

5310 

5122 

5503 

5585 

5067 

tr>748 

5830 

82 

2 

6912 

5993 

6075 

6156 

6238 

6320 

6401 

6483 

6564 

6646 

82 

S 

6727 

6809 

6890 

6972 

7<T>3 

7134 

7216 

7297 

7379 

7460 

81 

4 

7541 

7623 

7704 

7785 

7806 

7948 

8029 

8110 

8191 

8273 

81 

535 

8354 

8435 

8516 

8597 

8078 

8759 

8841 

8922 

9003 

9084 

81 

6 

0165 

9216 

9327 

9408 

9  189 

9570 

9651 

9732 

9813 

9893 

81 

7 

9974 

730055 

730136 

730217 

730298 

730378 

730  HO 

730540 

730621 

730702 

81 

8 

730782 

0803 

0944 

1024 

11  05 

1180 

l'2f>0 

1347 

1428 

1508 

81 

9 

1589 

1009 

1750 

1830 

1911 

1991 

2072 

2152 

2233 

2313 

81 

540 

732394 

732474 

732555 

732G35 

732715 

732796 

732870 

732956 

733037 

733117 

80 

1 

3197 

327H 

3358 

3138 

3518 

3598 

3079 

3759 

3839 

3919 

80 

2 

391)9 

4079 

4160 

4210 

43JO 

4400 

4480 

4560 

4010 

4720 

80 

3 

4800 

4880 

4900 

50  10 

5120 

5200 

5270 

537) 

5130 

5310 

80 

4 

6599 

6079 

5759 

5838 

5918 

5998 

6078 

6157 

6237 

0317 

80 

545 

6307 

6470 

6556 

6035 

6715 

6795 

6874 

6954 

7034 

7113 

80 

6 

7193 

7272 

7352 

7KU 

7511 

75<K) 

7070 

7749 

7829 

790S 

79 

7 

7987 

80«7 

8140 

8225 

8305 

8384 

8403 

8543 

8622 

8701 

79 

8 

87  SI 

8800 

8'.nO 

9018 

9007 

9177 

9250 

9335 

9414 

9493 

79 

9 

9572 

9651 

9731 

9810 

9889 

9UC8 

740047 

740120 

740205 

740284 

79 

550 

740363 

740442 

740521 

740600 

740078 

740757 

740830 

740915 

740991 

741073 

79 

1 

u;>2 

1230 

1309 

1388 

1407 

1516 

1024 

1703 

1782 

1860 

79 

2 

1939 

2018 

201)6 

2175 

2254 

2332 

2411 

2  189 

2508 

2(547 

79 

3 

2725 

2801 

2HS2 

2JWI 

3039 

3118 

3190 

3275 

33.T.1 

3431 

78 

4 

3510 

3588 

3667 

3715 

3823 

3902 

3980 

4058 

4136 

4215 

78 

555 

4293 

4371 

4449 

4528 

4606 

4684 

4762 

4840 

4919 

4997 

78 

6 

5075 

5153 

5231 

6301) 

5387 

5405 

5543 

5021 

569<J 

5777 

78 

7 

6855 

6933 

6011 

6089 

6167 

6245 

6323 

6401 

0479 

6356 

78 

8 

6634 

6712 

6790 

68(>8 

691.") 

7023 

7101 

7179 

7230 

7334 

78  V 

9 

7412 

7489 

7507 

7645 

7722 

7800 

7878 

7935 

8033 

8110 

78 

560 

748188 

748206 

748343 

748421 

748498 

748576 

748653 

748731 

748808 

748885 

77 

1 

81163 

9040 

9118 

9195 

9272 

9350 

9427 

9504 

9582 

9639 

77 

2 

9730 

9814 

9891 

9%8 

750015 

750123 

750200 

750277 

750334 

750431 

77 

3 

750508 

750586 

7500(53 

750740 

0817 

0891 

0971 

1048 

1125 

1202 

77 

4 

1279 

1356 

1433 

1510 

1587 

16C4 

1741 

1818 

1895 

1972 

77 

565 

2048 

2125 

2202 

2279 

2356 

2433 

2509 

2386 

2663 

2740 

77 

6 

281G 

2893 

2970 

3047 

3123 

3200 

3277 

3333 

3430 

3300 

77 

7 

3583 

3600 

3730 

3813 

8889 

3966 

4042 

4119 

4193 

4272 

77 

8 

4348 

4425 

4501 

4578 

4654 

4730 

4807 

4883 

4960 

5030 

76 

9 

6112 

0189 

5205 

5341 

5417 

5494 

6570 

5046 

5722 

5799 

76 

570 

755875 

755951 

756027 

756103 

756180 

756256 

756332 

756408 

756484 

756560 

76 

1 

6630 

6712 

6788 

(5>S64 

6910 

7016 

7092 

7168 

7244 

7320 

76 

2 

7396 

7472 

7548 

7624 

7700 

7775 

7851 

7927 

8003 

8079 

76 

S 

8155 

8230 

8306 

8382 

8458 

8533 

8609 

8685 

8761 

8836 

70 

4 

8912 

8988 

90(53 

9139 

9214 

9290 

9S66 

9441 

9517 

9592 

76 

675 

9668 

9743 

9819 

9894 

9970 

700045 

760121 

760196 

760272 

760347 

75 

6 

760422 

700408 

760573 

700049 

760724 

0799 

0875 

0950 

1025 

1101 

75 

7 

1176 

1251 

1326 

1402 

1477 

1552 

1627 

1702 

1778 

1833 

73 

8 

1928 

2003 

2078 

2153 

22*28 

2303 

2378 

2453 

2529 

2604 

75 

9 

2679 

2754 

2829 

2904 

2978 

3053 

3128 

3203 

3278 

3353 

75 

N. 

0 

1 

2 

3 

4 

5 

6 

7 

8 

9 

D. 

TABLE  OF  LOGARITHMS 


505 


N. 

0 

1 

2 

3 

4 

5 

6 

7 

8 

9 

D. 

580 

763428 

763503 

763578 

763653 

763727 

763802 

763S77 

763952 

764027 

764101 

75 

1 

4176 

4251 

4326 

4400 

4475 

4550 

4621 

4699 

4774 

4848 

75 

2 

4023 

4908 

5072 

5147 

5-221 

5-2'  >6 

5370 

5445 

5520 

5594 

75 

3 

5609 

5743 

5S18 

5892 

5966 

6041 

6115 

6190 

6264 

6388 

74 

4 

6413 

6487 

6562 

6636 

6710 

6785 

6859 

6933 

7007 

7082 

74 

585 

7156 

7230 

7304 

7379 

7453 

7527 

7601 

7675 

7749 

7823 

74 

6 

7808 

7972 

8046 

8120 

8194 

8268 

8312 

8416 

8400 

8564 

74 

7 

8l)38 

8712 

8786 

8860 

8934 

9008 

9082 

9156 

9230 

0303 

74 

8 

9377 

9451 

9525 

9599 

9073 

9746 

9820 

9891 

0968 

770042 

74 

9 

770115 

770189 

770263 

770336 

770410 

770484 

770557 

770631 

770705 

0778 

74 

590 

770852 

770926 

770999 

771073 

771146 

771220 

771293 

771367 

771440 

771514 

74 

1 

K>S7 

1661 

1731 

1808 

1881 

1955 

202S 

2102 

2175 

2248 

73 

2 

9300 

2395 

2168 

2512 

2615 

I2te8 

276'2 

2835 

2908 

2981 

73 

3 

3055 

31*28 

3201 

3274 

3348 

3121 

3404 

3.r)'J7 

3640 

3713 

73 

4 

3786 

3860 

3933 

4006 

4079 

4152 

4225 

4208 

4371 

4444 

73 

595 

4517 

4590 

4663 

4736 

4809 

4882 

4055 

5028 

5100 

5173 

73 

6 

5210 

5319 

5.192 

5465 

5538 

5010 

5683 

5756 

6820 

6902 

73 

7 

5974 

6017 

6120 

6193 

0265 

6338 

6111 

6183 

6556 

6620 

73 

8 

6701 

6771 

68  1  6 

6919 

6902 

7064 

7137 

7209 

7282 

7354 

73 

9 

7127 

7199 

7572 

7614 

7717 

7769 

7862 

7931 

8006 

8079 

72 

600 

778151 

778224 

778296 

778368 

778141 

778513 

778585 

778658 

778730 

778802 

72 

1 

8874 

8947 

9019 

OO'Jl 

0163 

0236 

9308 

9380 

0452 

0524 

7'' 

2 

9596 

9669 

9741 

9813 

0885 

0957 

78002!) 

780101 

780173 

780245 

72 

3 

780317 

78MS9 

780  J61 

780533 

78CMnOr> 

780677 

0710 

0821 

0893 

0965 

72 

4 

1037 

1109 

1181 

1253 

1324 

1396 

1468 

1510 

1612 

1684 

72 

605 

1755 

1827 

1899 

1971 

2042 

2111 

21R6 

2258 

2329 

2401 

72 

6 

2173 

2544 

2616 

2688 

2759 

2831 

2902 

2974 

3046 

8117 

72 

7 

3189 

3-260 

3332 

3103 

3475 

3546 

3618 

3689 

3761 

8832 

71 

8 

3904 

3975 

4016 

1118 

41S9 

4'261 

1332 

4  103 

4475 

4546 

71 

9 

4617 

4689 

4760 

4831 

4902 

4974 

5045 

6116 

5187 

5259 

71 

610 

785330 

785401 

785172 

785543 

785615 

785686 

785757 

785828 

785899 

785970 

71 

1 

6041 

6112 

6183 

6'25  1 

6325 

6396 

6167 

6538 

6600 

6680 

71 

2 

6751 

6822 

6893 

6964 

7035 

7106 

7177 

7248 

7310 

7390 

71 

3 

7400 

7531 

760-2 

7673 

7714 

7815 

7885 

7956 

8027 

8098 

71 

4 

8168 

8239 

8310 

8381 

8-151 

8522 

8593 

8663 

8734 

8804 

71 

615 

8875 

8946 

9016 

9087 

0157 

9228 

9299 

0360 

0440 

9510 

71 

6 

9581 

%51 

9722 

9792 

0863 

9033 

790<H)4 

700074 

700114 

790215 

70 

7 

790285 

790356 

7904'26 

7904  96 

700567 

790637 

0707 

07  78 

08-18 

0918 

70 

8 

09SK 

1059 

1129 

1199 

1269 

1310 

1410 

1-180 

1550 

1620 

70 

9 

1691 

1761 

1831 

1901 

1971 

2041 

2111 

2181 

2252 

2322 

70 

620 

792392 

792402 

792532 

702602 

792672 

792742 

792812 

792882 

702952 

793022 

70 

1 

3092 

31  62 

3*231 

3301 

3371 

3441 

3511 

3581 

3651 

S721 

70 

2 

3790 

38(50 

3930 

4000 

4070 

4139 

4209 

4279 

4349 

4418 

70 

3 

4488 

4558 

4627 

4697 

4767 

4836 

4906 

4976 

5045 

5115 

70 

4 

5185 

5254 

5324 

5393 

6463 

6532 

5602 

6672 

6741 

6811 

70 

625 

5880 

5949 

6019 

6088 

6158 

6227 

6297 

6306 

6436 

6505 

69 

6 

6574 

6644 

6713 

6782 

6852 

6921 

6990 

7060 

7129 

71  08 

60 

7 

72f>8 

7337 

7406 

7475 

7545 

7614 

7683 

7752 

7821 

7890 

fl» 

8 

7900 

8029 

8098 

8167 

8236 

8305 

8371 

8443 

8513 

8,182 

69 

9 

fe651 

8720 

8789 

8858 

8927 

8096 

9065 

9134 

0203 

9272 

69 

630 

790341 

709409 

799478 

709517 

799616 

799685 

799754 

700823 

790892 

799961 

60 

1 

800029 

80009S 

800167 

800236 

800305 

800373 

800442 

800511 

800580 

800648 

69 

2 

0717 

0786 

0854 

0923 

0992 

1061 

1129 

1198 

1266 

1335 

69 

3 

140* 

1172 

1541 

1609 

1678 

1747 

1815 

1884 

1952 

2021 

60 

4 

2089 

2158 

2226 

2295 

2363 

2432 

2500 

2668 

2637 

2705 

68 

635 

2774 

2842 

2910 

2079 

3047 

3116 

3184 

3252 

3321 

3389 

68 

6 

3457 

3525 

3594 

3662 

3730 

3798 

3867 

3035 

4003 

4071 

68 

7 

4139 

4208 

4276 

4344 

4412 

4480 

4548 

4616 

4685 

4753 

68 

8 

4821 

4889 

4957 

5025 

6093 

5161 

5229 

5297 

5365 

6433 

68 

9 

6501 

5569 

5637 

5705 

6773 

6841 

6908 

6976 

6044 

0112 

68 

N. 

0 

1 

2 

3 

4 

5 

6 

7 

8 

e 

D. 

506 


APPENDIXES 


N. 

0 

1 

2 

5 

4 

5 

6 

7 

8 

9 

D. 

640 

806180 

806248 

806316 

806384 

806451 

806519 

806587 

806655 

806723 

806790 

68 

1 

6858 

6926 

6994 

7061 

7129 

7197 

7264 

7332 

7400 

7467 

68 

2 

7535 

7603 

7670 

7738 

7806 

7873 

7941 

8008 

8076 

8143 

68 

3 

8211 

8279 

8346 

8414 

8481 

8549 

8616 

8684 

8751 

8818 

67 

4 

8886 

8953 

9021 

9088 

9156 

9223 

9290 

9358 

9425 

9492 

67 

645 

9560 

9627 

9604 

9762 

9829 

9896 

9964 

810031 

810098 

810165 

67 

6 

810233 

810300 

810367 

810434 

810501 

810569 

810636 

0703 

0770 

0837 

67 

7 

01)04 

0071 

1039 

1106 

1173 

1240 

1307 

1374 

1441 

1508 

67 

8 

1575 

1612 

1709 

1776 

1843 

1910 

1977 

2044 

2111 

2178 

67 

9 

2243 

2312 

2379 

2445 

2512 

2579 

2646 

2713 

2780 

2847 

67 

650 

812013 

812080 

81  -SO  17 

813114 

813181 

813247 

813314 

813381 

813448 

813514 

67 

1 

3581 

3618 

3714 

3781 

3818 

3914 

3981 

4018 

4114 

4181 

67 

2 

4218 

4314 

4381 

4447 

4514 

45S1 

4617 

1714 

4780 

4847 

67 

3 

4913 

4980 

504(5 

5113 

5179 

5216 

5312 

5378 

5445 

5511 

60 

4 

5578 

5644 

5711 

5777 

5843 

5910 

5976 

6042 

6109 

6175 

6(5 

655 

6241 

6308 

6374 

6440 

6506 

6573 

6639 

6705 

6771 

6838 

66 

6 

6!XH 

6970 

7036 

7102 

71f»r 

7235 

7301 

7367 

7133 

7499 

66 

7 

7565 

7631 

7698 

7764 

7830 

78% 

7062 

8028 

8034 

8160 

66 

8 

8220 

8202 

8358 

8124 

8490 

8Vifi 

8622 

8688 

8754 

8820 

66 

9 

8885 

8951 

9017 

9083 

9149 

9215 

9281 

9346 

9412 

9478 

66 

660 

819544 

810610 

819676 

819741 

819807 

810873 

819939 

820004 

820070 

820136 

66 

1 

820P01 

820267 

8203I-53 

820399 

820  1  04 

820530 

820505 

0661 

0727 

0792 

66 

2 

0858 

0924 

0989 

1055 

1120 

1186 

1251 

1317 

1382 

1418 

6ti 

3 

lf>  I  I 

1579 

1015 

1710 

1775 

1841 

1006 

1072 

2037 

2103 

65 

4 

2168 

2233 

2299 

2364 

2430 

2495 

2560 

2626 

2691 

2756 

65 

665 

2822 

2887 

2952 

3018 

3083 

3148 

3213 

3279 

3314 

3409 

65 

6 

3474 

3539 

3605 

3670 

3735 

3800 

asfio 

3930 

39% 

4061 

65 

7 

412(5 

4191 

4256 

4321 

4386 

4451 

4516 

4581 

46  lf> 

47J1 

6,") 

8 

4776 

4841 

4906 

4971 

5036 

5101 

51  06 

5231 

5206 

5361 

6') 

0 

6426 

5491 

5556 

5621 

5686 

5751 

5815 

5SSO 

5915 

6010 

65 

670 

826075 

826140 

826204 

826269 

826334 

826399 

826464 

826528 

826593 

826658 

65 

1 

6723 

6787 

6852 

6917 

6081 

7016 

7111 

7175 

7210 

7305 

65 

2 

7309 

7434 

7499 

7563 

762S 

7692 

7757 

7821 

7886 

7951 

6r> 

3 

8015 

8080 

8144 

8209 

8273 

8338 

8402 

8467 

8r>:u 

8505 

61 

4 

8CGO 

8724 

8789 

8853 

8918 

8982 

9016 

9111 

9175 

V239 

64 

675 

930  1 

9368 

9132 

9497 

9501 

9625 

9690 

9754 

9818 

9882 

64 

6 

9947 

830011 

830075 

830139 

830201 

8302(& 

830332 

830396 

8304  (JO 

830525 

61 

7 

830589 

0653 

0717 

0781 

0845 

0909 

0973 

1037 

1102 

1166 

61 

8 

1230 

129* 

1358 

1422 

1186 

1550 

1611 

1678 

1742 

1806 

61 

9 

1870 

1934 

1998 

2062 

2126 

2189 

2253 

2317 

2381 

2415 

64 

680 

832509 

832573 

832637 

832700 

832764 

832828 

832802 

832056 

833020 

833083 

64 

1 

3147 

3'>11 

IV275 

3338 

3402 

3466 

3530 

359I-! 

3657 

3721 

64 

2 

3784 

3818 

3912 

3975 

4039 

4103 

4166 

4230 

4294 

4357 

61 

3 

4421 

4484 

4548 

4611 

4675 

4730 

4802 

4866 

40'29 

4993 

61 

4 

5056 

5120 

5183 

5247 

5310 

5373 

5437 

5500 

5564 

5627 

63 

685 

5(591 

5751 

5817 

5881 

5941 

6007 

6071 

6134 

6197 

6261 

63 

6 

6324 

6387 

645  I 

6514 

6577 

6641 

6704 

6767 

6830 

6P»4 

63 

7 

6957 

7020 

7083 

7146 

7210 

7273 

7336 

7399 

7162 

7525 

63 

8 

758K 

7652 

7715 

7778 

7811 

7904 

7967 

8030 

8093 

8156 

63 

9 

8219 

8282 

8345 

8408 

8471 

8534 

8597 

8660 

8723 

8786 

63 

690 

838849 

838912 

838975 

839038 

839101 

839164 

839227 

839289 

839352 

839115 

63 

1 

0178 

9541 

9604 

96(57 

9729 

9792 

9855 

9918 

9981 

840043 

63 

2 

840106 

840169 

840232 

840294 

840357 

840420 

840482 

840545 

840608 

0671 

63 

3 

0733 

0796 

0859 

0921 

0984 

1046 

1109 

1172 

1234 

1297 

63 

4 

1359 

1422 

1485 

1547 

1610 

1672 

1735 

1797 

1860 

1922 

63 

695 

1985 

2047 

2110 

2172 

2235 

2297 

2360 

2422 

2484 

2547 

62 

6 

2609 

2672 

2734 

2796 

2859 

2921 

2983 

3046 

3108 

3170 

62 

7 

3233 

8295 

8357 

3420 

3482 

3544 

3606 

3669 

3731 

3793 

62 

8 

8855 

3918 

3980 

4042 

4104 

4166 

4229 

4291 

4353 

4415 

62 

9 

4477 

4539 

4601 

4664 

4726 

4788 

4850 

4912 

4974 

5036 

62 

N. 

0 

1 

2 

3 

4 

5 

6 

7 

8 

9 

D. 

TABLE  OF  LOGARITHMS 


507 


N. 

0 

1 

2 

3 

4 

5 

6 

7 

8 

9 

D. 

700 

846098 

845160 

845222 

8452S4 

845346 

845408 

845470 

846532 

845594 

845656 

62 

1 

5718 

5780 

5842 

5904 

5966 

6028 

6090 

6151 

6!213 

6275 

62 

2 

6337 

6399 

6401 

6523 

6585 

6646 

6708 

6770 

6832 

6894 

62 

3 

6955 

7017 

7079 

7141 

7202 

7264 

7326 

7388 

7449 

7511 

62 

4 

7573 

7634 

7696 

7758 

7819 

7881 

7943 

8004 

8066 

8128 

62 

705 

8189 

8251 

8312 

8374 

8435 

8497 

8659 

8620 

8682 

8743 

62 

6 

8805 

880)6 

8928 

8989 

9051 

9112 

9174 

9235 

9297 

9358 

61 

7 

9419 

9481 

9542 

9604 

9665 

9720 

9788 

9819 

9911 

9972 

61 

8 

850033 

850095 

850156 

850217 

850279 

850340 

850401 

850462 

850524 

850585 

61 

9 

0646 

0707 

0709 

0830 

0891 

0952 

1014 

1075 

1136 

1197 

61 

710 

851258 

851320 

851381 

851442 

851503 

851564 

851625 

851686 

851747 

851809 

61 

1 

1870 

1931 

1992 

2053 

2114 

2175 

2236 

2297 

2358 

2419 

61 

2 

2480 

2541 

2602 

26(53 

2724 

2785 

2840 

2907 

2LHJS 

3029 

61 

3 

3090 

3150 

3211 

3272 

3333 

3394 

3455 

3516 

3577 

S037 

61 

4 

8698 

3759 

3820 

3881 

3941 

4002 

4063 

4124 

4185 

4245 

61 

715 

4306 

4367 

4428 

4488 

4549 

4610 

4670 

4731 

4792 

4852 

61 

6 

4913 

4974 

5034 

6095 

5156 

5216 

5277 

6337 

5398 

6459 

61 

7 

5519 

6580 

5640 

5701 

5701 

5822 

5882 

5913 

(5003 

(5064 

61 

8 

6124 

6185 

6245 

6306 

6306 

6427 

6487 

6548 

6008 

6008 

60 

9 

6729 

6789 

6850 

6910 

6970 

7031 

7091 

7152 

7212 

7272 

60 

720 

857332 

857393 

857453 

857513 

857574 

857634 

857694 

857755 

857815 

857875 

60 

1 

7935 

7995 

8056 

8116 

8176 

8236 

8297 

8357 

8-117 

8477 

60 

2 

8537 

8597 

8657 

8718 

8778 

bS38 

8Sl>8 

8958 

9018 

9078 

60 

3 

9138 

9198 

9258 

9318 

9379 

9  439 

9-199 

9559 

9019 

9679 

60 

4 

9739 

9799 

9859 

9918 

9978 

860038 

860098 

860158 

860218 

860278 

GO 

725 

860338 

860398 

860458 

860518 

860578 

0637 

0697 

0757 

0817 

0877 

60 

6 

0937 

OU96 

1056 

1116 

1176 

12.i6 

1295 

1355 

1415 

1475 

60 

7 

1534 

1594 

1664 

1714 

1773 

1833 

1893 

1952 

2012 

2072 

60 

8 

2131 

2191 

2251 

2310 

2,170 

2  130 

21M> 

2519 

2608 

20(58 

60 

9 

2728 

2787 

2847 

2906 

2966 

3025 

3085 

3144 

3204 

3263 

60 

730 

863323 

86.3382 

863442 

863501 

863561 

863620 

863680 

863739 

863799 

863858 

59 

1 

3917 

8977 

40IJ6 

4096 

4155 

4214 

4274 

4333 

4392 

4452 

69 

2 

4511 

4.070 

4  030 

4689 

4748 

4808 

4867 

4920 

4985 

5045 

5» 

3 

5104 

6163 

6222 

5282 

5341 

5100 

515') 

5519 

5578 

5037 

69 

4 

5696 

5755 

5814 

5874 

6933 

5992 

6051 

6110 

6169 

6228 

69 

735 

6287 

6346 

6405 

6165 

6521 

6583 

6642 

6701 

6760 

6819 

59 

6 

6878 

6937 

6990 

7055 

7114 

7173 

7232 

7291 

7350 

7409 

69 

7 

71(57 

752G 

75K5 

7014 

770.'* 

7702 

7H21 

78HO 

7939 

7998 

59 

8 

8056 

8115 

8174 

8233 

8292 

8350 

8409 

8408 

8527 

8580 

69 

9 

8644 

8703 

8702 

8821 

8879 

8938 

8997 

9056 

9114 

9m 

59 

740 

869232 

869290 

869349 

869408 

869166 

869525 

869584 

869642 

869701 

869760 

69 

1 

9818 

9877 

9935 

9994 

870053 

870111 

870170 

870228 

870287 

870345 

69 

2 

870404 

870102 

870521 

870579 

OG38 

0696 

0755 

(J813 

0872 

O'JUO 

68 

3 

09b9 

1017 

110(5 

1164 

1223 

1281 

1339 

1398 

1456 

1515 

68 

4 

1573 

1G31 

1690 

1748 

1800 

1865 

1923 

1981 

2040 

2098 

68 

745 

2156 

2215 

2273 

2331 

2389 

2448 

2506 

2564 

2622 

2681 

68 

6 

27,19 

2797 

2855 

2913 

2972 

3030 

3088 

3146 

32O4 

3262 

68 

7 

3321 

3379 

3437 

34'JG 

3553 

3611 

3«i9 

3727 

3785 

3844 

68 

8 

3902 

3900 

4018 

4076 

4134 

4192 

4250 

4308 

4366 

4421 

58 

9 

4482 

4510 

4598 

4656 

4714 

4772 

4830 

4888 

4945 

6003 

58 

750 

875061 

875119 

875177 

875235 

875293 

875351 

875409 

875466 

875524 

875582 

68 

1 

5640 

5G98 

5756 

5813 

5871 

5929 

5987 

6045 

6102 

6160 

68 

2 

6218 

6276 

6333 

6391 

6419 

6507 

6564 

6622 

6680 

6737 

58 

3 

6795 

6853 

6910 

6968 

7026 

7083 

7141 

7199 

7256 

7314 

68 

4 

7371 

7429 

7487 

7544 

7602 

7659 

7717 

7774 

7832 

7889 

68 

755 

7947 

8004 

8062 

8119 

8177 

8234 

8292 

8349 

8407 

8464 

67 

6 

8522 

8579 

8637 

8694 

8752 

8809 

8866 

8924 

8981 

9039 

57 

7 

9096 

9153 

9211 

9268 

9325 

9383 

9440 

9497 

9565 

9612 

67 

8 

9669 

9726 

9784 

9841 

9898 

9956 

880013 

880070 

880127 

880185 

57 

9 

880242 

880299 

880356 

880413 

880471 

880528 

0585 

0642 

0699 

0750 

67 

N. 

0 

1 

2 

3 

4 

6 

6 

7 

8 

9 

D. 

508 


APPENDIXES 


N. 

0 

1 

2 

3 

4 

6 

6 

7 

8 

9 

D. 

760 

880814 

880871 

880928 

880985 

881042 

881099 

881156 

881213 

881271 

881328 

57 

1 

1385 

1442 

1499 

1556 

1613 

1670 

1727 

1784 

1841 

1898 

57 

2 

1955 

2012 

2069 

2126 

2183 

2240 

2297 

2354 

2411 

2468 

57 

3 

2525 

2581 

2638 

2695 

2752 

2809 

2866 

2923 

2980 

3037 

57 

4 

3093 

3150 

3207 

3264 

3321 

3377 

3434 

3491 

3548 

3605 

57 

765 

3661 

3718 

3775 

3832 

3888 

3945 

4002 

4059 

4115 

4172 

57 

6 

4229 

4285 

4342 

4399 

4455 

4.')  12 

4569 

4625 

4682 

4739 

57 

7 

4795 

4852 

4909 

4965 

5022 

5078 

5135 

5192 

5248 

5305 

57 

8 

5361 

5418 

5474 

5531 

5587 

5644 

5700 

5757 

5813 

5870 

57 

9 

5926 

5983 

6039 

6096 

6152 

6209 

6265 

6321 

6378 

6434 

56 

770 

886491 

886547 

886604 

886660 

886716 

886773 

886829 

886885 

886942 

886998 

56 

1 

7054 

7111 

7167 

7223 

7280 

7336 

7392 

7449 

7505 

7561 

5(5 

2 

7617 

7674 

7730 

7786 

7842 

7898 

7955 

8011 

8067 

8123 

5<5 

3 

8179 

8236 

8202 

8348 

8404 

84fiO 

8516 

8573 

8629 

8685 

56 

4 

8741 

8797 

8853 

8909 

8965 

9021 

9077 

9134 

9190 

9246 

56 

775 

9302 

9358 

9414 

9470 

9526 

9582 

9638 

9604 

9750 

0806 

56 

6 

98«£ 

9918 

9974 

890030 

890086 

800141 

8901f>7 

890253 

890.10!) 

890365 

56 

7 

890421 

890477 

890533 

0589 

00-15 

0700 

07  50 

0812 

0868 

0024 

56 

8 

0980 

1035 

1091 

1117 

1203 

1  •>"){> 

1314 

1870 

1426 

1482 

56 

9 

1537 

1593 

164U 

1705 

1700 

1816 

1872 

1928 

1983 

2039 

56 

780 

892005 

892150 

892206 

892262 

892317 

S9237& 

892429 

892184 

892.540 

892595 

56 

1 

2651 

2707 

2702 

281H 

2873 

2929 

2!  18.") 

3010 

3096 

3151 

56 

2 

3207 

3262 

3318 

3373 

3129 

3184 

3540 

3505 

3651 

3706 

56 

3 

3702 

3817 

3873 

3928 

3<)S4 

4039 

4094 

4150 

42(>.> 

4261 

55 

4 

4316 

4371 

4427 

4482 

4538 

4593 

4648 

4704 

4759 

4814 

55 

786 

4870 

4925 

4980 

503fi 

5091 

5146 

5201 

5257 

5312 

5367 

55 

6 

6423 

5478 

5533 

5588 

5644 

5699 

5754 

5809 

5864 

5920 

65 

7 

5975 

6030 

6085 

6140 

6195 

6251 

C306 

63(U 

6416 

6471 

55 

8 

6526 

6581 

6636 

66<>2 

6747 

6802 

6857 

6912 

69f,7 

7022 

55 

9 

7077 

7132 

7187 

7242 

7297 

7332 

7407 

7462 

7517 

7372 

55 

790 

897627 

897682 

897737 

897792 

897847 

897902 

897957 

80S012 

898067 

898122 

55 

1 

8176 

8231 

8286 

8341 

8396 

8451 

850(> 

8561 

8015 

8670 

55 

2 

8725 

8780 

8835 

8890 

8944 

891)0 

9054 

910!) 

9164 

9218 

55 

3 

9273 

9328 

9383 

9437 

9492 

9547 

9602 

9656 

9711 

9766 

55 

4 

9821 

9875 

9930 

9985 

900039 

900094 

900149 

900203 

900258 

900312 

65 

795 

900367 

900422 

900476 

900531 

0586 

0640 

0695 

0749 

0804 

0859 

55 

6 

0913 

0968 

1022 

1077 

1131 

1186 

1240 

1295 

1349 

1404 

55 

7 

1458 

1513 

1567 

1622 

1(576 

1731 

1785 

1840 

1894 

1948 

54 

8 

2003 

2057 

2112 

2166 

2221 

2275 

2329 

2384 

2438 

24fl2 

64 

9 

2547 

2601 

2655 

2710 

2764 

2818 

2873 

2927 

2981 

3036 

54 

800 

903090 

903144 

903199 

903253 

903307 

903361 

903416 

903470 

903524 

903578 

54 

1 

3633 

3687 

3741 

3795 

38-19 

3904 

3958 

4012 

4066 

4120 

54 

2 

4174 

4229 

4283 

4337 

4391 

4445 

4499 

4553 

4007 

4661 

54 

3 

4716 

4770 

4824 

4878 

4932 

4986 

5040 

5094 

5148 

5202 

54 

4 

5250 

5810 

5364 

5418 

5472 

5526 

6580 

5634 

5688 

5742 

54 

805 

5796 

5850 

5904 

5958 

6012 

6066 

6119 

6173 

6227 

6281 

54 

6 

6335 

6389 

6443 

6497 

6551 

6004 

6658 

6712 

6766 

6820 

51 

7 

6874 

6927 

6981 

7035 

7089 

7143 

7196 

7250 

7304 

7358 

54 

8 

7411 

7465 

7519 

7573 

7626 

7680 

7734 

7787 

7841 

7895 

54 

9 

7949 

8002 

8056 

8110 

8163 

8217 

8270 

8324 

8378 

8431 

54 

810 

908485 

908539 

908592 

908646 

908699 

908753 

908807 

908860 

908914 

908967 

54 

1 

9021 

9074 

9128 

9181 

9235 

9289 

9342 

9396 

9449 

9503 

54 

2 

9556 

9610 

9663 

9716 

9770 

9823 

9877 

9930 

9984 

910037 

53 

3 

910091 

910144 

910197 

910251 

910304 

910358 

910411 

910464 

910518 

0571 

53 

4 

0624 

0678 

0731 

0784 

0838 

0891 

0944 

0998 

1051 

1104 

53 

815 

1168 

1211 

1264 

1317 

1371 

1424 

1477 

1530 

1584 

1637 

53 

6 

1690 

1743 

1797 

1850 

1903 

1956 

2009 

2063 

2116 

2169 

53 

7 

2222 

2275 

2328 

2381 

2435 

2488 

2541 

2594 

2647 

2700 

53 

8 

2753 

2806 

2859 

2913 

2966 

3019 

3072 

3125 

3178 

3231 

53 

9 

3284 

8337 

3390 

3443 

3496 

S549 

3602 

3655 

3708 

3761 

53 

K. 

0 

1 

2 

3 

4 

5 

6 

7 

8 

9 

D. 

TABLE  OF  LOGARITHMS 


509 


N. 

0 

1 

2 

3 

4 

5 

6 

7 

8 

9 

D. 

820 

913814 

913867 

913920 

913973 

914026 

914079 

914132 

914184 

314237 

914290 

53 

1 

4.m 

4396 

4419 

4502 

4555 

4008 

4660 

4713 

4766 

4819 

53 

2 

4872 

4925 

4977 

5030 

5083 

51  86 

5189 

5211 

5294 

6347 

63 

3 

5400 

5453 

5505 

6558 

5611 

5664 

5716 

5769 

5822 

5875 

53 

4 

5927 

5080 

6033 

6085 

6188 

6191 

6243 

6296 

6349 

6401 

53 

825 

6454 

6507 

6559 

6612 

6664 

6717 

6770 

6822 

6875 

6927 

53 

6 

6980 

7033 

7oar> 

7138 

7190 

7243 

7295 

7348 

7400 

7453 

53 

7 

7506 

7,">8 

7611 

7663 

7716 

7768 

7820 

7873 

7025 

7978 

52 

8 

8030 

8083 

8135 

8188 

8210 

8293 

8315 

8397 

8450 

8502 

52 

9 

8555 

8607 

8659 

8712 

8764 

8816 

8869 

8921 

8973 

9026 

52 

830 

919078 

919130 

919183 

919235 

919287 

919340 

919?92 

919444 

910496 

919549 

52 

1 

9001 

9653 

9706 

9758 

9810 

9862 

9914 

9967 

920019 

920071 

5'> 

2 

9201  23 

920176 

920228 

9202SO 

920332 

920384 

920136 

920489 

0511 

0593 

52 

3 

0645 

0697 

0749 

OS01 

0853 

0906 

0958 

1010 

1062 

1114 

52 

i 

1166 

1218 

1270 

1322 

1374 

1426 

1478 

1530 

1582 

1634 

52 

835 

1686 

1738 

1790 

1«I2 

1894 

1946 

1998 

2050 

2102 

2154 

52 

6 

2206 

2258 

2310 

2362 

2111 

2466 

2518 

2570 

2622 

2674 

r>2 

7 

2725 

2777 

2829 

2881 

2933 

2985 

3037 

3089 

3140 

3192 

62 

8 

3214 

3296 

3348 

3399 

3451 

3503 

3555 

3607 

3658 

3710 

52 

0 

3762 

3811 

3865 

3917 

3969 

4021 

4072 

4124 

4176 

4228 

52 

840 

921279 

924331 

924383 

924434 

924186 

924538 

924589 

924641 

924693 

924744 

52 

1 

4706 

4818 

1800 

4(T>1 

5003 

5054 

5106 

5157 

5201) 

5261 

52 

2 

5312 

5364 

5415 

5467 

5518 

5570 

5621 

5673 

5725 

5776 

52 

3 

5828 

5879 

5931 

5982 

6034 

6085 

6137 

6188 

6240 

6291 

61 

4 

6342 

6394 

6445 

6497 

6548 

6600 

6651 

6702 

6754 

6805 

61 

845 

6857 

6908 

6959 

7011 

7062 

7114 

7165 

7216 

7268 

7319 

61 

6 

7370 

7422 

7473 

7524 

7576 

7627 

7678 

7730 

7781 

7832 

51 

7 

7883 

7035 

7086 

8037 

808H 

8140 

8191 

8242 

8293 

8345 

51 

8 

83f>6 

8417 

8198 

8510 

8601 

8652 

8703 

8754 

8805 

8857 

51 

9 

8908 

8959 

9010 

9061 

9112 

9163 

9215 

9266 

9317 

9368 

51 

850 

920410 

92')  170 

929521 

020572 

9?9623 

929071 

929725 

929776 

929827 

929879 

51 

1 

9930 

9981 

930032 

930083 

930134 

D301H5 

930236 

930287 

930338 

930389 

51 

2 

930440 

930101 

0542 

0.712 

0613 

060  1 

0715 

0796 

0847 

0898 

51 

3 

0049 

1000 

1051 

1102 

1153 

1201 

1254 

1305 

1356 

1407 

51 

4 

i  ir>8 

1509 

1500 

1610 

1661 

1712 

1763 

1814 

1865 

1915 

61 

855 

1966 

2017 

2068 

2118 

2169 

2220 

2271 

?322 

2372 

2423 

51 

6 

2474 

2V21 

257r> 

2626 

2677 

2727 

277S 

282!) 

2879 

2930 

51 

7 

2981 

3031 

3082 

31  33 

3183 

323  1 

3285 

3335 

3386 

3437 

61 

8 

3487 

3538 

3589 

3639 

3690 

3740 

3791 

3841 

3892 

3913 

61 

9 

3993 

4014 

4094 

4145 

4195 

4246 

4296 

4347 

4397 

4418 

51 

860 

934498 

934549 

934599 

934650 

934700 

931751 

934801 

934852 

934902 

931953 

50 

1 

5003 

5O54 

5101 

5154 

5205 

5255 

5306 

5350 

5406 

5  157 

50 

2 

5507 

5558 

5608 

5658 

5709 

5759 

5809 

5860 

5910 

5960 

50 

3 

roil 

6061 

6111 

6162 

6212 

6262 

6313 

6363 

6413 

6163 

50 

4 

6514 

6564 

6614 

6665 

6715 

G705 

6815 

6865 

6916 

6966 

60 

865 

7016 

7066 

7117 

7167 

7217 

7267 

7317 

7367 

7418 

7468 

50 

6 

7518 

7568 

7618 

7668 

7718 

7760 

781  9 

7860 

791!) 

79(11) 

50 

7 

8019 

8069 

8110 

81  M 

821  <> 

8269 

8320 

8370 

8120 

8470 

60 

8 

8520 

8570 

8620 

8670 

8720 

8770 

8820 

8H70 

8920 

8970 

50 

9 

9020 

9070 

9120 

9170 

9220 

9270 

9320 

9369 

9419 

9169 

50 

870 

OWU9 

939560 

939619 

939669 

930719 

939769 

930819 

939869 

939918 

1)39968 

50 

1 

940018 

940068 

940118 

940168 

910218 

940267 

940317 

940367 

940417 

940167 

50 

2 

0516 

0566 

0616 

0666 

0716 

0765 

0815 

0865 

0015 

0964 

50 

3 

10H 

1064 

1114 

1163 

1213 

1263 

1313 

1362 

1112 

1462 

60 

4 

1511 

!.;<>] 

1611 

1660 

1710 

1760 

1809 

1859 

1909 

1958 

50 

875 

2008 

2058 

2107 

2157 

2207 

2256 

2306 

2355 

2405 

2455 

50 

6 

2504 

2554 

2603 

2653 

2702 

2752 

2801 

2851 

2901 

2950 

50 

7 

3000 

3049 

3099 

3148 

3198 

3247 

3297 

3346 

3306 

3445 

49 

8 

3495 

3544 

3593 

3643 

3692 

3742 

3791 

3841 

3890 

3939 

49 

9 

3989 

4038 

4088 

4137 

4186 

4236 

4285 

4335 

4384 

4433 

49 

N. 

0 

1 

2 

3 

4 

5 

6 

7 

8 

9 

D. 

510 


APPENDIXES 


N. 

0 

1 

2 

3 

4 

5 

6 

7 

8 

9 

D. 

880 

944483 

944532 

944581 

944631 

944680 

944729 

944779 

944828 

944877 

944927 

49 

1 

4976 

5025 

5074 

5124 

5173 

5222 

5272 

5321 

5370 

5419 

49 

2 

5169 

5518 

5567 

5616 

56(>5 

5713 

5701 

5813 

5862 

5912 

49 

3 

5961 

6010 

6030 

6108 

61.37 

6207 

6256 

6305 

6354 

6403 

49 

4 

6452 

6501 

6551 

6600 

6649 

6t)')8 

6747 

6796 

6845 

685)4 

49 

885 

6913 

6992 

7011 

7000 

7140 

7180 

7238 

7287 

7336 

7385 

49 

6 

7434 

7483 

7532 

7581 

76.W 

7b7() 

7728 

7777 

7826 

7875 

49 

7 

7924 

7973 

8022 

8070 

8119 

8168 

8217 

8266 

8315 

8364 

40 

8 

8413 

8462 

8511 

8500 

8609 

8057 

8706 

8755 

8804 

8853 

49 

9 

8902 

8951 

8999 

9048 

9097 

9146 

9195 

92  H 

9292 

9341 

49 

890 

949390 

949139 

949488 

910536 

049585 

940634 

940683 

940731 

949780 

949829 

49 

1 

9878 

9926 

5)975 

950024 

950073 

950121 

950170 

950219 

950267 

950316 

49 

2 

9503C55 

95041  4 

950162 

0511 

0560 

0608 

0657 

0706 

0754 

0803 

49 

3 

0851 

0900 

0919 

0907 

1046 

1005 

1143 

1192 

1240 

1289 

49 

4 

1338 

1386 

1435 

1183 

1532 

1580 

1629 

1677 

1726 

1775 

49 

895 

1823 

1872 

1920 

1960 

2017 

2066 

2114 

2163 

2211 

2260 

48 

6 

2308 

2356 

2405 

2453 

2.302 

25,30 

2399 

2647 

2696 

2714 

48 

7 

2792 

2811 

2880 

293* 

29*6 

303  1 

30*3 

3131 

3180 

3228 

48 

8 

3276 

3325 

3373 

3121 

3(70 

3318 

3506 

3615 

3663 

3711 

48 

9 

3760 

3808 

3856 

3905 

3953 

4001 

4049 

4098 

4146 

4194 

48 

900 

954243 

954291 

954339 

954387 

954135 

954484 

954532 

954580 

054628 

951677 

48 

1 

4725 

4773 

4821 

486!) 

4918 

49(>0 

.3014 

5002 

5110 

5158 

48 

2 

5207 

5255 

5303 

53.)1 

539<) 

5147 

5405 

5543 

530*2 

5()10 

4* 

3 

5(588 

673(5 

57H  I 

5832 

5880 

59'28 

59  7( 

002  1 

6072 

6120 

48 

4 

6168 

6216 

6265 

6313 

6361 

640!) 

6457 

6305 

6553 

6001 

48 

905 

6640 

6697 

6745 

6703 

6810 

6888 

6936 

698  1 

7032 

70*0 

48 

6 

7128 

7176 

7224 

7272 

7320 

7368 

74  16 

7464 

7512 

73.3!) 

48 

7 

7607 

7655 

7703 

7751 

7790 

7*17 

7w<)j 

7')  12 

7'J'K) 

8038 

48 

8 

8086 

81.  -14 

8181 

822!) 

8-277 

8325 

837,i 

8121 

8408 

8516 

48 

9 

8561 

8612 

8059 

8707 

8755 

8803 

fe*50 

8808 

8946 

8994 

48 

910 

959041 

959089 

959137 

950185 

059232 

939280 

5)59328 

959375 

950123 

959471 

48 

1 

9518 

9.3(>(> 

9(514 

!)<>()] 

9709 

9757 

9*04 

9852 

91*  )0 

9917 

4* 

2 

9995 

960012 

960000 

9601,-!* 

960185 

5)(>02,-!3 

9(>02*1 

9()0.-2M 

960376 

9601  '23 

48 

3 

960171 

05  1  * 

0566 

0613 

0(561 

0709 

0750 

0*04 

0*31 

0899 

48 

4 

0946 

0994 

1041 

10*9 

1136 

11*1 

1231 

1279 

1326 

1374 

48 

915 

1421 

1460 

1516 

1563 

1611 

1658 

1706 

1733 

ISO! 

1848 

47 

6 

18i»5 

1913 

195JO 

2038 

20*5 

2132 

21*0 

2227 

2275 

2322 

47 

7 

2361) 

2417 

2  464 

•2511 

255<> 

2<>0h 

2(>"i3 

2701 

2748 

2795 

47 

8 

28  13 

2H90 

2937 

25)85 

3032 

307!) 

3126 

3171 

3221 

3268 

47 

9 

3316 

3363 

3410 

3457 

3504 

3552 

3599 

3646 

3(593 

3741 

47 

920 

963788 

963835 

963882 

963929 

963977 

964024 

964071 

964118 

964165 

964212 

47 

1 

42(50 

4307 

4354 

4401 

4448 

44')f> 

451-2 

15'M> 

46.17 

4GM 

47 

2 

4731 

4778 

4*23 

4*72 

4919 

45)06 

5013 

5061 

5108 

5155 

47 

3 

5202 

5249 

62% 

5343 

53'K) 

5437 

518  4 

5.3.U 

5578 

5625 

47 

4 

5672 

5719 

5766 

5813 

5*00 

5907 

5954 

6001 

6018 

6095 

47 

925 

6112 

6180 

6236 

62S3 

6320 

6376 

6123 

6470 

6517 

6564 

47 

6 

6611 

6(>58 

6705 

6752 

67!)!) 

(>M5 

6*92 

6i)3!> 

698(5 

7033 

47 

7 

70*0 

71'27 

7173 

7  '2  20 

7*267 

7314 

7361 

7408 

74.34 

7501 

47 

8 

7548 

7395 

7042 

7(5*8 

7733 

77*? 

7*2') 

7875 

7922 

7969 

47 

9 

8016 

800'2 

8109 

8156 

8203 

*'24!) 

8296 

8313 

8390 

8436 

47 

930 

968483 

968530 

96857(5 

96*T>23 

96S670 

968716 

968763 

968810 

968*56 

968903 

47 

1 

8950 

8996 

90  IH 

90590 

9136 

5J183 

92'29 

9276 

9323 

9369 

47 

2 

9416 

9163 

9.300 

95.36 

9602 

9619 

9151)3 

9742 

9789 

9M5 

47 

3 

9882 

9928 

9975 

970021 

970068 

970114 

970101 

970207 

970254 

970300 

47 

4 

9703  17 

970393 

970440 

0186 

0533 

0579 

0026 

0672 

0719 

0765 

46 

935 

0812 

0858 

0004 

0051 

0007 

1044 

1000 

1137 

1183 

1220 

46 

6 

1276 

1322 

1360 

1413 

1461 

1508 

1551 

1601 

1647 

1693 

46 

7 

1740 

1786 

1832 

1870 

1025 

1071 

2018 

2064 

2110 

2157 

46 

8 

2203 

2249 

2205 

2342 

2388 

2434 

2481 

2527 

2573 

2619 

46 

9 

2<J66 

2712 

2758 

2804 

2851 

2897 

2943 

2989 

3035 

3082 

46 

N, 

0 

1 

2 

3 

4 

5 

6 

7 

8 

9 

D. 

TABLE  OF  LOGARITHMS 


511 


N. 

0 

1 

2 

3 

4 

5 

6 

7 

8 

9 

D. 

940 

973128 

973174 

973220 

973266 

973313 

973359 

973405 

973451 

973497 

973543 

46 

1 

3590 

3636 

3682 

3728 

3774 

3820 

3866 

3913 

3959 

4005 

46 

2 

4051 

4097 

4143 

4181) 

4235 

4281 

4327 

4374 

4420 

4466 

46 

3 

4512 

4558 

4604 

4650 

4696 

4742 

4788 

4834 

48SO 

4D26 

46 

4 

4972 

5018 

5064 

5110 

6156 

5202 

5248 

5294 

5340 

5386 

46 

945 

5432 

5478 

5524 

5570 

5616 

5662 

5707 

5753 

5799 

5845 

46 

6 

5891 

5937 

5983 

6029 

6075 

6121 

6167 

6212 

6268 

6304 

46 

7 

6350 

6396 

6442 

6488 

6533 

6579 

6625 

6671 

6717 

6763 

46 

8 

6808 

6854 

6900 

6946 

6992 

7037 

7083 

7129 

7175 

7220 

46 

9 

7266 

7312 

7358 

7408 

7449 

7495 

7541 

7586 

7632 

7678 

46 

950 

977724 

977769 

977815 

977861 

977906 

977952 

977998 

978013 

978089 

9781  35 

46 

1 

8181 

8226 

8272 

8317 

8363 

8409 

8454 

8500 

8546 

8591 

46 

2 

8637 

8683 

8728 

8774 

8819 

8865 

8911 

8956 

9(H)2 

9047 

46 

3 

9093 

9138 

9184 

9230 

9275 

9321 

9366 

9412 

9457 

9503 

46 

4 

9548 

9594 

9639 

9685 

9730 

9776 

9821 

9867 

9912 

9958 

46 

955 

980003 

980049 

980094 

980140 

980185 

980231 

980270 

980322 

980367 

980412 

45 

6 

0458 

0503 

0549 

0591 

0640 

0685 

0730 

0776 

0821 

0867 

45 

7 

0912 

0957 

1003 

1048 

1093 

1139 

11S4 

1229 

1275 

1320 

4 

8 

1366 

1411 

1  456 

1501 

i:>47 

1592 

1637 

1683 

1728 

1773 

4 

9 

1819 

1864 

1909 

1954 

2000 

2045 

2090 

2135 

2181 

2226 

4 

960 

982271 

982316 

982362 

982407 

982152 

9S2497 

982543 

982588 

982633 

982678 

4 

1 

2723 

2769 

2hl4 

2859 

290  1 

29  1<) 

2994 

3010 

3085 

SI  30 

4 

2 

3175 

3220 

3265 

3310 

3356 

3401 

3446 

3491 

3536 

35S1 

3 

3626 

3671 

3716 

3762 

3807 

.iS")2 

3h')7 

3942 

3987 

4032 

4 

4077 

4122 

4167 

4212 

4257 

4302 

4347 

4392 

4437 

4482 

965 

4527 

4572 

4617 

4662 

4707 

4752 

4797 

4842 

4887 

4932 

6 

4977 

5022 

5067 

5112 

5157 

5202 

5247 

5292 

5337 

5382 

7 

5426 

5471 

5516 

5561 

5606 

5651 

5696 

5711 

57hfl 

5H30 

8 

5875 

5920 

5965 

6010 

6055 

(.100 

6144 

(llcS'J 

6°34 

C279 

9 

6324 

6369 

6413 

6458 

6503 

h;>18 

f.593 

WW7 

(5682 

6727 

970 

986772 

986817 

986861 

986906 

986951 

98(W6 

987010 

987085 

987130 

987175 

4 

1 

7219 

7264 

7309 

7353 

7398 

7443 

7188 

75H2 

7577 

7622 

4 

2 

7666 

7711 

7756 

7800 

7845 

7HIK) 

793  1 

7(>7() 

N)2^ 

80(58 

4 

3 

8113 

8157 

8202 

8247 

8291 

S336 

835^1 

8425 

8170 

85  11 

4 

4 

8559 

8604 

8648 

8693 

8737 

8782 

8826 

8871 

8916 

8960 

4 

975 

9005 

9049 

9094 

9138 

9183 

9227 

92  <  2 

9316 

9361 

9405 

45 

6 

94.50 

9194 

9539 

95M 

9628 

9(572 

9717 

9761 

9806 

9S50 

44 

7 

9895 

9939 

9983 

99W2S 

9(H)072 

<)<)01  J7 

990161 

990206 

990250 

99029  4 

44 

8 

90033!) 

990383 

990428 

0172 

0516 

0561 

OG05 

0650 

0694 

0738 

44 

9 

0783 

0827 

0871 

0916 

09(>0 

1001 

1019 

1093 

1137 

1182 

44 

980 

991226 

991270 

991315 

991359 

991403 

991448 

991492 

991536 

991580 

U91625 

44 

1 

1  669 

1713 

1  758 

1802 

1846 

1890 

1935 

1979 

2023 

2067 

44 

2 

2111 

2156 

2200 

2244 

2288 

2333 

2377 

2121 

2  465 

2509 

44 

3 

2554 

2598 

2612 

2686 

2730 

2774 

2819 

2863 

2907 

2951 

44 

4 

2995 

3039 

3083 

3127 

3172 

3216 

3260 

3304 

3348 

8392 

44 

985 

3436 

3480 

3524 

3568 

3613 

3657 

3701 

3745 

3789 

3833 

44  J 

6 

3877 

3921 

3965 

4009 

4053 

4097 

4141 

41H5 

4229 

4278 

44  j 

7 

4317 

4361 

4405 

4449 

4193 

4537 

4581 

4625 

40W) 

4713 

44 

8 

4757 

4801 

4845 

4889 

4933 

4977 

5021 

fX)65 

5108 

5152 

44 

9 

5196 

5240 

5284 

5328 

5372 

5416 

5160 

5504 

5517 

5591 

44 

990 

995635 

995679 

995723 

995767 

995811 

995854 

995898 

995942 

995986 

996030 

44 

1 

6074 

6117 

6161 

(.205 

6219 

6293 

(>337 

()'.MO 

6124 

6468 

44 

2 

6512 

6555 

6599 

6643 

6687 

6731 

6774 

(»H1S 

6H62 

6WO 

44 

3 

6949 

6993 

7037 

7080 

7124 

7168 

7212 

7255 

7299 

7343 

44 

4 

7386 

7430 

7474 

7517 

7561 

7605 

7648 

7692 

7736 

777'J 

44 

995 

7823 

7867 

7910 

7954 

7998 

8041 

8085 

8129 

8172 

8216 

44 

6 

8259 

8303 

8347 

8390 

8134 

8477 

8521 

8564 

H(X)8 

8652 

4* 

7 

8695 

8739 

8782 

8826 

8869 

8913 

8956 

9000 

9043 

9087 

44 

8 

9131 

9174 

9218 

9261 

9305 

9348 

9392 

9435 

9479 

9522 

44 

9 

9565 

9609 

9652 

9696 

9739 

9783 

9826 

9870 

9913 

9957 

43 

N. 

0 

1 

2 

3 

4 

5 

6 

7 

8 

9 

D. 

512 


APPENDIXES 


Table  2 
COMPOUND  AMOUNT  OF  1 

s  =  (1  +  i)n 


n 

H% 

H% 

Jf4% 

H% 

«% 

1 

1.0012  5000 

.0025  0000 

1.0029  1667 

1.0033  3333 

1.0037  5000 

2 

1.0025  0156 

.0050  0625 

1.0058  4184 

1.0066  7778 

1  0075  1406 

3 

1.0037  5469 

0075  1877 

1.0087  7555 

1.0100  3337 

1.0112  9224 

4 

1.0050  0938 

.0100  3756 

1.0117  1781 

1  0134  0015 

1.0150  8459 

5 

1.0062  6564 

.0125  6266 

1.0146  6865 

1.0167  7815 

1  0188  9115 

6 

1.0075  2348 

.0150  9406 

1.0176  2810 

1.0201  6740 

1  0227  1200 

7 

1.0087  8288 

.0176  3180 

1.0205  9618 

1.0235  6797 

1.0265  4717 

8 

1.0100  4386 

.0201  7588 

1  0235  7292 

1.0269  7986 

1  0303  9672 

9 

1.0113  0641 

.0227  2632 

1.0265  5834 

1.0304  0313 

1.0342  6070 

10 

1.0125  7055 

.0252  U13 

1.0295  5247 

1.0338  3780 

1.0381  3918 

11 

1.0138  3626 

1.0278  4634 

1.0325  5533 

1.0372  8393 

1  0420  3220 

12 

1.0151  0356 

1.0304  1596 

0355  6695 

1.0407  4154 

1  0459  3983 

13 

1.0163  7244 

1.0329  9200 

.0385  8736 

1.0442  1068 

1.0498  6210 

14 

1.0176  4290 

1.0355  7448 

.0416  1657 

1.0476  9138 

1  0537  9908 

15 

1  0189  1495 

1.0381  6341 

.0446  5462 

1.0511  8369 

1.0577  5083 

16 

1.0201  8860 

1  0407  5882 

.0477  0153 

1  0546  8763 

1  0617  1739 

17 

1.0214  6383 

1.0433  6072 

.0507  5732 

1.0582  0326 

1  0656  9883 

18 

1  0227  4066 

1.0459  6912 

.0538  2203 

1.0617  3060 

1  0696  9521 

19 

1.0240  1909 

1.0485  8404 

.0568  9568 

1.0652  6971 

1  0737  0656 

20 

1.0252  9911 

1.0512  0550 

.0599  7829 

1.0688  2060 

1  0777  3296 

21 

1.0265  8074 

1  0538  3352 

1.0630  6990 

I  0723  8334 

1  0817  7446 

22 

1.0278  6396 

1.0564  6810 

1  0661  7052 

1  0759  5795 

1  0858  31  H 

23 

1.0291  4879 

1  0591  0927 

1.0692  8018 

1  0795  4448 

1  0899  0298 

24 

1  0304  3523 

1  0617  5704 

1.0723  9891 

1  0831  429(5 

1  0939  9012 

25 

1.0317  2327 

1.0644  1144 

1  0755  2674 

1.0867  5344 

1.0980  9258 

26 

1.0330  1293 

1.0670  7247 

1.0786  0370 

1  0903  7595 

1  1022  1043 

27 

1.0343  0419 

0697  4015 

1  0818  0980 

1  0940  1053 

1  1063  4372 

28 

1  0355  9707 

0724  1450 

1  0849  6508 

1  0976  5724 

1  1104  9251 

29 

1.0368  9157 

0750  9553 

1  0881  2956 

1  1013  1609 

1  1146  5685 

30 

1.0381  8768 

0777  8327 

1.0913  0327 

1  1049  8715 

1  1188  3682 

31 

1.0394  8542 

1.0804  7773 

1.0944  8624 

1.1086  7044 

1.1230  3245 

32 

1  0407  8478 

1.0831  7892 

1.0976  7849 

1.1123  6601 

1  1272  4383 

33 

1.0420  8576 

1  0858  8687 

1  1008  8005 

1.1160  7389 

1  1314  7099 

34 

1  0433  8836 

1.0886  0159 

1  1040  9095 

1.1197  9414 

1  1357  1401 

35 

1.0446  9260 

1.0913  2309 

1.1073  1122 

1.1235  2679 

1.1399  7293 

36 

1.0459  9847 

1  0940  5140 

1.1105  4088 

1.1272  7187 

1  1442  4783 

37 

1.0473  0596 

1.0967  8653 

1.1137  7995 

1  1310  2945 

1.1485  3876 

38 

1.0486  1510 

1.0995  2850 

1.1170  2848 

1.1347  9955 

1.1528  4578 

39 

1.0499  2586 

1.1022  7732 

1.1202  8648 

1.1385  8221 

1.1571  6895 

40 

1.0512  3827 

1.1050  3301 

1.1235  5398 

1  .  1423  7748 

1.1615  0834 

41 

1.0525  5232 

1.1077  9559 

1.1268  3101 

1.1461  8541 

1.1658  6399 

42 

1  0538  6801 

1.1105  6508 

1.1301  1760 

1.1500  0603 

1.1702  3598 

43 

1.0551  8535 

1.1133  4149 

1.1334  1378 

1.1538  3938 

1.1746  2437 

44 

1.0565  0433 

1.1161  2485 

1.1367  1957 

1  .  1576  8551 

1.1790  2921 

46 

1.0578  2496 

1.1189  1516 

1.1400  3500 

1.1615  4446 

1.1834  5057 

46 

1.0591  4724 

1.1217  1245 

1.1433  6010 

1.1654  1628 

1.1878  8851 

47 

1.0604  7117 

1.1245  1673 

1.1466  9490 

1.1693  0100 

1.1923  4309 

48 

1.0617  9676 

1.1273  2802 

1.1500  3943 

I  1731  9867 

1  1968  1438 

49 

1  0631  2401 

1.1301  4634 

1.1533  9371 

1  1771  0933 

1  2013  0243 

60 

1.0644  5291 

1.1329  7171 

1.1567  5778 

1  1810  3303 

1.2058  0732 

COMPOUND  AMOUNT  OF  1 

s  =  (1  +  i)° 


513 


n 

«2% 

J2% 

Jl'2% 

?8% 

?3% 

1 
2 
3 
4 
5 

1.0041  6667 
1.0083  5069 
1.0125  5216 
1.0167  7112 
1.0210  0767 

.0050  0000 
.0100  2500 
.0150  7513 
.0201  5050 
.0252  5125 

1.0058  3333 
1  0117  0069 
1.0176  0228 
1.0235  3830 
1.0295  0894 

1.0062  5000 
0125  3906 
0188  6743 
.0252  3535 
.0316  4307 

1.0066  6667 
1.0133  7778 
1.0201  3363 
1.0269  3452 
1.0337  8075 

6 
7 
8 
9 
10 

0252  6187 
0295  3379 
.0338  2352 
1.0381  3111 
.0424  5666 

1.0303  7751 
1.0355  2940 
1.0407  0704 
1.0459  1058 
1.0511  4013 

1.0355  1440 
1.0415  5490 
1.0476  3064 
1.0537  4182 
1.0598  8865 

1.0380  9084 
.0445  7891 
.0511  0753 
.0576  7695 
.0642  8743 

1.0406  7262 
1.0476  1047 
1.0545  9451 
1.0616  2514 
1.0687  0264 

11 
12 
13 
14 
16 

1.0468  0023 
1.0511  6180 
1  0555  4173 
1.0599  3983 
1.0643  5625 

1.0563  9583 
1.0616  7781 
1.0669  8620 
1.0723  2113 
1.0776  8274 

1.0660  7133 
1.0722  9008 
1.0785  4511 
1.0848  3662 
1  0911  6483 

1.0709  3923 
1.0776  3260 
1.0843  6780 
.0911  4510 
.0979  6476 

1.0758  2732 
1.0829  9951 
1.0902  1950 
1.0974  8763 
1  .  1048  0422 

16 
17 
18 
19 
20 

1.0687  9106 
1.0732  4436 
1.0777  1621 
1.0822  0670 
1.0867  1589 

0830  7115 
0884  8651 
0939  2894 
0993  9858 
1048  9558 

1.0975  2996 
1  1039  3222 
1.1103  7182 
1  .  1  1  68  4899 
1.1233  6395 

1048  2704 
.1117  3221 
.1186  8053 
.1256  7229 
1.1327  0774 

1.1121  6958 
1.1195  8404 
1.1270  4794 
1.1345  6159 
1.1421  2533 

21 
22 
23 
24 
25 

1.0912  4387 
1  0957  9072 
I  1003  5652 
1  1049  4134 
1  .  1095  4526 

.1104  2006 
1159  7216 
1215  5202 
1271  5978 
.1327  9558 

1.1299  1690 
1365  0808 
1431  3771 
1498  0602 
1.1565  1322 

1.1397  8716 
.1469  1083 
.1510  7902 
.1612  9202 
1.1685  5009 

1.1497  3950 
1.1574  0443 
1.1651  2046 
1.1728  8793 
1.1807  0718 

26 
27 
28 
29 
30 

1  1141  6836 
1  1188  1073 
1  1234  7211 
1  1281  5358 
1.1328  5422 

1384  5955 
.1441  5185 
1498  7261 
.1556  2197 
1.1614  0008 

1  1632  5955 
1.1700  4523 
1  1768  7049 
1837  3557 
.1906  4069 

1.1758  5353 
.  1832  0262 
.1905  9763 
1.1980  3887 
1.2055  2661 

1.1885  7857 
1.1965  0242 
1.2044  7911 
1.2125  0897 
1.2205  9236 

31 
32 
33 
34 
36 

1.1375  7444 
1.1423  1434 
1  1470  7398 
1  1518  5346 
1.1566  5284 

1  1672  0708 
1.1730  4312 
1  1789  0833 
1.1848  0288 
1.1907  2689 

1975  8610 
2045  7202 
2115  9869 
2186  6634 
1  2257  7523 

1.2130  61  i5 
1.2206  4278 
1.2282  7180 
1  .  2359  4850 
1.2436  7318 

1.2287  2964 
1.2369  2117 
1.2451  6731 
1.2534  6843 
1.2618  2489 

36 
37 
38 
39 
40 

1.1614  7223 
1  1663  1170 
1  1711  7133 
1.1760  5121 
1.1809  5143 

1  1966  8052 
1.2026  6393 
1  2086  7725 
1  2147  2063 
1.2207  9424 

1  2329  2559 
1  240  1  1765 
1  2473  5167 
1  2546  2789 
1  2619  4655 

1.2514  4614 
1  2592  6767 
1.2671  3810 
1.2750  5771 
1.2830  2682 

1.2702  3705 
1.2787  0530 
1.2872  3000 
1.2958  1153 
1.3044  5028 

41 
42 
43 
44 
46 

1  1858  7206 
1.1908  1319 
1.1957  7491 
1.2007  5731 
1  2057  6046 

1.2268  9821 
.2330  3270 
.2391  9786 
.2453  9385 
.2516  2082 

1.2693  0791 
1.2767  1220 
1.2841  5969 
1.2916  5062 
1.2991  8525 

1.2910  4574 
1.2991  1477 
1.3072  3424 
1.3154  0446 
1.3236  2573 

1  3131  4661 
1.3219  0092 
1.3307  1360 
1.3395  8502 
1.3485  1559 

46 
47 
48 
49 
60 
i  

1.2107  8446 
1.2158  2940 
1  2208  9536 
1.2259  8242 
1.2310  9068 

.2578  7892 
.2641  6832 
.2704  8916 
.2768  4161 
.2832  2581 

1.3067  6383 
1.3143  8662 
1.3220  5388 
1  3297  6586 
1.3375  2283 

1.3318  9839 
1.3402  2276 
1.3485  9915 
1.3570  2790 
1.3655  0932 

1.3575  0569 
1.3665  5573 
1.3756  6610 
1.3848  3721 
1.3940  6946 

514 


APPENDIXES 


D 

»/4% 

1% 

lVa% 

1%% 

la/8% 

1 

l,007,r> 

1.01 

1.0112  5 

1.0125 

1.0137  5 

2 

1.0150  5625 

1.0201 

1.02262656 

1.0251  5625 

1  0276  8906 

3 

1.02266917 

1.0303  01 

1.0341  3111 

1.0379  7070 

1  0418  1979 

4 

1.0303  3919 

1.0406  0401 

1  0457  6509 

1.0509  4531 

1  0561  4481 

5 

1.0380  6673 

1.0510  1005 

1.0575  2994 

1.0640  8215 

1.0706  6680 

6 

1.0438  5224 

1.06152015 

1.0694  2716 

1.07738318 

1.0853  8847 

7 

1.0536  9613 

1.0721  3535 

1  0814  5821 

1.0908  5047 

1  1003  1256 

8 

1  0615  9885 

1  0328  6671 

1  0936  2462 

1.1044  8610 

1  1154  4  ISO 

9 

1.0695  6081 

1  0936  8527 

1  1051)  278') 

1.11829218 

1.1307  7918 

10 

1.0775  8253 

1.10462213 

1.1183  0958 

1.1322  7083 

1.1463  2740 

11 

1.0856  6141 

1.11566835 

1.13005124 

1.1464  2422 

1  1C208910 

12 

1.  09380690 

1  126H  2503 

1  11367414 

1.1607  5452 

1  1780  6813 

13 

1.1020  1015 

1  1380  9.12H 

1  1565  4078 

1.17526395 

1  1912  6(r>6 

14 

1.1102  7553 

1  1491  7121 

1.16955186 

1.1  899  5475 

1.2106  8773 

15 

1.1186  025i> 

1.16096896 

1.18270932 

1.2048  2918 

1.2273  3469 

16 

1.12699211 

1.1725  7864 

1.19601480 

1.21988955 

1.2442  1054 

17 

1.1354  4155 

1.18430143 

1  20U4  MW7 

1.2351  38  17 

1.2613  1813 

18 

1.1439  6039 

1  1961  4748 

l.:I230  7ti50 

1  2505  7739 

1  278661.% 

19 

1.15254009 

1  2081  0895 

1.2368  36  11 

1  2662  0961 

1.2962  4316 

20 

1.1611  8114 

1.2201  9004 

1.2507  5052 

1.2820  3723 

1.3140  6630 

21 

1.1698  9302 

1.2323  9194 

1  2648  2146 

1  2980  6270 

1.3321  3402 

22 

1.1786  6722 

1.2447  1586 

1.27905071 

1.3142  8848 

1  35045177 

23 

1  1875  0723 

1.2571  6302 

1.2934  4003 

1.3307  1709 

1.3690  2048 

24 

l.i  iW  i:r>3 

1.2697  3465 

1  3079  9123 

1.3173  5105 

1  38784151 

25 

1.2053  8663 

1.2824  3200 

1.3227  0613 

1.3641  9294 

1.4069  2738 

26 

1.2144  2703 

1.29525631 

1.33758657 

1.3812  4535 

1  4262  7263 

27 

1.2235  3523 

1  3082  0888 

1  3526  3442 

1.3985  1092 

1  4  158  MhS 

28 

1.2327  1175 

1  3212  9097 

1.3678  5156 

1  4159  9230 

1  4657  (>478 

29 

1.2419  5709 

1  3345  0388 

1.3832  3989 

1.4336  9221 

1.4859  1  «>(),-) 

30 

1.2512  7176 

1.3478  4892 

1.3988  0134 

1.4516  1336 

1.5063  5013 

31 

1.2606  5630 

1.3613  2740 

1.4145  3785 

1.4697  5853 

1  5270  6275 

32 

1  2701  1122 

1  3719  4068 

1.4304  5140 

1  4881  3031 

1  5480  5986 

33 

1.2796  3706 

1  .$880  9009 

1  44  65  4398 

1  3007  321  1 

1  5693  4569 

34 

1.2892  3131 

1  4025  7699 

1  4628  1760 

1.5255  6629 

1.5909  1M1!> 

35 

1.2989  0359 

1,4166  0276 

1.4792  7430 

1.5440  3587 

1.6127  9940 

36 

1.3086  4537 

1.4307  6878 

1.4959  1613 

1  5639  4382 

1.6349  7539 

37 

1.8184  6021 

1.4450  7647 

1.5127  4519 

1.5834  W12 

1.6374  5030 

38 

1.3283  4866 

1  4595  2724 

1  5297  6357 

I  6032  8678 

1  6802  4633 

39 

1.3383  1128 

1.4741  2251 

1.5469  7341 

l.G'233  2787 

1.7033  4971 

40 

1.3483  4861 

1.4888  6373 

1.5643  7687 

1.643o  1946 

1.7267  7077 

41 

1.3584  6123 

1.5037  5237 

1.5819  7611 

1.6641  6471 

1.7505  1387 

42 

1.3686  4969 

1  5187  8989 

1.5997  7334 

1  6849  6677 

1.7745  8343 

43 

1.3789  1456 

1.5339  7779 

1.6177  7079 

1.7060  2885 

1.7989  8396 

44 

1.3892  5642 

1.5493  1757 

1.6339  7071 

1.7273  5421 

1.8237  1999 

45 

1.3996  7584 

1.5648  1075 

1.6543  7538 

1.7489  4614 

1.8487  9614 

46 

1.4101  7341 

1.5804  5885 

1.6721)  8710 

1  7708  0797 

1.8742  1708 

47 

1.4207  4971 

1.5962  6344 

1.6918  0821 

1.7929  4306 

1.89998757 

48 

1.4314  0533 

1.6122  2608 

1.7108  4105 

1.8153  5485 

1.9261  1240 

49 

1.4421  4087 

1.6283  4834 

1.7300  8801 

1.8380  4679 

1.9525  9644 

50 

1.4529  5693 

1.6446  3182 

1.7495  5150 

1.8610  2237 

1.9794  4464 

COMPOUND  AMOUNT  OF  1 

s  =  (14-  i)» 


515 


r 

a 

ll/2% 

1%% 

!3/4% 

X7/B% 

2% 

i 

2 
3 
4 
5 

1.015 
1  0302  25 
1  0456  7838 
1  0(513  6355 
1.0772  8400 

1.0162  5 
1  0327  6106 
1.04954648 
1.0666  0161 
1.0839  3388 

1  0175 
1  0353  0625 
1  0534  2411 
1  0718  5903 
1  0906  1656 

1.0187  5 
1.0378  5156 
1  0573  1128 
1.0771  3587 
1.0973  3216 

102 
1.0404 
1.0612  OR 
1.0824  321(5 
1.1040  8080 

6 
7 
8 
9 
10 

1  0934  4326 
1  1098  4491 
1  1264  9259 
1.1433  8098 
1.16054083 

1  1015  4781 
1.111)1  4796 
1.13763899 
1.1561  2563 
1.1749  1267 

1.10970235 
1  1291  2215 
1  1488  8178 
1  1(589  8721 
1.1894  4449 

1.11790714 
1.13886790 
1  1602  2167 
1.1819  7588 
1  2011  3788 

1.1261  6242 

1.118(58567 
1  17165938 
1  1950  9257 
1.2189  9142 

11 
12 
13 
14 
15 

1  1779  4894 
1  1!>.')6  1817 
1  21355241 
1.2317  5573 
1  2502  3207 

1.19400500 
1.2134  0758 
1.2.W1  2545 
1  2531  (5374 
1.2735  2765 

1  2102  5977 
1  2314  3931 
1  2529  8950 
1  274<>  H.82 
1.2972  2786 

1.2207  1546 
1  2197  1638 
1  2731  4856 
1  2')70  2009 
1.3213  3922 

1  2433  7131 
1  2<i82  4179 
1  2936  0(5(53 
1.3194  787(5 
1.3158  toil1,! 

16 
17 
18 
19 
20 

1.26898555 
1,28802033 
1  3073  1061 
1.32(i<>  5075 
1.346S  5501 

1  2942  22  18 
i  3i:>2  5359 
1  3366  2b46 
1  35S3  4664 
1.3801  1977 

1  3  1  99  2935 
1  34  30  2S  11 
1.3()(i,r)31U 
1  ;!i)04  4540 
1  4147  7820 

1  3161  H33 
1.37135398 
1  3970  6686 
1  1232  6187 
1  1  199  4803 

1.3727  857) 
I  4002  1142 
1  1282  1(525 
1.45(>8  1117 
1.1859  1740 

21 
22 
23 
24 
25 

1  3670  5783 
1  3875  (-370 
1.10837715 
1  4295  0281 
1  1509  4535 

1  10285160 
1  1256  4793 
1  418H  1471 
1  17235795 
1  491.2  8377 

1.43%  3081 
1  4617  2871 
1  1<»03  61  1(, 
1  5164  4279 
1  5J29  8054 

1  4771  3155 
1  50  IK  3082 
1  f>330  4640 
I.f><il7  9102 
1.5910  7460 

1  5156  6684 
1  5459  7967 
1.57689926 
1  6081  3725 
1.64060599 

26 

1  4727  0953 

1  5205  <»838 

1  5(599  8269 

1  (5209  0725 

1.6731  1811 

27 
28 
29 
30 

1  4918  0018 
1  51722218 
1.53998051 
1.5630  8022 

1  5153  0810 
1  5701  l')36 
1  5%9  3868 
1  6218  7268 

1  5971  5739 
1  (>T>4  1290 
1  <!538  57(52 
1.6828  0013 

1.6512  992(5 
1  6822  fill  2 
1  71380352 
1.74593731 

1.7068  8648 
1.74102121 
1.7758  1469 
1.81136158 

31 
32 
33 
34 
35 

1.586526-12 
1  6103  2432 
1  6344  7918 
1  6589  9637 
1  6838  8132 

1.6482  2811 
1  (i7.r>0  1182 
1  7022  3076 
1  72'  W  9201 
1.7580  0275 

1  7122  4913 
1  7122  1319 
1.7727  0223 
1  8037  2152 
1.8352  8970 

1  7780  7366 
1  8120  2379 
1.8150  <)<)2l 
1.8806  1172 
1.9158  7311K 

1  8175  8882 
1.8815  4059 
1  92223110 
1  960(5  7(503 
1  9998  8955 

36 
37 
38 
39 
40 

1  7091  3954 
1  7347  7663 
1.76079828 
1  7872  1025 
1.81401841 

1  7865  7030 
1  81500207 
1  8111  0560 

1  8750  8857 
1.9055  5875 

1  8671  0727 
|  9000  8(!89 
1  9333  3841 
1  9671  7184 
2.0015  9734 

1  95179582 
1  9883  9199 
2  0256  7134 
2  0(i3(i  5573 
2.1023  4928 

2  0398  8734 
2  080(5  8509 
2  l'J22  9879 
2  1617  1177 
2.2080  3966 

41 
42 
43 
44 
45 

1  8412  2868 
1  8688  1712 
1  .8968  7982 
1  9253  3302 
1.9542  1301 

1.93652408 
1  9679  9260 
1  9999  7248 
2.0324  7203 
2.0654  9970 

2  0366  2530 
2  0722  (.624 
2  1085  3090 
2  1H4  3019 
2.1829  7322 

2.1417  6833 
2.18102618 
2  2228  37(50 
2.201  5  1581 
2.3009  7548 

2.2522  0046 
2.2972  4447 
23131  8936 
2.3900  5314 
2  1378  5421 

46 
47 
48 
49 
50 

1  9835  2621 
2.0132  7910 
2.0434  7829 
2  0741  3046 
2.1052  4242 

2  01)90  6407 
2  1331  7387 
2.1678  3794 
2  2030  6531 
2.2388  6512 

2.2211  7728 
2  2fJOO  1789 
2.2995  9872 
2  3398  4170 
2.3807  8893 

2.3502  3127 
23912  9811 
2.4391  9120 
2.4849  2603 
2.5315  1839 

2.48661129 
2  53(53  4351 
2.5870  7039 
2.63881179 
2.6915  8808 

516 


APPENDIXES 

s  =  (1  +  i)n 


n 

2'/8% 

2*/4% 

2%% 

2Va% 

2%% 

1 
2 
3 
4 
5 

1.0212  5 
1.0429  5156 
1.0651  1428 
1.0877  4796 
1,11086201 

1.0225 
1.0455  0625 
1.06903014 
1.09308332 
1.1176  7769 

1.0237  5 
1.0480  6406 
1.0729  5558 
1.0984  3828 
1.1245  2619 

1.025 
1.0506  25 
1.0768  9068 
1.1038  1289 
1.1314  0821 

1.0275 
1.0557  5625 
1,0847  8955 
1.1146  2126 
1.1452  7334 

6 
7 
8 
9 
10 

1.1344  6844 
1.1585  758!) 
1.1831  9563 
1.2083  3854 
1.23401573 

1.1428  2544 
1.16853901 
1.19483114 
1.2217  1484 
1.2492  0343 

1.15123369 
1  17857519 
1.2065  6(565 
J.2U52  2261 
1  2645  5915 

1  1596  9342 
1.18868575 
1.21840290 
1.21886297 
1.28008454 

1.1767  6836 
1.2091  2949 
1.2423  8055 
I.'27(i5  4602 
1.31  1C  5103 

11 
12 
13 
14 
15 

1.2602  3856 
1.2870  1863 
1.3143  6778 
1  3422  9809 
1.3708  2193 

1.27731050 
1.30604999 
133543611 
1.36  vl  8343 
1.39620680 

1  2945  9243 
1.3253  3900 
1.3568  1580 
1.3890  4017 
1  4220  2988 

1.31208666 
1.34488882 
1.3785  1101 
1.41297382 
1.4482  9817 

1  3477  2141 
1  3847  8378 
1.12286533 
1.4619  9413 
1.5021  9896 

16 
17 
18 
19 
20 

1.3099  5180 
1.4297  0087 
1.46008202 
1.4911  0876 
1.5227  9482 

1.42762146 
1.4597  4294 
1.4925  8716 
1.5261  7037 
1.5605  0920 

1.45580309 
1.4903  7841 
1.5257  7490 
1.5620  1205 
1.5991  0984 

1  48150562 
1.5216  1826 
1  5596  5872 
1.59865019 
1.6386  1644 

1.5435  0911 
1.5859  559  j 
1  6295  6973 
1.674H  8290 
1.7204  2843 

21 

22 
23 
24 
25 

1.5551  5421 
1.5882  0124 
1.6219  5051 
1.6564  1696 
1.6916158'? 

1  .5956  2066 
1.6315  2212 
1  (1682  3137 
1  7057  6658 
1  74-U  4632 

1.63708870 
1  6759  6955 
1.7157  7383 
1  7565  2346 
1  79b2  4089 

1.6795  8185 
1.72157140 
1.7646  1068 
1  .8087  2595 
1.8539  4410 

1  7677  4021 
1.8163  5307 
1.86(>3  0278 
1.9176  2610 
1.9703  6082 

26 
27 
28 
29 
30 

1.7275  6266 
1.7642  7336 
1.8017  6417 
1.8400  5166 
1.8791  5276 

1.7833  8962 
1.8235  1588 
1  8615  4499 
1  9064  9725 
1.9493  9341 

1.81094911 
1.88467165 
1.9291  3261 
1.9752  5663 
2.0221  6898 

1  9002  9270 
1.9478  0002 
1  9961  9502 
'2.0t(i  4  0739 
2  0975  6758 

2.0245  4575 
2  0802  2075 
2  1374  2682 
2  1962  0000 
225660173 

31 
32 
33 
34 
35 

1.9190  8476 
1.95986531 
2.00151245 
2  0440  4458 
2.0874  8053 

1  9932  5  1  79 
2  0381  0303 
2.0839  603  4 
2.1308  4945 
2.1787  9356 

2.0701  9549 
2.11936263 
2.16969749 
2.22122781 
2.2739  8197 

2  1500  0677 
2.2037  5694 
2,2588  5086 
2  3153  2213 
237320519 

2  31  8(5  5828 
2  3824  2138 
2  4479  3797 
2.5152  5626 
2.5844  2581 

36 
37 
38 
39 
40 

2.1318  3949 
2.1771  4108 
2.2234  0533 
2.2706  5269 
2.3189  0406 

2  2278  1642 
2  2779  4229 
2  3291  9599 
2  3816  0290 
2.4351  8S97 

2  3279  8904 
2.3832  7878 
2  4398  8165 
2  4978  2884 
2.5571  5228 

2  4325  3532 

2.4933  4870 
2.5556  8242 
2.6195  7448 
2.6850  6384 

2.6554  9752 
2  7285  2370 
2.8035  5810 
2  8806  5595 
2.9598  7399 

41 
42 
43 
44 
45 

2.3681  8077 
2  4185  0462 
2.4698  9784 
2.5223  8317 
2.5759  8381 

2  1899  8072 
2  5460  0528 
2  6032  00  U) 
2.66180444 
2.72175639 

2  6178  8464 
2.ti800  5940 
2  7437  1081 
2  8088  7395 
2.8755  8470 

2.7521  9043 
2.8209  9520 
2  8915  2008 
2.9638  0808 
3.0379  0328 

3.0412  7052 
3.1249  0546 
3  2108  4036 
3.2991  3847 
3.3898  6478 

46 
47 
48 
49 
50 

2.6307  2347 
2.6866  2634 
2.7437  1715 
2.8020  21U 
2.8615  6409 

2.7829  9590 
2.8456  1331 
2.9096  3961 
2.9751  0650 
3.0420  4640 

2.9438  7984 
3.013V  9699 
3.0853  7466 
3.1586  5231 
3.2336  7030 

3.1138  5086 
3.1916  9713 
3.2714  8956 
3.3532  7680 
3.4371  0872 

3.4830  8606 
3  5788  7093 
3.6772  8988 
3.7784  1535 
3.8823  2177 

COMPOUND  AMOUNT  OF  1 

s  =  (1  +  i)« 


517 


n 

3% 

3V4% 

3Va% 

3»/4% 

4% 

1 

2 

3 
4 

5 

1.03 

1.0609 
1.0927  27 
J  1*55  0881 
1  1592  7407 

1.0325 
1.0660  5625 
1.1007  0308 
1.1364  7593 
1.1734  1140 

1.035 
1.0712  25 
1.1087  1788 
1.1475  2300 
1.1876  8631 

1.0375 
1  .0764  0625 
1.1167  7148 
1.15S6  5042 
1.2020  9981 

1.04 
1.0816 
1.1248  64 
1.16985856 
1.2166  5290 

6 
7 
P 
9 
10 

1.1940  :230 
1  2298  7387 
1.2667  7008 
1.3047  7318 
1.3439  1638 

1.2115  4727 
1.2509  2255 
1.2915  7754 
1.33355381 
1.3768  9430 

1.2292  5533 
1  2722  7926 
1  3168  0904 
1.3628  9735 
1.4105  9876 

1.2471  7855 
1.2939  4774 
1.3424  7078 
1.39281344 
1.4450  4394 

1.2653  1902 
1.3159  3178 
1.3685  6905 
1.4233  1181 
1.4802  4428 

11 
12 
13 
14 
15 

1.3842  3387 
1  4257  6089 
1  4685  3371 
1.5125  8972 
1.5579  6742 

1.42164337 
1.4678  4678 
1  5155  5180 
1.5648  0723 
1.6156  6347 

1.4599  6972 
1.51106866 
1.5639  5606 
1.6186  9152 
1.6753  4883 

1.49923309 
1.  55515133 
1.61378387 
1.6743  0076 
1.7370  8704 

1.5394  5406 
1.60103222 
1  6650  7351 
1.7316  7645 
1.8009  4351 

16 
17 
18 
19 
20 

1  6047  0644 
1  6528  4763 
1.7024  3306 
1.753.'}  0605 
1.8061  1123 

1.6681  7253 
1  7223  8814 
1.77836575 
1.8361  6264 
1.8958  3792 

1.7339  8604 
1.7916  7555 
1.8574  8920 
1.9225  0132 
1.9897  8886 

1.8022  2781 
1.8698  1135 
1.9399  2927 
2.01267662 
2.0881  5200 

1.87298125 
1.9479  0050 
2.0258  1652 
2.1068  4918 
2.1911  2314 

21 

22 
23 
24 
25 

1.8602  9457 
1  9161  03  11 
1  9735  b65l 
203279111 
2  0937  7793 

1.95745266 
2  0210  6987 
2  0867  5464 
2  1545  7416 
2  2245  9782 

20591  3147 
2.1315  1158 
2.2061  1448 
2  2833  2849 
2  3632  4498 

2.1664  5770 
2.2476  9986 
2.3319  8860 
2.4194  3818 
2.51016711 

2.2787  6807 
2.3699  1879 
2.4647  1551 
2.5633  041fl 
2.6658  3633 

26 

27 
28 
29 
30 

215659127 
2.2212  8901 
2  2879  2708 
2  3565  0551 
2  4272  6247 

2  2968  9725 
2  3715  4611 
2  44862167 

2  52820188 
2  6103  6844 

2  4459  5856 
2.5315  6711 
2.6201  7196 
271187798 
2.8067  9370 

2.6042  9838 
2.70195956 
2  8032  8305 
2,9084  0616 
3.0174  7139 

2.7724  6978 
2.8833  6858 
2.9987  0332 
3.1186  5145 
3.2433  9751 

31 
32 
33 
34 

35 

2  5000  8035 
2.5750  8276 
2.6523  :r>24 
2.73190530 
2.8138  6245 

2.6952  0541 
2.78279959 
2  8732  4058 
2  9666  2089 
3.0630  3607 

2.9050  3148 
3.00<>7  0759 
3.1119  4235 
3.2208  6033 
3.3335  9045 

3.1306  2657 
3.2480  2507 
3.3698  2601 
3.4961  9448 
3.6273  0178 

3.3731  3341 
3.5080  5875 
3  6483  8110 
3.7943  1631 
3.9460  8899 

36 
37 
38 
39 
40 

2  8982  7838 
2  9852  2668 
3.0747  8348 
3.1670  2698 
3.2620  3779 

3.1625  8475 
3  2653  6875 
3.3714  9323 
3.4810  6676 
3.5942  0143 

3.4502  6611 
3.5710  2543 
36960  1132 
3  8253  7171 
3.9592  5972 

3.7633  2559 
3.9044  5030 
4.0508  6719 
4.2027  7471 
4.3603  7876 

4.1039  3255 
4.2680  8986 
4.4388  1345 
4.6163659') 
4.8010  2063 

41 
42 
43 
44 

45 

3  3598  9893 
3.4606  9589 
3.5645  1677 
3.6714  5227 
3.78159584 

3.71101298 
3.8316  2090 
3  9561  4858 
4.0847  2341 
4.2174  7692 

4  0978  3381 
4.24125799 
1.3897  0202 
4.5433  4160 
4.7023  5855 

4.5238  9296 
4.6935  3895 
4.8695  4666 
5.0521  5466 
5.2416  1046 

4.9930  6145 
5.1927  8391 
5.4004  9527 
5.6165  1508 
5.8411  7568 

46 
47 
48 
49 
60 

3.8950  4372 
4.0118  9503 
4.1322  5188 
4.2562  1944 
4.3839  0602 

4.3545  4492 
4.4960  6763 
4.6421  8983 
4.7930  6100 
4.9488  3548 

4.86694110 
5  0372  8404 
5.2135  8898 
5.3960  6459 
5.5849  2686 

5.4381  7085 
5  6421  0226 
5.8536  8109 
6.0731  9413 
6.3009  3891 

6.0748  2271 
6.3178  1562 
6.5705  2824 
6.a333  4937 
7.1066  8335 

518 


APPENDIXES 

s  -  (14-  i)n 


n 

4V  V 

437  r 

5% 

6%% 

1 

1.0425 

1  .045 

1.0475 

1.05 

1.055 

2 

1.0868  0625 

1.092023 

1.0972  5025 

1.1025 

1.1130  2> 

3 

1.13299552 

1.1411  6613 

1  1493  759'2 

1  1576  25 

1.1742  4138 

4 

1.1811  4783 

1.1925  1860 

1.2039/128 

1.2155  0625 

1.2388  240") 

5 

1,2313  4661 

1.2461  8194 

1.2611  5991 

1.2762  8156 

1.3069  6001 

6 

1.28367884 

1.3022  6012 

1.32106501 

1.3400  9564 

1.3788  12S1 

7 

1.33823519 

1.30080183 

1.3838  1500 

1.4071  0042 

1.4546  7910 

8 

1.3951  1018 

1  4221  0061 

1.4495  4084 

1  4774  5544 

1.53408051 

9 

1.4544  0237 

1.4860  9514 

1.5184  0031 

1.5513  2822 

1.6190  9427 

10 

1.5162  1147 

1.5529  6942 

1.5905  2133 

1.6288  9463 

1.7081  4440 

11 

1.5806  5358 

1  0228  5305 

1.0060  7423 

1.7103  3936 

1.8020  1)240 

12 

1.04783136 

1  0458  HI  43 

1.7452  1270 

1.7958  5633 

1.9012  ()74'J 

13 

1.71786119 

1.7721  9010 

1  82S1  1037 

1  885o  4914 

2  0057  7390 

14 

1.7908  7342 

1  8319  4492 

1.4141)4361 

1  9799  3100 

2  11  GO  9140 

15 

1.8609  8554 

1.9352  8244 

2  0059  0552 

2.0789  2818 

2  2324  7649 

16 

1  9163  3243 

2  0223  7015 

2.1011  8004 

2.1828  7459 

2  3552  0270 

17 

2.02905156 

211337081 

2  2009  9237 

2.2920  1832 

2.  4848  02  H 

18 

2.11528625 

2  2084  7877 

2  3035  :{431 

2  106G  1923 

2  0214  01)27 

19 

2  2051  8591 

2  3078  6031 

2  1150  5204 

2.5209  5020 

2.7656  l()9l 

20 

2.2989  0631 

2.41171402 

2.5297  6764 

2.6532  9771 

2.9177  5749 

21 

2.3966  0983 

2.5202  4116 

2.64993160 

2.7859  6259 

307823115 

22 

2.4984  6575 

2  6330  5201 

2.7758  0335 

2.9252  6072 

3  2475  3703 

23 

2.6040  5054 

2.7521  0635 

2.9076  5401 

3  0715  2376 

3.4261  5157 

24 

2.7153  4819 

2.8700  1383 

3  0457  6758 

3  2250  9994 

3.0145  8990 

25 

2.8307  5049 

3.0054  3446 

3.1904  4154 

3.3863  5494 

3.8133  9233 

26 

2.9510  5739 

3.14007901 

3.3119  8751 

3.5550  7269 

4.0231  2893 

27 

3.0764  7732 

3.2820  0966 

3.5007  3192 

3  7334  5632 

4.21110102 

28 

3.2072  2761 

3.4296  9998 

3  0070  1068 

3  9201  2914 

4  4778  4307 

29 

3.3435  3478 

3.5840  3049 

3.8111  9998 

4.11«1  3500 

4.7211  2111 

30 

3.4856  3501 

3.7453  1813 

4.0236  5098 

4.3219  4238 

4.9839  5129 

31 

3.6337  7450 

3.9138  5745 

4.2147  8068 

4.5380  3949 

5  2580  0801 

32 

3.7882  0992 

4.0899  8104 

4.4149  8276 

4  7619  4147 

5  5472  0238 

33 

3,9492  0884 

4  2740  3018 

4.0246  9445 

5  0031  8854 

5.8523  0181 

34 

4.1170  5021 

4  4663  6154 

4.8443  6743 

5.2533  4797 

6.1742  4171 

35 

4.2920  2485 

4.6073  4781 

5.0744  7488 

5.5160  1537 

6.5138  2501 

36 

4.4744  3590 

4.8773  7846 

5.3155  1244 

5.7918  1614 

0.8720  8538 

37 

4.0645  9943 

5.09680019 

5  5079  9928 

608140094 

7  2500  5008 

38 

4  8628  4491 

5.3202  1921 

5.8324  7925 

0  3854  7729 

7.0488  02M 

39 

5.0695  1581 

5.5058  9408 

6.1095  2201 

6.7047  5115 

8.0094  8099 

40 

5.2849  7024 

5.8163  6454 

0.3997  2431 

7.0399  8871 

8.5133  0877 

41 

5.5005  8147 

0.0781  0094 

67037  1121 

7.3919  8815 

8.9815  4076 

42 

5.7437  3868 

6  3510  1548 

7.0221  3750 

7.7013  8756 

9.4755  2550 

43 

5.9878  4758 

6.6374  3818 

7.3556  8903 

8.1490  66WI-4 

9  9966  7940 

44 

6.24233110 

6.9361  2290 

7.7050  8426 

8.5571  5028 

10.5464  9677 

45 

6.5076  3017 

7.2482  4843 

8.0710  7576 

8.9850  0779 

11.1265  5409 

46 

6.7842  0445 

7.5744  1961 

8.4544  5186 

9.4342  5818 

11.7385  1456 

47 

7.0725  3314 

7  9152  6849 

8  8560  3832 

9  9059  7109 

12  3841  3287 

48 

7.3731  1580 

8  2714  5557 

9.2767  0014 

10.40126965 

13.0652  0017 

49 

7.6864  7J22 

8.6436  7107 

9  7173  4340 

10.9213  3313 

13.7838  4948 

50 

8.0131  4834 

9.0326  3627 

10.1789  1721 

11.4673  9979 

14.5419  6120 

COMPOUND  AMOUNT  OF  1 

s  =  (1  +  i)n 


519 


n 

6% 

*V%% 

7% 

8% 

9% 

2 
3 

4 
5 

1.06 
1.1236 
1.1910  16 
1.2624  7696 
1.3382  2558 

1.065 
1.1342  25 
1.2079  4963 
1.2864  6635 
1.37008666 

1.07 
.1449 
2250  43 
3107  9601 
.4025  5173 

1.08 
1.1664 
1  2597  12 
1.36048896 
1.4693  2808 

1.09 
1.1881 
1.2950  29 
1.4115  8161 
1.5386  2395 

6 
7 
8 
9 
10 

1.4185  1911 
1.5036  3026 
1.5938  4807 
1  .685)4  7896 
1.7908  4770 

1  4591  4230 
1.55398655 
1.6549  9567 
1.7625  7039 
1.8771  3747 

.5007  3035 
6057  8148 
71  SI  8618 
.83S4  5921 
1.9671  5136 

1  5868  7432 
1  7138  2427 
1  8509  3021 
1  9990  0163 
2.1589  2500 

1.6771  0011 
1  8280  3912 
1  9925  626  1 
2  1718  9328 
2.3673  6367 

11 
12 
13 
14 
15 

1  8982  9856 
2.0121  9647 
2  1329  2826 
2.2609  0396 
2.39655819 

1  9991  5140 
2  12909624 
2  2674  875O 
2.41487418 
2.5718  4101 

2.1048  5195 
2.2521  9159 
2  4098  4500 
2.5785  3415 
2.7590  3154 

2.33163900 
2  5181  70  J  2 
2  7196  2373 
2.9371  9362 
3.1721  6911 

2.5804  2641 
281-266478 
3.0658  0101 
3  3U7  2703 
3.6424  8246 

16 
17 
18 
19 
20 

2.5403  5168 
2  6927  7279 
2  8543  3915 
3  0255  9950 
3.2071  3547 

2  7390  1067 
201704637 
3  1066  5438 
3  3085  8691 
3.52.-S6  4500 

2.9521  6375 
3.15881521 
3.3799  3228 
H  Hl<>5  2751 
3.8696  8446 

3.4259  4264 
3  7000  1805 
3  9960  1950 
4.3157  0106 
4.6609  5714 

3  9703  0588 
4.3276  3341 
4  7171  204'2 
5.1116  6125 
5.6044  1077 

21 
22 
23 
24 
25 

3.3995  6360 
3  6035  3742 
3  8197  4966 
4.0489  3464 
4.2918  7072 

3  7526  8199 
3  9966  0632 
4  2563  8573 
4  5330  5081 
4.82769911 

4.14056237 
4  4304  0174 
.7405  2986 
5.0723  G695 
5.4274  3264 

5  0338  3372 
5  1365  4041 
5  H71  1  6365 
63411  8074 
6.8484  7520 

6.10880771 
6.6586  0043 
7  2578  7447 
791108317 
8.6230  8066 

26 
27 
28 
29 
30 

4.5493  8296 
4.8223  4594 
511168670 
5.4183  8790 
5.7434  9117 

5.1414  9955 
5  475(5  9702 
583161733 
6  2106  7245 
6.6143  6616 

5.8073  5292 
f,  2  1  US  6763 
6  6488  3836 
7.11425705 
7.6122  5504 

7  3903  5321 
7  9880  6147 
8  6271  0639 
9  3172  7190 
10.06265689 

9  3991  5792 
10  21508213 
11  1671  3952 
12  1721  H?OH 
13.2676  7847 

31 
32 
33 
34 
35 

6.0881  0064 
6  4533  8668 
6  8405  8988 
7.2510  2528 
7.6860  8679 

7  0442  9996 
7  5021  7946 
7  9898  2113 
8.  5091  5950 
9.0622  5487 

8.1451  1290 
871527080 
9.3253  3975 
9.9781  1354 
10.67658148 

10.8676  6944 
1  1  7370  8300 
126700  4964 
13  6901  3361 
14.7853  4429 

14.4617  6953 
15  7633  2879 
17  1820  2838 
18.7284  1093 
20.4139  6792 

36 
37 
38 
39 
40 

8.1472  5200 
8.6360  8712 
9.1542  5235 
9.7035  0749 
10.2857  1794 

9.65130143 
10  2786  3(503 
10.91674737 
11.65828595 
12  4160  7453 

11.42394219 
12.22361814 
13.0792  7141 
13  9918  2011 
14.9744  5784 

15.9681  7184 
17  24562558 
18  6252  7563 
20.1152  9768 
21.7245  2150 

22  2512  2503 
21  2538  3528 
26  4366  8046 
28  8159  8170 
31.1094  2005 

41 
42 
43 
44 

45 

10.9028  6101 
11.55703267 
12  2.504  5463 
12.9854  8191 
13.7646  1083 

13  2231  1938 
14.0826  2214 
li.  9979  9258 
15.9728  6209 
17.0110  9813 

16.0226  6989 
17.14425678 
18.3443  5475 
19.6284  5959 
21.0024  5176 

23.4624  8322 
25  3394  8187 
27.3666  4042 
29.5559  7166 
31.9204  4939 

34  2302  6786 
37.31753197 
40.6761  0()84 
44.3369  5973 
48.3272  8610 

46 
47 
48 
49 
50 

14.5904  8748 
15.4659  1673 
16.3938  7173 
17.3775  0403 
18.4201  5427 

18.11681951 
19.2944  1278 
20.5485  4961 
21.8842  0533 
23.3066  7868 

22.4726  2338 
24  0457  0702 
25.7289  0651 
27.5299  2997 
29.4570  2506 

34.4740  8534 
37.2320  1217 
40.2105  7314 
43.4274  1899 
46.9016  1251 

52.6767  4185 
5741764862 
62,5852  3700 
68.2179  0833 
74.3575  2008 

520 


APPENDIXES 


Table  3 
PRESENT  VALUE  OF  1 


n 

Vs% 

%% 

3/a% 

V*% 

6/8% 

1 

2 
3 
4 
5 

0.9987  5156 
0.9975  0468 
0.9962  5936 
0.99ft)  1059 
0.9937  7337 

0.9975  0023 
0.99501869 
0  9925  3734 
0.99006219 
0.9875  9321 

0.9962  6401 
0.9925  4198 
0  9888  3385 
0  9851  3958 
0.9814  5911 

0.9950  2488 
0  9900  7450 
0  9851  4876 
0  9802  4752 
0.9753  7067 

0.9937  8882 
0  9876  1622 
0  9814  8196 
0.9753  8580 
0.9693  2750 

6 

7 
8 
9 
10 

0  9925  3270 
0.9912  9359 
0  9900  5602 
09HHH1999 
0.9875  8551 

0.9851  3038 
0  9h2h  7370 
0.9WV2  231  1 
0.9777  7869 
0  9753  4034 

0  9777  9238 
09741  3936 
0.9701  9999 
0  !)668  7-121 
0.9632  6198 

0  9705  1808 
0.9056  8963 
0.9608  8520 
0  9561  0468 
0  9513  4794 

0.06S3  06S'< 
0  9573  23.jf> 
0  9513  771.". 
0  9454  6827 
0  9395  9580 

11 

12 
13 
14 
15 

0.9863  5257 
0  9851  2117 
0.9838  9130 
0.9826  6297 
0  9814  3618 

0  9729  0807 
0.9701  8187 
0  9680  6171 
0  96r>6  4759 
0.9632  3949 

0  9596  6324 
0  9560  7795 
0.9525  0605 
0.9489  4750 
091.310224 

0  9466  1  189 
0  94l90r>34 
0<)372  1921 
01)325  5616 
0.9279  1688 

0  9337  .7)80 
0  9279  600") 
0  0221  9632 
0.9164  6840 
0.9107  7604 

16 
17 
18 
19 
20 

0.9802  1092 
0.9789  8718 
0  9777  0498 
0.9765  4430 
0.9753  2514 

0  9608  3740 
0.9584  4130 
0.95605117 
0.9536  6700 
0.9512  8878 

0  9118  7022 
0.9383  5141 
093484573 
09313  5316 
0.9278  7363 

0.9233  0037 
01)187  0684 
0  9141  3616 
0.9095  8822 
0.9050  629O 

0  9051  1905 
08994  9710 
0.893')  102') 
0.8883  5802 
0.8828  4027 

21 
22 
23 
24 
25 

0.9741  0750 
0.9728  9139 
0.9716  7679 
0  9701  6371 
0.9692  5215 

0.9489  1649 
09465  5011 
0  9441  8964 
0.9418  3505 
0.9394  8634 

09214  0711 
0  9209  5353 
0  9175  12S6 
0.9140  8504 
0.9106  7003 

0  9005  6010 
0  8960  7971 
0  81)16  2160 
0.8871  8507 
0.8827  7181 

0.8773  5670 
0.8719  0736 
0  8664  0170 
0  8611  098.-) 
0.85576135 

26 
27 
28 
29 
30 

0.9680  4210 
0  9668  3355 
0.9656  2652 
09611  2100 
0.9632  1697 

0.9371  4318 
0  9348  0646 
0.9324  7527 
0.9301  4990 
0.9278  3032 

0  9072  6777 
0.9038  7823 
0  90050135 
0.8971  3709 
0  8937  8539 

0  8783  7901 
0  8740  0986 
0  8696  6155 
0  86r>3  3188 
0  8610  2973 

0  8504  4606 
0  HI  51  6378 
0  8399  1432 
0  8346  9746 
0.8295  1300 

31 
32 
33 
34 
35 

0.9620  1446 
0.9(508  1344 
0.9590  1392 
0  9581  1590 
0.9572  1938 

0.92551653 
0.9232  0851 
0.9209  0624 
0  91860972 
0.9103  1892 

0  8904  4622 
0  8871  19.V2 
0  8838  0525 
0.8805  033C 
0.8772  1381 

0.8567  4600 
08521  8358 
0  8182  4237 
08410  2226 
0.8398  2314 

0.8243  6075 
0.8192  4050 
0  8141  5205 
0  8090  9520 
0.8010  6976 

36 
37 
38 
39 
40 

0  9560  2435 
0.9548  3081 
0  9">36  3876 
0  9524  4820 
0.9512  5913 

0  91  10  3384 
09117  5145 
0  9094  8075 
0.9072  1272 
0.9049  5034 

0  873!)  3f>:>r> 
08706  7153 
0  8674  1871 
0.8611  7804 
0.8609  4948 

0.8356  4492 
0  S3  14  8748 
0  8273  5073 
0  8232  34.").") 
0.8191  3886 

0.7990  7551 
0  7941  1234 
0.7891  7997 
0  7842  7823 
0.7794  0693 

41 
42 
43 
44 
45 

0  9500  7154 
0.9488  8543 
0.9477  0080 
0.9465  1766 
0.9453  3599 

0.9026  9361 
0.9004  4250 
0  8981  9701 
0  8959  5712 
0.8937  2281 

0  8577  3298 
0  8545  2850 
0.8513  3599 
08181  5511 
0.8449  8671 

0.8150  6354 
0  81  10  0850 
0.8069  7363 
0.8029  5884 
0.7989  6402 

0.7715  6500 
0.7697  5493 
0.7649  7384 
0.7602  2245 
0.7555  0057 

46 
47 
48 
49 
60 

0.9441  5579 
0.9429  7707 
0.94179982 
0.9406  2404 
0.9394  4973 

0.8914  9407 
0  8892  7090 
0.8870  5326 
0  8848  4116 
0.8826  3457 

0  8418  2985 
0  8386  8478 
0.8355  5146 
0.8324  2985 
0.8293  1990 

0.7949  8907 
0.7910  3390 
0.7870  9841 
0.7831  8250 
0.7792.8607 

0.7508  0802 
0  7461  4462 
0  7415  1018 
0.7369  0453 
0.7323  2748 

j 

PRESENT  VALUE  OF  1 


521 


n 

3/4% 

1% 

1V8% 

iy«% 

!3/a% 

1 

2 
3 

4 
5 

0.9925  5583 
0.9851  6708 
0.9778  3333 
0.9705  5417 
0.9633  2920 

0.9900  9901 
0.9802  9605 
0.9705  9015 
0.9609  8034 
0.9514  6569 

0.9888  7515 
0  9778  7407 
0.9669  9537 
0.9562  3770 
0.9455  9970 

0.9876  5432 
0.9754  6106 
0.9634  1833 
0.9515  2428 
0.9397  7706 

0.9864  3650 
0.9730  5C96 
0.9598  5890 
0.9468  3986 
0  9339  9739 

6 
7 

8 
9 
10 

0.9561  5802 
0  9490  4022 
0.9419  7540 
0.93  JO  6318 
0.92800315 

0.9420  4524 
0.9327  1805 
0  9234  8322 
09143  3982 
0  9052  8095 

0  9350  8005 
0.9246  7743 
0  9143  9054 
0.9042  1808 
0  8941  5880 

0.9281  7488 
0.9167  1593 
0.0053  9845 
0.8942  2069 
0.8831  8093 

092132912 
0  9088  3267 
0.8965  0571 
0.8843  4596 
0,87235113 

11 
12 
13 
14 

15 

0  9210  9494 
0.9142  3815 
09074  3211 
0.9006  7733 
0.8939  7254 

0.8963  2372 
0  8874  4923 
0  8786  G2(JO 
0  8699  C297 
0.8613  4947 

08842  1142 
0  8743  7470 
0  8646  4742 
0  8550  2835 
0.8455  1629 

0  8722  7746 
0  8615  0860 
0.8508  7269 
0.8103  6809 
0.8299  9318 

0.8605  1899 
0.8488  4734 
0  8373  3400 
0.8259  7682 
0.8147  7368 

16 
17 
18 
19 
20 

0.8873  1766 
0  8807  1231 
08741  5(514 
0  8676  4878 
0.8611  8985 

0  8528  2126 
08113  7749 
0  8360  1731 
0  8277  3992 
081954447 

0.8361  1005 
0  82(58  0846 
08176  1034 
0  8085  1455 
0.7995  1995 

0.8197  4635 
0  8096  2602 
0.7996  3064 
0.7897  5866 
0.7800  0855 

0.8037  2250 
0  7928  2120 
0.7820  6777 
0.7714  6020 
0.7609  9619 

21 

22 
23 
24 
25 

0.8547  7901 
0  8484  1589 
0.8421  0014 
0.8358  3140 
0.8296  0933 

08114  3017 
0  8033  9621 
0  79,14  4179 
078756613 
0.7797  6844 

0.7906  2542 
0  7S18  2983 
0  7731  3210 
0.76453112 

0.7560  2583 

0  7703  7881 
0  7608  6796 
0.7514  7453 
0.7421  9707 
0.7330  3414 

0,7506  7472 
0.7404  9294 
0.7304  4926 
0.72054181 
0.7107  6874 

26 
27 
28 
29 
30 

0.8234  3358 
0.81730380 
0  8112  1906 
0  8051  8080 
0.7991  8690 

0  7720  4796 
0  7644  0392 
0  7568  3557 
074934215 
0.7419  2292 

074761516 

0  7392  9806 
0.7310  7348 
0  7229  4040 
0  7148  9780 

0.7239  8434 
0.7150  4626 
0.7062  1853 
0  6974  9978 
0.6888  8867 

07011  2823 
0.6916  1847 
0.6822  3771 
0.67298417 
0.6638  5015 

31 
32 
33 
34 
35 

0  7932  3762 
0.7873  3262 
0.7814  7158 
0.77.')C  5418 
0.7698  8008 

0.73457715 
072730411 
0  7201  0307 
0.7129  733  t 
0.7059  1420 

0.7069  4467 
0  6990  8002 
0  6913  0287 
06836  1223 
0.67600715 

0.6803  8387 
0  6719  8407 
0.6636  8797 
0.6554  9429 
0.6474  0177 

0.6518  5194 
0.6459  6985 
0.63720821 
0.6285  6540 
0.6200  3991 

36 
37 
38 
39 
40 

0.7611  4896 
0.7584  6051 
0  7528  1440 
0.7472  1032 
0.7416  4796 

0.6989  2495 
0  6920  0490 
0  6851  5337 
0.6783  6967 
0  6716  5314 

0  6684  8667 
0.6610  4986 
0  6536  9578 
0.6464  2352 
0.6392  3216 

0.6394  0916 
O.C315  1522 
0.6237  1873 
0.6160  1850 
0.6084  1334 

0.61103000 
0.6033  3416 
0.5951  5083 
0.5870  7850 
0.5791  1566 

41 
42 
43 
44 
46 

0.7361  2701 
0.7306  4716 
0.7252  0809 
0.7198  0952 
0.7144  5114 

0  6650  0311 
0.6584  1892 
065189992 
0  6454  4546 
0.6390  5492 

0.6321  2080 
0.6250  8855 
0.6181  3454 
0.6112  5789 
0.6044  5774 

0.6009  0206 
0.5934  8352 
0.5861  5656 
0.5789  2006 
0.5717  7290 

0.5712  6083 
0.5635  1253 
0.5558  6933 
0.5483  2979 
0.5408  9252 

46 

47 
48 
49 
50 

0.7091  3264 
0.7038  5374 
0.6986  1414 
0.6934  1353 
0.6882  5165 

0.6327  2764 
0.6264  6301 
0  6202  6041 
0.6141  1921 

0.6080  3882 

0.5977  3324 
0.5910  8356 

0  5845  0784 
0.5780  0528 
0.5715  7506 

0.5647  1397 
0.5577  4219 
0.5508  5649 
0.5440  5579 
0.5373  3905 

0.5335  5612 
0.6263  1923 
0.5191  8050 
0.5121  3800 
0.5051  9220 

522 


APPENDIXES 

v-  =  _J— 

T  e  +          i         »\ x 


n 

iya% 

1%% 

!3/4% 

178% 

2% 

1 
2 
3 
4 

5 

0.9852  2167 
0.9706  6175 
0.9563  1699 
0.9421  8423 
0.9282  6033 

0.9840  0984 
0.9682  7537 
0.9527  9249 
0.9375  5718 
0.9225  6549 

0,98280098 
0.9658  9777 
0.9492  8528 
0.9329  5851 
0.9169  1254 

0.9815  9509 
0  9635  2892 
0.9457  9526 
0.9283  8799 
0.91130109 

098039216 
0.9611  6878 
0.91232233 
0.9238  4543 
0.9057  30C1 

6 
7 
8 
9 
10 

0.9145  4219 
0.9010  2679 
0.8877  1112 
0  8745  9224 
0.8616  6723 

0.9078  1352 
0  8932  9744 
0.87901317 
0  8649  5791 
0.8511  2709 

0.9011  4254 
0  8856  4378 
0.8704  1157 
0  8r>->4  4135 
0.8407  2860 

0  8045  2868 
0.8780  6496 
0  8619  0426 
0.8400  4099 
0.8304  6968 

0.8879  7138 
087056018 
0.8534  9037 
0.8367  5527 
0.8203  4830 

11 
12 
13 

14 
15 

0  8489  3323 
0  836.3  8742 
0.8240  2702 
0.8118  4928 
0  7998  5150 

0.83751743 
0  8241  2539 
0  8109  4750 
0.7979  8032 
0.7852  2048 

0  8262  6889 
0  8120  5788 
0  7()8()  9128 
0  7813  6490 
0.7708  7459 

0  8151  8496 
0  80<)1  8156 
0  7854  5429 
0.7709  9808 
0.7568  0793 

080126304 
0.7884  93  1H 
0  7730  32r>3 
0.7578  7502 
0.7430  1473 

16 
17 
18 
19 
20 

0  7880  3104 
0.7763  8526 
0.76491159 
0.7536  0747 
0.7424  7042 

0.7726  6468 
0  7603  0965 
0.7481  5218 
0.7361  8911 
0.7244  1732 

0  7576  1631 
0  744.3  8605 
0  7317  7990 
0.7191  9401 
0.7068  2458 

0.7428  7895 
0  7292  0633 
0.7157  8536 
0.7026  1139 
0.6896  7989 

0.7284  4581 
0  7141  6256 
0.7001  5937 
0.6864  3076 
0.6729  7133 

21 
22 
23 
24 
25 

0.7314  9795 
0  7206  8763 
0.7100  3708 
0.6995  4392 
0.6892  0583 

0  7128  3378 
0.701  1  3545 
06902  1938 
0  6791  8267 
0.6683  2243 

0  6940  6789 
0.6827  2028 
067097817 
0  659  1  3800 
0.6480  9632 

0.6769  8640 
0  6G45  2653 
0  6522  9598 
0  6402  9053 
0.6285  0604 

0  6597  7582 
0  6468  3904 
0  (>341  5592 
0  6217  2149 
0.6095  3087 

26 
27 
28 
29 
30 

0.6790  2052 
0  6689  8574 
0  6590  9925 
0.6493  5887 
0.6397  6243 

0  6576  3584 
06471  2014 
0.6367  7259 
0.6265  9049 
0.6165  7121 

0  6369  4970 
0.6259  9479 
0  6152  2829 
0.6046  4697 
0.5942  4764 

0  6169  3844 
0  0().,5  8375 
0.5944  3804 
0  5834  9746 
0.5727  5824 

0  5975  7928 
0  5858  6201 
0  5743  7455 
0.5631  1231 
0.5520  7089 

31 
32 
33 
34 
35 

0.6303  0781 
0.6209  9292 
0.61181568 
0  6027  7407 
0.5938  6608 

0.60671214 
0.5970  1071 
0  5874  6442 
0  5780  7077 
0.5688  2732 

0.5840  2716 
0  5739  8247 
0  5011  1053 
0.5544  0839 
0.5448  7311 

0.5622  1668 
0  5518  6913 
0  5417  1203 
0.5317  4187 
0.5219  5521 

0  5112  4597 
0  5306  3330 
0.5202  2873 
O..r>100  2817 
0.5000  2761 

36 
37 
38 
39 
40 

0.5850  8974 
0.5764  4309 
0.5679  2428 
0.5595  3126 
0.5512  6232 

0.5597  3168 
0  5507  8148 
05119  7440 
0.5333  0814 
0.5247  8046 

0.5355  0183 
0  52(52  9172 
0  5172  4002 
0.5083  4400 
0.4996  0098 

0.5123  4867 
0.5029  1894 
0  1936  6277 
0.18457695 
0.17565836 

0.4902  2315 
0.4806  1093 
0.4711  8719 
0.4619  4822 
0  4528  9042 

41 
42 
43 
44 
45 

0.5431  1559 
0.5350  8925 
0.5271  8153 
0.5193  9067 
0.5117  1494 

0  5163  8914 
0.5081  3199 
0.5000  0688 
0.49201169 
0.4841  4434 

0  4910  0834 
0  4825  6348 
0  4742  6386 
0.4661  0699 
0.4580  9040 

0  4669  0391 
0.4583  1058 
0.4498  7542 
0.44159550 
0.4334  6798 

0.4440  1021 
0.4353  0413 
0.4267  6875 
0.4184  0074 
0.4101  9680 

46 
47 
48 
49 
50 

0.5041  5265 
0.4967  0212 
0.4893  6170 
0.4821  2975 
0.4750  0468 

0.4764  0280 
0.4687  8504 
0.4612  8909 
0.4539  1301 
0.4466  5487 

0.4502  1170 
0.4  1246850 
0.4348  5848 
0.4273  7934 
0.4200  2883 

0.4254  9004 
0.4176  5894 
0.4099  7196 
0.4024  2647 
0.3950  1984 

0.4021  5373 
0.3942  6836 
0.3865  3761 
0.3789  5844 
0.3715  2788 

PRESENT  VALUE  OF  1 
i 


52, 


vn  = 


(1  +  i)" 


n 

2y«% 

2V*% 

23/8% 

2V2% 

2%% 

1 

0.9791  9217 

0.9779  9511 

0  9768  0098 

0.9756  0976 

0.9732  3601 

2 

0.9588  1730 

0.9564  7444 

0.9541  4015 

0.9518  1440 

0  9471  8833 

3 

0.9388  6639 

0.9354  2732 

0.9320  0503 

0.9285  9941 

0.9218  3779 

4 

0.91  <«  3061 

0.9148  4335 

0  9103  8342 

090595064 

0.8971  6573 

5 

0.9002  0133 

0.8947  1232 

0.88<J2  6342 

0.8838  5429 

0.8731  5400 

6 

0.881  1  7009 

0.8750  2427 

0.8686  3337 

0  8622  968V 

0.8497  8191 

7 

0  8031  2861 

0  8557  6946 

08  484  Sl!>3 

0  SI  12  6524 

0.8270  4128 

8 

0.81,11  6878 

0  8369  3835 

0  8287  9798 

0  8207  4657 

0  8049  0635 

9 

0  8275  8264 

0.81  S3  2161 

0  S0l>:>  70(57 

0  8fX)7  2836 

0  7833  (538  "> 

10 

0.8103  6244 

0.8005  1013 

0.7907  8942 

0.7811  9840 

0.7623  9791 

11 

0  7935  0056 

0.7828  94  99 

0.7721  4388 

0.7621  4t78 

0.74191)310 

12 

0  7769  8953 

0  7(>.">(i(>748 

0  7~»i:>  2H94 

0.71355589 

0.7221  3440 

13 

0.7M)S  2206 

0  7lh8  11)05 

0  7370  1  972 

0  7254  2038 

0.7028  0720 

14 

0  71499100 

0  7o23  4137 

0.71992158 

0.7077  2720 

0  6839  9728 

15 

0.7294  8935 

0  7102  2628 

0.7032  2010 

0.6904  6556 

0.6656  9078 

16 

0  7143  1026 

0  700  1  6580 

0  6869  0609 

0  6736  2493 

06478  7421 

17 

0.6994  4701 

0  6850  5212 

0  «.7<)«>  7053 

0.0571  9506 

0.6305  3454 

18 

068189303 

0  <>W)9  77(>3 

().h.Ml  0467 

0.6411  6591 

0.6136  5892 

19 

0.6706  4189 

0  6552  3  184 

06101  9993 

0  6255  2772 

0.59723196 

20 

0.6566  8729 

0.64081(147 

0.6253  4791 

0.6102  7094 

0.5812  5057 

21 

0.6(30  2305 

0  6267  1538 

0.6108  4045 

0  5953  8629 

0.5656  9398 

22 

0  6296  4313 

0.6129  2457 

0  59<>6  6955 

0.5808  6467 

0.5505  5375 

23 

0.61(>5  4K)2 

05991  3724 

0  58282710 

0  5666  9724 

0  5358  1874 

24 

0.6037  1273 

0  5862  4668 

0.5693  0637 

0.5528  7535 

0.5214  7809 

25 

0.5911  5077 

0.5733  4639 

0.5560  9902 

0.5393  9059 

0.50752126 

26 

0  5788  5021 

0  5607  2997 

05131  9807 

0  5262  3472 

0.4939  3796 

27 

0  5668  0559 

054839117 

0  5305  9(540 

0  51  33  9973 

0.4807  1821 

28 

0.5550  1159 

0.5363  2388 

0.5182  8708 

0.5008  7778 

0.4678  5227 

29 

0.5434  6300 

0  52  i5  2213 

0.5062  6333 

0  18H6  6125 

0  4553  3068 

30 

0.5321  5471 

0.5129  8008 

0.4945  1852 

0.4767  4269 

0.4431  4421 

31 

0.5210  8173 

0  5016  9201 

0.4830  4617 

0.4651  1481 

0.4312  8391 

32 

0.51023915 

0.  1906  5233 

0  4?  1  8  3997 

0.4537  7055 

0  4197  4103 

33 

0.4996  2217 

0.  1  798  5558 

0  4608  9374 

044270298 

0  4085  0708 

34 

0.4892  2612 

0.4692  9641 

0  n(>2  OH6 

0.4319  0534 

0  3975  7380 

35 

0.4790  4638 

0.4589  6960 

0.4397  5722 

0.4213  7107 

0.3869  3314 

36 

0.4690  7846 

0.4488  7002 

0.4295  5529 

0.41109372 

0.3765  7727 

37 

0.4593  1796 

0.4389  9268 

0  1195  JXJ02 

0  4010  6705 

0  3664  9856 

38 

0.4497  6055 

0  4293  3270 

0.4098  5594 

0.3912  8492 

0  3566  8959 

39 

0.4404  0201 

0.4198  8528 

0  4003  4769 

0  3817  4139 

0.3471  4316 

40 

0.4312  3819 

0.4106  4575 

0.3910  6001 

0.3724  3062 

0.3378  5222 

41 

0.4222  6506 

0.40160954 

0.3819  8780 

0.3633  4695 

0  3288  0995 

42 

0.4134  7864 

0.3927  7216 

0.3731  2006 

0.3544  8483 

0.32M  0968 

43 

0.4048  7505 

0.3841  2925 

0.364  \  6990 

0.3458  3886 

03114  4495 

44 

0.3964  5047 

0.3756  7653 

0  3560  1455 

0.3374  0376 

0.3031  0944 

45 

0.3882  0120 

0.3674  0981 

0.3477  5536 

0.3291  7440 

0.2949  9702 

46 

0.3801  2357 

0.3593  2500 

0.3396  8778 

0.3211  4576 

0.2871  0172 

47 

0.3722  1402 

0.3514  1809 

033180735 

0.3133  1294 

0.2794  1773 

48 

0.3644  6906 

0.3436  8518 

0.3241  0975 

0.3056  7116 

0.2719  3940 

49 

0.3568  8524 

0.3361  2242 

0  3165  9072 

0.2982  J576 

0.26466122 

50 

0.3494  5924 

0.3287  2608 

0.3092  4612 

0.2909  4221 

0.2575  7783 

524 


APPENDIXES 

1 


v» 


(1  +  i) 


n 

3% 

3*4% 

3V2% 

33/4% 

4% 

\ 
2 
3 
4 

5 

0.9708  7379 
0.9425  9591 
0.9151  4166 
0  8884  8705 
0.8626  0878 

0.9685  2300 
0.9380  3681 
0.9085  1022 
0.87991305 
0.8522  1603 

0.9661  8357 
0.9335  1070 
0.9019  4271 
0.8714  4223 
0.8419  7317 

0.9638  5542 
0.9290  1727 
0.89.51  3834 
0  8630  7310 
0.8318  7768 

0.9615  3846 
0  9245  5621 
0.8889  9636 
0.85480419 
0.8219  2711 

6 
7 
8 
9 
10 

0.8374  8426 
0.8130  9151 
0.7894  0923 
0.7064  1673 
0.7440  9391 

0.8253  9083 
0.7994  1000 
0  7742  4698 
0.7498  7601 
0.7262  7216 

0.8135  0064 
0.7859  9096 
0.7594  1156 
0.7337  3097 
0.7089  1881 

0.80180981 
0  7728  2874 
0  71489517 
071797125 
O.G920  2048 

0.7903  1  153 
0.759917M 
0  730(5  9021 
0  7025  8674 
0.6755  G417 

11 
12 
13 
14 
15 

0.7224  2128 
0.7013  7988 
0.6809  5134 
0.6611  1781 
0.6418  6195 

0.7034  1129 
0.6812  7002 
0.6598  2568 
0.635)0  5635 
0.6189  4078 

0.6849  4571 
0  6617  8330 
063940415 
061778179 
0.5968  9062 

0  6670  0769 
0  6428  9898 
O.()l%(jl67 
O.W26I26 
0.5756  7639 

0.6495  8093 
0  0245  9705 
0  6005  71  09 
0.5774  7508 
0.5552  6450 

18 
17 
18 
19 
20 

0  6231  6694 
0.6050  1645 
O.f>873  9461 
0.5702  8603 
0.5536  7576 

0.5994  5838 
0.5805  8923 
0  5023  1402 
0.54161407 
0.5274  7125 

0.5767  0591 
0.5572  0378 
05383  6114 
0.5201  5569 
0.502o  6588 

0  5548  6881 
0  5348  1331 
0  5154  8271 
0.  1968  5080 
0.4788  9234 

0  5339  0818 
0  5133  7325 
0.1936281-2 
0.1746  4242 
0.4563  8095 

21 

22 
23 
24 
25 

0  5375  4928 
0.5218  9250 
0.50669176 
0.49193374 
0.4776  0557 

0  5108  6804 
0.4947  8745 
047921302 
0.4641  2884 
0.14951945 

0.4855  7090 
0.4691  5063 
0.4532  8.363 
0.13795713 
0.4231  4699 

0.4615  8298 
0  1448  9925 
0.4288  1856 
04U3  1910 
0.3983  7985 

0  4388  3300 
0.4219  5539 
0  4057  2633 
03901  2147 
0.3751  KiSO 

26 
27 
28 
29 
30 

0.4636  9478 
0  4501  8906 
0.4370  7675 
0.4243  4636 
0.4119  8676 

0  4353  6993 
0.4216  6579 
0.4083  9302 
0  3955  3803 
0.3830  8768 

0.4088  3767 
039501224 
0.3816  5434 
0.3687  4815 
0.3562  7811 

0.3839  8058 
0..-S701  0176 
<Ur>(i7  2159 
0.3438  3093 
0.331  1  0331 

0  3606  8923 
03168  1657 
0.3331  7747 
0  3206  5141 
0.3083  1867 

31 
32 
33 
34 
35 

0.3999  8715 
0.3883  3703 
0.3770  2625 
0.3660  4490 
0.3553  8340 

0  3710  2923 
0  3593  5035 
0.3480  3908 
0.3370  8385 
0.3264  7346 

0.3442  3035 
0.33258971 
0.3213  4271 
0.3104  7GOr> 
0.2999  7686 

0.3191  2187 
0  3078  7940 
0  2967  5123 
0  '2860  2528 
0.2756  8702 

0.2964  6026 
0.2850  5794 
0.27409117 
0.2635  5209 
0.2534  1547 

36 
37 
38 
39 
40 

0.3450  3243 
0.3349  8294 
0  3252  2615 
0.3157  5355 
0.3065  5684 

0.3161  9706 
0.3062  4413 
0  21)66  0448 
0.2872  6826 
0.2782  2592 

0.2898  3272 
0.28003161 
0  2705  GUM 
0.2014  1?50 
0.2525  7217 

0.2657  2242 
0.2561  1800 
0  2468  (5072 
0.2379  3805 
0.2293  3788 

0.2436  6872 
0  2342  9685 
0  2252  8543 
0  2166  2061 
0  2082  8904 

41 
42 
43 
44 
45 

0.2976  2800 
0.2889  5922 
0.2805  4294 
0.2723  7178 
0.2644  3862 

0.2694  6820 
0  2609  8615 
0.2527  7109 
0.2448  1462 
0.2371  0859 

024403137 
0  '2:^7  7910 
0.2278  0590 
0.2201  0-231 
0.2126  5924 

0.2210  4855 
0  2130  5885 
0.2053  5793 
0.1979  3535 
0.1907  8106 

0.2002  7793 
0.1925  7493 
0,1851  6820 
0.1780  4635 
0.1711  9841 

46 
47 
48 
49 
50 

0.2567  3653 
0.2492  5876 
0.2419  9880 
0.2349  5029 
0.2281  0708 

0.2296  4512 
0.2224  1658 
0  2154  1558 
0  2086  3494 
0.2020  6774 

0/2054  6787 
0.1985  1968 
0  1918  0645 
0.1853  2024 
0.1790  5337 

0.18388536 
0.1772  3890 
0.1708  3268 
0.1646  5800 
0.1587  0651 

0.1646  1386 
0.1582  8256 
0.1521  9476 
0.14634112 
0.1407  1262 

PRESENT  VALUE  OF  1 
1 


525 


n 

4V*% 

4y2% 

4*/4% 

5% 

6V2% 

1 
2 
3 
4 
5 

0.9592  3201 
0  9201  2721 
0  8826  1603 
0.8466  3408 
0.8121  11)02 

0.9569  3780 
0.9157  2995 
0.8762  9660 
0.8385  6134 
0.8024  5105 

0.9546  5394 
0.9113  6414 
0  8700  3737 
0  8305  8460 
0.7929  2086 

0  9523  8095 
0.9070  2948 
0.8638  37(50 
0  8227  0247 
0.7835  2617 

0.9478  6730 
0  8984  5242 
0  8516  136<i 
08072  1674 
0.7651  3135 

6 
7 
8 
9 
10 

0.7790  1105 
0  7472  5281 
0.71678926 
0  6875  6764 
0.6595  3780 

0.7678  9574 
0.7348  2846 
0  7031  8513 
0.6729  0443 
0.6439  2768 

0.7569  6502 
0  7226  3964 
0.6898  7077 
0  <r>s5  8785 
0.6287  2349 

0.7462  1540 
0.7106  8133 
0.6768  3936 
0  6446  0892 
0.0139  1326 

0.7252  4583 
0  6874  3681 
0  65159887 
0.61762920 
0.5854  3058 

11 
12 
13 
14 
15 

0.6326  4969 
0.6068  5822 
05821  1819 
0  5583  8676 
0.5356  2279 

0.6161  9874 
0  5896  6386 
05012  7164 
0  5399  7280 
0.5167  2044 

0  6002  1335 
0.5729  9604 
0  5470  1293 
0  5222  0804 
0.4985  2797 

0.5846  7929 
0.5568  3742 
0  5303  2135 
0.5050  6795 
0.4810  1710 

0.5549  1050 
0.5259  8152 
0.4985  6068 
0.4725  6937 
0.4479  3305 

16 
17 
18 
19 
20 

0  5137  8685 
0  1928  4110 
0  4727  4926 
0  4531  7650 
0.4349  8945 

0  4944  6932 
0  4731  7639 
0.4528  0037 
04333  0179 
0.41464286 

0  4759  2169 
0.4543  1051 

0.4337  3796 
0.41406965 
0.3952  9322 

0.4581  1152 
0.  1362  9669 
0.4155  2065 
0  3957  3396 
0.3768  8948 

0.4245  8101) 
0.  4024  4653 
0.3814  6590 
0.3615  7900 
0.3427  2890 

21 
22 
23 
24 
25 

0.4172  5607 
0.4002  4563 
0  3830  2866 
0.3682  7689 
0.3532  6321 

0.3967  8743 
0  3797  0089 
0  3633  5013 
0  3177  0347 
0.3327  3060 

0.3773  6823 
0  3602  5607 
0.3439  1987 
03283  2116 
03131  3624 

0.3589  4236 
0.3418  4987 
0.3255  7131 
0.3100  6791 
0.2953  0277 

0.3248  6158 
0.3079  2567 
0.2918  7267 
0.27665(550 
0.2022  3370 

26 
27 
28 
29 
30 

0  3388  6159 
0.3250  4709 
031179577 
0.2990  8467 
0.2868  9177 

0.3181  0218 
0  3046  9137 
0  2915  7069 
0  2790  1502 
0  2670  0002 

0.2992  2314 
0.2856  5455 
0  2727  0124 
0  2003  3531 
0.2485  3013 

0.2812  4073 
0  2678  4832 
0.2550  9364 
0.2429  4632 
0.2313  7745 

0.2485  6275 
0.23560450 
0.2233  2181 
0.21167944 
0.2006  4402 

31 
32 
33 
34 
35 

0.2751  9594 
0.2639  7692 
0.2532  1527 
0.21289235 
0.2329  902C 

0  2555  0241 
0  2  1  H  9991 
023397121 
0.2238  9589 
0.2142  5444 

0.2372  6027 
0.2265  0145 
0  2162  3050 
0  2004  2530 
0.1970  6473 

0.2203  5947 
0.2098  6617 
0  1998  7254 
0  1903  5480 
0.1812  9029 

0.1901  8390 
0.18026910 
0.17087119 
0.1619  6321 
0.  1535  1963 

36 
37 
38 
39 
40 

0.2234  9186 
0  2143  8068 
0  2056  4094 
0.1972  5750 
0.1892  1582 

0  2050  2817 
0  1961  9921 
0.1877  5044 
0.1796  6549 
0.1719  2870 

0.1881  2862 
0  1795  9772 
0.1714  5367 
0  1636  7893 
0.1562  5673 

0.17265741 
0.1641  3503 
0.1566  0530 
0.1491  4797 
0.1420  4568 

0.14551624 
0.1379  3008 
0.1307  3941 
0.12392362 
0.11746314 

41 
42 
43 
44 
45 

0  18150199 
0.1741  0263 
0.1670  0492 
0.1601  9657 
0.1536  6577 

0  1645  2507 
0.1574  4026 
0.1506  0054 
0.1441  7276 
0.1379  6437 

0.1491  7110 
0  1424  0078 
0.1359  4919 
0.1297  8443 
0.1238  9922 

0.1352  8160 
0.  1  L'88  3962 
0.1227  0440 
0.11686133 
0.1112  9651 

0.11133947 
0.1055  3504 
0  1000  3322 
0.0948  1822 
0.0898  7509 

46 
47 
48 
49 
50 

0.1474  0122 
0  1413  9206 
0.1356  2787 
0.13009868 
0.1247  9489 

0.1320  2332 
0.1263  3810 
0  12089771 
011569158 
0.1107  0965 

0.1182  8088 
0.1129  1731 
0.1077  9695 
0.1029  0878 
0.0982  4228 

0.1059  9668 
0.10094921 
0.0961  4211 
0.0915  6391 
0.0872  0373 

0.0851  8965 
0.0807  4849 
0.0765  3885 
0.0725  4867 
0.0687  6652 

526 


APPENDIXES 


n 

6% 

6»/a% 

7% 

8% 

9% 

1 
2 
3 
4 
5 

0.9433  9623 
0.8899  9644 
0.8396  1928 
0.7920  9366 
0.7472  5817 

0.9389  6714 
0.8816  5928 
0.8278  4909 
0.7773  2309 
0.7298  8084 

0.9345  7944 
0.8734  3873 
0.8162  9788 
0.7628  9521 
0.7129  8618 

0.9259  2593 
0.8573  3882 
0.7938  3224 
0.7350  2985 
0.6805  8320 

0.9174  3119 

0.8416  7999 
0.7721  8348 
0  7084  2521 
0.6499  3139 

6 
7 
8 
9 
10 

0.7049  6054 
O.C650  5711 
0.6274  1237 
0.5918  9846 
0.5583  9478 

0.6853  3412 
0.6435  0621 
0.60123119 
0.5673  5323 
0.5327  2604 

0.6663  4222 
0.6227  4974 
0.5820  0910 
0.5439  3374 
0.5083  4929 

0.6301  6963 
0.5834  9040 
0.5402  6888 
0.5002  4897 
0.4631  9349 

0.5962  6733 
0.5470  3424 
0.5018  GG28 
0.4604  2778 
0.4224  1081 

11 
12 
13 
14 
15 

0.5267  8753 
0.4969  6936 
0.4f>88  3902 
0.4423  0096 
0.4172  6506 

0  5002  1224 
0.4696  8285 
0  4410  1676 
0.4141  0025 
0.3888  2652 

0.4750  9280 
0.4440  1196 
0.4149  6445 
0.3878  1724 
0.3624  4602 

0.4288  8286 
0.3971  1376 
0  367(5  9792 
0  3404  6104 
0.31524170 

0  3875  3285 
0.3555  3473 
0  3261  7865 
0  2992  4647 
0  2745  3804 

16 
17 
18 
19 
20 

0.3936  4628 
0  3713  6442 
0.3503  4379 
0  3305  1301 
0.3118  0473 

0  3650  9533 
034281251 
0.3218  8969 
0.3022  4384 
0.2837  9703 

0.3387  3460 
0  3165  7439 
0.2958  6392 
0.2765  0832 
0.2584  1900 

0.2918  9047 
0  2702  6895 
0.2502  4903 
0.2317  1206 
0.2145  4821 

0  2518  6976 
0.2310  7318 
0.2119  9374 
0  1944  8967 
0.1784  3089 

21 

22 
23 
24 

25 

0.2941  5540 
0.2775  0510 
0.2617  9726 
0.2169  7855 
0.2329  9863 

0.2664  7608 
0.2502  1228 
0.2349  4111 
0.2206  0198 
0.2071  3801 

0.2415  1309 
0  2257  1317 
0.2109  4688 
0.1971  4G62 
0.1842  4918 

0.1986  5575 
0.1839  4051 
0.1703  1528 
0.1576  9934 
0.1460  1790 

0.1636  9806 
(I  1501  8171 
0.1377  8139 
0  1264  0191 
0.1159  6784 

26 
27 
28 
29 
30 

0.2198  1003 
0.2073  6795 
0  1950  3014 
018155674 
0.1741  1013 

0.1944  9579 
0.1826  2515 
0.1714  7902 
0.1610  1316 
0.1511  8607 

0.1721  9549 
0.16093037 
0.1501  0221 
0.1405  6282 
0.1313  6712 

0.1352  0176 
0  1251  8682 
0.1159  1372 
0.1073  2752 
0  0993  7733 

0.1063  9251 
0  0976  0781 
0  0895  4845 
0  0821  5454 
0.0753  7114 

31 
32 
33 
34 
35 

0.1642  5484 
0.154!)  5740 
0.1461  8622 
0.13791153 
0.1301  0522 

0.1419  5875 
0.1332  9460 
0.1251  5925 
0.1175  2042 
0.1103  4781 

0.1227  7301 
0.1147  4113 
0.1072  3470 
0.1002  1934 
0.0936  6294 

0  0920  1605 
0  0852  0005 
0  0788  8893 
0  0730  4531 
0.0676  3454 

0  0691  4783 
0.0634  3838 
0  0582  0035 
0.0533  9481 
0.0489  8607 

36 
37 
38 
39 
40 

0.1227  4077 
0  1157  9318 
0,10923885 
0.1030  5552 
0.0972  2219 

0.1036  1297 
0  0972  8917 
0.0913  5134 
0.0857  7590 
0.0805  4075 

0.0875  3546 
008180881 
0.0764  5686 
0.0714  5501 
0  0667  8038 

0.0626  2458 
0  0579  8572 
0.0536  9048 
0  0497  1341 
0.0460  3093 

0.0449  4135 
004123059 
0.0378  2623 
0  0347  0296 
0.0318  3758 

41 
42 
43 
44 
45 

0.0917  1905 
0.0865  2740 
0.0816  2962 
0.0770  0908 
0.0726  5007 

0.0756  2512 
0.0710  0950 
0.0666  7559 
0.0626  0619 
0.0587  8515 

0.0624  1157 
0.0583  2857 
0.0545  1268 
0.0509  4643 
0.0476  1349 

0.0426  2123 
0.0394  6411 
0.0365  4084 
0.0338  3411 
0.0313  2788 

0.0292  0879 
0.0267  9706 
0  0245  8446 
0.0225  5455 
0.0206  9224 

46 
47 
48 
49 
50 

0.0685  3781 
0.0645  5831 
0.0609  9840 
0.0575  4566 
0.0542  8886 

0.0551  9733 
0.0518  2848 
0.0486  6524 
0.0456  9506 
0.0429  0616 

0.0444  9859 
0.0415  8747 
0.0388  6679 
0.0363  2410 
0.0339  4776 

0.0290  0730 
0.0268  5861 
0.0248  6908 
0.0230  2693 
0.0213  2123 

0.0189  8371 
0.0174  1625 
0.0159  7821 
0.0146  5891 
0.0134  4854 

AMOUNT  OF  ANNUITY  OF  1 


527 


Table  4 
AMOUNT  OF  ANNUITY  OF  1 

(1  +  i)n  -  1 


"Sli                  j 

n 

1 
2 
3 
4 
5 

1.000  0000 
2  005  0000 
3.015  0230 
4.030  1001 
5.050  2506 

1% 

1  0000000 
20100000 
3.030  1000 
4.060  4010 
5.101  0050 

1  0000000 
2  OT2  5000 
3  037  656'- 
4.075  6270 
5.126  5723 

1.000  0000 
2.01  5  OlHW 
3  043  2230 
4.090  9034 
5.152  2660 

1.000  0000 
2  017  5000 
3  052  8M3 
4.106  2304 
5.178  0894 

2% 

1.0000000 
2.020  1HXK) 
3  0(50  4<K)0 
4  121  6080 
5.204  0402 

6 
7 
8 
9 
10 

6.075  5019 
7.103  8794 
8.141  4088 
9.182  1158 
10.228  0264 

6.152  0151 
7.213  5352 
8.285  6706 
9.368  5273 
10.462  2125 

6.190  6544 
7.268  0376 
8.358  8881 
9.463  3742 
10.581  6664 

6.229  5509 
7  322  9942 
8  432  8391 
9.559  3317 
10.702  7217 

6.268  7060 
7  378  4083 
8  507  5305 
9.(>56  4122 
10.825  3993 

6.308  1210 
7.134  2831 
8  5829691 
9  751  6'284 
10.949  7210 

11 
12 
13 
14 
15 

11  279  1665 
12  335  5624 
13.397  2402 
14.464  2264 
15  536  5475 

11.566  8347 
12.682  5030 
13  809  3280 
14  947  1213 
16  O')0  8955 

11.713  9372 
12.800  3614 
14.021  1159 
15.196  3799 
16.386  3346 

11  863  2625 
13041  2114 
14  236  8296 
15.450  3821 
16.682  1378 

12.0148139 
13.225  1037 
14  456  5430 
15.709  5325 
16.984  4494 

12.168  7151 
13.412  0897 
14  6803313 
15.973  9382 
17.293  4169 

16 
17 
18 
19 
20 

16  614  2303 
17.697  3014 
18.785  78~9 
19  879  7168 
20.979  1154 

17  257  8615 
18.4304431 
19  611  7476 
20  810  8930 
22.019  0040 

17  591  1638 
18  811  0534 
20016  1915 
21  '2<)6  7689 
22.562  9785 

17.932  3698 
19.201  3554 
20  489  3757 
21.75)6  7164 
23.123  6671 

18.281  6772 
19  601  6066 
20.911  6347 
22  3  1  1  1  658 
23.701  6112 

18.639  2853 
20.0120710 
21.4123121 
22.840  5586 
24.297  3698 

21 
22 
23 
24 
25 

22  084  01  10 
23  194  1311 
24  310  4032 
26.431  9552 
26.559  1150 

23239  1910 
24  471  5860 
25  716  3018 
20.973  4648 
28  243  1995 

23  8450138 
25  143  0785 
26  437  3(»70 
27  788  0840 
29.135  4351 

24.470  5221 
25  837  5799 
27.225  1186 
28  633  5208 
30  063  0236 

25.1163894 
26  335  92(52 
28.0  '20  6549 
29  511  0164 
31.027  4592 

25.783  3172 

27  298  9835 
28.844  9()32 
30.421  8625 
32.030  2907 

26 
27 
28 
29 
30 

27  691  9106 
28.830  8702 
29.974  5220 
31  124  3946 
32.280  0166 

295256315 
30.820  8878 
32.1290967 
33  450  3877 
34.784  8915 

30.  499  (3280 
31.8808734 
33  279  3843 
31  695  3766 
36.129  0688 

31  513  9690 
32  986  6783 
84.481  4787 
35  998  7009 
37.538  6614 

32.570  4397 
31.1404224 
35.737  8798 
37  303  2927 
39.017  1503 

33.670  9037 
35.344  3238 
87.051  2103 
38.792  2315 
40.568  0792 

31 
32 
33 
34 
35 

33.441  4167 
34  008  6238 
35  781  6669 
36  960  5752 
38.145  3781 

36.132  7404 
37.494  0678 
38  869  0085 
40.257  6986 
41.660  2756 

37.580  6822 
39  050  4407 
40.538  5712 
42.015  3033 
43.570  8696 

39.101  7616 
40.688  2880 
42  298  6123 
43.983  0915 
45  592  0879 

40.699  9504 
42.4121996 
41.151  4131 
45.927  1153 
47.730  8398 

42.3794108 
4  1  227  029(5 
46  1  1  1  5702 
48.033  80  16 
49994  4776 

36 
37 
38 
39 
40 

39.336  1050 
40.532  7855 
41  735  4494 
42.944  1267 
44.158  8473 

43  076  8784 
44  507  6471 
45.952  7236 
47  412  2308 
48.886  3734 

45115  5055 
46  679  4493 
48  262  9424 
49.866  2292 
51.489  5571 

47.275  9692 
48.985  1087 
50  719  8854 
52  480  6837 
54.267  8939 

49.566  1295 
51.433  5368 
53.333  623(5 
65  2C6  9621 
67.234  1339 

51  994  3672 
54  034  2545 
5(5  1  1  1  939(5 
58  237  2384 
60.401  9832 

41 
42 
43 
44 
45 

45.379  6415 
46.606  5397 
47.839  5724 
49.078  7703 
56.324  1642 

50.375  2371 
51.878  9895 
53  397  7794 
54.931  7572 
56.481  0747 

53  133  1765 
54  797  3412 
56  482  8080 
58.188  3369 
59.9156911 

56.081  9123 
57.923  1410 
59  791  9881 
61.688  8679 
63.614  2010 

59  235  7312 
61  272  3565 
63  344  fi228 
65.453  1537 
67.598  5839 

62.610  0228 
64.862  2233 
67  159  4678 
69.502  6571 
71.892  7103 

46 
47 
48 
49 
50 

51.575  7850 
52  833  6639 
54.097  8322 
55.368  3214 
56.645  1630 

58.045  8855 
59.626  3443 
61.222  6078 
62.834  8338 
64.463  1822 

61.664  6372 
63.435  4452 
65.228  3882 
67.043  7431 
68.881  7899 

65.568  4140 
67.551  9402 
69.565  2193 
71  608  6976 
73.682  8280 

69.781  5591 
72  002  7364 
74.262  7843 
76.562  3830 
78.902  2247 

74.330  5645 
76  817  1758 
79.353  5193 
81.940  5897 
84.679  4016 

528 


APPENDIXES 

(1  +  i)°  -  1 


n 

2V4% 

2V3% 

2'/4% 

3% 

3V2% 

4% 

1 
2 
3 

4 
5 

1.0000000 
2.022  6000 
3.068  0063 
4.137  0364 
6.230  1197 

1.0000000 
2.025  0000 
3.075  6250 
4.152  5156 
5.256  3285 

1.0000000 
2.027  5000 
3.083  2563 
4.168  0458 
5.282  6671 

10000000 
2  030  0000 
3.090  9000 
4.1836270 
5.309  1358 

1.0000000 
2.035  0000 
3.106  2250 
4.214  9429 
5.362  4659 

1.000  0000 
2.040  0000 
3.121  6000 
4.246  4640 
5.416  3226 

6 
7 
8 
9 
10 

6.347  7974 
7.490  6228 
8.659  1619 
9.853  9930 
11.075  7078 

6.387  7367 
7.547  4302 
8.736  1159 
9.954  5188 
11.203  3818 

6.427  9404 
7.604  7088 
8.813  8383 
10  056  2188 
11.332  7648 

6.468  4099 
7.662  4622 
8.892  3361 
10.159  1061 
11.463  8793 

6.550  1522 
7.779  4075 
9  Ool  6868 
10  368  4958 
11.731  3932 

6.632  9755 
7.898  2945 
9  214  220.1 
10.582  7953 
12.006  1071 

11 
12 
13 
14 
15 

12.324  9113 
13.602  2218 
14.908  2718 
16  243  7079 
17.609  1913 

12.483  4663 
13.795  5530 
15.1404418 
16  518  9528 
17.931  9267 

12  6U  4159 
13.992  1373 
15.3769211 
16  799  7864 
18.261  7805 

12  807  7957 
11  1920296 
15.617  7905 
17.086  3242 
18.598  9139 

13.141  9919 
14.601  9616 
16.1130303 
17  676  9864 
19.295  6809 

13.486  3514 
15.025  8035 
16.626  8377 
182919112 
20.023  5876 

16 
17 
18 
19 
20 

19.0053981 
20.433  0196 
21.892  7625 
23.385  3497 
24.911  5200 

19.380  2248 
20.8(54  7305 
22  386  3187 
23  91  6  0074 
25.544  6576 

10.763  9795 
21  307  4889 
22  893  4449 
24.5230116 
26.197  3975 

20.156  8813 
21.701  5877 
23  414  4354 
2.')  1168684 
26.870  3745 

20  971  0297 
22  705  0158 
24  499  6913 
26  357  1805 
28.279  6818 

21.8245311 
23  097  5121 
25  6454129 
27  671  22'  n 
29.778  0786 

2l 
22 
23 
24 

25 

26  472  0292 
28  067  6499 
29  699  1720 
31.3674034 
33.073  1700 

27.183  2711 

28  8(i2  8559 
30  584  4273 
32  349  0380 
34.157  7639 

27.917  8259 
29.08,-)  5662 
31  501  9192 
33  368  2220 
35.285  8481 

28  676  4857 
30  53(5  7SOH 
32  152  8837 
34  426  4702 
36.459  2643 

30  269  4707 
32  328  9022 
31  1604137 
36  606  5282 
38.949  8567 

31.969  2017 
34  247  9f>98 
36.617  8886 
39.082  6041 
41.645  9083 

26 
27 
28 
29 
30 

34.8173163 
36.600  7059 
38  424  2218 

40  28H  7668 
42.195  2640 

36.011  7080 
37.912  0007 
39  859  8008 
41.8562958 
43.902  7032 

37.2.56  2089 
39.280  7547 
41.360  9754 
43.498  4022 
45.694  6083 

38.553  0423 
40.709  6335 
42  930  922.1 
45  218  8:j02 
47.575  4157 

41.313  1017 

43.759  0602 
46.290  6273 
48  910  79U3 
51.622  6773 

44.311  7446 
47  084  2144 
49.967  5830 
52.966  2803 
56.084  9378 

31 
32 
33 
34 
35 

44.144  6575 
46.137  9123 
48.1760153 
60.259  9756 
62.390  8251 

46.000  2707 
48.150  2775 
50351  0345 
52.612  8853 
64.928  2074 

47.951  2100 
50.269  8683 
52  652  2897 
55.100  2277 
57.615  4839 

50  002  6782 
52.502  7585 
55.0778113 
57.730  1765 
60.462  0818 

54.429  4710 
57  334  5025 
60.341  2101 
63  453  152  1 
66.674  0127 

59.328  3353 
62.701  4687 
firt  209  5274 
69  857  9085 
73  052  2249 

36 
37 
38 
39 
40 

54.569  6186 
56.797  1351 
59.075  3774 
61.4045733 
63.786  1762 

57.301  4126 
59.733  9479 
62.227  2966 
64  782  9791 
67.402  5535 

60.199  9097 
62.8:>r>  4072 
65.583  9309 
68.387  4890 
71.2(58  1450 

63.275  9443 
66.174  2226 
69.1594193 
72  2H4  2328 

75.401  2597 

70.007  6032 
73  457  8693 
77.028  8947 
80  724  90(50 
84.550  2778 

77.598  3139 
81.702  2464 
85  970  3363 
90  409  1497 
95.025  5157 

41 
42 
43 
44 
45 

66.221  3652 
68.711  3459 
71.257  3512 
73.860  6416 
76.522  5061 

70.087  6174 
72.839  8078 
75.600  8030 
78.552  3231 
81.516  1312 

74.228  0190 
77.26')  2895 
80  394  1950 
83  605  0353 
86.904  1738 

78.663  2975 
82.023  1965 
85.483  8923 
89.048  4091 
92.719  8614 

88.509  5375 
92.607  3713 
96.848  6293 
101.238  3313 
105.781  6729 

99.826  53(s3 
104.819  5978 
110.0123817 
115.412  8770 
121.029  3920 

46 
47 
48 
49 
50 

79.244  2624 
82.027  2583 
84.872  8717 
87.782  5113 
90.757  6178 

84.554  0344 
87.667  8853 
90.859  5824 
94.131  0720 
97.484  3488 

90.294  0386 
93.777  1246 
97  355  9056 
101.033  2854 
104.811  7008 

96  501  4572 
100.396  5010 
104.408  3960 
108.540  6479 
112.796  8673 

110.484  0315 
115.350  9726 
1203882566 
125.601  8456 
130.997  9102 

126.870  5677 
132  945  3904 
139.263  2060 
145.833  7343 
152.667  0837 

AMOUNT  OF  ANNUITY  OF  1 

(1  +  i)n  -  i 


529 


S51i 


i 


n 

4Va% 

5% 

5V2% 

6% 

7% 

8% 

1 

1.0000000 

1.000  0000 

1.0000000 

1.0000000 

1.000  0000 

1.0000000 

2 

2.045  0000 

2.050  0000 

2.053  0000 

2.060  0000 

2.070  0000 

2.080  0000 

3 

3.137  0250 

3.152  5000 

3.168  0250 

3.183  6000 

3  214  9000 

3.246  4000 

4 

4.278  1911 

4  310  1250 

4.342  2664 

4.374  6160 

4.439  9430 

4.506  1120 

5 

5.470  7097 

5.525  6313 

5.581  0910 

5.637  0930 

5,750  7390 

5.866  6010 

6 

6.716  8917 

6801  9128 

6  888  0510 

6.975  3185 

7.153  2907 

7.335  92vO 

7 

8.0191518 

8.1420085 

8.206  8938 

8.393  8377 

8.6540211 

8.922  8034 

8 

9.380  0136 

9.549  1089 

9.721  5730 

9.897  4679 

10.259  8026 

10.63C  6276 

9 

10602  1142 

11  026  5643 

11  2562595 

11.491  3160 

11.977  9888 

12.487  5578 

10 

12.288  2094 

12.577  8925 

12.8753538 

13.180  7949 

13.816  4480 

14.486  5625 

11 

13.841  1788 

14.206  7872 

14  583  4983 

14.971  6426 

15.783  5993 

16.645  4875 

12 

15.464  0318 

15.917  1265 

10.385  5907 

168699412 

17.888  4513 

18.977  1265 

13 

17.159  9133 

17.712  9829 

18  '286  7981 

18.882  1377 

20  140  0429 

21.495  2966 

14 

18.932  1094 

19.598  6320 

20.292  5720 

21.0150659 

22.550  4879 

24.214  9203 

15 

20.784  0543 

21,578  5636 

22.408  0635 

23.275  9699 

25.129  0220 

27.152  1139 

16 

22  719  3367 

23.657  4918 

24  641  1400 

25  672  5281 

27  888  0536 

30.324  2830 

17 

24  741  7069 

25  840  3004 

20.990  4027 

282128798 

30.840  2173 

33.750  2257 

18 

20  855  0837 

28.132  3847 

29.481  2048 

30  905  6526 

33.999  0325 

37.450  2437 

19 

29.003  5025 

30  539  0039 

321020711 

33.7599917 

37  378  9648 

41.446  2632 

20 

31.371  4228 

33.005  954  1 

34.8083180 

30.785  5912 

40.995  4923 

45.761  9643 

21 

33  783  1368 

35.719  2518 

37.786  0755 

39.992  7267 

44.865  1768 

50.422  9214 

22 

36.303  3780 

38  50:>  2144 

40  8(54  3097 

43  392  2903 

49.005  7392 

55.456  7552 

23 

38.937  0300 

41  130  4751 

44  111  8107 

46.995  8277 

53  436  1409 

60.893  2950 

24 

41.0891903 

44  501  9989 

47  537  9983 

50.81o  5/74 

58.1766708 

66.764  7592 

25 

44.565  2102 

47.727  0988 

51.152  588ii 

54.864  5120 

63.249  0377 

73.105  9400 

26 

47.570  6146 

51.1134538 

54.965  9805 

69.156  8827 

68.676  4704 

79.954  4152 

27 

50.711  3236 

r>4  6091265 

58  989  1094 

63.705  7657 

74.483  8233 

87.350  7684 

28 

53.993  3332 

58  102  5828 

63  233  5105 

68628  1116 

80.697  6909 

95.338  8298 

29 

57.423  0332 

62.3227119 

67.711  3535 

73.639  7983 

87.3165298 

103.965  9362 

30 

61.007  OG97 

66.438  8475 

72.435  4780 

79.058  1862 

94.400  7803 

113.283  2111 

31 

64.7523878 

70  760  7899 

77.4194293 

84.801  6774 

102  073  0414 

123.345  8680 

32 

68.666  2452 

75.298  8294 

82.677  4979 

90.889  7780 

110  218  1543 

134.213  6374 

33 

72  756  2263 

80  063  7708 

8H  224  7603 

97.343  1647 

118.933  4251 

145  950  6204 

34 

77.030  2565 

85  066  9594 

94.077  1221 

104.183  7546 

128.258  7648 

158.0266701 

35 

81.4966180 

90.320  3074 

100.251  3638 

111.434  7799 

138.236  8784 

172.310  8037 

36 

86.163  9658 

95  836  3227 

106  765  1888 

119.120  8667 

i  48.91  3  4598 

187.1021480 

37 

91  Oil  3*43 

101.628  1389 

113637  2742 

127.268  1187 

160.337  4020 

203.070  3198 

38 

96.138  2048 

107.709  5458 

120  887  3243 

135901  2058 

172  561  0202 

220.3159454 

39 

101.464  4240 

1140950231 

128.530  1271 

145.058  4581 

185.640  2916 

238.941  2210 

40 

107.030  3231 

120.799  7742 

136.605  6141 

151.701  9656 

199.635  1120 

259.056  5187 

41 

112.846  6876 

127  839  7630 

14o.ll8  9229 

165.047  6836 

214.609  5698 

280.781  0402 

42 

118.924  7885 

135  23  1  7511 

154.100  4636 

175.950  5446 

230.632  2397 

304.243  5234 

43 

125  276  4040 

142.993  3387 

163.575  9891 

187.507  5772 

247.776  4695 

329.583  0053 

44 

131.913  8422 

151.143  0056 

173.5726685 

199.758  0319 

266.120  8513 

356.949  6457 

45 

138.849  9651 

159.700  1559 

184.119  1653 

212.743  5138 

285.749  3108 

386.505  6174 

46 

146.098  2135 

168.685  1637 

195.245  7194 

226.508  1246 

306.751  7626 

418.4260668 

47 

153.672  6331 

178.119  4219 

206.984  2339 

241.0986121 

329.224  3860 

452.900  1521 

48 

161.5879016 

188  025  3929 

219.368  3668 

256.564  5288 

353.270  0930 

490.1321643 

49 

169.859  3572 

198.426  6626 

232.433  6270 

272.958  4006 

378.998  9995 

530.342  7374 

50 

178  503  0283 

209  347  9957 

246.217  4765 

290.335  9046 

406.528  9295 

673.770  1564 

510 


APPENDIXES 


Table  6 
PRESENT  VALUE  OF  ANNUITY  OF  1 

1  * 

1~(T+l)i 


n 

y*% 

1% 

1V4% 

ll/2% 

!3/4% 

2% 

1 

2 
3 
4 
5 

0.99.r>  0249 
1  .985  0994 
2  970  2481 
3.950  4957 
4.923  8663 

0.990  0990 
1.970  3951 
2.940  9852 
3  901  9656 
4.853  4312 

0.987  6543 
1.963  1154 
2.926  5337 
3.878  0580 
4  817  8360 

0.985  2217 
1.955  8834 
2.912  2004 
3.854  3847 
*.782  6450 

0.982  8010 
1.948  6988 
•2  897  9840 
3  830  9425 
4.747  8551 

0.980  3922 
1  941  5609 
2.883  8833 
3  807  7287 
4.713  4595 

6 

7 
8 
9 
10 

5  896  3844 
6  862  0740 
7.822  9592 
8.779  0639 
9.730  4119 

5.795  4765 
6.728  1945 
7.651  6778 
85660176 
9.471  3045 

5.746  0099 
6  662  7258 
7  568  1243 
8  462  3150 
9.345  5259 

5  697  1872 
6  598  21  10 
7  485  9251 
8  300  5173 
9.222  1846 

5.618  9976 
6534  641! 
7.405  0530 
8  200  4943 
9.101  2229 

5,601  4309 
6  471  9911 
7.325  4814 
8  102  2367 
8.982  5850 

11 
12 
13 
14 
15 

10.677  0267 
11  618  9321 
12  556  1513 
13.488  7078 
14.416  C246 

10.367  6282 
11.255  0775 
12  133  7401 
13  003  7030 
13.865  05?5 

10.217  8034 
11.079  3120 
11  930  1847 
12.770  5528 
13.600  5459 

10.071  1178 
10.907  5052 
11.731  5322 
12.543  3815 
13.343  2330 

9.927  4918 
10.739  5497 
11.537  6410 
12  322  0059 
13  092  8805 

9.786  8481 
10  575  3412 
11  348  3738 
12  106  2488 
12.849  2035 

16 
17 
18 
19 
20 

15.339  9250 
16  258  6319 
17.172  7680 
18.082  3562 
18.987  4192 

14.717  8738 
15  562  2513 
16  398  2686 
17.226  0085 
18.045  5530 

14.420  2923 
15  22»  9183 
16  029  5489 
16.819  3076 
17.599  3161 

14  131  2641 
14  907  0493 
15  672  5609 
16.420  1684 
17.1686388 

13.850  4968 
14  5'>5  0828 
15.326  8627 
160460567 
16.752  8813 

13.577  7093 
14.291  8719 
14.9U2  0313 
15  678  4620 
16.351  4333 

21 
22 
23 
24 
25 

19  887  9792 
20.784  0590 
21  675  6tJ06 
22.562  8(562 
23.445  6380 

18  856  9831 
19.660  3793 
20.455  8211 
21.243  3873 
22.023  1557 

18  369  6950 
19.130  5629 
19  882  0374 
20.624  2345 
21.357  2686 

17.900  1367 
18.620  8244 
19  330  8615 
20.030  4054 
20.7196112 

17.447  5492 
18.130  2095 
18  801  2476 
19  400  6857 
20.108  7820 

17  Oil  2092 
17.658  0482 
18  292  2041 
18.913  9250 
19.523  4505 

26 
27 
28 
29 
SO 

24.324  0179 
25.198  0278 
?(J.067  6894 
26.933  0242 
27.794  0540 

22.795  2037 
23.551)  6076 
24  ,*Ki  1132 
25.065  7853 
25.807  7082 

22  081  2530 
22  790  2992 
23  502  5178 
24.200  0176 
24.888  9062 

21  398  6317 
22.067  6175 
22  726  7167 
23.376  075d 
24.015  8380 

20.745  7317 
21  371  7264 
21.9869547 
22  591  6017 
23.185  8493 

20.121  0358 
20  706  8978 
21.281  2721 
21  844  3847 
22.396  4556 

31 
32 
33 
34 
35 

28.650  8000 
29.503  2836 
30.351  5259 
31.195  5482 
32.035  3713 

26.542  2854 
27.269  5895 
27.989  6926 
28.702  6659 
29.408  5801 

25.569  2901 
20  241  2742 
26.904  9622 
27.560  4564 
28  207  8582 

24.646  1458 
25.267  1387 
25.878  9544 
26.481  7285 
27.075  5946 

23.769  8765 
24.343  8590 
24.907  9695 
25.462  3779 
26.007  2510 

22.937  7015 
23.468  3348 
23.988  5636 
24.498  5917 
24.998  6193 

36 
37 
38 
39 
40 

32.871  0162 
33.702  5037 
34.529  8544 
35.353  0890 
36.172  2279 

30  107  5050 
30.799  5099 
31.484  6633 
32  163  0330 
32.834  6861 

28  847  2674 
29.478  7826 
30  102  5013 
30.718  5198 
31.326  9332 

27.660  6843 
28  237  1274 
28.805  0516 
29.364  5829 
29.915  8452 

26  542  7528 
27.009  0446 
27.586  2846 
28.094  6286 
28.594  2296 

25.488  8425 
25.969  4f>34 
26.440  6406 
26.902  5888 
27.355  4792 

41 

42 
43 
44 
45 

36.987  2914 
37.798  2999 
38.605  2735 
39.408  2324 
40.207  1964 

33.499  6892 
34.158  1081 
34.810  0081 
35.455  4535 
36.094  5084 

31.927  8352 
32.521  3187 
33.107  4753 
33.686  3954 
34.258  1682 

30.458  9608 
30.994  0500 
31.521  2316 
32.040  6222 
32.552  3372 

29.085  2379 
29.567  8014 
30.042  0652 
30.508  1722 
30.966  2626 

27.799  4895 
28  234  7936 
28.661  5623 
29.079  9631 
29.490  1599 

46 
47 
48 
49 
50 

41.002  1855 
41.793  2194 
42.580  3178 
43.363  5003 
44.142  7864 

86.727  2361 
37.353  6991 
37.973  9595 
38.588  0787 
39.196  1175 

34.822  8822 
35.380  6244 
35.931  4809 
36.475  5867 
37.012  8757 

33.056  4898 
33.553  1920 
34.042  5537 
34.524  6834 
34.999  6881 

31.416  4743 
31.858  9428 
32.293  8013 
32.721  1806 
33.141  2095 

29.892  3136 
30.286  5820 
30.673  1196 
31.052  0780 
31.423  6059 

PRESENT  VALUE  OF  ANNUITY  OF  1 

i 


531 


"; 

^          (1 

.  +  i)n 

3i      " 

i 

• 

| 

n 

2J/4% 

2Va% 

2%% 

3% 

3Va% 

...         « 
4% 

1 

0.977  9951 

0.975  6098 

0.973  2360 

0.970  8738 

0  966  1830 

OJ615385 

2 

1.934  4696 

1.927  4242 

1.920  4243 

1.913  4697 

1.899  6943 

1  886  0947 

3 

2  869  8969 

2.856  0236 

2.842  2621 

2  828  fil!4 

2  801  6370 

2  775  0910 

4 

3.784  7402 

3.761  5)742 

3.739  4279 

3.717  0984 

3.673  0792 

3.629  8952 

5 

4.679  4525 

4.645  8285 

4.612  5819 

4.579  7072 

4.515  0524 

4.451  8223 

6 

5.554  4768 

5.508  1254 

5.462  3668 

5.417  1914 

5  328  5530 

5242  1369 

7 

6.410  2463 

6.349  3906 

6  289  4081 

6  230  2830 

6  114  5440 

6  002  0547 

8 

7.247  1846 

7.170  1372 

7.094  3144 

7.019  6922 

6.873  9555 

6.732  7449 

9 

8.065  7062 

7.970  8655 

7.877  6783 

7.786  1089 

7.607  6865 

7.135  3316 

10 

8.866  2164 

8.752  0639 

8.640  0762 

8.530  2028 

8.316  6053 

8.110  8958 

11 

9.649  1113 

9.514  2087 

9  382  0693 

9  252  6241 

9  001  5510 

8  760  4767 

12 

10.414  7788 

10.257  7646 

10.104  2037 

9.954  0040 

9.663  3343 

9.385  0738 

13 

11  163  5979 

10.9831850 

108070109 

10.634  9553 

10.302  7385 

9.985  6479 

14 

11.895  9392 

11.690  0122 

11.491  0081 

11.296  0731 

10.920  5203 

10.563  1229 

15 

12  612  1655 

12.381  3777 

12.156  6989 

11  937  9351 

11.517  4109 

11.118  3874 

16 

13.312  6313 

13.055  0027 

12  804  5732 

12  561  1020 

12.094  1168 

11.652  2956 

17 

13  997  6834 

13.712  1977 

13.435  1077 

13.106  1185 

12.651  3206 

12.165  d689 

18 

14  667  6611 

14  353  3636 

11.018  7666 

13.753  5131 

13.1896817 

12.659  2970 

19 

15.322  8959 

14.978  8013 

14  646  0016 

14.323  7991 

13.709  8374 

13.133  9394 

20 

15.963  7124 

15.589  1623 

15.227  2521 

14.877  4749 

14.212  4033 

13.590  3263 

21 

16.590  4278 

16.181  5486 

15.792  9461 

154150241 

14  697  9742 

14.029  1600 

22 

17.203  3523 

16  765  4132 

16  343  4999 

15.9369166 

15  167  1248 

14.451  1153 

23 

17.802  7896 

17  332  1105 

168793186 

16443  (K)84 

15  620  4105 

14.856  84  17 

24 

18  389  0362 

17.884  9858 

17.400  7967 

16  935  5421 

16.058  3676 

15.246  9631 

25 

18.962  3826 

18.424  3764 

17.908  3180 

17  413  1477 

1(U81  5140 

15.622  0799 

26 

19.523  1126 

18.9506111 

18  402  2559 

17  8768424 

16.890  3523 

15  9H2  7692 

27 

20.071  5038 

19.464  0109 

18.8H2  9741 

18327  0315 

17  285  3645 

18  32^5858 

28 

20.  (507  8276 

19  Wi4  8887 

19.350  8264 

18  764  1082 

176670189 

16.663  0632 

29 

21.1323198 

20.453  5499 

19  80(5  1571 

19  188  4546 

18  035  7670 

16983  714<5 

30 

21.645  3299 

20/J30  2926 

20.249  3013 

19.600  4414 

18.392  0454 

17.292  0333 

31 

22.147  0219 

21.395  4074 

20.680  5852 

20  000  4285 

18  736  2758 

17.588  4936 

32 

22.637  6742 

21.819  1780 

21.100  3262 

20.388  7655 

19.068  8655 

17873  5515 

33 

23  117  5298 

22  291  8809 

21.508  8333 

20  705  7918 

19.390  2082 

18  147  0457 

34 

23.586  8262 

22.723  7863 

21.906  4071 

21.131  8367 

19.700  6842 

18.111  I97H 

35 

24.045  7958 

23.1451573 

22.2U3  3403 

21.487  2201 

20.000  6611 

18.661  (5132 

36 

24.494  6658 

23.556  2511 

22.6699175 

21.8322525 

20.290  4938 

18.908  2820 

37 

24.933  6585 

23  057  3181 

23  03C  4161 

22  167  2354 

20.570  5254 

10  142  57HH 

38 

25  362  9912 

24.348  6030 

23.393  1057 

22.492  4616 

20.841  0874 

19.367  8642 

39 

25.782-3765 

24.730  3444 

23  740  2488 

22.808  2151 

21.1024999 

19.584  4H48 

40 

26.193  5222 

25.102  7751 

24.078  1011 

23.114  7720 

21.355  0723 

19.792  773U 

41 

26.505  1317 

25.466  1220 

24  40t)  9110 

23.412  4000 

21.599  1037 

19.9930518 

42 

26.987  9039 

25.820  6068 

24.726  9207 

23.701  3592 

21.834  8828 

20.185  (>267 

43 

27.372  0332 

26.166  4457 

25  038  3656 

23  981  9021 

22.062  6887 

20  370  7949 

44 

27.747  7097 

26  503  8495 

25.341  4751 

24.254  2739 

22.282  7910 

20.548  8413 

45 

28.115  1195 

26.833  0239 

25.636  4721 

24.518  7125 

22.495  4503 

20.720  0397 

46 

28.474  4445 

27.1541696 

25  923  5738 

24.775  4491 

22.700  9181 

20.884  6536 

47 

28.825  8626 

27.467  4826 

26  202  9915 

25.024  7078 

22.899  4378 

21.042  9361 

48 

29.169  5478 

27.773  1537 

26.474  9309 

25  266  7066 

23.091  2443 

21.195  1309 

49 

29.505  6702 

28.071  3695 

26.739  5922 

25  501  6569 

23.276  5645 

21.341  4720 

50 

29.834  3963 

28.362  3117 

26.997  1700 

25.729  7640 

23.455  6179 

21.482  1846 

532 


APPENDIXES 

i 


l~ 


n 

4y3% 

5% 

5'/2% 

6% 

7% 

8% 

1 
2 
3 
4 
5 

0.956  0378 
1.872  6678 
2.748  9644 
3.587  5257 
4.389  9767 

0.952  3810 
1.859  4104 
2.723  2480 
3.545  9505 
4.329  4767 

0.947  8673 
1  8463197 
2  ($97  9334 
3  505  1501 
4.270  2845 

0.943  3962 
1  S33  3927 
2.073  0120 
3  1(5.')  1056 
4.2123638 

0  934  5794 
1.808  0182 
2.624  3160 
33872113 
4.100  1974 

0.925  9259 
1.783  2648 
2  577  0970 
3.312  12(58 
3.992  7100 

6 
7 
3 
9 
10 

5.157  8725 
5.892  7(309 
6  505  8861 
7.268  7905 
7.912  7182 

5.075  6923 
5.78G  3731 
0.4(53  2128 
7.107  8217 
7.721  7349 

4  995  5303 
5.682  9671 
6  334  5660 
G952  1953 
7.537  6258 

4  9173243 
5.5H2  3814 
6  U09  7(J38 
0.801  6923 
7.300  0871 

4.7(56  5397 
5  389  2894 
5.971  2985 
65152323 
7.023  581C 

4.622  8797 
5.206  3701 
5  746  6389 
6  246  887!) 
6.710  0814 

11 
12 
13 
14 
15 

8.528  9169 
9  118  5808 
9.682  8524 
10.222  8253 
10.739  5457 

8.306  4142 
8.863  2516 
9.393  5730 
9  808  6  109 
10.379  6580 

8  092  5363 
8.618  5179 
9.1170785 
9  589  6479 
10.037  5809 

7.886  8746 
8  383  8439 
8.852  6830 
9  29  1  9839 
9  712  2190 

7.498  6744 
7.942  6863 
8.357  6508 
8.745  4680 
9.1079140 

7.138  9643 
7  536  0780 
7.903  7759 
8.241  2370 
8.559  4787 

16 
17 
18 
19 
20 

11.234  0151 
11.707  1914 
12.159  9918 
12.593  2936 
13.007  9365 

10.837  7696 
11.274  0663 
11  689  5869 
12.085  3209 
12.462  2103 

10  462  1620 
10.861  6086 
11.21(5  0745 
11.  (5076535 
11.950  3825 

10  1058953 
10  477  2597 
10.827  (3035 
11.158  1165 
11.4699212 

9  446  6486 
9.763  2230 
10.059  0809 
10.335  5953 
10  594  0143 

8.851  3692 
9.121  6381 
9  371  8871 
9  603  5992 
9.818  1474 

21 
22 
23 
24 
25 

13.404  7239 
13.784  4248 
14.147  7749 
14.495  4784 
14.828  2090 

12.821  1527 
13.1(53  0026 
13.488  5739 
13.798  6418 
14.093  9446 

12.275  2441 
12  583  1697 
12.8750124 
13.151  6990 
13.413  9327 

11.7640766 
12041  5817 
12.303  3790 
12  5503575 
12.783  3562 

10  835  5273 
11.001  2405 
11.2721874 
11.469  3340 
11.653  5832 

10  016  8032 
10.2007137 
10.371  0590 
K)  528  7583 
10.674  7762 

26 
27 
28 
29 
30 

15.146  6115 
15.451  3028 
15.742  8735 
16.021  8885 
.6.288  8885 

14.375  1853 
14.6130336 
14.898  1273 
15.141  0736 
15.372  4510 

13  662  4954 
13  898  0999 
14  121  4217 
14.333  101? 
14.533  7452 

13  003  1662 
13  210  5341 
13.406  1643 
13  590  7210 
13.764  8312 

11.8257787 
11.986705)1 
12  137  1113 
12.277  6741 
12.40U  0412 

10.809  9780 
10935  1618 
11.051  078.3 
11.158  40fiO 
11.257  7833 

31 
32 
33 
34 
35 

16.544  3910 
16.788  8909 
17.022  8621 
17.246  7580 
17.461  0124 

15.592  8105 
15.802  6767 
16.002  5492 
16  192  9040 
16.374  1943 

14.723  9291 
14.904  1982 
15.075  0694 
15  237  0326 
15.390  5522 

13.929  0860 
14  084  0434 
1  1  230  2290 
14.368  1411 
14.498  2464 

12531  8112 
126465553 
12  753  7900 
12  854  0094 
12.947  6723 

11.349  7994 
11  434  9994 
11  5138884 
11  5869337 
11.654  5682 

36 
37 
38 
39 
40 

17.666  0406 
17.862  2398 
18.049  9902 
18.229  6557 
18.401  5844 

16.546  8517 
16.711  2873 
16  867  8927 
17.017  0407 
17.159  0864 

15.536  0684 
15.673  9985 
15  804  7379 
15  928  6615 
16.046  1247 

14.G20  9871 
1  1  736  7803 
14  8160192 
14.949  0747 
15  046  2969 

13.035  2078 
131170166 
13.193  4735 
13.264  9285 
13  331  7089 

11.717  1928 
11  775  1785 
11.828  H690 
11  878  5824 
11.924  6133 

41 
42 
43 

44 
46 

18.566  1095 
18.723  5498 
18.874  2103 
19.018  3831 
19.156  3474 

17  294  3680 
17.423  2076 
17.545  9120 
17.662  7733 
17.774  0698 

16.157  4642 
16  2(52  9992 
16.363  0324 
16  457  8506 
16  547  7257 

15  138  0159 
15.224  5433 
15.306  1729 
15.383  1820 
15.455  8321 

13.394  1*204 
13.452  4490 
13.506  9617 
13.557  9081 
13.603  5216 

11.967  2346 
12.006  6987 
12.043  2395 
12.077  0736 
12.108  4015 

46 
47 
48 
49 
50 

19.288  3707 
19  414  7088 
19.535  6065 
19.651  2981 
19.762  0078 

17.880  0665 
17.981  0157 
18.077  1578 
18  168  7217 
18.255  9255 

16.632  9154 
16.713  6639 
16.790  2027 
16  862  7514 
16.931  5179 

15  524  3699 
15.589  0282 
15.650  0266 
15.707  5723 
15.761  8606 

13  650  0202 
13.691  6077 
13.730  4744 
13.766  7986 
13.800  7463 

12.137  4088 
12.164  2674 
12.189  1365 
12.212  1634 
12.233  4846 

RENT  OF  PRESENT  VALUE  OF  ANNUITY  OF  1       533 


Table  6 

RENT  OF  PRESENT  VALUE  OF  ANNUITY  OF  1 
!  L 


n 

*% 

1% 

ivi% 

iy.% 

1H% 

2% 

1 

1.0050000 

1  010  0000 

1  012  5000 

1.0150000 

1.017  5000 

1.020  0000 

2 

0  503  7531 

0507  5121 

0.509  394  i 

0.511  27  7;> 

0.513  1630 

0.515  0195 

3 

0.336  6722 

0.340  0221 

0.311  7012 

0.343  3830 

0.345  0675 

0  346  7547 

4 

0.253  1328 

0.256  2811 

0  257  8610 

0  259  4448 

0.261  0324 

0.262  6238 

5 

0.203  0100 

0.206  0398 

0.207  5621 

0.209  OS93 

0.210  6214 

0.212  1584 

6 

0.169  5955 

0  1725184 

0  174  0338 

0  175  5252 

0.1770226 

0  178  5258 

7 

0.145  7285 

0.1486283 

(U5008S7 

0.151  5562 

0.1530306 

0.151  5120 

8 

0.127  8289 

0  130  6903 

0.132  1331 

0.133  5840 

0.135  0429 

0.136  5098 

9 

0  113  9074 

0.116  7101 

0.118  1700 

0.119  6098 

0.121  0581 

0.122.5154 

10 

0.102  7706 

0.105  5821 

0.107  0031 

0.108  4342 

0.109  8764 

0.111  3265 

11 

0.093  6590 

0  096  4541 

0.097  8684 

0  099  2938 

0.100  7304 

0.102  1779 

12 

0.086  0()64 

0  08*  8188 

0.090  25cS3 

0  091  6800 

0.093  1138 

0.094  5590 

13 

0  079  0122 

0  082  4148 

0.083  8210 

0.085  2  10  1 

0  086  6728 

0.088  1184 

14 

0074  1361 

00769012 

0  078  3052 

0.079  7233 

0.081  1556 

0.082  6020 

15 

0.069  3644 

0.072  1238 

0.073  52(15 

0.074  9444 

0.076  3774 

0.077  8255 

16 

0.065  1894 

0  067  9446 

0.069  3467 

0  070  7051 

0.072  1996 

0.073  6501 

17 

0.061  5058 

0  064  25H1 

0  065  6fi02 

0  0(37  0797 

0.068  51(52 

0.069  9698 

18 

0.058  2317 

0  OtiO  9820 

0  0(52  3848 

0.063  8058 

0.065  2449 

0.066  7021 

19 

0  055  3025 

0.0580518 

0  059  4555 

0  000  8785 

0.062  3206 

0.063  7818 

20 

0052  600  1 

0  055  4153 

0  05(5  8204 

0.058  2457 

0.059  6912 

0.061  1567 

21 

0.050  2816 

0  053  0308 

0  054  4375 

0  055  8655 

0  057  3146 

0.058  7818 

22 

0048  1138 

0  050  8(>37 

0  052  2724 

0  053  7033 

0.055  1564 

0.056  6314 

23 

0.046  1346 

0  048  8858 

0  050  2967 

0  051  7308 

0.053  1880 

0.054  6681 

24 

0  044  320(5 

0017  0735 

0018  4866 

0.049  924  1 

0.051  3857 

0052  8711 

25 

0.012  6519 

0  045  4068 

0  0168225 

0.048  2635 

0.049  7295 

0.051  2204 

26 

0041  1116 

0  04.?  8689 

0  045  2873 

00167320 

0  048  2027 

0  049  6992 

27 

0.039  6856 

0  012  4455 

0.043  8668 

0045  3153 

0  046  7908 

0  048  2931 

28 

0.0383617 

0.011  1241 

0.012  5186 

0.044  001  1 

0.045  4815 

0  046  9897 

29 

0037  1291 

0  039  8950 

0  04  1  3223 

0.042  7788 

0.044  2642 

0.045  7781 

30 

0.035  9789 

0.038  748  L 

0.010  1785 

0.041  6392 

0.043  1298 

0.044  6499 

31 

0.034  9030 

0.037  6757 

0.039  1094 

0  040  5743 

0.042  0701 

0  043  5963 

32 

0.033  8945 

0  036  6709 

0.038  1079 

0  039  5771 

00410781 

0042  6106 

33 

0032  9173 

0  035  7274 

0.037  1679 

0038  6414 

0.040  1478 

0011  6865 

34 

0.032  0559 

0034  8  KM) 

0  036  2839 

0037  7619 

0.039  2736 

0.0408187 

35 

0.031  2155 

0.034  0037 

0.035  4511 

0.036  9336 

0.038  4508 

0.040  0022 

36 

0.030  4219 

0033  2143 

0  03  1  G653 

0.0361524 

0.037  6751 

0  039  2329 

37 

0.029  6714 

0  032  4680 

0.033  9227 

0.035  4114 

0.036  9426 

0  038  5068 

38 

0.028  9604 

0  031  7615 

0033  2198 

0.034  7161 

0.036  2499 

0.037  820(1 

39 

0.028  2861 

0031  0916 

0  032  5536 

0  034  0546 

0.035  5940 

0037  1711 

40 

0.027  6455 

0  030  4556 

0.031  9214 

0.033  4271 

0.034  9721 

0.036  5558 

41 

0.027  0363 

0  029  8510 

0.031  3206 

0.0328311 

0  034  3817 

0.035  9719 

42 

0.026  4562 

0  029  2756 

0.030  749L 

0.032  2643 

0.033  8206 

0  035  4173 

43 

0.025  9032 

0.0287274 

0  030  2047 

0.031  7247 

0.033  2867 

0.031  8899 

44 

0.025  3751 

0.028  2044 

0.029  6856 

0.031  2104 

0  032  7781 

0  034  3879 

45 

0.024  8712 

0  027  7050 

0.029  1901 

0.030  7198 

0.032  2932 

0.033  9090 

4f 

0.024  3889 

0  027  2278 

0.028  7168 

0.0302512 

0.031  8304 

0.033  4534 

47 

0.023  9273 

0  026  771  1 

0.0282611 

(J.029  8034 

0  031  3884 

0.033  0179 

48 

0.023  4850 

0.026  3338 

0  027  8307 

0.029  3750 

0  030  9657 

0.032  6018 

49 

0  023  0609 

0025  9147 

0  027  4156 

0  028  9618 

0.030  5612 

0  032  2040 

50 

0.022  6538 

0.0255127 

0.027  0176 

0.028  5717 

0.030  1739 

0.031  8232 

1 


Note:     For  Rent  of   Annuity    (1  -j-  i)n  —  1   .subtract   the    rate    per  cent. 
Example:    Rent  of  1  at  1  Vfe%  for  25  years  is  .0482035  —  .015  —  .0332B35. 


534 


APPENDIXES 
L  i 


flHll 


1- 


n 

2y4% 

2y2% 

23/4% 

3% 

3Va% 

4% 

1 

2 
3 
4 
5 

1  022  5000 
0.510  9376 
0.3184446 
0.204  2189 
0.213  7002 

1  025  0000 
0.518  8272 
0.350  1372 
0.265  8179 
0.215  2469 

1.027  5000 
0.520  7183 
0.351  8324 
0.267  4206 
0.216  7983 

1.0300000 
0.522  6108 
0  353  524 
0.269  0271 
0.218  3546 

1.035  0000 
0  526  4005 
0.356  9342 
0.272  2511 
0.221  4814 

1.040  0000 
0  530  1961 
0.360  3485 
0.275  4901 
0.224  0271 

6 

7 
8 
9 
10 

0.180  0350 
0  156  0003 
0.137  9846 
0.123  9817 
0.112  7877 

0.181  5500 
0.157  495J 
0.139  4674 
0.1254569 
0.114  2588 

0.183  0708 
0.158  9975 
0.140  9580 
0.126  9410 
0.115  7397 

0  184  5975 
0.160  5064 
0.142  4564 
0  128  4330 
0.117  2305 

0  187  6682 
0  163  544f 
0.145  4767 
0.131  4460 
0.120  2414 

0.190  7619 
0  166  60  W 
0.148  5278 
0.134  4930 
0.1232909 

11 
12 
13 
14 
15 

0.103  6365 
0.096  0174 
0.089  5769 
0.084  0623 
0.079  2885 

0.105  1060 
0.097  4871 
0.091  0483 
0.085  5365 
0.080  7665 

0.106  5863 

0.098  9687 
0.092  5325 
0.087  0246 
0.082  2592 

0  108  0775 
0.100  4621 
0  094  0295 
0.088  5263 
0.083  7666 

0.111  0920 
0.103  4840 
0.097  0616 
0.091  5707 
0.086  8251 

0.114  1490 
0  106  5522 
0.100  1437 
0  094  6690 
0,089  9411 

16 
17 
18 
19 
20 

0  075  1166 
0.071  4404 
0.068  1772 
0.065  2618 
0.062  6421 

0.076  5990 
0.072  9278 
0.069  6701 
0  066  7606 
0.064  1471 

0.078  0971 
0  074  4319 
0.071  1806 
0.068  2780 
0.065  6717 

0  079  6109 
0.075  95'25 
0  072  7087 
0.069  8139 
0.067  2157 

0.082  6848 
0.079  0431 
0  075  8168 
0.072  9403 
0.070  3611 

0  085  8200 
0.082  19H5 
0  078  9933 
00761380 
0  073  5818 

21 
22 
23 
24 
25 

0.060  2757 
0.058  1282 
0.056  1710 
0.054  3802 
0.052  7860 

0.061  7873 
0.059  6466 
0.057  6964 
0  055  9128 
0.054  2759 

0.063  3194 
0  061  1864 
0.059  2441 
0.05-7  4686 
0.055  8400 

0.064  8718 
0.062  7474 
0  060  8139 
0  059  0474 
0.057  4279 

0.068  0366 
0  065  9321 
0  064  0188 
0  062  2728 
0.060  6740 

0.071  2801 
0.069  1988 
0  067  3091 
0  065  5868 
0.064  0120 

26 
27 
28 
29 
30 

0.051  2213 
0.0198219 
00185253 
0.047  3208 
0.046  199? 

0  052  7688 
0  051  3769 
0  050  0879 
0.0488913 
0.047  7776 

0  054  3412 
0.052  9578 
0.051  6774 
0  050  4894 
0.049  3844 

0.055  938S 
0.054  5642 
0  Oo3  2932 
0.0521147 
0.051  0193 

0  059  2054 
0.057  85i!4 
0.056  6027 
0.055  4454 
0.054  3713 

0  062  5674 
0.061  2385 
0  060  0130 
0.058  8799 
0.057  8301 

31 
32 
33 
34 
35 

0.045  1528 
0.044  1742 
0.043  2572 
0.042  3966 
0.041  5873 

0.046  7390 
0.045  7683 
0.044  8594 
0.044  0068 
0.043  2056 

00183545 
0  047  3926 
0.046  4925 
0.045  6488 
0.044  8565 

0  049  9989 
0.049  0466 
0  048  1561 
0.047  3220 
0.016  5393 

0  053  3724 
0  052  4415 

0  051  5724 
0.050  7597 
0.049  9984 

0.056  8554 
0  055  9486 
0  055  1036 
0.051  3148 
0.053  5773 

36 
37 
38 
39 
40 

0  040  8252 
0.040  1064 
0  039  4275 
0.038  7854 
0,038  1774 

0.042  4516 
0.041  7409 
0.041  0701 
0.010  4362 
0.039  8362 

0  044  1113 
0.043  4095 
0.0127476 
0  042  1226 
0.041  5315 

0  043  8038 
0.045  1116 
0  044  4593 
0.043  8439 
0.043  2624 

0.049  2842 
0.048  6133 
0.047  9821 
0  047  3878 
0.046  8273 

0.052  8869 
0  052  2396 
0.051  6319 
0  051  0608 
0.050  5235 

41 
42 
43 
44 
45 

0.037  6009 
0.037  0536 
0.036  5336 
0.036  0390 
0.035  5681 

0  039  2679 
0  038  7288 
0  038  2169 
0  037  7304 
0.037  2C75 

0.040  9720 
0  040  4418 
0.039  9387 
0  039  4610 
0.039  0069 

0.042  7124 
0  042  1917 
0  041  6981 
0  041  2299 
0.040  7852 

0.046  2982 
0  045  7983 
0  045  3254 
0.044  8777 
0.044  4534 

0  050  0174 
0.049  5402 
0049  0899 
0.048  6645 
0.048  2625 

46 
47 
48 
49 
50 

0.035  1192 
0  034  6911 
0.034  2823 
0.033  8918 
0.033  5184 

0  036  8268 
0.036  4067 
0.036  0060 
0.035  6235 
0.035  2581 

0.038  5749 
0.038  1636 
0  037  7716 
0.037  3977 
0.037  0409 

0  040  3625 
0  039  9605 
0.039  5778 
0.039  2131 
0.038  8655 

0.044  0511 
0  043  6692 
0.043  3065 
0.042  9617 
0.042  6337 

0.047  8821 
0  047  5219 
0.047  1807 
0.046  8571 
0.046  5502 

Note  : 


PCI  Rent  of  Annuity  (1  -f  i)n  —  1  subtract  the  rate  per  cent. 

i 
*         Rent  of  1  at  2%  for  20  years  is  .0611567  —  .02  —  .0411567- 


RENT  OF  PRESENT  VALUE  OF  ANNUITY  OF  1 
i  i 


535 


1  - 


(1  + 


n 

4V«% 

5% 

5V2% 

6% 

7% 

8% 

1 

1  045  0000 

J  050  0000 

1  055  0000 

1  060  (KICK) 

1.070  0000 

1  080  0000 

2 

0.533  9976 

0.537  8049 

0  541  61  SO 

0  545  4369 

0  553  0918 

0  5flO  7lV)2 

3 

0.363  7734 

0  367  2086 

0  370  6541 

0  374  1098 

0381  0517 

0.388  0335 

4 

0.278  7437 

0  282  0118 

0  285  2945 

0  288  5915 

0  295  2281 

0  301  9208 

5 

0  227  7916 

0.230  9748 

0.234  1764 

0  237  3964 

02138907 

0.250  4564 

6 

0  193  8784 

0  197  0175 

0  20v>  1790 

0  203  3(526 

0  209  795S 

0  216  3151 

7 

0.1697015 

0  172  8198 

0  175  «)644 

0  179  1350 

0.185  5532 

0  1920721 

8 

0.151  6097 

0  154  7218 

0  157  8640 

0  161  0359 

0  167  467S 

0171  0148 

9 

0.137  5745 

0  140  6901 

0  143  8395 

0  147  0222 

0  153  4865 

0.1600797 

10 

0.126  378H 

0.129  5016 

0  132  6678 

0.135  8680 

0.112  3775 

0.149  0295 

11 

0  117  2t82 

0  ]  20  3889 

0  \'2t  f»707 

0  126  7929 

0  133  3569 

0  1  10  0763 

12 

0  109  6662 

0  112  8254 

0  1  16  0292 

0  119  2770 

0  1  25  9020 

0  13269;>0 

13 

0  103  2754 

0.106  4558 

0  109  6843 

0.112  9601 

0  119650!) 

0  1265218 

14 

0  097  8203 

0  101  0240 

0  lot  2791 

0.107  5819 

0  114  3149 

0  121  2968 

15 

0  093  1138 

0  096  3423 

0  099  6256 

0.102  9028 

0.109  7916 

0.1168295 

16 

00890151 

0  092  2049 

0  09")  5825 

0  098  9521 

0  105  8577 

0  112  9760 

17 

0  085  4176 

0  088  6999 

0  092  0120 

0095  4H8 

0  102  12.V2 

0.109  6291 

18 

0  OS2  2369 

0  085  5  162 

0  088  9199 

0  0')2  3565 

0  099  1126 

0  106  7021 

19 

0079  1073 

0  082  7450 

0086  1501 

0  089  6209 

0  096  75:50 

0  101  1276 

20 

0.076  8761 

0.080  24  26 

0  083  6793 

0.0&7  1846 

0  094  3929 

0.101  8522 

21 

0  074  6006 

0  077  9961 

OOH]  4618 

0  085  0046 

0  092  2890 

0  099  83"2 

22 

0072  5157 

0  075  9705 

0079  4712 

0083  0156 

0  090  4058 

0  ()«w  0321 

23 

0  070  6825 

0.074  1368 

0  077  0690 

0  081  2785 

0  088  7139 

0  096  4222 

24 

0  068  9870 

0  072  4709 

0  076  0358 

0  079  6790 

0  087  1890 

0  091  9780 

26 

0.067  4390 

0.070  9525 

0  074  5494 

0  078  2267 

0085  8105 

0.093  6788 

26 

00660214 

0.069  5643 

0  073  1931 

0  076  9041 

0084  5610 

0.092  5071 

27 

0064  7195 

0  068  2919 

0  071  9523 

0  07r>  0<)72 

0.083  4257 

0091  4181 

28 

0  063  5208 

0  067  122', 

0  070  81-11 

0  071  5926 

0  082  3919 

0  0!K)  4889 

29 

0.062  1146 

0.066  0455 

0  069  7686 

0  073  5796 

0081  4187 

0089(5185 

30 

0.061  3915 

0.065  051  1 

0  068  8054 

0  072  6189 

0  080  58(54 

0.088  8274 

31 

0.0604135 

0.064  1321 

0  067  9167 

0  07  1  7922 

0  079  7969 

0.088  1073 

32 

0.059  5632 

0063  280  4 

0  067  0952 

0  071  0023 

0  079  0729 

0087  4508 

33 

0  058  7445 

0.062  4900 

0  006  3347 

0  070  2729 

0  078  1081 

0086  85  1  (i 

34 

0  057  9819 

0  061  7554 

0  065  6296 

0  0(59  5981 

0  077  7%7 

0  086  30  1  1 

35 

0.057  2705 

0.061  0717 

0  (Mil  9719 

0.068  9739 

0.077  2310 

0  085  8033 

36 

0.056  6058 

0.060  4345 

0061  3664 

00683918 

00767153 

0.0853417 

37 

0.055  9840 

0  059  8398 

0  063  7999 

0  067  8574 

0.070  23(511 

0  081  9211 

38 

00554017 

0.059  2842 

0  063  2722 

0  067  3581 

0.075  7951 

0  084  53K9 

39 

0.054  8557 

0  058  7646 

0  062  7799 

0  066  8938 

0  075  3868 

0084  18"jl 

40 

0.044  3432 

0.058  2782 

0.062  3203 

0.066  4615 

0.075  0091 

0.083  8602 

41 

0.053  8616 

0.057  8223 

0  061  8<X)9 

0  066  0589 

0  074  6590" 

0  083  561  5 

42 

0.053  4087 

0.057  3947 

0  061  4893 

0  065  ($834 

0.074  3359 

0  083  2868 

43 

0  052  9824 

0.056  9933 

0  061  1134 

0  065  3331 

0  074  0359 

0  083  0311 

44 

0  052  5807 

0  056  6163 

0.060  7613 

0.065  0061 

0  073  7577 

0082  8015 

45 

0.052  2020 

0.056  2617 

0.060  4313 

0.064  7005 

0.073  1990 

0.082  5873 

46 

0.051  8447 

0.055  9282 

0  0601218 

0064  4119 

0  073  2600 

0  082  3899 

47 

0  051  5073 

0  055  6142 

O0')98313 

0064  1477 

0073  0371 

0  082  2080 

48 

0051  1886 

0.055  3184 

0  059  5585 

0.063  8977 

0  072  8307 

0.082  0103 

49 

0.050  8872 

0  055  0397 

0  059  3023 

0  063  6636 

0  072  6385 

0  081  8856 

50 

0  050  6022 

0.054  7767 

0.059  0615 

0  063  4443 

0.072  4599 

0.081  7429 

1 


Note:    For  Rent  of  Annuity   (1  -f  i)n  —  1   subtract  the  rate  per  cent. 

i 
.Example :   Rent  of  1  for  30  years  at  3%  is  .0510193  —  .03  =  .0210193. 


536  APPENDIXES 

Table  7 

AMERICAN  EXPERIENCE  TABLE  OF  MORTALITY 
(Based  on  100,000  living  at  age  of  10) 


Yearly 

Yearly 

Age 

Number 
living 

Number 
dying 

proba- 
bility of 

proba- 
bility of 

X 

1* 

dx 

dying 

living 

q* 

Px 

10 

100  000 

749 

0.007  490 

0.992  510 

11 

99  251 

746 

0.007  516 

0.992  484 

12 

98  505 

743 

0.007  543 

0.992  457 

13 

97  762 

740 

0.007  569 

0.992  431 

14 

97  022 

737 

0.007  596 

0.992  404 

15 

96  285 

735 

0.007  634 

0.992  366 

16 

95  550 

732 

0.007  661 

0.992  339 

17 

94  818 

729 

0.007  688 

0  992  312 

18 

94  089 

727 

0.007  727 

0.992  273 

19 

93  362 

725 

0.007  765 

0.992  235 

20 

92  637 

723 

0.007  805 

0.992   195 

21 

91  914 

722 

0  007  855 

0.992  145 

22 

91   192 

721 

0  007  906 

0  992  094 

23 

90  471 

720 

0.007  958 

0  992  042 

24 

89  751 

719 

0.008  Oil 

0.991   989 

25 

89  032 

718 

0.008  065 

0  991   935 

26 

88  314 

718 

0.008  130 

0  991   870 

27 

87  596 

718 

0  008  197 

0  991   803 

28 

86  878 

718 

0.008  264 

0.991   736 

29 

86  160 

719 

0.008  345 

0.991   655 

30 

85  441 

720 

0.008  427 

0.991   573 

31 

84  721 

721 

0.008  510 

0.991   490 

32 

84  000 

723 

0.008  607 

0  991   393 

33 

83  277 

726 

0.008  718 

0  991   282 

34 

82  551 

729 

0.008  831 

0.991    169 

35 

81  822 

732 

0  008  946 

0  991  054 

36 

81  090 

737 

0.009  089 

0  990  911 

37 

80  353 

742 

0.009  234 

0  990  766 

38 

79  611 

749 

0.009  408 

0.990  592 

39 

78  862 

756 

0.009  586 

0.990  414 

40 

78  106 

765 

0.009  794 

0.990  206 

41 

77  341 

774 

0.010  008 

0  989  992 

42 

76  567 

785 

0  010  252 

0.989  748 

43 

75  782 

797 

0.010  517 

0  989  483 

44 

74  985 

812 

0.010  829 

0.989  171 

45 

74  173 

828 

0  Oil   163 

0  988  837 

46 

73  345 

848 

0.011  562 

0  988  438 

47 

72  497 

870 

0.012  000 

0.988  000 

48 

71  627 

896 

0.012  509 

0.987  491 

49 

70  731 

927 

0.013  106 

0.986  894 

60 

69  804 

962 

0.013  781 

0  986  219 

51 

68  842 

1  Oil 

0.014  541 

0  985  459 

62 

67  841 

1  044 

0.015  389 

0.984  611 

53 

66  797 

1  091 

0.016  333 

0.983  667 

54 

65  706 

1  143 

0.017  396 

0.982  604 

AMERICAN  EXPERIENCE  TABLE  OF  MORTALITY 

AMERICAN  EXPERIENCE  TABLE  OF  MORTALITY 


537 


Yearly 

Yearly 

Age 

Number 
living 

Number 
dying 

proba- 
bility of 

proba- 
bility of 

1, 

d* 

dying 

living 

q* 

P. 

55 

64  563 

1  199 

0.018  571 

0  981  429 

56 

63  364 

1  260 

0.019  885 

0.980  115 

57 

62  104 

1  325 

0  021  335 

0.978  665 

58 

60  779 

1  394 

0.022  936 

0.977  064 

59 

59  385 

1  468 

0.024  720 

0.975  280 

60 

57  917 

1  546 

0.026  693 

0.973  307 

61 

56  371 

1  628 

0.028  880 

0.971  120 

62 

54  743 

1  713 

0  031  292 

0.968  708 

63 

53  030 

1  800 

0.033  943 

0.966  057 

64 

51  230 

1  889 

0.036  873 

0.963  127 

65 

49  341 

1  980 

0.040  i29 

0.959  871 

66 

47  361 

2  070 

0  013  707 

0.956  293 

67 

45  291 

2  158 

0  047  647 

0.952  353 

68 

43  133 

2  243 

0.052  002 

0.947  998 

69 

40  890 

2  321 

0.056  762 

0.943  238 

70 

38  569 

2  391 

0.061  993 

0.938  007 

71 

36  178 

2  448 

0.067  665 

0.932  335 

72 

33  730 

2  487 

0.073  733 

0.926  267 

73 

31  243 

2  505 

0.080  178 

0.919  822 

74 

28  738 

2  501 

G.087  028 

0.912  972 

75 

26  237 

2  476 

0.094  371 

0.905  629 

76 

23  761 

2  431 

0.102  311 

0.897  689 

77 

21  330 

2  369 

0.1  11  064 

0.888  93h 

78 

18  961 

2  291 

0.120  827 

0.879  173 

79 

16  670 

2  196 

0.131  734 

0.868  266 

80 

14  474 

2  091 

0.144  466 

0.855  534 

81 

12  383 

1  964 

0  158  605 

0.841  395 

82 

10  419 

1  816 

0.174  297 

0.825  703 

83 

8  603 

1  648 

0.191  561 

0.808  439 

84 

6  955 

1  470 

0  211  359 

0.788  641 

85 

5  485 

1  292 

0.235  552 

0.764  448 

86 

4  193 

1  114 

0.265  681 

0.734  319 

87 

3  079 

933 

0  303  020 

0  696  980 

88 

2  146 

744 

0.346  692 

0.65'*  308 

89 

1  402 

555 

0.395  863 

0.604  137 

90 

847 

385 

0.454  545 

0.545  455 

91 

462 

246 

0.532  466 

0.467  ,534 

92 

216 

137 

0  634  259 

0.365  741 

93 

79 

58 

0  734  177 

0.265  823 

94 

21 

18 

0.857  143 

0.142  857 

95 

3 

3 

1.000  000 

0.000  000 

538  APPENDIXES 

Table  8 

COMMUTATION  COLUMNS,  3^  PER  CENT 


Age 

X 

Dx 

N, 

c, 

M, 

1  -ha, 

A* 

10 

70891.9 

1575  535 

513.02 

17612.9 

22.2245 

0.24845 

11 

67981.5 

1504  643 

493  69 

17099  9 

22  1331 

0.25154 

12 

65189.0 

1436  662 

475  08 

16606.2 

22.0384 

0.25474 

13 

62509.4 

1371  473 

457  16 

16131,1 

21.9403 

0.25806 

14 

59938.4 

1308  963 

439.91 

15674.0 

21.8385 

0.26151 

15 

57471.6 

1249  025 

423.88 

15234.1 

21.7329 

0  26508 

16 

55104.2 

1191  553 

407.87 

14810.2 

21.6236 

0  26877 

17 

52832  9 

1136  449 

392.47 

14402.3 

21.5102 

0.27261 

18 

50653.9 

1083  616 

378.15 

14009.8 

21  3926 

0  27659 

19 

48562.8 

1032  962 

364.36 

13631.7 

21.2707 

0.28071 

20 

46556.2 

984  400 

351  07 

13267.3 

21.1443 

0  28497 

21 

44630.8 

937  843 

338  73 

12916  3 

21.0134 

0  28940 

22 

42782.8 

893  213 

326  82 

12577  5 

20  8779 

0  29399 

23 

41009.2 

850  430 

315  33 

12250.7 

20  7375 

0.29873 

24 

39307.1 

809  421 

304  24 

11935.4 

20.5922 

0.30365 

25 

37673.6 

770  113 

293  55 

11631.1 

20.4417 

0  30873 

26 

36106.1 

732  440 

283  62 

11337.6 

20  2858 

0  31401 

27 

34601.5 

696  334 

274  03 

11054  0 

20.1244 

0.31947 

28 

33157.4 

661  732 

264  76 

10779  9 

19.9573 

0.32512 

29 

31771.3 

628  575 

256  16 

10515.2 

19.7843 

0  33097 

30 

30440  8 

596  804 

247  85 

10259.0 

19.6054 

0  33702 

31 

29163  5 

566  363 

239  797 

10011  2 

19.4202 

0.34328 

32 

27937.5 

537  199 

232  331 

9771.38 

19  .  2286 

0  34976 

33 

26760.5 

509  262 

225.406 

9539  04 

19.0304 

0.35646 

34 

25630.1 

482  501 

218  683 

9313  64 

18.8256 

0.36339 

35 

24544  7 

456  871 

212  157 

9094.96 

18.6138 

0  37055 

36 

23502.5 

432  326 

206  383 

8882  80 

18  3949 

0  37795 

37 

22501  4 

408  824 

200  757 

8676  42 

18.1688 

0  38560 

38 

21539.7 

386  323 

195.798 

8475  .  66 

17.9354 

0.39349 

39 

20615.5 

364  783 

190.945 

8279.86 

17.6946 

0.40163 

40 

19727.4 

344  167 

186.684 

8088.92 

17.4461 

0  41003 

41 

18873.6 

324  440 

182  493 

7902.23 

17.1901 

0  41869 

42 

18052.9 

305  566 

178  828 

7719  74 

16.9262 

0  42762 

43 

17263.6 

287  513 

175  421 

7540.91 

16.6543 

0  43681 

44 

16504.4 

270  250 

172.680 

7365.49 

16.3744 

0  44628 

45 

15773.6 

253  745 

170.127 

7192.81 

16.0867 

0.45600 

46 

15070.0 

237  972 

168.345 

7022  68 

15.7911 

0  46600 

47 

14392.1 

222  902 

166  872 

6854.34 

15.4878 

0.47626 

48 

13738.5 

208  510 

166  047 

6687.47 

15  1770 

0  48677 

49 

13107.9 

194  771 

165  983 

6521.42 

14.8591 

0.49752 

50 

12498.6 

181  663 

166.424 

6355  44 

14.5346 

0.50849 

51 

11909.6 

169  165 

167.316 

6189.01 

14.2041 

0  51967 

52 

11339.5 

157  255 

168  601 

6021.70 

13.8679 

0  53104 

53 

10787.4 

145  916 

170.234 

5853.10 

13.5264 

0  54258 

54 

10252.4 

135  128 

172.317 

5682.86 

13.1801 

0  55430 

COMMUTATION  COLUMNS,  3}$  PER  CENT  539 

COMMUTATION  COLUMNS,  3}-'2'  PER  CENT 


Age 

Dx           NX           Cx          MX 

1+ax 

Ax 

55 
56 
67 
58 
59 

9733. 
9229. 
8740. 
8264. 
7801. 

40 
60 
17 
44 
83 

124876 
115142 
105912. 
97172. 
88908. 

8 
6 
2 

174 
177 
180. 
183 
186 

646 
325 
167 
140 
340 

5510 
5335 
5158 
4978. 
4795. 

54 
90 
57 
40 
27 

12  8296 
12  4753 
12  1179 
11.7579 
11.3958 

0  56615 
0  57813 
0  59022 
0  60239 
0.61463 

60 
61 
62 
63 
64 

7351. 
6913. 
6486. 
6071 
5666. 

65 
44 
75 
27 

85 

81106,4 
73754.7 
66841.3 
60354.5 
54283.3 

189 
192 
196 
199 
201 

604 
909 
117 
109 

887 

4608. 
4419. 
4226 
4030. 
3831  . 

93 
32 
41 
30 
19 

11. 
10 
10. 
9, 
9. 

0324 
6683 
3043 
9410 
5791 

0.62692 
0.63924 
0.65155 
0.66383 
0.67607 

65 
66 
67 
68 
69 

5273. 

4890 
4518. 
4157 
3808. 

33 
55 
65 
82 
32 

48616.4 
43343.1 
38452.5 
33933.9 
29776.1 

204 
206 
208 

208 
208 

457 
522 
022 
903 

858 

3629. 
3424 
3218 
3010 
2801. 

30 

84 
32 
30 
40 

9. 

8. 
8. 
8. 
7. 

2193 
8626 
5097 
1615 

8187 

0  68824 
0  70030 
0.71223 
0  72401 
0.73560 

70 
71 
72 
73 

74 

3470. 
3145 
2833 
2535 
2253 

67 
43 

42 
75 
57 

25967 
22497 
19351 
16518 
13982 

7 
1 
6 
2 
5 

207 

205 
201 
196 

189 

881 
639 
851 
430 
491 

2592 
2384 
2179 
1977 
1780 

54 
66 
02 
17 
73 

7. 
7. 
6. 
6 
6. 

4820 
1523 
8298 
5141 
2046 

0.74698 
0  75813 
0.76904 
0  77972 
0  79018 

75 
76 
77 
78 
79 

1987 
1739 
1508 
1295 
1100 

87 
39 
63 
73 
647 

11728 

9741 
8001 
6493 
5197 

9 
03 
63 
00 
27 

181 
171 
161 
151 
140 

253 
940 
889 
2646 
0891 

1591 
1409 
1238 
1076 
924 

24 
99 
05 
158 

894 

5 
5. 
5. 
5. 
4. 

9002 
6002 
3039 
0111 
7220 

0  80048 
0  81062 
0  82064 
0.83054 
0  84032 

80 
81 
82 
83 
84 

923 
763 
620 
494 
386 

338 
234 

465 
995 
641 

4096 
3173 
2410 
1789 
1294 

62 
29 
05 
59 
59 

128 
116 
104 

91 

78 

8801 
9588 
4881 
6152 
.9565 

784 
655 
538 
434 
342 

805 
924 
966 

478 
862 

4. 
4. 
3 
3 
3. 

4368 
1577 
8843 
6154 
3483 

0  84997 
0  85940 
0  86865 
0  87774 
0  88677 

85 
86 
87 
88 
89 

294 
217 
154 
103 
65 

610 
598 
383 
963 
.6231 

907 
613 
395 
241 
137 

95 
34 
74 
36 
.398 

67 
55 
45 
34 
25 

0490 
8566 
1992 
82425 
09929 

263 
196 
141 
95 
60 

906 
857 
000 
8011 
9768 

3 
2 
2 
2 

0819 
8187 
5634 
3216 
0937 

0  89578 
0  90468 
0  91332 
0  92149 
0  92920 

90 
91 
92 
93 
94 

38 
20 
9 
3 
0 

.3047 

.  18692 
.11888 
.22236 
.827611 

71 
33 
13 
4 
0 

.775 
.4700 
.2831 
.16420 
.94184 

16 
10 
5 
2 
0 

82244 
385393 
588150 
285784 
685393 

35 
19 
8 
3 
0 

8775 
05509 
66970 
08155 
.79576 

1 
1 
1 
1 
1 

8738 
6580 
4567 
2923 
1380 

0  93664 
0  94393 
0  95074 
0.95630 
0  96152 

95 

0 

114232 

0 

.114232 

0 

110369 

0 

110369 

1 

0000 

0.96618 

Index 


Abbreviated  methods  (see  Short  methods) 
Accounts: 

asset  valuation    (see  Asset  valuation 
accounts) 

balancing,  12 

current  (see  Current  account) 

foreign  exchange,  297-307 
averaging,  298-300 
branch,  conversion  of,  300-307 

receivable,  turnover,  175-176 

running,  storage,  141-142 
Accrual  basis,  income  reported  on,  167 
Accumulation  of  simple  interest,  81-82 
Accurate  interest,  81 
Actuarial  science,  311 
Addends,  3 

Adding  machine,  subtraction  on,  14 
Addition,  3-;10 

combinations,  sum  10,  4—5 

group,  6 

of  fractions,  40 
decimal,  44-45 

positive  and  negative  numbers,  230 

practical  applications,  9-10 

recording,  by  columns.  6-8 

same  number  repeated,  5 

streamline,  4 

two  columns  at  a  time,  6 

verification  of,  30,  32 
Agent,  insurance,  commission  of,  96-97 
Agreement,  partnership : 

lack  of,  181-182 

profit-sharing,  181-182 
Agricultural  indexes,  279-280.  283-284 
Algebra,  229-234 

coefficient,  230-231 

division,  234 

multiplication,  233-234 

parentheses,  brackets,  braces,  231-232 

positive  and  negative  numbers,  229- 

230 
addition  of,  230 

subtraction,  232-233 

symbols  and  terms,  229 
Aliquot  parts: 

calculating  interest  by  using,  78-79 

defined,  48 

division  by,  50-51 

multiplication  by,  49-50 

table  of,  49 

use  of,  48 

American  experience  mortality  table,  454, 
536-537 


Amortization: 
annuities,  337-338 
of  discount: 

bonds  outstanding  method,  398-399 
scientific  method,  400-404 
serial  redemption  bonds,  398-404 
Amount : 

annuity  due,  349-352 

of  ordinary  annuity,  327-331,  344-346 

table,  331 
simple,  82-83 
Analysis: 

of  compound  interest,  327-328 

table,  317 
of  statements  (see  Statements,  analysis 

of) 
Annuities: 

compound  interest  and,  327-328 
deferred,  359-364 
due  (see  Annuities  due) 
kinds  of,  327 
life  (see  Life  annuities) 
ordinary  (see  Ordinary  annuities) 
rent  of,'  327 
special,  349-366 
Annuities  due,  349-359 
amount  of,  349 

analysis  of,  350-352 
denned,  327,  349 
effective  interest  on,  358-359 
present  value  of: 

comparison  with  ordinary  annuity, 

352 

computing,  353-355 
rent  of,  357-358 

Annuity  method,  depreciation,  410-413 
Anticipation,  discounts,  75-76 
Antilogarithm,  finding,  254 
Appraisal,  valuation  by,  203 
Appropriations,  223-224 
Arithmetical  progression  (see  Progression: 

arithmetical) 

Arithmetical  solution,  problems  with  un- 
known quantities,  246-248 
Assets : 

current,  172-176 

perpetuity,  ordinary  annual  expenses 

and  replacement  of,  420-422 
replacement  of,  perpetuity  providing 
for   ordinary    annual    expenses 
and  for,  420-422 

wasting,  capitalization  of,  422-424 
Asset  valuation  accounts,  405-424 
asset  valuation,  405 
capitalization,  wasting  asset,  422-424 
composite  life,  414-418 


540 


INDEX 


541 


Asset  valuation  accounts  (cont.)i 

depletion,  405 

calculation  of,  418-419 
defined,  405,  418 

depreciation  (see  Depreciation) 

perpetuity,  ordinary  annual  expenses 
and  replacement  of  asset,  420- 
422 
Average,  123-129 

approximation  by,  bonds,  397-398 

compound  (see  Compound  average) 

cost  method,  inventories,  144-146 

due  date,  131 

moving,  124-126 

periodic,  126-127 

progressive.  126 

sales,  clerk 's  per  cent  of,  57 

simple,  123-124 

weighted,  127-129 
Averaging: 

dates  of  invoices,  131-133 

foreign  <  \rliii  w  accounts,  298-300 
Axiom,  d  rimed,  'J29 

B 

Bad    debts,    gross   profit    computations, 

162-166 

Balancing  an  account,  12 
Bank  discount,  87--90 
counting  time,  87 
denned,  87 
proceeds,  88-89 
finding  both,  89 
finding  face  of  note,  proceeds,  time, 

rate  given,  89 
table    for    finding    difference    between 

dates,  87-88 
Bar  chart,  267-269,  270 
Base.  53-54 
Bonds,  367-404 

allotment,  goodwill,  207-209 
approximation  by  averages,  397-398 
bought  on  yield  basis,  382-  383 
computation  without  bond  table,  397 
definitions,  367 
discount: 

between  interest  dates,  380 
value  of  bond  bought  at,  380-381 
interest  on : 

accrued  between  interest  dates,  380 
discount  or  premium  between  dates, 

380 
effective  rate: 

bonds  sold  at  discount,  396-397 
bonds  sold  at  premium,  396 
values  of  bonds  between  dates,  379- 

380 

premium,  between  interest  dates,  380 
price,  369 
purchased    at    discount   or   premium, 

368-369 

rate  of  yield,  369 
redeemable  from  fund,  394-3P5 
redeemed  above  par,  383-385 
redemption  periods,  387-390 


Bonds  (cotU): 

serial  redemption   (see  Serial  redemp- 
tion bonds) 
sold  at  discount,  372-376 

effective  rate  of  interest  on,  396-397 
sold  at  par,  368 
sold  at  premium,  376-379 

effective  rate  of  interest  on,  396 
tables: 

first  form,  369 
interpolating  in,  370-372 
of  values,  369 
second  form,  369-370 
use  of,  369 
values : 

computed  without  tables,  372 
table,  369 
yield,  rate  of,  369 
Book  value,  shares  of  stock,  220 
Braces,  in  algebra,  231-232 
Brackets,  in  algebra,  231-232 
Briggs'  table,  272 
Buiigeling.  62-64 

Building  MIK!  loan  associations,  425-437 
classified  problems  on,  434-437 
control,  425 
distribution  of  profits: 

Dcxtcr's  method,  427,  429,  431 
methods  compared,  430 
partnership  method,  427-429 
effective  rate  of  interest  on  money  in- 
vested   in    installment    shares. 
433-434 

funds,  withdrawal  of,  425-426 
plans  of  oig.'ini/Mliori,  426 
Dayton  or  Ohio,  431-432 
serial,  427-431 

Dexter's  method,  427,  429,  431 
methods  compared,  430 
partnership  method,  427-429 
withdrawal  value,  431 
terminating,  426 
stock,  classes  of: 
fully-paid,  425 
installment,  425 
time  required  for  stock  to  mature,  432- 

433 
Business  finance,  213-221 

book  value,  shares  of  stock,  220 
cumulative  voting,  219-220 
profits  distribution,  220-221 
stock  rights,  213-214 
working  capital,  216-219 
Business  insurance,  95-105 
agent's  commission,  96-97 
coinsurance,  97-99,  100 
finding  premium,  96 
fire,  95 
group  iif«,  102-103  (see  also  Group  life 

insurance) 
health,  104-105 
kinds  of,  95 
policy,  95 

cancellation  of,  97,  ;J5 
form  of,  95 
rates,  95-96 


542 


INDEX 


Business  insurance  (cont.): 
use  and  occupancy,  99-102 
workmen's  compensation,  104,  105 

Business  measurements,  practical,  481- 
488 


Cancellation,  37 

mark-up,  152,  153 

method,  interest,  79-80 
Capital : 

contribution,  adjustments  of,  184-186 

how  invested,  175 

interest  on,  profits  not  covering,  183- 
184 

sources  of.  1 74 

total  employed: 

ratio,  operating  profit  to,  170 
turnover,  175 

working,  210-219 

ratio,  169,  172-174 

Capitalization  of  wasting  assets,  422-424 
Capitalized  cost,  419-420 
Cash  discount,  71-72 
Cash  payment,  120-121 
Casting  out  elevens,  31-32 
Casting  out  nines,  29-30 
Cham  discounts,  72 
Change  making,  subtraction  method,  10- 

11 
Characteristic,    positive    and    negative, 

250-251 
Charts: 

bar,  267-269,  270 

circle,  265-267 

coordinate,  rules  for,  270-272 

curve,  269-270 

graphs  and,  265 

line,  26&-270 

].-i:  ,-:-:  -.'•    272  275 

]..•«.,  -T.,  -J7..  -J77 
Checking  computations,  29-34 

absolute  check,  29 

addition,  30,  32 

by  casting  out  elevens,  31-32 

by  casting  out  nines,  29-30 

check  number  thirteen,  33-34 

division,  31,  33 

where  remainder,  31 

methods,  29 

multiplication,  30-31,  33 

rough  check,  29 

subtraction,  30,  32-33 
Checks,  pay,  118-120 
Circle  chart,  265-267 
City  taxes,  223 

Clerk's  per  cent  of  average  sales,  57 
Clock  card,  in  and  out,  108 

time,  107-109 

Coefficient,  defined,  230-231 
Coin  sheet,  120-121 
Coinsurance,  97-99,  100 
Combinations,  442-444 

in  probability,  446-448 


C Commercial  discounts,  71-76 
anticipation,  75—76 
cash,  71-72 

finding  net  price,  73-75 
invoices,  transportation  charges  on,  75 
single,  equivalent  to  series,  72-73 
trade,  72 

Commissions,  percentage,  68-69 
Commodity  taxes,  224 
Common  divisor,  greatest,  36 
Common  fractions,  39-51  (see  also  Frac- 
tions: common) 

Common  multiple,  least,  36-37 
Commutation   columns,   table,   460-463 

538-539 

Complement  method,  subtraction,  12-14 
Complex  fractions,  clearing  of,  240-241 
Composite  price  indexes,  281 
Compound  average,  135-137 

dating  forward  or  backward,   135—136 
defined,  135 

product  method,  135,  136 
C /om pound  discount,  321-322 
Compound  interest,  311-325 
actuarial  science,  311 
analysis  of,  327-328 

table,  317 
compound  amount: 

calculation  of,  313-315 

fractional  part  of  conversion  period, 

324 

of  given  principal,  315-316 
tables,  313 

compound  discount,  321—322 
computation  of,  316-317 
conversions  of  interest,  results  of,  317 
effective  rates,  317-320 
method.  311 
nominal  rates,  317-319 
present  uorth,  320-321 

of  1,  table,  321 
principal,  312 
rate,  312-313,  322-323 
ratio  of  increase,  313 
relation  to  annuities,  328 
symbols,  311-312 
Computations,  checking,  29-34  (see  also 

Checking  computations) 
Conversion: 

foreign     exchange     (see    Foreign     ex- 
change: conversion) 
of  interest,  frequent,  317 
period,    compound    amount   for   frac- 
tional part  of,  324 

Coordinate  charts,  rules  for,  270-272 
Cost: 

capitalized,  419-420 
gross  profit  test: 
goods  sold,  156-157 
sales,  rate  per  cent,  157 
or  market,  inventories,  143-144 
to  retail,  determining  ratio  of,  153—154 
County  taxes,  223 
Cross    multiplication,    21-23     (see    also 

Multiplication:  cros?) 
Cumulative  voting,  219-220 


INDEX 


543 


Currency  memorandum,  120-121 
Current  account,  139-140 

denned,  139 

methods,  139-140 
Current  assets,  172-176 
Current  Tax  Payment  Act  of  1943,  107 
Curve  chart,  269-270 

D 

Oaily  cost  card,  108 
Hates: 

forward  or  backward,  135 
of  invoices,  averaging,  131-133 
Dating  terms,  75 
Dayton  or  Ohio  plan,  431-432 
Death  taxes,  225 
Debts: 
bad,   gross  profit   computations,    162- 

166 

installment  payment  of,  339-340 
Decimal  fractions,  44-48 
addition  of,  41—45 
changing: 

common  fraction  to  decimal,  48 
to  equivalent  common  fraction,  48 
defined,  4  4 
division  of,  45—48 
multiplication  of,  45 

abbreviating,  46-47 
relation  between  percentage  and  com- 
mon and,  53 
subtraction  of,  44—45 
Decrease,  per  cent  of,  59-60 
Deductions  on  payroll  records,  109,  118 
Deferred  annuity,  359-364 
Deferred  life  annuity,  463-464 

due,  464 

Denominator,  39 

Departmental  sales,  daily  record  of,  56 
Depletion: 

calculation  of,  418-419 
denned,  405,  418 
Depreciation: 
defined,  405 

methods  of  calculating,  405 
annuity,  410-413 

f  ixed-pe  re  en  tage-of -diminish  ing- 
value  method,  413-414 
sinking-fund,  409-410 
straight-line,  405-407 
sum-of-digits,  408 
unit-product,  407-408 
MoikiiiK-hou"*.  407-408 
tabl--,  ini,,  107.  408,409-410,412,414 
Dexter 's  rule,  427',  429,  431 

comparison  of  partnership  plan  and, 

430 

Discount: 
amortization     of.     serial     redemption 

bonds,  398-404 
table,  402 

bond,  between  interest  dates,  380 
bonds  purchased  at,  368-369 

calculating,  380-381 
bonds  sold  at,  372-376 


Discount  (cpnt.): 

commercial,  71-76  (see  also  Commer 
cial  discounts) 

compound,  321-322 

series  of,  72 
Distribution  of  profits,  methods  of,  427- 

431 
Dividends,  24 

and  net  cost,  life  insurance  policies,  474 
Divisibility,  tests  of,  35 
Division: 

abbreviated,  25 

by  algebra,  234 

by  aliquot  parts,  50-51 

by  *   j       ' '  •    .    255,  256-257 

by  .'•    .M,  '.'-,  24-25 

of  fractions,  42-43 
decimal,  45-46,  47-48 

on  slide  rule,  263-264 

reciprocals  in,  26-28 

short  methods,  24-28 

use  of  tables  in,  25-26 

verification  of,  31,  33 

where  remainder,  31 
Divisor,  24 

greatest  common,  36 
Dollars-times-days  method,  interest,  80 


E 


Earning  power,  determining  from  profit 

and  loss  statements,  201-202 
Effective  rate  of  interest: 

compound  interest,  317-320 

on  annuities,  332-334,  341-343 

on  annuities  due,  358—359 

on  bonds,  396-397 
Elevens,  casting  out,  31—32 
Empirical  probability,  450-452 
Endowment: 

insurance,  471-472 

pure,  459-460 
Envelopes,  pay,  120 
Equation  of  accounts,  135—137  (see  also 

Compound  average) 
Equations,  235-248 

defined,  229 

fractions,  239 

complex,  clearing  of,  240-241 

problems  containing  unknown  quanti- 
ties, 246-248 

simple,  235-238 

simultaneous,  two  or  more  unknowns 

241-246 
Events : 

compound,  448 

independent,  448 

mutually  exclusive,  449-450 
Exact  interest,  81 
Expenses,  169 

per  cent  of,  58-59 
Exponents: 

denned,  229 

logarithms,  249 


544 


INDEX 


Factoring,  35 

Factors  and  multiples,  35-37 

Federal  Farm  Loan  Act,  432 

Federal  income  tax,   sale  of  stock  and 

rights,  214-216 

Federal  Old  Age  Benefit  Tax,  107 
Financial  indexes,  278 
Fire  insurance,  95 
Fire  losses,  157-158 
First-in,     first-out    method,     inventory, 

146-147 
Fixed  -percentage  -of  -dim  inishing-valuc 

method,  depreciation,  413-414 
Fixed  property  investment,  turnover  of, 

176-180 
Focal  date,  131 
Forborne    temporary    life    annuity    due, 

467-468 

Foreign  exchange,  293-307 
accounts : 

averaging,  298-300 
branch,  conversion  of,  300-307 
conversion : 

branch  accounts,  300-307 
decimals  of  one  monetary  unit  into 
monetary  units  of  smaller  .de- 
nomination, 295 
one   monetary   unit    into    terms   of 

another,  294-295 
interest  on,  295-296 
par  of  exchange,  293-294 

current,  294 

problems,  classes  of,  294-307 
rate  of  exchange,  293 
time  bill,  value  of,  296-297 
Fractions,  239 
addition  of,  40 

changing  mixed  number  to,  40 
common,  39-51 

relation    between    percentage    and 

decimal  and,  53 
complex,  clearing  of,  240—241 
decimal : 

adding,  44-45 
subtracting,  44—45 
division  of,  42—43 
kinds  of,  39 
multiplication  of,  41—42 

two    mixed    numbers   ending    in    J, 

43-44 

reduction  of,  39-40 
subtraction  of,  40-41 
terms  explained,  39 
Frequency  ratio,  453 
Fully-paid  stock,  425 


G 


Geometrical  progression  (see  Piogression: 

geometrical) 
Gold  standard,  293,  294 
Goodwill,  201-212 
basis  of  stock  allotment,  205 


Goodwill,  basis  of  stock  allotment  (con/.) : 
bonas,     preferred     stock,     common 

stock,  207-209 
common  stock  only,  205-206 
preferred  stock  for  net  assets,  206  - 

207 

basis  of  valuation,  101 
conclusions  concerning,  209 
defined,  201 
determining  earning  power  from  profit 

and  loss  statements,  201-202 
in  partnership,  181 
methods  of  valuing,  202-204 
by  appraisal,  203 
by  number  of  years'  purchase  price 

of  net  profits,  203-204 
examples,  202-203 
excess  of  profits  over  interest  on  net 

assets,  204 
Government,     finance     of     (see     Public 

finance) 

Governmental  functions,  223 
Graphs,  265-277 

Greatest  common  divisor  (G.C.D.),  36 
Gross  and  net  income,  bar  chart,  270 
Gross  profit  computations,  155-168 
bad  debts,  162-166 
cost  of  goods  sold,  156-157 
cost  of  sales,  rate  per  cent,  157 
deferring  income,  effect  on  tax,  166-168 
fire  losses,  157-158 
installment   sales,    personal   property, 

160-161 
procedure,  155 
test  of  inventory,  155 
in  verifying    taxpayer's    inventory, 

,158-160 

unearned  gross  profit,  reserve  for,  162 
uses,  155-156 

Group  life  insurance,  102-103 
amount  of,  102 
cost  of,  103 
eligibility  for,  102 

H 

Health  insurance,  104-105 
Highway  taxes,  225 

I 

Improper  fractions,  39 
In-and-out  clock  cards,  108-109 
Income,  per  cent  of,  by  source,  57-58 
Income  tax,  225 

federal,  sale  of  stock  and  rights,  214- 

216 

Increase,  per  cent  of,  59-60 
Index  numbers,  278-284 

agricultural,  279-280 

composite  price,  281 

construction  of,  280-281 

crop  production,  283-284 

nature  of,  278-279 

production,  278 

weighted,  281-284 


INDEX 


545 


Industrial    production,    index    numbers, 

278-279 
Installment  basis,   income  reported  on, 

167 

Installment  selling,  160-161 
Installment  stock,  425 
Insurance: 

business  (see  Business  insurance) 

endowment,  471-472 

life  (see  Life  insurance) 

policies   (see  Life  insurance  policies) 

policies,  transformation,  476-477 

reserves,  473-474 

term,  470,  471 

Insurance  Principles  and  Practice,  474 
Interest: 

and  premium,   life  insurance  policies, 
474 

bond,  valuation   (see  Bonds:  interest) 

compound  (see  Compound  interest) 

effective,  use  of,  in  annuities,  332-334 

exact  or  accurate,  81 

method  (see  Interest  method) 

on  capital,  deducting,  partnership,  183 

on  foreign  exchange,  295-296 

on    investment,    profits   not   covering, 
183-184 

simple,  77-86  (see  aiso  Simple  interest) 
Interest  method: 

account  current,  139-140 

averaging  dates  of  invoices,  131,  133 

cancellation,  79-80 
Inventories,  143-154 

average  cost  method,  144-146 

cost  or  maikct,  143-144 

determining  ratio  of  cost  to  retail,  153- 
151 

first-in,  first-out  method,  146-147 

last-in,  first-out  method,  147 

mark-down,  per  cent  of,  to  net  cost,  150 

merchandise  turnovers,  118 
number  of,  348-150 

retail  method,  151-153 

turnover,  statement  analysis,  175 

valuation  of,  143 
Investment: 

average,  profit  sharing  in  ratio  of,  186- 
188 

ratio  of,  partnership,  182-183 
Invoices: 

dates    of,    averaging    (see    Averaging 
dates  of  invoices) 

discount,  transportation  charges  on,  75 

K 

Kent,  Chancellor,  181 
L 

Last-in,  first-out  method,  inventory,  147 
Least  common  multiple  (L.C.M.),  36-37 
Legal  reserve,  defined,  473 
Life  annuities,  459-468 

commutation  table,  460-461,  462-463 

deferred,  463-464 
due,  464 


Life  annuities  (cont.): 
due,  462 

use  of  commutation  table,  462-463 
nature  of,  459,  460 
payments  m  times  a  year,  466-407 
pure  endowment,  459-460 
table,  commutation  columns  in,  460- 

461 

temporary,  465 
due,  465-466 
Life  insurance,  453 
group, 102-103 
limited  payment,  477 
policies,  valuation  of,  473-478 
Life  insurance  policies,  473-478 
dividends  and  net  cost,  474 
interest  and  premium,  474 
loading,  474 

mortality  and  level  premium,  473 
preliminary  term  valuation,  477-478 
reserves,  473-474 

limited  payment  life  insurance,  477 
terminal,  475 
retrospective  method,  476 
Line  chart,  269-270 
Liquidation  of  partnership,  188-200 
by  periodic  distribution,  190 
by  total  distribution,  189-190 
Loading,  474 
Logarithm-.  249-264 
charts,  272-275 
division  of,  255,  256-257 
exponents,  249 
finding  number: 

when  log  is  given,  254 

when  mantissa  is  not  in  table,  254- 

255 

multiplication  by,  255-256 
parts  of  a  logarithm,  250 
characteristic,  250-251 
mantissa,  251-252 
powers  of  numbers,  225,  257 

process  with  negative  characteristic;, 

257 
roots  of  numbers,  258 

process  with  negative  characteristics, 

258 

rules  for  computation  of,  255 
slide  rule  (see  Slide  rule) 
table,  249 

how  to  use,  252-253 
of  numbers,  274 
use  of,  249 
Losses,  fire,  gross  profit  test,  157-158 

M 
Making  change,  subtraction  method,  10- 

Mantissa,  251-252 
Mark-down,  152,  153 

per  cent  of,  to  net  cost,  150 
Market,  cost  or,  inventories,  143-144 
Marking  goods,  percentage,  66-68 
Mark-on,  152,  153 
Mark-up,  151-152,  153 


546 


INDEX 


Measurements,  practical  business,  481- 

488 

Merchandise  turnover,  148-150 
Merchants'  rule,  partial  payments,  91,  93 
Minuend,  10 
Mixed  number,  39 
Mortality: 

and  level  premium,  473 
table,  453-454,  536-537 

notation,  454-455 
Moving  average,  124-126 
Multiples: 

factors  arid,  35-37 
least  common,  36-37 
table  of,  17,  23-24,  25,  34 
Multiplicand,  16 
Multiplication: 
algebra,  233-234 
any  number  by  11,  18 
by  aliquot  parts,  49-50 
by  factors  of  multiplier,  17-18 
by  15,  19 

by  logarithms,  255-256 
by  numbers  a  little  larger  than   100, 

1000,  etc.,  21 

by  numbers  near  100,  1000,  etc.,  19-20 
by  25,  19 

contractions  in,  17 
cross,  21-23 

number  of  throe  digits  by  number  of 

three  digits,  22-23 
number  of  three  digits  by  number  of 

two  digits,  22 

number  of  two  figures  by  11,  18 
numbers  ending  with  ciphers,  19 
of  fractions,  41-42 
decimal,  45.  46-47 
mixed  number  by  mixed  number,  44 
two    mixed    numbers    ending    in    \, 

43-44 
of  two  numbers  each  a  little  larger  than 

100,  1000,  etc.,  21 
of  two  numbers  near  100,  1000,  etc., 

20-21 

on  slide  rule,  262-263 
part  of  multiplier  factor  or  multiple*  of 

another  part,  18 

table,  multiples  of  number,  23-24 
divisicn,  25 
multiplication,  23-24 
verification  of,  30-31,  33 
Multiplier,  16 

N 

Negative  and  positive  numbers,  229-230 
Net  cost,  per  cent  of  mark-down  to,  150 
Net  income,  gross  and,  bar  chart,  270 
Net  premiums,  469-472 
annual,  469-470 

for  endowment  insurance,  471-472 
for  term  insurance,  471 
single,  469 

for  endowment  insurance,  471 
term  insurance,  470,  471 
Net  price,  finding,  73-75 


Net  profit  on  sales,  bar  chart,  268 

Nines,  casting  out,  29-30 

Nominal  rates,  compound  interest,  317- 

319 
Notes,"1   V   j  "      •    *  proceeds,  time,  and 

!         .       -re  given,  89 
Numbers,  logarithms  of,  275 
Numerator,  39 


Ohio  plan,  431-432 
Old-line  companies,  473 
Operating  statistics,  percentage.  61-62 
Ordinary  annuities,  327-348 
amortization,  337-338 
amount  of,  327,  328,  331 

computing  procedure,  328-329 
selection  of  rate  by  calculation  of, 

344  346 
semiannual  or  quarterly  basis,  329- 

331 

table,  331 
defined,  327 

installment  payment  of  debt,  339-340 
present  value  of,  335-337 

rents,  computation  of,  338 
problem  with  limited  data,  346-347 
rate: 

by  calculating  amounts  of  annuities. 

344 

computation,  343-346 
effective,  use  of,  332,  341-343 
sinking  fund  contributions,  334 
term  of,  computation,  340-341 
use  of  effective  interest  in,  332-334 

P 

Par: 

bonds  redeemed  above,  383—385 
bonds  sold  at,  368 
of  exchange,  293-294 
Parentheses,  in  algebra,  231-232 
Parity  ratio,  281 
Partial  payments,  91-94 
methods,  91-93 
merchants'  rule,  93 
United  States  rule,  91-92 
Partnership,  181-200 

agreements,  profit-sharing,  181-188 
capital  contribution,  adjusting,  184 

186 

deducting  interest  on  capital,  183 
lack  of,  181-182 

profits  not  covering  interest  on  in- 
vestment, 183-184 
ratio  of  average  investment,  186-188 
defined,  181 
goodwill,  181 
liquidating,  188-200 
losses,  ratios: 
arbitrary,  182 
of  investment,  182-183 
mathematical  calculations,  181 
method  of  distributing  profits,  427-429 
compared  with  Dexter 's  rule,  430 


INDEX 


547 


Partnership  (cont.): 

profits  distribution,  220-221,  427-430 
Pay  checks,  118-120 
Payroll  records  and  procedure: 

cash  payment,  120-121 

coin    sheet,     currency    memorandum, 
120-121 

deductions,  109,  118 

forms,  110,  111,  112,  113,  114,  llf>,  116, 
117,  118 

pay  checks,  118-120 

pay  envelopes  and  receipts,  120 

piecework  system,  116 

records,  107-116 
sheets  for,  114 

remittance  voucher,  118 

withholding  exemptions,  109 
Percentage,  53-69 

applications  of,  53 

average  sales,  clerk's  per  cent,  f)7 

budgeting,  62-64 

commissions,  68-  69 

computations,  54-56 

daily  record,  departmental  sales,  56 

decrease  or  increase,  59-60 

definitions,  53-54 

expense,  58-59 

fundamental  processes,  54 

income,  by  source,  57  58 

increase  or  decrease,  59-60 

marking  goods,  (Hi -08 

operating  statistics,  61-62 

profits  based  on  sales,  64-66 

returned  sales,  by  departments,  56  57 
Periodic  average,  126-127 
Permutations,  439-442 

in  probability,  446-448 

nature  of,  439-441 

number  of  ways  of  doing  two  or  more 

things  together,  441-442 
Perpetuity: 

ordinary  annual  expenses  and  replace- 
ment of  asset,  420-422 

paid  at  longer-than-year  intervals,  364— 

365 
Personal   property,   installment  sales  of, 

160-161" 

Piecework  system,  payroll  records,  116 
Policy,  insurance,  95 

cancellation  of,  97 

form  of,  95 

life     insurance     (see     Life     insurance; 

policies) 

Positive  and  negative  numbers,  229-230 
Powers  of  numbers,  225,  257 
Premium : 

bond,  between  interest  dates  380 

bonds  purchased  at,  368-369 
calculating,  381-382 

bonds  sold  at,  376-379 

insurance,  96 

interest  and.  474 

level,  mortality  and,  473 

net  (see  Net  premiums) 
Present  value: 

annuity  due,  352 


Present  value,  annuity  due  (cont.): 

compared    with    ordinary    annuity. 

352 

computation  of,  352-355 
rent  of,  357-358 
of  annuity  of  1,  table,  530-532 
of  1,  table.  520-526 
ordinary  annuity,  335  337,  338 
Present  worth: 

compound,  320-321 
of  1,  table,  321 
simple  interest,  84 

simple  amount  and,  comparison.  84- 

85 
Principal,  77,  312 

installment  payment  of,  339  340 
interchanging  time  and,  interest,  81 
Probability,  445  452 

and     mortality,     453  458      (w     aho 

Mortality) 
life  insurance,  453 
combinations  in,  446  4  18 
compound  events,  4-18 
empirical,  450  452 
events  (sec  Kvents) 
joint  life,  457-458 
of  dying,  450-457 
of  living,  455-456 
permutations  in,  416-448 
theory  of,  445-446 
Proceeds,  bank  discount,  88-89 
Product,  16 

Production  indexes,  278 
Product  method: 

account  current,  139  140 
averaging  dates  of  invoices,  131—132 
compound  average,  136 
Profit   and  loss  statements,  determining 

earning  power  from,  201  202 
Profits,  169 

based  on  sales,  percentage.  64  (56 
gross,   computations    (see   GrosM  profit 

computations) 

partnership,  distribution  of,  220  221 
ratios  concerning,  169,  170-  171 
Progression,  285-291 
arithmetical: 

decreasing  series,   finding  values  of 

terms,  287-288 
defined,  285 

elements,  relation  of,  2S.1 
increasing  series,  28.r>  2S(> 

finding  values  of  terms,  285-280 
symbols,  285 
decreasing  series,  28f>  2XS 

defined,  285 
defined,  285 
geometrical,  288-2!)  1 

decreasing  series  nnding  values  of 

terms,  290  291 
defined,  288 
elements  of,  280 
increasing  8<-n<>s,   finding  values  of 

terms,  289 

increasing  series  289 
problems,  use  of  logarithms,  291 


548 


INDEX 


Progressive  average,  126 
Proper  fractions,  39 
Property  taxes,  225-228 
Public  finance: 

and  taxation,  223-228  (See  also  Taxes) 

appropriations,  223-^224 

governmental  functions,  223 

Q 

Quarterly  basis,  computing  annuity,  329- 

331 
Quotient,  24 

R 

Rate,  54,  312-313 

compound  interest,  312-313,  322-323 
effective,  ordinary  annuity,  332,  341- 

343 

insurance,  95—96 
of  exchange.  293 
of  interest,  77 
effective,  317-320 
nominal,  317-319 
of  yield,  bonds,  369 
ordinary  annuity,  343-346 
Ratios : 

average  investment,  profit  sharing  in, 

186-188 

chart,  275,  276-277 
cost  to  retail,  determining,  153-154 
financial  and  operating,   169-180   (see 
also    Statements,    analysis    of: 
ratios) 

frequency,  453 

geometrical  progression,  290-291 
of  increase,  compound  interest,  313 
parity,  281 
partnership: 
arbitrary,  182 
of  investment,  182-183 
working  capital,  169,  172-174 
Receipts,  pay,  120 
Reciprocals,  in  division,  26-28 
Record,  daily,  departmental  sales,  56 
Redemption,   serial,    bonds    (see   Bonds: 

serial  redemption) 
Redemption  periods,  bonds,  387 
Reduction  of  fractions,  39-40 
Remainder,  24 
Rent: 
Aof  annuity,  327 

present  value  of,  338 
of  annuity  due,  355-357 

present  value  of,  357-358 
Repeater,  67 
Reserves : 
denned,  473 

for  unearned  gross  profit,  162 
policy,  473-^74 
terminal,  475 
Retail,  determining  ratio  of  cost  to,  153- 

154 

Rider,  insurance  policy,  95 
Roots  of  numbers,  225,  258 
Running  account,  141-142 


S 


Sales: 

average,  clerk's  per  cent,  57 

bar  charts,  268 

departmental,  daily  record,  56 

profits  based  on,  percentage,  64-66 

returned,  by  departments,  56-57 
Serial  redemption  bonds: 

amortization  of  discount,  premium,  or 
discount  and  expense  on,  398 
404 

nature  of,  385 

redeemed  by  other  than  equal  annual 
payments,  390 

redemption  periods,  387-390 

value  of,  analysis  of  calculation  of,  38.3 
387,  388 

value  of  series,  verification  of  calcula 
tion,  393-394 

with   interest   payments,    analysis   of, 

391-393 

Series  of  discounts,  72 
Short  methods,  3-28 

addition,  3-10  (see  also  Addition) 

calculating  simple  interest,  77 

calculating  single  discount,  equivalent 
to  any  two  discounts,  73 

division,  21-28  (see  also  Division) 

multiplication,   16-24   (see  also  Multi- 
plication) 

subtraction,  10-16  (see  also  Subtraction) 
Signs,  arithmetic,  229 
Simple  amount,  82-83 
Simple  average,  1 23—1 2 1 
Simple  equations,  235-238 
Simple  interest,  77-86 

accumulation  of,  Sl-82 

aliquot-parts  method,  78 

cancellation  method,  79-80 

defined,  77 

doilars-times-davs  method,  80 

interchanging  principal  and  time,  81 

present  worth,  84 

simple  amount  and,  comparison,  84- 
85 

rate,  83 

short  method,  77 

simple  amount,  82-83 

sixty-day  method,  77-78 

table  of,'  82 

time,  83-84 

true  discount,  85-^86 
Simultaneous  equations,  241-246 
Sinking  fund  contributions,  334 
Sinking-fund  method,  depreciation,  409- 

410 
Sixty-day  method  of  calculating  interest, 

77-78 
Slide  rule: 

accuracy  of  calculations  made  by,  259 

described,  258-259 

division  on,  263-264 

learning  to  use,  260 

model,  constructing,  261-262 

multiplication  on,  262-263 


INDEX 


549 


Slide  rule  (co nt.): 
reading,  261 
theory  of,  259-260 
use  of,  259 

Social  Security  Act,  107 
Statements,  analysis  of,  161M80 
cost,  expenses,  profits,  169-170 
ratios,  169-170 

costs,  expenses,  to  net  sales,  169-170 
financial  and  operating,  71 
gross  profit  to  net  sales,  170 
net  profit  to  net  sales,  170 
net  profit  to  net  worth,  170 
operating  profit  to  net  sales,  170 
operating    profit     to    total    capital 

em  ploy  od,  170 
working  capital,  172-174 
relationships: 

capital,  now  invested,  175 
earnings   on    comimm    stockholders' 

investments,  170 
sources  of  capital,  174 
turnover : 

accounts  receivable,  175-176 
fixed  property  investment,  176   180 
inventories,  175 
total  capital  employed,  175 
State  taxes,  225 

Statistics,  operating,  percentage,  61   62 
Stock: 

allotment,  goodwill,  205  209 
classes  of,  building  and  loan   associa- 
tions,    425 
rights,  213  211 

sale  of  stock  and,  federal  income  tax, 

214-216 

shares  of,  book  value  of,  220 
time  reciiured  to  mature,  432-433 
Stock  card,  145 

Stockholders,   common,   earnings  on   in- 
vestments, 170 
Storage,  141-142 
defined,  141 

running  account,  141-142 
Stores  ledger,  form,  146 
Straight-line  method,  depreciation,   405- 

407 

Streamline  addition,  4 
Subtraction,  10-16 
balancing  account,  12 
by  algebra,  232-233 
complement  method,  12-14 
difference  between  given  minuend  and 

several  subtrahends,  11-12 
errors,  avoiding,  11 
of  fractions,  40-41 

decimal,  44-^45 
on  adding  machine,  14 
practical  problems,  14—16 
verification  of,  30,  32-33 
Subtrahend,  10 
Sum,  3 

Sum-of-digits  method,  depreciation,  408 
Symbols: 

and  terms,  algebra,  229 
compound  interest,  311-312 


T 

Tables: 

aliquot  parts,  49 
amount  of  annuity,  331 

of  1,  527-529 
annuity,  section  of,  344 
asset  valuation  accounts,  406,  407,  408, 

409-410,  412,  414,  423 
bond  (see  Bonds:  tables) 
Briggs',  272 

cancellation,  short  rate,  98 
compound  amount,  313 

of  1,  512-519 

compound  interest,  analysis  of,  317 
drill,  addition,  3  6 

for  finding  difference  between  dates,  88 
installment  payments,  339 
life  annuities,  462  403 

commutation  columns  in,   160  4(11 
logarithms,  249,  252  253,  197  511 
%mortality,   American  experience,  536  - 

537 

multiples,  17,  23  24,  25,  31 
present  value  of  annuity  of  1,  530  532 
present  value  of   1,  520-520 
present  worth  of  1,  321 
rent,  present  value  annuity  of  1,  533  • 

r  o  *• 

5Jo 

simple  interest,  82 

sinking  fund  contributions,  321 

use  of,  in  division,  25  20 

weights,  measures,  values,  489  490 
Taxes: 

deferring  income,  effect  on,  166   168 

in  U.S.,  1943,  221 

kinds  of,  224-225 

public  finance  and  taxation  (xrc,  Public, 
finance) 

purposes  of,  223 
Taxpayer's  inventory,  gross  profit  test  in, 

158-100 
Temporary  lift1  annuities,  -4(55 

due,  465-466 

forborne,  467  -468 
Terminating  plan,  12'»    127 
Term  insurance,  -170,  471 
Term  of  credit,  131 
Thirteen,  check  number,  33  3-1 
Time: 

bill  of  exchange,  value  of,  290  297 

compound  interest,  312,  323  324 

counting,  bank  discount,  87 

interchanging  principal  and,   interest, 
81 

simple  interest,  83-84 
Timebooks,  107 
Time-clock  cards,  107-109 

forms,  108 
Town  taxes,  223 
Trade: 

discount.  72 

foreign  (see  Foreign  exchange) 

indexes,  278 

Transportation  charges,  on  discount  in- 
voices, 75 


550 


INDEX 


True  discount,  simple  interest.  85-86 
Turnover,  merchandise,  148-150 

U 

Unearned  gross  profit,  reserve  for,  102 

Unit,  39 

United  States  rule,  partial  payments,  91- 
92,  93 

Unit-product  method,  depreciation,  407- 
408 

Unknown  quantities,  solution  of  prob- 
lems containing,  246-248 

Use  and  occupancy  insurance,  99-102 


Valuation: 
of  asset  accounts  (see  Asset  valuation 

accounts) 

of  bonds,  369,  372  (see  also  Bonds) 
of  goodwill  (see  Goodwill:  methods ^>f 

valuing) 
of  iuvtntories,  143 


Valuation  (cont.): 

of  life  insurance  policies  (see  Life  in- 
surance policies) 
Voting,  cumulative,  219-220 


W 


Wasting  assets,  capitalization  of,  422-424 
Weighted  average,  127-129 
Weighted  index  numbers,  281-284 
Withholding  exemptions,  payroll  records, 

109 

Withholding  tax,  107 
Working  capital,  216-219 

ratio,  169,  172-174 
W.          .•        .•       method,     depreciation 

4U7-408 
Workmen's  compensation,  104,  105 


Y 


Yield  basis,  bond  bought  on, 
Yields  of  Bonds  and  Stocks,  < 


382-383 
370,  371